Sie sind auf Seite 1von 554

D

O
-n

m
"n

m
>

D
O
GO
CO
>

J-5rL

*
O 7
V
WILEY
TOPPAN

V
Wiley International Edition

PRINCIPLES OF
REFRIGERATION
ROY J. DOSS AT, Associate Professor of Refrigeration
and Air Conditioning, University of Houston, Houston, Texas

W
JOHN WILEY & SONS, INC.
NEW YORK and LONDON
TOPPAN COMPANY, LTD.
TOKYO, JAPAN
Authorized reprint of the edition published by John
Wiley & Sons, Inc., New York and London.

Copyright © 1961 by John Wiley ft Sons, Inc.


All Rights Reserved. No part of this book may
be reproduced in any form without the written
permission of John Wiley & Sons, Inc.

Wiley International Edition

This book is not to be sold outside the country


to which it is consigned by the publisher.

Library of Congress Catalog Card Number: 61-15396

Printed in Singapore by Toppan Printing Co. (S) Pte. Ltd.


Preface

This textbook has been written especially for use in programs where a full
curriculum in refrigeration is offered. However, the material covered and the
method of presentation are such that the text is also suitable for adult evening
classes and for on-the-job training and self-instruction. Furthermore, the
material is so arranged and sectionalized that this textbook is readily adaptable
to any level of study and to any desired method or sequence of presentation.
Despite a rigorous treatment of the thermodynamics of the cycle, application
of the calculus is not required nor is an extensive background in physics and

thermodynamics presupposed. The first four chapters deal with the funda-
mental principles of physics and thermodynamics upon which the refrigeration
cycle is based. For those who are already familiar with these fundamentals,
the chapters will serve as review or reference material.
Chapter 21 motors and control circuits as they apply to refrigera-
treats electric
tion and This material is presented from the viewpoint
air conditioning systems.
of practical application, the more mathematical approach being left to companion
electrical courses.
Throughout this textbook emphasis is placed on the cyclic nature of the
refrigeration system, and each part of the system is carefully examined in relation

to the whole. Too, care is taken continually to correlate theory and practice
through the use of manufacturer's catalog data and many sample problems. To
this end, certain pertinent catalog data are included.

Roy J. Dossat

July, 1961
:

Acknowledgments

Most of the material in this textbook Ingersoll-Rand Company


is based on information gathered from Kennard Division, American Air Filter
publications of the American Society of Company, Inc.
Heating, Refrigerating, and Air Condition- Kramer Trenton Company
ing Engineers and of the following equip- McQuay, Inc.
ment manufacturers The Marley Company
Marsh Instrument Company
Acme Industries, Inc. Mueller Brass Company
Alco Valve Company Penn Controls, Inc.
Anaconda Metal Hose Division, The Recold Corporation
American Brass Company Sporlan Valve Company
Bell & Gossett Company Tecumseh Products Company
Carrier Corporation Tranter Manufacturing, Inc.
Controls Company of America Tubular Exchanger Manufacturers
Dean Products, Inc. Association, Inc.
Detroit Controls Division, American Tyler Refrigeration Corporation
Radiator & Standard Sanitary The Vilter Manufacturing Company
Corporation worthington corporation
Detroit Ice Machine Company York Corporation, Subsidiary of Borg-
Dole Refrigerating Company Warner Corporation
Dunham-Bush, Inc.
Edwards Engineering Corporation Appreciation is expressed to all these
E. I. du Pont de Nemours & Company organizations for their contributions in the
Freezing Equipment Sales, Inc. form of photographs and other art work,
Frick Company and for granting permission to reproduce
General Controls Company proprietary data, without which this text-
General Electric Company book would not have been possible.
Halstead & Mitchell

vi
Contents

1. Pressure, Work, Power, Energy I

2. Matter, Internal Energy, Heat, Temperature 10

3. Thermodynamic Processes 24

4. Saturated and Superheated Vapors 43

5. Psychrometric Properties of Air 57

6. Refrigeration and the Vapor Compression System 71

7. Cycle Diagrams and the Simple Saturated Cycle 89

8. Actual Refrigerating Cycles 107

9. Survey of Refrigeration Applications 121

10. Cooling Load Calculations 144

11. Evaporators 164

12. Performance of Reciprocating Compressors 203

13. System Equilibrium and Cycling Controls 225

14. Condensers and Cooling Towers 244

15. Fluid Flow, Centrifugal Liquid Pumps, Water and Brine Piping 274

16. Refrigerants 284

17. Refrigerant Flow Controls 298


vii
viii PRINCIPLES OF REFRIGERATION

18. Compressor Construction and Lubrication 334

19. Refrigerant Piping and Accessories 365

20. Defrost Methods, Low-Temperature Systems, and Multiple Temperature


Installations388

21. Electric Motors and Control Circuits 407

Tables and Charts 430

Index 535
where P « the pressure expressed in units of F
per unit of A
F — the total force in any units of force
A = the total area in any units of area
1-3. Measurement of Pressure. As indi-
cated by equation 1-1, pressure is measured
in units of force per unit of area. Pressures are
most frequently given in pounds per square
inch, abbreviated psi. However, pressure, like
I
force, as a matter of convenience and depending
on the magnitude of the pressure, may be stated
in terms of other units of force and area, such
Pressure, Work, as pounds per square foot, tons per square foot,
grams per square centimeter, etc.
Power, Energy Example l-l. Arectangular tank, measur-
ing 2 ft by 3 ft at the base, is filled with water.
If the total weight of the water is 432 lb, deter-
mine the pressure exerted by the water on the
bottom of the tank in
(a) pounds per square foot
l-l. Force. A
force is defined as a push or a (b) pounds per square inch
pull. anything that has a tendency to set
It is
Solution
a body in motion, to bring a moving body to
rest, or to change the direction of motion. A (a) Area of tank base 2x3
= 6sqft
force may also change the size or shape of a
Total weight of water = 432 lb
body. That is, the body may be twisted, bent,
432
stretched, compressed, or otherwise distorted Applying Equation 1-1, P —^
by the action of a force.
= 72psf
The most familiar force is weight. The
(b) Area of the tank base 24 x 36
weight of a body is a measure of the force
exerted on the body by the gravitational pull
= 864 sq in.
of the earth (Fig. 1-1).
Total weight of water = 432 lb
432
There are many forces other than the force of Applymg Equation 1-1, P
gravity, but all forces are measured in weight 864
units. Although the most commonly used unit
= 0.5 psi
of force measure is the pound, any unit of The problem described in Example 1-1 is

weight measure may be used, and the particular Notice that the pressure
illustrated in Fig. 1-2.
unit- used at any time will usually depend on on the bottom of the tank in pounds per square
the magnitude of the force to be measured. foot is equivalent to the downward force
1-2. Pressure. Pressure is the force exerted exerted by the weight of a column of water
per unit of area. It may be described as a having a cross section of one square foot, where-
measure of the intensity of a force at any given as the pressure in pounds per square inch is

point on the contact surface. Whenever a force equivalent to the downward force exerted by a
is evenly distributed over a given area, the column of water having a cross section of 1 sq in.
pressure at any point on the contact surface is Further, since there are 144 sq in. in 1 sq ft, the
the same and can be calculated by dividing the force exerted per square foot is 1 44 times as great
total force exerted by the total area over which as the force exerted per square inch.
the force is This relationship
applied. is 1-4. Atmospheric Pressure. The earth is
expressed by the following equation surrounded by an envelope of atmosphere or air
which extends upward from the surface of the
earth to a distance of some 50 mi or more. Air
'-h (1-1)
has weight and, because of its weight, exerts a
PRINCIPLES OF REFRIGERATION
cross section will be less when taken at an
altitude of one mile above sea level than when
taken at sea level. Therefore, it follows that
atmospheric pressure decreases as the altitude
increases.
1-5. Barometers. Barometers are instruments
used to measure the pressure of the atmosphere
and are of several types. A simple barometer
which measures atmospheric pressure in terms
of the height of a column of mercury can be
constructed by filling with mercury a hollow
-Spring scale
glass tube 36 in. or more long and closed at one
Pointer -
end. The mercury is held in the tube by placing
the index finger over the open end of the tube
while the tube is inverted in an open dish of
mercury. When the finger is removed from the
tube, the level of the mercury in the tube will
fall, leaving an almost perfect vacuum at the

closed end. The pressure exerted downward by


the atmosphere on the open dish of mercury will
cause the mercury to stand up in the evacuated
tube to a height depending upon the amount of
pressure exerted. The height of the mercury
column in the tube is a measure of the pressure
exerted by the atmosphere and is read in inches

Weight
of mercury column (abbreviated in. Hg). The
normal pressure of the atmosphere at sea level
(14.696 psi) pressing down on the dish of mercury
will cause the mercury in the tube to rise to a

Fig. 1-1, Because of gravity, the suspended weight


exerts a\downward force of 7 lb.

pressure on the surface of the earth. The pressure


exerted by the atmosphere is known as atmos-
pheric pressure.
The weight of a column of air having a cross
section of 1 sq in. and extending from the surface
of the earth at sea level to the upper limits of the
atmosphere is 14.696 lb. Therefore, the pressure
on the surface of the earth at sea level resulting
from the weight of the atmosphere is 14.696 psi
(14.7). This is understood to be the normal or
standard atmospheric pressure at sea level and
is sometimes referred to as a pressure of one
atmosphere. Actually, the pressure of the
atmosphere does not remain constant, but will
usually vary somewhat from hour to hour Fig. 1-2. Of the total weight of the water in the
depending upon the temperature, water vapor container, that part which is exerted on a sq I ft area
content, and several other factors. is the pressure in pounds per square foot. Likewise,
Because of the difference in the height of the that part which is exerted on a sq in. area is the
I

column, the weight of a column of air of given pressure in pounds per square inch.
PRESSURE, WORK, POWER, ENERGY 3

height of 29.921 in. (Fig. 1-3). A column of brated in inches to read the deviation of the
mercury 29.921 in. high is, then, a measure of a mercury columns from the zero condition in
pressure equivalent to 14.696 psi. By dividing either direction (Fig. l-4a).
29.921 in. Hg by 14.696 psi, it is determined that When in use, one side of the U-tube is
a pressure of 1 psi is equivalent to a pressure of connected to the vessel whose pressure is to be
2.036 in. Hg. Therefore, 1 in. Hg equals measured. The pressure in the vessel, acting on
1/2.036, or 0.491 psi, and the following equa- one leg of the tube, is opposed by the atmos-
tions are established: pheric pressure exerted on the open leg of the
psi tube. If the pressure in the vessel is greater
in. Hg = (1-2)
than that of the atmosphere, the level of the
0.491
= mercury on the vessel side of the U-tube is
and psi in. Hg x 0.491 (1-3)
depressed while the level of the mercury on the
Example 1-2. What isthe pressure of the
open side of the tube is raised an equal amount
atmosphere in psi if a barometer reads 30.2
(Fig. l-4b). If the pressure in the vessel is less
in. Hg?
than that of the atmosphere, the level of the
Solution. Applying Equation 1-3,
mercury in the open leg of the tube is depressed
P- 30.2 x 0.491
while the level of the mercury in the leg con-
= 14.83 psi
nected to the vessel is raised by an equal amount
Example 1-3. In Fig. 1-3, how high will
(Fig. l-4c). In either case, the difference in the
the mercury stand in the tube when the atmos-
pheric pressure is 14.S psi? heights of the two mercury columns is a
measure of the difference in pressure between
Solution. Applying Equation 1-2,
the total pressure of the fluid in the vessel and
the pressure of the atmosphere.
0.491
= In Fig. l-4b, the level of the mercury is 2 in.
29.53 in. Hg
1-6. Pressure Gages. Pressure gages are
instruments used to measure the fluid pressure
(either gaseous or liquid) inside a closed vessel.
Pressure gages commonly used in the refriger-
ation industry are of two types: (1) manometer
and (2) bourdon tube.
1-7. Manometers. The manometer type gage
utilizes a column of liquid to measure the
pressure, the height of the column indicating
the magnitude of the pressure. The liquid used
in manometers is usually either water or mercury. Scale (inches)

When mercury is used, the instrument is known


as a mercury manometer or mercury gage and,
when water is used, the instrument is a water
manometer or water gage. The simple barom-
eter described previously is a manometer type
instrument.
A simple mercury manometer, illustrated in
Figs. 1-4a, 1-46and l-4c, consists of a U-shaped
glass tube open at both ends and partially filled
with mercury. When both legs of the U-tube
are open to the atmosphere, atmospheric pres- Dish of mercury
sure is exerted on the mercury in both sides of
Fig. 1-3. The pressure exerted by the weight of the
the tube and the height of the two mercury
atmosphere on the open dish of mercury causes the
columns is the same. The height of the two mercury to stand up into the tube. The magnitude
mercury columns at this position is marked as of the pressure determines the height of the mercury
the zero point of the scale and the scale is cali- column.
PRINCIPLES OF REFRIGERATION

Manometers using water as the measuring


Atmospheric
/pressure \ fluid are particularly useful for
measuring very
small pressures. Because of the difference in the
density of mercury and water, pressures so
slight that they will not visibly affect the height
of a mercury column will produce easily
detectable variations in the height of a water
column. Atmospheric pressure, which will
support a column of mercury only 29.921 in.
high, will lift a column of water to a distance of
approximately 34 ft. A
pressure of 1 psi will
raise a column of water 2.31 ft or 27.7 in. and a

Atmospheric
pressure
Fig. l-4o. Simple U-tube manometer. Since both
30 in. Hg
legs of the manometer are open to the atmosphere
Vessel
and are at the same pressure, the level of the mercury - pressure
is the same in both sides. 26 in. Hg

Atmospheric
pressure
30 in. Hg

Fig. l-4c Manometer indicates that the vessel


pressure is 4 in. Hg less than the atmospheric
pressure of 30 in. Hg.

Fig. I -4b. Simple manometer indicates that the


vessel pressure exceeds the atmospheric pressure
by 4 in. Hg.

below the zero point in the side of the U-tube


connected to the vessel and 2 in. above the zero
point in the open side of the tube. This indi-
cates that the pressure in the vessel exceeds the
pressure of the atmosphere by 4 in. Hg (1.96
psi). In Fig. l-4c, the level of the mercury is
depressed 2 in. in the side of the tube open to
the atmosphere and raised 2 in. in the side con-
nected to the vessel, indicating that the pressure
in the vessel is 4 in. Hg (1.96 psi) below (less
than) atmospheric. Pressures below atmospheric
are usually called "vacuum" pressures and may Fig. 1-5. Bourdon tube gage mechanism. (Courtesy
be read as "inches of mercury, vacuum." Marsh Instrument Company.)
PRESSURE, WQRK, POWER, ENERGY 5

pressure of only 0.036 psi is sufficient to support inches of mercury (Fig. 1-66). In many cases,
a column of water 1 in. high. Hence, 1 in. single gages, known as "compound" gages, are
of water column is equivalent to 0.036 psi. designed to measure pressures both above and
Table 1-1 gives the relationship between the below atmospheric (Fig. l-6c). Such gages are
various units of pressure measurement. calibrated to read in psi above atmospheric and
1-8. Bourdon Tube Gages. Because of the in inches of mercury below atmospheric.
excessive length of tube required, gages of the 1-9. Absolute and Gage Pressures. Absolute

manometer type are not practical for measuring pressure is understood to be the "total" or
pressures above IS psi and are more or less "true" pressure of a fluid, whereas gage pressure

<°) (b) (c)

bourdon tube gages,


Fig. 1-6. Typical (a) Pressure gage, (b) Vacuum gage, (c) Compound gage. (Courtesy
Marsh instrument Company.)

limited to the measurement of relatively small is the pressure as indicated by a gage. It is


pressures in air ducts, etc. important to understand that gages are cali-
Gages of the bourdon tube type are widely brated to read zero at atmospheric pressure and
used to measure the higher pressures en- that neither the manometer nor the bourdon
countered in refrigeration work. The actuating tube gage measures the "total" or "true"
mechanism of the bourdon tube gage is illus- pressure of the fluid in a vessel; both measure
trated in Fig. 1-5. The bourdon tube, itself, is a only the difference in pressure between the total
curved, elliptical-shaped, metallic tube which pressure of the fluid in the vessel and the atmos-
tends to straighten as the fluid pressure in the pheric pressure. When the fluid pressure is
tube increases and to curl tighter as the pressure greater than the atmospheric pressure, the
decreases. Any change in the curvature of the absolute pressure of the fluid in the vessel is
tube is transmitted through a system of gears determined by adding the atmospheric pressure
to the pointer. The direction and magnitude to the gage pressure, and, when the fluid
of the pointer movement depend on the direc- pressure is less than atmospheric, the absolute
tion and magnitude of the change in the curva- pressure of the fluid is found by subtracting the
ture of the tube. gage pressure from the atmospheric pressure.
Bourdon tube gages are very rugged and will The relationship between absolute pressure and
measure pressures either above or below gage pressure is shown graphically is Fig. 1-7.
atmospheric pressure. Those designed to Example 1-4. A pressure gage on a
measure pressures above atmospheric are known refrigerant condenser reads 120 psi. What is
as "pressure" gages (Fig. l-6a) and are gener- the absolute pressure of the refrigerant in the
ally calibrated in psi, whereas those designed to condenser?
read pressures below atmospheric are called Solution. Since the barometer reading is not
"vacuum" gages and are usually calibrated in given, it is assumed that the atmospheric
PRINCIPLES OF REFRIGERATION
Gage Absolute Example 1-6. During compression the
pressure pressure pressure of a vapor is increased from 10 in. Hg
gage to 125 psi gage. Calculate the total increase
45- 59.7 in pressure in psi.

Solution. Since the pressure increases from


40 54.7
10 in. Hg below atmospheric to 125 psi above
35- 49.7 atmospheric, the total increase in pressure is
the sum of the two pressures.
30 -44.7
Initial pressure = 10 in. Hg
25- 39.7
Initial pressure in psi below 1 x 0.491
Pressures above atmospheric = 4.91 psi
atmospheric in psi
20- 34.7 Final pressure in psi above
atmospheric =125 psi
15- 29.7
Total increase in pressure = 129.91 psi
10- 24.7 Absolute pressure in psi is abbreviated psia,

5-
whereas gage pressure in psi is abbreviated psig.
•19.7
1-10. Work. Workis done when a force
Atmospheric pressure 29.92 in. Hg
(14.7 psi)
acting on a body moves the body through a
5-
distance. The amount of work done is the
10 -Pressures below-
15- - atmospheric in— product of the force and the distance through
20 in. Hg which the force acts. This relationship is shown
25
(14.7 psi) by the following equation:
29.92 in. Hg~~
W=Fx 1 (1-4)
Fig. 1-7. Relationship between absolute and gage
where F= the force applied in any units of
pressures.
force
1 = the distance through which the force
pressure is normal at sea level, 14.696 psi, and, acts in any linear unit
since the pressure of the refrigerant is above W = the work done expressed in units of
atmospheric, the absolute pressure of the force and linear measure
refrigerant is equal to the gage pressure plus
the atmospheric pressure. The work done is always expressed in the

=120 same unit terms used to express the magnitude


Gage pressure in psi
Atmospheric pressure in psi = 14.696 of the force and the distance. For instance, if
the force is expressed in pounds and the dis-
Absolute pressure of
refrigerant = 134.696 psi tance in feet, the work done is expressed in
foot-pounds. The foot-pound is the most
Example 1-5. A compound gage on
the
frequently used unit of work measure.
suction side of a vapor compressor reads
5 in. Hg, whereas a barometer nearby reads ExampleU7. A
ventilating fan weighing
29.6 in. Hg. Determine the absolute pressure 315 lb hoisted to the roof of a building 200 ft
is
of the vapor entering the compressor. above the level of the ground. How much
Solution. Since the pressure of the vapor work is done?
entering the compressor is less than atmospheric,
Solution. By applying
the absolute pressure of the vapor is computed weight of
Equation 1-4, the
by subtracting the gage pressure from atmos- the fan = 315 lb
pheric pressure. Distance through which
Atmospheric pressure in the fan is hoisted = 200 ft
in. Hg = 29.6 Work done 315 x 200
Gage pressure in in. Hg = 5.0 = 63,000 ft-lb

Absolute pressure in of doing


= -I I. Power. Power is the rate
24.6 in. Hg
I
in. Hg
24.6 x 0.491 work. That is, it is the work done divided by the
Absolute pressure
= 12.08 psi time required to do the work. The unit of power
PRESSURE, WORK, POWER, ENERGY 7

is the horsepower. One horsepower is defined as where JT — the kinetic energy in foot-pounds
the power required to do work at the rate of M = the weight of the body in pounds
33,000 ft-lb per minute or (33,000/60) 550 ft-lb V = the velocity in feet per second (fps)
per second. The power required in horsepower g = gravitational constant (32. 1 74 ft/sec2)
may be found by either of the following equa-
tions: Example 1-9. An automobile weighing
3500 lb is moving at the rate of 30 mph. What
(1 " 5) is its kinetic energy?
^-TSMTT,
Solution. Velocity 5280 ft/mi x 30 mi
where Hp = the horsepower
W = the work done in foot-pounds in fps V 3600sec/hr
= the time in minutes = 44 fps
t
Applying Equa- 3500 lb x (44 fps)*
or
W tion 1-7, the kinetic 2 x 32.174 ft/sec2
Hp = (1-6) energy AT = 105,302 ft-lb
550 x t

where t = the time in seconds 1-14. Potential Energy. Potential energy is

Example the energy a body possesses because of its


In Example 1-7, if the time
1-8.
required to hoist the fan to the roof of the position or configuration. The amount of work
building is 5 minutes, how much horsepower a body can do in passing from a given position
is required? or condition to some reference position or
work done
Solution. Total =63,000 ft-lb
condition is a measure of the body's potential
Time required to do the energy. For example, the driving head of a pile-
work = 5 min driver has potential energy of position when
raised to some distance above the top of a
„ J
Horsepower required
. 63,000
piling. If released, the driving head can do the
33,000 x 5
= 0.382 hp
work of driving the piling. A compressed steel
spring or a stretched rubber band possesses
1-12. Energy. In order to do work or to
potential energy of configuration. Both the
cause motion of any kind, energy is required.
steel spring and the rubber band have the
A body is when it has
said to possess energy
ability to do work because of their tendency to
the capacity for doing work. Hence, energy is
return to their normal condition.
described as the ability to do work. The
The potential energy of a body may be evalu-
amount of energy required to do a given amount
ated by the following equation:
of work is always equal to the amount of work
done and the amount of energy a body possesses
is equal to the amount of work a body can do
P =M xZ (1-8)

in passing from one condition or position to where P = the potential energy in foot-pounds
another. M = the weight of the body in pounds
Energy may be possessed by a body in either Z = the vertical distance above some
or both of two basic kinds: (1) kinetic and (2) datum or reference
potential.
1-13. Kinetic Energy. Kinetic energy is the Example 1-10. Ten thousand gallons of
energy a body possesses as a result of its motion
water are stored in a tank located 250 ft above
the ground. Determine the potential energy of
or velocity. For instance, a hammer swinging
the water in relation to the ground.
through an arc, a bullet speeding toward a
target, and the moving parts of machinery all Solution. The
have kinetic energy by virtue of their motion. weight of the
The amount of kinetic energy a body possesses water in pounds 10,000 gal x 8.33 lb/gal
is a function of its mass and its velocity and may M 83,300 lb
be determined by the following equation: Applying
Equation 1-8,
Mx V 2
the potential 83,300 lb x 250 ft
energy P 20,825,000 ft-lb
8 PRINCIPLES OF REFRIGERATION
1-15. Energy as Stored Work. Before a energy, chemical energy, heat energy, etc., and
body can possess energy, work must be done on is readily converted from one form to another.
the body. The work which is done on a body Electrical energy, for instance, is converted into
to give the body its motion, position, or con- heat energy in an electric toaster, heater, or
figuration is stored in the body as energy. range. Electrical energy is converted into
Hence, energy is stored work. For instance, mechanical energy in electric motors, solenoids,
work must be done to stretch therubber band, to and other electrically operated mechanical
compress the steel spring, or to raise the driving devices. Mechanical energy, chemical energy,
head of a pile-driver to a position above the and heat energy are converted into electrical
piling. In any case, the potential energy stored energy in the generator, battery, and thermo-
is equal to the work done. couple, respectively. Chemical energy is con-
The amount of energy a body possesses can be verted into heat energy in chemical reactions
ascertainedby determining the amount of work such as combustion and oxidation. These are
done on the body to give the body its motion, only a few of the countless ways in which the
position, or configuration. For example, assume transformation of energy can and does occur.
that the driving head of a pile-driver weighing There are many fundamental relationships
200 lb is raised to a position 6 ft above the top which exist between the various forms of energy
of a piling. The work done in raising the driving and their transformation, some of which are of
head is 1200 ft-lb (200 lb x 6 ft). Therefore, particular importance in the study of refrigera-
1200 ft-lb of energy are stored in the driving- tion and are discussed in detail later.
head in its raised position and, when released,
neglecting friction, the driving-head will do PROBLEMS
1200 ft-lb of work on the piling.
1-16. Total External Energy. The total 1. The cooling tower on the roof of a building
weighs 1360 lb when filled with water. If the
external energy of a body is the sum of its
basin of the tower measures 4 ft by 5 ft, what
kinetic and potential energies.
is the pressure exerted on the roof
Example l-l I. Determine the total external (a) in pounds per square foot? Ans. 68 psf.
energy of an airplane weighing 10,000 lb and (£) in pounds per square inch?
flying 6000 ft above the ground at a speed of Ans. 0.472 psi.
300 mph. 2. If theatmospheric pressure is normal at sea
Apply-
Solution. leveland a gage on an R-12 condenser reads
ing Equation 1-7, 130 psi, what is the absolute pressure of the
the kinetic energy 10,000 lb x (440 fps), Freon in the condenser in pounds per square
K 2 x 32.174 ft/sec* inch? Ans. 144.7 psia.
= 30,086,436 ft-lb 3. What isthe total force exerted on the top- of
Applying a piston if the area of the cylinder bore is 5
Equation 1-8, the 10,000 lb x 6000 ft sq in. and the pressure of the gas in the cylinder
potential energy P - 60,000,000 ft-lb islSQpsi? Ans. 7501b.
Adding, the 4. A
barometer reads 10 in. Hg. What is the
total external atmospheric pressure in psi? Ans. 4.91 psi.
energy = 90,086,436 ft-lb 5. Abarometer on the wall reads 29.6 in. Hg
1-17. Law of Conservation of Energy. The while a gage on the tank of an air compressor
indicates 105 psi. What is the absolute pressure
First Law of Thermodynamics states in effect
of the air in the tank in pounds per square foot?
that the amount of energy is constant. None
Ans. 119.53 psia.
can be either created or destroyed. Energy is
expended only in the sense that it is converted
6. Agage on the suction inlet of a compressor
reads 10 in. Hg. Determine the absolute pres-
from one form to another.
sure of the suction vapor in psi. Ans. 9.79 psia.
1-18. Forms of Energy* All energy can be
classified as being of either of the two basic
7. A
gage on the suction side of a refrigeration
compressor reads 5 in. Hg. If a gage on the
kinds, kinetic or potential. However, energy discharge side of the compressor reads 122 psi,
may appear in any one of a number of different what is the increase in pressure during the
forms, such as mechanical energy, electrical compression? Ans. 124.46 psi.
PRESSURE, WORK, POWER, ENERGY 9

8. An electric motor weighing 236 lb is hoisted a mean distance of 135 ft above the
to the roof of a building in 2 min. If the roof ground? Ans. 8,996,400 ft-lb
is 125 ft above the ground,
12. Water in a river 800 ft above sea level is
(a) How much work is done? flowing at the rate of 5 mph. Calculate the sum
Ans. 29,500 ft-lb of the kinetic and potential energies per pound
(b) Neglecting friction and other losses, what of water in reference to sea level.
is tiie horsepower required? Ans. 800.84 ft-lb
Ans. 0.447 hp
13. A water pump delivers 60 gal per minute
9. Compute the kinetic energy of an automobile of water to a water tank located 100 ft above
weighing 3000 lb and moving at a speed of the level of the pump. If water weighs 8.33 lb
75 mph. Ans. 567,188 ft-lb per gallon and if the friction of the pipe and
10. What is the total external energy of the other losses are neglected,
automobile in Problem 9 if the automobile is (a) How much work is done?
traveling along a highway 6000 ft above Ans. 49,980 ft-lb
sea level? Ans. 18,000,000 ft-lb (6) Compute the horsepower required.
11. What is the total potential energy of 8000 Ans. 1.5 hp.
gal of water confined in a tank and located
The molecule is the smallest, stable particle
of matter into which a particular substance can
be subdivided and still retain the identity of
the original substance. For example, a grain
of table salt (NaCl) may be broken down into
individual molecules and each molecule will
be a molecule of salt, the original substance.
However, all molecules are made up of atoms,

2 so that it is possible to further subdivide a


molecule of salt into its component atoms. But,
a molecule of salt is made up of one atom of
sodium and one atom of chlorine. Hence, if a
Matter, Internal molecule of salt is divided into its atoms, the
atoms will not be atoms of salt, the original

Energy, Heat, substance, but atoms of two entirely different


substances, one of sodium and one of chlorine.

Temperature There are some substances whose molecules


are made up of only one kind of atoms. The
molecule of oxygen (0 2), for instance, is
composed of two atoms of oxygen. If a mole-
cule of oxygen is divided into its two component
atoms, each atom will be an atom of oxygen,
2-1. Heat. Heat is a form of energy. This the original substance, but the atoms of oxygen
is evident from the fact that heat can be con- will not be stable in this condition. They will

verted into other forms of energy and that other not remain as free and separate atoms of oxygen,
forms of energy can be converted into heat. but, if permitted, will either join with atoms or
However, there is some confusion as to exactly molecules of another substance to form a new
what energy shall be termed heat energy. compound or rejoin each other to form again a
Popular usage has made the concept of heat as molecule of oxygen.
internal or molecular energy almost universally It is assumed that the molecules that make up

accepted. Because of this, referring to heat as a substance are held together by forces of
internal energy is almost unavoidable at times. mutual attraction known as cohesion. These
On the other hand, from a strictly thermo- forces of attraction that the molecules have for
dynamic point of view, heat is denned as energy each other may be likened to the attraction that
in transition from one body to another as a exists between unlike electrical charges or

result of a difference in temperature between the between unlike magnetic poles. However,
two bodies. Under this concept, all other despite the mutual attraction that exists between
energy transfers occur as work. Both these the molecules and the resulting influence that
concepts of heat will evolve in this and the each molecule has upon the others, the mole-
following chapters. The term heat will be used cules are not tightly packed together. There is a
book in either sense.
hereafter in this certain amount of space between them and they
2-2. Matter and Molecules. Everything in are relatively free to move about. The mole-
the universe that has weight or occupies space, cules are further assumed to be in a state of

all matter, is composed of molecules. Mole- rapid and constant vibration or motion, the
cules, in turn, are made up of smaller particles rate and extent of the vibration or movement
called atoms and atoms are composed of still being determined by the amount of energy
smaller particles known as electrons, protons, they possess.
neutrons, etc. The study of atoms and sub- 2-3. Internal Energy. It has been pre-

atomic particles beyond the scope of this


is viously stated that energy is required to do work
book and the discussion will be limited for the or to cause motion of any kind. Molecules,
most part to the study of molecules and their like everything else, can move about only if they

behavior. possess energy. Hence, a body has internal


10
1

MATTER, INTERNAL ENERGY, HEAT, TEMPERATURE 1

energy as well as external energy. Whereas 2-5. States of Matter. Matter can exist in
a body has external mechanical energy because three different phases or states of aggregation:
of its velocity, position, or configuration in solid, liquid, or a vapor or gas. For example,
relation to some reference condition, it also has water is a liquid, but this same substance can
internal energy as a result of the velocity, exist as ice, which is a solid, or as steam, which
position, and configuration of the molecules of is a vapor or gas.
the materials which make up the body. 2-6. The Effect of Heat on the State of
The molecules of any material may possess Aggregation. Many materials, under the
energy in both kinds, kinetic and potential. proper conditions of pressure and temperature,
The total internal energy of a material is the can exist in any and all of the three physical
sum of its internal kinetic and potential energies. states of matter. It will be shown presently
This relationship is shown by the equation that the amount of energy the molecules of the
material have determines not only the tem-
U =K +P (2-1)
perature of the material but also which of the
where U = the total internal energy three physical states the material will assume at
K = the internal kinetic energy any particular time. In other words, the addi-
P = the internal potential energy tion or removal of heat can bring about a change
in the physical state of the material as well as a
2-4. Internal Kinetic Energy. Internal kinetic
change in its temperature.
energy is the energy of molecular motion or
That heat can bring about a change in the
velocity. When heat energy flowing into a
physical state of a material is evident from the
material increases the internal kinetic energy,
fact that many materials, such as metals will
the velocity or motion of the molecules is

increased. The increase in molecular velocity is


become molten when sufficient heat is applied.
Furthermore, the phenomenon of melting ice
always accompanied by an increase in the
temperature of the material. Hence, a material's
and boiling water is familiar to everyone. Each
of these changes in the physical state is brought
temperature is, in a sense, a measure of the
average velocity of the molecules which make
about by the addition of heat.
2-7. Internal Potential Energy. Internal
up the material. The more kinetic energy the
potential energy is the energy of molecular
molecules have, the greater is their movement
separation or configuration. It is the energy the
and the faster they move. The more rapid the
molecules have as a result of their position in
motion of the molecules, the hotter is the
material and the more internal kinetic energy
relation to one another. The greater the degree
of molecular separation, the greater is the inter-
the material has. It follows, then, that if the
nal potential energy.
internal kinetic energy of the material is di-
minished by the removal of heat, the motion of
When a material expands or changes its

physical state with the addition of energy, a


the molecules will be slowed down or retarded
and the temperature of the material rearrangement of the molecules takes place
will be
decreased.
which increases the distance between them.
According to the kinetic theory if the removal Inasmuch as the molecules are attracted to one
another by forces which tend to pull them to-
of heat continues until the internal kinetic
energy of the material is reduced to zero, the gether, internal work must be done in order to
separate further the molecules against their
temperature of the material will drop to Abso-
lute Zero (approximately —460° 10 and the
attractive forces. An amount of energy equal to
motion of the molecules will cease entirely.* the amount of internal work done must flow
into the material. This energy is set up in the
* It isnow known that the energy is not zero at
material as an increase in the internal potential
Absolute Zero. It is the disorganization (entropy)
which diminishes to zero. Heat is sometimes
energy. It is "stored" energy which is
defined as "disorganized energy." Both the energy accounted for by the increase in the mean dis-
and the disorganization decrease as the temperature tance between the molecules. The source of
decreases. However, the disorganization decreases this energy is the heat energy supplied.
faster than the energy and therefore diminishes It is important to understand that in this
to zero before the energy reaches zero. instance the energy flowing into the material
12 PRINCIPLES OF REFRIGERATION

has no effect on molecular velocity (internal 2-11. Temperature. Temperature is a prop-


kinetic energy); only the degree of molecular erty of matter. a measure of the level of
It is

separation (the internal potential energy) is heat intensity or the thermal pressure of a body.
affected. A high temperature indicates a high level of heat
2-8. The Solid State. A material in the solid intensity or thermal pressure, and the body is

state has a relatively small amount of internal said to be hot. Likewise, a low temperature
potential energy. The molecules of the material indicates a low level of heat intensity or thermal
are rather closely bound together by each other's pressure and the body is said to be cold.
attractive forces and by the force of gravity. 2-12. Thermometers. The most frequently
Hence, a material in the solid state has a rather used instrument for measuring temperature is
rigid molecular structure in which the position the thermometer. The operation of most ther-
of each molecule is more or less fixed and the mometers depends upon the property of a liquid
motion of the molecules is limited to a vibratory to expand or contract as its temperature is

type of movement which, depending upon the increased or decreased, respectively. Because
amount of internal kinetic energy the molecules of their low freezing temperatures and relatively
possess, may be either slow or rapid. constant coefficients of expansion, alcohol and
Because of its rigid molecular structure, a mercury are the liquids most frequently used in
solid tends to retain both its size and its shape. thermometers. The mercury thermometer is the
A solid is not compressible and will offer more accurate of the two because its coefficient
considerable resistance to any effort to change of expansion is more constant through a greater
its shape. temperature range than is that of alcohol.
M. The Liquid The molecules of a
State. However, mercury thermometers have the dis-
material in the liquid state have more energy advantage of being more expensive and more
than those of a material in the solid state and difficult to read. Alcohol is cheaper and can be

they are not so closely bound together. Their colored for easy visibility.

greater energy allows them to overcome each Two temperature scales are in common use
other's attractive forces to some extent and to today. The Fahrenheit scale is used in English
have more freedom to move about. They are speaking countries, whereas the Centigrade
free to move over and about one another in scale is widely used in European countries as
such a way that the material is said to "flow." well as for scientific purposes.
Although a liquid is noncompressible and will 2-13. Centigrade Scale. The point at which
retain because of its fluid molecular
its size, water freezes under atmospheric pressure is
structure, it will not retain its shape, but taken as the arbitrary zero point on the Centi-
will assume the shape of any containing grade and the point at which water boils
scale,

vessel. is The distance on the scale


designated as 100.
2-10. The Vapor or Gaseous State. The between these two points is divided into one
molecules of a material in the gaseous state have hundred equal units called degrees, so that the
an even greater amount of energy than those distance between the freezing and boiling points
of a material in the liquid state. They have of water on the Centigrade scale is 100°. Water
sufficient energy to overcome all restraining freezes at 0° Centigrade and boils at 100° Centi-
forces. They are no longer bound by each grade.
other's attractive forces, neither are they bound 2-14. Fahrenheit Scale. Although there is

by the force of gravity. Consequently, they some disagreement as to the actual method used
fly about at high velocities, continually collid- by Fahrenheit in designing the first temperature
ing with each other and with the walls of the scale, it was arrived at by means similar to those
container. For this reason, a gas will retain described in the previous section. On the
neither nor its shape. It is readily com-
its size Fahrenheit scale, the point at which water
pressible and will completely fill any container freezes is marked as 32, and the point at which
regardless of size. Further, if the gas is not water boils 212. Thus, there are 180 units
stored in a sealed container, it will escape from between the freezing and boiling points of
the container and be diffused into the surround- water. The zero or reference point on the
ing air. Fahrenheit scale is placed 32 units or degrees
MATTER, INTERNAL ENERGY, HEAT, TEMPERATURE 13

below the freezing point of water and is assumed Boiling point of water
212* 100*
to represent the lowest temperature Fahrenheit
could achieve with a mixture of ammonium
chloride and snow.
2-15. Temperature Conversion. Temper-
ature readings on one
can be converted to
scale Freezing point of water
32* 0*
reading on the other scale by using the appro- 0* -17.8*
priate of the following equations: Scales coincide
-40" -40*
° F = 9/5° C + 32 (2-2)

°
C= 5/9(° F - 32) (2-3)

It should be noted that the difference between


the freezing and boiling points of water on the
Fahrenheit scale is 180°, whereas the difference
between these two points on the Centigrade
scale is only 100°. Therefore, 100 Centigrade
degrees are equivalent to 180 Fahrenheit
degrees. This establishes a relationship such
that 1° C equals 9/5° F (1.8° F) and1° F equals
Absolute zero
5/9° C (0.555° C). This is shown graphically -460* -273'
in Fig. 2-1. Since 0° on the Fahrenheit scale is

32° F below the freezing point of water, it is Fig. 2-1. Comparison of Fahrenheit and Centigrade
necessary to add 32° F to the Fahrenheit equiva- temperature scales.

lent after converting from Centigrade. Like-


wise,it is necessary to subtract 32° F from a

Fahrenheit reading before converting to Centi- or Centigrade scales are in respect to arbitrarily

grade. selected zero points which, as has been shown,


are not even die same for the two scales. When
Example 2-1. Convert a temperature read-
it is desired to know only the change in tem-
ing of 50° C to the equivalent Fahrenheit
perature that occurs during a process or the
temperature.
temperature of a substance in relation to some
Solution. Applying known reference point, such readings are
Equation 2-2, ° F 9/5(50° Q + 32 entirely adequate. However, when temperature
= 122° F
readings are to be applied in equations dealing
Example 2-2. A thermometer on the wall with certain fundamental laws, it is necessary
of a room reads 86° F. What is the room to use temperature readings whose reference
temperature in degrees Centigrade? point is the true or absolute zero of tempera-
ture. Experiment has indicated that such a
Solution. Applying Formula
point, known as Absolute Zero, exists at approxi-
2-3, the room temperature 5/9(86-32)
in ° C = 30° C mately -460° F or -273° C (Fig. 2-1).
Temperature readings in reference to Abso-
Example 2-3. A thermometer indicates
lute Zero are designated as absolute tempera-
that the temperature of a certain quantity of
tures and may be in either Fahrenheit or
water is increased 45° F by the addition of
heat. Compute the temperature rise in Centi-
Centigrade degrees. A temperature reading on
the Fahrenheit scale can be converted to
grade degrees.
absolute temperature by adding 460° to the
Solution. Temperature rise in ° F Fahrenheit reading. The resulting temperature
= 45°F is in degrees Rankine (° R).
Temperature rise in ° C 5/9(45° F) Likewise, Centigrade temperatures can be
= 25°C
converted to absolute temperatures by adding
2-16. Absolute Temperature. Tempera- 273° to the Centigrade reading. The resulting
ture readings taken from either the Fahrenheit temperature is stated in degrees Kelvin (° K).
14 PRINCIPLES OF REFRIGERATION
In converting to and from absolute tempera- ture of the two bodies is the same, there is no
tures, the following equations will apply: transfer of heat.
Heat always flows down the temperature
T= t+460 (2-4)
scale from a high temperature to a low tem-
t = r-460 (2-5) perature, from a hot body to a cold body, and
never in the opposite direction. Since heat is
T « / + 273 (2-6)
energy and cannot be destroyed, heat
if is to
/ - T - 273 (2-7) leave one body of material, it must flow into
and be absorbed by another body of material
where T= absolute temperature in degrees whose temperature is below that of the body
Rankine or Kelvin being cooled.
t — temperature in degrees Fahrenheit The
rate of heat transfer between two bodies
or Centigrade is always directly proportional to the difference
Equations 2-4 and 2-5 apply to the Rankine and in temperature between the two bodies.
Fahrenheit scales, whereas Equations 2-6 and Methods of Heat Transfer.
2-18. The trans-
2-7 apply to the Kelvin and Centigrade scales. fer of heat from one place to another occurs
Hereafter in this book Rankine and Fahrenheit in three ways: (1) conduction, (2) convection,
temperatures are used unless otherwise specified. and (3) radiation.
2-19. Conduction. Heat transfer by con-
Example 2-4. A thermometer on the tank
of an air compressor indicates that the tempera- duction occurs when energy is transmitted by
ture of the air in the tank is 95° F. Determine direct contact between the molecules of a single
the absolute temperature in degrees Rankine. body or between the molecules of two or more
bodies in good thermal contact with each other.
Solution. Applying In either case, the heated molecules communi-
Equation 2-4, T- 95° F + 460°
cate their energy to the other molecules im-
= 555° R mediately adjacent to them. The transfer of
Example 2-5. The temperature of the energy from molecule to molecule by conduction
vapor entering the suction of a refrigeration is similar to that which takes place between the
compressor is -20° F. Compute the tempera- balls on a billiard table, wherein all or some
ture of the vapor in degrees Rankine. part of the energy of motion of one ball is trans-
mitted at the moment of impact to the other
Solution. Applying
balls that are struck.
Equation 2-4, T = -20° F + 460°
= 440°R When one end of a metal rod
heated over a
is

flame, some of the heat energy from


the heated
Example 24. If the temperature of a gas end of the rod will flow by conduction from
is 100° C, what is its temperature in degrees molecule to molecule through the rod to the
Kelvin? cooler end. As the molecules at the heated end
of the rod absorb energy from the flame, their
Solution. Applying
energy increases and they move faster and
Equation 2-6, T= 100° C+ 273°
through a greater distance. The increased
= 373° K
energy of the heated molecules causes them to
Example 2-7. The temperature of steam strike against the molecules immediately ad-
leaving a boiler is 610° R. What is the tempera- jacent to them. At the time of impact and
ture of the steam on the Fahrenheit scale? because of it, the faster moving molecules trans-
mit some of their energy to their slower moving
Solution. Applying
neighbors so that they too begin to move more
Equation 2-5, / = 610° R - 460° rapidly. In this manner, energy passes from
= 150° F
molecule to molecule from the heated end of the
2-17. Direction and Rate of Heat Flow. rod to the cooler end. However, in no case
Heat will flow from one body to another when, would it be possible for the molecules furthest
and only when, a difference in temperature from the heat source to have more energy than
exists between the two bodies. If the tempera- those at the heated end.
MATTER, INTERNAL ENERGY, HEAT, TEMPERATURE IS

As heat passes through the metal rod, the air Cooler portions of water descend to
replace the lighter portions that rise
immediately surrounding the rod is also heated gas^fgje^*- ^^

by conduction. The rapidly vibrating particles


of the heated rod strike against the molecules
of the air which are in contact with the rod.
The energy so imparted to the air molecules
causes them to move about at a higher rate and
communicate their energy to other nearby air
molecules. Thus, some of the heat supplied to
the metal rod is conducted to and carried away
by the surrounding air.
Heat is conducted
If the heat supply to the rod is interrupted, from flame to
Flame
heat will continue to be carried away from the water through
bottom of vessel
rod by the air surrounding until the tempera-
ture of the rod drops to that of the air. When
Heated portions of water become
this occurs, there will be no temperature differ-
lighter and rise toward surface,
ential, the system will be in equilibrium, and thereby distributing the heat
no heat will be transferred. throughout the entire mass

The rate of heat transfer by conduction, as


Fig. 2-2. Convection currents set up in a vessel of
previously stated, is in direct proportion to the
water when the vessel is heated at bottom center.
difference in temperature between the high and
low temperature parts. However, all materials

rate. Some When any portion of a fluid is heated, it ex-


do not conduct heat at the same
materials, such as metals, conduct heat very pands and itsvolume per unit of weight
readily, whereas others, such as glass, wood, and increases. Thus, the heated portion becomes
the top, and is immediately
lighter, rises to
cork, offer considerable resistance to the con-
duction of heat. Therefore, for any given tem- replaced by a cooler, heavier portion of the

perature difference, the rate of heat flow by fluid. For example, assume that a tank of
conduction through different materials of the water heated on the bottom at the center
is

same length and cross section will vary with the (Fig. 2-2). The heat from the flame is conducted

particular ability of the various materials to through the metal bottom of the tank to the
conduct heat. The relative capacity of a material water inside. As the water adjacent to the heat
to conduct heat is known as its conductivity. source absorbs heat, its temperature increases
Materials which are good conductors of heat and it expands. The heated portion of the
have a high conductivity, whereas materials water, being lighter than the water surrounding,
rises to the top and replaced by cooler, more
which are poor conductors have a low con- is

ductivity and are used as heat insulators. dense water pushing in from the sides. As this
In general, solids are better conductors of new portion of water becomes heated, it too
heat than liquids, and liquids are better con- rises to the top and is replaced by cooler water
from the As this sequence continues, the
ductors than gases. This is accounted for by sides.
heat distributed throughout the entire mass
the difference in the molecular structure. Since is

the molecules of a gas are widely separated, the of the water by means of the convection currents
transfer of heat by conduction, that is, from established within the mass.

molecule to molecule, is difficult. Warm air currents, such as those which occur
2-20. Convection. Heat by con-
transfer over stoves and other hot bodies, are familiar to
vection occurs when heat moves from one place everyone. How convection currents are utilized

to another by means of currents which are set to carry heat to all parts of a heated space is

up within some fluid medium. These currents illustrated in Fig. 2-3.

are known as convection currents and result


2-21. Radiation. Heat transfer by radiation
from the change in density which is brought occurs in the form of a wave motion similar to
about by the expansion of the heated portion of light waves wherein the energy is transmitted

the fluid. from one body to another without the need for
16 PRINCIPLES OF REFRIGERATION

exists,the energy exchange is in equilibrium and


the body neither gains nor loses energy.
Heat transfer through a vacuum is impossible
by either conduction or convection, since these
processes by their very nature require that
matter be the transmitting media. Radiant
energy, on the other hand, is not dependent
upon matter as a medium of transfer and there-
fore can be transmitted through a vacuum.
Furthermore, when radiant energy is transferred
from a hot body to a cold body through some
Steam coils' intervening media such as air the temperature of
Fig. 2-3. Room heated by natural convection. the intervening media is unaffected by the
passage of the radiant energy. For example,
heat is radiated from a "warm" wall to a "cold"
intervening matter. Heat energy transmitted wall through the intervening air without having
by wave motion is called radiant energy. any appreciable effect upon the temperature of
It is assumed that the molecules of a body are the air. Since the molecules of the air are rela-
in rapid vibration and that this vibration sets tivelyfew and widely separated, the waves of
up a wave motion in the ether surrounding the radiant energy can easily pass between them so
body.* Thus, the internal molecular energy of that only a very small part of the radiant energy
thebody is converted into radiant energy waves. isintercepted and absorbed by the molecules of
When these energy waves are intercepted by the air. By far the greater portion of the radiant
another body of matter, they are absorbed by energy impinges upon and is absorbed by the
that body and are converted into its internal solid wall whose molecular structure is much
molecular energy. more compact and substantial.
The
earth receives heat from the sun by Heat waves are very similar to light waves,
radiation. The energy of the sun's molecular differing from them only in length and frequency.
vibration is imparted in the form of radiant Light waves are radiant energy waves of such
energy waves to the ether of interstellar space sur- length as to be visible to the human eye. Thus,
rounding the sun. The energy waves travel across light waves are visible heat waves. Whether
billions of miles of space and impress their heat waves are visible or invisible depends upon
energy upon the earth and upon any other the temperature of the radiating body. For
material bodies which intercept their path. The example, when metal is heated to a sufficiently
radiant energy is absorbed and transformed high temperature, it will "glow," that is, emit
into internal molecular energy, so that the visible heat waves (light).
vibratory motion of the hot body (the sun) is When radiant energy waves, either visible or
reproduced in the cooler body (the earth). invisible, strike a material body, they may be
All materials give off and absorb heat in the reflected, refracted, or absorbed by it, or they
form of radiant energy. Any time the tempera- may pass through it to some other substance
ture of a body is greater than that of its sur- beyond.
roundings, it will give off more heat by radiation The amount of radiant energy which will pass
than it absorbs. Therefore, it loses energy to through a material depends upon the degree of
its surroundings and its internal energy de- transparency. A highly transparent material,
creases. If the temperature of the body is below such as clear glass or air, will allow most of the
that of its surroundings, it absorbs more radiant radiant energy to pass through to the materials
energy than it loses and its internal energy beyond, whereas opaque materials, such as
increases. When no temperature difference wood, metal, cork, etc., cannot be penetrated
* Ether is the name
given to that which fills all by radiant energy waves and none will pass
space unoccupied by matter, such as interstellar through.
space and the space between the molecules of every The amount of radiant energy which is either
material. reflected or absorbed by a material depends
MATTER, INTERNAL ENERGY, HEAT. TEMPERATURE 17

upon the nature of the material's surface, that change the temperature of 1 pound of the
texture and its color. Materials having a
is, its material 1° F. For instance, the specific heat of
light-colored, highly polished surface, such as a aluminum is 0.226 Btu/lb/°F, whereas that of
mirror, reflect a maximum of radiant energy, brass is 0.089 Btu/lb/°F. This means that 0.226
whereas materials having rough, dull, dark Btu is required to raise the temperature of 1
surfaces will absorb the maximum amount of pound of aluminum 1° F, whereas only 0.089
radiant energy. Btu is necessary to change the temperature of
2-22. British Thermal Unit. It has already 1 pound of brass 1° F. Note that by the defini-
been established that a thermometer measures tion of the Btu the specific heat of water is 1
only the intensity of heat and not the quantity. Btu per pound per degree Fahrenheit.
However, in working with heat it is often The specific heat of any material, like that of
necessary to determine heat quantities. Obvi- water, varies somewhat throughout the tem-
ously, some unit of heat measure is required. perature scale. Here again, the variation is so
Heat is a form of energy, and as such is slight that it is sufficiently accurate for most
intangible and cannot be measured directly. calculations to consider the specific heat to be
Heat can be measured only by measuring the a constant amount. This not true, however,
is
effects it has on a material, such as the change in as the material passes through a change in
temperature, state, color, size, etc. physical state. The specific heat of a material
The most universally used unit of heat measure is approximately one-half that
in the solid state
is the British thermal unit, abbreviated Btu. A of the same -material in the liquid state. For
Btu is denned as the quantity of heat required to instance, the specific heat of ice is 0.S Btu,
change the temperature of 1 lb of water 1° F. whereas that of water is one. The specific heat
This quantity of heat, if added to 1 lb of water, values of materials in the gaseous state are dis-
will raise the temperature of the water 1°F. cussed in another chapter.
Likewise, if 1 Btu is removed from 1 lb of water, 2-24. Calculating Heat Quantity. The
the temperature of the water will be lowered quantity of heat which must be added to or
1°F. removed from any given mass of material in
The quantity of heat required to change the order to bring about a specified change in its
temperature of 1 lb of water 1° F
is not a con- temperature can be computed by using the
stant amount. with the tem-
It varies slightly following equation:
perature range at which the change occurs. For
this reason, a Btu is more accurately defined as Q, - MC(tt - t0 (2-8)
being l/180th of the quantity of heat required to
raise the temperature of 1 lb of water from the where Q, = the quantity of heat either absorbed
freezing point (32° F) to the boiling point or rejected by the material
(212° F). This is identified as the "mean Btu" M = the weight of the material in
and is the exact amount of heat required to raise pounds
the temperature of 1 lb of water from 62 to C = the specific heat of the material
63° F. If the change in temperature occurs at ?! = the initial temperature

any other point on the temperature scale, the tt = the final temperature

amount of heat involved is either more or less


than the mean Btu, depending upon the par- Example2-8. Twenty pounds of water at
ticular point on the temperature scale that the an temperature of 76° F are heated until
initial

change takes place. However, the variation the temperature is increased to 180° F. How
from the mean Btu is so slight that it may be much heat must be supplied?
neglected and, regardless of the temperature
Solution.
range, for all practical purposes it is sufficiently
Applying Q, - 20 lb x 1 x (180 - 76)
accurate to assume that the temperature of lb
1
Equation 2-8, = 2080 Btu
of water is changed 1° F by the addition or
removal of 1 Btu. Example 2-9. If water weighs 8.33 lb per
2-23. Specific Heat. The specific heat of a gallon, how much heat is rejected by 30 gal of
material is the quantity of heat required to water in cooling from 80° F to 35° F?
18 PRINCIPLES OF REFRIGERATION
Solution. accompanies a change in the temperature of the
Weight of material, the heat transferred is identified as
water in 30 gal x 8.33 lb/gal sensible heat. The term sensible is applied to
pounds = 250 lb this particular heat because the change in tem-
Applying Q, = 250 lb x 1 x (35 - 80)
perature it causes can be detected with the sense
Equation 2-8, = 250 lb x 1 x (-45)
of touch and can, of course, be measured with a
= 11,250 Btu
thermometer.
Note: Since the heat of a material is
specific
2-27. Latent Heat. When heat, either added
given in terms of Btu/lb/° F, the weight of the
to or rejected by a material, brings about or
material must be determined before Equation
2-8 can be applied. accompanies a change in the physical state of the
material, the heat is known as latent heat. The
Where than t l7 the answer obtained
tt is less
name latent, a Latin word meaning hidden, is
by applying Equation 2-8 will be negative,
said to have been given to this special kind of
indicating that heat is rejected by rather than
heat by Dr. Joseph Black because it apparently
absorbed by the material. In this type of
problem, where the direction of heat flow is disappeared into a material without having any
obvious, the negative sign can be ignored and effect on the temperature of the material.
the answer assumed to be positive. Many materials progressing up the tempera-
Example 2-10. ture scale will pass through two changes in the
pounds of cast iron
Fifteen
are cooled from 500° F to 250° F by being im- state of aggregation: first, from the solid to the
mersed in 3 gallons (25 lb) of water whose liquid phase and then, as the temperature of
initial temperature is 78° F. Assuming that the the liquid is further increased to a certain level
specific heat of the cast iron is 0.101 Btu/lb/° F beyond which it cannot exist as a liquid, the
and that all of the heat given up by the cast iron liquid will change into the vapor state. When
is absorbed by the water, what is the final the change occurs in either direction between
temperature of the water? the solid and liquid phases, the heat involved is
Solution. By known as the latent heat of fusion. When the
applying Equation change occurs between the liquid and vapor
2-8 to compute the
phases, the heat involved is the latent heat of
total quantity of heat Q, = 15 lb x 0.101
vaporization.
given up by the cast x 250
= 2-28. Sensible Heat of a Solid. To obtain a
iron, 378.75 Btu
better understanding of the concept of molecular
By rearranging and
applying Equation 2-8 h MC + 'i energy, consider the progressive effects of heat
as it is taken in by a material whose initial
to determine the final
378.75
temperature of the + 78°F thermodynamic condition is such that its energy
25 x 1
water after absorbing content is zero. Assume that a solid in an open
the heat given up by = 15.15° +78 container is at a temperature of — 460° F
the cast iron, = 93°F (Absolute Zero). Theoretically, at this tem-
2-25. Heat Divided into Two Kinds or perature the molecules of the material have no
Categories. has been previously stated
It energy and are completely at rest.

(Section 2-6) that heat has the ability to bring When heat energy flows into the solid, the
about a change in the physical state of a material molecules Of the solid begin to move slowly and
as well as the ability to cause a change in its the temperature of the solid begins to climb.
temperature. Heat is divided into two kinds or The more heat energy taken in by the solid, the
categories, depending upon which of these two faster the molecules vibrate and the warmer the
effects it has on a material which either absorbs solid becomes. The increase in molecular velo-
or rejects The division of heat into several
it. cityand in the temperature of the solid continues
classifications is made only to facilitate and as more heat is absorbed, until the solid reaches
simplify certain necessary calculations and does its melting or fusion temperature. The total
not stem from any difference in the nature of quantity of heat energy required to bring the
heat itself. temperature of the solid from the original con-
2-26. Sensible Heat. When heat either ab- dition of Absolute Zero to the melting or fusion
sorbed or rejected by a material causes or temperature is known as the sensible heat of the
MATTER, INTERNAL ENERGY, HEAT, TEMPERATURE 19

solid. As previously shown, the quantity of heat It is important at this point to emphasize that
which must be transferred in order to bring the change of phase occurs in either direction at
about a specified change in the temperature of the fusion temperature, that is, the temperature
any given mass of any material can be calculated at which the solid will melt into the liquid phase
by applying Equation 2-8. is the same as which the liquid will
that at
2-29. The Melting or Fusion Temperature. freeze into the solid phase. Further, the quan-
Upon reaching the fusion temperature, the tity of heat that must be rejected by a certain

molecules of the solid are moving as rapidly as weight of liquid at the fusion temperature in
is possible within the rigid molecular structure order to freeze into the solid state is exactly
of the solid state. not possible to increase
It is equal to the amount of heat that must "be ab-
further the motion of the molecules or the tem- sorbed by the same weight of the solid in melting
perature of the solid beyond this point without into the liquid state.
firstovercoming partially the forces of mutual None of the heat absorbed or rejected during
attraction which exists between the molecules. the change of phase has any effect on molecular
Hence, the material cannot exist in the solid velocity. Therefore the temperature of the
state at any temperature above its melting or material remains constant during the phase
fusion temperature. On reaching the fusion change, and the temperature of the resulting
temperature, any additional heat absorbed by liquid or solid is the same as the fusion tem-
the material will cause some part of the solid perature.*
to revert to the liquid phase. The quantity of heat that is absorbed by a
The exact temperature at which melting or given weight of a solid at the fusion temperature
fusion occurs varies with the different materials in melting into the liquid phase, or, conversely,
and with the pressure. For instance, at normal the quantity of heat that is rejected by a given
atmospheric pressure, the fusion temperature of weight of liquid at the fusion temperature in
lead is approximately 600° F, whereas copper freezing or solidifying, can be determined by
melts at approximately 2000° F and ice at only applying the following equation:
32° F. In general, the melting temperature
decreases as the pressure increases except for
QL = Mxh it (2-9)

noncrystalline solids, whose melting tempera- where Ql = the quantity of heat in Btu
tures increase as the pressure increases. M= the mass or weight in pounds
2-30. Latent Heat of Fusion. When heat h it = the latent heat in Btu per pound
is absorbed by a solid at the fusion temperature, Example 2-11. Calculate the quantity of
the molecules of the solid utilize the energy to heat required to melt 12 lb of ice at 32° F into
overcome partially their attraction for one water at 32° F. The latent heat of fusion of
another. They break away from one another to water under atmospheric pressure is 144 Btu
some extent and become more widely separated. per pound.
As the molecules flow over and about one
Solution. Apply-
another, the material loses the rigidity of the
ing Equation 2-9, the
solid state and becomes fluid. It can no longer quantity of heat re-
support itself independently and will assume the quired to melt 12 lb 12 lb x 144 Btu/lb
shape of any containing vessel. of ice = 1728 Btu
The attraction which exists between the mole- Note. Since 12 lb of ice absorb 1728 Btu in
cules of a solid is considerable and a relatively
melting into water, it follows that 12 lb of water
large quantity of energy is required to overcome at 32° F will reject 1728 Btu in returning to the
that attraction. The quantity of heat required solid state.
to melt one pound of a material from the solid
* This applies with absolute accuracy only to
phase into the liquid phase is called the latent
crystalline solids. Noncrystalline solids, such as
heat of fusion. The latent heat of fusion, along glass, have indefinite fusion temperatures. That
with other values such as specific heat, fusion is, the temperature will vary during the change of
temperature, etc., for the different materials has phase. However, for the purpose of calculating heat
been determined by experiment and may be quantities, the temperature is assumed to remain
found in various tables. constant during the phase change.
20 PRINCIPLES OF REFRIGERATION
Example 2-12. If SO lb of ice at 32° F 2-33. Latent Heat of Vaporization. Any
absorb 6000 Btu, what part of the ice will be heat taken in by a liquid after the liquid reaches
melted? the saturation temperature is utilized to increase
Solution. By rearranging = _G the degree of molecular separation (increases the
and applying Equation 2-9, h if internal potential energy) and the substance
the part of the ice melted, M _
~
6000 Btu changes from the liquid to the vapor phase.*
144 Btu/lb There is no increase in molecular velocity and,
- 41.66 lb therefore, no change in the -internal kinetic
2-31. Sensible Heat of the Liquid. When a energy during the change in phase. Hence,
material passes from the solid to the liquid the temperature remains constant during the
phase, the resulting liquid is at the fusion tem- phase change and the vapor which results is at
perature. The temperature of the liquid may the vaporizing temperature.
then be increased by the addition of heat. Any As the material changes state from a liquid
heat absorbed by a liquid after the change of to a vapor, the molecules of the material acquire
state is set up in the liquid as an increase in the sufficient energy to overcome all restraining
internal kinetic Molecular velocity
energy. forces, including the force of gravity. The
increases and the temperature of the liquid rises. amount of energy required to do the internal
But here again, asin the case of the solid, the work necessary to overcome these restraining
temperature of the liquid eventually reaches a forces is very great. For this reason, the capacity
point beyond which it cannot be further in- of a material to absorb heat while undergoing
creased. A liquid cannot exist as a liquid at any a change from the liquid to the vapor phase is
temperature above its vaporizing temperature enormous, many times greater even than its
for a given pressure and, upon reaching the capacity to absorb heat in changing from the
vaporizing temperature, if additional heat is solid to the liquid phase.
taken in by the liquid, some part of the liquid The quantity of heat which 1 lb of a liquid
will change to the vapor phase. absorbs while changing into the vapor state is

The total quantity of heat taken in by a liquid known as the latent heat of vaporization. The
as its temperature is increased from the fusion latent heat of vaporization, like the saturation
to the vaporizing temperature is called the temperature, is each material. It
different for
sensible heat of the liquid. Here again, Equation will be shown both the latent heat
later that
2-8, sometimes known as the "sensible heat value and the saturation temperature of any
equation," can be applied to determine the particular liquid vary with the pressure over the
quantity of heat necessary to change the tem- liquid. When the pressure increases, the satur-
perature of any given weight of liquid through ation temperature increases and the latent heat
any specified temperature range. value decreases.
2-32. Saturation Temperature. The tem- The quantity of heat required to vaporize any
perature at which a liquid will change into the given weight of liquid at the saturation tem-
vapor phase is called the saturation temperature, perature is calculated by the following equation:
sometimes referred to as the "boiling point"
or "boiling temperature." A liquid whose tem- QL =M xh fa (2-10)

perature has been raised to the saturation tem- where Ql = the quantity of heat in Btu
perature is called a saturated liquid. M = the mass or weight -in pounds
The saturation temperature, that is, the tern' hfg = the latent heat of vaporization in
perature which vaporization occurs, is
at Btu/lb
different for each liquid. Iron, for example,
Example 2-13. of vapori-
If the latent heat
vaporizes at 4450° F, copper at 4250° F, and
zation of water is 970 Btu per pound, how
lead 3000° F. Water, of course, boils at 212° F,
* Some of the energy added to the material leaves
and alcohol at 170° F. Some liquids boil at
the material as external work and has no effect on
extremely low temperatures. few of these A the internal energy of the material. When the
are ammonia, oxygen, and helium, which boil pressure is constant, the amount of external work
at temperatures of -28° F, -295° F, and done is proportional to the change in volume.
—452° F, respectively. External work is discussed in detail later.
:

MATTER, INTERNAL ENERGY, HEAT, TEMPERATURE 21

much heat is requited to vaporize 3 gal of water material at any particular condition is the sum
at the saturation temperature of 212° F? total of all the sensible and latent heat required
to bring it to that condition from an initial
Solution. Total 3 gal x 8.33 lb/gal condition of Absolute Zero.*
weight of water M = 25 lb
Applying Equation 25 lb x 970 Btu/lb Example 2-16. Compute the total heat
= content of 1 lb of steam at 212° F.
2-10, QL 24,250 Btu

Solution. The total heat of 1 lb of saturated


Example 2-14. One gallon of water at
steam is the sum of the following heat quantities
200° F in an open container absorbs 1200 Btu.
(a) To raise the temperature of 1 lb of ice from
How much water is vaporized? -460° F to 32° F,
applying Equation = 1 x 0.5 x
Solution. Since the saturation temperature -
2-8, [32 (-460)]
of water at atmospheric pressure is 212° F, the = 1 x 0.5 x 492
entire mass of the water must be raised to this
= 246 Btu
temperature before any water will vaporize
(6) To melt 1 lb of ice at 32° F into water at
Weight of 1 gal of water = 8.33 lb 32° F,
Applying Equation 2-8, applying Equation 2-9, QL = 1 x 144
the heat required to raise = 144 Btu
the temperature of the 8.33 lb x 1
(c) To increase temperature of water from 32° F
water from 200° F to x 12°
to 212° F,
212° F, 0, = 100 Btu
=
applying Equation 1 x 1 x
2-8, (212 - 32)
Heat available to vapor-
= 1 x x 180
1200 - 100
1
ize some portion of the
= 180 Btu
water 1100 Btu
(d) To vaporize 1 lb of water,
1100
Rearranging and apply- applying Equation 2-10, QL = 1 x 970
ing Equation 2-10, the 970 = 970 Btu
weight of water vaporized, 1.135 lb or (e) Summation:
M 0.136 gal Sensible heat of the solid = 246 Btu
Latent heat of fusion = 144 Btu
Example 2-15. If 5000 Btu are removed Sensible heat of the liquid = 180 Btu
from 8 lb of saturated steam at atmospheric Latent heat of vaporization = 970 Btu
pressure, how much of the steam will condense Total heat of 1 lb
into water? of steam = 1540 Btu

Solution. By rearrang- 5000 Btu Through the use of a temperature-heat


ing and applying Equation M = 970 Btu/lb
diagram, the solution to Example 2-15 is shown
graphically in Fig. 2-4.
2-10, = 5.15 lb

2-36. Mechanical Energy Equivalent.


2-34.Superheat —the Sensible Heat of a
Normally the external energy of a body is ex-
Vapor. Once a liquid has been vaporized,
pressed in mechanical energy units (work),
the temperature of the resulting vapor can be
whereas the internal energy of a body is ex-
further increased by the addition of heat. The
pressed in heat energy units.
heat added to a vapor after vaporization is the
The fact that internal energy is usually ex-
sensible heat of the vapor, more commonly
pressed in heat energy units gives rise to the
called superheat. When the temperature of a definition of heat as molecular or internal energy.
vapor has been so increased above the satur-
As previously stated, from a thermodynamic
ation temperature, the vapor is said to be
superheated and is called a superheated vapor.
* The total heat of a material is commonly known
Superheated vapors are discussed at length in as "enthalpy," and is computed from some
another chapter. arbitrarily selected zero point rather than from
2-35. Total Heat. The total heat of a Absolute Zero. See Section 4-18.
22 PRINCIPLES OF REFRIGERATION

- .8
i
J'S E
m
=Ss"
8.
M *

#I~ I .3- 3i£Z c> 3~


C/5

300 7

r
Constant temperature
212
200

100

Constant temperature
32

A
-400

-460
F
J 1
J L I
J L _L J L J_
100 200 300 400 500 600 700 800 900 1000 1100 1200 1300 1400 1500 1600 1700

Heat content (Btu)

Fig. 2-4. Graphical analysis of the relationship of heat content to the temperature and state of a
material.

point of view energy is heat energy only when part of the mechanical energy of the hammer
it is in transition from one body to another blow converted to the internal kinetic energy
is

because of a difference in temperature between of the nail head. As the molecules of the metal
thetwo bodies. Once the energy flows into a that make upthe nail head are jarred and agi-
body it becomes "stored" thermal energy. tated by the blow of the hammer, their motion
Hence, thermodynamically speaking, internal or velocity is increased and the temperature of
energy is not heat but thermal energy in storage. the nail head increases. If a wire is bent rapidly
Not all the heat energy flowing into a body is back and forth, the bent portion of the wire
body as internal energy. In many
stored in the becomes hot because of the agitation of the
some or all of the energy flowing into
instances, molecules. Also, everyone is familiar with the
the body passes through or leaves the body as increase in temperature which is brought about

work (mechanical energy). This is made clear by the of two surfaces rubbing together.
friction
in another section. Often the external energy of a body is con-
Furthermore, up to this point it has been verted to internal energy and vice versa. For
assumed that the internal energy of a body is example, a bullet speeding toward a target has
increased only by the addition of heat energy kinetic energy because of its mass and velocity.
directly, as from a flame or some other heat At the time of impact with the target, the bullet
source. However, this is not the case. The losesits velocity and a part of its kinetic energy

internal or molecular energy of a body may also isimparted to the molecules of both the bullet
be increased when work is done on the body. and the target so that the internal energy of
That is, the mechanical energy of the work done each is increased.
on a body may be converted to the internal Since heat energyis often converted into

energy of the body. For example, the head of a mechanical energy (work) and vice versa, and
nail struck by a hammer will become warm as a since it is often desirable to express both the
MATTER, INTERNAL ENERGY, HEAT, TEMPERATURE 23

internal and external energies of a body in PROBLEMS


terms of the same energy unit, a factor which
can be used to convert from one energy unit 1. A Fahrenheit thermometer reads 85°. What
is the temperature in degrees Centigrade?
to the other is useful.
It has been determined by experiment that
Ans. 29.44° C
2. Convert 90° Centigrade to degrees Fahren-
one Btu of heat energy is equivalent to 778
heit. Ans. 194° F
ft-lb of mechanical energy, that is, one Btu is the
3. The temperature of a gas is 40° F. What is
amount of heat energy required to do 778 ft-lb
its temperature on the Rankine scale?
of work. This quantity known as the mecha-
is
Ans. 500° R
nical energy equivalent and is usually repre-
4. The temperature of the suction vapor enter-
sented in equations by the symbol J.
ing a refrigeration compressor is —20° F. What
To convert energy in Btu into energy in is the temperature of the vapor in degrees
foot-pounds, the energy in Btu is multiplied by Rankine? Ans. 440° R
778 and, to convert energy in foot-pounds to 5. Thirty gallons of water are heated from 75° F
energy in Btu, the energy in foot-pounds is to 180° F. Determine the quantity of heat
divided by 778. Expressed as equations, these required? Ans. 26,240 Btu
relationships become 6. In a certain industrial process, 5000 gal of
W
- -7
water are cooled from 90° F to 55° F each hour.
Q (2-11)
Determine the quantity of heat which must be
removed each hour to produce the required
and W = Q xJ (2-12) cooling. Ans. 1,457,750 Btu
7. Calculate the quantityof heat which must
where Q = the quantity of heat energy in Btu
be removed from 60 gal of water in order to
W = mechanical energy or work in foot- cool the water from 42° F and freeze it into
pounds Ans. 77,000 Btu
ice at 32° F.
J = the mechanical energy equivalent
8. If 12,120 Btu are added to 3 gal of water at
of heat 200° F, what fraction of the water in pounds
Example 2-17. Convert 36,000 ft-lb of will be vaporized? Ans. 9.4 lb or 1.13 gal
mechanical energy into heat energy units. 9. Twenty-five pounds of ice are placed in
10 gal of water and allowed to melt. Assuming
36,000
Solution. Applying Equa- Q = 778 that there is no loss of heat to the surroundings,
tion2-ll, if the initial temperature of the water is 80° F,
= 46.3 Btu to what temperature will the water be cooled
by the melting of the ice ? Ans. 35.7°
Example 2-18. Express 12 Btu of heat
energy as work in mechanical energy units. 10. Agas expanding in a cylinder does 25,000
ft-lbof work on the piston. Determine the
Solution. Applying W= 12 x 778 quantity of heat required to do the work.
Equation 2-12, = 9336 ft-lb Ans. 32.13 Btu
that, although the molecules of water are
actually closer together in the solid state than
they are in the liquid state, they are grouped
together to form crystals. It is the relatively
large spaces between the crystals of the solid,
rather than any increase in the mean distance
between the molecules, which accounts for the

3 unusual increase in volume during solidification.


This is true also for crystalline solids other than
ice.
3-2. Expansion of Solids and Liquids. When
Thermodynamic a solid or a liquid is heated so that its tempera-
ture is increased, it will expand a given amount
Processes for each degree of temperature rise. As stated
earlier, many temperature measuring devices
are based upon this principle. The amount of
expansion which a material experiences with
each degree of temperature rise is known as its
coefficient of expansion. The coefficient of
3-1. The Effects of Heat on Volume. When expansion is different for every material, and
either the velocity of the molecules or the degree moreover it will vary for any particular material
of molecular separation is increased by the depending upon the temperature range in
addition of heat, the mean distance between the which the change occurs.
molecules is increased and the material expands Since solids and liquids are not readily com-
so that a unit weight of the material occupies a pressible, if a solid or a liquid is restrained or
greater volume. This effect is in strict accord- confined so that volume is not allowed to
its
ance with the theory of increased or decreased change normally with a change in temperature,
molecular activity as described earlier. Hence, tremendous pressures are created within the
when heat isadded to or removed from an material itself and upon the restraining bodies,
unconfined material in any of the three physical which is likely to cause buckling or rupturing
states, it will expand or contract, respectively. of either the material, the restraining bodies, or
That is, its volume will increase or decrease with both. To provide for the normal expansion and
the addition or removal of heat. contraction occurring with temperature changes,
One of the few exceptions to this rule is water. expansion joints are built into highways,
If water is cooled, its volume will decrease bridges, pipelines, etc. Likewise, liquid con-
normally until the temperature of the water tainers are never completely filled.
Space must
drops to 39.2° F. At this point, water attains be allowed for the normal expansion. Other-
its maximum density and, if further cooled, its wise the tremendous expansive forces generated
volume will again increase. Furthermore, after by a temperature increase will cause the con-
being cooled to 32° F, it will solidify and the taining vessel to rupture, sometimes with
solidification will be
accompanied by still explosive force.
further expansion. In fact, 1 cu ft of water will 3-3. Specific Volume. The specific volume
freeze into approximately 1.085 cu ft of ice. of a material is the volume occupied by a 1 lb
This accounts for the tremendous expansive mass of the material. Each material has a
force created during solidification which is different specific volume and, because of the
sufficient to burst steel pipes or other restraining change in volume which accompanies a change
vessels. in temperature, the specific volume of every
The peculiar behavior of water as it solidifies material varies somewhat with the temperature
appears to contradict the general laws govern- range. For instance, at 40° F, 1 lb of water has
ing molecular activity as described previously. a specific volume of 0.01602 cu ft, whereas the
However, this is not the case. The unusual volume occupied by 1 lb of water at 80° F is
behavior of water is explained by the hypothesis 0.01608 cu ft.

24
THERMODYNAMIC PROCESSES 25

3-4. Density. The density of a material is the 3-5. Pressure-Temperature-Volume Rela-


weight in pounds of 1 cu ft of the material. tionships of Gases. Because of its loose
Density is die reciprocal of specific volume, that molecular structure, the change in the volume of
is, the specific volume divided into one. The a gas as the gas is heated or cooled is much
density of any material, like specific volume, greater than that which occurs in the case of a
varies with the temperature, but in the opposite solid or a In the following sections, it
liquid.
direction. For example, at 40° F, the density will be shown that a gas may change its con-
of water 62.434 lb per cubic foot (1/0.01602),
is dition in a number of different ways and that
whereas water at 80° F has a density of 62.20 certain laws have been formulated which govern
lb per cubic foot (1/0.01608). Since density and the relationship between the pressure, temper-
specific volume are reciprocals of each other, ature, and volume of the gas during these
as one increases the other decreases. The changes. It should be noted at the outset that
volume of many
density and/or the specific in applying the fundamental gas laws it is
common materials can be found in various always necessary to use absolute pressures and
tables. absolute temperatures in degrees Rankine.
The relationship between density and specific Further, in studying the following sections, it

volume is given by the following equations: should be remembered that a gas always
completely fills any container.
The relationship between the pressure,
p= ~v
(3-1)
temperature, and volume of a gas is more
easily understood when considered through a
1
V = - (3-2)
series of processes in which the gas passes
p from some initial condition to some final con-
dition in such a way that only two of these
V=M x v (3-3)
properties vary during any one process, whereas

M= V x P (3-4)
the third property remains unchanged or con-
stant.

where v = the specific volume in cubic feet per 3-6. Temperature-Volume Relationship at
pound a Constant Pressure. If a gas is heated
(cu ft/lb)
= the under such conditions that its pressure is kept
p density in pounds per cubic foot
(lb/cu ft)
constant, its volume will increase 1/492 of its

V = the total volume in cubic feet volume at 32° F for each 1° F increase in its

M = the total weight in pounds temperature. Likewise, if a gas


constant pressure, its volume will decrease 1/492
is cooled at a

Example If the specific volume of dry


3-1 . of its volume at 32° F for each 1 ° F decrease in
saturated steam at 212° F is 26.80 cuft per its temperature.
pound, what is the density of the steam? In order to better visualize a constant pressure
change in condition, assume that a gas is con-
Solution. Applying fined in a cylinder equipped with a perfectly
Equation 3-1, the density p 26.80 fitting, frictionless piston (Fig. 3-la). The
= 0.0373 lb/cu ft pressure of the gas which is exerted on the
is that
gas by the weight of the piston and by the weight
Example 3-2. The basin of a cooling tower, of the atmosphere on top of the piston. Since
measuring 5 ft x 4 ft x 1 ft, is filled with water. the piston is free to move up or down in the
If the density of the water is 26.8 lb per cubic
cylinder, the gas is allowed to expand or con-
foot, what is the total weight of the water in the
tract, that is, to change its volume in such a way
basin?
that the pressure of the gas remains constant.
Solution. The total volume V 5 ft x 4 ft As the gas is heated, its temperature and volume

x 1ft increase and the piston moves- upward in the


= 20 cu ft cylinder. As the gas is cooled, its temperature
Applying Equation 3-4, the 20 X 26.8 and volume decrease and the piston moves down-
total weight of water M - 536 lb ward in the cylinder. In either case, the pressure
26 PRINCIPLES OF REFRIGERATION

[tttl
Hi I i
Pistoh||

Perfectly fitting
frictionless
ft? '.
'Of/^'"""-"-"^
1
Volume
Volu
piston
change
chai

Piston
:

JL
?< P = 100 psia i£f P = 100 psia $
£fr = 500*R J& $v;T=1000*R 4.

&V=Icuft

fTTT
Heat Heat
added removed

(a) (b) (c)

Fig. 3-1. Constant pressure process, Gas confined In a cylinder with a perfectly fitting, frictionless piston,
(a)

(b) As gas is heated, both the temperature and the volume of the gas Increase. The increase in volume
is

exactly proportional to the increase in absolute temperature, (c) As gas is cooled, both the temperature and
the volume of the gas decrease. The decrease in volume is exactly proportional to the decrease in absolute
temperature.

of the gas remains the same or unchanged during When any three of the preceding values are
the heating or cooling processes. known, the fourth may be calculated by applying
3-7. Charles' Law for a Constant Pressure Equation 3-5.

Process. Charles' law for a constant pressure


Example A gas, whose initial tem-
3-3.
process states in effect that, when the pressure
perature is 520° R
and whose initial volume is
of the gas remains constant, the volume of the 5 cu ft, is allowed to expand at a constant
gas varies directly with its absolute temperature. pressure until its volume is 10 cu ft. Determine
Thus, if the absolute temperature of a gas is the final temperature of the gas in degrees
doubled while its pressure is kept constant, its Rankine.
volume will also be doubled. Likewise, if the Solution. By rearranging and
absolute temperature of a gas is reduced by applying Equation 3-5, the final
one-half while the pressure is kept constant, its temperature of the gas T2 520 x 10
volume will also be reduced by one-half. This
relationship is illustrated in Figs. 3-16 and = 1040°
3-lc.
Example 3-4. A
gas, having an initial
Charles' law for a constant pressure process
temperature of 80° F, is cooled at a constant
written as an equation is as follows: When the
pressure until its temperature is 40° F. If the
pressure is kept constant, initial volume of the gas is 8 cu ft, what is its
final volume?
TrVt = T^ (3-5)
Solution. Since the temperatures are given
where T± = the initial temperature of the gas in in degrees Fahrenheit, they must be converted
degrees Rankine to degrees Rankine before being substituted in
r2 = the final temperature of the gas in Equation 3-5.

degrees Rankine By rearranging and apply- _ ^i^i


Vx = the initial volume of the gas in ing Equation 3-5, the final Tt
cubic feet volume V2 = 500 x 8
Vs = the final volume of the gas in cubic 540
feet = 7.4074 cu ft
THERMODYNAMIC PROCESSES 27

3-8.Pressure-Volume Relationship at a and frequency of the impacts, the greater is the


Constant Temperature. When the volume pressure. The number of molecules confined in
of a gas is increased or decreased under such a given space and their velocity will, of course,
conditions that the temperature of the gas does determine the force and the frequency of the
not change, the absolute pressure will vary impacts. That is, the greater the number of
inversely with the volume. Thus, when a gas is molecules (the greater the quantity of gas) and
compressed (volume decreased) while its tem- the higher the velocity of the molecules (the
perature remains unchanged, its absolute higher the temperature of the gas), the greater
pressure will increase in proportion to the isthe pressure. The force with which the mole-
decrease in volume. Similarly, when a gas is cules strike the container walls depends only
expanded at a constant temperature, its abso- upon the velocity of the molecules. The higher
lute pressure will decrease in proportion to the the velocity the greater is the force of impact.
increase in volume. This is a statement of The greater the number of molecules in a given
Boyle's law for a constant temperature process space and the higher the velocity the more often
and is illustrated in Figs. 3-2a, 3-26, and 3-2c. the molecules will strike the walls.
has been previously stated that the mole-
It When a gas is compressed at a constant
cules of a gas are flying about at random and at temperature, the velocity of the molecules re-
high velocities and that the molecules of the mains unchanged. The increase in pressure
gas frequently collide with one another and with which occurs is accounted for by the fact that the
the walls of the container. The pressure exerted volume of the gas is diminished and a given
by the gas is a manifestation of these molecular number of gas molecules are confined in a smaller
collisions. Billions and billions of gas mole- space so that the frequency of impact is greater.
cules, traveling at high velocities, strike the The reverse of this holds true, of course, when
walls of the container during each fraction of a the gas expanded at a constant temperature.
is

second. It is this incessant molecular bombard- Any thermodynamic process which occurs in
ment which produces the pressure that a gas such a way that the temperature of the working
exerts upon the walls of its container. The substance does not change during the process is
magnitude of the pressure exerted depends upon called an isothermal (constant temperature)
the force and frequency of the molecular im- process.
pacts upon a given area. The greater the force Boyle's law for a constant temperature process

Fig. 3-2. Constant tempera-


ture process, (a) Initial condi-
L
tion, (b) Constant temperature
mi: piston: mi
expansion —volume
change is 1

inversely proportional to the !! i l

i i

change in absolute pressure. -, 1 I-'


'
1 1

Heat must be added during ':: II ;'.:.•


'

expansion to keep tempera- "^:'l l:---'v


-__-l-L_
ture constant, (c) Constant miipiswiiu
temperature compression
<p=ibopsi | P * 50 psi
volume change is inversely
proportional to the change in HI|liPiston:i||
V=lcuft . V«2cuft $
absolute pressure. Heat must iP m 200 psi<2
be removed during com- IV = 0.5cuft1
pression to keep temperature
!

T=70*F ? T=70*F ; !§J>.70*F


constant.
ft 'If H'l ii
Heat idded Heat re moved
(c>> (1 U)
28 PRINCIPLES OF REFRIGERATION

Volume = 1 cu ft Volume = 1 cu ft @> Volume = 1 cu ft

Pressure = 100 psia Pressure = 200 psia Pressure = 50 psia

Temperatures 500° R, Temperature= 1000° R Temperature = 250'

Heat added
TT TTT
Heat removed

(a) (b) (c)

Fig. 3-3. Constant volume process, (a) Initial condition, (b) The absolute pressure increases in direct
proportion to the increase in absolute temperature, (c) The absolute pressure decreases in direct proportion
to the decrease in absolute temperature.

is represented by the following equation: if initialpressure of the gas is 3000 psfa, determine
the temperature is constant, the final pressure in psig.

Solution. By rearranging PiVi


PiVi = P*r* (3-6)
and applying Equation 3-6,

where Px = the initial absolute pressure the final pressure Pt 3000 x 10


Pa = the final absolute pressure 4
Vx = the initial volume in cubic feet = 7500 psfa
K2 = the final volume in cubic feet
Dividing by 144
7500
144
Example Five pounds of air are ex-
3-5. = 52.08 psia
panded at a constant temperature from an = -
Subtracting the atmos- 52.08 14.7
initial volume of 5 cu ft to a final volume of
pheric pressure = 37.38 psig
10 cu ft. If the initial pressure of the air is
20 psia, what is the final pressure in psia? 3-9. Pressure-Temperature Relationship
at a Constant Volume. Assume that a gas
Solution. By rearranging and
is confined in a closed cylinder so that its
applying Equation 3-6, the final
pressure P2 20 x 5 volume cannot change as it is heated or cooled
10 (Fig. 3-3a). When the temperature of a gas is

= 10 psia increased by the addition of heat, the absolute


pressure will increase in direct proportion to the
Example 3-6. Four cubic
of gas are feet
increase in absolute temperature (Fig. 3-3b).
allowed to expand at a constant temperature
If the gas is cooled, the absolute pressure of the
from an initial pressure of 1500 psfa to a final
gas will decrease in direct proportion to the
pressure of 900 psfa. Determine the final
decrease in absolute temperature (Fig. 3-3c).
volume of the gas.
Whenever the temperature (velocity of the
Solution. By rearranging and _ PiVi molecules) of a gas is increased while the volume
applying Equation 3-6, the P* of the gas (space in which the molecules are
final volume Vz _ 1500 x 4
confined) remains the same, the magnitude of
900 the pressure (the force and frequency of mole-
= 6.67 cu ft
cular impacts on the cylinder walls) increases.
Example 3-7. A
given weight of gas, whose Likewise, when a gas is cooled at a constant
initial volume 10 cu ft, is compressed iso-
is volume, the force and frequency of molecular
thermally until its volume is 4 cu ft. If the impingement on the walls of the container
THERMODYNAMIC PROCESSES 29

diminish and the pressure of the gas will be less is used, then V will become the specific volume
than before. The reduction in the force and the v, and Equation 3-8 may be written:
frequency of molecular impacts is accounted
for by the reduction in molecular velocity. Pv
(3-9)
3-10. Charles' Law for a Constant Volume T
Process. Charles' law states in effect that
when a gas is heated or cooled under such con- where R= the gas constant
ditions that the volume of the gas remains
The gas constant R is different for each gas.
unchanged or constant, the absolute pressure
The gas constant for most common gases can
varies directly with the absolute temperature.
may be
be found in tables. A few of these are given
Charles' law written as the following
in Table 3-1.
equation: when the volume is the same,
Multiplying both sides of Equation 3-9 by M
produces
Vi = Vi (3-7)
PMv = MRT
where Tx = the initial temperature in degrees but since
Rankine Mv = V
r2 = the final temperature in degrees then
Rankine PV = MRT (3-10)
Px = the initial pressure in pounds per
square inch absolute where P = the pressure in psfa
P2 = the final pressure in pounds per V = the volume in ft3
square inch absolute M = the mass in pounds
R = the gas constant
Example 3-8. A certain weight of gas con- T = the temperature in ° R
fined in a tank has an initial temperature of
80° F and an pressure of 30 psig. If the
initial Equation 3-10 is known as the general gas law
gas is heated until the final gage pressure is and is very useful in the solution of many
50 psi, what is the final temperature in degrees problems involving gases. Since the value of R
Fahrenheit? for most gases can be found in tables, if any three

Solution. By rearrang- T, xi». of the four properties, P, V, M, and T, are known,


T, =
ing and applying Equa- Pi the fourth property can be determined by Equa-
tion 3-7, (80 + 460) tion 3-10. Notice that the pressure must be in
_ x(50 + 14.7) pounds per square foot absolute.
(30 + 14.7)
= 782° R Example 3-9. The tank of an
compres- air

- sor has a volume of 5 cu with air


ft and is filled
Converting Rankine to 782 460
= at a temperature of 100° F. If a gage on the
Fahrenheit 322° F
tank reads 151.1 psi, what is the weight of the
air in the tank?
3-11. The General Gas Law. Combining
Charles' and Boyle's laws produces the following
Solution. From
equation: Table 3-1, R for air = 53.3
P P (151.1 + 14.7)
-P= -P 0-8) By rearranging
and applying Equa- x 144x5
tion 3-10, the weight
~ 53.3 x (100 + 460)
Equation 3-8 is a statement that for any given
weight of a gas, the product of the pressure in
of air M _ 165.8 x 144 x 5
53.3 x 560
psfa and the volume in cubic feet divided by the
= 4 lb
absolute temperature In degrees Rankine will
always be a constant. The constant, of course, Example 3-10. Two pounds of air have a
will be different for different gases and, for any volume of 3 cu ft. If the pressure of the air is
one gas, will vary with the weight ofgas involved. 135.3 psig, what is the temperature of the air in
However, if, for any one gas, the weight of 1 lb degrees Fahrenheit?
30 PRINCIPLES OF REFRIGERATION
Solution. From amount of work done. The increase in the tem-
Table 3-1, R for air = 53.3 perature of a gas as the gas is compressed is a
common phenomenon and may
By
rearranging
and applying Equa-
T- PV
~ MR feeling the valve stem of a tire being
be noted by
filled with a
tion 3-10, the tem- (135.3 + 14.7) hand pump, or the head of an air compressor,
perature of the air x 144x3 etc.
in °R, 2 x 53.3 3-13. The General Energy Equation. The
law of conservation of energy clearly indicates
150 x 144 x 3
that the energy transferred to a body must be
2 x 53.3
accounted for in its entirety. It has been shown
= 607.9° R that some part (or all) of the energy taken in by a
Converting to t = 607.9 - 460 material may leave the material as work, and
Fahrenheit, = 147.9° F that only that portion of the transferred energy
which is not utilized to do external work remains
3-12.External Work. Whenever a material
in the body as "stored thermal energy." It is
undergoes a change in volume, work is done. If
evident then that of the energy transferred to
all
the volume of the material increases.work is done
a body must be accounted for in some one or in
by the material. volume of the material
If the
some combination of the following three ways:
decreases, work
done on the material. For
is
(1) as an increase in the internal kinetic energy,
example, consider a certain weight of gas con-
(2) as an increase in the internal potential energy,
fined in a cylinder equipped with a movable
and (3) as external work done. The general
piston (Fig. 3-la). As the gas is heated, its
energy equation a mathematical statement of
is
temperature increases and it expands, moving
this concept and may be written:
the piston upward in the cylinder against the
pressure of the atmosphere. Work is done in AQ = AK + AP + AW (3-11)
that the weight of the piston is moved through a
distance(Fig. 3-1A).* The agency doing the work where AQ = the heat energy transferred to the
is the expanding gas. material in Btu
In order to do work, energy is required AK = that fraction of the transferred
(Section 1-12). In Fig. 3-lZ>, the energy required energy which increases the in-
to do the work is supplied to the gas as the gas ternal kinetic energy
isheated by an external source. It is possible, AP = that fraction of the transferred
however, for a gas to do external work without energy which increases the in-
the addition of energy from an external source. ternal potential energy
In such cases, the gas does the work at the AW = that fraction of the transferred
expense of its own energy. That is, as the gas energy which is utilized to do
expands and does work, its internal kinetic energy external work
(temperature) decreases in an amount equal to
The Greek letter, A (delta), used in front of a
the amount of energy required to do the work.
term in an equation identifies a change of
When a gas is compressed (its volume de-
creased), a certain amount of work must be done
condition. For instance, where K represents
the internal kinetic energy, AK represents the
on the gas in order to compress it. And, an
change in the internal kinetic energy.
amount of energy equal to the amount of work
Depending upon the particular process or
done will be imparted to the molecules of the
change in condition that the material undergoes,
gas during the compression. That is, the
any of the terms in Equation 3-11 may have any
mechanical energy of the piston motion will be
value either positive or negative, or any may be
transformed into the internal kinetic energy of
equal to zero. This will be made clear later.
the gas (molecular motion) and, unless the gas is
3-14. External Work of a Solid or Liquid.
cooled during the compression, the temperature
When heat added to a material in either the
of the gas will increase in proportion to the
solid or liquid state increases the temperature
* Some work is done, also, in overcoming friction of the material, the material expands somewhat
and in overcoming the pressure of the atmosphere. and a small amount of work is done. However,
.

THERMODYNAMIC PROCESSES 31

the increase in volume and the external work the changes in the condition of a gas. Many
done is so slight that the portion of the trans- complex problems in mechanics are made simple
ferred energy which is utilized to do external by die assumption that no friction exists, the
work or to increase the internal potential energy effects of friction being considered separately.
is negligible. For all practical purposes, it can The function of an ideal gas is the same as that

be assumed that all the energy added to a solid of the frictionless surface. An ideal gas is
or a liquid during a temperature change in- assumed to undergo a change of condition
creases the internal kinetic energy. None leaves without internal friction, that is, without the

the material as work and none is set up as an performance of internal work in the overcoming
increase in the internal potential energy. In of internal molecular forces.
this instance, both AP and AW of Equation The idea of internal friction is not difficult to
3-11 are equal to zero and, therefore, AQ is comprehend. Consider that a liquid such as oil
equal to AK will not flow readily at low temperatures. This

When a solid melts into the liquid phase, is because of the internal friction resulting from
the change in volume is again so slight that the strong intermolecular forces within the liquid.
external work done may be neglected. Further- However, as the liquid is heated and the mole-
more, since the temperature also remains cules gain additional energy, the intermolecular

constant during the phase change, none of the forces are overcome somewhat, internal friction
transferred energy increases the internal kinetic diminishes, and the liquid flows more easily.
energy. All the energy taken in by the melting Vaporization of the liquid, of course, causes
solid is set up as an increase in the internal a greater separation of the molecules and
potential energy. Therefore, AK and AW are brings about a substantial reduction in internal
both equal to zero and AQ is equal to AP. friction, but some interaction between the

This is not true, however, when a liquid molecules of the vapor still exists. In the
changes into the vapor phase. The change in gaseous state, intermolecular forces are greatest
volume that occurs and therefore the external when the gas is near the liquid phase and
work done as the liquid changes into a vapor is diminish rapidly as the gas is heated and its

considerable. For example, when 1 lb of water temperature rises farther and farther above the
at atmospheric pressure changes into a vapor, saturation temperature. A
gas approaches the
its volume increases from 0.01671 cu ft to 26.79 ideal state when it reaches a condition such that

cu ft. Of the 970.4 Btu required to vaporize 1 the interaction between the molecules and
lb of water, approximately 72 Btu of this energy hence, internal friction, is negligible.
are required to do the work of expanding Although no such thing as an ideal or perfect
against the pressure of the atmosphere. The many gases, such as air,
gas actually exists,
remainder of the energy vapor as
is set up in the nitrogen, hydrogen, helium, etc., so closely
an increase in the internal potential energy. In approach the ideal condition that any errors
this instance, only AK is equal to zero, so that which may result from considering them to be
AQ is equal to AP plus AW. ideal are of no consequence for all practical

3-15. "Ideal" or "Perfect" Gas. The various purposes.


laws governing the pressure-volume-temperature Although important that the student of
it is

relationships of gases as discussed in this chapter refrigeration understand and be able to apply
apply with absolute accuracy only to a hypo- the laws of perfect gases, it should be under-
thetical "ideal" or "perfect" gas. A "perfect" stood that gases as they normally occur in the
gas is described as one in such a condition that mechanical refrigeration cycle are close to the
is, they are vapors, and
saturation curve, that
there is no interaction between the molecules of
the gas. The molecules of such a gas are do not even approximately approach the
entirely free and independent of each other's condition of an ideal or perfect gas.* They
attractive forces. Hence, none of the energy follow the gas laws in only a very general way,
transferred either to or from an ideal gas has * Avapor is sometimes defined as a gas at a
any effect on the internal potential energy. condition close enough to the saturation curve so
The concept of an ideal or perfect gas greatly that it does not follow the ideal gas laws even
simplifies the solution of problems concerning approximately.
32 PRINCIPLES OF REFRIGERATION
and therefore the use of the gas laws to deter- transferred to the gas increases the internal
mine the pressure-volume-temperature relation- kinetic energy of the gas. None of the energy
ships of such vapors will result in considerable leaves the gas as work.
inaccuracy. In working with vapors, it is usually When
a gas is cooled (heat removed) while
necessary to use values which have been deter- itsvolume remains constant, all the energy
mined experimentally and are tabulated in removed is effective in reducing the internal
saturated and superheated vapor tables. These kinetic energy of the gas. It should be noted
tables are included as a part of this textbook that in Equation 3-12, AQ represents heat
and are discussed later. transferred to the gas, AK represents an in-
3-16. Processes for Ideal Gases. A gas is crease in the internal kinetic energy, and AW
said to undergo a process when it passes from represents work done by the gas. Therefore,
some initial state or condition to some final if heat is given up by the gas, AQ is negative.
state or condition. A change in the condition of Likewise, if the internal kinetic energy of the gas
a gas may occur in an infinite number of ways, decreases, AK is negative and, if work is done on
only five of which are of interest. These are the the gas, rather than by it, A W is negative. Hence,
(1)constant pressure (isobaric), (2) constant in Equation 3-13, when the gas is cooled, both
volume (isometric), (3) constant temperature AQ and AK are negative.
(isothermal), (4) adiabatic, and (5) polytropic 3-18. Constant Pressure Process. If the
processes. temperature of a gas is increased by the addition
In describing an ideal gas, it has been said of heat while the gas is allowed to expand so
that the molecules of such a gas are so far apart that its pressure is kept constant, the volume of
that they have no attraction for one another, the gas will increase in accordance with Charles'
and that none of the energy absorbed by an law (Fig. 3-1). Since the volume of the gas
ideal gas has any effect on the internal potential increases during the process, work is done by
energy. It is evident, then, that heat absorbed the gas at the same time that its internal energy
by an ideal gas will either increase the internal is increased. Hence, while one fraction of the
kinetic energy (temperature) of the gas or it will transferred energy increases the store of internal
leave the gas as external work, or both. Since kinetic energy, another fraction of the trans-
the change in the internal potential energy, AP, ferred energy leaves the gas as work. For a
willalways be zero, the general energy equation constant pressure process, identified by the
for an ideal gas may be written: subscript/!, the energy equation may be written
AQ = AK + AW (3-12) AQP =AK 1I
+AW (3-14)
In order to better understand the energy 3-19. Specific Heat of Gases. The quantity
changes which occur during the various pro- of heat required to raise the temperature of
cesses, it should be kept in mind that a change in 1 lb of a gas 1° F while the volume of the gas
the temperature of the gas indicates a change in remains constant is known as the specific heat
the internal kinetic energy of the gas, whereas a at a constant volume (C„). Similarly, the
change in the volume of the gas indicates work quantity of heat required to raise the tempera-
done either by or on the gas. ture of 1 lb of a gas 1° F while the gas expands
3-17. Constant Volume Process. When a at a constant pressure is called the specific
gas is heated while it is so confined that its heat at a constant pressure (C„). For any
volume cannot change, its pressure and tempera- particular gas, the specific heat at a constant
ture will vary according to Charles' law (Fig. pressure is always greater than the specific heat
3-3). Since the volume of the gas does not at a constant volume. The reason for this is
change, no external work is done and is AW easily explained.
equal to zero. Therefore, for a constant The quantity of energy required to increase
volume process, indicated by the subscript v, the internal kinetic energy of a gas to the extent
that the temperature of the gas is increased 1 ° F
*Qv - Mv (3-13)
is exactly the same for all processes. Since,
Equation 3-13 is a statement that during a during a constant volume process, no work is
constant volume process all of the energy done, the only energy required is that which
THERMODYNAMIC PROCESSES 33

increases the internal kinetic energy. However, Example 3-12. Twelve pounds of air are
during a constant pressure process, the gas cooled from an initial temperature of 95° F to
expands a fixed amount for each degree of a final temperature of 72° F. Compute the
increase in the internal kinetic energy.
temperature rise and a certain amount of
external work is done. Therefore, during a
Solution. Apply- = 12 x 0.169
constant pressure process, energy to do the ing Equation 3-15, x (72 - 95)
work thatdone must be supplied in addition
is the increase in in- = 12 x 0.169 x (-23)
to that which increases the internal kinetic ternal kinetic energy = -46.64 Btu
energy. For example, the specific heat of air AK
at a constant volume is 0.169 Btu per pound,
In Example 3-12, AK is negative, indicating
whereas the specific heat of air at a constant
that the gas is cooled and that the internal
pressure is 0.2375 Btu per pound. For either
kinetic energy is decreased rather than increased.
process, the increase in the internal energy of
Heat Transferred during a Constant
3-21.
the air per degree of temperature rise is 0.169
Volume Process. For a constant volume
Btu per pound. For the constant pressure pro-
process, since
cess, the additional 0.068S Btu per pound
AQV = AKV
(0.2375 — 0.169) is the energy required to do
then
the work resulting from the volume increase
AQV = MCv{ti - /,) (3-16)
accompanying the temperature rise.
The specific heat of a gas may take any value Example 3-13. If, in Example 3-11, the gas

either positive or negative,depending upon the is heated while its volume is kept constant,
amount of work that the gas does as it expands. what is the quantity of heat transferred to the
3-20. The Change in Internal Kinetic gas during the process ?
Energy. During any process in which the
Solution. Apply- AQV = 5 x 0.169
temperature of the gas changes, there will be a x (140 - 75)
ing Equation 3-16,
change in the internal kinetic energy of the gas. the heat transferred = 5 x 0.169 x 65
Regardless of the process, when the tempera- to the gas, = 54.9 Btu
ture of a given weight of gas is increased or
decreased, the change in the internal kinetic Alternate Solution.

energy can be determined by the equation From Example


3-12, AKV = 54.9 Btu
AK = MCv(tt - fj (3-15) Since
AQ V = AKV AQV = 54.9 Btu
where AK = the increase in the internal kinetic
,

energy in Btu Example 3-14. If,in Example 3-12, the


M = the weight pounds
in air is cooled while its volume remains constant,
Cv = constant volume specific heat what is the quantity of heat transferred to the

tt = the final temperature air during the process?


fx = the initial temperature
Solution. From
Note. The temperature may be in either Example 3-12, AKV = -46.67 Btu
Fahrenheit or Rankine, since the difference in Since AQ„ = AKV . AQV 46.67 Btu
temperature will be the same in either case as
In Example 3-14, notice that since AKV is
long as the units are consistent.
negative, indicating a decrease in the internal
Example 3-11. The temperature of 5 lb kinetic energy, AQV must of necessity also be
of air increased by the addition of heat from
is negative, indicating that heat is transferred
an initial temperature of 75° F to a final tem- from the gas rather than to it.
perature of 140° F. If C„ for air is 0.169 Btu, 3-22, External Work during a Constant
what is the increase in the internal energy? Pressure Process. It will now be shown that
Solution. Applying =5 x 0.169 the work done during a constant pressure
Equation 3-15, the in- x (140 - 75) process may be evaluated by the equation
crease in internal kinetic = 5 x 0.169 x 65
energy AK = 54.9 Btu W'PiVt-VJ (3-17)
34 PRINCIPLES OF REFRIGERATION
where W = the work done in foot-pounds the sum of AKP , the increase in internal kinetic
P= the pressure in psfa energy,and AW
9 the heat energy equivalent of
,

Kg = the final volume in cubic feet the work of expansion.


Vx = the initial volume in cubic feet
Example 3-17. Compute the total heat
Assume that the piston in Fig. 3-lc has an energy transferred to the air during the constant
area of A square feet and that the pressure of pressure process described in Example 3-15.

the gas in the cylinder is P pounds per square Solution. Con-


foot. Then, the total force exerted on the top of verting 70° F to ° R = 70 + 460
the piston will be PA pounds, or degrees Rankine, = 530° R
F = P xA Applying T2 — *1 X ^1
Charles' law, " vx
Assume now that the gas in the cylinder, having Equation 3-5, to _ 530 x 15
an initial volume Vly is heated and allowed to determine the final 13.34
expand to volume V2 while its pressure is kept temperature, = 596"
constant. In doing so, the force PA acts through
Applying Equa- AA: = 1 x 0.169
the distance 1 and work is done. Hence,
tion 3-15, the in- x (596 - 530)
W = P xA x crease in internal = 1 x 0.169 x 66

but since
kinetic energy, = 11.154 Btu

A x 1 = (Vs - VJ From Example


then
3-15 and 3-16, DW, =4.52 Btu
W = P(V -VJ 2
Applying Equa- AQ V = 11.15 + 4.52
tion 3-14, = 15.67 Btu
Example 3-15. One pound of air having
an initial volume of 13.34 cu ft and an initial Since the specific heat at a constant pressure
temperature of 70° F is heated and allowed to CP takes into account not only the increase in
expand at a constant pressure of 21 17 psfa to a internal energy per pound but also the work
final volume of 15 cu ft. Determine the amount done per pound per degree of temperature rise
of external work in foot-pounds. during a constant pressure expansion, for the
Solution. Applying = 2117 x constant pressure process only, AQ P may be
Equation 3-17, the work in (15 - 13.34) determined by the following equation
foot-pounds W = 2117 x 1.66
AQ P - MCp (t 2 -
= 3514 ft-lb tj) (3-20)

In Equation 3-12, AWS& always given in heat Hence, an alternate solution to Example 3-17 is

energy units. By application of the mechanical Applying Equation AQ V = 1 x 0.2375


energy equivalent (Section 3-16), W in foot- 3-20, x (596 - 530)
pounds may be expressed as A W in Btu. The = 1 x 0.2375
relationship is x 66
W
AW = — (3-18)
= 15.67 Btu

3-24. Pressure-Volume (PV) Diagram.


W = AW xJ (3-19)
Equation 3-8 is a statement that the thermo-
Example 3-16. Express the work done in dynamic state of a gas is adequately described
Example 3-15 in terms of heat energy units. by any two properties of the gas. Hence, using
any two properties of the gas as mathematical
Solution. Applying Equation _ 3514
coordinates, the thermodynamic state of a gas
3-18, the work in Btu W 778
at any given instant may be shown as a point
= 4.52 Btu
on a chart. Furthermore, when the conditions
3-23.Heat Transferred during a Constant under which a gas passes from some initial state
Pressure Process. According to Equation to some final state are known, the path that the
3-14, AQ„ the total heat transferred to a gas process follows may be made to appear as a line
during a constant pressure process is equal to on the chart.
THERMODYNAMIC PROCESSES 35

The graphical representation of a process or 4000


cycle is a process diagram or a cycle
called
diagram, respectively, and is a very useful tool
in the analysis and solution of cyclic problems.
_ Final
work
Since is a function of pressure and condition

volume, when it is the work of a process or

cycle which is of interest, the properties used as


coordinates are usually the pressure and the
volume. When the pressure and volume are
used as coordinates to diagram a process or
cycle, it is called a pressure-volume (PV)
12 13 ! 14 16
diagram.
*i v2
To illustrate the use of the PV diagram, a
Volume (cubic feet)
pressure-volume diagram of the process de-
Fig. 3-4. Pressure-volume diagram of constant
scribed in Example 3- IS is shown in Fig. 3-4.
pressure process. Crosshatched area between
Notice that the pressure in psfa is used as the process diagram and base line represents external
vertical coordinate, whereas the volume in cubic work done during the process.
feet is used as the horizontal coordinate.
In Example 3-1 5, the initial condition of the

gas such that the pressure is 21 17 psfa and the


is two dimensions. In Fig. 3-4, the area of the
volume is 13.34 cu ft. To establish the initial rectangle, 1-2-Kg-^ (crosshatched), is the
state of the gas on the PV chart, start at the product of its altitude P and its base (Kg — Fj).
origin and proceed upward along the vertical But according to Equation 3-17, the product
pressure axis to the given pressure, 2117 psfa. P(Vt — VJ is the external work done during a
Draw a dotted line parallel to the base line constant pressure process. It is evident then
through this point and across the chart. Next, that the area between the process diagram and
from the point of origin proceed to the right the volume axis a measure of the external
is

along the horizontal volume axis to the given work done during the process in foot-pounds.
volume, 13.34 cu ft. Through this point draw a This area is frequently referred to as "the area
vertical dotted line across the chart. The inter- under the curve."
section of the dotted lines at point 1 establishes Figure 3-5 is a PV diagram of a constant
the initial thermodynamic state of the gas. volume process. Assume that the initial
According to Example 3-1 5, the gas is heated condition of the gas at the start of the process
and allowed to expand at a constant pressure is such that the pressure is 2000 psfa and the
until its volume is IS cu ft. Since the pressure volume is 4 cu ft. The
is heated while its
gas
remains the same during the process, the state volume is kept constant until the pressure
point representing the final state of the gas must increases to 4000 psfa. The process takes place
fall somewhere along the line of constant along the constant volume line from the initial
pressure already established. The exact point condition 1 to the final condition 2.
on the pressure line which represents the final It has been stated that no work is done during
state 2
determined by the intersection of the
is a process unless the volume of the gas changes.
line drawn through the point on the volume axis Examination of the PV diagram in Fig. 3-5 will
that identifies the final volume. show that no work is indicated for the constant
In passing from the initial state 1 to the final volume process. Since a line has only the
state 2 the air passes through a number of dimension of length, there is no area between
intermediate thermodynamic states, all of which the process diagram and the base or volume
can be represented by points which will fall axis. Hence, no work is done.
along line 1 to 2. Line 1 to 2, then, represents 3-25. Constant Temperature Process.
the path that the process will follow as the ther- According to Boyle's law, when a gas is com-
modynamic state of the gas changes from 1 to 2, pressed or expanded at a constant temperature,
and is the PV diagram of the process described. the pressure will vary inversely with the volume.
The area of a rectangle is the product of its That is, the pressure increases as the gas is
36 PRINCIPLES OF REFRIGERATION

4500 shown in Fig. 3-6. In a constant temperature


P* ^~ Final condition process the pressure and volume both change in
4000 accordance with Boyle's law. The path followed
by an isothermal expansion is indicated by
3500
line 1 to 2 and the work of the process in foot-

3000 pounds is represented by the area l-2-V^-V^


The area, 1-2-V^-V^ and therefore the work
32500 of the process, may be calculated by the equa-
£
Px ^— Initial condition tion
| 2000

1500
fF = P1 K1 xln-^ (3-22)

where In = natural logarithm (log to the base e)



500

3 4
II
5 6
1

7
Example
having an
initial volume
3-18.
initial
A certain weight of gas
pressure of 2500 psfa and an
of 2 cu ft is expanded isothermally
Volume (cubic feet) to a volume of 4 cu Determine:
ft.

(a) the final pressure of the gas in psfa


Fig. 3-5. Pressure-volume diagram of constant
volume (b) the work done in heat energy units.
process. no area between the
Since there is

process diagram and the volume axis, there is no


work done during a constant volume process. Solution
(a)By applying Boyle's PiVx
law, Equation 3-6, Vi
compressed and decreases as the gas the final pressure Pa 2500 x 2
expanded. is

Since the gas will do work as it expands, if 4


the temperature is to remain constant, energy = 1250 psfa
with which to do the work must be absorbed (b) By applying Equa-
from an external source (Fig. 3-26). However, tion 3-22, the ex- = 2500 x 2 x lnf
since the temperature
gas remains of the
ternal work of the = 2500 x 2 x In 2
process in foot- = 2500 x 2 x 0.693
constant, of the energy absorbed by the gas
ail

during the process leaves the gas as work;


pounds W = 3465 ft-Ib
By Applying Equa- 3465
none is stored in the gas as an increase in the ~ 778
tion 3- 1 8 the work in
,
internal energy.
heat energy units Alf = 4.45 Btu
When a gas is compressed, work done on is

the gas, and if the gas is not cooled during ihe


compression, the internal energy of the gas will
be increased by an amount equal to the work of
compression. Therefore, if the temperature of
the gas is to remain constant during the com-
pression, the gas must reject to some external
body an amount of heat equal to the amount of Final
work done upon it during the compression condition

(Fig. 3-2c).
There is no change in the internal kinetic
energy during a constant temperature process.
Therefore, in Equation 3-12, AK is equal to zero
and the general energy equation for a constant
temperature process may be written

AQ = f
AW t (3-21)
Fig. 3-6. Pressure-volume diagram of constant
3-26. Work of an Isothermal Process. A temperature process. Crosshatched area represents
PV diagram of an isothermal expansion is the work of the process.
THERMODYNAMIC PROCESSES 37

Example 3-19. A
certain weight of gas Solution. From Example
having an pressure of 12S0 psfa and an
initial 3-18, AW = 4.45 Btu
initial volume of 4 cu ft is compressed isotherm- Since, in the isothermal
ally to a volume of 2 cu ft. Determine: process, AW t equals AQ t,
AQ = t 4.45 Btu
(a) the final pressure in psfa
(b) the work done by the gas in Btu. Example 3-21. What is the quantity of
heat transferred to the gas during the constant
Solution temperature process described in Example 3-19.
(a)By applying PiVi
Boyle's law, Solution. From
Equation 3-6, 1250 x 4 Example 3-19, AW = -4.45 Btu
the final pressure 2 Since A W t equals AQ t,
AQ = t -4.45 Btu
^2 = 2500 psfa
Again, notice that a negative amount of heat
(b) By Applying Equa- is transferred to the gas, indicating that heat in
tion 3-22, the ex- = 2500 x 2 x In | this amount is actually given up by the gas
ternal work of the = 2500 x 2 x In 0.5
during the process.
process in foot- = 2500 x 2 x -0.693
pounds W = -3465 ft-lb 3-28. Adiabatic Process. An adiabatic pro-
cess is described as one wherein the gas
By applying
Equation 3-18, -3465 changes its condition without absorbing or
the work in heat 778 rejecting heat, as such,from or to an external
energy units AW = -4.45 Btu body during the process. Furthermore, the
pressure, volume, and temperature of the gas all
Notice that the process in Example 3-19 is the vary during an adiabatic process, none of them
exact reverse of that of Example 3-18. Where remaining constant.
the process in Example 3-18 is an expansion, When a gas expands adiabatically, as in any
the process in Example 3-19 is a compression. other expansion, the gas does external work and
Both processes occur between the same two energy is required to do the work. In the
conditions, except that the initial and final processes previously described, the gas absorbed
conditions are reversed. Notice also that the energy to do the work from an external
whereas work is done by the gas during the source. an adiabatic process, no
Since, during
expansion process, work is done on the gas heat is absorbed from an external source, the gas
during the compression process. But since the must do the external work at the expense of its
change of condition takes place between the own energy. An adiabatic expansion is always
same limits in both cases, the amount of work accompanied by a decrease in the temperature
done in each case is the same. of the gas as the gas gives up its own internal
3-27.Heat Transferred during a Constant energy to do the work (Fig. 3-7).
Temperature Process. Since there is no When a gas is compressed adiabatically,
change in the temperature during an isothermal work is done on the gas by an external body.
process, there is no change in the internal The energy of the gas is increased in an amount
kinetic energy and AK equals zero. According equal to the amount of work done, and since no
to Equation 3-21, the heat energy transferred heat energy is given up by the gas to an external
during a constant temperature process is exactly body during the compression, the heat energy
equal to the work done in Btu. During an equivalent of the work done on the gas is set up
isothermal expansion heat is transferred to the as an increase in the internal energy, and the
gas to supply the energy to do the work that is temperature of the gas increases.
done by the gas, whereas during an isothermal Because no heat, as such, is transferred to or
compression heat is transferred from the gas so from the gas during an adiabatic process, AQa is
that the internal energy of the gas is not in- always zero and the energy equation for an
creased by the performance of work on the gas. adiabatic process is written as follows:
Example 3-20. Determine the quantity of AKa +AW =0 (3-23)
a
heat transferred to the gas during the constant
Therefore,
temperature expansion described in Example
3-18. AW. = -AK. and AKa - -AWa
38 PRINCIPLES OF REFRIGERATION
Example 3-23. A
gas having an initial
pressure of 945 psfa and an initial volume of
4 cu ft is compressed adiabatically to a volume
of 2 cu ft. If the final pressure of the air is
2500 psfa, how much work is done in heat
energy units?

Solution. From
Example 3-22, k for air 1.406
Final
condition Applying Equation 3-24, (945 x 4) -
the work done in foot- (2500 x 2)
pounds W a 1.406 - 1

3780 - 5000
0.406
-1220
Fig. 3-7. Pressure-volume diagram of adiabatic
0.406
process. An isothermal curve is drawn in for
comparison.
-3005 ft-lb

Applying Equation 3-18, -3005


the work in heat energy 778
3-29. Work of an Adiabatic Process. The units A(C. -3.86 Btu
work of an adiabatic process may be evaluated
by the following equation: 3-30. Comparison of the Isothermal and
Adiabatic Processes. A comparison of the
P1 V1 -P2 V2
W a =
k - 1
(3-24) isothermal and adiabatic processes is of interest.
Whenever a gas expands, work is done by the
where k — the ratio of the specific heats of the gas, and energy from some source is required to
gas in question, CJCV do the work. In an isothermal expansion, all
Example 3-22. A gas having an initial of the energy to do the work is supplied to the
pressure of 2500 psfa and an volume of
initial gas as heat from an external source. Since the
2 cu ft is expanded adiabatically to a volume energy is supplied to the gas from an external
of 4 cu ft. If the final pressure is 945 psfa, source at exactly the same rate that the gas is
determine the external work done in heat doing work, the internal energy of the gas
energy units.
and the tempera-
neither increases nor decreases
Solution ture of the gas remains constant during the
Cj, for air = 0.2375 Btu/lb process. On the other hand, in an adiabatic
C„ for air = 0.169 Btu/lb expansion there is no transfer of heat to the gas
The ratio of the specific c, during the process and all of the work of expan-
heats, k sion must be done at the expense of the internal
0.2375 energy of the gas. Therefore, the internal
0.169 energy of the gas is always diminished by an
= 1.406 amount equal to the amount of work done and
Applying Equation 3-24, (2500 x 2) the temperature of the gas decreases accordingly.
the work of adiabatic ex- - (945 x 4) Consider now isothermal and adiabatic
pansion in foot-pounds 1406 - 1 compression processes. In any compression
w a 5000 - 3780 process,work is done on the gas by the com-
0.406 pressing member, usually a piston, and an
1220 amount of energy equal to the amount of work
0.406 done on the gas is transferred to the gas as work.
= 3005 ft-lb During an isothermal compression process,
Applying Equation 3-18, 3005 energy is transferred as heat from the gas to an
the work in heat energy ~ 778 external sink at exactly the same rate that work
units A Wa = 3.86 Btu is being done on the gas. Therefore, the internal
:

THERMODYNAMIC PROCESSES 39

energy of the gas neither increases nor decreases


during the process and the temperature of the
gas remains constant. On the other hand,
during an adiabatic compression, there is no Polytropic N> 1 and <K
transfer of energy as heat from the gas to an
external sink. Therefore, an amount of energy
equal to the amount of work done on the gas is
set up in the gas as an increase in the internal
energy, and the temperature of the gas increases
accordingly.
3-34. The Polytropic Process. Perhaps the
simplest way of denning a polytropic process is

by comparison with the isothermal and adia-


batic processes. The isothermal expansion, in
which the energy to do the work of expansion is
Fig. 3-8. Pressure-volume diagram of a polytropic
supplied entirely from an external source, and
process. Adiabatic and isothermal curves are drawn
the adiabatic expansion, in which the energy to
in for comparison.
do the work of expansion is supplied entirely by
the gas itself, may be thought of as the extreme
limits between which all expansion processes will
rate of heat rejection and move the path of the
fall. Then, any expansion process in which the
compression closer to the isothermal.
energy to do the work of expansion is supplied
3-32. PVT Relationship during Adiabatic
partly from an external source and partly from
Processes. Since the temperature, pressure,
the gas itself will follow a path which will fall
and volume all change during an adiabatic
somewhere between those of the isothermal and
process, they will not vary in accordance with
adiabatic processes (Fig. 3-8). Such a process is
Charles' and Boyle's laws. The relationship
known as a polytropic process. If during a
between the pressure, temperature, and volume
polytropic' expansionmost of the energy to do
during an adiabatic process may be evaluated by
the work comes from an external source, the
the following equations
polytropic process will more nearly approach
the isothermal. On the other hand, when the * -1)
H'
greater part of the energy to do the external Tt = Tx x (3-25)
work comes from the gas itself, the process F(*-l)
more nearly approaches the adiabatic.
This is true also for the compression process. (3-26)
When a gas loses heat during a compression
process, butnotataratesumcient to maintain the
temperature constant, the compression is poly-
tropic. The greater the loss of heat, the closer
p*=Pi (3-27)

the polytropic process approaches the iso-

thermal.' The smaller the loss of heat, the closer


-P (3-28)
the polytropic process approaches the adiabatic.
Of course, with no heat loss, the process
'*W
becomes adiabatic.
The actual compression of a gas in a compres- (3-29)
sor will usually very nearly approach adiabatic
compression. This is because the time of com-

pression normally very short and there is not


is
- v, . (3>r (3-30)
sufficient time for any significant amount of heat
to be transferred from the gas through the
cylinder walls to the surroundings. Water Example 3-24. Air is expanded adiabati-
jacketing of the cylinder will usually increase the cally from a volume of 2 cu ft to a volume of
R

40 PRINCIPLES OF REFRIGERATION
4 cu ft. If the initial pressure of the air is 24,000 gas undergoing the process.*
' Usually, the
psfa, what is the final pressure in psfa? value of « must be determined by actual test of
the machine in which the expansion or com-
Solution. From
pression occurs. In some instances average
Table 3-1, k for air = 1.406
,1.406 values of n for some of the common gases
Applying Equation
3-27, the final pressure
= 24,000 x undergoing changes under more or less standard
conditions are given in tables. If the values of
= 24,000 x 0.5 1 *06
two properties are known for both
= initial and
24,000 x 0.378
= 9072 psfa
final conditions, the value of n may be calcu-
lated. The following sample equation shows
Example 3-25. Air is expanded adia- the relationship:
batically from a volume of 2 cu ft to a volume
of 4 cu ft. If the initial temperature of the air
« = -^
log
(Pj/Pj,)
(3-31)
logC^/^)
is 600° R, what is the final temperature in
degrees Rankine? Example 3-27." Air, having an initial
pressure of 24,000 psfa and an initial tempera-
Solution. From
ture of 600° R, is expanded polytropically from
Table 3-1, k for air = 1.406
a volume of 2 cu ft to a volume of 4 cu ft. If
Applying Equation (2)« .406-1)
= 600x the exponent of polytropic expansion is 1.2,
3-25, the final tempera- (4)(1.406-
determine:
ture T2 (2)0.406
(a) The weight of the air in pounds
= 600x
(4)0.408 (6) The final pressure in psfa
1.325 (c) The final temperature in degrees Rankine
= 600x (d) The work done by the gas in Btu
1.756
= 600 x 0.755 (e) The increase in internal energy

= 453" (/) The heat transferred to the gas.

Example Air
expanded adia-
3-26.
is Solution
batically from anpressure of 24,000 psfa
initial (a) From Table 3-1,
to a final pressure of 9072 psfa. If the initial R for air = 53.3
temperature is 600° R, what is the final tem- Rearranging and
perature? applying Equation RT
3-10, the weight of _ 24,000 x 2
Table
Solution. From air M 53.3 x 600
3-1, k for
= 1.5 lb
air = 1.406
Applying 1.406-1 * The value of n depends upon the specific heat
Equation 3-26, \ **
/ 9072 \ i«o«
= 600 x of the gas during the process. Since the specific
the final tempera-
124 000/
\24,000/ heat may take any value, it follows that theoretically
ture T9 = 600 x (0.378)< 0M »» n may have any value. In actual machines, however,
= 600 x 0.755 n will nearly always have some value between 1
= 453° R and A:.

Broadly- defined,
a polytropic process is any
Exponent of Polytropic Expansion
3-33. process during which the specific heat remains
and Compression. The presssure-tempera- constant. By this definition, all five processes
ture-volume relationships for the polytropic discussed in this chapter are polytropic processes.
process can be evaluated by Equations 3-25 It is general practice today to restrict the term

through 3-30, except that the polytropic ex- polytropic to mean only those processes which follow

pansion or compression exponent n is substi- a path between those of the isothermal and
falling

tuted for k. Too, the work of a polytropic


adiabatic processes. The exponents of isothermal
and adiabatic expansions or compressions are 1
process can be determined by Equation 3-24 if
and k, respectively. Hence, the value of n for the
n is substituted for k.
polytropic process must fall between 1 and k. The
The exponent n will always have a value closer the polytropic process approaches the
somewhere between 1 and k for the particular adiabatic, the closer n will approach k.

THERMODYNAMIC PROCESSES 41

(b) Applying Equation 2. The volume of a certain weight of air is kept


3-27, the final
- 24,000 xg)" constant while the temperature of the air is
pressure Pa = 24,000 x (0.SY* increased from 55° F to 100° F. If the initial
- 24,000 x 0.435 pressure is 25 psig, what is the final pressure of
= 10,440 psfa the air in psig ? Am. 28 .47 psig
3. A
certain weight of air confined in a con-
Applying Equation (2)U.»-i) tainer is cooled from 150° F to 70° F. If the
(c)
— x
600 *
"""
3-25, the tempera- (4)<i.«-i) initial pressure of the air is 36.3 psig, what is the

ture Tt final pressure of the air in psig ? Am. 29.6 psig


-«0x3K (4)
o.» 4. One pound of air at atmospheric pressure
,„ — 1.149 has a volume of 13.34 cu ft at a temperature
= 600 x of 70° F. If the air is passed across a heat
1.32
= 522°R exchanger and is heated to a temperature of
150° F while its pressure is kept constant, what
is the final volume of the air ? Ans. 15.35 cu ft
(d) Applying Equation (24,000 x 2)
3-24, the work - (4 x 10,440) 5.A cylinder of oxygen has a volume of 5 cu ft.

done W 1.2 -1 A gage on the cylinder reads 2200 psi. If the


temperature of the oxygen is 85° F, what
_ 48,000 - 41,760 is the
weight of the oxygen in the cylinder?
0.2
Am. 60.6 lb
6240 6. In Problem 4, determine:
0.2 (a) The work done by the air during the
= 31,200 ft-lb heating. Am. 4254.8 ft-lb or 5.47 Btu
(b) The increase in the internal kinetic energy.
Applying Equation 31,200 Am. 13.52 Btu
3-18, the work in 778 (c) The quantity of heat transferred to the air.
BtuAW = 40.10 Btu Am. 19 Btu
7. A certain weight of air having an initial
(e) From Table 3-1, = 0.169 Btu/lb volume of 0.1334 cu ft and an initial tempera-
C„ for air ture of 70° F is drawn into the suction side of
Applying Equation = 1.5 x 0.169 x an air compressor. If the air enters the cylinder
3-15, the increase (522 - 600) at standard atmospheric pressure and is com-
in internal energy = 1.5x 0.169 x pressed isothermally to a final pressure of 150
AK (-78) psia, determine:
= -19.77 Btu (a) The weight of air in the cylinder at the
start of the compression stroke.
(/) Applying Equation = AJSr + AfF Am. 0.01 lb
3-9, the heat (b) The final temperature of the air in degrees
energy trans- = -19.77 + 40.10 Rankine. Am. 530° R
ferred to the gas AQ = 20.33 Btu (c) The volume of
the air at the end of the
compression stroke. Am. 0.0131 cu ft
(d) The work of compression in Btu.
Notice in Example 3-27 that the work done
by the air in the polytropic expansion is equiva-
Am. 0.843 Btu
(e) The increase or decrease in internal energy.
lent to 40.10 Btu. Of this amount, 20.33 Btu
Am. None
is supplied from an external source, whereas the The energy
(/) transferred to the gas during
other portion, 19.77 Btu, is supplied by the gas the compression. Am. —0.843 Btu
itself, thereby reducing the internal kinetic
8. Assumethat the air in Problem 7 is com-
energy by this amount. pressed adiabatically rather than isothermally.
Compute:
PROBLEMS (a) The final temperature of the air in degrees

1. Three pounds of air occupy a volume of


Rankine. Am. 1038° R
(b) The volume of
the air at the end of the
24 cu ft. Determine: compression stroke. Am. 0.0256 cu ft
(a) The density of the air. Arts. 0.125 lb/cu ft (c) The work of compression in Btu.
(b) The specific volume. Am. 8 cu ft/lb Am. 0.86 Btu
42 PRINCIPLES OF REFRIGERATION

(d) The increase in the internal kinetic energy. (b) The volume of the air at the end of the
Arts. 0.86 Btu compression stroke. Ans. 0.0192 cu ft
(c) The heat energy transferred to or from (c) The work of compression is Btu.
the gas during the compression. Ans. 0.8S Btu
Ans. None (d) The decrease in the internal kinetic energy.
Ans. 0.45 Btu
9. Assuming that the air in Problem 7 is com-
(e) The heat energy transferred from the gas
pressed polytropically rather than isothermally.
during the compression. Ans. 0.40 Btu
If n equals 1.2, compute:
(«) The final temperature of the air in degrees 10. Compare the results of Problems 7, 8,

Rankine. Ans. 119 A" R and 9.


For example, in Fig. 4-1 , the water in the heated
vessel is saturated and is vaporizing at 212° F as
the latent heat of vaporization is supplied by the
burner. The water vapor (steam) rising from
the water is saturated and remains at the satura-

4 tion temperature (212° F) until


condenser. As the saturated vapOr gives
it reaches the
up
heat to the cooler water in the condenser, it

condenses back into the liquid state. Since


Saturated and condensation occurs at a constant temperature,
the water resulting from the condensing vapor

Superheated is also at 212° F. The latent heat of vapor-

ization,absorbed as the water vaporizes into

Vapors steam, given up by the steam as the steam


is

condenses back into water.


4-3. Superheated Vapor. A vapor at any
temperature above its saturation temperature
is a superheated vapor (Section 2-34). If, after

vaporization, a vapor is heated so that its

4-1. Saturation Temperature. When the temperature is raised above that of the vapor-
temperature of a liquid is raised to a point such izing liquid, the vapor is said to be superheated.
that any additional heat added to the liquid will In order to superheat a vapor it is necessary to
cause a part of the liquid to vaporize, the separate the vapor from the vaporizing liquid as
liquid is said to be saturated. Such a liquid is shown in Fig. 4-2. As long as the vapor remains
known as a saturated liquid and the tempera- in contact with the liquid it will be saturated.

ture of the liquid at that condition is called the This because any heat added to a liquid-
is

saturation temperature (Sections 2-31and 2-32). vapor mixture will merely vaporize more liquid
4-2. Saturated Vapor. The vapor ensuing and no superheating will occur.

from a vaporizing liquid is called a saturated Before a superheated vapor can be condensed,
vapor as long as the temperature and pressure the vapor must be de-superheated, that is, the
of the vapor are the same as those of the satur- vapor must first be cooled to its saturation
ated liquid from which it came. A saturated temperature. Heat removed from a super-
vapor may be described also as a vapor at a heated vapor will cause the temperature of the
temperature such that any further cooling of vapor to decrease until the saturation tempera-

the vapor will cause a portion of the vapor to ture is reached. At this point, any further
condense and thereby resume the molecular removal of heat will cause a part of the vapor to
structure of the liquid state. It is important condense.
to understand that the saturation temperature 4-4. Subcooled Liquid. If, after condensa-

of the liquid (the temperature at which the liquid tion, a liquid is cooled so that its temperature
will vaporize if heat is applied) and the satura- •is reduced below the saturation temperature, the

tion temperature of the vapor (the temperature liquidis said to be subcooled. Thus, a liquid at

at which the vapor will condense if heat is any temperature below the saturation tempera-
removed) are the same for any given pressure ture and above the fusion point is a subcooled
and that the liquid cannot exist as a liquid at any liquid.

temperature above its saturation temperature, 4-5.The Effect of Pressure on the Satura-
whereas a vapor cannot exist as a vapor at any tion Temperature. The saturation tempera-
temperature below its saturation temperature.* ture of a liquid or a vapor varies with the

* pressure. Increasing the pressure raises the


Under certain conditions it is possible to "super-
cool" water vapor momentarily below its saturation saturation temperature and decreasing the pres-
temperature. However, this is a very unstable sure lowers the saturation temperature. For
condition and cannot be maintained except momen- example, the saturation temperature of water at
tarily. atmospheric pressure (0 psig or 14.7 psia) is
43
44 PRINCIPLES OF REFRIGERATION
212° F. If the pressure over the water is in- the water at atmospheric pressure is 212° F,
creased from psig to 5.3 psig (20 psiaX the the temperature of the water will rise as the
saturation temperature of the water increases water is heated until it reaches 212° F. At this
from 212° F to 228° F. On the Other hand, if the point, if the heating is continued, the water will
pressure over the water reduced from 14.7
is begin to vaporize. Soon the space above the
psia to 10 psia, the new saturation temperature water will be filled with billions and billions of
of the water will be 193.2° F. Figure 4-3 is a water vapor molecules darting about at high

Condenser
water out""^
Steam gives up heat to
, cold water in condenser
/ and condenses into water

Cold water in

Saturated steam
-*
at212"F .{
^Con densed steam leaving
condenser-212°F

Heat added

Fig. 4VI. Saturated vapor.

graphical representation of the relationship velocities. Some of the vapor molecules will fall
between the pressure and the saturation tempera- back into the water to become liquid molecules
ture of water. again, whereas others will escape through the
To illustrate the effect of pressure on the opening to the outside and be carried away by
saturation temperature of a liquid, assume that air currents. If the opening at the top of the
water is confined in a closed vessel which is vessel is of sufficient size to allow the vapor to
equipped with a throttling valve at the top escape freely, the vapor will leave the vessel at
(Fig. 4-4a). A
compound gage is used to deter- the same rate that the liquid is vaporizing. That
mine the pressure exerted in the vessel and two is, the number of molecules which are leaving

thermometers are installed so that one records the liquid to become vapor molecules will be
the temperature of the water and the other the exactly equal to the number of vapor molecules
temperature of the vapor over the water. With which are leaving the space, either by escaping
the throttling valve wide open, the pressure to the outside or by falling back into the liquid.
exerted over the water is atmospheric (0 psig or Thus, the number of vapor molecules and the
14.7 psia). Since the saturation temperature of density of the vapor above the liquid will
SATURATED AND SUPERHEATED VAPORS 45

« r- iOiA-.t!j>.^.
:
-^j^~ -± Jt.i-^x4j—^ ^.j^Ls ^Steam superheated
in superheater

Saturated steam

/T7-X
5 "
TfT"/
H eat
Superheated
Fig. 4-2. Superheated vapor. steam
added

tint
Heat added

remain constant and the pressure exerted by the Suppose that the throttling valve is partially

vapor will be equal to that of the atmosphere closed so that the escape of the vapor from the
outside of the vessel. vessel is impeded somewhat. For a time the
Under this condition the water vapor ensuing equilibrium will be disturbed in that the vapor
from the vaporizing liquid will be saturated, will not be leaving the vessel at the same rate the
that temperature and pressure will be the
is, its liquid is vaporizing. The number of vapor
same as that of the water, 212° F and 14.7 psia. molecules in the space above the liquid will
The density of the water vapor at that tempera- increase, thereby increasing the density and the
ture and pressure will be 0.0373 lb/cu ft and its pressure of the vapor over the liquid and raising
specific volume will be 1 /0.0373 or 26.8 ft*/lb. the saturation temperature.
Regardless of the rate at which the liquid is If it is assumed that the pressure of the vapor
vaporizing, as long as the vapor is allowed to increases to 5.3 psig (20 psia) before equilibrium
escape freely to the outside so that the pressure is again established, that is, before the rate at
and density of the vapor over the liquid does which the vapor is escaping to the outside is

not change, the liquid will continue to vaporize exactly equal to the rate at which the liquid is

at 212° F. vaporizing, the saturation temperature will be

DUU

500

-.400
re
'3»
o.

Is 300 Fig. 4-3. Variation in the


saturation temperature of
a. 200 water with changes in pressure.

100

14.7
50 100 150 200| 250 300 350 400 450 500
212
Saturation temperature (*F)
46 PRINCIPLES OF REFRIGERATION

xtt v'
Valve wit,e °P en Valve partially closed

« /S^ Steam <„'•> ^i'-'i- Steam


^ 20 psia

Density-0.0373 Ib/cu ft Density-0.0498 Ib/cu ft

212'F
>228'F

Zz_Water_ H>:
:
>^^
:

(a) <b)

Fig. 4-4.

228° F, the density of the vapor will be 0.0498 and outside of the vessel is sufficient to allow
lb/cu ft, and 1 lb of vapor will occupy a volume the vapor to escape at a rate equal to that at
of 20.08 cu ft. This condition is illustrated in which the liquid is Vaporizing. The increase in
Fig. 4-46. pressure, of course, results in an increase in the
By comparing the condition of the vapor in saturation temperature and in the density of the
Fig. 4-4A with that of the vapor in Fig. 4-4a, it vapor. Likewise, any decrease in the rate of
will be noted that the density of the vapor is vaporization will have the opposite effect. The
greater at the higher pressure and saturation pressure and density of the vapor over the liquid
temperature. Furthermore, it is evident that the will decrease and the saturation temperature
pressure and the saturation temperature of will be lower.
liquid or vapor can be controlled by regulating Assume now that the throttling valve on the
the rate at which the vapor escapes from over container isagain opened completely, as in
the liquid. Fig. 4-4a, so that the vapor is allowed to escape
In Fig. 4-4«, the rate of vaporization will have freely and unimpeded from over the liquid. The
or no effect on the pressure and saturation
little density and pressure of the vapor will decrease
temperature because the vapor is allowed to until the pressure of the vapor is again equal to
escape freely so that the density and pressure of that of the atmosphere outside of the container.
the vapor over the liquid will neither increase Since the saturation temperature of water at
nor decrease as the rate of vaporization is atmospheric pressure is 212° F and since a liquid
changed. On the other hand, in Fig. 4-46, any cannot exist as a liquid at any temperature above
increase in the rate of vaporization will cause an its saturation temperature corresponding to its
increase in the density and pressure of the vapor pressure, evident that the water must cool
it is
and result in an increase in the saturation itself from 228° F to 212° F at the instant that
temperature. The reason any increase
is that the pressure drops from 20 psia to atmospheric
in the rate of vaporization will necessitate the pressure. To accomplish this cooling, a portion
escape of a greater quantity of vapor in a given of the liquid will "flash" into a vapor. The
length of time. Since the size of the vapor outlet latent heat necessary to vaporize the portion of
is fixed by the throttling action of the valve, the the liquid that flashes into the vapor state is
pressure of the vapor in the vessel will increase supplied by the mass of the liquid and, as a
until the pressure difference between the inside result of supplying the vaporizing heat, the
SATURATED AND SUPERHEATED VAPORS 47

temperature of the mass of the liquid will be


High energy molecules
reduced to the new saturation temperature. escape from surface
Enough of the liquid will vaporize to provide S" of liquid and become
f vapor molecules
the required amount of cooling.
4-6. Vaporization. The vaporization of a
liquid may occur in two ways: (1) by evapora-
tion and (2) by ebullition or "boiling." The
vaporization of a liquid by evaporation occurs
only at the free surface of the liquid and may
take place at any temperature below the satura-
tion temperature. On the other hand, ebullition
or boiling takes place both at the free surface
Water
and within the body of the liquid and can occur
Fig. 4-5. Evaporation from surface of a liquid.
only at the saturation temperature. Up to this
point, only ebullition or boiling has been
considered.
4-7. Evaporation. Evaporation is taking than others. Liquids havingthelowest "boiling"
place continually and the fact that water points, that is, the lowest saturation temperature

evaporates from lakes, rivers, ponds, clothes, for a given pressure, evaporate at the highest

etc., is sufficient evidence that evaporation can rate. However, for any particular liquid, the
and does occur at temperatures below the rate of vaporization varies with a number of
saturation temperature. Any liquid open to the factors. In general, the rate of vaporization
atmosphere, regardless of its temperature, will increases as the temperature of the liquid
gradually evaporate and be diffused into the increases and as the pressure over the liquid

air. decreases. Evaporation increases also with the

The vaporization of liquids at temperatures amount of exposed surface. Furthermore, it


below their saturation temperature can be will be shown later that the rate of evaporation

explained in this manner. The molecules of a is dependent on the degree of saturation of the

liquid are in constant and rapid motion, their vapor which is always adjacent to and above
velocities being determined by the temperature the liquid.
of the liquid. In the course of their movements 4-9. The Cooling Effect of Evaporation.

the molecules are continually colliding with one Since the higher velocity molecules (those
it is

another and, as a result of these impacts, some having the most energy) which escape from the
of the molecules of the liquid momentarily surface of an evaporating liquid, it follows that
attain velocities much higher than the average the average energy of the mass is thereby

velocity of the other molecules of the mass. reduced and the temperature of the mass
Thus, their energy is much greater than the lowered. Whenever any portion of a liquid
average energy of the mass. If this occurs vaporizes, an amount of heat equal to the latent

within the body of the liquid, the high velocity heat of vaporization must be absorbed by that
molecules quickly lose their extra energy in portion, either from the mass of the liquid,
subsequent collisions with other molecules. from the surrounding air, or from adjacent
However, if the molecules attaining the higher objects. Thus, the energy and temperature of
than normal velocities are near the surface, they the mass are reduced as it supplies the latent
may project themselves from the surface of the heat of vaporization to that portion of the
liquid and escape into the become vapor
air to liquid which vaporizes. The temperature of the

molecules. (Fig. 4-5). The molecules so mass is reduced to a point slightly below that of
escaping from the liquid are diffused throughout the surrounding media and the temperature
the air. They occupy the relatively large spaces difference so established causes heat to flow
which exist between the molecules of the air and from the surrounding media into the mass of the
become a part of the atmospheric air. liquid. The energy lost by the mass during
4-8. Rate of Vaporization. For any given vaporization thereby replenished and evapora-
is

temperature, some liquids will evaporate faster tion becomes a continuous process as long as
48 PRINCIPLES OF REFRIGERATION

Molecules escaping from


Molecules cannot
surface are carried away escape-fall back
by air so that evaporation into liquid to replace
is continuous those leaving

Vapor pressure
Water temperature
0.74 in. Hg
slightly below air >. Absolute density Saturated vapor
temperature >
0.001148 Ib/cu ft

70° F

Saturated liquid

(a)
(b)

Vapor pressure-
Vapor pressure—
1.13 in. Hg
Absolute density-
0.52 in. Hg
Absolute density—
0.001570 Ib/cu ft
60' F / 0.000828

Saturated vapor
Saturated vapor

60° F

Saturated liquid

(c)
(d)
Fig. 4-6.

any of the liquid remains. The vapor resulting


saturated, and no further evaporation will
from evaporation is diffused into and carried
occur. The energy of the liquid will be increased
away by the air. by the vapor molecules which are returning to
4-10. Confined Liquid-Vapor Mixtures. the liquid in exactly the same amount
that it is
When a vapor is confined in a container with a diminished by the molecules that are leaving.
portion of its own liquid, both the vapor and the Since no further cooling will take place
by
liquid will be saturated. To illustrate, assume evaporation, the liquid will assume the tempera-
that an open container is partially filled
with ture of the surrounding air and' heat
water and is stored where the ambient tempera- transfer
will cease. (Fig. 4-66).
ture is 70° F (Fig. 4-6a). The water will
be If, at this point, the ambient temperature
evaporating at 70° F and, as described in the
80° F, heat transfer will again
rises to, say,
previous section, the vapor molecules leaving take
place between the surrounding air and
the liquid will be diffused into the surrounding the
The temperature and average molecular
liquid.
air so that evaporation will continue
until all velocity of the liquid will be increased
of the liquid and
is evaporated. However, if a evaporation will be resumed. The number
tightly fitting cover is placed over the of
container, molecules leaving the liquid will again
the vapor molecules will be unable to be
escape to greater than the number returning and
the outside and they will collect above theliquid. the
density and pressure of the vapor
Soon the space above the liquid will be so filled above the
liquid will be increased. As the
density and
with vapor molecules that there will be as many
pressure of the vapor increase, the saturation
molecules falling back into the liquid as there
temperature of the liquid increases. Eventually,
are leaving the liquid. A condition of equi- when the saturation temperature reaches 80°
F
librium will be attained, the vapor will be and is equal to the ambient temperature, no
SATURATED AND SUPERHEATED VAPORS 49

further heat transfer will occur and evaporation 4-12. Condensation. Condensation of a
will cease. Equilibrium will have again been vapor may be accomplished in several ways: (1)
established. The density and pressure of the by extracting heat from a saturated vapor, (2)
vapor be greater than before, the saturation
will by compressing the vapor while its temperature
temperature of the liquid-vapor mixture will be remains constant, or (3) by some combination
higher, and there will be more vapor and less of these two methods.
liquid in the container than previously (Fig. 4-13.Condensing by Extracting Heat from
4-6c). a Saturated Vapor. A saturated vapor has
Suppose now that the ambient temperature been previously described as one at a condition
60° F.
falls to When this occurs, heat will flow such that any further cooling will cause a part
from the 80° F liquid-vapor mixture to the of the vapor to condense. This is because a
cooler surrounding air. As the liquid-vapor vapor cannot exist as a vapor at any tempera-
mixture loses heat to the surrounding air, its ture below its saturation temperature. When
temperature and average molecular velocity will the vapor is cooled, the vapor molecules cannot
be decreased and many of the vapor molecules, maintain sufficient energy and velocity to over-
lacking sufficient energy to remain in the vapor come the attractive forces of one another and
state, will fall back into the liquid and resume remain as vapor molecules. Some of the
the molecular arrangement of the liquid state; molecules, overcome by the attractive forces,
that is, a part of the vapor will condense. The will revert to the molecular structure of the
density and pressure of the vapor will be liquid state. When condensation occurs while
diminished and the saturation temperature of the vapor is confined so that the volume remains
the mixture will be reduced. When the satura- constant, the density and pressure of the vapor
tion temperature of the mixture falls to 60° F will decrease so that there is a decrease in the
it will be the same as the ambient temperature saturation temperature. If, as in a vapor
and no further heat flow will occur. Equilibrium condenser (Fig. 4-1), more vapor is entering
will have been established and the number of the vessel as the vapor condenses and drains
molecules re-entering the liquid will exactly from the vessel as a liquid, the density, pressure,
equal those which are leaving. At this new and saturation temperature of the vapor will
condition, the density and pressure of the vapor remain constant and condensation will continue
willbe less than before, the saturation tempera- as long as heat is continuously extracted from
ture will be lower,and since a part of the vapor the vapor.
condensed into liquid, there will be more liquid 4-14. Condensing by Increasing the Pres-
and less vapor comprising the mixture than at sure at a Constant Temperature. When a
the previous condition (Fig. 4-6d). vapor is compressed at a constant temperature,
4-11. Sublimation. It is possible for a its volume diminishes and the density of the
substance to go directly from the solid state vapor increases as the molecules of the vapor
to the vapor state without apparently passing are forced into a smaller volume. The satura-
through the liquid state. Any solid substance tion temperature of the vapor increases as the
will sublime at any temperature below its pressure increases until a point is reached where
fusion temperature. Sublimation takes place the saturation temperature of the vapor is equal
in a manner similar to evaporation, although to the actual temperature of the vapor. When
much slower, in that the higher velocity this occurs, the density of the vapor will be at a
molecules near the surface escape from the mass maximum value for that condition, and any
into the surrounding air and become vapor further compression will cause a part of the
molecules. One of the most familiar examples vapor to assume the more restrained molecular
of sublimation is that of solid C0 2 (dry ice), structure of the liquid state. Thereafter,
which, at normal temperatures and pressures, condensation will continue as long as com-
sublimes directly from the solid to the vapor pression continues so that the density and
state. Damp wash frozen on the line in the pressure of the remaining vapor cannot be
winter time will sublime dry. During freezing further increased.If the temperature of the
weather ice and snow will sublime from streets vapor is to remain constant, heat must be
and sidewalks, etc. removed from the vapor during the compression
50 PRINCIPLES OF REFRIGERATION

(Section 3-25). If heat not removed from the


is cause they can actually be measured. Enthalpy,
vapor, the of the vapor will
temperature internal energy, and entropy cannot be measured.

increase and condensation will not occur. They must be calculated and are therefore
A careful analysis
of Sections 4-13 and 4-14 known as calculated properties.
will show that in either case the vapor is Pressure, temperature, volume, and internal
brought to a saturated condition before conden- energy have already been discussed to some
sation begins and that heat is removed from extent. A
discussion of enthalpy and entropy
the vapor in order to bring about condensation. follows.
Furthermore, the vapor is saturated in each 4-18. Enthalpy. Enthalpy is a calculated

case only when the saturation temperature and property of matter which is sometimes loosely
the actual temperature of the vapor are the defined as "total heat content." More specifi-
same. cally, the enthalpy H
of a given mass of material
In Section 4-13, heat is removed from the at any thermodynamic condition is an
specified

vapor at a constant pressure until the tempera- expression of the total heat which must be
ture of the vapor falls to the saturation tempera- transferred to the material to bring the material
ture corresponding to its pressure, whereupon to the specified condition from some initial
the continued removal of heat causes a part of condition arbitrarily taken as the zero point of
the vapor to condense. enthalpy.
In Section 4-14, the pressure of the vapor Whereas the total enthalpy H represents the
is increased while the temperature of the vapor enthalpy of M pounds, the specific enthalpy
remains constant until the saturation tempera- h is the enthalpy of 1 lb. Since it is usually the
ture of the vapor corresponding to the increased specific enthalpy rather than the total enthalpy
pressure is equal to the actual temperature of which is of interest, hereafter in this text the
the vapor. In both cases, since the vapor must term enthalpy shall be used to mean specific
give up the latent heat of vaporization in order enthalpy, h, the enthalpy of 1 lb.

to condense, heat must be removed from the Since little is known about the specific heat or
vapor. the other properties of materials at low tempera-
4-15. Critical Temperature. The tempera- tures, it is not possible to determine absolute

ture of a gas may be raised to a point such values for the calculated properties. For this

that it cannot become saturated regardless of reason, values for the calculated properties
the amount of pressure applied. The critical must be determined from some arbitrarily
temperature of any gas is the highest tempera- selected zero point rather than from absolute
ture the gas can have and still be condensible zero.* For example, the zero point of enthalpy
, by the application of pressure. The critical for water and its vapor, steam, is taken as
a
temperature is different for every gas. Some water at 32 F under atmospheric pressure.
gases have high critical temperatures while the The enthalpy of 1 lb of water at 60° F then is the
^critical" temperatures of others are relatively total amount of heat which must be transferred

low. For example, the critical temperature of to the water in order to raise the temperature of
water vapor is 706° F, whereas the critical the water from 32° F to 60° F. According to
temperature of air is approximately -225° F. Equation 2-9, this is 28 Btu (1 x 1 x 28).
4-16. Critical Pressure. Critical pressure is Hence, based on the assumption that the
the lowest pressure at which a substance can enthalpy of water is zero at 32° F, the enthalpy
exist in the liquid state at its critical tempera- of water at 60° F is 28 Btu/lb.
ture; ihit is, it is the saturation pressure at the Mathematically, enthalpy is defined as
critical temperature.
Pv
44J. Igriportant Properties of Gases and (4-1)
Vapors. Although a gas or vapor has many
properties, only six are of particular importance * Since it is required to know the change in the
in the study of refrigeration. These are pressure, enthalpy of the working fluid during a process,
temperature, volume, enthalpy, internal energy, rather than the absolute enthalpy at some par-
and entropy. Pressure, temperature, and ticular condition, the fact that absolute enthalpy

volume are called measurable properties be- cannot be calculated is of little consequence.
SATURATED AND SUPERHEATED VAPORS 51

Standard atmospheric pressure


2105 psfa (14.6% x 144)
i
s- Final condition
Fig. 4-7. Pressure-volume « P
diagram showing the external
work done by fluid expansion
as lb of water is vaporized
I

at atmospheric pressur
S
approximately 59,000 ft-lb. a. Specific volume of
1 lb of steam at 212°
26.8 cu ft/lb

Volume cubic feet

where h = the specific enthalpy in Btu/lb The enthalpy of vaporization (latent heat of
u = the specific internal energy in Btu/lb vaporization) of water at 212° F is 970.4 Btu. Of
P= the pressure in psfa this amount, only 897.6 Btu actually increases
v = the specific volume in cubic feet the internal energy and represents energy in
J =
the mechanical energy equivalent storage in the vapor. The other 72.8 Btu leaves
Ithas been demonstrated (Section 3-12) that the vapor as the work of expansion and is
all the heat transferred to a fluid is not neces- represented by the term PvjJ. A PV diagram of
sarily stored in the fluid as an increase in the the vaporization process is shown in Fig. 4-7.
internal energy of the fluid. In many cases, 4-19. Entropy. Entropy, like enthalpy, is a
some part or all or the heat transferred to the calculated property of matter. The entropy S
fluid passes through the fluid and leaves the of a given mass of material at any specified
fluid aswork. In Equation 4-1, that part of the condition is an expression of the total heat

transferred energy which is stored in the fluid transferred to the material per degree of abso-
as an increase in the internal energy is repre- lute temperature to bring the material to that
sented by the term u, whereas that part of the condition from some initial condition taken as
transferred heat which leaves the fluid as work the zero of entropy.
is represented by the term PvjJ. Notice that, Since it is not possible to calculate the
although the energy represented by the term absolute value of entropy, entropy values, like
PvjJ, does not increase the internal energy of the those of enthalpy, are based on an arbitrarily
fluid and is not stored in the fluid, it neverthe- selected zero point. The zero points of entropy
less represents energy which must be trans- and enthalpy are the same for any one fluid.
ferred to the fluid in order to bring the fluid Hence, for water and its vapor, steam, the zero
to the specified condition from the initial point of entropy is taken as water at 32° F.
condition at the arbitrarily selected zero point of Again, as in the case of enthalpy, it is the
enthalpy. Furthermore, even though the specific entropy s rather than the total entropy
external work energy is not stored in the fluid, S which is useful. Therefore, in this book, the
itmust pass back through the fluid and be given term entropy shall be used to mean specific
up by the fluid as the fluid returns to the initial entropy s rather than the total entropy S.
condition. It has been shown (Section 3-22) that the

Consider, for example, the vaporization of 1 mechanical energy or work of a process can be
lb of water into steam at 212° F under atmos- expressed as the product of the change in
pheric pressure. The volume of 1 lb of water at volume and the average absolute pressure.
212° F is 0.01670 cu ft whereas the volume of 1 Likewise, it is often convenient to express the
lb of steam at 212° F is 26.82 cu ft. Hence, the heat energy transferred during a process as the
fluid expands from a volume of 0.0167 cu ft to a product of two factors. The concept of entropy
volume of 26.82 cu ft during the vaporization makes this possible. The heat energy trans-
thereby doing work in expanding against the ferred during a process can be expressed as the
pressure of the atmosphere. product of the change in entropy and the
52 PRINCIPLES OF REFRIGERATION

672*R(212'F + 460)

Fig. 44

Specific entropy of
saturated steam at 212° F

g]/ 1.7566 Btu/lb/°R

Entropy (Btu/lb/'R)

average absolute temperature. * Mathematically may take place entirely within the fluid itself

the relationship is expressed by the following as a result of internal friction.However, the


equations: entropy of a fluid is not affected by external
AQ = As x Tm (4-2) work done either by or on the fluid. Thus in a
frictionless (occurring without either internal
A <2
a (4-3) or external friction), adiabatic (no heat transfer
* m to or from an external body) compression, as in
the ideal compression of the refrigerant vapor
AC

T =
lm (4-4) ina refrigeration compressor, the entropy of the
As
fluid willremain the same or constant.
where AQ = the heat energy transferred in Btu 4-20. Vapor Tables. It has been stated
As = the change in entropy in Btu per previously that a vapor does not approach the
pound per ° R condition of an ideal gas because of the inter-
Tm = the average absolute temperature molecular forces which exist between the
in°R molecules of the vapor. Therefore, internal
On a pressure-volume diagram (Fig. 4-7), the friction is presentwhenever a vapor undergoes a
"area under the curve," which is the product change of condition so that the various proper-
of the change in volume and the average abso- ties of a vapor at the different conditions cannot

lute pressure, represents the work of the process. be determined by applying the laws of ideal
Similarly, on a temperature-entropy diagram
(temperature plotted against entropy), the The properties of vapors at various conditions
"area under the curve," which is the product have been determined by experiment for all
of the entropy change and the average absolute common vapors and these data are published in
temperature, represents the heat transferred the form of tables. Separate tables are used for
during the process (Fig. 4-8). saturated and superheated vapors.
Although the mathematical treatment of 4-21. Saturated Vapor Tables. Saturated
entropy not required in the study of refrigera-
is vapor tables (Fig. 4-9) deal only with saturated
tion and is beyond the scope of this book, it is liquids and vapors, and usually give values for
important to note that according to Equation the following properties: (1) temperature, (2)
4-2 the entropy changes only when heat is pressure, (3) specific volume, (4) enthalpy
transferred during the process. If there is no (specific), and (5) entropy (specific). Normally,
heat energy transfer, there is no change in the the temperature in degrees Fahrenheit is listed

entropy. The heat energy transfer may occur in the extreme left-hand column. The pressure
either to or from an external source or sink or it is given in the second and third columns,
* followed by the specific volume in cubic feet
The average absolute temperature is not merely
the mean of the initial and final temperatures of the for both the liquid and the vapor in the fourth
process, but is the average of all of the absolute and fifth columns, respectively. Some tables
temperatures through which the process passes. list the density in addition to or in place of the
SATURATED AND SUPERHEATED VAPORS 53

Properties of Saturated Steam

Absolute Pressure Specific Volume Enthalpy Entropy

Temp., Sat. Sat. Sat. Sat. Sat. Sat.

"F, Psi, In. Hg, liquid, Evap., vapor, liquid, Evap., vapor, liquid, Evap., vapor,

P Vg h, hfg A. s, St, s.
t P
(1) (2) (3) (4) (5) (6) (7)« (8) (9) (10) (11) (12)

200 11.526 23.467 0.01663 33.62 33.64 167.99 977.9 1145.9 0.2938 1.4824 1.7762
202 12.011 24.455 0.01665 32.35 32.37 170.00 976.6 1146.6 0.2969 1.4760 1.7729
204 12.512 25.475 0.01666 31.14 31.15 172.02 975.4 1147.4 0.2999 1.4697 1.7696
206 13.031 26.531 0.01667 29.97 29.99 174.03 974.2 1148.2 0.3029 1.4634 1,7663
208 13.568 27.625 0.01669 28.86 28.88 176.04 972.9 1148.9 0.3059 1.4571 1.7630

210 14.123 28.755 0.01670 27.80 27.82 178.05 971.6 1149.7 0.3090 1.4508 1.7598
212 14.696 29.922 0.01672 26.78 26.80 180.07 970.3 1150.4 0.3120 1.4446 1.7566
214 15.289 31.129 0.01673 25.81 25.83 182.08 969.0 1151.1 0.3149 1.4385 1.7534
216 15.901 32.375 0.01674 24.88 24.90 184.10 967.8 1151.9 0.3179 1.4323 1.7502
218 16.533 33.662 0.01676 23.99 24.01 186.11 966.5 1152.6 0.3209 1.4262 1.7471

220 17.186 34.992 0.01677 23.13 23.15 188.13 965.2 1153.4 0.3239 1.4201 1.7440
222 17.861 36.365 0.01679 22.31 22.33 190.15 963.9 1154.1 0.3268 1.4141 1.7409
224 18.557 37.782 0.01680 21.53 21.55 192.17 962.6 1154.8 0.3298 1.4080 1.7378
226 19.275 39.244 0.01682 20.78 20.79 194.18 961.3 1155.5 0.3328 1.4020 1.7348
228 C2O.O160 40.753 0.01683 20.06 20.07 196.20 960.1 1156.3 0.3357 1.3961 1.7318

230 20.780 42.308 0.01684 19.365 19.382 198.23 958.8 1157.0 0.3387 1.3901 1.7288
240 24.969 50.837 0.01692 16.306 16.323 208.34 952.2 1160.5 0.3531 1.3609 1.7140
250 29.825 60.725 0.01700 13.804 13.821 218.48 945.5 1164.0 0.3675 1.3323 1.6998
260 35.429 72.134 0.01709 11.746 11.763 228.64 938.7 1167.3 0.3817 1.3043 1.6860
270 41.858 85.225 0.01717 10.044 10.061 238.841 931.8 1170.6 0.3958 1.2769 1.6727

Fig. 4-9. Excerpt from typical saturated vapor table.

Reproduced from Thermodynamic Properties of Steam by Keenan and Keyes, published by John Wiley and
Sons, 1936, with permission.

specific volume. If the density only is given latent heat of vaporization at the pressure and
and the volume is wanted, the specific
specific temperature indicated; and (3) the enthalpy of
volume is determined by dividing the density the vapor (h g ), which is the sum of the enthalpy
into one. Likewise, when the specific volume is of the liquid (hf) and the enthalpy of vapor-
given and the density is wanted, the density is ization (hfa). For example, the enthalpy of the
found by dividing the specific volume into one liquid (hf) for water at 212° F under atmospheric
(Section 3-4). pressure is 180 Btu (1 x 1 x 180), whereas the

Three values for enthalpy h are usually enthalpy of the saturated water vapor at 212° F
given in the saturated vapor tables: (1) the under atmospheric pressure is 1050 Btu, which
enthalpy of the liquid (ht), which is the heat is the sum of the enthalpy of the liquid (180 Btu)
required to raise the temperature of the liquid and the enthalpy of vaporization (970 Btu).
from the temperature at the assumed zero point Two values of entropy are usually given:
of enthalpy to the saturation temperature corre- sf , the entropy of the liquid and s g , the entropy
sponding to the pressure of the liquid; (2) the of the vapor, the difference between the two
enthalpy of vaporization (hfg), which is the being the change in entropy during vaporization.
54 PRINCIPLES OF REFRIGERATION

Dichlorodifluoromethane (Refrigerant-12)
Properties of Superheated Vapor

Abs. Pressure 36 lb/in. s Abs. Pressure 38 lb/in.* Abs. Pressure 40 lb/in. 2 Abs. Pressure 42 lb/in. 2
Temp. Gage Pressure 21.3 lb/in. 8 Gage Pressure 23.3 lb/in. 8 Gage Pressure 25.3 lb/in. 8 Gage Pressure 27.3 lb/in. 8
°F (Sat. Temp. 20.4° F) (Sat. Temp. 23.2° F) (Sat. Temp. 25.9° F) (Sat. Temp. 28.5° F)

/ V H S V H S V H S V H S

(at
sat'n) {1.113) {80.54) {0.16947) {1.058) {80.86) {0.16931) {1.009) {81.16) (0.16914) (0,963) (81.44) (0.16897)
30 1.140 81.90 0.17227 1.076 81.82 0.17126 1.019 81.76 0.17030 0.967 81.65 0.16939
40 1.168 83.35 0.17518 1.103 83.27 0.17418 1.044 83.20 0.17322 0.991 83.10 0.17231

50 1.196 84.81 0.17806 1.129 84.72 0.17706 1.070 84.65 0.17612 1.016 84.56 0.17521
60 1.223 86.27 0.18089 1.156 86.19 0.17991 1.095 86.11 0.17896 1.040 86.03 0.17806
70 1.250 87.74 0.18369 1.182 87.67 0.18272 1.120 87.60 0.18178 1.063 87.51 0.18086
80 1.278 89.22 0.18647 1.208 89.16 0.18551 1.144 89.09 0.18455 1.087 89.00 0.18365
90 1.305 90.71 0.18921 1.234 90.66 0.18826 1.169 90.58 0.18731 1.110 90.50 0.18640

100 1.332 92.22 0.19193 1.260 92.17 0.19096 1.194 92.09 0.19004 1.134 92.01 0.18913
110 1.359 93.75 0.19462 1.285 93.69 0.19365 1.218 93.62 0.19272 1.158 93.54 0.19184
120 1.386 95.28 0.19729 1.310 95.22 0.19631 1.242 95.15 0.19538 1.181 95.09 0.19451
130 1.412 96.82 0.19991 1.336 96.76 0.19895 1.267 96.70 0.19803 1.204 96.64 0.19714
140 1.439 98.37 0.20254 1.361 98.32 0.20157 1.291 98.26 0.20066 1.227 98.20 0.19979

ISO 1.465 99.93 0.20512 1.387 99.89 0.20416 1.315 99.83 0.20325 1.250 99.77 0.20237
160 1.492 101.51 0.20770 1.412 101.47 0.20673 1.340 101.42 0.20583 1.274 101.36 0.20496
170 1.518 103.11 0.21024 1.437 103.07 0.20929 1.364 103.02 0.20838 1.297 102.96 0.20751
180 1.545 104.72 0.21278 1.462 104.67 0.21183 1.388 104.63 0.21092 1.320 104.57 0.21005
190 1.571 106.34 0.21528 1.487 106.29 0.21433 1.412 106.25 0.21343 1.343 106.19 0.21256

200 1.597 107.97 0.21778 1.512 107.93 0.21681 1.435 107.88 0.21592 1.365 107.82 0.21505
210 1.623 109.61 0.22024 1.537 109.57 0.21928 1.459 109.52 0.21840 1.388 109.47 0.21754
220 1.650 111.27 0.22270 1.562 111.22 0.22176 1.482 111.17 0.22085 1.411 111.12 0.22000
230 1.676 112.94 0.22513 1.587 112.89 0.22419 1.506 112.84 0.22329 1.434 112.80 0.22244
240 1.702 114.62 0.22756 1.612 114.58 0.22662 1.530 114.52 0.22572 1.457 114.49 0.22486

250 1.728 116.31 0.22996 1.637 116.28 0.22903 1.554 116.21 0.22813 1.480 116.19 0.22728
260 1.754 118.02 0.23235 1.662 117.99 0.23142 1.577 117.92 0.23052 1.502 117.90 0.22967
270 1.780 119.74 0.23472 1.687 119.71 0.23379 1.601 119.65 0.23289 1.524 119.62 0.23204
280 1.807 121.47 0.23708 1.712 121.45 0.23616 1.625 121.40 0.23526 1.547 121.36 0.23441
290 1.833 123.22 0.23942 1.737 123.20 0.23850 1.649 123.15 0.23760 1.570 123.11 0.23675

300 1.762 124.95 0.24083 1.673 124.92 0.23994 1.592 124.87 0.23909

Fig. 4-10. Excerpt from typical superheated vapor table.

Copyright by E. I. du Pont de Nemours and Co., Inc. Reprinted by permission.

It has been stated previously that the con- Fig. 4-9and reading across the table, the values
dition of a gas or a vapor can be determined (set off by the heavy lines) for all the other
when any two of its properties are known. How- properties of the vapor at this condition can be
ever, for a saturated liquid or vapor at any one obtained.
pressure, there is only one temperature that the 4-22. Superheated Vapor Tables. A super-
fluid can have and still satisfy the conditions of heated vapor table deals with the properties of
saturation. This is true also for the other a superheated vapor rather than those of a
properties of a saturated liquid or vapor. saturated vapor, and the arrangement of a
Therefore, if any one property of a saturated superheated vapor table is somewhat different
liquid or vapor is known, the value of the other from that of a saturated vapor table. One
properties can be read directly from the satu- common form of the superheated vapor table is
rated vapor table. For instance, assume that illustrated in Fig. 4-10.
the pressure of one pound of dry saturated Before examining the superheated vapor
steam is 20 psia. By locating 20 psia (encircled) table, it important to take note of one
is

in the second column of the abbreviated table in significant difference between a saturated and a
SATURATED AND SUPERHEATED VAPORS 55

Whereas, for a saturated (d) The amount of superheat in the vapor in


superheated vapor.
vapor at any one pressure there is only one Btu
(e) The change in the volume during the
temperature which will satisfy the conditions
superheating
of saturation, a superheated vapor may have
(/) The change in entropy during the super-
any temperature above the saturation tempera- heating
ture Corresponding to its pressure. The specific

volume, enthalpy, and entropy of a superheated


Solution
vapor at any one pressure will vary with the (a) From the head of
temperature. This does not mean that the the table, the satura-
properties of a superheated vapor are entirely tion temperature
independent of the pressure of the vapor but corresponding to
only that the properties of the superheated 40 psia = 25.9° F
vapor at any one pressure will vary with the From the body of
temperature. As a matter of fact, superheated the table (first

vapor tables are based on the pressure of reading, itali-

cized),thespecific
the vapor, and before the properties of a super-
volume of the
heated vapor can be determined from a table, the
vapor at satura-
pressure of the vapor or one of the properties
tion = 1.009 cu ft/lb
of the vapor at saturation must be known.
The enthalpy of the
When one of the properties of the vapor at
vapor at satura-
saturation known, the pressure of the vapor
is
= 81.16 Btu/lb
tion
can be found by consulting a saturated vapor
The entropy of the
table.
vapor at satura-
In addition to the properties of the super- tion = 0.16914 Btu/lb/° R
heated vapor at various temperatures above the
saturation temperature corresponding to the (b) From the body
pressure, superheated vapor tables usually list of the table, the
properties of the
some or all of the properties of the vapor at the
vapor superheated
saturation temperature. For example, in Fig.
to 50° F (offset by
4-10, the absolute and gage pressures, along
heavy lines in Fig.
with the saturation temperature corresponding 4-10) the specific
to these pressures, are given at the head of volume = 1.070 cu ft/lb
the table. The first readings in the body of the = 84.65 Btu/lb
The enthalpy
table (italicized) lists the specific volume, the = R
The entropy 0.17612 Btu/lb/
enthalpy, and the entropy of the vapor at
saturation. The specific volume, enthalpy, and (c) The superheated
entropy of the superheated vapor at various temperature = 50.0° F
temperatures above the saturation temperature The temperature at
make up the body of the table. Notice that the saturation = 25.9° F
temperature of the superheated vapor, given in The degree of super-
the extreme left-hand column, is listed in 10° F heat of the vapor
increments. in degrees Fahr-
enheit = 24.1°F
Example 4-1. One pound of superheated (rf)The enthalpy of
Refrigerant-12 vapor is at a temperature of
the superheated
50° F and its pressure is 40 psia. From the = 84.65 Btu/lb
vapor
abbreviated table in Fig. 4-10, determine:
The enthalpy of the
(a) The temperature, volume, enthalpy, and
vapor at satura-
entropy of the vapor at saturation
tion = 81.16 Btu/lb
(fc) The volume, enthalpy, and entropy of the

vapor at the superheated condition The amount of


(c) The degree of superheat of the vapor in superheat in the
degrees Fahrenheit vapor in Btu = 3.49 Btu/lb
56 PRINCIPLES OF REFRIGERATION
(e) The entropy of the
(/) The volume of the
superheated vapor = 0.17612 Btu/lb/°R superheated vapor = 1.070 cu ft/lb
The entropy of the The volume of the
vapor at satura- vapor at satura-
tion = 0.16914 Btu/lb/° R tion = 1.009 cu ft/lb
The change in The change in vol-
entropy during ume during the
the superheating = 0.00698 Btu/lb/° R superheating = 0.061 cu ft/lb
The volume occupied by any given weight of
air depends upon the pressure and temperature
of the air, and varies inversely with the baro-
metric pressure and directly with the absolute
temperature. Air very nearly approaches the
condition of an ideal gas and will follow the gas
laws with sufficient accuracy for all practical

5 purposes. Therefore, the volume occupied by


any given weight of air at any given pressure
and temperature can be determined by applying
Equation 3-10.
Psych rometric Example 5-1. Determine the volume oc-
cupied by 1 lb of air having a temperature of
Properties of Air 70° F at standard sea level pressure (14.7 psia).

Solution. Rearranging y_ MxR x T


and applying Equation P
3-10, 1 x 53.3 x
(70 + 460)
5-1. Composition of Air. Air is a mecha- 14.7 x 144
nical mixture of gasesand water vapor. Dry = 13.34 cu ft
air (air without water vapor) is composed
Example 5-2. Determine the volume of the
chiefly of nitrogen (approximately 78% by
air in Example 5-1 if the barometric pressure is
volume) and oxygen (approximately 21 %), the 12.6 psia.
remaining 1 % being made up of carbon dioxide
and minute quantities of other gases, such as Solution. Applying 1 x 53.3 x
Equation 3-10, (70 + 460)
hydrogen, helium, neon, argon, etc. With V=
regard to these dry air components, the com- 12.6 x 144
position of the air is practically the same = 15.57 cu ft

everywhere. On the other hand, the amount of Example 5-3. Determine the volume of the
water vapor in the air varies greatly with the air in Example 5-1 if the temperatureof the air
particular locality and with the weather con- is 100° F.
ditions. Since the water vapor in the air results
Solution. Applying 1 x 53.3 x
primarily from the evaporation of water from Equation 3-10,
K=
(100 + 460)
the surface of various bodies of water, atmos-
14.7 x 144
pheric humidity (water vapor content) is
= 14.10 cu ft
greatest in regions located near large bodies of
water and is less in the more arid regions. The
relationship between the volume and the
Since all air in the natural state contains a weight of a given quantity of air at any condition
certain amount of water vapor, no such thing is expressed by the following equations:

as "dry air" actually exists. Nevertheless, the V= Mxv (5-1)


concept of "dry air" a very useful one in that
is
V
it greatly simplifies psychrometric calculations. M =-
v
(5-2)
Hereafter in this book the term "dry air"
used to denote air without water vapor, whereas
is
where M = the weight of air in pounds
the term "air" is used to mean the natural V= the volume of M pounds of air in

mixture of "dry air" and water vapor. cubic feet


5-2. Air Quantities. Air quantities may be v = the specific volume of the air in
stated either in units of volume (cubic feet) or in cubic feet per pound
units of weight (pounds) so that the need for
Example 5-4. Air at a temperature of
converting air quantities from one unit of 95° F circulated over a cooling coil at the
is
measure to the other occurs frequently. rate of 2000 cu ft/min (cfm). If the specific
57
58 PRINCIPLES OF REFRIGERATION

volume of the air is 14.38 cu ft/lb, determine partial pressures exerted by the individual
the weight of air passing over the coil in pounds gases or vapors.
per hour. Air, being a mechanical mixture of gases and
2000 water vapor, obeys Dalton's law. Therefore,
Solution. Applying
Equation 5-2, the weight
M _ 14.38 the total barometric pressure is always equal to
of air passing over the = 139.2 lb/min the sum of the partial pressures of the dry gases
cooling coil and the partial pressure of the water vapor.
Multiplying by 60 min M- 139.1 x 60 Since psychrometry is the study of the properties

= 8346 lb/hr of air as affected by the water vapor content,


partial exerted by pressures the
5-3. Standard Air. Because of the difference the individual
dry gases are unimportant and, for all practical
in the volume of any given weight of air at
-purposes, the total barometric pressure may be
various temperatures and pressures, an air
considered to be the sum of only two pressures:
standard has been established for use in the
(1) the partial pressure exerted by the dry gases
rating of air handling equipment so that all
and (2) the partial pressure exerted by the water
equipment is rated at equal conditions. Dry air
vapor.
having a specific volume of 1 3.34 cu ft per pound
or a density of 0.07496 (0.075) lb per cu ft
5-5. Dew Point Temperature. It is im-
portant to recognize that the water vapor in
(1/13.34) is defined as standard air. Air at a
is actually steam at low pressure and that
the air
temperature of 70° F and at standard sea level
thislow pressure steam, like high pressure steam
pressure has this specific volume and density
will be in a saturated condition when its
(see Example 5-1).
temperature is the saturation temperature corre-
A given volume of air at any condition can be
sponding to its pressure. Since all of the com-
converted to an equivalent volume of standard
ponents in a gaseous mixture are at the same
air by applying the following equation
temperature, it follows that when air is at any

(5-3) temperature above the saturation temperature


corresponding to the partial pressure exerted by
where Vs = the equivalent volume of standard the water vapor the water vapor in the air will
air be superheated. On the other hand, when air
Va = the actual volume of the air at any is at a temperature equal to the saturation

given condition temperature corresponding to the partial


v a =* the specific volume of the air at the pressure of the water vapor, the water vapor in
given condition the air is saturated and the air is said to be
v, = the specific volume of standard air saturated (actually it is only the water vapor
(13.34 cu ft/lb) which is saturated). The temperature at which
the water vapor in the air is saturated is known
Example 5-5. Example 5-4,
For the air in
as the dew point temperature of the air. Ob-
determine the equivalent volume of standard
viously, then, the dew point temperature of the
air.
air isalways the saturation temperature corre-
Solution. Applying sponding to the partial pressure exerted by the
Equation 5-3, the equiv- _ 2000 x 14.38
water vapor. Hence, when the partial pressure
alent volume of standard 13.34
exerted by the water vapor is known, the dew
airK, = 2155 cfm
point temperature of the air can be determined
5-4. Dalton's Law of Partial Pressure. from the steam tables. Likewise, when the dew
Dalton's law of partial pressures states in effect point temperature of the air is known, the
that in any mechanical mixture of gases and partial pressure exerted by the water vapor can
vapors (those which do not combine chemically) be determined from the steam tables.
(1) each gas or vapor in the mixture exerts an
Example 5-6. Assume that a certain quan-
individual partial pressure which is equal to the has a temperature of 80° F and that
tity of air
pressure that the gas would exert if it occupied the partial pressure exerted by the water vapor
the space alone and (2) the total pressure of the in the air is 0.17811 psia. Determine the dew
gaseous mixture is equal to the sum of the point temperature of the air.
PSYCHROMETRIC PROPERTIES OF AIR 59

Solution. From Table 4-1, the saturation will be saturated. It is important to notice that
temperature of steam corresponding to a pres- the higher the temperature of the air, the higher
sure of 0.17811 psia is 50° F. Therefore, 50° F is the maximum possible vapor pressure and the
is the dew point temperature of the air.
greater is the maximum possible water vapor
content.
Example 5-7. A
certain quantity of air has
a temperature of 80° F and a dew point tem- 5-7. Absolute Humidity. The water vapor
perature of 40° F. Determine the partial pres- in the air is called humidity. The absolute
sure exerted by the water vapor in the air. humidity of the air at any given condition is
denned as the actual weight of water vapor
Solution. From Table 4-1, the saturation contained in 1 cu ft of air at that condition.
pressure corresponding to 40° F is 0.12170 psia
Since the weight of water vapor contained in the
and therefore 0.12170 psia is the partial pressure
air is relatively small, it is often measured in
exerted by the water vapor.
grains rather than in pounds (7000 grains equal
It has been shown (Section 4-5) that the lib).
pressure exerted by any vapor is directly 5-8. The Psychrometric Tables. It was
proportional to the density (weight per unit shown in Section 5-5 that the actual weight of
volume) of the vapor. Since the dew point water vapor contained in a unit volume of air is
temperature of the air depends only on the solely a function of the dew point temperature
partial pressure exerted by the water vapor, it of the air. Because of this fixed relationship
follows that, for any given volume of air, the between the dew point temperature and the
dew point temperature of the air depends only absolute humidity of the air, when the value
upon the weight of water vapor in the air. As of one is known, the value of the other can be
long as the weight of water vapor in the air readily computed.
remains unchanged, the dew point temperature The absolute humidity of air at various dew
of the air will also remain unchanged. If the point temperatures is listed and
in Tables 5-1
amount of water vapor in the air is increased or 5-2. The dew point temperatures are listed in
decreased, the dew point temperature of the air column (1) of the tables, and the absolute
will alsobe increased or decreased, respectively. humidity corresponding to each of the dew
Increasing the amount of water vapor in the air point temperatures is given in columns (4) and
will increase the pressure exerted by the water (5). The values given in column (4) are in
vapor and raise the dew point temperature. pounds of water vapor per cubic foot of air,
Likewise, reducing the amount of water vapor whereas the values given in column (5) are in
in the air will reduce the pressure of the water grains of water vapor per cubic foot of air.
vapor and lower the dew point temperature. Too, the partial pressure (saturation pressure)
5-6. Maximum Water Vapor Content. of the vapor corresponding to each dew point
The maximum amount of water vapor that can temperature is given in inches of mercury in
be contained in any given volume of air depends column (2) and in pounds per square inch in
only upon the temperature of the air. Since the column (3).
amount of water vapor in the air determines the 5-9. Relative Humidity. Relative humidity
partial pressure exerted by the water vapor, it (RH), expressed in percent, is the ratio of the
is evident that the air will contain the maximum actual weight of water vapor per cubic foot of
amount of water vapor when the water vapor air relative to the weight of water vapor con-
in the air exerts the maximum possible pressure. tained in a cubic foot of saturated air at the
Since the maximum pressure that can be same temperature, viz:
exerted by any vapor is the saturation pressure Actual weight of
corresponding to its temperature, the air will water vapor per
contain the maximum weight of water vapor
when the pressure exerted by the water vapor Relative humidity
J
= ...
cubic foot of air
. .
Weight of water
-z
—x 100
is equal to the saturation pressure corresponding
vapor in 1 cu ft of
to the temperature of the air. At this condition
saturated air at the
the temperature of the air and the dew point
same temperature
temperature will be one and the same and the air
60 PRINCIPLES OF REFRIGERATION
For instance, if air at a certain temperature given in Tables 5-1 and 5-2 apply only to air at
contains only half as much water vapor per standard barometric pressure.
cubic foot of air as the air could contain at that The specific humidity of the air at any given
temperature if it were saturated, the relative dew point temperature increases as the total
humidity of the air is 50%. The relative barometric pressure decreases and decreases as
humidity of saturated air, of course, is 100%. the total barometric pressure increases. The
reason for this is easily explained. It has been
Example Air at a temperature of
5-8.
shown (Examples and
5-1 5-3) that the volume
80° F has a dew point temperature of 50° F.
occupied by 1 lb of air increases as the total
Determine the relative humidity.
barometric pressure decreases. Since the density
Solution. From of a vapor varies inversely with the volume, it
Table 5-2, absolute follows that the weight of water vapor required
humidity correspond- to produce a given vapor density and vapor
ing to dew point tem-
pressure increases as the volume of the air
perature of 50° F = 4.106 grains/cu ft
increases. Likewise, as the volume occupied by
Absolute humidity 1 lb of air diminishes, the weight of water vapor
of saturated air at
required to produce a certain vapor density and
80° F 11.04 grains/cu ft
vapor pressure also diminishes.
Applying Equation _
— 4.106 Percentage Humidity.
5-4, the relative humid-
x 1/MV
100 5-11. Percentage
11,04 humidity is defined as the ratio of the actual
ity of the air = 37.1 % weight of water vapor in the air per pound of dry

Example 5-9. Determine the air to the weight of water vapor required to
relative hu-
midity of the air in Example 5-8, if the air is saturate completely 1 lb of dry air at the same
cooled to 60° F. (Note the dew point tempera-
: temperature. Percentage humidity, like relative
ture of the air does not change because the humidity, is given in percent. Notice, however,
moisture content does not change.) that percentage humidity is associated with the

weight of water vapor per unit weight of air,


Solution. From Table
whereas relative humidity is associated with the
5-2, absolute humidity
corresponding to dew weight of water vapor per unit volume of air.

point of 50° F = 4.106 grains/cu ft For reason the percentage humidity varies
this

Absolute humidity of with the total barometric pressure, whereas


saturated air at 60° F = 5.795 grains/cu ft relative humidity does not.
Applying Equation 5-4, _ 4.106
Example 5-10. Air at standard sea level
the relative humidity of 5.795 pressure has a temperature of 80° F and a dew
the air = 70.8% point temperature of 50° F. Determine the
specific humidity and percentage humidity of
5-10. Specific Humidity. The specific humid-
the air.
ity is the actual weight of water vapor mixed
with 1 lb of dry air and is usually stated in Solution. From Table
grains per pound, that is, grains of water vapor 5-2, the specific humidity
per pound of dry air. For any given barometric of the air in grains per
pressure, the specific humidity is a function of pound corresponding to
the dew point temperature alone. The specific a 50° F dew point tem-
humidity of air at various dew point tempera- perature (Column 7) 53.38 grains/lb

tuies Columns 6 and 7 of Tables 5-1


is listed in Specific humidity of
and 5-2. In Column 6, the specific humidity is saturated air at 80° F
given in pounds of water vapor per pound of dry
(Column 7) 155.50 grains/lb

air, whereas in Column 7 the specific humidity


Percentage humidity 53.38
x 100
is given in grains of water vapor per pound of 155.50
dry air. Since the specific humidity correspond- 34.3%
ing to any given dew
point temperature varies Note. Compare this value with the relative
with the total barometric pressure, the values humidity obtained in Example 5-8.
PSYCHROMETRIC PROPERTIES OF AIR 61

5-12. Dry Bulb and Wet Bulb Tempera-


tures. The dry bulb (DB) temperature of the
air is the temperature as measured by an ordinary
dry bulb thermometer. When measuring the dry
Swivel
bulb temperature of the air, the bulb of the ther-
connection
mometer should be shaded to reduce the effects
of direct radiation.
The wet bulb (WB) temperature of the air is
the temperature as measured by a wet bulb ther-
mometer. A wet bulb thermometer is an ordi-
nary thermometer whose bulb is enclosed in a
wetted cloth sac or wick. To obtain an accurate
reading with a wet bulb thermometer, the wick
should be wetted with clean water at approxi-
mately the dry bulb temperature of the air and
the air velocity around the wick should be main-
tained between 1000 and 2000 ft per minute. As
a practical matter, this velocity can be simulated
in still air by whirling the thermometer about on
the end of a chain. An instrument especially
designed for this purpose is the sling psychrom-
eter (Fig. 5-1). The sling psychrometer is made .Dry bulb
"thermometer
up of two thermometers, one dry bulb and one
wet bulb, mounted side by side in a protective
case which is attached to a handle by a swivel
Wet bulb
connection so that the case can be easily rotated Wetted wick
thermometer
about the hand. After saturating the wick with
clean water, the instrument is whirled rapidly in
the air for approximately one minute, after which Fig. 5-1. Sling psychrometer.
time readings can be taken from both the dry
bulb and wet bulb thermometers. The process
should be repeated several times to assure that any given dry bulb temperature, the lower the
the lowest possible wet bulb temperature has moisture content of the air, the lower is the wet
been recorded. bulb temperature. The reason for this is easily

Unless the air is 100 %saturated, in which case explained.


the dry bulb, wet bulb, and dew point tempera- When unsaturated air is brought into contact
tures of the air willbe one and the same, the with water, water will evaporate into the air at a
temperature recorded by a wet bulb thermom- rate proportional to the difference in pressure
eter will always be lower than the dry bulb between the vapor pressure of the water and the
temperature of the air. The amount by which vapor pressure of the air. Hence, when a wet
the wet bulb temperature is reduced below the bulb thermometer is whirled rapidly about in
dry bulb temperature depends upon the relative unsaturated air, water will evaporate from the
humidity of the air and is called the wet bulb wick, thereby cooling the water remaining in the
depression. wick (and the thermometer bulb) to some tem-
Whereas a dry bulb thermometer, being un- perature below the dry bulb temperature of the
by humidity, measures only the actual
affected air.

temperature of the air, a wet bulb thermometer, It is important to recognize the fact that the

because of its wetted wick, is greatly influenced wet bulb temperature of the air is a measure of
by the moisture in the air; thus a wet bulb tem- the relationship between the dry bulb and dew
perature is in effect a measure of the relationship point temperatures of the air, and as such it
between the dry bulb temperature of the air and provides a convenient means of determining the
the moisture content of the air. In general, for dew point temperature of the air when the dry
62 PRINCIPLES OF REFRIGERATION

bulb temperature is known. Too, it will be and the total heat content of the air at any
shown later that the wet bulb temperature is also condition is the sum of the sensible and latent
an index of the total heat content of the air. heat contained therein.
In order to understand why the wet bulb tem- The sensible heat of the air is a function of the
perature is a measure of the relationship between dry bulb temperature. For any given dry bulb
the dry bulb and dew point temperatures, a know- temperature, the sensible heat of the air is taken
ledge of the theory of the wet bulb thermometer as the enthalpy of dry air at that temperature as
isrequired. When water evaporates from the calculated from 0° F. Air sensible heat at various
wick of a wet bulb thermometer, heat must be temperatures is given in Btu per pound of dry air
supplied to furnish the latent heat of vaporiza- in Column 10 of Tables 5-1 and 5-2. With regard
tion. Before the temperature of the water in the to Column 10, the temperatures listed in Column
wick is reduced below the dry bulb temperature 1 are used as dry bulb temperatures.
of the air, the source of the heat to vaporize the
water is the water itself. Therefore, as water Example 5-1 1. Using Table 5-2, determine
evaporates from the wick, the water remaining the sensible heat in 10 lb of air at 80° F.
in the wick is cooled below the dry bulb tempera-
ture of the air. When this occurs, a temperature Solution. From Table
5-2, the sensible heat of 1 lb
and heat begins to flow
differential is established
of air at 80° F = 19.19 Btu/lb
from the air to the wick. Under this condition, a
part of the vaporization heat is being supplied by
For 10 lb of air, the sen- = 10 x 19.19
sible heat at 80° F = 191 .9 Btu
the air while the other part is supplied by the
water in the wick. As the temperature of the added or
The quantity of sensible heat
wick continues to decrease, the temperature heating or cooling a given weight of
removed in
difference between the air and the wick increases through a given temperature range may be
air
progressively so that more and more of the computed by applying Equation 2-8. The mean
vaporization heat is supplied by the air and less 0.24
specific heat of air at constant pressure is
and less is supplied by the water in the wick.
Btu/lb. (Although the specific heat of any vapor
When the temperature of the wick is reduced to
or gas varies somewhat with the temperature
the point where the temperature difference
mean specific heat value is
range, the use of a
between the air and the "wick is such that the
sufficiently accurate for all practical purposes.)
flow of heat from the air is sufficient to supply all
of the vaporizing heat, the temperature of the Example 5-12. Compute the quantity of
wick will stabilize even though vaporization from sensible heat required to raise the temperature
the wick continues. The temperature at which of 10 lb of air from 0° F to 80° F.
the wick stabilizes is called the temperature of
Solution. Applying 10 x 0.24
adiabatic saturation and is the wet bulb tempera-
Equation 2-8, Qs x (80 - 0)
ture of the air.
192 Btu
Through careful analysis of the foregoing, it
Alternate Solution. From
can be seen that the wet bulb temperature Table 5-2, the sensible heat
depends upon both the dry bulb temperature and of 1 lb of air at 80° F = 19.19 Btu/lb
the amount of water vapor in the air. For Sensible heat of 1 lb of
example, the lower the relative humidity of the air at 0° F = Btu/lb
air, the greater is the rate of evaporation from
For 1 lb of air, Qa = 19.19 -0
the wick and the greater is the amount of heat = 19.19 Btu/lb
required for vaporization. Obviously, the greater For 10 lb of air, Q, = 10 x 19.19
the need for heat, the greater is the wet bulb = 191.9 Btu
depression below the dry bulb temperature. Too,
it follows also that the lower the dry bulb tem- Since all the components of dry air are non-
perature, the lower the wet bulb temperature for condensable at normal temperatures and pres-
any given wet bulb depression. sures, for all practical purposes the only latent
5-13. The Heat Content or Enthalpy of heat in the air is the latent heat of the water
Air. Air has both sensible and latent heat, vapor in the air. Therefore, the amount of
PSYCHROMETRIC PROPERTIES OF AIR 63

latent heat in any given quantity of air depends Latent heat per
upon the weight of water vapor in the air and pound of dry air at

upon the latent heat of vaporization of water


50° F DP = 0.007626 x 1081.7
= 8.25 Btu/lb
corresponding to the saturation temperature of
the water vapor.
For 10 lb of dry air, = 10 x 8.25
total latent heat = 82.5 Btu
Since the saturation temperature of the water
vapor is the dew point temperature of the air, Since the total heat of the air is the sum of the
the point temperature determines not only
dew sensible and latent heat contained therein, the
the weight of water vapor in the air but also the total heat of the air in Examples 5-12 and 5-13
value of the latent heat of vaporization. Hence, is the sum of the sensible heat of the dry air, as
the latent heat content of the air is a function of computed in Example 5-12, and the latent heat
the dew point temperature alone. As long as the of the water vapor mixed with the dry air, as
dew point temperature of the air remains un- computed in Example 5-13, viz:
changed, the latent heat content of the air also
Sensible heat of 10 lb of dry
remains unchanged.
air at 70° F, from Example
The total heat content of water vapor at -
5-12 191.9 Btu
various temperatures as computed from 32° F is
Latent heat of water vapor
given in Btu per pound in Column 1 1 of Tables
mixed with 10 lb of dry air at
5-1 and 5-2. Although the values given in 50° F DP, from Example 5-13 82.5 Btu
Column include the sensible heat of the
11
Total heat of 10 lb of air at 191.9 + 82.5
liquid above 32° F as well as the latent heat of 70° F DB and 50° F DP* 274.4 Btu
vaporization at the given temperature, common
practice is to treat the entire heat content of the 5-14. Wet Bulb Temperature as a Measure
water vapor as latent heat.* of Total Heat. It has been shown in pre-
The latent heat content of any given quantity ceding sections that the sensible heat of the air
of air can be computed by multiplying the actual (the heat content of the dry air) is a function of
weight of water vapor in the air in pounds by the the dry bulb temperature and that the latent heat
total heat of the water vapor as given in Column of the air (the heat content of the water vapor
11 of Tables 5-1 and 5-2. mixed with the dry air) is a function of the dew
point temperature. Since, for any given com-
Example 5-13. Compute the latent heat
bination of dry bulb and dew point temperatures,
content of the air in Example 5-12, if the dew
the wet bulb temperature of the air can have only
point temperature of the air is 50° F.
one value, it is evident that the wet bulb tempera-
Solution. From ture is an index of the total heat content of the
Table 5-2, the actual air. However, it is important to recognize that
weight of water vapor
although there is only one wet bulb temperature
per pound of dry air
that will satisfy any given combination of dry
(specific humidity) at
50° F DP (Column 6) = 0.007626 lb bulb and dew point temperatures, there are
many combinations of dry bulb and dew point
Total heat per
temperatures which will have the same wet bulb
pound of saturated
water vapor at 50° F temperature (see Fig. 5-2). This means in effect
(Column 11) = 1081.7 Btu/lb that different samples of air having the same wet

* Although the any con-


total heat of the air at * The actual weight of air involved is slightly in
dition is the sum
of the sensible and latent heat excess of 10 lb, being 10 lb of dry air plus the weight
contained therein, as a practical matter it is more of water vapor (0.007626 lb) mixed with the dry air.
convenient to consider the total enthalpy of the air Too, since the temperature of the water vapor is the
as being the sum of the enthalpy of the dry air and same as that of the dry air (70° F), the water vapor
the enthalpy of the water vapor mixed with the dry contains a certain amount of superheat (50° F to
air. Since the amount of sensible heat is com- 70° F) which is not included in the total heat. How-

paratively small, the error which accrues from ever, sinceboth of these values are very small, the
assuming all of the heat of the vapor to be latent error incurred by neglecting them has no practical
heat is of no practical consequence. significance.
64 PRINCIPLES OF REFRIGERATION
Temperature, °F Heat Content, Btu/lb humidity of the air is known, the specific volume
Dry Dew Wet of partially saturated air at any condition can be
Bulb Point Bulb Sensible Latent Total computed by applying these values in the follow-
ing equation:
60 60 60 14.39 11.98 26.37
65 57 60 15.59 10.78 26.37
70 53.5 60 16.79 9.58 26.37 va = v* + K»» - vd ) x %RH] (5-5)
75 50 60 17.99 8.38 26.37
80 45.5 60 19.19 7.18 26.37 where va = the specific volume of partially
85 40.5 60 20.39 5.98 26.37
90 34.5 60 21.59 4.78 26.37 saturated air
vd = the specific volume of dry air at the
Fig. 5-2 same temperature
vs = the specific volume of saturated air
at the same temperature
bulb temperature have the same total heat, even
though the ratio of sensible to latent heat may be Example 5-15. Compute the specific vol-
different for the different samples. ume of air at 95° F DB and 50% RH.
The values of total heat listed for various
Solution. From Table
temperatures in Column 12 of Tables 5-1 and
5-2, specific volume of
5-2 are for 1 lb of saturated air at the tempera- dry air at 95° F (Column
ture shown. However, since all samples of air, =
8) 1 3.97 cu ft/lb
saturated or unsaturated, having the same wet Specific volume of
bulb temperature have the same total heat, the saturated air at 95° F
values given inColumn 12 will apply to any (Column 9) = 14.79 cu ft/lb
sample of air when the temperatures listed in =
Applying Equation 13.97+ [(14.79
Column 1 are used as wet bulb temperatures. 5-5, va - 13.97) x 0.5]
= 14.38 cu ft/lb
Example 5-14.
100 lb of air having an
If
initial wet bulb temperature of 78° F are cooled 5-16. The Psychrometric Chart. Psychro-
to a final wet bulb temperature of 60° F, deter-
metric charts (Fig. 5-3) are graphical repre-
mine the total heat removed from the air during
sentations of psychrometric data such as those
the cooling process.
contained in Tables 5-1 and 5-2. The use of
Solution. From Table psychrometric charts permits graphical analysis
5-2, total heat of 1 lb of of psychrometric data and thereby facilitates the
air corresponding to 78" F solution of many practical problems dealing with
WB (Column 12) - 63.05 Btu/lb air which would otherwise require tedious mathe-
Total heat per pound of matical calculation.
air at 60° F WB
(Column Basically, the psychrometric chart shows the
12) = 26.37 Btu/lb relationship between four fundamental proper-
Total heat removed per = 63.05 - 26.37 ties of air: (1) dry bulb temperature,
(2) dew
pound of air, Q t = 36.68 Btu/lb point temperature, (3) wet bulb temperature, and
For 1001b of air, the - 100 x 36.68 (4) relative humidity. When any two of these
four properties are known, the other two can be
total heat removed, Q t = 366.8 Btu
determined directly from the psychrometric chart
5-15. Specific Volume of Air.
has already It without using mathematical calculations.
been shown that the volume occupied by a given The skeleton chart in Fig. 5-4 illustrates the
weight of air depends upon the temperature of general construction of the psychrometric chart
the air and upon the total barometric pressure. which is based primarily upon the relationship
For standard sea level pressure, the volume of that exists between the aforementioned four pro-
1 lb of dry air at various temperatures is listed perties. Notice that the lines of dry bulb tem-
in Column 8 of Tables 5-1 and 5-2. The volume perature are vertical while the lines of dew point
of 1 lb of saturated air (1 lb of dry air and the temperature are horizontal. The lines of wet
water vapor to saturate it) is listed for various bulb temperature run diagonally across the chart
temperatures in Column 9. When the relative as do the lines of constant volume. The curved
PSYCHROMETRIC PROPERTIES OF AIR 65

lines are lines of constant relative humidity. The (b) From Example 5-16,
curved line bounding the chart on the left side is total heat per pound of
the line of 100% relative humidity and is called air, Q t
= 40.50 Btu/lb
the saturation curve. Air at any condition such The latent heat per = Qt - Q,
that its state can be identified by a point falling pound of air, & = 40.50 - 22.80
= 17.7 Btu/lb
anywhere along the saturation curve is saturated
air. Values for dry bulb, wet bulb, and dew Example 5-18. If the air in Example 5-16 is
point temperatures are read at the saturation cooled to 75° F, determine:
curve. Values for dry bulb temperature are also (a) The final dew point temperature
given at the base of the chart. Notice that the (b) The final wet bulb temperature
dry bulb, wet bulb, and dew point temperatures (c) The final relative humidity
for saturated air coincide. Values of specific (d) The final total heat per pound
volume and relative humidity are given along
below the
Solution. Since the air is not cooled
the lines of constant volume and relative humid-
initial dew
point temperature, no moisture is
Values of specific humidity and
ity, respectively.
removed from the air. Therefore, the specific
vapor pressure are given on the right and left humidity, dew point temperature, and latent
margins of the chart. For any given air condi- heat of the air remain unchanged. Hence, the
tion, the specific humidity and vapor pressure initial dew point temperature and the new dry
corresponding to the dew point temperature can bulb temperature can be used as coordinates to
be determined by following the dew point tem- locate the new condition of the air on the
perature line to the specific humidity and vapor psychrometric chart (point B
in Fig. 5-6). The

pressure scales. The total heat corresponding to following properties of the air at the new con-
dition are taken from the psychrometric chart
any wet bulb temperature is found by following
as indicated in Fig. 5-6:
the wet bulb lines to the total heat scale above
(a) Wet bulb temperature = 71.4° F
the saturation curve. The following example will = 85
illustrate the use of the psychrometric chart.
(b) Relative humidity %
(c) Total heat per pound = 35.69 Btu/lb
Example 5-16. A certain quantity of air has Example 5-19. With respect to Fig. 5-6, in
a dry bulb temperature of 95° F and a wet bulb cooling the air from condition "A," as described
temperature of 77° F. From the psychrometric
in Example 5-16, to condition "B," as described
chart determine all of the following values: (1)
in Example 5-18, compute:
dew point temperature, (2) specific humidity, (a) The total heat removed per pound of air
(3) vapor pressure, (4) specific volume, (5) total
(b) The sensible heat removed per pound of
heat, and (6) relative humidity.
air.

Solution. Using the two known properties of


Solution.
the air as coordinates the condition of the air
(a) From Example 5-16,
can be established as a point on the chart. Once = 40.50 Btu/lb
air total heat at A
this point has been established, the other pro-
perties of the air at this condition can be read
From Example 5-18, air

directly from the chart as shown in Fig. 5-5, viz:


total heat at B = 35.69 Btu/lb

= The total heat removed


Dew point temperature 70° F
=110 per pound of air
Specific humidity grains/lb
Vapor pressure = 0.37 psia in cooling from A = 40.50 - 35.69
Specific volume = 14.33 cu ft/lb toB = 4.81 Btu/lb
Total heat = 40.5 Btu/lb (b) Since there is no change in the latent heat of
Relative humidity = 45 % the the sensible heat removed per pound
air,

Example 5-17. For the air in Example 5-16, of air is equal to the total heat removed
determine: (a) the sensible heat per pound of per pound of air.
air and (6) the latent heat per pound of air.
Example 5-20. Assume that the air in
Solution, Example 5-16 is cooled to 40° F and determine:
(a) From Table 5-2, en- (a) The total heat removed per pound
thalpy of 1 lb of dry air (b) The sensible heat removed per pound
at 95° F DB, Q, = 22.80 Btu/lb (c) The latent heat removed per pound.
66 PRINCIPLES OF REFRIGERATION

smejS-Jie /tip jo punod auo uj jode/\ jsjem jo )q8j9/\A

cm cm cm cm cm (\j t-H t-ti-i t-i,-t,-i.-irH,-ir-ja» oo r^vo in v m cm »-• ©o

>>
c
rt
a
E
5

e
v
c
O

in Q-
io E M

»! o
£•
ir>
ID
Q .

r
M
o 6
u
r
S
E

•5
s
IT)
m 1
A
R £

i
**
ii utri it I n i liin i luirli ii ii in 1 in 1
1 1 1 1 1 1 li ii

83R82888888SR8S88i8R88SR8S°
I I' i i I i i i i I

8
o
uj
i i

bs jad qi-jodeA
8°°°
suiej8-j|e Ajp jo punod auo ui JOdeA jsjbm jo )i|8j3M

u I

o
i i i i I

o
i i

iaje/w jo ajnsssjj
i i

o
I i i i i I

ci
i i i i

o
*-<
o
I i i i il

8
o
PSYCHROMETRIC PROPERTIES OF AIR 67

su|Bj8-im Aip jo punod mm u| iodeA jajeM jo )u8i»m


§i8§S§§§§§§ §8 §§§888888883
ll|IIM|IMI|llll|llll|llll|llll|llll|Mll||lll|llll|l|ll|l

8.
E
o
u

e
c

o
U

8
E

llllllllllllllllllllllllllllllllllll

8388S8.88I5 8 8s9 888888888388S


sujuS-Jie ftp 10. punoaauo ui jod«A jojbm jo w3»m
i i i i i I i i i. i i i i i i I i i i i rxi i i i i i i i i i i i i I i i i i i

8 8 5 o 8
o o o 8
o 8
o
t-l
d o
ui bs isd q|-jodeA iv\b* jo ainssaid
68 PRINCIPLES OF REFRIGERATION

su!bj8-j!b Ajp jo punod auo in JodBA jsjbm jo )M,8ja/\f\

ll|IHI|llll|llll|llll|llll|llll|flll|llll|IIM|lll||lll||||||;

CL
E
o
(J
u
1.

s
UJ

o
u

E
8

suibj8-j!b <jp jo punod suo uj jodBA jaiEM jo )(]8ja/v\

I I i i i I i i i i I i i i i I i i i i I i i i i I i i i i I i i i i I i i
jlU
o 8 8 3 8 o
d o o
§j 8
d d d d
u; bs jad q|-jodBA jajBM jo ainssaid
PSYCHROMETRIC PROPERTIES OF AIR 69

suiafl-JiB &p jo punod auo u; jodBA ts\m jo juSbm

CM
ll
CNJ CM CM CM CM
lllllllllllllllllllll
f-i i-t^H t-Hi-H,^
l lllll l llll l lllllllllllllll l llllllllllll
(-Ht-l^rHatOOl^tOin^
Mil !
AN H
iiii|mi|iiii|iiii|iiii|iiii|iiii|iiii|iiii|iiiijiiii|im|iiii|iiiiyiiii|iiii|iiii|Hii|iiii|iiii|iiii

L f\

•Sg-1
"51
C, /Si
mi 8
?
&
2 /
- /

>T J.S6-8Q
a.
£
o
u

O
>s

a.
o
U
t

E
p

iiiliiiiliiiiliiiiliiiiliiiiliiiilliiiliiiiliiiiliiiiliiiiliiiiliiiiliiiiliiiiliiilliiiilllllliiiilniiliiiiliiiilillllilN

(MNNNNSrtHHHHHHHHH
sme.i8-.ne Aip jo punod auo u; jode/v j8}bm jo jtjSraAA

I i i i i I i i i I I I I I I I i I i i I i i i i I i I i i I i i i i I- i i I I I

8 S ^ m o 8
o o o o o o
ui bs iad q|—jodeA jsjbm jo sjnssaid
70 PRINCIPLES OF REFRIGERATION
Since the air is cooled below the
Solution. min (cfm). If the specificvolume of the air is
dew point temperature, moisture will be con- 14.10 cu ft/lb, compute the weight of air passing
densed out of the air and the air at the final over the coil in pounds per hour.
condition will be saturated. Therefore, the dry Ans. 4255 lb/hr
bulb temperature, the dew point temperature, 5. Compute the equivalent volume of standard
and the wet bulb temperature will coincide at air for the conditions of Problem 4.
40° F and the relative humidity of the air will be Ans. 946 cfm
100%. On the psychrometric chart, the con- 6. Compute the quantity of sensible heat re-
dition of the air falls on the saturation curve at
quired to raise the temperature of 10 lb of air
40° F (point B in Fig. 5-7).
from a temperature of 35° F to a temperature of
(a) From the psychromet- 100° F. Ans. 156 Btu
ric chart, the totalheat
7. If80 lb of air having an initial wet bulb tem-
of the air at the initial
= perature of 80° F are cooled to a final wet bulb
condition (point A) 40.50 Btu/lb
temperature of 65° F, determine the total heat
The total heat of the air
removed from the air during the cooling process.
at the final condition Ans. 1085.6 Btu
(point B) =15.19 Btu/lb
removed - 8. A
certain quantity of air has "a dry bulb
The total heat 40.50 15.19
temperature of 90° F and a wet bulb tempera-
per pound = 25.31 Btu/lb
ture of 77° F. From the psychrometric chart
(b) Applying Equation 2-8, determine all of the following values:
the sensible heat re- = x 0.24
1 (a) dew point temperature, (£>) specific humidity,
moved per pound of x (95 — 40) (c) vapor pressure, (d) specific volume, (e) total
air =13.2 Btu/lb heat, and (/) relative humidity.
(c) The latent heat re- = Q - Q,t
Ans. (a) 72.3° F; (6) 119.5 gpp; (c) 0.395 psia;
removed per pound = 25.31 — 13.20 (a") 14.23 cu ft/lb; (e) 40.5 Btu/lb; (/) 58%
of air =12.11 Btu/lb 9. Assume that the air in Problem 8 is cooled to
75° F anddetermine:
PROBLEMS (a) the finaldew point temperature Ans. 72.3° F
(b) the final wet bulb temperature Ans. 73° F
1. Determine the volume occupied by 1 lb of air (c) the final relative humidity Ans. 92 %
having a temperature of 80° F at standard sea id) the final total heat per pound of air
level pressure. Ans. 13.59 cu ft Ans. 36.6 Btu/lb
2. Compute the volume of the air in Problem 1 10. Assume that the air in Problem 8 is cooled
if the barometric pressure is 13.5 psia. to 55° F and
determine:
Ans. 14.80 cu ft pound of air
(a) the total heat removed per
3. Determine the volume of the air in Problem 1 Ans. 17.2 Btu/lb
if the temperature of the air is 120° F. (6) the sensible heat removed per pound of air
Ans. 14.60 cu ft Ans. 8.40 Btu/lb
4. Air at a temperature of 90° F is circulated (c) the latent heat removed per pound of air
over a cooling coil at the rate of 1000 cu ft per Ans. 8.8 Btu/lb
material in order to produce and maintain the
desired temperature conditions is called the heat
load. In most refrigerating applications the total
heat load on the refrigerating equipment is the
sum of the heat that leaks into the refrigerated
space through the insulated walls, the heat that
enters the space throughdoor openings, and the
heat that must be removed from the refrigerated

6 product in order to reduce the temperature of


the product to the space or storage conditions.
Heat given off by people working in the re-
frigerated space and by motors, lights, and other
Refrigeration equipment also contributes to the load
electrical
on the refrigerating equipment.
and the Vapor Methods of calculating the heat load are
discussed in Chapter 10.

Compression 6-4. The Refrigerating Agent.


frigerating process the
In any re-
body employed as the
System heat absorber or cooling agent
refrigerant.
is called the

All cooling processes may be classified as


either sensible or latent according to the effect
the absorbed heat has upon the refrigerant.
When the absorbed heat causes an increase in
6-1. Refrigeration. In general, refrigeration the temperature of the refrigerant, the cooling
isdenned as any process of heat removal. More process is said to be sensible, whereas when the

specifically, refrigeration is denned as that absorbed heat causes a change in the physical
branch of science which deals with the process state of the refrigerant (either melting or vapor-
of reducing and maintaining the temperature of izing), the cooling process .is said to be latent.

a space or material below the temperature of the With either process, if the refrigeratingeffectisto
surroundings. be continuous, the temperature of the refriger-
To accomplish this, heat must be removed ating agent must be maintained continuously
from the body being refrigerated and transferred below that of the space or material being
to another body whose temperature is below that refrigerated.
of the refrigerated body. Since the heat removed To illustrate, assume that 1 lb of water at
from the refrigerated body is transferred to 32° F placed in an open container inside an
is

another body, it is evident that refrigerating and insulated space having an initial temperature of
heating are actually opposite ends of the same 70° F (Fig. 6-1). For a time, heat will flow from
process. Often only the desired result distin- the 70° F space into the 32° F water and the
guishes one from the other. temperature of the space will decrease. How-
6-2. for Thermal Insulation. Since
Need ever, for each one Btu of heat that the water
heat will always travel from a region of high absorbs from the space, the temperature of the
temperature to a region of lower temperature, water will increase 1° F, so that as the tempera-
there is always a continuous flow of heat into ture of the space decreases, the temperature of
the refrigerated region from the warmer sur- the water increases. Soon the temperatures of
roundings. To limit the flow of heat into the the water and the space will be exactly die same
refrigerated region to some practical minimum, and no heat transfer will take place. Refrigera-
it is usually necessary to isolate the region from tion will not be continuous because the tempera-
its surroundings with a good heat insulating ture of the refrigerant does not remain below the
material. temperature of the space being refrigerated.
6-3. The Heat Load. The rate at which heat Now assume that 1 lb of ice, also at 32° F, is
must be removed from the refrigerated space or substituted for the water (Fig. 6-2). This time

71
72 PRINCIPLES OF REFRIGERATION

Insulation by heat conducted to it from these materials.


As the air is warmedexpands and rises to the
it

top of the space carrying the heat with it to the


ice compartment. In passing over the ice the air
is cooled as heat is conducted from the air to the

ice. On cooling, the air becomes more dense and

falls back into the storage space, whereupon it

absorbs more heat and the cycling continues.


The air in carrying the heat from the warm walls
and stored product to the melting ice acts as a
heat transfer agent.
To insure adequate air circulation within the
refrigerated space, the ice should be located near
Heat leaking the top of the refrigerator and proper baffling
through insulation
should be installed to provide direct and un-
from warm space to cold water.
Fig. 6-1. Heat flows restricted paths of air flow. A drip pan must be
Water temperature rises as space temperature located beneath the ice to collect the water which
decreases. Refrigeration will not be continuous.
results from the melting.
Ice has certain disadvantages which tend to
limit its usefulness as a refrigerant. For instance,
the temperature of the refrigerant does not with ice it is not possible to obtain the low tem-
change as it absorbs heat from the space. The peratures required in many refrigeration applica-
ice merely changes from the solid to the liquid tions. Ordinarily, 32° F is the minimum tem-
state while its temperature remains constant at perature obtainable through the melting of ice
32° F. The heat absorbed by the ice leaves the alone. In some cases, the melting temperature
space in the water going out the drain and the of the ice can be lowered to approximately 0° F
refrigerating effect will be continuous until all by adding sodium chloride or calcium chloride
the ice has melted. to produce a freezing mixture.
It is both possible and practical to achieve Some of the other more obvious disadvantages
continuous refrigeration with a sensible cooling of ice are the necessity of frequently replenishing
process provided that the refrigerant is con- the supply, a practice which is neither convenient
tinuously chilled and recirculated through the nor economical, and the problem of disposing of
refrigerated space as shown in Fig. 6-3. the water resulting from the melting.
Latent cooling may be accomplished with
Insulation
either solid or liquid refrigerants. The solid
refrigerants most frequently employed are ice
and solid carbon dioxide (dry ice). Ice, of
course, melts into the liquid phase at 32° F,
whereas solid carbon dioxide sublimes directly
into the vapor phase at a temperature of —109° F
under standard atmospheric pressure.
6-5. Ice Refrigeration. Melting ice has been
used successfully for many years as a refrigerant.
Not too many years ago ice was the only cooling
agent available for use in domestic and small
commercial refrigerators.
In a typical ice refrigerator (Fig. 6-4) the heat Heat leaking
entering the refrigerated space from all the through insulation

various sources reaches the melting ice primarily


Fig. 6-2. Heat flows from warm space to cold ice.
by convection currents set up in the air of the Temperature of space decreases as ice melts. Tem-
refrigerated space. The air in contact with the perature of ice remains at 32° F. Heat absorbed by
warm product and walls of the space is heated ice leaves space in water going out the drain.
REFRIGERATION AND THE VAPOR COMPRESSION SYSTEM 73

Another less obvious, but more important, 6-6. Liquid Refrigerants. The ability of
disadvantage of employing ice as a refrigerant is liquids to absorb enormous quantities of heat as
the difficulty experienced in controlling the rate they vaporize is the basis of the modern mechani-
of refrigeration, which in turn makes it difficult cal refrigerating system. As refrigerants, vapor-
to maintain the desired low temperature level izing liquids have a number of advantages over
within the refrigerated space. Since the rate at melting solids in that the vaporizing process is
which the ice absorbs heat is directly propor- more easily controlled, that is, the refrigerating
tional to the surface area of the ice and to the effect can be started and stopped at will, the rate

Fig. 6-3. Continuous sensible cooling. Heat taken in by the water in the space is given up to the ice.

temperature difference between the space tem- of cooling can be predetermined within small
perature and the melting temperature of the ice, limits, and the vaporizing temperature of the
the rate of heat absorption by the ice diminishes liquid can be governed by controlling the pres-
as the surface area of the ice is diminished by the sure at which the liquid vaporizes. Moreover,
melting process. Naturally, when the refriger- the vapor can be readily collected and condensed
ating rate diminishes to the point that the heat back into the liquid state so that the same liquid
isnot being removed at the same rate that it is can be used over and over again to provide a
accumulating in the space from the various heat continuous supply of liquid for vaporization.
sources, the temperature of the space will Until now, in discussing the various properties
increase. of fluids, water, because of its familiarity, has
Despite its disadvantages, ice is preferable to been used in examples. However, because of
all
mechanical refrigeration in some applications. its relatively high saturation temperature, and
Fresh vegetables, fish, and poultry are often for other reasons, water is not suitable for use as
packed and shipped in cracked ice to prevent a refrigerant in the vapor-compression cycle. In
dehydration and to preserve appearance. Too, order to vaporize at temperatures low enough to
ice has tremendous eye appeal and can be used satisfy most refrigeration requirements, water
to considerable advantage in the displaying and would have to vaporize under very low pres-
serving of certain foods such as salads, cocktails, sures, which are difficult to produce and main-
etc., and in chilling beverages. tain economically.
74 PRINCIPLES OF REFRIGERATION
v;;;;;;;m//m;//;;;;;;m/m/;;//. a liquid at ordinary temperatures only if confined
under pressure in heavy steel cylinders.
Table 16-3 is a tabulation of the thermo-
I? - Baffle
dynamic properties of R-12 saturated liquid and
vapor. This table lists, among other things, the

53£ saturation temperature of R-12 corresponding


,1 fs^i\ - to various pressures. Tables 16-4 through 16-6
I V /40* Ti an° » .
list the thermodynamic properties of some of the
other more commonly used refrigerants. These
\ and
tables are similar to the saturated liquid

^ Drain vapor tables previously discussed and are em-


ployed in the same manner.
6-7. Vaporizing the Refrigerant. An in-
44'
sulated space can be adequately refrigerated
by merely allowing liquid R-12 to vaporize in a
container vented to the outside as shown in Fig.
WW///////MWA
6-5. Since the R-12under atmospheric pres-
is

sure, its saturation temperature is — 21. 6° F.


Fig. 6-4. Ice refrigerator. Heat is carried from
Vaporizing at this low temperature, the R-12
warm walls and product to the ice by air circulation
readily absorbs heat from the 40° F space
within the refrigerated space. Air circulation is by
through the walls of the containing vessel. The
gravity.
heat absorbed by the vaporizing liquid leaves the
space in the vapor escaping through the open
There are numerous other fluids which have vent. Since the temperature of the liquid
lower saturation temperatures than water at the remains constant during the vaporizing process,
same pressure. However, many of these fluids refrigeration will continue until all the liquid is

have other properties that render them unsuitable vaporized.


for use as refrigerants. Actually, only a relatively Any container, such as the one in Fig. 6-5, in
few fluids have properties that make them desir-
able as refrigerants, and most of these have been Refrigerant vapor
js^v,
at atmospheric
compounded specially for that purpose. /
( pressure
There is no one refrigerant which is best suited
for all the different applications and operating
conditions. For any specific application the
refrigerant selected should be the one whose
properties most closely fit the particular require-
ments of the application.
Of all of the fluids now in use as refrigerants,
the one fluid which most nearly meets all the
qualifications of the ideal general-purpose re-
frigerant is a fluorinated hydrocarbon of the
methane having the chemical name di-
series
chlorodifluoromethane (CC1 2F 2). It is one of a
group of refrigerants introduced to the industry
under the trade name of "Freon," but is now
manufactured under several other proprietary
designations. To avoid the confusion inherent
jn the use of proprietory or chemical names, this
compound is now referred to as Refrigerant-12. The Refrigerant-12 liquid vaporizes as it
Fig. 6-5.
Refrigerant-12 (R-12) has a saturation tempera- takes heat from the 40° F space. The heat taken in
in
ture of — 21. 6° F at standard atmospheric by the refrigerant leaves the space in the vapor
pressure. For this reason, R-12 can be stored as escaping through the vent.
REFRIGERATION AND THE VAPOR COMPRESSION SYSTEM 75

which a refrigerant is vaporized during a re-


Pressure of
frigerating process is called an evaporator and refrigerant vapor

is one of the essential parts of any mechanical below atmospheric

refrigerating system.
6-8. Controlling the Vaporizing Tempera- Refrigerant-12 liquid
'boiling at -100"F
ture. The temperature at which the liquid
vaporizes in the evaporator can be controlled by
controlling the pressure of the vapor over the
liquid, which in turn is governed by regulating
the rate at which the vapor escapes from the
evaporator (Section 4-5). For example, if a hand
valve is installed in the vent line and the vent is
partially closed off so that the vapor cannot
escape freely from the evaporator, vapor will
collect over the liquid causing the pressure in the
evaporator to rise with a corresponding increase
in the saturation temperature of the refrigerant
(Fig. 6-6). By carefully adjusting the vent valve
to regulate the flow of vapor from the evaporator,
Fig. 6-7. Pressure
of refrigerant in evaporator
it is possible to control die pressure of the vapor
reduced below atmospheric by action of a vapor
over the liquid and cause the R-12 to vaporize at
pump.
any desired temperature between —21.6° F and
the space temperature. Should the vent valve be
completely closed so that no vapor is allowed to and no heat will flow from the space to the
escape from the evaporator, the pressure in the refrigerant. Vaporization will cease and no
evaporator will increase to a point such that the further cooling will take place.
saturation temperature of the liquid will be equal When vaporizing temperatures below —21.6°
to the space temperature, or 40° F. When this F are required, it is necessary to reduce the
occurs, there will be no temperature differential pressure in the evaporator to some pressure
below atmospheric. This can be accomplished
through the use of a vapor pump as shown in
Fig. 6-7. By this method, vaporization of the
liquid R-12 can be brought about at very low
Refrigerant temperatures in accordance with the pressure-
vapor above temperature relationships given in Table 16-3.
atmospheric
pressure 6-9. Maintaining a Constant Amount of
Liquid in the Evaporator. Continuous
Refrigerant-12
vaporization of the liquid in the evaporator
liquid boiling at 30*F
requires that the supply of liquid be continuously
replenished if the amount of liquid in the
evaporator is to be maintained constant. One
method of replenishing the supply of liquid in
the evaporator is through the use of a float valve

assembly as illustrated in Fig. 6-8. The action


of the float assembly is to maintain a constant
level of liquid in the evaporator by allowing
liquid to flow into the evaporator from the
storage tank or cylinder at exactly the same rate
Fig. 6-6. The boiling temperature of the liquid
that the supply of liquid in the evaporator is

refrigerant in evaporator is controlled by


the being depleted by vaporization. Any increase in
controlling the pressure of the vapor over the liquid the rate of vaporization causes the liquid level in
with the throttling valve in the vent. the evaporator to drop slightly, thereby opening
76 PRINCIPLES OF REFRIGERATION
in Fig. 6-8, used to regulate the flow of liquid
High pressure
S liquid refrigerant into the evaporator is called a
refrigerant
refrigerant flow control. The refrigerant flow
control is an essential part of every mechanical
refrigerating system.

Needle valve There are of refrigerant


five different types
~ assembly
flow controls, of which are in use to some
all

extent at the present time. Each of these


_ Low pressure distinct types is discussed at length in Chapter
liquid refrigerant

17. The float type of control illustrated in Fig.


6-8 has some disadvantages, mainly bulkiness,
which tend to limit its use to some few special
applications. The most widely used type of
refrigerant flow control is the thermostatic
KaSSsiSfc''
expansion valve. A flow diagram illustrating the
use of a thermostatic expansion valve to control
the flow of refrigerant into a serpentine coil
type evaporator is shown in Fig. 6-9.
6-10. Salvaging the Refrigerant. As a matter
Fig. 6-8. Float valve assembly maintains constant of convenience and economy it is not practical
liquid level in evaporator. The pressure of the to permit the refrigerant vapor to escape to the
refrigerant is reduced as the refrigerant passes outside and be lost by diffusion into the air.
through the needle valve. The vapor must be collected continuously and
condensed back into the liquid state so that the
same refrigerant is used over and over again,
the needle valve wider and allowing liquid to thereby eliminating the need for ever replenish-
flow into the evaporator at a higher rate. Like- ing the supply of refrigerant in the system. To
wise, any decrease in the rate of vaporization
High pressure
causes the liquid level to rise slightly, thereby liquid

moving the needle valve in the closing direction


to reduce the flow of liquid into the evaporator.
When vaporization ceases entirely, the rising
liquid level will close the float valve tightly and
stop the flow of liquid completely. When vapori-
Low pressure
zation is resumed, the liquid level will fall allow- Hiquid-vapor
ing the float valve to open and admit liquid to mixture

the evaporator.
The liquid refrigerant does not vaporize in the
storage cylinder and feed line because the pres-
sure in the cylinder
is such that the saturation

temperature of the refrigerant is equal to the


temperature of the surroundings (see Section
4-10). The high pressure existing in the cylinder
forces the liquid to flow through the feed line
and the float valve into the lower pressure
evaporator. In passing through the float valve,
the high pressure refrigerant undergoes a pres-
sure drop which reduces its pressure to the

evaporator pressure, thereby permitting the re-


frigerant liquid to vaporize in the evaporator at
the desired low temperature. Fig. 6-9. Serpentine coil evaporator with thermo-
Any device, such as the float valve illustrated static expansion valve refrigerant control.
REFRIGERATION AND THE VAPOR COMPRESSION SYSTEM 77

provide some means of condensing the vapor, low-pressure,


Low-pressure,
another piece of equipment, a condenser, must low-temperature
low-temperature
be added to the system (Fig. 6-10).
Since the refrigerant vaporizes in the evapor-
L liquid-vapor mixture

Refrigerant control -s /
high-temperature
ator because it absorbs the necessary latent heat
from the refrigerated space, all that is required
in order to condense the vapor back into the
liquid state is that the latent heat be caused to
flow out of the vapor into another body. The
body of material employed to absorb the latent
heat from the vapor, thereby causing the vapor
to condense, is called the condensing medium.
The most common condensing media are air
and water. The water used as a condensing
medium is usually supplied from the city main
or from a cooling tower. The used as a
air
condensing medium is ordinary outdoor air at
normal temperatures.
For heat to flow out of the refrigerant vapor
into the condensing medium the temperature
of the condensing medium must be below that
of the refrigerant vapor. However, since the
pressure and temperature of the saturated
vapor leaving the evaporator are the same as High-pressure,

J
.

High-pressure,
high-temperature'
those of the vaporizing liquid, the temperature high-tempenhii
liquid-vapor mixture

of the vapor will always be considerably below High-pressure,

that of any normally available condensing Mgh-temperitun! -


liquid
medium. Therefore, heat will not flow out of
the refrigerant vapor into the air or water used Fig. 6-10. Collecting and condensing the refrigerant

medium vapor. Refrigerant absorbs heat in evaporator and


as the condensing until the saturation
gives off heat in the condenser.
temperature of the refrigerant vapor has been
increased by compression to some temperature
above the temperature of the condensing cannot be cooled to a temperature below its
medium. The vapor pump or compressor saturation temperature, the continuous loss of
shown in Fig. 6-10 serves this purpose. heat by the refrigerant vapor in the condenser
Before compression, the refrigerant vapor is causes the vapor to condense into the liquid
at the vaporizing temperature and pressure. state at the new, higher pressure and saturation
Since the pressure of the vapor is low, the temperature. The heat given off by the vapor in
corresponding saturation temperature is also the condenseris carried away by the condensing

low. During compression the pressure of the medium. The resulting condensed liquid, whose
vapor is increased to a point such that the temperature and pressure will be the same as
corresponding saturation temperature is above those of the condensing vapor, flows out of the
the temperature of the condensing medium being condenser into the liquid storage tank and is
employed. At the same time, since mechani- then ready to be recirculated to the evaporator.
cal work is done on the vapor in compressing Notice that the refrigerant, sometimes called
it to the higher pressure, the internal energy of the working fluid, is merely a heat transfer
the vapor is increased with a corresponding agent which carries the heat from the refriger-
increase in the temperature of the vapor. ated space to the outside. The refrigerant
After compression, the high-pressure, high- absorbs heat from the refrigerated space in the
temperature vapor is discharged into the con- evaporator, carries it out of the space, and
denser where up heat to the lower
it gives rejects it to the condensing medium in the

temperature condensing medium. Since a vapor condenser.


78 PRINCIPLES OF REFRIGERATION
6-11. Typical Vapor-Compression System. 4-12. Service Valves. The suction and dis-
A flow diagram of a simple vapor-compression charge sides of the compressor and the outlet
system is shown in Fig. 6-11. The principal of the receiver tank are usually equipped with
parts of the system are: an evaporator,
(1) manual shut-off valves for use during service
whose function it is to provide a heat transfer operations. These valves are known as the
surface through which heat can pass from the "suction service valve," the discharge service
refrigerated space or product into the vaporizing valve," and the "receiver tank valve," respec-
refrigerant; (2) a suction line, which conveys tively. Receiver tanks on large systems frequent-
the low pressure vapor from the evaporator ly have shut-off valves on both the inlet and the
to the suction inlet of the compressor; (3) a outlet.
vapor compressor, whose function it is to 6-13. Division of the System. A refriger-
remove the vapor from the evaporator, and to ating system is divided into two parts according
raise the temperature and pressure of the vapor to the pressure exerted by the refrigerant in the
to a point such that the vapor can be condensed two parts. The low pressure part of the system
with normally available condensing media; (4) consists of the refrigerant flow control, the
a "hot-gas" or discharge line which delivers the evaporator, and the suction line. The pressure
high-pressure, high-temperature vapor from the exerted by the refrigerant in these parts is the
discharge of the compressor to the condenser; low pressure under which the refrigerant is
(5) a condenser, whose purpose it is to provide vaporizing in the evaporator. This pressure is
a heat transfer surface through which heat known variously as the "low side pressure,"
passes from the hot refrigerant vapor to the the "evaporator pressure," the "suction pres-
condensing medium; (6) a receiver tank, which sure," or the "back pressure." During service
provides storage for the liquid condenser so that operations this pressure is usually measured
a constant supply of liquid is available to the at the compressor by installing a compound
evaporator as needed; (7) a liquid line, which gage on the gage port of the suction service
carries the liquid refrigerant from the receiver valve.
tank to the refrigerant flow control; (8) a The high pressure side or "high side" of the
refrigerant flow control, whose function it is to system consists of the compressor, the discharge
meter the proper amount of refrigerant to the or "hot gas" line, the condenser, the receiver
evaporator and to reduce the pressure of the tank, and the liquid line. The pressure exerted
liquid entering the evaporator so that the liquid by the refrigerant in this part of the system is
will vaporize in the evaporator at the desired low the high pressure under which the refrigerant is
temperature. condensing in the condenser. This pressure is

Refrigerant
flow control"!

Evaporator-]

<8K
j
Liquid
Fig. 6-11. Flow diagram of simple
,ine
Suction dj~ vapor compression system show-
ine~V.
line ing the principal parts.
® rDischargeline
"SB", Receiver
valve (-Condenser
| (g) J/tank valve
Suction _
service valve

Compressor ®
VJteceiver
tank
REFRIGERATION AND THE VAPOR COMPRESSION SYSTEM 79

Receiver ir- coo led


tank condenser

Fig, 6-12. Air-cooled condens-


ing unit. Note fan mounted on
motor shaft to circulate air
Compressor
Compressor
over condenser. driver

called the "condensing pressure," the "dis- ing to condensing medium used to condense the

charge pressure," or, more often, the "head refrigerant. A condensing unit employing air as
pressure." the condensing medium (Fig. 6-12) is called an
The dividing points between the high and low air-cooled condensing unit, whereas one employ-
pressure sides of the system are the refrigerant ing water as the condensing medium is a water-
flow control, where the pressure of the refriger- cooled condensing unit.
ant reduced from the condensing pressure to
is 6-IS. Hermetic Motor-Compressor Assem-
the vaporizing pressure, and the discharge valves blies. Condensing units of small horsepower
in the compressor, through which the high are often equipped with hermetically sealed
pressure vapor is exhausted after compression.* motor-compressor assemblies. The assembly
It should be noted that, although the compressor consists of a direct-driven compressor mounted

isconsidered to be a part of the high side of the on a common shaft with the motor rotor and
system, the pressure on the suction side of the the whole assembly hermetically sealed in a
compressor and in the crankcase is the low side welded steel shell (Fig. 6-13).
pressure. The change in pressure, of course, Condensing units equipped with hermetically
occurs in the cylinder during the compression sealed motor-compressor assemblies are known
process. as "hermetic condensing units" and are em-
6-14, Condensing Units. The compressor, ployed on a number of small commercial
hot gas line, condenser, and receiver tank, refrigerators and on almost all household

along with the compressor driver (usually an refrigerators, home freezers, and window air

electric motor), are often combined into one conditioners. For reasons that will be shown
compact unit as shown in Fig. 6-12. Such an later, many hermetic condensing units are not
assembly is called a condensing unit because its equipped with receiver tanks,
function in the system is to reclaim the vapor A variation of the hermetic motor-compressor
and condense it back into the liquid state. assembly is the "accessible hermetic." It is

Condensing units are often classified accord- similar to the full hermetic except that the shell
is bolted together rather
enclosing the assembly
Care should be taken not to confuse the suction
• than seam welded. (Fig. 6-14). The bolted
and discharge valves in the compressor with the construction permits the assemblies to be
suction and discharge service valves. The suction opened in the field for servicing.
and discharge valves in a reciprocating compressor 6-16. Definition of a Cycle. As the refriger-
perform the same function as the intake and exhaust ant circulates through the system, it passes
valves in an automobile engine and are vital to the
through a number of changes in state or
operation of the compressor, whereas the suction and
condition, each of which is called a process.
discharge service valves serve no useful purpose inso-
is concerned.
The refrigerant starts at some initial state or
far as the operation of the compressor
The latter valves are used only to facilitate service
condition, passes through a series of processes
operations, as their nomenclature implies. in a definite sequence, and returns to the initial
80 PRINCIPLES OF REFRIGERATION

Fig. 4-13. Air-cooled condensing unitemploying hermetic motor-compressor. Note separate fan to circulate
air over condenser. (Courtesy Tecumseh Products Company.)

condition. This series of processes is called a part of the liquid vaporizes as it passes through
cycle. The simple vapor-compression refriger- the refrigerant control in order to reduce the
ation cycle is made up of four fundamental temperature of the liquid to the evaporating
processes: expansion, (2) vaporization, (3)
(1) temperature.
compression, and (4) condensation. In the evaporator, the liquid vaporizes at a
To understand properly the refrigeration constant pressure and temperature as heat to
cycle it is necessary to consider each process in supply the latent heat of vaporization passes
the cycle both separately and in relation to the from the refrigerated space through the walls of
complete cycle. Any change in any one process the evaporator to the vaporizing liquid. By the
in the cycle will bring about changes in all the action of the compressor, the vapor resulting
other processes in the cycle. from the vaporization is drawn from the
6-17. Typical Vapor-Compress ton Cycle. evaporator through the suction line into the
A typical vapor-compression cycle is shown in suction inlet of the compressor. The vapor
Fig. 6-15. Starting at the receiver tank, high- leaving the evaporator is saturated and its
temperature, high-pressure liquid refrigerant temperature and pressure are the same as those
flows from the receiver tank through the liquid of the vaporizing liquid. While flowing through
line to the flow control.
refrigerant The the suction line from the evaporator to the
pressure of the liquid is reduced to the evapor- compressor, the vapor usually absorbs heat
ator pressure as the liquid passes through the from the air surrounding the suction line and
refrigerant flow control so that the saturation becomes superheated. Although the tempera-
temperature of the refrigerant entering the ture of the vapor increases
somewhat in the
evaporator will be below the temperature of the suction line as the result of superheating, the
refrigerated space. It will be shown later that a pressure of the vapor does not change so that
REFRIGERATION AND THE VAPOR COMPRESSION SYSTEM 81

u
I
8

r
I
5
8
E

II
!i
«
£
L-
C V

I*
9- 3

£f
2<5
i •
82 PRINCIPLES OF REFRIGERATION

Liquid-vapor mixture
30'F-28.46 psig
Subcooled liquid
86°F-120.6 psig
Liquid-vapor mixture"
30T-28.46 psig

Saturated vapor
30°F-28.46 psig

Superheated vapor
Saturated vapor
132*^-120.6 psig"
102"F-120.6 psig
Superheated vapo r
70°F-28.46
t

psig~Xi£l(£
^ >->

Liquid-vapor mixture
"
102°F-120.6 psig

Saturated liquid
102°F-120.6 psig

Fig. 6-15. Typical refrigeration system showing the condition of the refrigerant at various points.

the pressure of the vapor entering the compres- differential between the refrigerant vapor and
sor is the same as the vaporizing pressure.* the cylinder wall is small, the flow of heat
In the compressor, the temperature and either to or from the refrigerant during com-
by compression
pressure of the vapor are raised pression is usually negligible. Therefore, com-
and the high-temperature, high-pressure vapor pression of the vapor in a refrigeration com-
is discharged from the compressor into the pressor is assumed to occur adiabatically.
hot-gas line. The vapor flows through the Although no heat as such is transferred either
hot-gas line to the condenser where it up
gives to or from the refrigerant during the compres-
heat to the relatively cool air being drawn sion, the temperature and enthalpy of the vapor
across the condenser by the condenser fan. are increased because of the mechanical work
As the hot vapor gives off heat to the cooler air, done on the vapor by the piston. Whenever a
its temperature is reduced to the new saturation vapor is compressed, unless the vapor is cooled
temperature corresponding to its new pressure during the compression, the internal kinetic
and the vapor condenses back into the liquid energy of the vapor is increased by an amount
state as additional heat is removed. By the equal to the amount of work done on the vapor
time the refrigerant reaches the bottom of the (Section 3-12). Therefore, when a vapor is
condenser, all condensed and the
of the vapor is compressed adiabatically, as in a refrigeration
liquid passes into the receiver tank, ready to be compressor, wherein no heat is removed from
recirculated. the vapor during the compression, the tempera-
6-18. The Compression Process. In ture and enthalpy are increased in direct
modern, high speed compressors, compression proportion to the amount of work done during
takes place very rapidly and the vapor is in the compression. The greater the work of
contact with the compressor cylinder for only a compression, the greater is the increase in
short time. Because the time of compression temperature and enthalpy.
is short and because the mean temperature The energy equivalent of the work done is
* Actually, the pressure of the vapor decreases called the heat of compression. The energy to

slightlybetween the evaporator and compressor do the work of compression, which is trans-
because of the friction loss in the suction line ferred to the vapor during the compression
resulting from the flow. process, is supplied by the compressor driver,
REFRIGERATION AND THE VAPOR COMPRESSION SYSTEM 83

usually an electric motor. It will be shown Since the condensing temperature is always
later that the theoreticalhorsepower required equal to the temperature of the condensing
to drive the compressor can be calculated from medium plus the temperature difference between
the heat of compression. the condensing refrigerant and the condensing
6-19. Discharge Temperature. Care should medium, it follows that the condensing tempera-
be taken not to confuse discharge temperature ture varies directly with the temperature of the
with condensing temperature. The discharge condensing medium and with the required rate
temperature is that at which the vapor is dis- of heat transfer at the condenser.
charged from the compressor, whereas the 6-21. Condensing Pressure. The condensing
condensing temperature is that at which the pressure is always the saturation pressure
vapor condenses in the condenser and is the corresponding to the temperature of the liquid-
saturation temperature of the vapor correspond- vapor mixture in the condenser.
ing to the pressure in the condenser. Because When the compressor is not running, the
the vapor is usually superheated as it enters the temperature of the refrigerant mixture in the
compressor and because it contains the heat of condenser will be the same as that of the
compression, the vapor discharged from the surrounding air, and the corresponding satura-
compressor is highly superheated and its tion pressure will be relatively low. Conse-
temperature is considerably above the satura- quently,when the compressor is started, the
tion temperature corresponding to its pressure. vapor pumped over into the condenser will not
The discharge vapor is cooled to the condensing begin to condense immediately because there is
temperature as it flows through the hot-gas line no temperature differential between the refriger-
and through the upper part of the condenser, ant and the condensing medium, and therefore
whereupon the further removal of heat from no heat transfer between the two. Because of
the vapor causes the vapor to condense at the the throttling action of the refrigerant control,
saturation temperature corresponding to the the condenser may be visualized as a closed
pressure in the condenser. container, and as more and more vapor is
6-20. Condensing Temperature. To provide pumped into the condenser without condensing,
a continuous refrigerating effect the refrigerant the pressure in the condenser increases to a
vapor must be condensed in the condenser at point where the saturation temperature of
the same rate that the refrigerant liquid is vapor is sufficiently high to permit the required
vaporized in the evaporator. This means that rate of heat transfer between the refrigerant
heat must leave the system at the condenser at and the condensing medium. When the required
the same rate that heat is taken into the system rate of heat transfer is reached, the vapor will
in the evaporator and suction line, and in the condense as fast as it is pumped into the
compressor as a result of the work of compres- condenser, whereupon the pressure in the con-
sion. Obviously, any increase in the rate of denser will stabilize and remain more or less
vaporization will increase the required rate of constant during the balance of the running cycle.
heat transfer at the condenser. 6-22. Refrigerating Effect. The quantity of
The rate at which heat will flow through the heat that each pound of refrigerant absorbs
walls of the condenser from the refrigerant from the refrigerated space is known as the
vapor to the condensing medium is the function refrigerating effect. For example, when 1 lb of
of three factors: (1) the area of the condensing ice melts it will absorb from the surrounding air
surface, (2) the coefficient of conductance of the and from adjacent objects an amount of heat
condenser walls, and (3) the temperature differ- equal to its latent heat of fusion. If the ice melts
ence between the refrigerant vapor and the at 32° F it will absorb 144 Btu per pound, so that
condensing medium. For any given condenser, the refrigerating effect of 1 lb of ice is 144 Btu.
the area of the condensing surface and the Likewise, when a liquid refrigerant vaporizes
coefficient of conductance are fixed so that the as it flows through the evaporator it will absorb
rate of heat transfer through the condenser an amount of heat equal to that required to
walls depends onlyon the temperature differ- vaporize it; thus the refrigerating effect of 1 lb
ence between the refrigerant vapor and the of liquid refrigerant is potentially equal to its

condensing medium. latent heat of vaporization. If the temperature


84 PRINCIPLES OF REFRIGERATION

of the liquid entering the refrigerant control the condensing pressure, 120.6 psig. Since the
from the liquid line is exactly equal to the saturation temperature corresponding to 120.6
vaporizing temperature in the evaporator the psig is 102° F, the 86° F liquid at the refrigerant
entire pound of liquid will vaporize in the control is subcooled 16° F (102 - 86) below its

evaporator and produce useful cooling, in saturation temperature.


which case the refrigerating effect per pound Since the saturation pressure corresponding
of refrigerant circulated be equal to the
will to 86° F is 93.2 psig, the R-12 can exist in the
total latent heat of vaporization. However, in its pressure is not reduced
liquid state as long as
an actual cycle the temperature of the liquid below 93.2 psig. However, as the liquid passes
entering the refrigerant control is always through the refrigerant control its pressure is
considerably higher than the vaporizing tem- reduced from 120.6 psig to 28.46 psig, the
perature in the evaporator, and must first be saturation pressure corresponding to the 30°
reduced to the evaporator temperature before vaporizing temperature of the refrigerant in the
the liquid can vaporize in the evaporator and evaporator. Since the R-12 cannot exist as a
absorb heat from the refrigerated space. For liquid at any temperature above the saturation
this reason, only a part of each pound of Hquid temperature of 30° F when its pressure is 28.46
actually vaporizes in the evaporator and psig, the hquid must surrender enough heat
produces useful cooling. Therefore, the refriger- to cool itself from 86° F to 30° F at the instant
ating effect per pound of liquid circulated is that its pressure is reduced in passing through
always less than the total latent heat of vapori- the refrigerant control.
zation. From Table 16-3, the enthalpy of hquid at
With reference to Fig. 6-15, the pressure of 86° F
and at 30° F is 27.73 Btu per pound and
the vapor condensing in the condenser is 120 14.76 Btu per pound, respectively, so that each
psig and the condensing temperature (satur- pound of liquid must surrender 12.97 Btu
ation temperature) of the R-12 vapor corre- (27.73 - 14,76) in order to cool from 86° F to
sponding to this pressure is 102° F. Since 30° F. Because the hquid expands through the
condensation occurs at a constant temperature, refrigerant control so rapidly, the liquid is not
the temperature of the liquid resulting from the in contact with the control for a sufficient length
condensation is also 102° F. After condensation, of time to permit this amount of heat to be
as the liquid flows through the lower part of transferred from the refrigerant to the control.
the condenser it continues to give up heat to the Therefore, a portion of each pound of hquid
cooler condensing medium, so that before the vaporizes as the liquid passes through the con-
liquid leaves the condenser its temperature is trol, and the heat to supply the latent heat of
usually reduced somewhat below the tempera- vaporization for the portion that vaporizes is

ture at which it condensed. The liquid is then drawn from the body of the hquid, thereby
said to be subcooled. The temperature at reducing the temperature of the refrigerant to the
which the liquid leaves the condenser depends evaporator temperature. In this instance, enough
upon the temperature of the condensing of each pound of liquid vaporizes while passing
medium and upon how long the hquid remains through the refrigerant control to absorb exactly
in contact with the condensing medium after the 12.97 Btu of sensible heat that each pound of
condensation. hquid must surrender in order to cool from 86° F
The liquid may be further subcooled in the to 30° F and the refrigerant is discharged from
receiver tank and by surrender-
in the liquid line the refrigerant control into the evaporator as a
ing heat to the surrounding air. In any case, liquid-vapor mixture. Obviously, only the liquid
because of the heat exchange between the portion of the liquid-vapor mixture will vaporize
refrigerant in the liquid line and the surrounding in the evaporator and produce useful cooling.
air, the temperature of the liquid approaching That portion of each pound of hquid circulated
the refrigerant control is likely to be fairly which vaporizes in the refrigerant control pro-
close to the temperature of the air surrounding duces no useful cooling and represents a loss of
the liquid line. In Fig. 6-1 5, the liquid ap- refrigerating effect. It follows, then, that the
proaches the refrigerant control at a temperature refrigerating effect per pound of hquid circulated
of 86° F, whereas its pressure is still the same as is equal to the total latent heat of vaporization
REFRIGERATION AND THE VAPOR COMPRESSION SYSTEM 85

less the amount of heat absorbed by that part of Example 6-2. If, in Example 6-1, the tem-
each pound that vaporizes in the control to perature of the liquid entering the refrigerant
reduce the temperature of the liquid to the control is 60° F rather than 86° F, determine the
refrigerating effect.
vaporizing temperature.
From Table 16-3, the latent heat of vaporiza- Solution. From Table
tion of 'R-12 at 30° F is 66.85 Btu per pound. 16-3, enthalpy of R-12
Since the loss of refrigerating effect is 12.97 Btu
saturated vapor at 30° F =81.61 Btu/lb
per pound, the refrigerating effect in this instance
Enthalpy of R-12 liquid
at 60° F = 21.57 Btu/lb
is (66.85 - 12.97) 53.88 Btu per pound.

The percentage of each pound of refrigerant Refrigerating effect = 60.04 Btu/lb

that vaporizes in the refrigerant control can be Example 6-3. If, in Example 6-1, the
determined by dividing the total latent heat of pressure in the evaporator is 21.05 psig, and the
vaporization into the heat absorbed by that part liquid reaching the refrigerant control is 86°" F,

of the pound that vaporizes in the control. In what is the refrigerating effect?
this instance the percentage of each pound Solution. From Table
vaporizing in the control is (12.97/66.85 x 100) 16-3, the saturation tem-
19.4%. Only 80.6% of each pound circulated perature of R-12 corre-
actually vaporizes in the evaporator and pro- sponding to 21.05 psig is
= 20° F and the enthalpy of
duces useful cooling (66.85 x 0.806 53.88
R-12 saturated vapor at that
Btu/lb).
temperature = 80.49 Btu/lb
Even though a portion of each pound cir-
Enthalpy of R-12 liquid
culated vaporizes as it passes through the re-
at 86° F = 27.73 Btu/lb
frigerant control, the enthalpy of the refrigerant
Refrigerating effect = 52.77 Btu/lb
does not change in the control. That is, since
there is no heat transfer between the refrigerant A comparison of Examples 6-1 and 6-2 in-

and the control, the enthalpy of the liquid-vapor dicates that the refrigerating effect increases as

mixture discharged from the control into the the temperature of the liquid approaching the
evaporator is exactly the same as the enthalpy of refrigerant control decreases, whereas a com-
the liquid approaching the control. Therefore, parison of Example 6-1 and 6-3 shows that the
the difference between the enthalpy of the re- refrigerating effect decreases as the vaporizing

frigerant vapor leaving the evaporator and the temperature decreases. Therefore, it is evident
enthalpy of the liquid approaching the control is that the refrigerating effect per pound of liquid
only the amount of heat absorbed by the re- circulated depends upon two factors: (1) the
frigerant in the evaporator, which is, of course, evaporating temperature and (2) the temperature
the refrigerating effect. Hence, for any given at which the liquid refrigerant enters the re-
conditions the refrigerating effect per pound can The higher the evaporating
frigerant control.

be easily determined by subtracting the enthalpy temperature and the lower the temperature of
of the liquid refrigerant entering the control the liquid entering the refrigerant control,
from the enthalpy of the saturated vapor leaving the greater will be the refrigerating effect.
the evaporator. 6-23. System Capacity. The capacity of any
refrigerating system is the rate atwhich it will
Example 6-1. Determine the refrigerating
remove heat from the refrigerated space and is
effect per pound if the temperature of the liquid
usually stated in Btu per hour or in terms of its
R-12 approaching the refrigerant control is
86° F and the temperature of the saturated ice-melting equivalent.

vapor leaving the evaporator is 30° F. Before the era of mechanical refrigeration, ice

Solution. From Table was widely used as a cooling medium. With the
16-3, enthalpy of R-12 satur- development of mechanical refrigeration, it was
ated vapor at 30° F = 81.61 Btu/lb only natural that the cooling capacity of
Enthalpy of R-12 liquid mechanical refrigerators should be compared
at 86° F = 27.73 Btu/lb with an ice-melting equivalent.
Refrigerating effect per When one ton of ice melts.it will absorb
pound = 53.88 Btu/lb 288,000 Btu (2000 lb x 144 Btu/lb). If one ton
86 PRINCIPLES OF REFRIGERATION

of ice melts in one day (24 hr), it will absorb heat the liquid occurs so that the temperature of the
at the rate of 12,000 Btu/hr (288,000 Btu/24 hr) liquid at the refrigerant control is also 100° F,

or 200 Btu/min (12,000 Btu/hr/60). Therefore, find:

a mechanical refrigerating system having the (a) The refrigerating effect per pound
capacity of absorbing heat from the refrigerated
(b) The weight of refrigerant circulated per
minute per ton
space at the rate of 200 Btu/min (12,000 Btu/hr)
(c) The weight of refrigerant circulated per
is cooling at a rate equivalent to the melting of
minute for a 10-ton system.
one ton of ice in 24 hr and is said to have a
capacity of one ton. Solution
The capacity of a mechanical refrigeration
(a) From Table 16-3, en-
thalpy of R-12 satur-
system, that is, the rate at which the system will
ated vapor at 20° F = 80.49 Btu/lb
remove heat from the refrigerated space, depends
Enthalpy of R-12
upon two factors: (1) the weight of refrigerant
liquid at 100° F = 31.16 Btu/lb
circulated per unit of time and (2) the refrigerat-
Refrigerating effect 49.33 Btu/lb
ing effect of each pound circulated.
(b) Weight of refrigerant cir-
culated per minute per
Example 6-4. A
mechanical refrigerating
ton 200
system is operating under conditions such that
the vaporizing temperature is 30° F while the 49.33
temperature of the liquid approaching the re- 4.05 lb
frigerant control is 86° F. If R-12 is circulated (c) Weight of refrigerant cir-
through the system at the rate of 5 lb/min, culated per minute for
determine: a 10-ton system 10 x 4.05
(«) the refrigerating capacity of the system in 40.5 lb
Btu per hour.
Example 6-6. If, in Example 6-5, the liquid
(b) the refrigerating capacity of the system in
is subcooled from 100° F before it
F to 80°
tons.
reaches the refrigerant control, calculate:
Solution (a) the refrigerating effect
(a) From Example 6-1, (b) the weight of refrigerant circulated per
refrigerating effect = 53.88 Btu/lb minute per ton
Weight of refrigerant Solution
circulated per minute = 5 lb (a) From Table 16-3,
Refrigerating capacity enthalpy of R-12
in Btu per minute = 5 x 53.88 saturated vapor at
= 269.40 Btu/min 20° F = 80.49 Btu/lb
Refrigerating capacity
Enthalpy of R-12 liquid
in Btu per hour = 269.40 x 60 at 80° F = 26J8 Btu/lb
= 16,164 Btu/hr
(b) Refrigerating capacity
Refrigerating effect = 54.21 Btu/lb
(b) Weight of refrigerant cir- 200
in tons _ 269.40
culated per minute per 54.21
200
= 1.347 tons
ton = 3.69 lb

In comparing Examples 6-5 and 6-6, it is


Weight of Refrigerant Circulated per
6-24.
apparent that the weight of refrigerant which
Minute per Ton. The weight of refrigerant
must be circulated per minute per ton of re-
which must be circulated per minute per ton of
frigerating capacity varies with the refrigerating
refrigerating capacity for any given operating
effectand depends upon the operating conditions
conditions is found by dividing the refrigerating
of the system. As the refrigerating effect per
effect per pound at the given conditions into 200.
pound increases, the weight of refrigerant cir-
Example 6-5. An R-12 system is culated per minute per ton decreases.
operating
at conditions such that the vaporizing tempera- 6-25. Volume of Vapor Displaced per Min-
ture is 20° F and the condensing temperature is ute per Ton. When 1 lb of liquid refrigerant
100° F. If it is assumed that no subcooling of vaporizes, the volume of vapor which results
REFRIGERATION AND THE VAPOR COMPRESSION SYSTEM 87

depends upon the vaporizing temperature. The evaporator too rapidly, the pressure in the
lower the vaporizing temperature and pressure, evaporator will decrease and result in a decrease
the greater is the volume of the vapor which is in the boiling temperature of the liquid. In either
produced. When the vaporizing temperature is case, design conditions will not be maintained
known, the specific volume of the saturated and the refrigerating system will be unsatis-
vapor which results from the vaporization can factory.
be found in the saturated vapor tables. For The maintenance of design conditions and
instance, from Table 16-3, the specific volume of therefore good refrigeration depends upon the
R-12 saturated vapor at 10° F is 1.351 cu ft per selection of a compressor whose capacity is such
pound. This means that each pound of R-12 that the compressor will displace in any given in-
that vaporizes at 10° F produces 1.351 cuft of terval of time a volume of vapor that is equal to
vapor. Therefore, if 10 lb of R-12 are vaporized the volume occupied by the weight of refrig-
at 10° F in an evaporator each minute, saturated erant which must be vaporized during the same
vapor will be produced at the rate of 13.51 cu ft time interval in order to produce the required
per minute (10 x 1.351). refrigerating capacity at the design conditions.
In order to produce one ton of refrigerating For instance, in Example 6-7, 4.05 lb of R-12
capacity, a definite weight of refrigerant must be must be vaporized each minute at 20° F for each
vaporized each minute. The volume of vapor one ton of refrigerating capacity desired. In
which must be removed from the evaporator vaporizing, the 4.05 lb of R-12 produce 4.55
each minute can be calculated by multiplying cuft of vapor (4.05 x 1.121). If the evaporator
the weight of refrigerant circulated per minute pressure and the boiling temperature of the
by the specific volume of the saturated vapor at liquid in the evaporator are to remain constant,
the vaporizing temperature. thisvolume of vapor must be removed from the
evaporator each minute for each one ton of
Example 6-7. Determine the volume of refrigerating capacity. Hence, the compressor
vapor to be removed from the evaporator per selected for a system operating at the conditions
minute per ton of refrigerating capacity for the
of Example 6-7 should have a capacity such that
system described in Example 6-5.
it will remove vapor from the evaporator at the

Solution. From rate of 4.55 cu ft per minute for each ton of


Table 16-3, specific refrigerating capacity required. For a 10 ton
volume of R-12 satur- system, the compressor would have to remove
ated vapor at 20° F = 1.121 cu ft/lb
vapor from the evaporator at the rate of 45.50
From Example 6-5,
cu ft per minute (10 x 4.55).
weight of refrigerant
circulated per minute
per ton = 4.05 lb/min/ton PROBLEMS
Volume of vapor The temperature of liquid R-12 entering the
1.
displaced per minute = 4.05 x 1.121
86° F and the vaporizing
refrigerant control is
per ton = 4.55 cu ft/min/ton
temperature 30° F. Determine:
(a) The refrigerating per pound of
effect
6-26. Compressor Capacity. In any mecha-
refrigerant circulated.Ans. 53.89 Btu/lb
nical refrigerating system the capacity of the
(6) The loss of refrigerating effect per pound.
compressor must be such that vapor is drawn
Ans. 12.96 Btu/lb
from the evaporator at the same rate that vapor (c) The weight of refrigerant circulated per
is produced by the boiling action of the liquid minute per ton. Ans. 3.71 lb/min/ton
refrigerant. If the refrigerant vaporizes faster (d)The volume of vapor displaced per
than the compressor is able to remove the vapor, minute per ton. Ans. 3.48 cu ft/min/ton
the excess vapor will accumulate in the evapora-
2. If saturated R-12 liquid reaches the refriger-
tor and cause the pressure in the evaporator to ant control at a pressure of 136 psig and the
increase, which in turn will result in raising the vaporizing pressure in the evaporator is 30.07
boiling temperature of the liquid. On the other psig, determine:
hand, the capacity of the compressor is such
if (a) The refrigerating effect per pound.
that the compressor removes the vapor from the Ans. 48.18 Btu/lb
88 PRINCIPLES OF REFRIGERATION
(*) The weight of refrigerant circulated per (c) The volume of vapor to be displaced per
minute per ton. Arts. 4.15 lb/min/ton minute per ton. Arts. 3.13 cuft/min/ton
(c) The volume of vapor displaced per minute 4. K , in Problem 2, the liquid is subcooled to
per ton. Arts. 3.77 cu ft/min/ton 70° F and the evaporating pressure is lowered to
3. If the approaching the refrigerant
liquid 16.35 psig, determine
control in Problem 2 is subcooled to 70° F, (a) The refrigerating effect. Arts. 45.25 Btu/lb
determine: (b) The weight of refrigerant circulated per
(a) The refrigerating effect. Arts. 57.93 Btu/lb minute per ton. Arts. 4.42 lb/min/ton
(b) The weight of refrigerant circulated per (c) The volume of vapor displaced per
minute per ton. Arts. 3.45 cu ft/min/ton minute per ton. Arts. 6.44 cu ft/min/ton
Fig. 7-1.* The condition of the refrigerant in
any thermodynamic state can be represented as
a point on the Ph chart. The point on the Ph
chart which represents the condition of the
refrigerant in any one particular thermodynamic
state may be located if any two properties of
the refrigerant at that state are known. Once the
state point has been located on the chart, all the

7 other properties of the refrigerant for that state


can be determined directly from the chart.
As shown by the skeleton Ph chart in Fig.
7-2, the chart is divided into three areas which
Cycle Diagrams are separated from each other by the saturated
liquid and saturated vapor curves. The area
and the Simple on the chart to the left of the saturated liquid
curve is called the subcooled region. At any
Saturated Cycle point in the subcooled region the refrigerant is
in the liquid state and its temperature is below
the saturation temperature corresponding to
its pressure.The area to the right of the satu-
rated vapor curve is the superheated region and
the refrigerant is in the form of a superheated
7-1. Cycle Diagrams. A
good knowledge of vapor. The center section of the chart, between
the vapor-compression cycle requires an inten- the saturated liquid and saturated vapor curves,
sive study not only of the individual processes represents the change in phase of the refrigerant
that make up the cycle but also of the relation- between the liquid and vapor states. At any
ships that exist between the several processes and point between the two curves the refrigerant is
of the effects that changes in any one process in in the form of a liquid-vapor mixture. The
the cycle have on all the other processes in the distance between the two curves along any
cycle. This is greatly simplified by the use of constant pressure line, as read on the enthalpy
charts and diagrams upon which the complete scale at the bottom of the chart, is the latent heat
cycle may be shown graphically. Graphical of vaporization of the refrigerant at that
representation of the refrigeration cycle permits pressure. The saturated liquid and saturated
the desired simultaneous consideration of all the vapor curves are not exactly parallel to each
various changes in the condition of the re- other because the latent heat of vaporization
frigerant which occur during the cycle and the of the refrigerant varies with the pressure at
effect that these changes have on the cycle with- which the change in phase occurs.
out the necessity of holding in mind all the On the chart, the change in phase from the
different numerical values involved in cyclic liquid to the vapor phase takes place progres-
problems. sively from left to right, whereas the change in
The diagrams frequently used in the analysis of phase from the vapor to the liquid phase occurs
the refrigeration cycle are the pressure-enthalpy from right to left. Close to the saturated liquid
(Ph) diagram and the temperature-entropy (Ts) curve the liquid-vapor mixture is nearly all
diagram. Of the two, the pressure-enthalpy liquid, whereas close to the saturated vapor
diagram seems to be the most useful and is the curve the liquid-vapor mixture is almost all
one which is emphasized in the following sec- vapor. The lines of constant quality (Fig. 7-3),
tions. The temperature-entropy diagram has extending from top to bottom through the
already been introduced (Section 4-19) and its center section of the chart and approximately
application to the refrigeration cycle will be parallel to the saturated liquid and vapor
discussed to some extent in this chapter. * The pressure-enthalpy chart for each refrigerant
7-2. The Pressure-Enthalpy Diagram. A is different, depending upon the properties of the
pressure-enthalpy chart for R-12 is shown in particular refrigerant.

89
90 PRINCIPLES OF REFRIGERATION

1
2
£

o.

(eisd) ajnsssjd
<

CYCLE DIAGRAMS AND THE SIMPLE SATURATED CYCLE 91

Subcooled region
(Refrigerant is in
the form of a Region of phase change
subcooled liquid) (Refrigerant is a liquid-
vapor mixture)

>— Liquid to vapor > >


Superheated region
(Refrigerant is in
the form of a
Vapor to liquid — «— superheated vapor)

Saturated liquid curve

Saturated vapor curve-

Specific enthalpy (Btu per lb)

Fig. 7-2. Skeleton Mi chart Illustrating the three regions of the chart and the direction of phase changing.

curves, indicate the percentage of vapor in the superheated vapor region, fall off sharply
mixture in increments of 10%. For example, at toward the bottom of the chart.
any point on the constant quality line closest to The straight lines which extend diagonally
the saturated liquid curve the quality of the and almost vertically across the superheated
liquid-vapor mixture is 10%, which means that vapor region are lines of constant entropy. The
10% (by weight) of the mixture is vapor. curved, nearly horizontal lines crossing the
Similarly, the indicated quality of the mixture superheated vapor region are lines of constant
at any point along the constant quality line volume.
closest to the saturated vapor curve is 90% and The values of any of the various properties
the amount of vapor in the liquid-vapor mixture of the refrigerant which are of importance in the
is 90%. At any point on the saturated liquid refrigerating cycle may be read directly from
curve the refrigerant is a saturated liquid and at the Ph chart at any point where the value of that
any point along the saturated vapor curve the particular property is significant to the process
refrigerant is a saturated vapor. occurring at that point. To simplify the chart,
The pressure is plotted along the vertical the number of lines on the chart is kept to a
axis, and the enthalpy is plotted along the minimum. For this reason, the value of those
horizontal axis. Hence, the horizontal lines properties of the refrigerant which have no real
extending across the chart are lines of constant significance at some points in the cycle are
pressure and the vertical lines are lines of omitted from the chart at these points. For
constant enthalpy. example, in the liquid region and in the region
The lines of constant temperature in the of phase change (center section) the values of
subcooled region are almost vertical on the entropy and volume are of no particular
chart and paralled to the lines of constant interest and are therefore omitted from the
enthalpy. In the center section, since the chart in these sections.
refrigerant changes state at a constant tempera- Since the Ph chart is based on a 1 lb mass of
ture and pressure, the lines of constant tempera- the refrigerant, the volume given is the specific
ture run horizontally across the chart and volume, the enthalpy is in Btu per pound, and
parallel to the lines of constant pressure. At the the entropy Btu per pound per degree of
is in
saturated vapor curve the lines of constant absolute Enthalpy values are
temperature.
temperature change direction again and, in the found on the horizontal scale at the bottom of
92 PRINCIPLES OF REFRIGERATION

the chart and the values of entropy and volume by using the simple saturated cycle as a standard
are given adjacent to the entropy and volume against which actual cycles may be compared,
lines, respectively. The values of both enthalpy the relative efficiency of actual refrigerating
and entropy are based on the arbitrarily selected cycles at various operating conditions can be
zero point of —40° F. readily determined.
The magnitude of the pressure in psia is A simple saturated cycle for a R-12 system is
read on the vertical scale at the left side of the plotted on a Ph chart in Fig. 7-4. The system is

chart. Temperature values in degrees Fahren- assumed -to be operating under such conditions
heit are found adjacent to the constant tempera- that the vaporizing pressure in the evaporator is

ture lines in the subcooled and superheated 35.75 psia and the condensing pressure in the
regions of the chart and on both the saturated condenser is 131.6 psia. The points A, B, C, D,
liquid and saturated vapor curves. and E on the Ph diagram correspond to points
7-3. The Simple Saturated Refrigerating in the refrigerating system as shown on the flow
Cycle. A simple saturated refrigerating cycle is diagram in Fig. 7-5.
it is assumed that the
a theoretical cycle wherein At point A, the refrigerant is a saturated
refrigerant vapor leaves the evaporator and liquid in the condenser at the condensing
enters the compressor as a saturated vapor (at pressure and temperature, and its properties, as

the vaporizing temperature and pressure) and given in Table 16-3, are:
the liquid leaves the condenser and enters the
refrigerant control as a saturated liquid (at p = 131.6 psia t = 100° F
the condensing temperature and pressure). h = 31.16 Btu/lb s = 0.06316 Btu/lb/° F
Although the refrigerating cycle of an actual v = 0.0127 cu ft/lb
machine will usually deviate
refrigerating
somewhat from the simple saturated cycle, the At point A, the values of p, t, and h may be
analysis of a simple saturated cycle is nonethe- read directly from the Ph chart. Since the
less worthwhile. In such a cycle, the funda- refrigerant is always a saturated liquid at point
mental processes which are the basis of every A, point A will always fall somewhere along
actual vapor compression refrigerating cycle are the saturated liquid curve and can be located on
easily identified and understood. Furthermore, the Ph chart if either/?, t, or h is known. Usually

25

Specific enthalpy (Btu per lb)

Fig. 7-3. Skeleton Ph chart showing paths of constant pressure, constant temperature, constant volume,
constant quality, constant enthalpy, and constant entropy. (Refrigerant- 1 2.)
CYCLE DIAGRAMS AND THE SIMPLE SATURATED CYCLE 93

hx ha he h e hd
Specific enthalpy (Btu per lb)

temperature of 20° F
Fig. 7-4. Pressure-enthalpy diagram of a simple saturated cycle operating at a vaporizing
and a condensing temperature of 100° F. (Refrigerant- 1 2.)

in actual practice, either p, t, or both will be expansion, frequently called "wire-drawing,"


measurable. in which the enthalpy of the working fluid does
7-4. The Expansion Process. In the simple not change during the process. This type of
saturated cycle there is assumed to be no change expansion occurs whenever a fluid is expanded
in the properties (condition) of the refrigerant through an orifice from a high pressure to a
liquid as it flows through the liquid line from lower pressure. It is assumed to take place
the condenser to the refrigerant control and without the gain or loss of heat through the
the condition of the liquid approaching the re- piping or valves and without the performance of
frigerant control is the same as its condition work.f
at point A. The process described by the initial Since the enthalpy of the refrigerant does not
and final state points A-B occurs in the refriger- change during process A-B, point B is located
ant control when the pressure of the liquid on the Ph chart by following the line of constant
isreduced from the condensing pressure to the enthalpy from point A to the point where the
evaporating pressure as the liquidpassesthrough constant enthalpy line intersects the line of
the control.* When the liquid is expanded into constant pressure corresponding to the evapor-
the evaporator through the orifice of the control, ating pressure. To locate point B on the Ph
the temperature of the liquid is reduced from the chart, the evaporating pressure or temperature

condensing temperature to the evaporating tem- must be known.


perature by the flashing into vapor of a small As a result of the partial vaporization of
portion of the liquid. the liquid refrigerant during process A-B, the
Process A-B is a throttling type of adiabatic
amount of work done by
t Actually, a certain
is

the fluid in projecting through the orifice of


* Process A-B is an irreversible adiabatic ex- itself

pansion during which the refrigerant passes through the control. However, since the heat equivalent
a series of state points in such a way that there is no of the work done in overcoming the friction of the
uniform distribution of any of the properties. orifice merely heats the orifice and is subsequently

Hence, no true path can be drawn for the process and reabsorbed by the fluid, the assumption that the
line A-B merely represents a process which begins enthalpy of the fluid does not change during the
at state point A and terminates at state point B. process is not in error.
:

94 PRINCIPLES OF REFRIGERATION
refrigerant at point B is a liquid-vapor mixture The values of s and v at point B are usually of
whose properties are no interest and are not given either on the Ph
p = 35.75 psia chart or in the vapor tables. If the values of s
t = 20° F and v are desired, they must be calculated.
h =31.16 Btu/lb (same as at point A) 7-5. The Vaporizing Process. The process

v = 0.1520 cu ft/lb B-C is the vaporization of the refrigerant in the


s - 0.06316 Btu/lb/°F evaporator. Since vaporization takes place at a
Note. The change in entropy during the
constant temperature and pressure, B-C is both
process A-B results from a
transfer of heat
isothermal and isobaric. Therefore, point C
energy which takes place within the refrigerant is located on the Ph chart by following the lines

of constant pressure and constant temperature


itself because of internal friction. transfer of A
energy which occurs entirely within the working from point B to the point where they intersect
fluid does not affect the enthalpy of the fluid,
the saturated vapor curve. At point C the
only the entropy changes. refrigerant is completely vaporized and is a
At point B, in addition to the values of p, t, saturated vapor at the vaporizing temperature
and h, the approximate quality of the vapor can and pressure. The properties of the refrigerant
be determined from the Ph chart by interpolat- at point C, as given in Table 16-3 or as read

ing between the lines of constant quality. In from the Ph chart, are:

this instance, the quality of the vapor as deter- p •* 35.75 psia (same as at point B)
mined from the Ph chart is approximately t = 20° F (same as at point B)
27%. h = 80.49 Btu/lb
Since the refrigerant at point B is a liquid- v =• 1.121 cu ft/lb
vapor mixture, only the values of p and t can be s = 0.16949 Btu/lb/°F
read directly from
Table 16-3. However, The enthalpy of the refrigerant increases
because the enthalpy of the refrigerant at points during process B-C as the refrigerant flows
A and B is the same, the enthalpy at point B through the evaporator and absorbs heat from
may be read from Table 16-3 as the enthalpy the refrigerated space. The quantity of heat
at the conditions of point A. The quality of the absorbed by the refrigerant in the evaporator
vapor at point B can be determined as in Section (refrigerating effect) is the difference between
6-22, using enthalpy values taken either from the enthalpy of the refrigerant at points B and
Table 16-3 or from the Ph chart directly. C. Thus, if h a , h b , h h d , h e and h x represent the
, ,

Refrigerant after
passing through
refrigerant control

Point at which
c In the simple saturated
cycle, the refrigerant
flows through the liquid
vaporization isv£~~ line from the condenser
complete ^> to the refrigerant
control without a
Suction vapor flows change in condition
from the evaporator
Point at which
to the compressor
Discharge vapor Fig. 7-5. Flow diagram of a
condensation-
through the suction
'from compressor begins simple saturated cycle.
line without a
change in condition
/-S

Z)
£l
Point at which r
condensation is—'
complete
CYCLE DIAGRAMS AND THE SIMPLE SATURATED CYCLE 95

enthalpies of the refrigerant at points A,B,C,D, pression process C-D, wherein no heat, as such,
E, and X, respectively, then is transferred either internally (within the vapor
itself) or externally (to or from an external
qi=hc - K (7-1)
source) Ag will always be equal to zero. If Ag
where qx = the refrigerating effect in Btu/lb. is equal to zero, then As must also be equal to
But since h t is equal to h a then ,
zero. Hence, there is no change in the entropy

^ = A. - K (7-2) of the vapor during a frictionless-adiabatic


(isentropic) compression.
When we substitute the appropriate values in
Since there is no change in the entropy of the
Equation 7-2 for the example in question,
vapor during process C-D, the entropy of the
ft
= 80.49 - 31.16 refrigerant at point D is the same as at point &
= 49.33 Btu/lb Therefore, point D can be located on the Ph
chart by following the line of constant entropy
On the Ph diagram, the distance between
from point C
to the point where the constant
point X and point C represents the total latent entropy line intersects the hne of constant pres-
heat of vaporization of 1 lb of R-12 at the sure corresponding to the condensing pressure.
vaporizing pressure of 35.75 psia (hfg in Table At point D, the refrigerant is a superheated
16-3). Therefore, since the distance B-C is the
vapor whose properties are
useful refrigerating effect, the difference between
X-C and B-C, which is the distance X-B, is p = 131.6 psia
t = 112° F (approximate)
the loss of refrigerating effect.

7-6. The Compression Process. In the simple h = 90.6 Btu/lb (approximate)


no v = 0.330 cu ft/lb (approximate)
saturated cycle, the refrigerant undergoes
s = 0.16949 Btu/lb/° F (same as at point C)
change in condition while flowing through the
suction line from the evaporator to the com- All of the properties of the refrigerant at the
pressor. Process C-D takes place in the condition of point D are taken from the Ph
compressor as the pressure of the vapor is chart. Since the values of t, h, and v require
increased by compression from the vaporizing interpolation, they are only approximations.

pressure to the condensing pressure. For the The properties of the superheated refrigerant

simple saturated cycle, the compression process, vapor cannot usually be read accurately from
C-D, is assumed to be isentropic* An isen- the vapor table unless the pressure of the vapor
tropic compression is a special type of adiabatic in question corresponds exactly to one of the

process which takes place without friction, f pressures listed in the table, This is seldom the
It is sometimes described as a "frictionless- case, particularly at the higher pressures where

adiabatic" or "constant-entropy" compression. the pressure listings in the table are in 10 lb


According to Equation 4-3, Section 4-19, the increments.
change in entropy (As) during any process is Work is done on the vapor during the com-
equal to the transferred heat (Ag) divided by the pression process, C-D, and the enthalpy of the
is increased by an amount equal to
C
average absolute temperature ( R). In any refrigerant

frictionless-adiabatic process, such as the com- the heat energy equivalent of the mechanical
work done on the vapor. The heat energy
be shown later that compression of the
* It will
work done during the com-
equivalent of the
vapor in an actual refrigerating compressor usually
pression is often referred to as the heat of
deviates somewhat from true isentropic compression.
As a general rule, compression is polytropic. compression and is equal to the difference in the
enthalpy of the refrigerant at points D and C.
t The term, adiabatic, is used to describe any
number of processes which take place without the Thus, where qt is the heat of compression per
transfer of energy as heat to or from the working pound of refrigerant circulated,
substance during the process. Thus, an isentropic =ha - h (7-3)
process is only one of a number of different processes
qt e

which may be termed adiabatic. For example, For the example in question,
compare process C-D with process A-B. Both are = -
q2 90.60 80.49
adiabatic, but C-D is frictionless, whereas A-B is
a throttling type of process which involves friction. = 10.11 Btu/lb
96 PRINCIPLES OF REFRIGERATION

The mechanical work done on the vapor The quantity of sensible heat (superheat)
by the piston during the compression may be removed from 1 lb of vapor in the condenser in
calculated from the heat of compression. If cooling the vapor from the discharge tempera-
h> is the work done in foot-pounds per pound of ture to the condensing temperature is the differ-
refrigerant circulated and / is the mechanical ence between the enthalpy of the refrigerant at
energy equivalent of heat, then point Dand the enthalpy at point (ha — ht). E
w = ft x / Process E-A is the condensation of the vapor
(7-4)
in the condenser. Since condensation takes
or w = J(ha - hj (7-5) place at a constant temperature and pressure,
process E-A follows along lines of constant
when we substitute in Equation 7-4,
pressure and temperature from point to E
w = 10.11 x 778 point A. The heat rejected to the condensing
- 7865.58 ft-lb medium during process E-A is the difference
between the enthalpy of the refrigerant at
As a result of absorbing the heat of com- points E and A (he — ha).
pression, the hot vapor discharged from the On returning to point A, the refrigerant has
compressor is in a superheated condition, that
completed one cycle and its properties are the
is, its temperature is greater than the saturation
same as those previously described for point A.
temperature corresponding to its pressure.
Since both processes D-E and E-A occur in
In this instance, the vapor leaves the compressor
the condenser, the total amount of heat rejected
at a temperature of 1 12° F, whereas the satura-
by the refrigerant to the condensing medium in
tion temperature corresponding to its pressure
the condenser is the sum of the heat quantities
of 131.6 psia is 100° F. Thus, before the vapor
rejected during processes D-E and D-A. The
can be condensed, the superheat must be
total heat given up by the refrigerant at the
removed and the temperature of the vapor
condenser is the difference between the enthalpy
lowered from the discharge temperature to the
saturation temperature corresponding to
of the superheated vapor at point and the D
its
saturated liquid at point A. Hence,
*
pressure.
7-7. The Condensing Process. %
Usually, both ?s =h ~K (7-6)

processes D-E and E-A take place in the where ft = the heat rejected at the condenser
condenser as the hot gas discharged from the per pound of refrigerant circulated.
compressor is cooled to the condensing tempera- In this instance,
tureand condensed. Process D-E occurs in the = 90.60 - 31.16
ft
upper part of the condenser and to some extent
- 59.44 Btu/lb
in the hot gas line. It represents the cooling of
the vapor from the discharge temperature to the If the refrigerant is to reach point A at the
condensing temperature as the vapor rejects end of the cycle in the same condition as it left

heat to the condensing medium. During point A at the beginning of the cycle, the total
processD-E, the pressure of the vapor remains heat rejected by the refrigerant to the condens-
constant and point E is located on the Ph chart ing medium in the condenser must be exactly
by following a line of constant pressure from equal to the heat absorbed by the refrigerant at
point D to the point where the constant pressure all other points in the cycle. In a simple
line intersects die saturated vapor curve. saturated cycle, the refrigerant is heated at only
At point E, the refrigerant is a saturated vapor two points in the cycle: (1) in the evaporator
at the condensing temperature and pressure. by absorbing heat from the refrigerated space
Its properties, as read from either the Ph chart (ft) and in the compressor by the heat of com-
or from Table 16-3, are: pression (ft).
Therefore,
p = 131.6 psia (same as at point D)
t = 100° F ft - ft + ft (7-7)
In this instance,
h = 88.62 Btu/lb
s - 0.16584 Btu/lb/°F ft =49.33 +10.11
»» 0.319 cu ft/lb = 59.44 Btu/lb
CYCLE DIAGRAMS AND THE SIMPLE SATURATED CYCLE 97

Where m is the weight of refrigerant to be A more convenient method of determining


circulated per minute per ton, the theoretical horsepower per ton is produced
200 Btu/min by combining Equations 1-5 and 7-15:
m (7-8)
— Aj)
1x m(hd
hp = (7-17)
For the cycle in question, 42.42

200
The compressor horsepower as calculated
above represents only the horsepower required
to compress the vapor. That is, it is the
= 4.05 lb/min/ton theoretical power which would be required per
ton of refrigerating capacity by a 100% efficient
Then, where Qa is the total quantity of heat
rejected at the condenser per minute per ton, system. It does not take into account the power
required to overcome friction in the compressor
Qa = »<fc) (7-9) and other power losses. The actual (brake)
or ft = mUhi ~ A„) (7-10) horsepower required per ton of refrigeration
will usually be from 30% to 50% more than
For the cycle in question,
the theoretical horsepower calculated, depend-
ft = 4.05 x 59.44 ing upon the efficiency of the compressor.
= 240.93 Btu/min/ton The factors governing compressor efficiency are
discussed later.
Where Q% is the heat of compression per 7-9. Coefficient of Performance. The coeffi-
minute per ton of refrigerating capacity,
cient of performance of a refrigerating cycle is

ea =/w(?8) (7-11) an expression of the cycle efficiency and is


stated as the ratio of the heat absorbed in the
or ft = m(fid - A,) (7-12)
refrigerated space to the heat energy equivalent
Substituting, of the energy supplied to the compressor, that is,

ft =4.05 x 10.11 Heat absorbed from


= 40.95 Btu/min/ton Coefficient of _ the refrigerated space
performance Heat energy equivalent
Where W is the work of compression done on
of the energy supplied
the vapor per minute per ton of refrigerating
to the compressor
capacity,
W= rriiw) (7-13) For the theoretical simple saturated cycle, this
may be written as
or, since w equals J(qJ or /(A„ - A„),
Refrigerating effect
W = JmUqd (7-14) c.o.p.
r
=
Heat of compression
(7-18)

or W = Jm(hd - h£ (7-15)
- kg)
(A,
or W = /(ft) (7-16) =
(Ad - A„)
When we substitute in Equation 7-15,

W = 778 x 4.05 x (90.60 - 80.49)


(ft)
= 31,856 ft-lb/min/ton Hence, for the cycle in question,
7-8. Theoretical Horsepower. The theoreti- 49.33
calhorsepower required to drive the compressor c.o.p.
10.11
per ton of refrigerating capacity may be found
by applying Equation 1-5 (Section 1-11): 4.88

7-10. Effect of Suction Temperature on


31,856
P ~ 33,000 x 1 Cycle Efficiency. The efficiency of the vapor-
compression refrigerating cycle varies consider-
= 0.965 hp/ton ably with both the vaporizing and condensing
98 PRINCIPLES OF REFRIGERATION

ha h c h c h e hd hd
'

Specific enthalpy (Btu/lb above - 40° F)


Fig. 7-4. Comparison of two simple saturated cycles operating at different vaporizing temperatures (figure

distorted). (Refrigerant- 12.)

temperatures. Of the two, the vaporizing ing effect for the cycle having the 10° F vaporiz-
temperature has by far the greater effect. ing temperature is 48.20 Btu/lb. When the
To illustrate the effect that varying the suction vaporizing temperature of the cycle is raised to
temperature has on cycle efficiency, cycle 40° F, the refrigerating effect increases to 51.55
diagrams of two simple saturated cycles operat- Btu/lb. This represents an increase in the
ing at different suction temperatures are drawn refrigerating effect per pound of
on the Ph chart in Fig. 7-6. One cycle, identified
by the points A, B, C, D, and E, is operating at a (he - K) - {hc - h a)
x 100
vaporizing temperature of 10° F and a condens- h c — ha
ing temperature of 100° F. A similar cycle
51.55 - 48.20
having the same condensing temperature but x 100
48.20
operating at a vaporizing temperature of 40° F
is set off by the points A, W, C", £>', and E.
= 6.95%
To facilitate a comparison of the two cycles, greater refrigerating effect per pound of
The
the following values have been determined from
refrigerant circulated obtained at the higher
the PA chart:
accounted for by the
vaporizing temperature is

(a) For the 10° F cycle, fact that there is a smaller temperature differen-

qi=h c -ha = 79.36 - 31.16 = 48.20 Btu/lb tial between the vaporizing temperature and the

q i =h d -h e
= 90.90 - 79.36 = 11.54 Btu/lb temperature of the liquid approaching the

qa "h d -h a = 90.90 - 31.16 = 59.74 Btu/lb refrigerant control. Hence, at the higher

F cycle, suction temperature, a smaller fraction of the


(6) For the 40°
= hc -ha = 82.71 - 31.16 = 51.55 Btu/lb
.
refrigerant vaporizes in the control and a greater
qi
= h* - A = 90.20 - 82.71 = 7.49 Btu/lb
<
portion vaporizes in the evaporator and pro-
qa
fl
8
= h, -ha = 90.20 - 31.16 = 59.04 Btu/lb duces useful cooling.
Since the refrigerating effect per pound is

In comparing the two cycles, note that the greater, the weight of refrigerant which must be
refrigerating effect per pound of refrigerant circulated per minute per ton of refrigerating
circulated is greater for the cycle having the capacity is less at the higher suction temperature

higher vaporizing temperature. The refrigerat- than at the lower suction temperature. Whereas
CYCLE DIAGRAMS AND THE SIMPLE SATURATED CYCLE 99

the weight of refrigerant circulated per minute per minute per ton are less at the higher suction
per ton for the 10° F cycle is temperature, the work of compression per
ton and therefore the theoretical horsepower
200 required per ton will be smaller at the higher
Ac — f>a suction temperature. The theoretical horse-
power required per ton of refrigerating capacity
200
= for the 10° F cycle is
48.20
= 4.151b/min
42.42
The weight of refrigerant circulated per minute
per ton for the 40° F cycle is only
4.15 x (90.90 - 79.36)
42.42
200 = 1.13

For the 40° F cycle, the theoretical horsepower


200 required per ton is

~ 51.55
"Kb* — hf)
- 3.88 lb/min
42.42
The decrease in the weight of refrigerant 3.88 x (90.20 - 82.71)
circulated at the higher suction temperature is 42.42
0.683
4.15 - 3.88
x 100
4.15 In this instance, increasing the vaporizing
= 6.5% temperature of the cycle from 10° F to 40° F
reduces the theoretical horsepower required per
Since the difference between the vaporizing ton by
and condensing pressures is smaller for the 1.13 -0.683
cycle having the higher suction temperature, X 10°
1.13
the work of compression per pound required to
compress the vapor from the vaporizing = 39.5%
pressure to the condensing pressure is less for
the higher temperature cycle than for the lower Later, when the efficiency of the compressor
temperature cycle. It follows then that the heat is taken into consideration, it will be shown that
of compression per pound for the cycle having the difference in the actual horsepower required

the higher vaporizing temperature is also less per ton at the various suction temperatures is
than that for the cycle having the lower vaporiz- even greater than that indicated by theoretical
ing temperature. The heat of compression per computations.
pound F cycle is 11.54 Btu, whereas
for the 10° Since the coefficient of performance is an

the heat of compression for the 40° F cycle is index of the power required per unit of refriger-

only 7.49 Btu. This represents a decrease in the ating capacity and, as such, is an indication of

heat of compression per pound of cycle efficiency, the relative efficiency of the two
cycles can be determined by comparing their
(A„ - he) - {hd - h<)
. of performance. The coefficient of
coefficients

hd — he performance for the 10° F cycle is

11.54 -7.49 hc — ha
x 100
11.54
-35.1%
= 48.20
Because both the work of compression per 11.54

pound and the weight of refrigerant circulated = 4.17


100 PRINCIPLES OF REFRIGERATION
F then, the smaller weight of
and the coefficient of performance for the 40° Obviously,
refrigerant circulated per minute per ton
cycle is

hf — ha accounts for only a very small part of the

- h(f reduction in the volume of vapor displaced per


hd >

minute per ton at the higher suction tempera-


51.55
ture. To a far greater extent, the decrease in
~ 7.49 the volume of vapor displaced per minute per
= 6.88 ton is a result of the lower specific volume of the
suction vapor which is coincident with the
It is evident that the coefficient of perform-
higher suction temperature (0.792 cu ft/lb at
ance, and hence the efficiency of the cycle, 40° F as compared to 1.351 cu ft/lb at 10° F).
improves considerably as the vaporizing tem- This aspect of system capacity and efficiency in
perature increases. In this instance, increasing relation to suction temperature will be further
the suction temperature from 10° F to 40° F investigated in conjunction with compressor
increases the efficiency of the cycle by
performance in Chapter 12.

6.88 - 4.17 The quantity of heat to be rejected at the


x 100 - 65% condenser per minute per ton is much smaller
4.17
for the cycle having the higher suction tempera-
ture. This is true even though the quantity of
Although the difference in the weight of
which must be circulated per minute
refrigerant heat rejected at the condenser per pound of
refrigerant circulatedis nearly the same for
per ton of refrigerating capacity at the various
usually relatively small, both cycles. For the 10° F cycle, the quantity
suction temperatures is

the volume of vapor that the compressor must of heat rejected at the condenser per minute per

handle per minute per ton varies greatly with ton is

changes in the suction temperature. This is Mha -h = 4.15 )


tt
x 59.74 = 247.92

probably one of the most important factors whereas for the 40° F cycle the heat rejected at
influencing the capacity and efficiency of a the condenser per minute per ton is only
vapor-compression refrigerating system and is
"(Ad- - A„) = 3.88 x 59.04 = 229.08 Btu
the one which is the most likely to be overlooked
by the student when studying cycle diagrams. The quantity of heat rejected per minute per
The difference in the volume of vapor to be ton at the condenser is less for the higher
displaced per minute per ton at the various suction temperature because of (1) the smaller

suction temperatures can be clearly illustrated weight of refrigerant circulated per minute per

by a comparison of the two cycles in question. ton and (2) the smaller heat of compression per
For the 10° F cycle, the volume of vapor to be pound.
It has been shown previously that the heat
displaced per minute per ton is
rejected at the condenser per pound of refriger-
m(v) =4.15 x 1.351 = 5.6cuft ant circulated is the sum of the heat absorbed
in the evaporator per pound (refrigerating
whereas, at the 40° F suction temperature, the
effect) and the heat of compression per pound.
volume of vapor to be displaced per minute per Since increasing the vaporizing temperature of
ton is
the cycle brings about an increase in the
ntv) = 3.88 x 0.792 - 3.075 cu ft refrigerating effect as well as a decrease in the
heat of compression, the quantity of heat
of interest to note that, whereas the
It is
rejected at the condenser per pound remains
decrease in the weight of refrigerant circulated
very nearly the same for both cycles (59.74 at
per minute per ton at the higher suction tempera- 10° F as compared to 59.04 at 40° F). In
ture is only 6.5%, the decrease in the volume
general, this is true for all suction temperatures
of vapor handled by the compressor per minute
because any increase or decrease in the heat
per ton is
usually accompanied by an
of compression is

~^—
5.6 - 3.075
x 100 =45% offsetting increase or decrease in the refrigerat-
ing effect.
CYCLE DIAGRAMS AND THE SIMPLE SATURATED CYCLE 101

171.8
x/** E'J D'

131.6
a/\iw Eh Z)/^137.5'F

/X !

//l 1
'

1 // ' i

/lO* B B 1

fc l!
]!
29.35
r i

/
/
/i
i
i
1

|S?
i
!
I
1

IS
Si /tol cyl iir>
'
|o
/mi id. 1— | 'cm
/ °> Sfi' Is 1 l<T>
CO
/ i II 1 1

Aa h a '
hc h e h e hi hi'
'

Specific enthalpy (Btu/lb above - 40° F)


Fig. 7-7. Comparison of two simple saturated cycles operating at different condensing temperatures (figure
distorted). (Refrigerant- 1 2.)

7-11. Effect of Condensing Temperature on the condensing temperature. Therefore, as


Cycle Efficiency. Although the variations in the condensing temperature is increased, the
cycle efficiency with changes in the condensing temperature of the liquid approaching the
temperature are not as great as those brought refrigerant control is increased and the refriger-
about by changes in the vaporizing temperature, ating effect per pound is reduced. In this
they are nonetheless important. In general, instance, the refrigerating effect is reduced from
if the vaporizing temperature remains constant, 48.20 Btu/lb to 43.20 Btu/lb when the condens-
the efficiency of the cycle decreases as the ing temperature is increased from 100° F to
condensing temperature increases, and increases 120° F. This is a reduction of
as the condensing temperature decreases.
48.20 - 43.20
To illustrate the effect of condensing tempera- X 10°
48.20
ture on cycle efficiency, cycle diagrams of two
saturated cycles operating at different condens-
- 10.37%
ing temperatures are drawn on the Ph chart in Because the refrigerating effect per pound is
Fig. 7-7. One
A, B, C, D, and E, has a
cycle, less for the cycle having the higher condensing

condensing temperature of 100° F, whereas the temperature, the weight of refrigerant to be


other cycle, A', B', C, D', and E', is operating at circulated per minute per ton must be greater.
a condensing temperature of 120° F. The evapo- For the cycle having the 100° F condensing
rating temperature of both cycles is 10° F. Values temperature the weight of refrigerant to be
for cycle A-B-C-D-E have been determined in circulated per minute per ton is 4.15 lb. When
the previous section. Values for cycle A'-B'-C- the condensing temperature is increased to
D'-E' are as follows: 120° F, the weight of refrigerant which must be
From the Ph diagram, circulated per minute per ton increases to

200
9! = he - ha = 79.36 - 36.16 = 43.20 Btu/lb
-
4 631b
-

qi = hd - he = 93.20 - 79.36 = 13.84 Btu/lb


.
4T20=
?s - h d - ha = 93.20 - 36.16 = 57.04 Btu/lb
. . This is an increase of

In a simple saturated cycle the liquid 4.63 - 4.15


x 100 = 11.57%
refrigerant reaches the refrigerant control at 4.15
102 PRINCIPLES OF REFRIGERATION

Since the weight of refrigerant which must be power required per ton at the 100° F condensing
circulated per minute per ton is greater at the temperature is only 1.13 hp when the condensing
higher condensing temperature, it follows that temperature is increased to 120° F, the theoreti-
the volume of vapor to be compressed per cal horsepower per ton increases to
minute per ton must also be greater. In a simple 4.63 x 13.84
saturated cycle the specific volume of the =L52hP
42.42
suction vapor varies only with the vaporizing
temperature. As the vaporizing temperature This is an increase in the power required per ton
is the same for both cycles, the specific volume
of the vapor leaving the evaporator is also the
same for both cycles and therefore the difference
in the volume of vapor to be compressed per
Note that the increase in the horsepower
minute per ton is in direct proportion to the required per ton at the higher condensing
difference in the weight of refrigerant circulated
temperature is greater than the increase in the
per minute per ton. At the 100° F condensing work of compression per pound. This is
temperature the volume of vapor to be com- accounted for by the fact that, in addition to the
pressed per minute per ton is 5.6 cu ft, whereas
20% increase in the work of compression per
at the 120° F condensing temperature the
pound, there is also a 6.5% increase in the
volume of vapor compressed per minute per ton weight of refrigerant circulated per minute per
increases to
ton.
4.63 x 1.351 =6.25cuft The coefficient of performance of the cycle
This represents an increase in the volume of at the 100° F
condensing temperature is 4.17.
vapor compressed per minute per ton of When the condensing temperature is raised to
120° F, the coefficient of performance drops to
6.25
—- 5.6
x 100 = 11.57% 43.20
5.6 = 3.12
13.84
Note that the percent increase in the volume
of vapor handled by the compressor is exactly Since the coefficient of performance is an
equal to the percent increase in the weight of index of the refrigerating capacity per unit of
refrigerant circulated. Contrast this with what power, the decrease in refrigerating capacity per
occurs when the vaporizing temperature is unit of power in this instance is

varied.
Since the difference between the vaporizing
4.17
—- 3.12
x 100 = 33.7%
and condensing pressures is greater, the work
of compression per pound of refrigerant Obviously, the effect of raising the condensing

circulated required to raise the pressure of the


temperature on cycle efficiency is the exact

vapor from the vaporizing to the condensing opposite of that of raising the evaporating

pressure is also greater for the cycle having the


temperature. Whereas raising the evaporating

higher condensing temperature. In this instance, temperature increases the refrigerating effect per
the heat of compression increases from 11.54 pound and reduces the work of compression so
Btu/lb for the 100° F condensing temperature that the refrigerating capacity per unit of power
increases, raising the condensing temperature
to 13.84 Btu/lb for the 120° F condensing
temperature. This is an increase of reduces the refrigerating effect per pound and
increases the work of compression so that the
-
—13.84
iL54—
11.54
Xl0°= 20%

As a result of the greater work of compression


refrigerating
creases.
capacity per unit of power de-

Although the quantity of heat rejected at the


per pound and the greater weight of refrigerant condenser per pound of refrigerant circulated
circulated per minute per ton, the theoretical varies only slightly with changes in the condens-
horsepower required per ton of refrigerating ing temperature because any change in the heat
capacity increases as the condensing tempera- of compression is accompanied by an offsetting
ture increases. Whereas the theoretical horse- change in the refrigerating effect per pound,
CYCLE DIAGRAMS AND THE SIMPLE SATURATED CYCLE 103

Td

Fig. 7-8. Temperature-entropy i


diagram of simple saturated £
cycle on skeleton Ts chart M T*
(figure distorted). (Refrigerant- J
12.) I

Specific entropy (Btu/lb *F)

the total heat rejected at the condenser per greater portion of the condenser surface is being
minute per ton varies considerably with changes used merely to reduce the temperature of the
in the condensing temperature because of the discharge vapor to the condensing temperature.
difference in the weight of refrigerant circulated 7-12. The Temperature-Entropy Diagram.
per minute per ton. It was shown in Section 7-7 Although the author is partial to the pressure-
that the total heat rejected at the condenser per enthalpy diagram, there are many who prefer
minute per ton (gj) is always the sum of the to use the temperature-entropy diagram to
heat absorbed in the evaporator per minute per analyze the refrigeration cycle. To acquaint
ton (Qx ) and the total heat of compression per the student with the use of Ts diagrams in'cycle
ton (Q 2 ). Since Q x is always 200 Btu/min/ton, analysis, a diagram of the simple saturated cycle
then Q3 will vary only with Q2 , the heat of described in the foregoing sections is drawn

compression per minute per ton. Furthermore, on Ts coordinates in Fig. 7-8. The state points
since Q 2 always increases as the condensing A, B, C, D, and E represent the points in the
temperature increases, it follows then that Q3 cycle as shown by the flow diagram in Fig. 7-5.
also increases as .the condensing temperature The state point X represents saturated liquid
increases. at the vaporizing temperature. Process A-B is
For the two cycles in question, the heat the irreversible adiabatic expansion through the
rejected at the condenser per minute per ton refrigerant control.* Process B-C is the iso-
at the 100° F
condensing temperature is 218.75 baric and isothermal vaporization in the
Btu. For the 120° F condensing temperature, evaporator. Process C-D is the reversible
the quantity of heat rejected at the condenser per adiabatic (isentropic) compression in thecom-
minute per ton increases to pressor and processes D-E and E-A are the
4.63 x (43.20 + 13.84) = 310.4 Btu desuperheating and condensing processes in the
condenser. Tt Tc and Td are the absolute
, ,
The percent increase is
suction, absolute condensing, and absolute

—lii™
-
310.40 218.75
>< 100 =41-8% discharge temperatures, respectively, whereas
sa , sb, se , sd , st , and s x are the specific entropies

It is interesting to note also that the amount of the refrigerant at the various state points.
of sensible heat rejected at the condenser in- The principal advantage of the Ts diagram is

creases considerably at the higher condensing that the areas shown on the chart represent

temperature, whereas the amount of latent heat actual heat quantities. In Fig. 7-8, the area
rejected diminishes slightly. This indicates that, * The line A-B does not necessarily follow the
at the higher condensing temperature, a actual path of process A-B. See Section 7-4.
104 PRINCIPLES OF REFRIGERATION

Condensing Temperature, 100° F Condensing Pressure, 136.16 Psia

Suction temperature 50° 40° 30° 20° 10° 0° -10° -20° -30° -40°

Absolute suction pressure 61.39 51.68 43.16 35.75 29.35 23.87 19.20 15.28 12.02 9.32

Refrigerating effect per


pound 52.62 51.55 50.45 49.33 48.20 47.05 45.89 44.71 43.54 42.34

Weight of refrigerant
circulated per minute per
ton 3.80 3.88 3.97 4.05 4.15 4.25 4.36 4.48 4.59 4.73

Specific volume of suction


vapor 0.673 0.792 0.939 1.121 1.351 1.637 2.00 2.47 3.09 3.91

Volume of vapor com-


pressed per minute per ton 2.56 3.08 3.77 4.55 5.60 6.96 8.72 11.10 14.20 18.50

Heat of compression per


pound 6.01 7.49 8.79 10.11 11.54 13.29 14.85 16.73 18.50 20.40

Theoretical horsepower per


ton 0.539 0.683 0.818 0.965 1.13 1.35 1.54 1.78 2.00 2.26

Coefficient of performance 8.76 6.88 5.74 4.88 4.17 3.54 3.09 2.67 2.36 2.07

Fig. 7-9

20

18

4 16

3.5 14 c

3.0 a 12 -
E
2.5 f 10
|
*
2-0
1Q. 8

1.5 6

1.0 4

0.5 2
^40 -30 -20 -10 10 20 30 40 50
Suction temperature

Fig. 7-IOa. For Refrigerant- 1 2, the refrigerating effect per pound, the weight of refrigerant circulated per
minute per ton, the specific volume of the suction vapor, and the volume of vapor compressed per minute
per ton are each plotted against suction temperature. Condensing temperature is constant at 100° F.
CYCLE DIAGRAMS AND THE SIMPLE SATURATED CYCLE 105

2.6

2.2
\V
8 g
\ a
Fig. 7-IOb. For Refrigerant- 1 2, V
the coefficient of performance
and the horsepower per ton are
I
1.6 ^ 1
plotted against suction tem- o.
Q) cop-P"* £
perature. Condensing tempera- £ 1.2 5 S
o
ture is constant at 100° F. I i
0.8 4$
0.4 ^Horserjower
per ton
1

-40 - 30-20-10 10 20 30 40 50
Suction temperature

A-X-C-D-E-A represents the heat energy pound, the weight of refrigerant circulated per
equivalent of the work of adiabatic compression minute per ton, the specific volume of the suction
and, since the distance between the base line and vapor, the volume of vapor compressed per
T„ foreshortened in the figure, represents the minute per ton, the horsepower required per
absolute vaporizing temperature of the liquid, ton, and the coefficient of performance of the
area B-C-Sc-sg-B represents the refrigerating cycle has been calculated for various suction
effect per pound. The sum of the areas A-X-C- temperatures. These values are given in tabular
D-E-A and B-C-Sg-sg-B, of course, represents form in Fig. 7-9 and are illustrated graphically
the heat rejected at the condenser per pound. and 7-106. In addition, the effect
in Figs. 7-10a
As in the case of the Ph diagram, it can be of condensing temperature on the horsepower
readily seenon the Ts chart that either lowering required per ton of refrigerating capacity is
the vaporizing temperature or raising the con- shown for several condensing temperatures in
densing temperature tends to increase the work Fig. 7-11.
of compression, reduce the refrigerating effect Since the properties of the refrigerant at point
per pound, and lower the efficiency of the cycle. D on the cycle diagram cannot ordinarily be
7-13. Summary. Regardless of the method obtained from the refrigerant tables and since
used to analyze the cycle, it is evident that the these properties are difficult to read accurately
capacity and efficiency of a refrigerating system from the Ph chart because of the size of the
improve as the vaporizing temperature increases chart, the approximate isentropic discharge
and as the condensing temperature decreases.
Obviously, then, a refrigerating system should
always be designed to operate at the highest
\
vs X v *r1
possible vaporizing temperature and the lowest a
possible condensing temperature commensurate
with the requirements of the application. This
will nearly always permit the most effective use
\ i
H„
^ *£
sjfcfc?
>o
of the smallest possible equipment and thereby
effect a savings not only in the initial cost fc^l
^"s;
§ -^_
-- — ~.
of the equipment but also in the operating
expenses.
1 ^jT
In any event, the influence of the vaporizing
-40 -30 -20 -10 10 20 30 40 50
and condensing temperatures on cycle efficiency Suction temperature
is of sufficient importance to warrant a more

intensive study. To aid the student in this, the Fig. 7-11. The effect of condensing temperature on
relationship between the refrigerating effect per the horsepower per ton.
106 PRINCIPLES OF REFRIGERATION

temperatures and the approximate enthalpy (3) The volume of vapor compressed per
of the refrigerant vapor at point have been D minute per ton.
compiled for a variety of vaporizing and con- Ans. 7.35 cu ft/min/ton
densing temperatures and are given in Table 7-1 B. (1) The heat of compression per pound of
more accurate refrigerant circulated.
to aid the student in arriving at
Ans. 14.39 Btu/lb
solutions to the problems at the end of the
(2) The heat of compression per minute
chapter.
per ton of refrigeration.
PROBLEMS Ans. 64.61 Btu/min/ton
1. ARefrigerant-12 system operating on a (3) The work of compression per minute
simple saturated cycle has an evaporating tem- per ton in foot-pounds.
perature of 0° F and a condensing temperature Ans. 50.267 lb/min/ton
of 110° F. Determine: (4) The theoretical horsepower per ton.
A. (1) The refrigerating effect per pound of Ans. 1.52hp/ton
refrigerant circulated. (5) The coefficient of performance.
Arts. 44.56 Btu/lb Ans. 3.1
(2) The weight of refrigerant circulated C. (1) The heat rejected per minute per ton
per minute per ton. at the condenser.
Arts. 4.49 lb/min/ton Ans. 264.61 Btu/min/ton
F

On the Ph diagram in Fig. 8-2, a simple


saturated cycle compared to one in which the
is

suction vapor is superheated from 20° F to


70° F. Points A B, C, D, and E mark the satur-
,

ated cycle, and points A, B, C", D', and E indi-


cate the superheated cycle.
If the slight pressure drop resulting from the
8 flow of the vapor in the suction piping
neglected, it may
be assumed that the pressure
is

of the suction vapor remains constant during


the superheating. That is, after the superheat-
Actual ing, the pressure of the vapor at the suction
inletof the compressor is still the same as
Refrigerating the vaporizing pressure in the evaporator.
With this assumption, point C'can be located on
Cycles the
pressure
Ph chart by following a line of constant
from point C to the point where the
line of constant pressure intersects the 70° F
constant temperature line. Point D' is found by
following a line of constant entropy from point
C to the line of constant pressure corresponding
8-1.Deviation from the Simple Saturated to the condensing pressure.
Cycle. Actual refrigerating cycles deviate some- In Fig. 8-2, the properties of the superheated
what from the simple saturated cycle. The reason vapor at points C" and D', as read from the Ph
for this is that certain assumptions are made for chart, are as follows:
the simple saturated cycle which do not hold At point C",
true for actual cycles. For example, in the
simple saturated cycle, the drop in pressure in
p = 35.75 psia / = 70° F
the lines and across the evaporator, condenser,
v = 1 .260 cu ft/lb h = 88.6 Btu/lb

etc., resulting from the flow of the refrigerant


s = 0.1840 Btu/lb/° R
through these parts is neglected. Furthermore,
the effects of subcooling the liquid and of super-
, 20*F
-*&
c 35.75 psia
heating the suction vapor are not considered.
Too, compression in the compressor is assumed
20* F
D 8
to be true isentropic compression. In the
c 35.75 psia

following sections all these things are taken into '


30"F
35.75 psia 4
C
\
20* F
J
account and their effect on the cycle is studied Saturated vapor
in detail.* 50* F
Superheated vapor 8
8-2. The Effect of Superheating the Suction
Vapor. In the simple saturated cycle, the suction
70*F ^ XT 164* F_
vapor is assumed to reach the suction inlet of /^\
the compressor as a saturated vapor at the 35.75 r^i 131.6 psi'a 100° E 3
psia
vaporizing temperature and pressure. In actual 100* F r
practice, this is rarely true. After the liquid 131.6 psia ")

has completely vaporized in the


refrigerant
evaporator, the cold, saturated vapor will
c A 100*
131.6 psia
usually continue to absorb heat and thereby
Fig. 8-1. Flow diagram of superheated cycle. Liquid
become superheated before it reaches the
compressor (Fig. 8-1).
completely vaporized at point C saturated vapor —
continues to absorb heat while flowing from C to
* The departure from true isentropic compression C— vapor reaches compressor in superheated
and the effect that it has on the cycle are discussed condition. Notice the high discharge temperature.
in Chapter 12. (Refrigerant- 1 2.)

107
108 PRINCIPLES OF REFRIGERATION

131.16

£ 35.75

Superheat

Enthalpy (Btu/lb)

Fig. 8-2. Ph diagram comparing simple saturated cycle to the superheated cycle. (Refrigerant- 1 2).

At point/)', ably higher for the superheated cycle than for

/> = 131.6 psia f = 164°F



the saturated cycle in this case, 164° F for the
superheated cycle as compared to 1 12° F for the
v - 0.380 cu ft/lb h = 99.2 Btu/lb
saturated cycle.
j = 0.1840 Btu/lb/° R 3. For the superheated cycle, a greater
quantity of heat must be dissipated at the
On the Ph chart, process C-C represents the
superheating of the suction vapor from 20° F
condenser per pound than for the saturated
cycle. This is because of the additional heat
to 70° F at the vaporizing pressure, and the
difference between the enthalpy of the vapor absorbed by the vapor in becoming superheated
amount of heat required to
at these points is the
and because of the small increase in the heat of
superheat each pound of refrigerant. In compression per pound. For the superheated
cycle, the heat dissipated at the condenser per
comparing the two cycles, the following
observations are of interest: pound is

h- ~ K - 99.2 - 31.16 = 68.04 Btu/lb


1 . The heat of compression per pound for the
superheated cycle is slightly greater than that and for the saturated cycle the heat dissipated
for the saturated cycle. For the superheated at the condenser per pound is

cycle, the heat of compression is


hd -ha = 90.6 - 31.16 = 59.44 Btu/lb
hx - h& = 99.2 - 88.6 = 10.6 Btu/lb
The percent increase in the heat dissipated at
whereas for the saturated cycle the heat of the condenser per pound for the superheated
compression is

In
hd -he = 90.6 - 80.49 =
this instance, the
10.11 Btu/lb

heat of compression per


cycle is


68.04-59.44
x tnn
100 = ijjd ,
14.4%

pound is greater for the superheated cycle by Note that the additional heat which must be
- dissipated per pound at the condenser in the
10.6 10.11
x 100 =» 4.84% superheated cycle is all sensible heat. The
10.11
amount of latent heat dissipated per pound is
2. For the same condensing temperature and the same for both cycles. This means that in the
pressure, the temperature of the discharge vapor superheated cycle a greater amount of sensible
leaving the head of the compressor is consider- heat must be given up to the condensing medium
ACTUAL REFRIGERATING CYCLES 109

before condensation begins and that a greater The weight of refrig- 200
portion of the condenser will be used in cooling erant circulated per fie ~K
the discharge vapor to its saturation temperature. minute per ton m 200
Notice also that, since the pressure of the
49.33
suction vapor remains constant during the
superheating, the volume of the vapor increases = 4.05 lb/min/ton
with the temperature approximately in accord-
ance with Charles' law.* Therefore, a pound Since the weight of refrigerant circulated is the
of superheated vapor will always occupy a same for both the superheated and saturated
greater volume than a pound of saturated vapor cycles and since the specific volume of the vapor
at the same pressure. For example, in Fig. 8-2, at the compressor inlet is greater for the super-

the specific volume of the suction vapor increases heated cycle than for the saturated cycle, it
from 1.121 cu ft per pound at saturation to follows that the volume of vapor that the com-
1 .260 cu ft per pound when superheated to 70° F.
pressor must handle per minute per ton of
This means that for each pound of refrigerant refrigerating capacity is greater for the super-

circulated, the compressor must compress a heated cycle than for the saturated cycle.
greater volume of vapor if the vapor is super-
For the saturated
heated than if the vapor is saturated. For this
cycle, the specific
reason, in every instance where the vapor is
volume of the suction
allowed to become superheated before it reaches
vapor v e = 1.121 cu ft/lb
the compressor, the weight of refrigerant cir-
culated by a compressor of any given displace- The volume of =m xv
ment will always be less than when the suction vapor compressed per = 4.05 x 1.121
vapor reaches the compressor in a saturated minute per ton V = 4.55 cu ft/min/ton
condition, provided the pressure is the same. For the superheated
The effect that superheating of the suction cycle, the specific
vapor has on the capacity of the system and on volume of the suction
the coefficient of performance depends entirely vapor «v = 1.260 cu ft/lb
upon where and how the superheating of the
The volume of vapor =m xv
vapor occurs and upon whether or not the
compressed per minute = 4.05 x 1.260
heat absorbed by the vapor in becoming super-
per ton V = 5.02 cu ft/min/ton
heated produces useful cooling.*
8-3. Superheating without Useful Cooling.
of the In regard to percentage, the increase in the
Assume first that the superheating
suction vapor occurs in such a way that no use- volume of vapor which must be handled by a
ful cooling results. When this is true, the refri- compressor operating on the superheated cycle
gerating effect per pound of refrigerant circulated
is the same for the superheated cycle as for a
same vaporizing
saturated cycle operating at the
and condensing temperatures, and therefore the
weight of refrigerant circulated per minute per This means, of course, that a compressor
ton will also be the same for both the superheated operating on the superheated cycle must be
and saturated cycles. Then, for both cycles 10.3% larger than the one required for the
illustrated in Fig. 8-2, saturated cycle.
Again, since the weight of refrigerant circu-
* The temperature and volume of the vapor do lated perminute per ton is the same for both
not vary exactly in accordance with Charles' law cycles and since the heat of compression per
because the refrigerant vapor is not a perfect gas.
pound is greater for the superheated cycle than
* The effects of superheating depend also upon
for the saturated cycle, it is evident that the
the refrigerant used. The discussion in this chapter
horsepower per ton is greater for the super-
is limited to systems using R-12. The effects of

superheating on systems using other refrigerants are heated cycle and the coefficient of performance
discussed later. is less.
110 PRINCIPLES OF REFRIGERATION
For the saturated cycle, m(h d - he) weight of refrigerant circulated per minute per
the horsepower per ton 42.42 ton for the superheated cycle is

4.05 x 10.11 200 200


= 3.48 lb/min/ton
42.42 h- - hn 57.44

= 0.965 hp/ton Notice that, even though the specific volume

The — of the suction vapor and the heat of compression


coefficient of per- he ha
formance per pound are both greater for the superheated
hd -K than for the saturated cycle, the volume of vapor
49.33 compressed per minute per ton and the horse-
10.11 power per ton are less for the superheated cycle

= 4.88 than for the saturated cycle. This is because of


the reduction in the weight of refrigerant circu-
For the superheated m(h d - - h C ')
lated. The volume of vapor compressed per
cycle, the horsepower per 42.42
minute per ton and the horsepower per ton for
ton
4.05 x 10.6 the saturated cycle are 4.55 cu ft and 0.965 hp,
42.42 respectively, whereas for the superheated cycle
= 1.01 hp/ton The volume of = m x vc -

The coefficient of per- hc ~ha vapor compressed per =3.48 x 1.260


formance hd ' - K-
minute per ton V = 4.38 cu ft/min/ton

49.33 The horsepower per m{h a > — A c .)


ton 42.42
10.60
= 3.48 x 10.60
4.65
42.42
In summary, when superheating of the vapor
occurs without producing useful cooling, the
= 0.870 hp/ton

volume of vapor compressed per minute per ton, For the superheated cycle, both the refrig-
the horsepower per ton, and the quantity of heat erating effect per pound and the heat of com-
given up in the condenser per minute per ton are pression per pound are greater than for the
all greater for the superheated cycle than for the saturated cycle. However, since the increase in
saturated cycle. This means that the com- the refrigerating effect is greater proportionally
pressor, the compressor driver, and the conden- than the increase in the heat of compression, the
sermust all be larger for the superheated cycle coefficient of performance for the superheated

than for the saturated cycle. cycle is higher than that of the saturated cycle.
8-4.Superheating That Produces Useful For the saturated cycle, the coefficient of per-
Cooling. Assume, now, that all of the heat formance is 4.69, whereas for the superheated
taken in by the suction vapor produces useful cycle
cooling. When this is true, the refrigerating effect The coefficient -
(h c . h a) 57.44
perpound is greater by an amount equal to the of performance = 5.42
amount of superheat. In Fig. 8-2, assuming that (4r - hc ) 10.60

the superheating produces useful cooling, the It will be shown in the following sections that

refrigerating effect per pound for the superheated the superheating of the suction vapor in an
cycle is equal to actual cycle usually occurs in such a way that a
part of the heat taken in by the vapor in becom-
h^ -h a = 88.60 - 31.16 = 57.44 Btu/lb
ing superheated is absorbed from the refrig-
Since the refrigerating effect per pound is erated space and produces useful cooling,
greater for the superheated cycle than for the whereas another part is absorbed by the vapor
saturated cycle, the weight of refrigerant circu- after the vapor leaves the refrigerated space and
lated per minute per ton is less for the super- therefore produces no useful cooling. The por-
heated than for the saturated cycle.
cycle tion of the superheat which produces useful
Whereas the weight of refrigerant circulated per cooling will depend upon the individual applica-
minute per ton for the saturated cycle is 4.05, the tion, and the effect of the superheating on the
ACTUAL REFRIGERATING CYCLES

cycle will vary approximately in proportion to of the refrigerated space should be eliminated
the useful cooling accomplished. whenever practical.

Regardless of the effect on capacity, except Superheating of the suction vapor in the
in some few special cases, a certain amount of suction line can be prevented for the most part
superheating is nearly always necessary and, in by insulating the suction line. Whether or not
most cases, desirable. When the suction vapor the loss of cycle efficiency in any particular

is drawn directly from the evaporator into the application is sufficient to warrant the additional
suction inlet of the compressor without at least expense of insulating the suction line depends
a small amount of superheating, there is a good primarily on the size of the system and on the
possibility that small particles of unvaporized operating suction temperature.
liquid will be entrained in the vapor. Such a When the suction temperature is relatively

vapor is "wet" vapor. It will be shown


called a high (35° F or 40° F), the amount of superheating
later that "wet" suction vapor drawn into the will usually be small and the effect on the
cylinder of the compressor adversely affects the efficiency of the cycle will be negligible. The
capacity of the compressor. Furthermore, since reverse is true, however, when the suction tem-

refrigeration compressors are designed as vapor perature is low. The amount of superheating is

pumps, if any appreciable amount of un- apt to be quite large.


vaporized liquid is allowed to enter the com- Too, at low suction temperatures, when the
pressor from the suction line, serious mechanical efficiency of the cycle is already very low, each
damage to the compressor may result. Since degree of superheat will cause a greater reduc-
superheating the suction vapor eliminates the tion in cycle efficiency percentagewise than when
possibility of "wet" suction vapor reaching the the suction temperature is high.It becomes

compressor a certain amount of super-


inlet, immediately apparent that any appreciable
heating is usually desirable. Again, the extent amount of superheating in the suction line of
to which the suction vapor should be allowed to systems operating at low suction temperatures
become superheated in any particular instance will seriously reduce the efficiency of the cycle
depends upon where and how the superheating and under these conditions, insulating of
that,

occurs and upon the refrigerant used. the suction line is not only desirable but
Superheating of the suction vapor may take absolutely necessary if the efficiency of the cycle
place in any one or in any combination of the is to be maintained at a reasonable level.
following places: Aside from any considerations of capacity,
even at the higher suction temperatures, insulat-
1. In the end of the evaporator ing of the suction line is often required to prevent
2. In the suction piping installed inside the frosting or sweating of the suction line. In
refrigerated space (usually referred to as a "drier flowing through the suction piping, the cold
loop") suction vapor will usually lower the temperature
3. In the suction piping located outside of the of the piping below the dew point temperature
refrigerated space of the surrounding air so that moisture will
4. In a liquid-suction heat exchanger. condense out of the air onto the surface of the
piping, causing the suction piping to either frost
8-5. Superheating in Suction Piping outside or sweat, depending upon whether or not the
the Refrigerated Space. When the cool re- temperature of the piping is below the freezing
frigerant vapor from the evaporator is allowed temperature of water. In any event, frosting or
to become superheated while flowing through sweating of the suction piping is usually un-
suction piping located outside of the refrigerated desirable and should be eliminated by insulating
by the vapor is absorbed
space, the heat taken in the piping.
from the surrounding air and no useful cooling 8-6. Superheating the Vapor inside the Re-
results. It has already been demonstrated that frigerated Space. Superheating of the suction
superheating of the suction vapor which pro- vapor inside the refrigerated space can take
duces no useful cooling adversely affects the place either in the end of the evaporator or in
efficiency of the cycle. Obviously, then, super- suction piping located inside the refrigerated
heating of the vapor in the suction line outside space, or both.
112 PRINCIPLES OF REFRIGERATION

great to a vapor as to a liquid, the capacity of the


evaporator is always reduced in any portion of

the evaporator where only vapor exists. There-


fore, excessive superheating of the suction vapor
in the evaporator will reduce the capacity of the
evaporator unnecessarily and will require either
that the evaporator be operated at a lower
vaporizing temperature or that a larger evapora-
tor be used in order to provide the desired
evaporator capacity. Neither of these is desir-
able nor practical. Since the space available for
evaporator installation is often limited and since
evaporator surface is expensive, the use of a
larger evaporator is not practical. Because of
the effect on cycle efficiency, the undesirability of
lowering the vaporizing temperature is obvious.
Often, a certain amount of suction piping,
Fig. 8-3. Flow diagram showing drier loop for super- usually called a drier loop, is installed inside the
heating suction vapor inside refrigerated space. refrigerated space for the express purpose of
superheating the suction vapor (Fig. 8-3). Use
of a drier loop permits more complete flooding
To assure the proper operation of the refri- of the evaporator with liquid refrigerant without
gerant control and to prevent liquid refrigerant the danger of the liquid overflowing into the
from overflowing the evaporator and being suction line and being drawn into the compres-

carried back to the compressor, when certain sor. This not only provides a means of super-

types of refrigerant controls are used, it is heating the suction vapor inside the refrigerated
necessary to adjust the control so that the liquid space so that the efficiency of the cycle is
iscompletely evaporated before it reaches the increased without the sacrifice of expensive eva-
end of the evaporator. In such cases, the cold porator surface, but it actually makes possible
vapor will continue to absorb heat and become more effective use of the existing evaporator
superheated as it flows through the latter portion surface. Also, in some instances, particularly
of the evaporator. Since the heat to superheat where the suction temperature is high and the
the vapor drawn from the refrigerated space,
is
relative humidity of the outside reasonably
air is

useful cooling results and the refrigerating effect low, superheating of the suction vapor inside the
of each pound of refrigerant is increased by an refrigerated space will raise the temperature of

amount equal to the amount of heat absorbed in the suction piping and prevent the formation of
the superheating. moisture, thereby eliminating the need for suction
has been shown that when the superheating
It line insulation. It should be noted, however,

of the suction vapor produces useful cooling the that the extent to which the suction vapor can be

efficiency of the cycle is improved somewhat.* superheated inside the refrigerated space is
However, in spite of the increase in cycle effi- limited by the space temperature. Ordinarily, if
ciency, it must be emphasized that superheating sufficient piping is used, the suction vapor can

the suction vapor in the evaporator is not be heated to within 4° F to 5° F of the space
economical and should always be limited to only temperature. Thus, for a 40° F space tempera-
that amount which is necessary to the proper ture, the suction vapor may be superheated to
operation of the refrigerant control. Since the approximately 35° F.
transfer of heat through the walls of the evapora- 8-7.The Effects of Subcooling the Liquid.
tor per degree of temperature difference is not as On the PA diagram in Fig. 8-4, a simple saturated
cycle is compared to one in which the liquid is
* Although this is true for systems using R-12 as subcooled from 100° F to 80° F before it reaches
a refrigerant, will be shown later that this is not
it the refrigerant control. Points A, B, C, D, and E
true for all refrigerants. designate the simple saturated cycle, whereas
ACTUAL REFRIGERATING CYCLES 113

points A', ff, C, D, and E designate the sub- for the subcooled cycle than for the saturated
cooled cycle. cycle,it follows that the volume of vapor which

Ithas been shown (Section 6-28) that when the compressor must handle per minute per ton
the liquid is subcooled before it reaches the will also be less for the subcooled cycle than for
refrigerant control the refrigerating effect per the saturated cycle.
pound is increased. In Fig. 8-4, the increase in
the refrigerating effect per pound resulting from For the saturated cycle,

the subcooling is the difference between h h and the specific volume of the
h v , and is exactly equal to the difference between suction vapor ve 1.121 cu ft/lb

h a and ha; which represents the heat removed The volume of vapor m x ve
from the liquid per pound during the subcooling. compressed per minute per 4.05 x 1.121
ton V 4.55 cu ft/min
For the saturated cycle, —K—K For the subcooled cycle,
the refrigerating effect per = 80.49 -31.16 the specific volume of the
pound, qx = 49.33 Btu/lb
suction vapor ve 1.121 cu ft/lb
For the subcooled cycle, = he — h >

the refrigerating effect per = 80.49 - 26.28


tt
The volume of vapor m x ve
compressed per minute per 3.69 x 1.121
pound, qt = 54.21 Btu/lb
ton V :
4.15 cu ft/min
Because of the greater refrigerating effect per
Because the volume of vapor compressed per
pound, the weight of refrigerant circulated per
minute per ton is less for the subcooled cycle,
minute per ton is less for the subcooled cycle
the compressor displacement required for the
than for the saturated cycle.
subcooled cycle is less than that required for the
For the saturated cycle, the 200 saturated cycle.
weight of refrigerant circulated ~ 49.33 Notice also that the heat of compression per
per minute per ton m = 4.05 lb pound and therefore the work of compression
For the subcooled cycle, the 200 per pound is the same for both the saturated and
— subcooled cycles. This means that the refri-
weight of refrigerant circulated 54.21
= gerating effect per pound resulting from the
per minute per ton m 3.69 lb
subcooling is accomplished without increasing
Notice that the condition of the refrigerant the energy input to the compressor. Any change
vapor entering the suction inlet of the compressor in the refrigerating cycle which increases the
is the same for both cycles. For this reason, the quantity of heat absorbed in the refrigerated
specific volume of the vapor entering the com- space without causing an increase in the energy
pressor will be the same for both the saturated input to the compressor will increase the c.o.p.
and subcooled cycles and, since the weight of of the cycle and reduce the horsepower required
refrigerant circulated per minute per ton is less per ton.

131.16

Fig. 8-4. Ph diagrams compar-


ing the subcooled cycle to
the simple saturated cycle. S
(Refrigerant- 1 2.)
: 35.75

Enthalpy (Btu/lb)
114 PRINCIPLES OF REFRIGERATION
Liquid-vapor mixture
35.75 psia-20°F

m<
K Subcooled liquid
131.16

20*
psia-SCF

Liquid subcooled
Fig. 8-5. Flow diagram illus-
by giving off heat to
surrounding air while trating subcooling of the liquid
Saturated vapor passing through
in the liquid line. (Refrigerant-
131.16 psia-lOOV liquid line, receiver,
Superheated vapor
etc. ,
12.)
131.16 paa-112'F

Saturated liquid
o
Saturated vapor
) 131.16 psia-100 F

3575psia-20*F

C K
For the saturated cycle, K-K subcooling is very often more than sufficient to

the coefficient of perfor- — hc offset the additional cost of the subcooler,


hd
mance 80.49 - 31.16 particularly for low temperature applications.
~ - The liquid subcooler may be piped either in
90.60 80.49
series or in parallel with the condenser. When
= 4.88
the subcooler is piped in series with the con-
The horsepower per ton _ m(h d — hc)
denser, the cooling water passes through the
42A2 subcooler first and then through the condenser,
= 4.05 x 10.51 thereby bringing the coldest water into contact
= 0.965 hp/ton with the liquid being subcooled (Fig. 8-7).
For the subcooled cycle, hc .
— h„' There is some doubt about the value of a sub-
the coefficient of perfor- hd -K cooler piped in series with the condenser. Since
mance 80.49 - 26.28 - the cooling water is warmed by the heat absorbed
in the subcooler, reaches the condenser at a
90.60 - 80.49
it

higher temperature and the condensing tempera-


54.21
ture of the cycle is increased . Hence the increase
10.51
in system efficiency resulting from the subcooling
= 5.16
is offset to some extent by the rise in the con-
The horsepower per ton m{h d - he) densing temperature.
42.42 When the subcooler is piped in parallel with
3.69 x 10.51 the condenser (Fig. 8-6), the temperature of the
42.42 water reaching the condenser is not affected by
= 0.914 hp/ton the subcooler. However, for either series or

In this instance, the c.o.p. of the subcooled cycle parallel piping, the size of the condenser water

is greater than that of the saturated cycle by pump must be increased when a subcooler is
added. If this is not done, the quantity of water
5.H5 -4.88
circulated through the condenser will be
~^M~ X 10° = 5 - 7%
diminished by the addition of the subcooler and
Subcooling of the liquid refrigerant can and the condensing temperature of the cycle will be
does occur in several places and in several ways. increased, thus nullifying any benefit accruing
Very often the liquid refrigerant becomes sub- from the subcooling.
cooled while stored in the liquid receiver tank Notice that in each case discussed so far,
or while passing through the liquid line by giving the heat given up by the liquid in becoming
off heat to the surrounding air (Fig. 8-5). In subcooled is given up to some medium external
some cases where water is used as the condensing to the system.
medium, a special liquid .subcooler is used to 8-8. Liquid-Suction Heat Exchangers. An-
subcool the liquid (Fig. 8-6). The gain in system othermethod of subcooling the liquid is to bring
capacity and efficiency resulting from the liquid about an exchange of heat between the liquid
»

ACTUAL REFRIGERATING CYCLES 115

and the cold suction vapor going back to the


compressor. In a liquid-suction heat exchanger,
the cold suction vapor is piped through the
heat exchanger in counterflow to the warm
liquid refrigerant flowing through the liquid
line to the refrigerant control (Fig. 8-8). In
flowing through the heat exchanger the cold
suction vapor absorbs heat from the warm
liquid so that the liquid is subcooled as the
vapor is superheated, and, since the heat
absorbed by the vapor in becoming superheated
is drawn from the liquid, the heat of the liquid

is diminished by an amount equal to the amount

of heat taken in by the vapor. In each of the


Water from^/ Saturated liquid
methods of subcooling discussed thus far, the condenser to subcooler

heat given up by the liquid in becoming sub-


Fig. 8-7. Flow diagram showing parallel piping for
cooled is given up to some medium external to
condenser and subcooler.
the system and the heat then leaves the system.
When a liquid-suction heat exchanger is used,
the heat given up by the liquid in becoming Since the heat given up by the liquid in
subcooled is absorbed by the suction vapor and the heat exchanger in becoming subcooled is
remains in the system. exactly equal to the heat absorbed by the vapor
On the Ph diagram in Fig.
a simple 8-9, in becoming superheated, h a — h a is equal to -

saturated cycle is compared to one in which a hC — h c and therefore is also equal to 5.71
'

liquid-suction heat exchanger is employed. Btu/lb. Since h a — ha represents an increase'

Points A, B, C, Z),and £ identify the saturated in the refrigerating effect, the refrigerating
cycle and points A', B', C", D', E identify the effect per pound for the heat exchanger cycle is
in which the heat exchanger is used. In the cycle
latter cycle, it is assumed that the suction vapor
hc - ha . = 80.49 - 25.45 = 55.04

is superheated from 20° F to 60° F in the heat The heat of compression per pound for the heat
exchanger. exchanger cycle is
The heat absorbed per pound of vapor in the
heat exchanger is
ha . - hd = 97.60 - 86.20 = 11.40

Therefore, the coefficient of performance is


hc . -h = c 86.20 - 80.49 = 5.71 Btu/lb

K - hn . 55.04
= 4.91
h,.-h, 11.40

The coefficient of performance of the satur-


D 5 ated cycle is 4.88. Therefore, the coefficient of
7 performance of the heat exchanger cycle is
S3
greater than that of the saturated cycle by only

4.91 -4.88
Water from subcooler 75' i tower
x 100 =0.5%
or city main
4.88
Water-cooled
condenser Depending upon the particular case, the
(10O*F condensing) t/80-

"^ 100* Liquid to


coefficient ofperformance of a cycle employing
'subcooler a heat exchanger may be either greater than,
90* water to cooling tower or sewer less than, or the same as that of a saturated
cycle operating between the same pressure
Fig. 8-4. Flow diagram illustrating subcooler piped limits. In any event, the difference is negligible,
in series with condenser. and it is evident that the advantages accruing
116 PRINCIPLES OF REFRIGERATION
20* vaporizing
temperature

Saturated
suctio n f
vapor- v
20'

Saturated Fig. 8-8. Flow diagram of refrigera-


liquid-lOOT tion cycle illustrating the use of a
Subcooled
liquid-75"F~ liquid-suction heat exchanger.

100* condensing
temperature

from the subcooling of the liquid in the heat always become superheated before the com-
exchanger are approximately offset by the pression process begins because nothing can be
disadvantages of superheating the vapor. done to prevent it. This is true even if no
Theoretically, then, the use of a heat exchanger superheating takes place either in the evapora-
cannot be justified on the basis of an increase in tor or in the suction line and the vapor reaches

system capacity and efficiency. However, since the inlet of the compressor at the vaporizing
in actual practice a refrigerating system does temperature. As the cold suction vapor flows
not (cannot) operate on a simple saturated into the compressor,it will become superheated

cycle, this does not represent a true appraisal of by absorbing heat from the hot cylinder walls.
the practical value of the heat exchanger. Since the superheating in the compressor cyl-
In an actual cycle, the suction vapor will inder will occur before the compression process

131.16

&

35.75 ,

Enthalpy (Btu/lb)

Hg. 8-». Ph diagrams comparing simple saturated cycle to cycle employing a liquid-suction heat exchanger.
The amount of subcooling is equal to the amount of superheating. (Refrigerant- 2.) 1
ACTUAL REFRIGERATING CYCLES 117

begins, the effect of the superheating on cycle vapor is always greater than the reduction in the
efficiency will be approximately the same as if temperature of the liquid. For instance, the
the superheating occurred in the suction line specific heat of R-12 liquid is approximately
without producing useful cooling.* 0.24 Btu per pound, whereas the specific heat of
The disadvantages resulting from allowing the the vapor is 0.15 Btu per pound. This means
suction to become superheated without pro- that the temperature reduction of the liquid will
ducing useful cooling have already been pointed be approximately 62% (0.15/0.24) of the rise in
out. Obviously, then, since superheating of the the temperature of the vapor, or that for each
suction vapor is unavoidable in an actual cycle, 24° F rise in the temperature of the vapor, the
whether or not a heat exchanger is used, any temperature of the liquid will be reduced 15° F.
practical means of causing the vapor to become For the heat exchanger cycle in Fig. 8-9, the
superheated in such a way that useful cooling vapor absorbs 5.71 Btu per pound in super-
results are worthwhile. Hence, the value of a heating from 20° F to 60° F. Assuming that all
heat exchanger lies in the fact that it provides a of the superheating takes place in the heat
method of superheating the vapor so that useful exchanger, the heat given up by the liquid is 5.71
cooling results. For this reason, the effect of a Btu, so that the temperature of the liquid is
heat exchanger on cycle efficiency can be reduced 23.8° F (5.71/0.24) as the liquid passes
evaluated only by comparing the heat exchanger through the heat exchanger.
cycle to one in which the vapor is superheated 8-9.The Effect of Pressure Losses Resulting
without producing useful cooling. from Friction. In overcoming friction, both
The maximum amount of heat exchange internal (within the fluid) and external (surface),
which can take place between the liquid and the the refrigerant experiences a drop in pressure
vapor in the heat exchanger depends on the while flowing through the piping, evaporator,
initial temperatures of the liquid and the vapor condenser, receiver, and through the valves and
as they enter the heat exchanger and on the passages of the compressor (Fig. 8-10).
length of time they are in contact with each A Ph diagram of an actual cycle, illustrating
other. the loss in pressure occurring in the various
The greater the difference in temperature, the parts of the system, is shown in Fig. 8-11. To
greater is the exchange of heat for any given simplify the diagram, no superheating or sub-
period of contact. Thus, the lower the vaporiz- cooling is shown and a simple saturated cycle is
ing temperature and the higher the condensing drawn in for comparison.
temperature, the greater is the possible heat Line J9'-C'represents the vaporizing process in
exchange. Theoretically, if the two fluids the evaporator during which the refrigerant
remained in contact for a sufficient length of undergoes a drop in pressure of 5.5 psi. Whereas
time, they would leave the heat exchanger at the the pressure and saturation temperature of the
same temperature. In actual practice, this is liquid-vapor mixture at the evaporator inlet is
not possible. However, the longer the two 38.58 psia and 24° F, respectively, the pressure of
fluids stay in contact, the more nearly the two the saturated vapor leaving the evaporator is
temperatures will approach one another. Since 33.08 psia, corresponding to a saturation tem-
the specific heat of the vapor is less than that of perature of 16° F. The average vaporizing tem-
the liquid, the rise in the temperature of the perature in the evaporator is 20° F, the same as
that of the saturated cycle.
* It will be shown later that some advantages As a result of the drop in pressure in the
accrue from superheating which takes place in the evaporator, the vapor leaves the evaporator at a
compressor: (1) When the suction vapor absorbs
lower pressure and saturation temperature and
heat from the cylinder walls, the cylinder wall tem-
with a greater specific volume than if no drop in
peratureis lowered somewhat and this brings about
pressure occurred.
a desirable change in the path of the compression
process. However, the change is slight and is The refrigerating effect per pound and the
difficult to evaluate. (2) When hermetic motor-
weight of refrigerant circulated per minute per
compressor assemblies are used, the suction vapor ton are approximately the same for both cycles,
should reach the compressor at a relatively low but because of the greater specific volume the
temperature in order to help cool the motor windings. volume of vapor handled by the compressor per
118 PRINCIPLES OF REFRIGERATION

V
38.58 psia
24* F (sat. temp.) *

Pressure drop through


evaporator, 5.5 psi

Average evaporating
c
temperature and
pressure
35.75 psia, 20° F
c 90° F (sat. temp.)

33.08 psia//
£L
,
Pressure drop through
158.9 psia
16* F (sat. temp.) liquid line, 77.3 psi
1 114"F (sat. temp.)

Pressure drop through


discharge valves, 8.2 psi
150.7 psia
|TTlO°F (sat. temp.)

Pressure drop through


hot gas line and

—*^\
condensers, 19.1 psi

131.6 psia
100°F (sat. temp.)

Average condensing
temperature and pressure
139 psia, 104° F
Fig. 8-10. Flow diagram illustrating the effect of pressure drop in various parts of the system. Pressure drops
are exaggerated for clarity. (Refrigerant- 2.) 1

minute per ton is greater for the cycle experi- through the suction line from the evaporator to
encing the pressure drop. Too, because of the the compressor inlet. Like pressure drop in the
lower pressure of the vapor leaving the evapora- evaporator, pressure drop in the suction line
tor, the vapor must be compressed through a causes the suction vapor to reach the compressor
greater pressure range during the compression at a lower pressure
and in an expanded condition
process, so that the horsepower per ton is also so that the volume of vapor compressed per
greater for the cycle undergoing the drop in minute per ton and the horsepower per ton are
pressure. both increased.
Line C'-C" represents the drop in pressure It is evident that the drop in pressure both in
experienced by the suction vapor in flowing the evaporator and in the suction line should be

Fig. 8-11. Ph diagram of refrig-


eration cycle illustrating the
effect of pressure losses in the
various parts of the system.
A simple saturated cycle is

drawn in for comparison.


(Refrigerant- 1 2.)

Enthalpy (Btu/lb)

Pressure drop
1. Compressor discharge valves 4. Evaporator
2. Discharge line and condenser 5. Suction line
3. Liquid line 6. Compressor suction valves
ACTUAL REFRIGERATING CYCLES 119

kept to an absolute minimum in order to obtain against the spring-loading and to force the vapor
the best possible cycle efficiency. This applies out through the discharge valves and passages
also to heat exchangers or any other auxiliary of the compressor into the discharge line.

device intended for installation in the suction Line D'-A represents the drop in pressure
line. resulting from the flow of the refrigerant through
In Fig. 8-11, the pressure drops are exagger- the discharge line and condenser. That part of
ated for clarity. Ordinarily,good evaporator line D'-A which represents the flow through the
design limits the pressure drop across the discharge line will vary with the particular case,
evaporator to 2 or 3 psi. Ideally, the suction line since the discharge line may be either quite long
should be designed so that the pressure drop is or very short, depending upon the application.
between 1 and 2 psi. In any event, the result of the pressure drop will

Fig. 8-12. Ph diagram of actual


refrigeration cycle illustrating
effects of subcooling, super-
heating, and losses in pressure.
I.
A simple saturated cycle is

drawn in for comparison.


(Refrigerant- 1 2).

Enthalpy (Btu/lb)

Pressure drop
1. Compressor discharge yalyes 4. Evaporator
2. Discharge line end condenser 5. Suction line
3. Liquid line 6. Compressor suction valves

Line C-C " represents the drop in pressure that be the same. Any drop in pressure occurring on
the suction vapor undergoes in flowing through the discharge side of the compressor (in the dis-
the suction valves and passages of the com- charge valves and passages, in the discharge line,
pressor into the cylinder. The result of the drop and in the condenser) will have the effect of
in pressure through the valves and passages on raising the discharge pressure and thereby
the suction side of the compressor is the same as increasing the work of compression and the
if the drop occurred in the suction line, and the horsepower per ton.
effect on cycle efficiency is the same. Here again, Line A-A' represents the pressure drop result-

good design requires that the drop in pressure be ing from the flow of the refrigerant through the
kept to a practical minimum. receiver tank and liquid line. Since the refrig-
Line C-D " represents the compression process erant at A' is a saturated liquid, the temperature
for the cycle undergoing the pressure drops. of the liquid must decrease as the pressure
Notice that the vapor in the cylinder is com- decreases. If the liquid is not subcooled by
pressed to a pressure considerably above the giving up heat to an external sink as its pressure
average condensing pressure. It is shown later drops, a portion of the liquid must flash into a
that this is necessary in order to force the vapor vapor in the liquid line in order to provide the
out of the cylinder through the discharge valves required cooling of the liquid. Notice that point
against the condensing pressure and against the A" lies in the region of phase-change, indicating
additional pressure occasioned by the spring- that a portion of the refrigerant is a vapor at
loading of the discharge valves. this point.
Line D"-D' represents the drop in pressure Despite the flashing of the liquid and the drop
required to force the discharge valves open in temperature coincident with the drop in
120 PRINCIPLES OF REFRIGERATION
pressure in the liquid line, the drop in pressure to 70°F in the suction line, whereas the liquid
no effect on cycle efficiency.
in the liquid line has issubcooled to 90° F by giving off heat to the
The pressure and temperature of die liquid must ambient air. Determine:
be reduced to the vaporizing condition before it
(a) The refrigerating effect per pound.
enters the evaporator in any case. The fact that Ans. 54.01 Btu/lb
a part of this takes place in the liquid line rather (b) The weight of refrigerant circulated per
than in the refrigerant control has no direct minute per ton. Ans. 3.70 lb/min/ton
effecton the efficiency of the system. It does, (c) The volume of vapor compressed per

however, reduce the capacity of both the liquid minute per ton. Ans. 2.93 cu ft/min/ton
lineand the refrigerant control. Furthermore, (d) The loss of refrigerating effect per pound
in the refrigerant control.
passage of vapor through the refrigerant control
Ans. 11.7 Btu/lb
will eventuallycause damage to the refrigerant
(c) Thequantity of superheat in the suction
control by eroding the valve needle and seat.
vapor. Ans. 4.39 Btu/lb
Ordinarily, even without the use of a heat
(/) The gain in refrigerating effect per pound
exchanger, sufficient subcooling of the liquid resulting from the liquid subcooling.
will occur in the liquid line to prevent the Ans. 4.93 Btu/lb
flashing of the liquid if the drop in pressure in (g) The adiabatic discharge temperature.
the line is not excessive. Flashing of the liquid Ans. 138.5° F
in the liquid line will usually not take place when (A) The heat of compression per pound.
the drop in the line does not exceed 5 psi. Ans. 9 Btu/lb
The effect of pressure drop in the lines and in
(0 The heat of compression per minute per
ton. Ans. 33.3 Btu/min/ton
the other parts of the system is discussed more
(J)
The work of compression per minute per
fully later in the appropriate chapters.
ton. Ans. 25.907 lb/min/ton
A Ph diagram of a typical refrigeratioircycle, (At) The theoretical horsepower per ton.
which illustrated the combined effects of pressure Ans. 0.755 hp/ton
drop, subcooling, and superheating, is compared (/)The heat rejected at the condenser per
to the PA diagram of the simple saturated cycle pound. Arts. 67 .4 Btu/lb
in Fig. 8-12. (m) The heat rejected at the condenser per ton.
Ans. 249.38 Btu/min/ton
PROBLEMS (») The coefficient of performance. Ans. 6
1. The vaporizing and condensing temperature Note: Some of the properties of the refriger-
of a Refrigerant-12 system are 40° F and 1 10° F, ant at various points in the cycle must be
respectively. The suction vapor is superheated determined from the Ph chart in Fig. 7-1.
America is today, nor do they realize the extent
to which such a society is dependent upon
mechanical refrigeration for its very existence.
It would not be possible, for instance, to pre-
serve food in sufficient quantities to feed the
growing urban population without mechanical
refrigeration.Too, many of the large buildings
which house much of the nation's business and
9 industry would become untenable in the summer
months because of the heat if they were not air
conditioned with mechanical refrigerating
equipment.
Survey In addition to the better known applications
of refrigeration, such as comfort air conditioning
of Refrigeration and the processing, freezing, storage, transpor-
tation, and display of perishable products,
Applications mechanical refrigeration is used in the processing
or manufacturing of almost every article or
commodity on the market today. The list of
processes or products made possible or improved
through the use of mechanical refrigeration is
almost endless. For example, refrigeration has
9-1. History and Scope of the Industry. In made possible the building of huge dams which
the early days of mechanical refrigeration, the
are vital to large-scale reclamation and hydro-
equipment available was bulky, expensive, and electric projects. It has made possible the con-
not too Also it was of such a nature as
efficient.
and tunnels and the sinking of
struction of roads
to require that a mechanic or operating engineer
foundation and mining shafts through and across
be on duty at all times. This limited the use of unstable ground formations. It has made
mechanical refrigeration to a few large applica- possible the production of plastics, synthetic
tions such as ice plants, meat packing plants, and
rubber, and many other new and useful mate-
large storage warehouses.
rials and products. Because of mechanical
In the span of only a few decades refrigeration refrigeration, bakers can get more loaves of
has grown into the giant and rapidly expanding bread from a barrel of flour, textile and paper
industry that it is today. This explosive growth manufacturers can speed up their machines and
came about as the result of several factors. First, get more production, and better methods of
with the development of precision manufacturing hardening steels for machine tools are available.
methods, it became possible to produce smaller, These represent only a few of the hundreds of
more equipment. This, along with the
efficient
ways in which mechanical refrigeration is now
development of "safe" refrigerants and the being used and many new uses are being found
invention of the fractional horsepower electric each year. In fact, the only thing slowing the
motor, made possible the small refrigerating growth of the refrigeration industry at the
unit which is so widely used at the present time present time is the lack of an adequate supply of
in such applications as domestic refrigerators
trained technical manpower.
and freezers, small air conditioners, and com- 9-2. Classification of Applications. For con-
mercial fixtures. Today, there are few homes or venience of study, refrigeration applications may
business establishments in the United States that be grouped into six general categories: (1)
cannot boast of one or more mechanical domestic refrigeration, (2) commercial refrigera-
refrigeration units of some sort. tion, (3) industrial refrigeration, (4) marine and
Few people outside of those directly connected transportation refrigeration, (5) comfort air con-
with the industry are aware of the significant ditioning, and (6) industrial air conditioning. It
part that refrigeration has played in the develop- will be apparent in the discussion which follows
ment of the highly technical society that that the exact limits of these areas are not
121
122 PRINCIPLES OF REFRIGERATION
as well
and that there is considerable for vessels transporting perishable cargo
precisely defined
as refrigeration for the ship's stores on
vessels of
overlapping between the several areas.
kinds.
9-3. Domestic Refrigeration. Domestic re- all
Transportation refrigeration is concerned
frigeration is rather limited in scope,
being
with refrigeration equipment as it is applied to
concerned primarily with household refrigera-
trucks, both long distance transports and
local
tors and home freezers. However, because
the
delivery, and to refrigerated railway cars. Typi-
number of units in service is quite large,
cal refrigerated truck bodies are shown in Fig.
domestic refrigeration represents a significant
11-8.
portion of the refrigeration industry.
9-7. Air Conditioning. As the name implies,
Domesticunits are usually small in size, having
condition
horsepower ratings of between ^
and J hp, and air conditioning is concerned with the
of the air in some designated area or space. This
these
are of the hermetically sealed type. Since
usually involves control not only of the space
applications are familiar to everyone, they will
temperature but also of space humidity and air
not be described further here. However, the
motion, along with the filtering and cleaning of
problems encountered in the design and main-
the
tenance of these units are discussed in appro-
air.

which follow. Air conditioning applications are of two types,


priate places in the chapters
their
Commer- either comfort or industrial, according to
9-4. Com.nercial Refrigeration.
concerned with the designing, purpose. Any air conditioning which has as its
cial refrigeration is
primary function the conditioning of air for
installation, and maintenance of refrigerated
fixtures of the type used by retail stores,
res- human comfort is called comfort air condition-
institutions for the storing, ing. Typical installations of comfort air
taurants, hotels, and
and dispensing of perish- conditioning are in homes, schools, offices,
displaying, processing,
churches, hotels, retail stores, public buildings,
able commodities of all types. Commercial
are described in more factories, automobiles, buses, trains, planes,
refrigeration fixtures
ships, etc.
detail later in this chapter.
9-5. Industrial Refrigeration. Industrial On the other hand, any air conditioning which
does not have as its primary purpose the con-
refrigeration is often confused with commercial
ditioning of air for human comfort is called
refrigeration because the division between these
industrial air conditioning. This does not
two areas is not clearly defined. As a general
necessarily mean that industrial air conditioning
rule, industrial applications are larger in
size

commercial applications and have the dis- systems cannot serve as comfort air conditioning
than
coincidentally with their primary function.
tinguishing feature of requiring an attendant on
Often this is the case, although not always so.
duty, usually a licensed operating engineer.
Typical industrial applications are ice plants,
The applications of industrial air conditioning
are almost without limit both in number
and in
large food-packing plants (meat, fish, poultry,
variety. Generally speaking, the functions
of
frozen foods, etc.), breweries, creameries, and
industrial air conditioning are to; (1)
control
industrial plants, such as oil refineries, chemical
the moisture content of hydroscopic materials;
plants, rubber plants, etc. Industrial refrigera-
(2) govern the rate of
chemical and biochemical
tion includes also those applications concerned
reactions; (3) limit the variations in the size of
with the construction industry as described in
precision manufactured articles because of ther-
Section 9-1.
9-6. Marine and Transportation Refrigera- mal expansion and contraction and (4) provide
;

essential to
Applications falling into this category clean, filtered air which is often
tion.
under commercial refrig- trouble-free operation and to the production of
could be listed partly
refrigera- quality products.
eration and partly under industrial
specialization 9-8. Food Preservation. The preservation of
tion. However, both these areas of
perishable commodities, particularly foodstuffs,
have grown to sufficient size to warrant special
is one of the most common uses
of mechanical
mention.
of course, refers to refrigeration. As such, it is a subject which
Marine refrigeration,
aboard marine vessels and includes, should be given consideration in any compre-
refrigeration
boats and hensive study of refrigeration.
for example, refrigeration for fishing
SURVEY OF REFRIGERATION APPLICATIONS 123

At the present time, food preservation is more The invention of the microscope and the sub-
important than ever before in man's history. sequent discovery of microorganisms as a major
Today's large urban populations require tremen- cause of food spoilage led to the development of
dous quantities of food, which for the most part canning in France during the time of Napoleon.
must be produced and processed in outlying With the invention of canning, man found a way
areas. Naturally, these foodstuffs must be kept to preserve food of all kinds in large quantities
in a preserved condition during transit and sub- and for indefinite periods of time. Canned foods
sequent storage until they are finally consumed. have the advantage of being entirely imperish-
This may be a matter of hours, days, weeks, able, easily processed, and convenient to handle
months, or even years in some cases. Too, many and store. Today, more food is preserved by
products, particularly fruit and vegetables, are canning than by all other methods combined.
seasonal. Since they are produced only during The one big disadvantage of canning is that
certain seasons of the year, they must be stored canned foods must be heat-sterilized, which
and preserved if they are to be made available frequently results in overcooking. Hence, al-
the year round. though canned foods often have a distinctive and
As a matter of life or death, the preservation delicious flavor all their own, they usually differ
of food has long been one of man's most pressing greatly from the original fresh product.
problems. Almost from the very beginning of The only means of preserving food in its
man's existence on earth, it became necessary original fresh state is by refrigeration. This, of
for him to find ways of preserving food during course, is the principal advantage that refrigera-
seasons of abundance in order to live through tion has over other methods of food preser-
seasons of scarcity. It is only natural, then, that vation. However, refrigeration too has its
man should discover and develop such methods disadvantages. For instance, when food is to
of food preservation as drying, smoking, pick- be preserved by refrigeration, the refrigerating
ling, and salting long before he had any know- process must begin very soon after harvesting
ledge of the causes of food spoilage. These or killing and must be continuous until the food
rather primitive methods are still widely used is finally consumed. Since this requires rela-
today, not only in backward societies where no tively expensive and bulky equipment, it is often
other means are available but also in the most both inconvenient and uneconomical.
modern societies where they serve to supplement Obviously, then, there is no one method of
the more modern methods of food preservation. food preservation which is best in all cases and
For instance, millions of pounds of dehydrated the particular method used ih any one case will
(dried) fruit, milk, eggs, fish, meat, potatoes, etc., depend upon a number of factors, such as the
are consumed in the United States each year, type of product, the length of time the product
along with huge quantities of smoked, pickled, is to be preserved, the purpose for which the

and salted products, such as ham, bacon, and product is to be used, the availability of trans-
sausage, to name only a few. However, although portation and storage equipment, etc. Very
these older methods are entirely adequate for the often it is necessary to employ several methods
preservation of certain types of food, and often simultaneously in order to obtain the desired
produce very unusual and tasty products which results.
would not otherwise be available, they nonethe- 9-9. Deterioration and Spoilage. Since the
less have inherent disadvantages which limit preservation of food is simply a matter of pre-

their usefulness. Since by their very nature they venting or retarding deterioration and spoilage
bring about severe changes in appearance, taste, regardless of the method used, a good knowledge
and odor, which in many cases are objectionable, of the causes of deterioration and spoilage is
they are not universally adaptable for the pres- a prerequisite to the study of preservation
ervation of all types of food products. Further- methods.
more, the keeping qualities of food preserved by It should be recognized at the outset that
such methods are definitely limited as to time. there are degrees of quality and that all perish-
Therefore, where a product is to be preserved able foods through various stages of
pass
indefinitely or for a long period of time, some deterioration before becoming unfit for con-
other means of preservation must be utilized. sumption. In most cases, the objective in the
124 PRINCIPLES OF REFRIGERATION

preservation of food not only to preserve the


is either eliminated or effectively controlled if the

foodstuff in an edible condition but also to foodstuff is to be adequately preserved.


9-10. Enzymes. Enzymes are complex, pro-
preserve it as nearly as possible at the peak of its
quality with respect to appearance, odor, taste, tein-like, chemical substances. Not yet fully

and vitamin content. Except for a few processed understood, they are probably best described
foods, this usually means maintaining the food- as chemical catalytic agents which are capable
stuff as nearly as possible in its original fresh of bringing about chemical changes in organic
state. materials. There are many different kinds of
to cause a detect-
Any deterioration sufficient enzymes and each one is specialized in that it
able change in the appearance, odor, or taste of produces only one specific chemical' reaction.
fresh foods immediately reduces the commercial In general, enzymes are identified either by the
value of the product and thereby represents an substance upon which they act or by the result
economic loss. Consider, for example, wilted of their action. For instance, the enzyme,
vegetables or overripe Although their
fruit. lactase, isso known because it acts to convert
edibility is little impaired, an undesirable change lactose (milk sugar) to lactic acid. This par-

in their appearance has been brought about ticular process is called lactic acid fermentation
and is the one principally responsible for the
which usually requires that they be disposed of
at a reduced price. Too, since they are well on "souring" of milk. Enzymes associated with
their way to eventual spoilage, their keeping the various types of fermentation are sometimes
reduced and they must be
qualities are greatly called ferments.
immediately or become Essential in the chemistry of all living pro-
consumed or processed
a total loss. cesses, enzymes are normally present in all

For obvious reasons, maintaining the vitamin organic materials (the cell tissue of all plants and
content at the highest possible level is always an animals, both living and dead). They are manu-
important factor in the processing and/or pres- factured by all living cells to help carry on the
ervation of all food products. In fact, many various living activities of the cell, such as res-
food processors, such as bakers and dairymen, piration, digestion, growth, and reproduction,

are now adding vitamins to their product to and they play an important part in such things
replace those which are lost during processing. as the sprouting of seeds, the growth of plants

Fresh vegetables, fruit, and fruit juices are some and animals, the ripening of fruit, and the
of the food products which suffer heavy losses in digestive processes of animals, including man.

vitamin content very quickly if they are not However, enzymes are catabolic as well as ana-
handled and protected properly. Although the bolic. That is, they act to destroy dead cell tissue
loss of vitamin content is not something which as well as to maintain live cell tissue. In fact,
in itself is apparent, in many fresh foods it is enzymes are the agents primarily responsible
for the decay and decomposition of all organic
usually accompanied by recognizable changes in
appearance, odor, or taste, such as, for instance, materials, as, for example, the putrification of

wilting in leafy, green vegetables. meat and fish and the rotting of fruit and
For the most part, the deterioration and even- vegetables.

tual spoilage of perishable food are caused by a Whether their action is catabolic or anabolic,
series of complex chemical changes which take enzymes are nearly always destructive to perish-
place in the foodstuff after harvesting or killing. able foods. Therefore, except in those few
These chemical changes are brought about by special cases where fermentation or putrification
both internal and external agents. The former is a part of the processing, enzymic action must

are the natural enzymes which are inherent in all be either eliminated entirely or severely inhibited
organic materials, whereas the latter are micro- if the product is to be preserved in good con-

organisms which grow in and on the surface of dition. Fortunately, enzymes are sensitive to

the foodstuff. Although either agent alone is the conditions of the surrounding media, par-
capable of bringing about the total destruction ticularly with regard to the temperature and the

of a food product, both agents are involved in degree of acidity or alkalinity, which provides a
most cases of food spoilage. In any event, the practical means of controlling enzymic activity.
activity of both of these spoilage agents must be Enzymes are completely destroyed by high
SURVEY OF REFRIGERATION APPLICATIONS 125

temperatures that alter the composition of the for consumption. Too, some microorganisms
organic material in which they exist. Since most secrete poisonous substances (toxins)which are
enzymes are eliminated at temperatures above extremely dangerous to health, causing poison-
160° F, cooking a food substance completely ing, disease, and often death.
destroys the enzymes contained therein. On the On the other hand, microorganisms have
other hand, enzymes are very resistant to low many useful and necessary functions. As a
temperatures and their activity may continue at matter of it were not for the work of
fact, if
a slow rate even at temperatures below 0° F. microorganisms, life of any kind would not be
However, it is a well-known fact that the rate of possible. Since decay and decomposition of all
chemical reaction decreases as the temperature dead animal tissue are essential to make space
decreases. Hence, although the enzymes are not available for new life and growth, the decaying
destroyed, their activity is greatly reduced at low action of microorganisms is indispensable to the
temperatures, particularly temperatures below life cycle.
the freezing point of water. Of all living things, only green plants (those
Enzymic action is greatest in the presence of containing chlorophyll) are capable of using
free oxygen (as in the air) and decreases as the inorganic materials as food for building their
oxygen supply diminishes. cell tissue. Through a process called photo-
With regard to the degree of acidity or alka- synthesis, green plants are able to utilize the
linity, some enzymes require acid surroundings, radiant energy of the sun to combine carbon
whereas others prefer neutral or alkaline en- dioxide from the air with water and mineral
vironments. Those requiring acidity are de- saltsfrom the soil and thereby manufacture
stroyed by and those requiring
alkalinity from inorganic materials the organic compounds
alkalinity are likewise destroyed by acidity. which make up their cell tissue.
Although an organic substance can be com- Conversely, all animals and all plants without
pletely destroyed and decomposed solely by the chlorophyll (fungi) require organic materials
action of its own natural enzymes, a process (those containing carbon) for food to carry on
known as autolysis (self-destruction), this sel- Consequently, they must of
their life activities.
dom occurs. More often, the natural enzymes upon the cell tissue of other plants
necessity feed
are aided in their destructive action by enzymes and animals (either living or dead) and are,
secreted by microorganisms. therefore, dependent either directly or indirectly
9-11. Microorganisms. The term micro- on green plants as a source of the organic
organism is used to cover a whole group of materials they need for lifeand growth.
minute plants and animals of microscopic and It is evident, then, that should the supply of
submicroscopic size, of which only the following inorganic materials in the soil, which serve as
three are of particular interest in the study of food for green plants, ever become exhausted,
food preservation: (1) bacteria, (2) yeasts, and all life would soon disappear from the earth.

(3) molds. These tiny organisms are found in This is not likely to happen, however, since

large numbers everywhere in the air, in the microorganisms, as a part of their own living
ground, in water, in and on the bodies of plants process, are continuously replenishing the supply
and animals, and in every other place where con- of inorganic materials in the soil.
ditions are such that living organisms can survive. With the exception of a few types of soil
Because they secrete enzymes which attack bacteria, all microorganisms need organic
the organic materials upon which they grow, materials as food to carry on the living process.
microorganisms are agents of fermentation, In most cases, they obtain these materials by
purification, and decay. As such, they are both decomposing animal wastes and the tissue of
beneficial and harmful to mankind. Then- dead animals and plants. In the process of
growth in and on the surface of perishable foods decomposition, the complex organic compounds
causes complex chemical changes in the food which make up the tissue of animals and plants
substance which usually results in undesirable are broken down step by step and are eventually
and appearance of
alterations in the taste, odor, reduced to simple inorganic materials which are
the food and which, allowed to continue for
if returned to the soil to be used as food by the
any length of time, will render the food unfit green plants.
126 PRINCIPLES OF REFRIGERATION

In addition to the important part they play in The rate at which bacteria and other micro-
the "food chain" by helping to keep essential organisms grow and reproduce depends upon
materials in circulation, microorganisms ale such environmental conditions as temperature,
necessary in the processing of certain fermented light, and the degree of acidity or alkalinity, and

foods and other commodities. For example, upon the availability of oxygen, moisture, and
bacteria are responsible for the lactic acid fer- an adequate supply of soluble food. However,
mentation required in the processing of pickles, there are many species of bacteria and they
olives,cocoa, coffee, sauerkraut, ensilage, and differ greatly both in their choice of environment
certain sour milk products, such as butter, and in the effect they have on their environment.
cheese, buttermilk, yogurt, etc., and for the Like the higher forms of plant life, all species of
acetic acid fermentation necessary in the pro- bacteria are not equally hardy with respect to
duction of vinegar from various alcohols. surviving adverse conditions of environment,
Bacterial action is useful also in the processing nor do all species thrive equally well under the
of certain other commodities such as leather, same environmental conditions. Some species
linen, hemp, and tobacco, and in the treatment prefer conditions which are entirely fatal to
of industrial wastes of organic composition. others. Too, some bacteria are spore-formers.

Yeasts, because of their ability to produce The spore is formed within the bacteria cell and
alcoholic fermentation, are of immeasurable is protected by a heavy covering or wall. In the

value to the brewing and wine-making industries spore state, which is actually a resting or
and to the production of alcohols of all kinds. dormant phase of the organism, bacteria are
Too, everyone is aware of the importance of extremely resistant to unfavorable conditions of
yeast in the baking industry. environment and can survive in this state almost
The chief commercial uses of molds are in the indefinitely. The spore will usually germinate

processing of certain types of cheeses and, more whenever conditions become favorable for the
important, in the production of antibiotics, such organism to carry on its living activities.
as penicillin and aureomycin. Most bacteria are saprophytes. That is, they
Despite their many useful and necessary are "free living" and feed only on animal wastes
functions, the fact remains that microorganisms and on the dead tissue of animals and plants.
are destructive to perishable foods. Hence, Some, however, are parasites and require a
their activity, like that of the natural enzymes, living host. Most pathogenic bacteria (those

must be effectively controlled if deterioration and causing infection and disease) are of the para-
spoilage of the food substance are to be avoided. sitic type. In the absence of a living host, some

Since each type of microorganism differs parasitic bacteria can live as saprophytes. Like-

somewhat in both nature and behavior, it is wise, some saprophytes can live as parasites

worthwhile to examine each type separately. when the need arises.


9-12. Bacteria. Bacteria are a very simple Since bacteria are not able to digest insoluble
form of plant life, being made up of one single food substances, they require food in a soluble
Reproduction is accomplished by cell
living cell. form. For this reason, most bacteria secrete
division. On
reaching maturity, the bacterium enzymes which are capable of rendering in-
divides into two separate and equal cells, each soluble compounds into a soluble state, thereby
of which in turn grows to maturity and divides making these materials available to the bacteria

into two cells. Bacteria grow and reproduce at as food. The deterioration of perishable foods
an enormous rate. Under ideal conditions, a by bacteria growth is a direct result of the action

bacterium can grow into maturity and repro- of these bacterial enzymes.
duce in as little as 20 to 30 min. At this rate a Bacteria, like all other living things, require
single bacterium is capable of producing as moisture as well as food to carry on their life
many as 34,000,000,000,000 descendants in a activities. As in other things, bacteria vary
24-hr period. Fortunately, however, the life considerably in their ability to resist drought.
cycle of bacteria is relatively short, being a Although most species are readily destroyed by
matter of minutes or hours, so that even under drying and will succumb within a few hours, the
ideal conditions they cannot multiply at any- more hardy species are able to resist drought
where near this rate. for several days. Bacterial spores can withstand
SURVEY OF REFRIGERATION APPLICATIONS 127

The Growth of Bacteria in Milk in Various Periods


Time, hours
Temp., °F 24 48 96 168

32 2,400 2,100 1,850 1,400


39 2,500 3,600 218,000 4,200,000
46 3,100 12,000 1,480,000
50 11,600 540,000
60 180,000 28,000,000
86 1,400,000,000

Fig. 9-1. From ASRE Data Book, Applications Volume, 1956-57. Reproduced by
permission of the American
Society of Heating, Refrigerating, and Air-Conditioning Engineers.

drought almost indefinitely, but will remain For each species of bacteria there is an opti-
dormant in the absence of moisture. mum temperature at which the bacteria will
In their need for oxygen, bacteria fall into two grow at the highest rate.
Too, for each species
groups (1) those which require free oxygen (air)
: there is a maximum and
a minimum tempera-
and (2) those which can
exist without free oxy- ture which will permit growth. At temperatures
gen. Some although having a prefer-
species, above the maximum, the bacteria are destroyed.
ence for one condition or the other, can live At temperatures below the minimum, the bac-
in the presence of free oxygen or in the absence teria are rendered inactive or dormant. The
of it. Those bacteria living without free oxygen optimum temperature for most saprophytes is
obtain the needed oxygen through chemical usually between 75° F and 85° F, whereas the
reaction which reduces one compound while optimum temperature for parasites is around
oxidizing another. Decomposition which occurs 99° F or 100° F. A few species grow best at
in the presence of freeoxygen is known as decay, temperatures near the boiling point of water,
whereas decomposition which takes place in the whereas a few other types thrive best at tempera-
absence of free oxygen is called putrification. tures near the freezing point. However, most
One of the products of putrification is hydrogen species are either killed off or severely inhibited
sulfide, a foul-smelling gas which is frequently at these temperatures. The effect of temperature
noted arising from decomposing animal on the growth rate of bacteria is illustrated by
carcasses. the chart in Fig. 9-1 which shows the growth
Bacteria are very sensitive to acidity or alka- rate of bacteria in milk at various temperatures.
linity and cannot survive in an either highly acid In general, the growth rate of bacteria is con-
or highly alkaline environment. Most bacteria siderably reduced by lowering the temperature.
require either neutral or slightly alkaline sur- 9-13. Yeasts. Yeasts are simple, one-cell plants
roundings, although some species prefer slightly of the fungus family. Of microscopic size,
acid conditions. Because bacteria prefer neutral yeast cells are somewhat larger and more com-
or slightly alkaline surroundings, nonacid plex than the bacteria cells. Although a few
vegetables are especially subject to bacterial yeasts reproduce by fission or by sexual process,
attack. reproduction usually by budding. Starting as
is
Light, particularly direct sunlight, is harmful a small protrusion of the mature cell, the bud
to all bacteria. Whereas visible light only enlarges and from the mother
finally separates
inhibits their growth, ultraviolet light is actually cell. Under ideal conditions,
budding is fre-
fatal to bacteria. Since light rays, ultraviolet or quently so rapid that new buds are formed before
otherwise, have no power of
penetration, they separation occurs so that yeast clusters are
are effective only in controlling surface bacteria. formed.
However, ultraviolet radiation (usually from Like bacteria, yeasts are agents of fermenta-
direct sunlight), when combined with drying, tion and decay. They secrete enzymes that bring
provides an excellent means of controlling about chemical changes in the food upon which
bacteria growth. they grow. Yeasts are noted for their ability
128 PRINCIPLES OF REFRIGERATION

to transform sugars into alcohol and carbon Mold spores are actually seeds and, under the
dioxide. Although destructive to fresh foods, proper conditions, will germinate and produce
particularly fruits and berries and their juices, mold growth on any food substance with which
the alcoholic fermentation produced by yeasts they come in contact. Since they are carried
is essential to the baking, brewing, and wine- about by air currents, mold spores are found
making industries. almost everywhere and are particularly abundant
Yeasts are spore-formers, with as many as in the air.
eight spores being formed within a single yeast Although molds are less resistant to high
cell. Yeasts are widespread in nature and yeast temperatures than are bacteria, they are more
spores are invariably found in the air and on the tolerant to low temperatures, growing freely at

skin of fruit and berries, for which they have a temperatures close to the freezing point of water.
particular affinity. They usually spend the Mold growth is inhibited by temperatures below
winter in the soil and are carried to the new fruit 32° F, more from the lack of free moisture than
in the spring by insects or by the wind. from the effect of low temperature. All mold
Like bacteria, yeasts require air, food, and growth ceases at temperatures of 10° F and
moisture for growth, and are sensitive to tem- below.
perature and the degree of acidity or alkalinity Molds flourish in dark, damp surroundings,
in the environment. For the most part, yeasts particularly in still air. An
abundant supply of
prefer moderate temperatures and slight acidity. oxygen is essential to mold growth, although a
In general, yeasts are not as resistant to unfavor- very few species can grow in the absence of
able conditions as are bacteria, although they oxygen. Conditions inside cold-storage rooms
can grow in acid surroundings which inhibit are often ideal for mold growth, especially in
most bacteria. Yeast spores, like those of bac- the wintertime. This problem can be overcome
teria, are extremely hardy and can survive for somewhat by maintaining good air circulation
long periods under adverse conditions. in the storage room, by the use of germicidal
9-14. Molds. Molds, like yeasts, are simple paints, and ultraviolet radiation, and by frequent

plants of the fungi family. However, they are scrubbing.


much more complex in structure than either Unlike bacteria, molds can thrive on foods
bacteria or yeasts. Whereas the individual containing relatively large amounts of sugars or
bacteria or yeast plants consist of one single acids. They are often found growing on acid
an individual mold plant is made up of a
cell, fruits and on the surface of pickling vats, and
number of cells which are positioned end to end are the most common cause of spoilage in
toform long, branching, threadlike fibers called apples and citrus fruits.

hypha. The network which is formed by a mass 9-15. Control of Spoilage Agents. Despite
of these threadlike fibersis called the mycelium complications arising from the differences in the
and is easily visible to the naked eye. The reaction of the various types of spoilage agents
hyphae of the mold plant are of two general to specific conditions in the environment, con-
types. Some are vegetative fibers which grow trolling these conditions provides the only means
under the surface and act as roots to gather food of controlling these spoilage agents. Thus, all
for the plant, whereas others, called aerial methods of food preservation must of necessity
hyphae, grow on the surface and produce the involve manipulation of the environment in and
fruiting bodies. around the preserved product in order to pro-
Molds reproduce by spore formation. The duce one or more conditions unfavorable to the
spores develop in three different ways, depending continued activity of the spoilage agents. When
on the type of mold: (1) as round clusters the product is to be preserved for any length of
within the fibrous hyphae network, (2) as a mass time, the unfavorable conditions produced must
enclosed in a sac and attached to the end of be of sufficient severity to eliminate the spoilage
aerial hyphae, and (3) as chainlike clusters on agents entirely or at least render them ineffective
the end of aerial hyphae. In any case, a single or dormant.
mold plant is capable of producing thousands of All types of spoilage agents are destroyed when
spores which break free from the mother plant subjected to high temperatures over a period of
and float away with the slightest air motion. time. This principle is used in the preservation
SURVEY OF REFRIGERATION APPLICATIONS 129

of food by canning. The temperature of the the drying effect of the smoke and partially

product is raised to a level fatal to all spoilage by antiseptics (primarily creosote) which are
agents and is maintained at this level until they absorbed from the smoke.
are all destroyed. The product is then sealed Too, some products are "cured" with sugar
in sterilized, air-tight containers to prevent or salt which act as preservatives in that they
recontamination. A product so processed will create conditions unfavorable to the activity of
remain in a preserved state indefinitely. spoilage agents. Some other frequently used
The exposure time required for the destruc- preservatives are vinegar, borax, saltpeter, bon-
tion of all spoilage agents depends upon the zoate of soda, and various spices. A
few of the
temperature level. The higher the temperature products preserved in this manner are sugar-
level,the shorter is the exposure period required. cured hams, salt pork, spiced fruits, certain
In this regard, moist heat is more effective than beverages, jellies, jams, and preserves.
dry heat because of its greater penetrating 9-16. Preservation by Refrigeration. The
powers. When moist heat is used, the tempera- preservation of perishables by refrigeration in-
ture level required is lower and the processing volves the use of low temperature as a means of
period is shorter. Enzymes and all living micro- eliminating or retarding the activity of spoilage
organisms are destroyed when exposed to the agents. Although low temperatures are not as

temperature of boiling water for approximately effective in bringing about the destruction of
five minutes, but the more resistant bacteria spoilage agents as are high temperatures, the
spores maysurvive at this condition for several storage of perishables at low temperatures
hours before succumbing. For this reason, some greatly reduces the activity of both enzymes and

food products, particularly meats and nonacid microorganisms and thereby provides a prac-
require long processing periods tical means of preserving perishables in their
vegetables,
which frequently result in overcooking of the original fresh state for varying periods of time.
The degree of low temperature required for
product. These products are usually processed
under pressure so that the processing tempera- adequate preservation varies with the type of
ture is increased and the processing time product stored and with the length of time the
shortened. product is to be kept in storage.
Another method of curtailing the activity of For purposes of preservation, food products
spoilage agents is to deprive them of the mois- can be grouped into two general categories (1) :

ture and/or food which is necessary for then- those which are alive at the time of distribution
continued activity. Both enzymes and micro- and storage and (2) those which are not. Non-
organisms require moisture to carry on their living food substances, such as meat, poultry,

activities. Hence, removal of the free moisture and fish, are much more susceptible to micro-

from a product will severely limit their activities. bial contamination and spoilage than are living
The process of moisture removal is called drying food substances, and they usually require more
(dehydration) and is one of the oldest methods of stringent preservation methods.

preserving foods. Drying is accomplished either With nonliving food substances, the problem
naturally in the sun and air or artificially in of preservation is one of protecting dead tissue
ovens. Dried products which are stored in a from all the forces of putrification and decay,

cool, dry place will remain in good condition both enzymic and microbial. In the case of
for long periods. living food substances, such as fruit and vege-

Pickling is essentially a fermentation process, tables, the fact of life itself affords considerable

the exhaustion of the protection against microbial invasion, and the


the end result of which is

substances which serve as food for yeasts and preservation problem one of keeping
is chiefly

bacteria. The product to be preserved by pick- the food substance alive while at the same time
ling is immersed in a salt brine solution and retarding natural enzymic activity in order to
fermentation allowed to take place, during
is slow the rate of maturation or ripening.
which the sugars contained in the food product Vegetables and fruit are as much alive after
are converted to lactic acid, primarily through harvesting as they are during the growing period.

the action of lactic acid bacteria. Previous to harvesting they receive a continuous
Smoked products are preserved partially by supply of food substances from the growing
130 PRINCIPLES OF REFRIGERATION
plant, some of which is stored in the vegetable its freezing point, whereas frozen storage re-
or fruit. After harvesting, when the vegetable or quires freezing of the product and storage at
fruit is cut off fromnormal supply of food,
its some temperature between 10° F and — 10° F,
the living processes continue through utilization with 0° F being the temperature most frequently
of the previously stored food substances. This employed.
causes the vegetable or fruit to undergo Short-term or temporary storage is usually
changes which will eventually result in deteriora- associated with retail establishments where
tion and complete decay of the product. The rapid turnover of the product is normally ex-
primary purpose of placing such products under pected. Depending upon the product, short-
refrigeration is to slow the living processes by term storage periods range from one or two days
retarding enzymic activity, thereby keeping the in some cases to a week or more in others, but
product in a preserved condition for a longer seldom for more than fifteen days.
period. Long-term storage is usually carried out by
Animal products (nonliving food substances) wholesalers and commercial storage warehouses.
are also affected by the activity of natural Again, the storage period depends on the type
enzymes. The enzymes causing the most of product stored and upon the condition of the
and
trouble are those which catalyze hydrolysis product on entering storage. Maximum storage
oxidation and are associated with the break- periods for long-term storage range from seven
down of animal fats. The principal factor to ten days for some sensitive products, such as
limiting the storage life of animal products, in ripe tomatoes, cantaloupes, and and up
broccoli,
both the frozen and unfrozen states, is rancidity. to six or eight months more durable
for the
Rancidity is caused by oxidation of animal fats products, such as onions and some smoked
and, since some types of animal fats are less meats. When perishable foods are to be stored
stable than others, the storage life of animal for longer periods, they should be frozen and
products depends in part on fat composition. placed in frozen storage. Some fresh foods,
For example, because of the relative stability of however, such as tomatoes, are damaged by the
beef fat, the storage life of beef is considerably freezing process and therefore cannot be success-
greater than that of pork or fish whose fatty fully frozen. When such products are to be
tissues are much less stable. preserved for long periods, some other method
Oxidation and hydrolysis are controlled by of preservation should be used.
placing the product under refrigeration so that 9-18. Storage Conditions. The optimum
the activity of the natural enzymes is reduced. storage conditions for a product held in either
The rate of oxidation can be further reduced in short- or long-term storage depends upon the
the case of animal products by packaging the nature of the individual product, the length of
products in tight-fitting, gas-proof containers time the product is to be held in storage, and
which prevent air (oxygen) from reaching the whether the product is packaged or unpackaged.
surface of the product. The packaging of fruit In general, the conditions required for short-
and vegetables in gas-proof containers, when term storage are more flexible than those
stored in the unfrozen state, not practical.
is required for long-term storage and, ordinarily,
Since these products are alive, packaging in gas- higher storage temperatures are permissible.
proof containers will cause suffocation and Recommended storage conditions for both
death. A dead fruit or vegetable decays very short- and long-term storage and the approxi-
quickly. mate storage life for various products are listed
As a general rule, the lower the storage tem- in Tables 10-10 through 10-13, along with other
perature, the longer is the storage life of the product data. These data are the result of both
product. experiment and experience and should be
9-17. Refrigerated Storage. Refrigerated followed closely, particularly for long-term
storage may be divided into three general cate- storage, if product quality is to be maintained
gories: (1) short-term or temporary storage, at a high level during the storage period.
(2) long-term storage, and (3) frozen storage. 9-19. Storage Temperature. Examination of
For short- and long-term storage, the product is show that the optimum storage
the tables will
chilled and stored at some temperature above temperature for most products is one slightly
SURVEY OF REFRIGERATION APPLICATIONS 131

above the freezing point of the product. There pressure of the surrounding air, the rate of
are, however, notable exceptions. moisture loss from the product being propor-
Although the effect of incorrect storage tem- tional to the difference in the vapor pres-
peratures generally is to lower product quality sures and to the amount of exposed product
and shorten storage life, some fruits and vege- surface.

tables are particularly sensitive to storage tem- The difference in vapor pressure between the
perature and are susceptible to so-called cold product and the air is primarily a function of
storage diseases when stored at temperatures the relative humidity and the velocity of the air
above or below their critical storage tempera- in the storage space. In general, the lower the
tures. For example, citrus fruits frequently relative humidity and the higher the air velocity,
develop rind pitting when stored at relatively the greater will be the vapor pressure differential
high temperatures. On the other hand, they are and the greater the rate of moisture loss from
subject to scald (browning of the rind) and the product. Conversely, minimum moisture
watery breakdown when stored at temperatures losses are experienced when the humidity in the
below their critical temperature. Bananas suffer storage space maintained at a high level with
is

peel injury when stored below 56° F, whereas low air velocity. Hence, 100% relative humidity

celery undergoes soggy breakdown when stored and stagnant air are ideal conditions for pre-

at temperatures above 34° F. Although onions venting dehydration of the stored product.
tend to sprout at temperatures above 32° F, Unfortunately, these conditions are also con-
Irish potatoes tend to become sweet at storage ducive to rapid mold growth and the formation
temperatures below 40° F. Squash, green beans, of slime (bacterial) on meats. Too, good
and peppers develop pits on their surface when circulation of the air in the refrigerated space

stored at or near 32° F. Too, whereas the best and around the product is necessary for ade-

storage temperature for most varieties of apples quate refrigeration of the product. For these
is 30° F to 32° F, some varieties are subject to reasons, space humidity must be maintained at
soft scald and soggy breakdown when stored somewhat less than 100% and air velocities

below 35° F. Others develop brown core at must be sufficient to provide adequate air
temperatures below 36° F, and still others circulation. The relative humidities and air
develop internal browning when stored below velocities recommended for the storage of
40° F. various products are listed in Tables 10-10

9-20. Humidity and Air Motion. The through 10-13.


storage of all perishables in their natural state When the product is- stored in vapor-proof
(unpackaged) requires close control not only of containers, space humidity and air velocity are

the space temperature but also of space humid- not critical. Some products, such as dried
ity and air motion. One of the chief causes of fruits, tend to be hydroscopic and therefore

the deterioration of unpackaged fresh foods, require storage at low relative humidities.

such as meat, poultry, fish, fruit, vegetables, 9-21. Mixed Storage. Although the main-

cheese, eggs, etc., is the loss of moisture from tenance of optimum storage conditions requires
the surface of the product by evaporation into separate storage facilities for most products,
not usually economically There-
the surrounding air. This process is known as this is feasible.

and
desiccation or dehydration. In fruit vege- fore, except when large quantities of product
accompanied by shriveling are involved, practical considerations often
tables, desiccation is
and wilting and the product undergoes a demand that a number of refrigerated products
considerable loss in both weight and vitamin be placed in common storage. Naturally, the

content. In meats, cheese, etc., desiccation difference in the storage conditions required by
causes discoloration, shrinkage) and heavy the various products raises a problem with
trim losses. It also increases the rate of oxida- regard to the conditions to be maintained in a
tion. Eggs lose moisture through the porous space designed for common storage.
shell, with a resulting loss of weight and general As a general rule, storage conditions in such
downgrading of the egg. spaces represent a compromise and usually
Desiccation will occur whenever the vapor prescribe a storage temperature somewhat above
pressure of the product is greater than the vapor the optimum for some of the products held in
132 PRINCIPLES OF REFRIGERATION
mixed storage. The higher storage tempera- ripening continue after harvesting, vegetables
tures are used in mixed storage in order to and fruit intended for storage should be
minimize the chances of damaging the more harvested before they are fully mature. The
sensitive products which are subject to the storage life of fully mature or damaged fruit
aforementioned "cold storage diseases" when and vegetables is extremely short even under
stored at temperatures below their critical the best storage conditions, and such products
temperature. should be sent directly to market to avoid
Although higher storage temperatures tend excessive losses.
to shorten the storage life of some of the products Since" a food product begins to deteriorate
held in mixed storage, this isnot ordinarily a very quickly after harvesting or killing, it is
serious problem when the products are stored imperative that preservation measures be taken
only for short periods as in temporary immediately. To assure maximum storage life
storage. with minimum loss of quality, the product should
For long-term storage, most of the larger be chilled to the storage temperature as soon
wholesale and commercial storage warehouses as possible after harvesting or killing. When
have a number of separate storage spaces products are to be shipped over long distances
available. General practice in such cases is to to storage, they should be precooled and shipped
group the various products for storage, and by refrigerated transport.
only those products requiring approximately 9-23. Product Chilling. Product chilling is
the same storage conditions are placed together distinguished from product storage in that the
in common storage. product enters the chilling room at a high
Another problem associated with mixed temperature (usually either harvesting or killing
storage is that of odor and flavor absorption. temperature) and is chilled as rapidly as possible
Some products absorb and/or give off odors to the storage temperature, whereupon it is
while in storage. Care should be taken not to normally removed from the chilling room and
store such products together even for short placed in a holding cooler for storage. The
periods. Dairy products in particular are very handling of the product during the chilling
sensitive with regard to absorbing odors and period has a marked influence on the ultimate
flavors from other products held in mixed quality and storage life of the product.
storage. On the other hand, potatoes are The recommended conditions for product
probably the worst offenders in imparting off- chilling rooms are given in Tables 10-10 through
flavors to other products in storage and should 10-13. Before the hot product is loaded into
never be stored with fruit, eggs, dairy products, the chilling room, the chilling room temperature
or nuts. should be at the "chill finish" temperature.
9-22. Product Condition on Entering Stor- During loading and during the early part of the
age. One of the principal factors determining chilling period, the temperature and vapor
the storage life of a refrigerated product is the pressure differential between the product and
condition of the product on entering storage. the chill room air will be quite large and the
It must be recognized that refrigeration merely product will give off heat and moisture at a
arrests or retards the natural processes of high rate. At this time, the temperature and
deteriorationand therefore cannot restore to humidity in the chill room will rise to a peak as
good condition a product which has already indicated by the "chill start" conditions in the
deteriorated. Neither can
it make a high tables.* At the end of the cycle, the chill room
quality product out of one of initial poor temperature will again drop to the "chill finish"
quality. Hence, only vegetables and fruit in conditions. It is very important that the
good condition should be accepted for storage. refrigerating equipment have sufficient capacity
Those that have been bruised or otherwise
* The temperatures listed in the tables as chill
damaged, particularly if the skin has been
start temperatures are average values and are in-
broken, have lost much of their natural protec-
tended for use in selecting the refrigerating equip-
tion against microbial invasion and are there-
ment. Actual temperatures in the chilling room
fore subject to rapid spoilage by these agents. during the peak chilling period are usually 3° F to
Too, as a general rule, since maturation and 4° F higher than those listed.
.

SURVEY OF REFRIGERATION APPLICATIONS 133

to prevent the chill room temperature from reason, eggs are sometimes dipped in a light
rising excessively during thepeak chilling period. mineral oil before chilling and storage. Too,
9-24. Relative Humidity and Air Velocity poultry, fish, and some vegetables are often
in Chill Rooms. The importance of relative packed in ice for chilling and storage. When
humidity in chilling rooms depends upon the products packed in ice are placed in refrigerated
product being chilled, particularly upon whether storage, the slowly melting ice keeps the surface
the product is packaged or not. Naturally, of the product moist and prevents excessive
when the product is chilled in vapor-proof dehydration.
containers, the humidity in the chilling room 9-25. Freezing and Frozen Storage. When a
is relatively unimportant. However, during product is to be preserved in its original fresh

loading and during the initial stages of chilling, long periods, it is usually
state for relatively
chilling room humidity will be high if the con- frozen and stored at approximately 0° F or
tainers are wet, but will drop rapidly once the below. The list of food products commonly
free moisture has been evaporated. frozen includes not only those which are
Products chilled in their natural state (un- preserved in their fresh state, such as vegetables,
packaged) lose moisture very rapidly, often fruit, fruit juices, berries, meat, poultry, sea

producing fog in the chilling room during the foods, and eggs (not in shell), but also many
early stages of chilling when the product tem- prepared foods, such as breads, pastries, ice
perature and vapor pressure are high. Rapid cream, and a wide variety of specially prepared
chilling and high air velocity are desirable and precooked food products, including full

during time so that the temperature and


this dinners.
vapor pressure of the product are lowered as The factors governing the ultimate quality
quickly as possible in order to avoid excessive and storage life of any frozen product are:
moisture loss and shrinkage. High air velocity
1 The nature and composition of the product
isneeded also in order to carry away the vapor
to be frozen
and thereby prevent condensation of moisture
2. The care used in selecting, handling, and
on the surface of the product.
preparing the product for freezing
Although high air velocity tends to increase
3. The freezing method
the rate of evaporation of moisture from the
4. The storage conditions.
product, it also greatly accelerates the chilling
rate and a more rapid reduction in
results in Only high quality products in good condition
product temperature and vapor pressure. Since should be frozen. With vegetables and fruit,
the reduction in vapor pressure resulting from selecting the proper variety for freezing is very
the higher chilling rate more than offsets the important. Some varieties are not suitable for
increase in the rate of evaporation occasioned freezing and will result in a low quality product
by the higher air velocity, the net effect of the or in one with limited keeping qualities.
higher air velocity during the early stages of Vegetables and fruit to be frozen should be
chilling is to reduce the over-all loss of moisture harvested at the peak of maturity and should be
from the product. However, during the final processed and frozen as quickly as possible
stages of chilling, when the temperature and after harvesting to avoid undesirable chemical

vapor pressure of the product are considerably changes through enzymic and microbial action.
lower, the effect of high air velocity in the chill- Both vegetables and fruit require considerable
ing room is to increase the rate of moisture loss processing before freezing. After cleaning and
from the product. Therefore, the air velocity washing to remove foreign materials leaves, —
in the chilling room should be reduced during dirt, insects, juices, etc. —
from their surfaces,
the final stages of chilling. vegetables are "blanched" in hot water or
As a general rule, the humidity should be steam at 212° F in order to destroy the natural
kept at a high level when products subject to enzymes. It will be remembered that enzymes
dehydration are being chilled. Some extremely are not destroyed by low temperature and,
sensitive products, such as poultry and fish, although greatly reduced, their activity con-
are frequently chilled in ice slush to reduce tinues at a slow rate even in food stored at 0° F
moisture losses during chilling. For the same and below. Hence, blanching, which destroys
134 PRINCIPLES OF REFRIGERATION

Fig. 9-2. Walk-in installation. Suspended blast


freezer provides high-velocity air for fast freezing,
saving valuable floor space in small areas. (Courtesy
Carrier Corporation.)

Fig. 9-3. Suspended blast free-


zer applied to reach-in cabinet
distributes blast air through
shelves. (Courtesy Carrier
Corporation.)

Fig. 9-4. Freezing in one room


and storage in another is

accomplished by single, floor-


mounted blast freezers. (Cour-
tesy Carrier Corporation.)
SURVEY OF REFRIGERATION APPLICATIONS 135

most of the enzymes, greatly increases the it to freeze slowly, usually in still air. The tem-
storage life of frozen vegetables. The time perature maintained in sharp freezers ranges
required for blanching varies with the type and from 0° F to —40° F. Since air circulation is
variety of the vegetable and ranges from 1 to l£ usually by natural convection, heat transfer
min for green beans to 1 1 min for large ears of from the product ranges from 3 hr to 3 days,
corn. Although much of the microbial popula- depending upon the bulk of the product and
tion is destroyed along with the enzymes during upon the conditions in the sharp freezer. Typi-
the blanching process, many bacteria survive. cal items which are sharp frozen are beef and
To prevent spoilage by these viable bacteria, pork half-carcasses, boxed poultry, panned and
vegetables should be chilled to 50° F immedi- whole fish, fruit in barrels and other large
ately after blanching and before they are pack- containers, and eggs (whites, yolks, or whole)
aged for the freezer. in 10 and 30 lb cans.
Like vegetables, fruit must also be cleaned Quick freezing is accomplished in any one or
and washed to remove foreign materials and in any combination of three ways: (1) immer-
to reduce microbial contamination. Although sion, (2) indirect contact, and (3) air blast.

fruit isperhaps even more subject to enzymic 9-27. Air Blast Freezing. Air blast freezing
deterioration than are vegetables, it is never utilizes the combined effects of low temperature
blanched to destroy the natural enzymes since and high air velocity to produce a high rate of
to do so would destroy the natural fresh quality heat transfer from the product. Although the
which is so desirable. method employed varies considerably with the
The enzymes causing the most concern with application, blast freezing is accomplished by
regard to frozen fruit are the ones which circulating high-velocity, low-temperature air
catalyze oxidation and result in rapid browning around the product. Regardless of the method
of the flesh. To control oxidation, fruit to be used, it is important that the arrangement of
frozen covered with a light sugar syrup. In
is the freezer is such that air can circulate freely
some cases, ascorbic acid, citric acid, or sulfur around all parts of the product.
dioxide are also used for this purpose. Packaged blast freezers are available in both
As a general rule, meat products do not suspended and floor-mounted models. Typical
require any special processing prior to freezing. applications are shown in Figs. 9-2 through 9-4.
However, because of consumer demand, speci- Blast freezing is frequently carried out in
ally prepared meats and meat products are being insulated tunnels, particularly where large

frozen in increasing amounts. This is true also quantities of product are tobe frozen (Figs.
of poultry and sea foods. 9-5 and 9-6). In some instances, the product

Because of the relative instability of their is carried through the freezing tunnel and
fatty tissue, pork and fish are usually frozen as frozen on slow-moving, mesh conveyor belts.
soon after chilling as possible. On the other The unfrozen product is placed on the con-
hand, beef is frequently "aged" in a chilling veyor at one end of the tunnel and is frozen
cooler for several days before freezing. During by the time it reaches the other end. Another
this time the beef is tenderized to some extent method is to load the product on tiered dollies.
by enzymic activity. However, the aging of The dollies are pushed into the tunnel and the
beef decreases its storage life, particularly if the product is frozen; whereupon they are pushed
aging period exceeds 6 or 7 days. out of the freezing tunnel into a storage room
With poultry, experiments indicate that (Fig. 9-5).

poultry frozen within 12 to 24 hr after killing Although blast freezing is used to freeze
types of products, particularly
is more tender than that frozen immediately nearly all it is

after killing. However, delaying freezing beyond suitable for freezing products of nonuniform
24 hr tends to reduce storage life without or irregular sizes and shapes, such as dressed
appreciable increasing tenderness. poultry.
9-26. Freezing Methods. Food products may 9-28. Indirect Contact Freezing. Indirect

be either sharp (slow) frozen or quick frozen. freezing is usually accomplished in plate freezers
Sharp freezing is accomplished by placing the and involves placing the product on metal
product in a low temperature room and allowing plates through which a refrigerant is circulated
.

136 PRINCIPLES OF REFRIGERATION

Fig. 9-5. Packaged blast freezers


applied to freezing tunnel. High
velocity, — 15° F air is blasted
through trucks. (Courtesy Car-
rier Corporation.)

(Fig. 9-7). Since the product is in direct thermal freezing is that juices tend to be extracted from
contact with the refrigerated plate, heat transfer the product by osmosis. This results in con-
from the product occurs primarily by conduction tamination and weakening of the freezing
so that the efficiency of the freezer depends, for solution. Too, where a sodium chloride brine
the most part, on the amount of contact surface. is used, salt penetration into the product may
This type of freezer is particularly useful when sometimes be excessive. On the other hand,
products are frozen in small quantities. when fruit is frozen in a sugar solution, sugar
One type of plate freezer widely used by the penetration into the fruit is entirely beneficial.
larger commercial freezers to handle small, The products most frequently
frozen by
flat, rectangular, consumer-size packages is the immersion are and shrimp. Immersion is
fish
multiplate freezer. The multiplate freezer particularly suitable for freezing fish and
consists of a series of horizontal, parallel, shrimp at sea, since the immersion freezer is
refrigerated plates which are actuated by relatively compact and space aboard ship is at
hydraulic pressure so that they can be opened a premium. In addition, immersion freezing
to receive the product between them and then produces a "glaze" (thin coating of ice) on the
closed on the product with any desired pressure. surface of the product which helps to prevent
When the plates are closed, the packages are dehydration of unpackaged products during
held tightly between the plates. Since both the the storage period.
top and the bottom of the packages are in good 9-30. Quick Freezing vs. Sharp Freezing.
thermal contact with the refrigerated plates, Quick frozen products are
nearly always
the rate of heat transfer is high and the product superior to those which are sharp (slow) frozen.
is quickly frozen. D. K. Tressler, in 1932, summarized the views
9-29. Immersion Freezing. Immersion freez- of R. Plank, H. F. Taylor, C. Birdseye, and
ing is accomplished by immersing the product in G. A. Fitzgerald, and stated the following as the
a low temperature brine solution, usually either main advantages of quick freezing over slow
sodium chloride or sugar. Since the refrigerated freezing:
liquid is a good conductor and is in good
thermal contact with all the product, heat 1 The ice crystals formed are much smaller,
transfer is rapid and the product is completely and therefore cause much less damage to cells.
frozen in a very short time. 2. The freezing period being much shorter,
Another advantage of immersion freezing is lesstime is allowed for the diffusion of salts and
that the product frozen in individual units
is the separation of water in the form of ice.
rather than fused together in a mass. 3. The product is quickly cooled below the
The principal disadvantage of immersion temperature at which bacterial, mold, and yeast
SURVEY OF REFRIGERATION APPLICATIONS 137

I
e

o
u

V
L.

&

I
138 PRINCIPLES OF REFRIGERATION

Fig. 9-7. Plate freezer for indirect


contact freezing. (Courtesy Dole
Refrigerating Company.)

growth occurs, thus preventing decomposition time the product temperature is lowered to
during freezing.* 25° F. The temperature range between 30° F
and 25° F is often referred to as the zone of
The principal difference between quick freez- maximum ice-crystal formation, and rapid heat
ing and sharp freezing is in the size,number, removal through this zone is desirable from the
and location of the ice crystals formed in the standpoint of product quality. This is particu-
product as cellular fluids are solidified. When larly true for fruits and vegetables because both
a product is slow frozen, large ice crystals are undergo serious tissue damage when slow frozen.
formed which result in serious damage to the Since animal tissue is much tougher and
tissue of some products through cellular break- much more elastic than plant tissue, the freezing
down. Quick freezing, on the other hand, rate is not as critical in the freezing of meats
produces smaller ice crystals which are formed and meat products as it is in fruits and vege-
almost entirely within the cell so that cellular tables. Recent experiment indicates that poultry
breakdown is greatly reduced. Upon thawing, and fish suffer little, if any, cellular damage
products which have experienced considerable when slow This does not mean, how-
frozen.
damage are prone to lose excessive
cellular ever, that quick frozen meats are not superior
amounts of fluids through "drip" or "bleed," to those which are slow frozen, but only that,
with a resulting loss of quality. for the standpoint of cellular damage, quick
Ice-crystal formation begins in most products freezing is not as important in the freezing of
at a temperature of approximately 30° F and, meats as it' is in fruits and vegetables. For
although some extremely concentrated fluids example, poultry that is slow frozen takes on a
still remain unfrozen even at temperatures below darkened appearance which makes it much less
—50° F, most of the fluids are solidified by the attractive to the consumer. This alone is
* Air Conditioning Refrigerating Data Book, enough to justify the quick freezing of poultry.
Applications Volume, 5th Edition, American Society Too, in all cases, quick freezing reduces the
of Refrigerating Engineers, 1954-55, p. 1-02. processing time and, consequently, the amount
SURVEY OF REFRIGERATION APPLICATIONS 139

of bacterial deterioration. This is especially fish must be reglazed approximately once a


worthwhile in the processing of fish because of month by dipping into fresh water or by
their tendency to rapid spoilage. spraying.
9-31. Packaging Materials. Dehydration, one 9-32. Frozen Storage. The exact temperature
of the principal factors limiting the storage life required for frozen storage is not critical,
of frozen foods, is greatly reduced by proper provided that it is sufficiently low and that it

packaging. Unpackaged products are subject does not fluxuate. Although 0°F is usually
to serious moisture losses not only during the adequate for short-term (retail) storage, -5° F
freezing process but also during the storage is the best temperature for all-around long-term
period. While in storage, unpackaged frozen (wholesale) storage. When products having
products lose moisture to the air continuously unstable fats (oxidizable, free, fatty acids) are
by sublimation. This eventually results in a stored in any quantity, the storage temperature
condition known as "freezer-burn," giving the should be held at —10° F or below in order to
product a white, leathery appearance. Freezer- realize the maximum storage life.

burn is accompanied by oxidation,


usually When products are stored above — 20° F,
flavor changes, and loss of vitamin content. which is normally the case, the temperature of
With few exceptions, all products are pack- the storage room should be maintained constant
aged before being placed in frozen storage. with a variation of not more than 1 ° F in either
Although most products are packaged before direction. Variations in storage temperature
freezing, some, such as loose frozen peas and lima cause alternate thawing and refreezing of some
beans, are packaged after the freezing process. of the juices in the product. This tends to
To provide adequate protection against increase the size of the ice crystals in the
dehydration and oxidation, the packaging product and eventually results in the same
material should be practically 100% gas and type of cellular damage as occurs with slow
vapor proof and should fit tightly around the freezing.
product to exclude as much air as possible. Too, Since many packaging materials do not offer
air spaces in packages have an insulating effect complete protection against dehydration, the
which reduce the freezing rate and increase relative humidity should be kept at a high level
freezing costs. (85 % to 90%) in frozen storage rooms, particu-
The compe-
fact that frozen products are in larly for long-term storage.
tition to products preserved by other methods Proper stacking of the product is also
introduces several factors which must be taken essential. Stacking should always be such that
into account when selecting packaging materials. it permits adequate air circulation around the
When the product is to be sold directly to the product. It is particularly important to leave a
consumer, the package must be attractive and good size air space between the stored product
convenient to use in order to stimulate sales. and the walls of the storage room. In addition
From a cost standpoint, the package should be to permitting air circulation around the product,
relatively inexpensive and of such a nature that this eliminates the possibility of the product
it permits efficient handling so as to reduce absorbing heat directly from the warm walls.
processing costs. 9-33. Commercial Refrigerators. The term
Some packaging materials in general use are "commercial refrigerator" is usually applied to
aluminum foil, impregnated paper-
tin cans, the smaller, ready-built, refrigerated fixtures of
board cartons, paper-board cartons over- the type used by retail stores and markets,
wrapped with vapor-proof wrappers, wax paper, hotels, restaurants, and institutions for the
cellophane, polyethylene, and other sheet and dispensing
processing, storing, displaying,
plastics. of perishable commodities. The term is some-
Frozen fish are often given an ice glaze (a times applied also to the larger, custom-built
thin coating of ice) which provides an excellent refrigerated fixtures and rooms used for these
protective covering. However, since the ice purposes.
glaze is very brittle, glazed fish must be handled Although there are a number of special
carefully to avoid breaking the glaze. Too, since purpose refrigerated fixtures which defy classi-
the ice glaze gradually sublimes to the air, the fication, in general, commercial fixtures can
140 PRINCIPLES OF REFRIGERATION

Fig. 9-8. Typical reach-in refrigera-


tor. (Courtesy Tyler Refrigeration
Corporation.)

be grouped into three principal categories: product or commodity as attractively as possible


(1) reach-in refrigerators, (2) walk-in coolers, in order to stimulate sales. Therefore, in the
and (3) display cases. design of refrigerated display fixtures, first

9-34. Reach-In Refrigerators. The reach-in consideration is given to the displaying of the
refrigerator is probably the most versatile and product. In many cases, this is not necessarily
the most widely used of all commercial fixtures. compatible with providing the optimum storage
Typical users are grocery stores, meat markets, conditions for the product being displayed.
bakeries, drug stores, lunch counters, res- Hence, the storage life of a product in a display
taurants, florists, hotels, and institutions of all fixture is frequently very limited, ranging from
kinds. Whereas some reach-in refrigerators a few hours in some instances to a week or more
serve only a storage function, others are used in others, depending upon the type of product
for both storage and display (Fig. 9-8). Those and upon the type of fixture.
serving only the storage function usually have Display fixtures are of two general types:
solid doors, whereas those used for display (1) the self-service case, from which the customer
have glazed doors. serves himself directly, and (2) the service case,
9-35. Walk-In Coolers. Walk-in coolers are from which the customer is usually served by
primarily storage fixtures and are available in a an attendant. The former is very popular in
wide variety of sizes to fit every need. Nearly supermarkets and other large, retail, self-service
all retail stores, markets, hotels, restaurants, establishments, whereas the service case finds
institutions, etc., of any size employ one or more use in the smaller groceries, markets, bakeries,
walk-in coolers for the storage of perishables etc. Typical service cases are shown in Figs. 9-9
of all types. Some walk-in coolers are equipped and 9-10.
with glazed reach-in doors. This feature is Self-service cases are of two types, open and
especially convenient for the storing, displaying, closed, with the open type gaining rapidly in
and dispensing of such items as dairy products, popularity. With the advent of the super-
eggs, and beverages. Walk-in coolers with market, the trend has been increasingly toward
reach-in doors are widely used in grocery stores, the open type self-service case, and the older,
particularly drive-in groceries, for handling closed type self-service cases are becoming
such items. obsolete. Several of the more popular types of
9-36. Display Cases. The principal function of open self-service cases are shown in Figs. 9-11
any kind of display fixture is to display the and 9-12. These are used to display meat
SURVEY OF REFRIGERATION APPLICATIONS 141

vegetables, fruit, frozen foods, ice cream, dairy


products, delicatessen items, etc. The design
of the case varies somewhat with the particular
type of product being displayed. Too, designs
are available for both wall and island installa-
tion. Although some provide additional storage
space, othersdo not.
9-37. Special Purpose Fixtures. Although all
the refrigerated fixtures discussed in the preced-
ing sections are available in a variety of designs
in order to satisfy the specific requirements of
individual products and applications, a number
of special purpose fixtures is manufactured
which may or may not fall into one of the three
general categories already mentioned. Some of
the more common special purpose fixtures are

Fig. 9-10. Double-duty service case for displaying


meats. (Courtesy Tyler Refrigeration Corporation.)

As a general rule, a locker plant furnishes all

or most of the following facilities and/or


services:

1. A chilling room for chilling freshly killed


meats.
2. A cold storage room for holding products
under refrigeration while awaiting preparation
and processing prior to freezing.
3. A processing room where the products are
processed and packaged for the freezer.

Fig. 9-9. Conventional single-duty service case for


displaying meats. (Courtesy Tyler Refrigeration
Corporation.)

beverage coolers, milk coolers (dairy farm), milk


and beverage dispensers, soda fountains, ice
cream makers, water coolers, ice makers, back-
bar refrigerators, florist boxes, dough retarders,
candy cases, and mortuary refrigerators.
9-38. Frozen Food Locker Plants. Normally,
the function of a frozen food locker plant is to
process and freeze foods for individual families
and other groups, either for take-home storage
or for storage at the locker plant. When
storageis at the plant, the customer rents a

and calls at the plant for


storage space (locker) Fig. 9-11. High multishelf produce sales

one or more packages as needed. (Courtesy Tyler Refrigeration Corporation.)


142 PRINCIPLES OF REFRIGERATION

Fig. 9-12. Open-type display case for


frozen foods and icecream. (Courtesy
Tyler Refrigeration Corporation.)

4. A freezing room or cabinet in which the 8. A curing and smoking room for handling
food is frozen prior to being placed in storage. bacon, ham, sausage, and other cured meats.
5. A low temperature room containing the Services such as slaughtering, lard rendering,
storage lockers. sausage making, etc., are also provided by some
6. A low temperature bulk storage room. plants.
7. An aging room where certain meats are The layout of a typical locker plant is shown
kept under refrigeration and allowed to age in Fig. 9-13.The recommended design condi-
(tenderize) for periods usually ranging from tions for the various spaces in the locker plant
7 to 10 days. are given in Fig. 9-14. The average size of the

M^
v//>y////Y//////^/Y//w//y/^///,/yy/s/y/>,

Fig. 9-13. Typical locker plant. (ASRE Data Book, Applications Volume, 1956-57.) Reproduced by permission
of American Society of Heating, Refrigerating, and Air-Conditioning Engineers.
SURVEY OF REFRIGERATION APPLICATIONS 143

Locker Plant Design Conditions


Room Refrigerant Insulation
Type of space
temperature temperature thickness, inches.

Work room, process room,


and kitchen Atmospheric None None
Chill room 34 to 36 F Design for 20 to 25 F below room 3 to 8
35 F temperature, for gravity
circulation; 10 to 15 F
below room tempera-
ture for forced air cir-
culation

Aging room 34 to 36 F Design for Same as chill room 3 to 8


35 F
Curing room 38 to 40 F Design for Same as chill room 3 to 8
40F
Freezing room (gravity -10 to -20 F -20 to -30 F 6 to 12
air circulation)

Freezer cabinet (in locker Not important -15 to -20 F 1 or 2


room)
Blast freezer Depends on type of -10 to -15 F 6 to 12
system used
Locker room or bulk OF -15 to -20 F 6 to 12
storage

Fig. 9-14. (ASRE Data Book, Applications Volume, 1956-57. Reproduced by permission of American Society
of Heating, Refrigerating and Air Conditioning Engineers.)

individual locker is 6 cu ft and the average same for all types of applications. Commerical
product storage capacity is approximately 35 refrigeration was selected for emphasis because
to 40 lb per cubic foot. Minimum product embraces a wide range of applications
this area
turnover is approximately 2 lb per locker per and because the problems encountered in this
day. Standard practice is to base chilling room area are representative of those in the other
and freezer capacities on the handling of 2 to 4 areas. Hence, even though the discussion in
lb of product per locker per day. this chapter and in those which follow deals
9-39. Summary. Recognizing that a thorough chiefly with commercial refrigeration, the
knowledge of the application itself is a pre- principals of system design and the methods of
requisite to good system design and proper equipment selection developed therein may be
equipment selection, we have devoted the mate- applied to all types of mechanical refrigeration
rial in this chapter to a brief survey of a few of applications.
the applications of mechanical refrigeration, Although no attempt is made in this book
with special emphasis being given to the area to discuss air conditioning as such except in a
of commercial refrigeration. very general way, it should be pointed out that
Obviously, the applications of mechanical most commercial refrigeration applications,
refrigeration are too many and too varied to particularly those concerned with product
permit detailed consideration of each and storage, involve air conditioning in that they
every type. Fortunately, this is neither necessary ordinarily include close control of the tempera-
nor desirable since methods of system designing ture, humidity, motion, and cleanliness of the air
and equipment selection are practically the in the refrigerated space.
application, it is essential that consideration be
given to all heat sources present and that all the
heat evolving from them be taken into account
in the over-all calculation.
10-2. Equipment Running Time. Although
refrigerating equipment capacities are normally
given in Btu per hour, in refrigeration applica-

10 tions the total cooling load


for a 24-hr period, that
is usually calculated
Btu per 24 hr.
is, in
Then, to determine the required Btu per hour
capacity of the equipment, the total load for the

Cooling Load 24-hr period is divided by the desired running


time for the equipment, viz:

Calculations RequiredBtu/hr
equipment = Totalcoolingload)Btu/24hr
2 : '.

Desired running time


capacity
(10-1)

Because of the necessity for defrosting the


10-1. The Cooling Load. The cooling load on evaporator at frequent intervals, it is not

refrigerating equipment seldom results from practical to design the refrigerating system in

any one single source of heat. Rather, it is the such a way that the equipment must operate
summation of the heat which usually evolves continuously in order to handle the load. In
from several different sources. Some of the most cases, the air passing over the cooling coil

more common sources of heat which supply is a temperature below its dew point
chilled to

the load on refrigerating equipment are: and moisture is condensed out of the air onto
the surface of the cooling coil. When the tem-
Heat that leaks into the refrigerated space
1.
perature of the coil surfaceabove the freezing
is
from the outside by conduction through the
temperature of water, the moisture condensed
insulated walls.
out of the air drains off the coil into the con-
2. Heat that enters the space by direct
densate pan and leaves the space through the con-
radiation through glass or other transparent
densate drain. However, when the temperature of
materials.
the cooling coil is below the freezing tempera-
3. Heat that is brought into the space by
ture of water, the moisture condensed out of
warm outside air entering the space through
the air freezes into ice and adheres to the surface
open doors or through cracks around windows
of the coil, thereby causing "frost" to accumu-
and doors.
late on the coil surface. Since frost accumulation
4. Heat given off by a warm product as its
on the coil surface tends to insulate the coil
temperature lowered to the desired level.
is
and reduce the coil's capacity, the frost must
5. Heat given off by people occupying the
be melted off periodically by raising the surface
refrigerated space.
temperature of the coil above the freezing point
6. Heat given off by any heat-producing
of water and maintaining it at this level until
equipment located inside the space, such as
the frost has melted off the coil and left the
electric motors, lights, electronic equipment,
space through the condensate drain.
steam tables, coffee urns, hair driers, etc.
No matter how the defrosting is accomplished,
The importance of any one of these heat the defrosting requires a certain amount of
sources with relation to the total cooling load time, during which the refrigerating effect of the
on the equipment varies with the individual system must be stopped.
application. Not all them will be factors in One method of defrosting the coil is to stop
every application, nor will the cooling load in the compressor and allow the evaporator to
any one application ordinarily include heat warm up to the space temperature and remain
from all these sources. However, in any given at this temperature for a sufficient length of
144
COOLING LOAD CALCULATIONS 145

time to allow the frost accumulation to melt off usually designed for continuous run and cooling
the coil. This method of defrosting is called loads for air conditioning applications are
"off-cycle" defrosting. Since the heat required determined directly in Btu per hour.
to melt the frost in off-cycle defrosting must 10-3. Cooling Load Calculations. To simplify
come from the air in the refrigerated space, cooling load calculations, the total cooling load
defrosting occurs rather slowly and a consider- is divided into a number of individual loads
able length of time is required to complete the according to the sources of heat supplying the
process. Experience has shown that when off- load. The summation of these individual loads
cycle defrosting is used, the maximum allowable is on the equipment.
the total cooling load
running time for the equipment is 16 hr out of In commercial refrigeration, the total cooling
each 24-hr period, the other 8 hr being allowed load is divided into four separate loads, viz:
for the defrosting. This means, of course, that (1) the wall gain load, (2) the air change load,
the refrigerating equipment must have sufficient (3) the product load, and (4) the miscellaneous
capacity to accomplish the equivalent of 24 hr of or supplementary load.
cooling in 1 6 hr of actual running time. Hence, 10-4. The Wall Gain Load, The wall gain

when off-cycle defrosting is used, the equipment load, sometimes called the wall leakage load,
running time used in Equation 10-1 is approxi- is a measure of the heat which leaks through the

mately 16 hr. walls of the refrigerated space from the outside


When the refrigerated space is to be main- to the inside. Since there is no perfect insulation,
tained at a temperature below 34° F, off-cycle there is always a certain amount of heat passing

defrosting is The variation


not practical. in from the outside to the inside whenever the
space temperature which would be required in inside temperature is below that of the outside.
order to allow the cooling coil to attain a The wall gain load is common to all refrigeration
temperature sufficiently high to melt off the applications and is ordinarily a considerable
frost during every off cycle would be detrimental part of the total cooling load. Some exceptions
to the stored product. Therefore where the to this are liquid chilling applications, where the
space temperature maintained below 34° F,
is outside area of the chiller is small and the walls
some method automatic defrosting is
of of the chiller are well insulated. In such cases,
ordinarily used. In such cases the surface of the the leakage of heat through the walls of the
coil is heated artificially, either with electric chiller is so small in relation to the total cooling
heating elements, with water, or with hot gas load that its effect is negligible and it is usually
from the discharge of the compressor (see neglected. On the other hand, commercial
Chapter 20). storage coolers and residential air conditioning
Defrosting by any of these means is accom- applications are both examples of applications
plished much more quickly than when off-cycle wherein the wall gain load usually accounts for
defrosting is used. Hence, the off-cycle time the greater portion of the total load.
required is less for automatic defrosting and 10-5. The Air Change Load. When the door
the maximum allowable running time for the of a refrigerated space is opened, warm outside
equipment is greater than for the aforementioned air enters the space to replace the more dense

off-cycle defrosting. For systems using auto- cold air which is lost from the refrigerated space
matic defrosting the maximum allowable through the open door. The heat which must
running time is from 18 to 20 hr out of each be removed from this warm outside air to
24-hr period, depending upon how often de- reduce its temperature to the space temperature
frosting is necessary for the application in becomes a part of the total cooling load on the
question. As a general rule, the 18 hr running equipment. This part of the total load is called
time is used. the air change load.
of interest to note that since the tempera-
It is The relationship of the change load to the
air

ture of the cooling coil in comfort air condition- total cooling load varies with the application.
ing applications is normally around 40° F, no Whereas in some applications the air change
frost accumulates on the coil surface and, there- load is not a factor at all, in others it represents
fore, no off-cycle time is required for defrosting. a considerable portion of the total load. For
For this reason, air conditioning systems are example, with liquid chillers, there are no doors
146 PRINCIPLES OF REFRIGERATION

or other openings through which air can pass The importance of the product load in relation
and therefore the air change load is nonexistent. to the total cooling load, like all others, varies

On the other hand, the reverse is true for air with the application. Although it is nonexistent
conditioning applications, where, in addition in some applications, in others it represents
to the air changes brought about by door practically the entire cooling load. Where the
openings, there is also considerable leakage of refrigerated cooler is designed for product
air into the conditioned space through cracks storage, the product is usually chilled to the
around windows and doors and in other parts storage temperature before being placed in the
of the structure. Too, in many air conditioning cooler and no product load need be considered
applications outside air is purposely introduced since the product is already at the storage
into the conditioned space to meet ventilating temperature. However, in any instance where
requirements. When large numbers of people the product enters a storage cooler at a tem-
are in the conditioned space, the quantity of perature above the storage temperature, the
fresh air which must be brought in from the quantity of heat which must be removed from
outside is quite large and the cooling load the product in order to reduce its temperature
resulting for the cooling of this air to the tem- to the storage temperature must be considered
perature of the conditioned space is often a as a part of the total load on the cooling
large part of the total cooling load in such equipment.
applications. In some few instances, the product enters the
In air conditioning applications, the air change storage fixture at a temperature below the
load is called either the ventilating load or the normal storage temperature for the product.
infiltration load. The term ventilating load is A case in point is ice cream which is frequently
used when the air changes in the conditioned chilled to a temperature of 0°F or -10° F
space are the result of deliberate introduction during the hardening process, but is usually
of outside air into the space for ventilating stored at about 10° F, which is the ideal dipping
purposes. The term infiltration load is used temperature. When such product enters
a
when the air changes are the result of the storage at a temperature below the space
natural infiltration of air into the space through temperature, it will absorb heat from the
cracks around windows and doors. Every air storage space as it warms up to the storage
conditioning application will involve either an temperature and thereby produce a certain
infiltration load or a ventilating load, but never amount of refrigerating effect of its own. In
both in the same application. other words, it provides what might be termed
Since the doors on commercial refrigerators a negative product load which could theoreti-
are equipped with well-fitted gaskets, the cracks callybe subtracted from the total cooling load.
around the doors are tightly sealed and there is This is never done, however, since the refriger-
little, if any, leakage of air around the doors of a ating effect produced is small and is not
commercial fixture in good condition. Hence, continuous in nature.
in commercial refrigeration, the air changes are The cooling load on the refrigerating equip-
usually limited to those which are brought about ment resulting from product cooling may be
by actual opening and closing of the door or either intermittent or continuous, depending
doors. on the application.The product load is a part
10-6. The Product Load. The product load is of the total cooling load only while the tempera-
made up of the heat which must be removed ture of the product is being reduced to the
from the refrigerated product in order to reduce storage temperature. Once the product is
the temperature of the product to the desired cooled to the storage temperature, it is no
level. The term product as used here is taken longer a source of heat and the product load
to mean any material whose temperature is ceases to be a part of the load on the equipment.
reduced by the refrigerating equipment and An exception to this is in the storage of fruit
includes not only perishable commodities, such and vegetables which give off respiration heat
as foodstuff, but also such items as welding for the entire time they are in storage even
electrodes, masses of concrete, plastic, rubber, though there is no further decrease in their
and liquids of all kinds. temperature (see Section 10-17).
COOLING LOAD CALCULATIONS 147

There of course, a number of refrigera-


are, human occupancy is frequently the largest single
tion where product cooling is
applications Too, many air condi-
factor in the total load.
more or less continuous, in which case the tioning systems are installed for the sole purpose
product load is a continuous load on the equip- of cooling electrical, electronic, and other types
ment. This is true, for instance, in chilling of heat-producing equipment. In such cases,
coolers where the primary function is to chill the equipment usually supplies the greater
the warm product to the desired storage tem- portion of the cooling load.
perature. When the product has been cooled 10-8. Factors Determining the Wall Gain
to the storage temperature, moved
it is usually Load. The quantity of heat transmitted through
out of the chilling room into a storage room and the walls of a refrigerated space per unit of time
the chilling room is then reloaded with warm is the function of three factors whose relation-

product. In such cases, the product load is ship is expressed in the following equation:
continuous and is usually a large part of the
total load on the equipment.
Q=A x U x D (10-2)

Liquid chilling is another application wherein where Q = the quantity of heat transferred in
the product provides a continuous load on the Btu/hr
refrigerating equipment. The flow of the liquid A — the outside surface area of the wall
being chilled through the chiller is continuous (square feet)
with warm liquid entering the chiller and cold U = the over-all coefficient of heat trans-
liquid leaving. In this instance, the product mission (Btu/hr/sq ft/° F)
load is practically the only load on the equip- D = the temperature differential across
ment since there is no air change load and the the wall (° F)
wall gain load is negligible, as is the miscel- The coefficient of transmission or "U" factor
laneous load. isa measure of the rate at which heat will pass
In air conditioning applications there is no through a 1 sq ft area of wall surface from the
product load as such, although there is often a air on one side to the air on the other side for

"pull-down load," which, in a sense, may be each 1° F


of temperature difference across the
thought of as a product load. wall. The value of the U factor is given in
10-7. The Miscellaneous Load. The miscel- Btu per hour and depends on the thickness of
laneous load, sometimes referred to as the the wall and on the materials used in the wall
supplementary load, takes into account all construction. Since it is desirable to prevent as
miscellaneous sources of heat. Chief among much heat as possible from entering the space
these are people working in or otherwise and becoming a load on the cooling equipment,
occupying the refrigerated space along with the materials used in the construction of cold
lights or other electrical equipment operating storage walls should be good thermal insulators

inside the space. so that the value of U is kept as low as is

In most commercial refrigeration applica- practical.

tions the miscellaneous load is relatively small, According to Equation 10-2, once the U
usually consisting only of the heat given off by factor is established for a wall, the rate of heat

lightsand fan motors used inside the space. flow through the wall varies directly with the
In air conditioning applications, there is no surface area of the wall and with the temperature
miscellaneous load as such. This is not to say differential across the wall. Since the value of

that human occupancy and equipment are not U is given in Btu/hr/sq ft/° F, the total quantity
a part of the cooling load in air conditioning of heat passing through any given wall in 1 hr
applications. On the contrary, people and can be determined by multiplying the U factor
equipment are often such large factors in the by the wall area in square feet and by the tem-
air conditioning load that they are considered perature difference across the wall in degrees
as separate loads and are calculated as such. Fahrenheit, that is, by application of Equation
For example, in those air conditioning applica- 10-2.

tions where large numbers of people occupy the Example10-1. Determine the total quan-
conditioned space, such as churches, theaters, tityof heat in Btu per hour which will pass
restaurants, etc., the cooling load resulting from through a wall 10 ft by 20 ft, if the U factor
148 PRINCIPLES OF REFRIGERATION
for the wall is 0.16 Btu/hr/sq ft/ F and the from top). In the next column select the desired
temperature on one side of the wall is 40° F thickness of clay tile (6 in.) and move to the
while the temperature on the other side is 95° F. right to the column listing values for 4 in. of

Solution
insulation. Read the U factor of the wall, 0.064
Btu/hr/sq ft/° F.
Total wall area 10 ft x 20 ft
200 sq ft
Temperature differ-
=

Should it be necessary, the U factor for any


type of wall construction can be readily calcu-
ential across wall, ° F . 95° - 40°
lated provided that either the conductivity or the
>55°F
Applying Equation conductance of each of the materials used in the
10-2, the heat gain 200 x 0.16 x 55 wall construction is known. The conductivity
through the wall 1760 Btu/hr or conductance of most of the materials used in
wall construction can be found in tables. Too,
Since the value of U in Equation 10-2 is in this information is usually available from the
Btu per hour, the result obtained from Equation
manufacturer or producer of the material.
10-2 is in Btu per hour. To determine the wall
Table 10-4 lists the thermal conductivity or the
gain load in Btu per 24 hr as required in refriger-
conductance of materials frequently used in the
ation load calculations, the result of Equation
construction of cold storage walls.
10-2 is multiplied by 24 hr. Hence, for calcula- The thermal conductivity or k factor of a
tion cooling loads in refrigeration applications,
material is the rate in Btu per hour at which
Equation 10-2 is written to include this multi-
heat passes through a 1 sq ft cross section of the
plication, viz:
material 1 in. thick for each 1 ° F of temperature
Q=AxUxDx24 (10-3)
difference across the material.
Whereas the thermal conductivity or k factor
10-9. Determination of the (/Factor. Over- is available only for homogeneous materials
all coefficients of transmission or U factors have
and the value given is always for a 1 in. thickness
been determined for various types of wall of the material, the thermal conductance or C
construction and these values are available in factor is available for both homogeneous and
tabular form. Tables 10-1 through 10-3 list U nonhomogeneous materials and the value given
values for various types of cold storage walls. is for the specified thickness of the material.
Example 10-2. From Table 10-1, determine For any homogeneous material, the thermal
the U factor for a wall constructed of 6-in. clay conductance can be determined for any given
tile with 4 in. of corkboard insulation. thickness of the material by dividing the k
by the thickness in
factor inches. Hence, for a
Solution. Turn to Table 10-1 and select the
homogeneous material,
appropriate type of wall construction (third

Concrete
C=\x (10-4)

Air spaces aggregate


where x = the thickness of material in inches.
Example 10-3. Determine the thermal
conductance for a 5 in. thickness of corkboard.

Solution
From Table 10-4,
k factor of cork-
board = 0.30 Btu/hr/sq ft/in/° F
Applying
Equation 10-4, C =_ Q ,

= 0.06 Btu/hr/sq ft/° F


Since the rate of heat transmission through
nonhomogeneous materials, such as the concrete
Fig. 10-1. Concrete aggregate building block. building block in Fig. 10-1, will vary in the
COOLING LOAD CALCULATIONS 149

several parts of the material, the C factor from Example Calculate the value of £/for
10-4.
nonhomogeneous materials must be determined a wall constructed of 8 in. cinder aggregate

by experiment. building blocks, insulated with 4 in. of cork-


The resistance that a wall or a material offers board, and finished on the inside with 0.5 in. of
cement plaster.
to the flow of heat is inversely proportional to
the ability of the wall or material to transmit Solution
heat. Hence, the over-all thermal resistance of From Table 10-4,
a wall can be expressed as the reciprocal of the 8 in. cinder
over-all coefficient of transmission, whereas the aggregate block C=0.60
thermal resistance of an individual material Corkboard k = 0.30
can be expressed as the reciprocal of its conduc-
Cement plaster k =8.00
tivity or conductance, viz: From Table
10-5,4,
inside
Over-all thermal resist-
surface
ance ~U conductance ft = 1-65
Thermal resistance of 1 1 x outside
or or surface
an individual material -l c k conductance /„ = 4.00

The terms I Ik and 1/C express the resistance Applying


to heat flow through a single material from
Equation 10-5, -I+-L+-L
the over-all 4 0.6 0.3
surface to surface only and do not take into
thermal resist- 0.5 1
+
T + L65
.

account the thermal resistance of the thin film


ance, 1/17
of air which adheres to all exposed surfaces. In
= 0.25 + 1.667 + 13.333
determining the over-all thermal resistance to
+ 0.0625 + 0.607
the flow of heat through a wall from the air on = 15.92
one side to the air on the other side, the resist- Therefore, U = 1/15.92
ance of the air on both sides of the wall should = 0.0622 Btu/hr/sq ft/ F
be considered. Air film coefficients or surface
For the most part, it is the insulating material
conductances for average wind velocities are
used in the wall construction that determines
given in Table 10-5/*.
the value of U for cold storage walls. The
When a wall is constructed of several layers
surface conductances and the conductances of
of different materials the total thermal resistance
the other materials in the wall have very little
of the wall is the sum of the resistances of the
individual materials in the wall construction,
effect on the value of U because the thermal
resistance of the insulating material is so large
including the air films, viz:
with relation to that of the air films and other
materials. Therefore, for small coolers, it is
\ \ x x x \
sufficiently accurate to use the conductance of
U fi **1 *2 kn Jo the insulating material alone as the wall U
Therefore factor.
1 10-10. Temperature Differential across Cold
U *
X X x Storage Walls. The temperature differential
I 1
across cold storage walls is usually taken as the
ft kx kt kn f„
difference between the inside and outside design
where = temperatures.
rr surface conductance of inside wall,
'
' floor, or ceiling
The inside design temperature is that which
is to be maintained inside the refrigerated space
1
— = surface conductance of outside wall, and usually depends upon the type of product
* * floor, or roof to be stored and the length of time the product
is to be kept in storage. The recommended
Note. When nonhomogeneous materials are storage temperatures for various products are
used, 1/C is substituted for xjk. given in Tables 10-10 through 10-13.
150 PRINCIPLES OF REFRIGERATION
The outside design temperature depends on that radiant energy waves are either reflected
the location of the cooler. For cold storage by or absorbed by any opaque material that
walls located inside a building, the outside they strike. Light-colored, smooth surfaces
design temperature for the cooler wall is taken will tend to reflect mbre and absorb less radiant
as the inside temperature of the building. When energy than dark, rough-textured surfaces.
cold storage walls are exposed to the outdoors, Hence, the surface temperature of smooth,
the outdoor design temperature for the region light-colored walls will be somewhat lower than
(Table 10-6) is used as the outside design tem- that of dark, rough-textured walls under the
perature. The outdoor design temperatures same conditions of solar radiation.
given in Table 10-6 are average outdoor tem- Since any increase in the outside surface
peratures and include an allowance for normal temperature will increase the temperature
variations outdoor design dry bulb
in the differential across the wall, the temperature
temperature during a 24-hr period. These differential across sunlit walls must be corrected
temperatures should not be used for calculating to compensate for the sun effect. Correction
air conditioning loads. factors for sunlit walls are given in Table 10-7.
1 0-1 1.TemperatureDifferential across Ceil- These values are added to the normal tempera-
ings and Floors. When a cooler is located ture differential. For walls facing at angles to
inside of a building and there is adequate the directions listed in Table 10-7, average
clearance between the top of the cooler and the values can be used.
ceiling of the building to allow free circulation of 10-13. Calculating the Wall Gain Load. In
air over the top of the cooler, the ceiling of the determining the wall gain load, the heat gain
cooler is treated the same as an inside wall. through all the walls, including the floor and
Likewise, when the top of the cooler is exposed ceiling, must be taken into account. When
to the outdoors, the ceiling is treated as an the several walls or parts of walls are of different
outdoor wall. The same holds true for floors construction and have different U factors, the
except when the cooler floor is laid directly on a heat leakage through the different parts is

slab on the ground. As a general rule, the computed Walls having identical
separately.
ground temperature under a slab varies only U factors may be considered together, provided
slightly the year round and is always consider- that the temperature differential across the
ably less than the outdoor design dry bulb walls is the same. Too, where the difference
temperature for the region in summer. Ground in the value of U is slight and/or the wall area
temperatures used in determining the tempera- involved is small, the difference in the U factor
ture differential across the floor of cold storage can be ignored and the walls or parts of walls
rooms are given in Table 10-6A and are based can be grouped together for computation.
on the regional outdoor design dry bulb Example 10-5. A walk-in cooler, 16 ft x
temperature for winter. 20 x 10 ft high is located in the southwest
ft
10-12. Effect of Solar Radiation. Whenever corner of a store building in Dallas, Texas (Fig.
the walls of a refrigerator are so situated that 10-2). The south and west walls of the cooler
they receive an excessive amount of heat by are adjacent to and a part of the south and west
radiation, either from the sun or from some walls of the store building. The store has a 14 ft
other hot body, the outside surface temperature ceiling so that there is a 4 ft clearance between

of the wall will usually be considerably above the top of the cooler and the ceiling of the store.

the temperature of the ambient air. familiar A The store is air conditioned and the temperature
inside the store is maintained at approximately
example of this phenomenon is the excessive
80° F. The inside design temperature for the
surface temperature of an automobile parked
cooler is 35° F. Determine the wall gain load
in the sun. The temperature of the metal for the cooler if the walls of the cooler are of the
surface is much higher than that of the sur- following construction:
rounding air. The amount by which the surface
South and west
temperature exceeds the surrounding air tem-
(outside walls) 6 in. clay tile
perature depends upon the amount of radiant 6 corkboard
in.
energy striking the surface and upon the 0.S cement plaster finish
reflectivity of the surface. Recall (Section 2-21) on inside
F

COOLING LOAD CALCULATIONS 151

North and east


(inside walls) 1 in. board on both sides '
, Outside design
16 H temperature, 92* F

^mm
of 2 x 4 studs
M I MM MIM
I

3f in. granulated cork 6 in. corkboard-' | 6 in. clay tile


Ceiling Same as north and east
walls Cooler 35° F
Floor 4 in. corkboard laid on 10 ft ceiling

5 in. slab and finished


with 3 in. of concrete 20' :

Solution
Wall surface area Partitions 3| in.

North wall 10 x 16 = 160 sq ft granulated cork-


1 in. board
West wall 10 x 20 = 200 sq ft on each side
South wall 10 x 16 = 160sqft Inside temperature
East wall 10 x 20 = 200 sq ft 80'
Ceiling 14
Ceiling 16 x 20 = 320 sq ft ft

Floor 16 x 20 = 320 sq ft
Wall U factors (Tables 10-1, 10-2, and 10-3)
North and east 0.079 Btu/hr/sq ft/° F
walls
South and west
walls 0.045
Ceiling 0.079
Floor 0.066
From Table 10-6,
outside summer de-
Fig. 10-2
sign dry bulb for
Dallas 92° F
From Table
10-6A, design A short method calculation may be used to
ground temperature determine the wall gain load for small coolers
for Dallas 70° F and for large coolers where the wall factor U

Outside Inside Normal Correction Design


Design Design Wall Factor from Wall
Temp. Temp. T.D. Table 10-7 T.D.

North wall 80° F 35° F 45° F 45° F


South wall 92° F 35° F 57°F 4°F 61° F
West wall 92° F 35* F 57= p 6°F 63° F
East wall 80° F 35° F 45° F 45° F
Ceiling 80° F 35° F 45° F 45° F
Floor 70° F 35° F 35° F 35° F

Applying Equation 10-2, and temperature difference are approximately


North wall 160 x 0.079 x 45 = 569Btu/hr the same for all the walls. Table 10-18 lists wall
West wall 200 x 0.045 x 63 = 567 gain factors (Btu/24 hr sq ft) based on the thick-
South wall 160 x 0.045 x 61 = 439 ness of the wall insulation and on the tempera-
East wall 200 x 0.079 x 45 = 7ll ture differential across the wall. To compute
Ceiling 320 x 0.079 x 45 = 1,137 the wall gain load in Btu/24 hr by the short
Floor 320 x 0.066 x 35 = 739 method, multiply the total outside wall area
Total wall gain load 4 162 Btu/hr
. (including floor and ceiling) by the appropriate
= 4,162 x 24 - 99,890 Btu/24 hr wall gain factor from Table 10-18, viz:
152 PRINCIPLES OF REFRIGERATION

Wall gain load = Outside surface area Solution


x wall gain factor Cubic feet of air per
per24hr cfm x 60 x 24
To select the appropriate wall gain factor
300 x 60 x 24
from Table 10-18, find the thickness of the wall
432,000 cu ft/24 hr
insulation in the extreme left-hand column of From Table 10-8.A
the table, move right to the column headed by heat gain per cubic
the design wall temperature difference, and read feet = 1.86Btu/cuft
the wall gain factor in Btu/24 hr/sq ft. For Ventilating (air
example, assume that the walls of a cooler are change) load = cu ft/24 hr
insulated with the equivalent of 4 in. of cork- x Btu/cu ft
board and that the temperature difference
= 432,000 x 1.86
= 803,520 Btu/24 hr
across the walls is 55° F. From Table 10-18,
read the wall gain factor of 99 Btu/24 hr/sq ft Except in those few cases where air is pur-
(see Example 10-18).
posely introduced into the refrigerated space for
10-14. Calculation the Air Change Load.
ventilation, the air changes occurring in the
The space heat gain resulting from air changes
space are brought about solely by infiltration
in the refrigerated space is difficult to determine
through door openings, The quantity of outside
with any real accuracy except in those few cases air entering a space through door openings in
where a known quantity of air is introduced a 24-hr period depends upon the number, size,
into the space for ventilating purposes. When and location of the door or doors, and upon
the weight of outside air entering the space in a
the frequency and duration of the door open-
24-hr period is known, the space heat gain ings. Since the combined effect of all these
resulting from air changes depends upon the
factors varies with the individual installation
difference in the enthalpy of the air at the
and is difficult to predict with reasonable ac-
inside and outside conditions and can be curacy, it is general practice to estimate the
calculated by applying the following equation:
air change quantity on the basis of experience
Air change load = W(h„ - hi) (10-6) with similar applications. Experience has shown
where W = weight of air entering space in 24 hr that, as a general rule, the frequency and dur-
(lb/24 hr)
ation of door openings and, hence, the air

h, = enthalpy of outside air (Btu/lb) change quantity, depend on the inside volume
hi = enthalpy of inside air (Btu/lb) of the cooler and the type of usage. Tables
10-9A and 10-9B list the approximate number
However, since air quantities are usually
of air changes per 24 hr for various cooler
given in cubic feet rather than in pounds, to
sizes. The values given are for average usage
facilitate calculations the heat gain per cubic
(see table footnotes). The ASRE Data Book
foot of outside air entering the space is listed in
defines average and heavy usage as follows:
Tables 10-8A and 10-8B for various inside and
Average usage includes installations not
outside air conditions. To determine the air
subject to extreme temperatures and where
change load in Btu per 24 hr, multiply the air
the quantity of food handled in the refrigerator
quantity in cubic feet per 24 hr by the appropri-
is not abnormal. Refrigerators in delicatessens
ate factor from Table 10-8 A or 10-8B.
and clubs may generally be classified under this
Where the ventilating air (air change) quantity
type of usage.
is given in cubic feet per minute (cfm), convert
Heavy usage includes installations such as
cfm to cubic feet per 24 hr by multiplying by
those in busy markets, restaurant and hotel
60 min and by 24 hr.
kitchens where the room temperatures are likely
Example 10-6. Three hundred cfm of air to be high, where rush periods place heavy loads
are introduced into a refrigerated space for on the refrigerator, and where large quantities
ventilation. If the inside of the cooler is main-
of warm foods are often placed in it.*
tained at 35° F and the outside dry bulb tem-
perature and humidity are 85° F and 50%, * The Refrigerating Data Book, Basic Volume,
respectively, determine the air change load in The American Society of Refrigerating Engineers,
Btu/24 hr. 1949, New York, p. 327.
COOLING LOAD CALCULATIONS 153

Example 10-7. A walk-in cooler 8 ft x Example 10-8. One thousand pounds of


IS x 10 ft high is constructed of 4 in. of cork-
ft fresh, lean beef enter a cooler at 55° F and are
board with 1 in. of wood on each side. The chilled to the cooler temperature of 35° F in
outside temperature is 95° F and the humidity 24 hr. Calculate the product load in Btu/24 hr.
is 50%. The cooler is maintained at 35° F and
the usage is average. Determine the air change Solution
loadinBtu/24hr. From Table 10-12, the
specificheat of lean
Solution. Since the walls of the cooler are beef above freezing = 0.75 Btu/lb/° F
approximately 6 in. thick (4 in. of corkboard
Applying Equation
and 2 in. of wood), the inside dimensions of the
10-7, the product load,
cooler are 1 ft less than the outside dimensions;
Btu/24 hr = 1000 x 0.75
therefore,
x (55 - 35)
Inside volume = 7 ft x 14 ft x 9 ft
= 1000 x 0.75 x 20
= 882 cu ft
= 15,000 Btu/24 hr
From Table 10-9A,
by interpolation, num- Notice that no time element is inherent in
ber of air changes per
Equation 10-7 and that the result obtained is
24 hr for cooler vol-
merely the quantity of heat the product will
ume of approximately
give off in cooling to the space temperature.
900cuft =19
However, since in Example 10-8 the product is
Total quantity of
to be cooled over a 24-hr period, the resulting
air change per 24 hr = Inside volume
x air changes heat quantity represents the product load for a
= 882 x 19 24-hr period. When the desired cooling time
= 16,758 cu ft/24 hr is less than 24 hr, the equivalent product load
From Table 10-8A, for a 24-hr period is computed by dividing the
heat gain per cubic heat quantity by the desired cooling time for the
feet = 2.49 Btu/cu ft product to obtain the hourly cooling rate and
Air change load = cu ft/24 hr then multiplying the result by 24 hr to determine
x Btu/cu ft the equivalent product load for a 24-hr period.
= 16,758 x 2.49 When adjusted to include these two factors,
= 41,727 Btu/24 hr Equation 10-7 is written:
10-15. Calculation the Product Load. When
WxCx(r,-r )x24hr
a product enters a storage space at a tempera-
ture above the temperature of the space, the
Q aav J (hr)
desired cooling time n*s
1
0°-8)

product will give off heat to the space until it


Example 104. Assume that it is desired to
cools to the space temperature. When the
beef in Example 10-8 in 6 hr rather than
chill the
temperature of the storage space is maintained in 24 hr. Determine the product load in
above the freezing temperature of the product, Btu/24 hr.
the amount of heat given off by the product in
cooling to the space temperature depends Upon Solution. Applying
1000 x 0.75
Equation 10-8, product
the temperature of the space and upon the
load, Btu/24 hr _ x (55 - 35) x 24
weight, specific heat, and entering temperature
6
of the product. In such cases, the space heat
= 60,000 Btu/24 hr
gain from the product is computed by the
following equation, (see Section 2-24): Compare this result with that obtained in
Q = W x C x(Ta -Tj (10-7) Example 10-8.
When a product is chilled and stored below
where Q = the quantity of heat in Btu
W = weight of the product (pounds) its freezing temperature, the product load
calculated in three parts:
is

C = the specific heat above freezing


(Btu/lb/° F) 1. The heat given off by the product in cooling

Tx = the entering temperature (° F) from the entering temperature to its freezing


Ta = the space temperature (° F) temperature.
154 PRINCIPLES OF REFRIGERATION

2. The heat given off by the product in solidi- To cool from freezing
fying or freezing. temperature to final
3. The heat given off by the product in cool- storage temperature,
applying Equation
ing from its freezing temperature to the final
10-7 500 x 0.37
storage temperature.
x[27-(-5)]
For parts 1 and 3, Equation 10-7 is used to 5920 Btu
determine the heat quantity. For part 1, Tx in Total heat given up
Equation 10-7 is the entering temperature of the by product (summa-
tion of 1,2, and 3) = 64,000 Btu
product, whereas Tz is the freezing temperature
of the product (Tables 10-10 through 10-13). Equivalent product
For part 3, Tx in Equation 10-7 is the freezing load for 24-hrperiod 64,000 x 24 hr
Btu/24 hr
temperature of the product and Tt is the final 12 hr
storage temperature. The heat quantity for = 128,000 Btu/24 hr
part 2 is determined by the following equation:
10-16. Chilling Rate Factor. During the early
Q = Wxh if (10-9) part of the chilling period, the Btu per hour load
on the equipment is considerably greater than the
where W = the weight of the product (pounds) average hourly product load as calculated in
h it = the latent heat of the product
the previous examples. Because of the high
(Btu/lb)
temperature difference which exists between the -

When the chilling and freezing are accom- product and the space air at the start of the
plished over a 24-hr period, the summation of chilling period, the chilling rate is higher and
the three parts represents the product load for the product load tends to concentrate in the
24 hr. When the desired chilling and freezing early part of the chilling period (Section 9-23).
time for the product are less than 24 hr, the Therefore, where the equipment selection is

summation of the three parts is divided by the based on the assumption that the product load
desired processing time and then multiplied by is evenly distributed over the entire chilling
24 hr to determine the equivalent 24-hr product period, me equipment selected will usually have
load. insufficient capacity to carry the load during
the initial stages of chilling when the product
Example 10-10. 500 pounds of poultry
load is at a peak.
F and are frozen and chilled
enter a chiller at 40°
to a final temperature of — 5° F for storage in
To compensate for the uneven distribution of

12 hr. Compute the product load in Btu/24 hr. the chilling load, a chilling rate factor is intro-
duced into the chilling load calculation. The
Solution
of the chilling rate factor is to increase the
effect
From Table 10-12,
product load calculation by an amount sufficient
Specific heat above
to make the average hourly cooling rate approxi-
freezing = 0.79 Btu/lb/° F
mately equal to the hourly load at the peak
Specific heat below
condition. This results in the selection of
freezing = 0.37 Btu/lb/° F
larger equipment, having sufficient capacity to
Latent heat = 106 Btu/lb
carry the load during the initial stages of chilling.
Freezing
Chilling rate factors for various products are
temperature = 27°F
listed in Tables 10-10 through 10-13. The
To cool poultry from factors given in the tables are based on actual
entering temperature tests and on calculations and will vary with the
to freezing tempera-
ratio of the loading time to total chilling time.
ture, applying Equa-
As an example, test results show that in typical
tion 10-7 500 x 0.79
- beef and hog chilling operations the chilling rate
x (40 27)
is 50% greater during the first half of the chill-
5135 Btu
To freeze, applying ing period than the average chilling rate for
Equation 10-9 500 x 106 the entire period. The calculation without the
53,000 Btu chilling rate factor will, of course, show the
COOLING LOAD CALCULATIONS 155

average chilling rate for the entire period. To 10-18. Containers and Packing Materials.
obtain this rate during the initial chilling period, When a product is chilled in containers, such as
itmuSt be multiplied by l.S. For convenience, milk in bottles or cartons, eggs in crates, fruit
the chilling rate factors are given in the tables and vegetables in baskets and lugs, etc., the
in reciprocal form and are used in the denomi- heat given off by the containers and packing
nator of the equation. Thus the chilling rate materials in cooling from the entering tempera-
factor for beef as shown in the table is 0.67 ture to the 'space temperature must be con-
0/1.5). sidered as a part of the product load. Equation
Where a chilling rate factor is used, Equation 10-7 is used to compute this heat quantity.
10-7 is written 10-19. Calculating the Miscellaneous Load.

W xC xjTi-TJ
the miscellaneous load consists primarily of the
Q ~ Chilling rate factor
(1 °" 10) heat given off by lights and electric motors
operating in the space and by people working
in the space. The heat given off by lights is
As a general rule chilling rate factors are not
used for the final stages of chilling from the 3.42 Btu per watt per hour. The heat given off
freezing temperature to the final storage tem- by electric motors and by people working in the
perature of the product. Too, chilling rate space is listed in Tables 10-15 and 10-16,

factors are usually applied to chilling rooms respectively. The following calculations are
only and are not normally used in calculation made to determine the heat gain from mis-

of the product load for storage rooms. Since cellaneous:

the product load for storage rooms usually Lights: wattage x 3.42 Btu/watt/hr x 24 hr
Electric motors factor (Table 10-1 5) x horse-
represents only a small percentage of the total :

load, the uneven distribution of the product power x 24 hr


load over the cooling period will not ordinarily People: factor (Table 10-16) x number of

cause overloading of the equipment and, there- people x 24 hr


10-20. Use of Safety Factor. The total cooling
fore, no allowance need be made for this
condition. load for a 24-hr period is the summation of the
10-17. Respiration Heat. Fruits and vegetables heat gains as calculated in the foregoing sections.

are still alive after harvesting and continue to It is common practice to add 5% to 10% to

undergo changes while in storage. The more this value as a safety factor. The percentage
important of these changes are produced by used depends upon the reliability of the informa-
respiration, a process during which oxygen
tion used in calculating the cooling load. As a

from the air combines with the carbohydrates general rule 10% is used.

in the plant tissue and results in the release of After the safety factor has been added, the

carbon dioxide and heat. The heat released is 24-hr load is divided by the desired operating

called respiration heat and must be considered time for the equipment to determine the average
as a part of the product load where considerable load in Btu per hour (see Section 10-2). The
quantities of fruit and/or vegetables are held in average hourly load is used as a basis for
storage. The amount of heat evolving from the equipment selection.
10-21. Short Method Load Calculations.
respiration process depends upon the type and
temperature of the product. Respiration heat
Whenever possible the cooling load should be
for various fruits and vegetables determined by using the procedures set forth
is listed in
Table 10-14. in the preceding sections of this chapter. How-
Since respiration heat given in Btu per
is
ever, when small coolers (under 1600 cu ft) are
pound per hr, the product load accruing from used for general-purpose storage, the product
load is frequently unknown and/or varies
respiration heat is computed by multiplying the
total weight of the product by the respiration somewhat from day to day so that it is not pos-
heat as given in Table 10-14, viz: sible tocompute the product load with any real
accuracy. In such cases, a short method of load
Q (Btu/24 hr) - Weight of product (lb) calculation can be employed which involves the
x respiration heat (Btu/lb/hr) use of load factors which have been determined
x24hr (10-11) by experience. When the short method of
156 PRINCIPLES OF REFRIGERATION
calculation is employed, the entire cooling load Average specific heat
is divided into two parts: (1) the wall gain load of vegetables (Table
and (2) the usage or service load. 10-11) = 0.9 Btu/lb/° F
The wall gain load is calculated as outlined Average respiration
in Section 10-13. The usage load is computed heat of vegetables

by the following equation:


(Table 10-14) = 0.09 Btu/lb/hr
Wall gain load:
Usage load = interior volume x usage factor Area x wall gain factor
(10-12) = 760 sq ft x 81 Btu/sq ft/24 hr
= 61,560 Btu/24 hr
Notice that the usage factors listed in Table Air change load:
10-17 vary with the interior volume of the cooler Inside volume
and with the difference in temperature between x air changes x Btu/cuft
the inside and outside of the cooler. Too, an = 1377cuft x 16.8
allowance is made for normal and heavy usage. x 1.69 Btu/cuft = 23,100 Btu/24 hr
Normal and heavy usage have already been Product load:
defined in Section 10-14. No safety factor is Temperature reduction
used when the cooling load is calculated by = Mxcx(r,-r1)
the short method. The total cooling load is
= 35001b x0.9Btu/lb/°F
divided by the desired operating time for the
x 10° F = 31,500 Btu/24 hr
equipment to find the average hourly load used Respiration heat

to select the equipment.


= Mx reaction heat x 24 hr
= 3500 x 0.09 Btu/lb/hr
Example 10-11. A walk-in cooler 18 ft x x24hr
10 x 10 ft high has 4 in. of corkboard insula-
ft 7,560 Btu/24 hr
tion and standard wall construction consisting Summation: 123,720 Btu/24 hr
of two layers of paper and 1 in. of wood on each Safety factor
side (total wall thickness is 6 in.). The tempera- (10%) =1 2,370 Btu
ture outside the cooler is 85° F. 35001b of Total cooling
mixed vegetables are cooled 10° F to the storage load = 136,100 Btu/24 hr
temperature each day. Compute the cooling
Required
load in Btu/hr based on a 16-hr per day opera-
cooling
ting time for the equipment. The inside
capacity
temperature is 40° F.
(Btu/hr) _ Total cooling load
Solution Desired running time
Outside surface area = 2 x 18 ft x 10 ft _ 136,100 Btu/24 hr
- 360 sq ft 16 hr
= 4 x 10 ft x 10 ft = 8,500 Btu/hr
= 400 sq ft
760 sq ft
Example 10-12. The dimensions of a
Inside volume (since
banana storage room located in New Orleans,
total wall thickness is
Louisiana are 22 ft x 32 ft x 9 ft. The walls
6 in., the inside dimen-
are 1 in. boards on both sides of 2 x 4 studs and
sions are 1 ft less than
insulated with 3$ in. granulated cork. The floor
the outside dimensions) 17ft x9ft x9ft and roof are of the same construction as the
1377 cu ft
walls. The floor is over a ventilated crawl space
Wall gain factor and the roof is exposed to the sun. The tempera-
(Table 10-18) (45° F
ture outside the storage room is approximately
TD and 4 in. insulation) 81 Btu/hr/sq ft
the same as the outdoor design temperature for
Air changes (Table the region. 30,000 lb of bananas are ripened at
10-9A) (by interpola- 70° F and then cooled to 56° F in 12 hr for
tion) 16.8 per 24 hr holding storage. Compute the Btu/hr cooling
Heat gain per cubic load. (Note: Because of the high storage tem-
foot of air (Table perature the evaporator will not collect frost and
10-8A)(50%RH) • 1.69 Btu/cuft the equipment is designed for continuous run.)
COOLING LOAD CALCULATIONS 157

Solution Required cooling capacity


Outside surface area (Btu/hr)
Ceiling = 704 sq ft Total cooling load
Walls and floor = 1676sqft Desired running time
Inside volume
(21 ft x 31 ft x 8 ft) - 5208 cu ft _ 1,476,860 Btu/24 hr

Outside design 24 hr
temperature for = 61,530 Btu/hr
New Orleans (Table
10-6) =*89°F Example 10-13. A chilling room 35 ft x
50 ft x 15 ft is used to chill 50,000 lb of fresh
Wall U factor beef per day from an initial temperature of
(Table 10-2) = 0.079Btu/hr/sqft/°F 100° F to a final temperature of 35° F in 18 hr.
Sun factor for roof Four people work in the chiller during the
(Table 10-7) (tar loading period. The lighting load is 1500 watts.
roofing) = 20°F The floor, located over an unconditioned space,
Air changes (Table is a 5 in. concrete slab insulated with 4 in. of
10-9 A) (by interpo- corkboard and finished with 3 in. of concrete.
lation) = 7/24hr The ceiling, situated beneath an unconditioned
Heat gain per space, isa 4 in. concrete slab with wood sleepers
cubic foot of air and insulated with 4 in. of corkboard. All of
change(Table 10-8A) = 1 .4 Btu/cu ft the walls are inside partitions adjacent to uncon-
(interpolated) ditioned spaces (90° F) except the east wall
Specific heat of which is adjacent to a 35° F storage cooler. Wall
bananas (Table construction is 4 in. cinder block insulated with
10-10) = 0.9Btu/lb/°F 4 in. of corkboard and finished pn one side with
plaster. Compute the cooling load in Btu per
Reaction heat of
hour based on a 16-hr operating time for the
bananas (Table
equipment.
10-14) = 0.5 Btu/lb/hr
Wall gain load: Solution
Area x U x TD x 24 hr Outside surface area
Ceiling Ceiling
704 x 0.079 x 53 x 24 (35 ft x50ft) = 1,750 sq ft
= 70,740 Btu/24 hr Floor
Walls and floor (35 ft x50ft) = 1,750 sq ft
1676 x 0.79 x 33 x 24 Walls (except
= 104,860 Btu/24 hr east)
Air change load: (120 ft x 15 ft) = 1,800 sq ft
Inside volume Inside volume
x air changes x Btu/cu ft (34 ft x 49 ft x
= 5208 x 7 x 1.4 Btu/cu ft 13.5 ft) - 22,491 cu ft
= 51,000 Btu/24 hr
Air changes
Product load:
(Table 10-9A) (by
Temperature reduction
=
_M
interpolation) 3.2 per 24 hr
x C xjTj-Tj) x24hr
Heat gain per
Chilling time (hr)
cubic foot (Table
_ 30,000 x 0.9 x 14 x 24 = 2.17 Btu/cu ft
10-8A)(50%RH)
12
Specific heat of
= 756,000 Btu/24 hr beef (Table 10-12) = 0.75 Btu/lb/° F
Respiration heat
= M
x reaction heat x 24 hr Chilling rate factor
(Table 10-12) = 0.67
= 30,000 x 0.5 x 24 = 360,000 Btu/24 hr
Summation: Occupancy heat
1,342,600 Btu/24 hr
gain (Table 10-16) = 900 Btu/hr/person
Safety factor (10%) 1 34,260 Btu Ceiling U factor
Total cooling load 1,476,860 Btu/24 hr (Table 10-3) = 0.069 Btu/hr/sqft/°F
158 PRINCIPLES OF REFRIGERATION
Floor U factor eously with the chilling peak. If the occupancy
(Table 10-3) = 0.066 Btu/hr/sq ft/°F occurred at any time other than at the peak,
Wall U factor the occupancy load could be ignored.
(Table 10-1) = 0.065 Btu/hr/sq ft/°F Example 10-14. Three thousand lug boxes
Wall gain load: of apples are stored at 35° F in a storage cooler
A x U xTD x 24 hr 50 ft x 40 ft x 10 ft. The apples enter the cooler
= 5300 x 0.067 x55 x24hr at a temperature of 90° F and at the rate of 200
= 468,700 Btu/24 hr lugs per day each day for the 15 day harvesting
Air change load: period. The walls including floor and ceiling
Inside volume are constructed of boards on both sides of
1 in.
x air changes x Btu/cu ft 2x4 studs and are insulated with 3$ in. of
=
22,491 x 3.2 x 2.17 rock wool. All of the walls are shaded and the
= 156,000 Btu/24 hr ambient temperature is 85° F. The average
Product load: weight of apples per lug box is 59 lb. The lug
M x C x (T %
- Tx) x 24 boxes have an average weight of 4.5 lb and a
Chilling time (hr) x chilling rate factor specific heat value of 0.60 Btu/lb/° F. Determine

50,000 x 0.75 x 65 x 24 the average hourly cooling load based on 16 hr


x 0.67 operating time for the equipment.
18
= 4,850,700 Btu/24 hr Solution
Miscellaneous:
Outside surface
Occupancy = 5800 sq
area ft
= No. of people x factor x 24 hr
= 4 x900 x24 Inside volume
= 86,400 Btu/24 hr (49 ft x 39 ft x 9 ft) = 17,200 cu ft

Lighting load Wall U factor


= watts x 3.4 x 24 hr (Table 10-2) = 0.072 Btu/hr/sq ft/ F
1500 x 3.4 x 24 = 122,400 Btu/24 hr Air changes
Summation: (Table 10-9A) (by
5,684,200 Btu/24 hr
Safety factor (10%)
interpolation) = 3.7 per 24 hr
= 568,420 Btu Heat gain per
6,252,620 Btu/24 hr cubic foot (Table
10-8 A) = 1.86 Btu/cu ft
Required cooling capacity (Btu/hr)
Specific heat of
_ Total cooling load
apples (Table 10-10) = 0.89 Btu/lb/° F
Desired operating time
Respiration heat
6,252,620 Btu/24 hr of apples (Table
18 hr 10-14) (by interpo-
= 390,800 Btu/hr lation) = 0.025 Btu/lb/hr
Wall gain load:
Note: (1) Since there is no temperature
differential across the east wall, there is no gain A x U x TD x 24 hr
or loss of heat through the wall and the wall
= 5800 x 0.072 x 50 x 24
is ignored in the cooling load calculations
= 501,100 Btu/24 hr
However, this wall should be insulated with at Air change load:
least the minimum amount of insulation to
Inside volume x air changes
prevent excessive heat gains through this wall
x Btu/cu ft

in the event that the adjacent refrigerated space


= 17,000 x 3.7 x 1.86

should become inoperative. (2) Since the TD


= 11 7,000 Btu/24 hr
across all the walls, including floor and ceiling,
Product load:
Temperature reduction
the same and since the difference in the wall
_M
is
x C x (Tt - Tx)
U factors is slight, the walls may be lumped
together for calculation. (3) Although the Chilling rate factor
workmen are in the space for only 4 hr each Apples
day for the purpose of load calculation, they are (200 x 59 lb) x 0.89 x 55
assumed to be in the cooler continuously. This 0.67
is because their occupancy occurs simultan- = 862,100 Btu/24 hr
COOLING LOAD CALCULATIONS 159

Fig. 103

Lug boxes and finished with 4 in. of concrete. The floor is


(200 x 4.5 lb) x 0.6 x 55 over a ventilated crawl space. Roof is exposed
0.67 to the sun. The equipment room is well venti-
= 44,300 Btu/24 hr lated so that the temperature inside is approxi-
Respiration mately the outdoor design temperature for the
= M
x reaction heat x 24 hr region. The storage room is maintained at 0° F,

= (3000 x 59 lb) x 0.025 x 24 whereas the temperature in the freezer is 10° F.
The locationHouston, Texas. Determine the
is
= 106,200 Btu/24 hr
average hourly refrigeration load based on 20 hr
Summation 1,630,700 Btu/24 hr per day operating time for the equipment.
Safety factor (10%) = 163,100 Btu Solution
Total cooling load = 1,793,800 Btu/24 hr Outside surface area
_ Roof
Average hourly load = Total cooling load (9 ft + 14 ft) =126sqft
Running time
Floor
_ 1,793,800 Btu/24 hr
(9 ft x 14 ft) =126sqft
16 hr N and E partitions
= 112,100 Btu/24 hr (23 ft x 10 ft) =230sqft
Note: Load and equipment
calculation
S and W partitions

selection is based on maximum loading which (23 ft x 10 ft) =230sqft


Inside volume
occurs on the fifteenth day.
(8ftx9ft x 13ft) = 936 cu ft
Example 10-15. Twenty-two thousand Summer outdoor
pounds of dressed poultry are blast frozen on design temperature 92° F
hand trucks each day (24 hr) in a freezing tunnel
14 ft x 9 ft x 10 ft high (see Fig. 10-3). The
U factors
Roof
poultry precooled to 45° F before entering the
is
(Table 10-3) = 0.036 Btu/hr/sq ft/° F
freezer where it is frozen and its temperature
Floor
lowered to 0° F for storage. The lighting load
(Table 10-3) = 0.035 Btu/hr/sq ft/° F
is 200 watts. The hand trucks carrying the
poultry total 1400 lb per day and have a specific
N and E partitions
(Table 10-2) = 0.035 Btu/hr/sq ft/° F
heat of 0.25 Btu/lb/° F. The partitions adjacent
to the equipment room and vestibule are con-
S and W partitions
(Table 10-2) =0.12 Btu/hr/sq ft/° F
structed of 6 in. clay tile insulated with 8 in. of
corkboard. Partitions adjacent to storage
Roof sun factor
cooler are 4 in. clay tile with 2 in. corkboard
(Table 10-7) = 20°F
insulation. The roof is a 6 in. concrete slab Air changes
insulated with 8 in. of corkboard and covered (Table 10-9B) 13.5 per 24 hr
with tar, felt, and gravel. The floor is a 6 in. Heat gain per cubic foot
concrete slab insulated with 8 in. of corkboard (Table 10-8B) = 3.56 Btu/cu ft
160 PRINCIPLES OF REFRIGERATION
—»-N Freezing = M
x latent heat
-32.5*-
Poultry = 22,000 x 106
= 2,332,000 Btu/24 hr
Lochtrs
Miscellaneous:
Cold O'F
Lockers Lighting: 200 watts x 3.4
18' storage
38*F Lockers >
Conditioned
Btu/watt/hr x 24 hr
Freezer
= 16,300 Btu/24 hr
Lockers N- space, 80* F
Summation:
Cold storage 3,014,100 Btu/24 hr
38"F
Safety factor (10%) = 301,400 Btu

Total cooling
Fig. 10-4. Frozen food locker plant. load = 3,315,500 Btu/24 hr
Average hourly
Specific heat of poultry
load _ 3,315,500 Btu/24 hr

(Table 10-12) 20 hr (running time)


Above freezing = 165,775 Btu/hr
= 0.79 Btu/lb/° F
Below freezing Example 10-16. A
frozen food locker plant
= 0.37 Btu/lb/° F 18 ft x 32.5 ft x 10 ft, containing 353 indi-
Latent heat of poultry vidual lockers and an 8 ft freezing cabinet, is
(Table 10-12) located in Tulsa, Oklahoma (see Fig. 10-4).
= 106 Btu/lb The north and west wall are constructed of 8 in.
Freezing temperature clay tile with 6 in. of corkboard insulation.
27° F South and east walls are 4 in. clay tile with 4 in.
Wall gain load: of corkboard insulation. The roof is exposed to
^xt7xri)x24hrs the sun and is constructed of 4 in. of concrete
Floor insulated with 8 in. of corkboard and covered
126 x 0.035 x 102 x 24 with tar, felt, and gravel. The floor is a 5 in.
= 10,800 Btu/24 hr concrete slab poured directly on the ground,
Roof insulated with 8 in. of corkboard, and finished
126 x 0.036 x (102 +20) x 24 with 3 in. of concrete. The product load on
= 13,300 Btu/24 hr cabinet freezer is 700 lb of assorted meats per
South and west partition day. (Standard practice is to allow for 2 lb of
230 x 0.035 x 102 x 24 product per locker per day.) In this instance,
= 19,700 Btu/24 hr the product is precooled to 38° F before being
North and east partition placed in the freezer. The lighting load is 500
230 x 0.12 x 10 x 24 watts and the average occupancy is three people.
= 6,600 Btu/24 hr Determine the average hourly refrigerating rate
Air change load: based on a 20-hr equipment operating time.
volume x
Inside air changes
x Btu/cu ft Solution
- 936 x 13.5 x 3.56 Outside surface area
= 45,000 Btu/24 hr Roof (18 ft x 32.5 ft)

Product Load: 585 sq ft

Temperature reduction Floor (18 ft x 32.5 ft)


=M x C x(rs -TJ 585 sq ft
Poultry South and east
_ 22,000 x 0.79 x (45 - 27) partition (50 .5 ft x 10 ft)
0.67 (chilling factor) 505 sq ft

= 302,700 Btu/24 hr North partition (18 ft x 10 ft)


22,000 x 0.37 x (27 - 0) 180 sq ft
- 219,700 Btu/24 hr West wall (32 .5 ft x 10 ft)
Trucks 325 sq ft
1,400 x 0.25 x (92 - 0) Inside volume
0.67 (16 ft x 30.5 ft x 8 ft)
.
48,000 Btu/24 hr 3904 cu ft
COOLING LOAD CALCULATIONS 161

Design outdoor temperature Product load:


(Table 10-6) = 92°F Temperature reduction
=M x C x (Tt - TJ
Design ground temperature
700 x 0.8 x (38 - 28)
(Table 10-6A) = 65° F = 5600 Btu/24 hr
Roof sun factor 700 x 0.4 x (28 - 0)
(Table 10-7) = 20°F 7840 Btu/24 hr
Freezing
West wall sun factor
- 6° F
= M x latent heat
(Table 10-7)
= 700 x 100 70,000 Btu/24 hr
U factors Miscellaneous:
Roof (Table 10-3) Lights
= 0.036 Btu/hr/sq ft/ F = 500 watts x 3.4 Btu/hr
Floor (Table 10-3) x 24 hr = 40,800 Btu/24 hr
= 0.046 Btu/hr/sq ft/° F Occupancy
North and west walls
= 3 x 1300 x 24
(Table 10-1) = 0.034 Btu/hr/sq ft/° F = 93,600 Btu/24 hr

South and east walls Summation: = 531,560 Btu/24 hr


(Table 10-1) = 0.066 Btu/hr/sq ft/° F Safety factor
(10%) = 53,150 Btu
Air changes Total cooling
(see Note 2 of Table 10-9B) load = 584,710 Btu/24 hr
= 12.6(6.3 x 2) Average hourly
per 24 hr
Heat gain per cubic foot
load _ 584,710 Btu/24 hr
20 hr
(Table 10-8B) = 3.0 Btu/cu ft
= 29,240 Btu/hr
Specific heat (average for meat) Load on freezer 91,780 Btu/24 hr
Above freezing = 0.8 Btu/lb/° F 20 hr
Below freezing = 0.4 Btu/lb/° F (product load
Latent heat
only, including
(average) = 100 Btu/lb
10% safety factor)
Freezing tempera- = 4,590 Btu/hr
ture (average) = 28° F Load on locker
Occupancy factor =
room only 29,240 Btu/hr
= 1300 Btu/hr/person
(total load less - 4,590 Btu/hr
Wall Gain Load: freezer load) = 24,650 Btu/hr
A x U x TD x 24
Example 10-17. Five hundred gallons of
Roof 585 x 0.036
partially frozen ice cream at 25° F are entering
x (92 + 20) x 24
a hardening room 10 ft x 15 ft x 9 ft each day.
= 56,600 Btu/24 hr
Hardening is completed and the temperature of
Floor 585 x 0.046 x 65 x 24 the ice cream is lowered to —20° F in 10 hr.
= 42,000 Btu/24 hr The walls, including floor and ceiling, are
South and east insulated with 8 in. of corkboard and the over-
partition 505 x 0.066 x 38 x 24 all thickness of the walls is 12 in. The ambient
= 30,400 Btu/24 hr temperature is 90° F and the lighting load is 300
North partition 180 x 0.034 x 80 x 24 watts. Assume the average weight of ice cream
= 11,750 Btu/24 hr is 5 lb per gallon, the average specific heat below

West wall 325 x 0.034 freezing is 0.5 Btu/lb/°F, and the average
x (92 + 6) x 24 latent heat per pound is 100 Btu.* Determine
= 26,000 Btu/24 hr the average hourly load based on 18 hr opera-
tion.
Air change load:
Insidevolume x air changes * These values are variable and depend upon the
x Btu/cu ft composition of the mix, the percent of overrun,
= 3904 x 12.6 x 3.01 and the temperature of the ice cream leaving the
= 147,570 Btu/24 hr freezer.
162 PRINCIPLES OF REFRIGERATION
Solution Solution
Outside surface Outside surface area
area = 750 cu ft = 636 sq ft

Inside volume Inside volume


(8 ft x 13 ft x 7 ft) (9 ft x lift x 8 ft)

= 728 cu ft 792 cu ft

Wall gain factor


Wall gain factor
(Table 10-18) 81 Btu/sq ft/24 hr
(Table 10-18) 99 Btu/sq ft/24 hr
Air changes
Usage factor
(Table 10-17) 50 Btu/cu ft/24 hr
(Table 10-9B)
(interpolated) = 16.7 per 24 hr Wall gain load:
Heat gain per cubic foot area x wall gain factor
(Table 10-8B) = 636 x 81 51,500 Btu/24 hr
(50 %RH) =3.88 Btu/cu ft Usage load:
Wall gain load: Inside volume x
area x wall gain factor usage factor
= 750 x 99 = 74,250 Btu/24 hr = 792 x 50 = 39,600 Btu/24 hr

Air change load: Total cooling load = 91,100 Btu/24 hr


Inside volumex air changes Average hourly load
x Btu/cu ft / 91,100 Btu/24 hr
= 728 x 16.7 x 3.88 )" 5,700 Btu/hr
= I 16 hr
47,170 Btu/24 hr
Product load:
Temperature reduction PROBLEMS
— M x C(T S
- 7\) x 24 hr
Chilling time 1. Acooler wall 10 ft by 18 ft is insulated with
(500 x 5) x 0.5 x (25 20) x 24 the equivalent of 3 in. of corkboard. Compute
the heat gain through the wall in Btu/24 hr if the
10
inside temperature is 37° F and the outside
= 135,000 Btu/24 hr
temperature is 78° F. Ans. 165,312 Btu/24 hr
Freezing
_ Mx latent heat x 24 2. The northwall of a locker plant is 12 ft by
Freezing time 60 ft and constructed of 8 in. hollow clay tile
is

insulated with 8 in. of corkboard. The locker


_ (500 x 5) x 100 x 24
plant is located in Houston, Texas and the
10 inside temperature is maintained at 0° F. Deter-
= 600,000 Btu/24 hr mine the heat gain through the wall in Btu/24 hr.
Miscellaneous load: Ans. 54,000 Btu/24 hr
Lighting:
300 watts x 3.4 x 24 3. A cold storage warehouse in Orlando,
= 24,480 Btu/24 hr Florida has a 30 ft by 50 ft flat roof constructed
Summation: of 4 in. of concrete covered with tar and gravel
880,900 Btu/24 hr
and insulated with the equivalent of 4 in. of
Safety factor (10%) = 88,090 Btu
corkboard. If the roof is unshaded and the
968,990 Btu/24 hr inside of the warehouse is maintained at 35° F,
Average hourly load compute the heat gain through the roof in
(968,990/1 8 hr) = 53,800 Btu/hr Btu/24 hr. Ans. 181,330 Btu/24 hr

4. Asmall walk-in cooler has an interior volume


Example 10-18. A cooler 10 ft x 12 ft x 9 of 400 cu ft and receives heavy usage. If the
ft is used for general purpose storage in a
inside of the cooler is maintained at 35° F and
grocery store. The cooler is maintained at 35° F the outside design conditions are 90° F and 60%
and the service load is normal. The walls are relative humidity, determine the air change load
insulated with the equivalent of 4 in. of cork- Ans. 50,300 Btu/24 hr
in Btu/24 hr.
board and the ambient temperature is 80° F.
Determine the cooling load in Btu per hour 5. A frozen storage room has an interior volume
based on a 16 hr operating time. of 2000 cu ft and is maintained at a temperature
COOLING LOAD CALCULATIONS 163

of -10° F. The usage is light and the outside The beef is to be frozen and its temperature
design conditions (anteroom) are 50° F and 70 % reduced to 0° F in 5 hr. Compute the product
relative humidity. Compute the air change load load. Ans. 267,900 Btu/hr
in Btu/24 hr. Arts. 16,100 Btu/24 hr
8. Fifty-five hundred crates of apples are in

6. Five thousand pounds of fresh, lean beef storage at 37° F. An additional 500 crates enter
enter a chilling cooler at 100° F and are chilled the storage cooler at a temperature of 85° F and
to 38° F in 24 hr. Compute the chilling load in are chilled to the storage temperature in 24 hr.
Btu/24 hr. Ans. 347,000 Btu/24 hr The average weight of apples per crate is 60 lb.
The crate weighs 10 lb and has a specific value
7. Five hundred pounds of prepared, packaged of 0.6Btu/lb/°F. Determine the total product
beef enter a freezer at a temperature of 36° F. load in Btu/24 hr. Ans. l,679,80aBtu/24 hr
valve or some other liquid level control (Fig.
11-1). The vapor accumulating from the boiling
action of the refrigerant is drawn off the top
by the action of the compressor. The principal
advantage of the flooded evaporator is that the
inside surface of the evaporator is always
completely wetted with liquid, a condition that
produces a very high rate of heat transfer. The
II principal disadvantage of the flooded evapo-
rator is that it is usually bulky and requires a
relatively large refrigerant charge.

Evaporators Liquid refrigerant is fed into the dry-expan-


sion evaporator by an expansion device which
meters the liquid into the evaporator at a rate
such that all the liquid is vaporized by the time
it reaches the end of the evaporator coil (Fig.
11-2). For either type, the rate at which the
1 1 —1 . Types of Evaporators. As stated pre- liquid is fed into the evaporator depends upon
viously, any heat transfer surface in which a the of vaporization and increases or
rate
refrigerant is vaporized for the purpose of decreases as the heat load on the evaporator
removing heat from the refrigerated space or increases or decreases. However, whereas the
material is called an evaporator. Because of flooded type is always completely filled with
the many different requirements of the various liquid, the amount of liquid present in the dry-
applications, evaporators are manufactured in expansion evaporator will vary with the load on
a wide variety of types, shapes, sizes, and the evaporator. When
the load on the evapo-
designs, and they may be classified in a number rator amount of liquid in the evapo-
is light, the
of different ways, such as type of construction, rator is small. As the load on the evaporator
operating condition, method of air (or liquid) increases, the amount of liquid in the evaporator
circulation, type of refrigerant control, and increases to accommodate the greater load.
application. Thus, for the dry-expansion evaporator, the
1 Flooded and Dry-Expansion Evapora-
1-2. amount of liquid-wetted surface and, therefore,
tors. Evaporators fall into two general cate- the evaporator efficiency, is greatest when the
gories, flooded and dry expansion, according to load is greatest.
their operating condition. The flooded type is 11-3. Types of Construction. The three
always completely filled with liquid refrigerant, principal types of evaporator construction are:
the liquid level being maintained with a float (1) bare-tube, (2) plate-surface, and (3) finned.

Fig. Il-I. Flooded evaporator.


Notice accumulator and float
Liquid from control. Circulation of the
receiver
refrigerant through the coil
Float control
is by gravity. The vapor accu-

mulated from the boiling action


inthe coil -escapes to the top
of the accumulator and is
drawn off by the suction of the
compressor.

164
EVAPORATORS l*S

Bore-lube and plate-surface evaporators are


sometimes dawned together as prime-surface
evaporaion in thai the entire surface of both
these types u more or leu in contact with the
vaporiring refrigerant inside. With the finned
evaporator, the refrigcran(<cafTying tube are
the only prime surface. The fins themselves are
not rilled wilh Tcfrigerani and are, therefore,
only secondary hat transfer surfaces whose
function J* (o pick up heat from the surrounding
air and conduct it to the refrigerant-carrying
tubes.
< U 'c
Atthough prune-surface evaporate** of both
(a)
the bare-tube and plait-surface types give salt*
factory service on a wide variety of applications
operating in any temperature range, they are
most frequently applied to applications where
the space temperature is maintained below
34* F and frost accumulation on the evaporator
surface cannot be readily prevented. Frost
accumulation on prime-surface evaporaion
does not affect the evaporator capacity to use
extent thai doc* on finned coils. Further'
it

more, most prime surface evaporators, particu-


larly the fiate-turface type, are easily cleaned
and can be readily defrosted manually by either
brushing or scraping off the frost accumulation.
This can be accomplished without interrupting
(be refrigerating process and jeopardizing the
c M
T
d6a

quality of the refrigerated product. Flf. 11-3. Common dmgm far b*rt-tub* «H».
(ML ftar*~Tub* Evaporator*. Bare- lube (o> fin cignf eo+l. {&} C»*»l trwnewi* coil.

evaporators a re usually constructed of cither steel


pipe or copper tubing. Steel pipe is used for
luge evaporators and for evaporators to be niton intended for use with refrigerants other
employed with ammonia, whereas copper tubing than ammonia. Bare-tube coils are available
it utilized in the manufacture of smaller evepo* tn a number of sues, shapes, and designs, and
are usually custom made lo the individual
Liquid *nm Rtfnetrtnt application. Common shapes for bare-tube
ntctiw j^Ro* contra* coils are fiat zigzag and oval trombone, as
shown in Fig. I!-), Spiral bare-tube coils are
3 often employed for liquid chilling.
11-5. Plate-Surface Evaporators. Plale-sur*
face evaporators arc of several lypes. Some
3 constructed of two fiat sheets of metal so
are

embossed and welded together as to provide a


3> path for refrigerant flow between the two sheets
Mr Mb (Fig. 11-4). This particular type of plate-
urfacc evaporator is widely uaed in household
Fig. 11-5. Dry-wpvtslun aviporuer. Liquid r#-
refrigerators and home freezers because it is

frlftrtftl refwrim prof r«tii inly u It flown through easily cleaned, economical to manufacture, and
OOll Ifld l»*yet toil u i vapor. F«cler bulb cpnireU can be readily formed into any one of the various
rata of now through tha orrflca at tha flow control. shapes required (Fig, 11-5).
(M PRINCIPLES OF REFRIGERATION

Flf. MM. Standard lerpenllne plate evaporator, (Courtety Kold-Hold Division —Truuer Manufacturing, Inc.)

Rg. 11-5. Soma


^^ r
typical iheocs «raJl*b!e in pfaie-typ*
mmm
eviporiion. (Courtesy D«s- Product*. Inc.)

(A) Outside jacket Of plate. Heavy, olnctrkally (E) Fitting whir* vacuum ll drawn and than permajv
wddfid steel. Smooth surface, ontlv (tiled.
(B) Continuous steel tubing through which refrlg- (F) Vacuum ip«« In dry pit to Spice in hold-over
eratit passei. K(
p.| contains lutectk solution under vacuum. No
{C) Inlet from compressor, maintenance required due. CO i!urdy, limp In con-
(D> Outfit to compressor. Copper connections for struct Jon.No moving parti; no th ing to wear or get
all refrigerant* except ammonia where stiel cormee- out of order; no service neeusiry.
lions ere vied.

Fig. II-*, Plate-type evapc-retor, (Courtesy Dolt Refrigerating Company.}


EVAPORATORS 1*7

Another type of plalc-surfacc evaporator plates is controlled by the melting point of the
consuls of formed tubing installed between two eutectic solution.
metal plates which arc welded together at the Plate-type evaporators may be used singly or
edges (Fig. It -6). In order to provide good in banks. Figure 11-8 illustrates how the plates
thermal contact between the welded plated tnd can be grouped together for ceiling mounting in
the tubing carrying the refrigerant, live space holding rooms, locker plants, freezers, etc. The
between the plates is either filled with a eutcciic plates may be manifolded for parallel flow of
solution or evacuated so that the pressure of the the refrigerant (Fig. 11-9) or they may be
atmosphere exerted on the outside surface of connected for series flow,
the plates holds the plates firmly against the Plate-surface evaporators provide excellent
tubing inside. Those containing the eutectic shelves in freezer rooms and similar applications
Solution are especially useful where a holdover (Fig. 11-10). They are also widely used a*
capacity is required. Many are used on refriger- partitions in freezers, frozen food display cases,
ated trucks, let such applications, the plates are Ice cream cabinets, uhIji fountains, etc. Plate
mounted either vertically or horizontally from evaporators are especially useful for liquid
the ceiling or walls of the truck { Fig. 1 1-7) and cooling installations where unusual peak load
are usually connected to a central plant refriger- conditions are encountered periodically. By
ation system while the trucks are parked at the building bank on the surface of the
up an ice

terminal during the night. The refrigerating plain during periods of light loads, a holdover
capacity thui stored in the eutectic solution is refrigerating capacity is established which will
sufficient to refrigerate the product during the help be refrigerating equipment carry the load
i

oral day's operations. The temperature of the through the heavy or peak conditions Fig, 11-11),

Pig, 1 1*1. Frtmitr plitu Iniullad In whottHlt Fc* crttm truck body, (Court aty Do4« H«tri| •r»tln| Company.)
IfiS PRINCIPLES OF REFRIGERATION

Flj. 11-$. P1»te b»nks employed in I ow MfnpcnLu r* itor-ifi roomt. (Ccmttmy Dqfa ftcfrt|enuifl| Company.)

Since this allows Che use of smaller capacity be connected to the tubing in such a manner
equipment than would ordinarily be required that good thermal contact between the fins and
by the peak load, a savings « affected in Initial the tubing is assured. In some instances, the
cost and, usually, also in operating expenses, flni are soldered directly to [he i ubing. In others,
1 1-6. Finned Evaporators. Finned eoib in the fins are slipped over ihe tubing and the
bare- tube coils upon which metal plates or fins tubing is expanded by pressure or some such
have been insialled (Fig, 13-15), The fins, means so that the fins bite imo the tube surface
serving as secondary heat-absorbing surfaces, and establish good thermal contact. A variation
have the effect of increasing the over-all surface of the latter method is to flare the fin hole
area of the evaporator, thereby improving its slightly to allow the fin to slip over the tube.
efficiency. With bare-lube evaporators, much After the fin is installed, the flare is straightened
of the air that circa kites
over the coil passes and the fin is securely locked to the tube.
through the open spaces between Ihe tubes and Fin Ni« and spacing dej^eiid in part on the
docs not come in contact with the cm surface. I particular type of application for which the coil
When added to a coil, the fins extend
fins arc is designed, The size of the tube determines the
out imo open spaces be (ween the tubes and
the size of the fin. Small tubes require small fins.
act as heat collectors. They remove heat from As the size of the tube increases, the size of the
that portion of the air which would not ordin- fin may be effectively increased, Fin spacing
arily conic in contact with the prime surface varies from one to fourteen fins per inch,
and conduct it back to the tubing, depending primarily on the operating tempera-
It is evident that to be effective the fins must ture of the coil.
fVAfORATOAS 16?

Fl|» 1 1-*. Ptau bank with pluu minifeJdtd for ptraJM rrfriftnr* ftow. Plat** may ttto b« Kmn«t*d for
ien« tlow. (Cduntiy Kold-Hdd Dickon—Trinwr H»nuf»ctur1ni, '"t-J

Fig. tl-10. P1*» fnpQjiwn


implayid u frtenr indvti.
Not* thu plitM »r* imngtdi
for Mhti ndrtftnni flow.
{Courtesy KoJd-Hold Division
—Trwwr Minuhcturinf , Jnc)
170 PRINCIPLES OF AEFAKJEAATTON

Ft t. 11.11. let-Cat Rtfrttar*-


tion hoMov«r- capacity It etnb-
li*h*d by buii^.ftf up tank
of l« on plite «aipenun,
(Court**)* Dol* Rafrlfarilinf
Cwnptflf.)

Frost accumulation on air-cooling colls restricting the air circuit son over the coil

operating at low temperatures is unavoidable, unnecessarily.


and since accumulation on finned coils
any frost Since their capacity is affected more by frost

tends to restrict the air passages between the accumulation than any other type of evaporator,
lint and to retard air circulation through the finned coils ore best suited to air-cooling applica-
coil, evaporators designed for tow temperature tion where the temperature is maintained above
application! must have wide fin spacing (two 34' F. When tinned soils are used for low
or Ihree fins per inch) in order to minimize the temperature operation, some means of defrost*
danger of blocking air circulation. On the ing the coil at regular inter val* must be provided.
other hand, coils designed for air conditioning This may be accomplished automatically by
and other installation* where the coil Optra ICi several means which are discussed in another
it temperatures high enough so that no frost chapter
accumulates On the coil surface may have as Because of the fins, finned coils have more
many as fourteen fins per inch. surface area per unit of length and width than
When air circulation over finned coils is by prime-surface evaporators and can therefore
gravity, it is important that the coil offer as little be built more compactly. Generally, a finned
resistance lo air How as is possible; therefore, coil will occupy less space than either a bare-
in general, fin spacing should be wider for lube or plate-surface evaporator Of* the same
natural convection coil* than for coils employing capacity. Hut provides for a considerable
fans. savings in space and makes tinned coils ideally
has been determined that a definite relation-
It suited for use with funs a* In reed convection
ship exists between the inside and outside units.
surfaces of an evaporator . Since external finning 11-7. Evaporator Capacity. The capacity of

affects only the outside surface, I he addition of any evaporator or cooling coil is the rate at
(ins beyond a certain limit will not materially which heat will pass th rough the evaporator
increase the capacity of the evaporator. In walls from the refrigeruk-n ipiioe or product to
fad. in some instances, excessive finning may the vaporizing liquid inside and is usually
actually reduce the evaporator capacity by expressed in Blu per hour An evaporator
EVAPORATORS 171

selected for any specific application must always where V = the over-all conductance factor in
have sufficient capacity to allow the vaporizing Btu/hr/sq ft/ F D
refrigerant to absorb heat at the rate necessary
ft = the conductance factor of the inside
to produce the required cooling when operating surface film in Btu/hr/sq ft of inside
at the design conditions. surface/" FD
Heat reaches the evaporator by all three LjK = resistance to heat flow offered by
methods of heat transfer. In air-cooling applica- metal of tubes and fins
tions most of the heat is carried to the evapo- /„ =.the conductance factor of the out-
rator by convection currents set up in the side surface film in Btu/hr/sq ft of
refrigerated space either by action of a fan or by outside surface/" F D
gravity circulation resulting from the difference R = ratio of outside surface to inside
in temperature between the evaporator and the surface
space. Too, some heat is radiated directly to
the evaporator from the product and from the Since a high rate of heat transfer through the
wall of the space. Where the product is in evaporator walls is desirable, the U or conduct-
thermal contact with the outer surface of the ance factor should be as high as possible.
evaporator, heat is transferred from the product Metals, because of their high conductance
to the evaporator by direct conduction. This factor, are always used in evaporator construc-
is always true for liquid cooling applications tion. However, a metal which will not react
where the liquid being cooled is always in with the refrigerant must be selected. Iron, steel,
contact with the evaporator surface. However, brass, copper, and aluminum are the metals
circulation of the cooled fluid either by gravity most commonly used. Iron and steel are not
or by action of a pump is still necessary for affected by any of the common refrigerants,
good heat transfer. but are apt to rust if any moisture is present in
Regardless of how the heat reaches the out- the system. Brass and copper can be used with
side surface of the evaporator, it must pass any refrigerant except ammonia, which dis-
through the walls of the evaporator to the solves copper. Aluminum may be used with
refrigerant inside by conduction. Therefore, any refrigerant except methyl chloride. Mag-
the capacity of the evaporator, that is, the rate nesium and magnesium alloys cannot be used
at which heat passes through the walls, is deter- with the fluorinated hydrocarbons or with
mined by the same factors that govern the rate methyl chloride.
of heat flow by conduction through any heat Of the three factors involved in Equation
transfer surface and is expressed by the formula 11-2, the metal of the evaporator walls is the
Q =A x U x D (11-1)
least significant. The amount of resistance to
heat flow offered by the metal is so small,
where Q = the quantity of heat transferred in
especially where copper and aluminum are
Btu/hr
concerned, that it is usually of no consequence.
A = the outside surface area of the
Thus, the U factor of the evaporator is deter-
evaporator (both prime and finned)
mined primarily by the coefficients of con-
U = the conductance factor in
over-all
ductance of the inside and outside surface
Btu/hr/sq ft of outside surface/" F D
films.
D = the logarithmic mean temperature
In general, because of the effect they have on
difference in degrees Fahrenheit
the inside and outside film coefficients, the value
between the temperature outside the
evaporator and the temperature of
of U for an evaporator depends on the type
of coil construction and the material used, the
the refrigerant inside the evaporator
amount of interior wetted surface, the velocity
11-8. Uor Over-All Conductance Factor.
of the refrigerant inside the coil, the amount
The resistance to heat flow offered by the evapo-
of oil present in the evaporator, the material
rator walls is the sum of three factors whose
being cooled, the condition of the external
relationship is expressed by the following:
surface, the fluid (either gaseous or liquid)
1 _R L 1
velocity over the coil, and the ratio of inside
+
U~fi K +f (11-2)
to outside surface.
172 PRINCIPLES OF REFRIGERATION
Heat transfer by conduction is greater through Any increase in the turbulence of flow either
liquids than through gases and the rate at which inside or outside the evaporator will materially
the refrigerant absorbs heat from the evaporator increase the rate of heat transfer through the
walls increases as the amount of interior wetted evaporator walls. In general, internal turbu-
surface increases. In this respect, flooded lence increases with the difference in temperature
evaporators, since they are always completely across the walls of the tube, closer spacing of
filled with liquid, are more efficient than the the tubes, and the roughness of the internal tube
dry-expansion type. This principle also applies surface. In some instances, heat transfer is

to the external evaporator surface. When the improved by internal finning.


outside surface of the evaporator is in direct Outside flow turbulence is influenced by fluid
contact with some liquid or solid medium, the velocity over the coil, tube spacing, and the
heat transfer by conduction to the outside shape of the fins.

surface of the evaporator is greater than when 11-9. The Advantage of Fins. The advantage
air is the medium in contact with the evaporator of finning depends on the relative values of the
surface. coefficients of conductance of the inside and
Any fouling of either the external or internal outside surface films and upon R, the ratio of
surfaces of the evaporator tends to act as ther- the outside surface to the inside surface. In any
mal insulation and decreases the conductance instance where the rate of heat flow from the
factor of the evaporator walls and reduces the inside surface of the evaporator to the liquid
rate of heat transfer. Fouling of the external refrigerant is such that it exceeds the rate at
surface of air-cooling evaporators is usually which heat passes to the outside surface from
caused by an accumulation of dust and lint the cooled medium, the over-all capacity of the
from the air which adheres to the wet coil evaporator is limited by the capacity of the
surfaces or by frost accumulation on the coil outside surface. In such cases, the over-all
surface. In liquid-cooling applications, fouling value of U can be increased by using fins to
of the external tube surface usually results from increase the outside surface area to a point such
scale formation and corrosion. Fouling of the that the amount of heat absorbed by the outside
internal surface of the evaporator tubes is surface is approximately equal to that which
usually caused by excessive amounts of oil in can pass from the inside surface to the liquid
the evaporator and/or low refrigerant velocities. refrigerant.
At low velocities, vapor bubbles, formed by the Because heat transfer is greater to liquids than
boiling action of the refrigerant, tend to cling to vapors, this situation often exists in air-
to the tube walls, thereby decreasing the amount cooling applications where the rate of heat flow
of interior wetted surface. Increasing the re- from the inside surface to the liquid refrigerant
frigerant velocity produces a scrubbing action on is much higher than that from the air to the
the walls of the tube which carries away the oil outside surface. For this reason, the use of
and bubbles and improves the rate of heat flow. finned evaporators for air-cooling applications
Thus, for a given tube size, the inside film coeffi- is becoming more and more prevalent. On the
cient increases as the refrigerant velocity in other hand, in liquid-cooling applications, since
creases. The refrigerant velocity is limited, how- liquid is in contact with both sides of the evapo-
ever, by the maximum allowable pressure drop ratorand the rate of flow is approximately equal
through the coil and, if increased beyond a certain for both surfaces, barenube evaporators per-
point, will result in a decrease rather than an in- form at high efficiency and finning is usually
crease in coil capacity. This depends to some unnecessary. In some applications, where fluid
extent on the method of coil circuiting and is dis- velocity over the outside of the evaporator is

cussed later. It can be shown also that the con- exceptionally high, the flow of heat to the outer
ductance of the outside surface film is improved surface may be greater than the flow from the
by increasing the fluid velocity over the outside inner surface to the refrigerant. When this
surface of the coil. But, here again, in many occurs, the use of inner fins will improve evapo-
cases the maximum velocity is limited, this time rator capacity in that the amount of interior
by consideration other than the capacity of the wetted surface is increased. Several methods
evaporator itself. of inner finning are shown in Fig. 11-12.
EVAPORATORS 173

(a) (b)

Fig. 11-12. Some methods of


inner finning.

(O
11-10. Logarithmic Mean Temperature For the values given in Fig. 11-13, the arith-
Difference. As illustrated in Fig. 11-13, the metic mean temperature difference is

temperature of air (or any other fluid) decreases


(40 - 20) + (30 - 20)
progressively as it passes through the cooling D = 15° F
coil. The drop in temperature takes place along
a curved line (A) approximately as indicated. It must be remembered that the MTD as
Assuming that the temperature of the refrigerant calculated by Equation 11-3 is slightly in error
remains constant, it is evident that the difference because of the curvature of the curved line A
in temperature between the refrigerant and the and would be the actual MTD only if the drop
air will be greater at the point where the air in air temperature occurred along a straight line,
enters the coil than at the point where it leaves, as indicated by the dotted line B in Fig. 11-13.
and that the average or mean difference in The actual logarithmic mean temperature,
temperature will fall along the curve (a) at a which is the midpoint of the curved line A, is

point somewhere between the two extremes. given by equation


Although the value obtained deviates slightly
(f. - t r) - (/x - tr)
from the actual logarithmic mean, an approxi- D= (11-4)
it. - tr)
mate mean temperature difference may be In
calculated by the following equation: 0l - t r)

D JU-t )^H-Q T
(U3)
For the values given in Fig. 1 1 - 1 3, the logarith-
mic mean temperature difference will be
where D = the arithmetic mean temperature
/„ = the temperature of the air entering D=
(40 - 20) - (30 - 20) JO
40-20
=
ln2
= — 14.43° F
the coil In
30 -20
fj = the temperature of the air leaving
the coil The preceding calculations were made on the
tr = the temperature of the refrigerant assumption that the refrigerant temperature
in the tubes remains constant. When this is not die case,
174 PRINCIPLES OF REFRIGERATION

The velocity of the air passing over the coil


has a considerable influence on both the value
of U and the METD
and is important in deter-
mining evaporator capacity. When air velocity

is low, the air passing over the coil stays in


contact with the coil surface longer and is

cooled through a greater range. Thus, the


METD and the rate of heat transfer is low. As
air velocity increases,a greater quantity of air
is brought in contact with the coil per unit of

time, the METD


increases, and the rate of heat
transfer improves. In addition, high air
velocities tend to break up the thin film of
Leaving air stagnant air which is adjacent to all surfaces.
temperature-30 F
Since this film of air acts as a heat barrier and
insulates the surface, its disturbance increases
Fig. 11-13. Mean temperature of air passing through the conductance of the outside surface film and
evaporator.
the over-all value of U improves.

tr will have two values. This condition is

discussed in another chapter.


The log mean temperature difference here-
after called mean effective temperature difference
(METD), may also be determined from Table
11-1.
11-11. The Effect of Air Quantity on
CoflA
Evaporator Capacity. Although not a part of
the basic heat transfer equation, there are other
factors external to the coil itself which greatly
affect coil performance. Principal among these
and distribution of
are the circulation, velocity,
air in the refrigerated space and over the coil.
These factors are closely related and in many
cases are dependent one on the other.
Except in liquid cooling and in applications
where the product is in direct contact with the CoilB
evaporator, most of the heat from the product
is carried to the evaporator by air circulation:

If air circulation
carried

the
air
coil.
is inadequate, heat
from the product to the evaporator
rate sufficient to allow the evaporator to per-
form at peak efficiency.
that the circulation of air
in all parts of the refrigerated space
It is

is
is

important also
not
at a

evenly distributed
and over
Poor distribution of the circulating
results in uneven temperatures and "dead
spots" in the refrigerated space, whereas the
AW Air

uneven distribution of air over the coil surface CoilC


causes some parts of the surface to function less Fig. 1 1-14. Coils B and C both have twice the surface
efficiently than others and lowers evaporator area of coil A. Coil C has twice the face area of coil A
capacity. or coil B.
EVAPORATORS 175

11-12.Surface Area. Equation 11-1 indicates ing the number of rows as in coil B, the METD
that the capacity of an evaporator varies will be decreased and the increase in capacity

directly with the outside surface area. This will not be nearly as great as when the surface

is true only if the U factor of the evaporator area is increased as in coil C. For the same
and the METD remains the same. In many a long, wide, flat coil will, in
total surface area,

cases, the value of U and the METD are affected general,perform more efficiently than a short,
when the surface area of the evaporator is narrow coil having more rows depth. However,
changed. In such cases, the capacity of the in many instances, the physical space available
evaporator does not increase or decrease in is limited and compact coils arrangements must
direct proportion to the change in surface area. be used. In applications where it it is permis-

To illustrate, in Fig. 11-14, coils B and C each sible, the loss of capacity resulting from increas-
have twice the surface area of coil A, yet the ing the number of rows can be compensated for

3rd 2nd 1st


Row Row Row

Leaving -<- < -< Entering

Air Air
C
Air -*r- to
2 Air
Fig. temperature
11-15. Air
32°
M 35°
no
drop across typical three-row
0)
cooling coil. Temperature -<- ^ CC ^ Temperature

C\J
o
CM
Air Air
30*-«- 40*

Coil

increase in capacity over the capacity of coil A to some extent by increasing the air velocity over
will be greater for coil C than for coil B. Pro- the coil. Too, in some applications, the use of

vided the air velocity is the same (the total deep coils is desirable for the purpose of
quantity of air circulated over coil C must be dehumidification.
twice that circulated over coil A), the METD 11-13. Evaporator Circuiting. It was demon-
across C
will be exactly the same as that across strated inChapter 8 that excessive pressure drop
A and the capacity of C will therefore be twice in the evaporator results in the suction vapor

the capacity of coil A. arriving at the suction inlet of the compressor

Figure 11-15 shows how the METD is af- at a lower pressure than is actually necessary,
fected when the surface area of the coil is thereby causing a loss of compressor capacity
increased by increasing the number of rows and efficiency.

(depth). Note that the drop in air temperature To avoid unnecessary losses in compressor
is much greater across the first row and dimin- capacity and efficiency, it is desirable to design
ishes as the air passes across each succeeding the evaporator so that the refrigerant experi-
row. This is accounted for by the fact that the ences a minimum drop in pressure. On the
temperature difference between the air and the other hand, a certain amount of pressure drop

refrigerant is much greater across the first row, isrequired to flow the refrigerant through the
becomes less and less as the temperature of the evaporator, and since velocity is a function of
air is reduced in passing across each row, and pressure drop, the drop in pressure must be
is least across the last row. It is evident then sufficient to assure refrigerant velocities high
that the rate of heat transfer is greater for the enough to sweep the tube surfaces free of vapor
first row and that the first row performs the bubbles and oil and to carry the oil back to the
most efficiently. For this reason, if the surface compressor. Hence, good design requires that
area of coil A in Fig. 11-14 is doubled by increas- the method of evaporator circuiting be such
176 PRINCIPLES OF REFRIGERATION

can be eliminated to a great extent by splitting


the single circuit into two circuits in the lower
portion of the evaporator (Fig. 11-17). When
this is done, the refrigerant travels a single series
path until the refrigerant velocity builds to the
allowable maximum, at which time the circuit
is split into two parallel paths for the balance of
the travel through the evaporator. This has the
effect of reducing the refrigerant velocity in the
two paths to one-half the value it would have
Refrigerant without the split, and the pressure drop per foot
out is reduced to one-eighth of the value it would

have in the lower part of the evaporator with a


Fig. 11-16. Evaporator with one series refrigerant
single single series circuit.*This, of course,
circuit.
willpermit greater loading of the coil without
exceeding the allowable pressure drop. At the
that the drop in pressure through the evaporator same time, the velocity in all parts of the coil
is the minimum necessary to produce refrigerant is maintained within the desirable limits so
velocities sufficient to provide a high rate of heat that the rate of heat transfer is not unduly
transfer and good oil return. affected.
In general, the drop in pressure through any Another method of reducing the pressure drop
one evaporator depend upon the size
circuit will through the evaporator is to install refrigerant
of the tube, the length of the circuit, and the headers at the top and bottom of the evaporator
circuit load. By circuit load is meant the time- so that the refrigerant is fed simultaneously
rate of heat flow through the tube walls of the through a multiple of parallel circuits (Fig.
circuit. The circuit load determines the quan- 11-18). However, this arrangement is not too
tity of refrigerant which must pass through the satisfactory and is not widely used. While the
circuit per unitof time. The greater the circuit pressure drop through the evaporator is low,
load, tiie must be the quantity of
greater this method of circuiting ordinarily results in
refrigerant flowing through the circuit and the
greater will be the drop in pressure. Hence,
for any given tube size, the greater the load on
the circuit, the shorter the circuit must be in
order to avoid excessive pressure drop.
Evaporators having only a single series
refrigerant circuit, such as the one illustrated
in Fig. 11-16, will perform satisfactorily within
certain load limits. When the load limit is

exceeded, the refrigerant velocity will be in-


creased beyond the desired range and the
pressure drop will be excessive.
Notice that the refrigerant enters at the top
of the evaporator as a liquid and leaves at the
bottom as a vapor. Since the volume of the Fig. 11-17. Evaporator with split refrigerant circuit.
refrigerant increases as the refrigerant vaporizes,
the refrigerant velocity and the pressure drop
* Pressure
drop increases as the square of the
per foot increase progressively as the refrigerant
velocity. Reducing the velocity to one-half reduces
travels through die circuit, and are greatest at
the pressure drop to one-quarter of its original value.
the end of the coil where the refrigerant is 100 % Then, since the length of each parallel branch is
vapor. only one-half the length of a single circuit, the drop
The excessive pressure drop occurring in the in pressure in the lower portion of the split coil is
latter part of a single series circuit evaporator only one-eighth of the single circuit value.
EVAPORATORS 177

reducing the refrigerant velocity below the


desired minimum so that the inside film coefficient
and the rate of heat transfer are also low.
Another disadvantage of this type of circuiting
is that the loading of the circuits is uneven. Since
the temperature difference between the air
Refrigerant
passing over the coil and the refrigerant in the
in
tubes is much greater across the first circuit (first
row) than across the last circuit (last row), the Refrigerant
loading of the first circuit is much greater than distributor

the loading of the last circuit. Hence, the


refrigerant velocity and the drop in pressure >
Sk. Refrigerant
.

through the several circuits are uneven and a out

large portion of the coil operates inefficiently. Fig. 11-19. Evaporator with refrigerant distributor

This criticism can be applied to some extent and suction header. Notice counterflow arrangement
also for the circuit arrangements in Figs. H-16 for refrigerant and air.

and 1 1-17. In all three arrangements, the lower


portion of the evaporator will not perform as
effectively as the upper portion because wetting an air conditioning coilwhere the temperature
of the internal tube surface will not be as great differentialbetween the refrigerant and the air
in the lower portion. This is because the refrig- is large and where external finning is heavy.

erant in the lower portion contains a high Notice that the air passes in counterflow to the
percentage of vapor, whereas in the upper refrigerant so that the warmest air is in contact
portion the refrigerant is nearly all liquid. with the warmest part of the coil surface. This
It is for this reason also that the outside provides the greatest mean temperature differ-
surface temperature of the coil is always lowest ential and the highest rate of heat transfer.
near the refrigerant inlet and highest near the Notice also that loading of the circuits is even.
of the fact that the saturation
outlet, in spite The number and length of the circuits that such
temperature of the refrigerant is lowest at the a coil should have are determined by the size
outlet due to the drop in pressure through the of the tube and the load on the circuits.
coil. For the multipass, headered evaporator, the
The circuit arrangement shown in Fig. 11-19 arrangement shown in Fig. 11-20 is much more
is very effective and is widely used, particularly desirable than that shown in Fig. 11-18. Coun-
when circuit loading is heavy, as in the case of terflowing of the air and the refrigerant increases
the METD and permits more even loading of
the circuits.
1 1-14. Use of Manufacturer's Rating Tables.
The mathematical evaluation of all the factors
Refrigerant which influence evaporator capacity is usually
out V^ impractical and in many cases impossible. For
the most part, evaporator capacities must be
determined by actual testing of the evaporator.
Refrigerant The results of such tests are contained in the
in rating tables published by the various evapo-
rator manufacturers.
The method of rating evaporators varies
somewhat with the type of evaporator and with
Circuits the particular manufacturer involved. How-
ever, the various rating methods are very similar

Fig. 11-18. Four-circuit evaporator with refrigerant and most manufacturers include, along with the
headers on both inlet and outlet. Crossflow of air evaporator rating tables, instructions as to how
and refrigerant results in uneven circuit loading. to use the ratings. In most cases, where the
178 PRINCIPLES OF REFRIGERATION

Fig. 11-20. Counter-flow of


refrigerant and air results in
more even circuit loading and
a higher mean temperature
differential. Compare this
arrangement with the cross-
flow arrangement in Fig. 11-18.

evaporators are rated in accordance with ASRE 10° F, provided that all other conditions are the
Standards, the capacity data are reliable and same.t
are for operating conditions as normally en- It is evident that a coil with a relatively small
countered. surface area operating at a relatively large TD
The selection of evaporators from manufac- can have the same capacity as another coil
turer's rating tables is relatively simple once the having a larger surface area but operating at a
conditions at which the evaporator is to operate smaller TD. Thus, insofar as Btu per hour
are known. Typical evaporator rating tables, capacity alone is concerned, a small coil will
along with methods of evaporator selection, are have that same refrigerating effect as a larger
discussed at the end of the chapter. one, provided that the TD at which the small
11-15. Evaporator TD. One of the most coil operates is greater in proportion. However,
important factors to consider in selecting the it will be shown in the following sections that
proper evaporator for any given application the temperature difference between the evapo-
is the evaporator TD. Evaporator TD is rator and the refrigerated space has considerable
defined as the difference in temperature between influence on the condition of the stored product
the temperature of the air entering the evapo- and upon the operating efficiency of the entire
rator and the saturation temperature of the system, and is usually, therefore, the determining
refrigerant corresponding to the pressure at the factor in coil selection. Before an evaporator
evaporator outlet.* can be selected, it is necessary to first determine
Although more exact methods of rating the TD at which it is expected to function. Once
evaporators are necessary in order to select the desired temperature difference is known, an
evaporators for air conditioning applications
and some product storage applications where Care should be taken not to confuse METD
t
space temperature and humidity are especially with evaporator TD. According to Equation 11-1,
the Btu per hour capacity of any given evaporator
critical, ratings for most evaporators designed
(whose U factor and surface area are fixed at the
for product cooling applications are based on
time of manufacture) varies directly with the
evaporator TD.
METD. However, assuming that the refrigerant
The relationship between evaporator capacity temperature and all else remains constant, the
and evaporator TD is shown by the curve in METD between the air passing over the evaporator
Fig. Notice that the capacity of the
11-21. and the refrigerant in the evaporator will vary
evaporator (Btu/hr) varies directly with the directly with the temperature of the air entering the
evaporator TD. That is, if an evaporator has a evaporator. That is, if the temperature of the air
certain capacity at a 1 ° F TD, it will have exactly entering the evaporator increases, the METD
ten times that capacity if the TD is increased to increases. Hence, the METD varies in proportion
to the evaporator TD and, therefore, the capacity
* ASRE Standard 25-56, Methods of Rating Air of the evaporator also varies in proportion to the
Coolers For Refrigeration. evaporator TD.
EVAPORATORS 17?

evaporator having sufficient surface area to quate, the capacity of the evaporator is de-
provide the required cooling capacity at the creased, the product is not cooled at a sufficient
design TD
can be selected. rate, the growth of mold and bacteria is encour-
11-16. The Effect of Coil TD on Space aged, and sliming occurs on some products. On
Humidity. The preservation of food and other the other hand, too much air circulation can be
products in optimum condition by refrigeration as detrimental as too little. When the circula-
depends not only upon the temperature of the tion of air, is too great, the rate of moisture
refrigerated space but also upon space humidity. evaporation from the product surface increases
When the humidity in the space is too low, and excessive dehydration of the product
excessive dehydration occurs in such products results. Excessive dehydration can be very
as cut meats, vegetables, dairy products, flowers, costly in that it causes deterioration in product
fruits, etc. On the other hand, when the humid- appearance and quality and shortens the life
ity in the refrigerated space is too high, the of the product. Furthermore, the loss of weight
growth of mold, fungus, and bacteria is encour- resulting from shrinkage and trimming is a
aged and bad sliming conditions occur, particu- considerable factor in dealer profits and in the
larly on meats and especially in the wintertime. price of perishable foods.
Space humidity is of little importance, of course, The desired rate of air circulation varies with
when the refrigerated product is in bottles, cans, the different applicationsand depends upon the
or other vapor-proof containers. space humidity, the type of product, and the
The most important factor governing the length of the storage period.
humidity in the refrigerated space is the evapo- With respect to product condition, air circula-
ratorTD.* The smaller the difference in tempera- tionand space humidity are closely associated.
ture between the evaporator and the space, the Poor air circulation has the same effect on the
higher is the relative humidity in the space. product as high humidity, whereas too much air
Likewise, the greater the evaporator TD, the circulation produces the same effect as low
lower is the relative humidity in the space. humidity. In many instances, it is difficult to
When the product to be refrigeratedis one determine whether product deterioration in
that will be affected by the space humidity, an a particular application is caused by faulty air
evaporator TD that will provide the optimum circulation or poor humidity conditions. For
humidity conditions for the product should be the most part, product condition depends upon
selected. In such cases, the evaporator TD is the combined effects of humidity and air
the most important factor determining the circulation, rather than upon the effect of either
evaporator selection. The design evaporator one alone, and either of these two factors can
TD required for various space humidities is be varied somewhat, provided that the other is
given in Table 11-2 for both natural-convection varied in an off-setting direction. For example,
and forced-convection evaporators. higher than normal air velocities can be used
In applications where the space humidity is without damage to the product when the space
of no importance, the factors governing evapo- humidity is also maintained at a higher level.
rator selection are: (1) system efficiency and The type of product and the amount of
economy of operation, (2) the physical space
35
available for evaporator installation, and (3)
initial cost. £ 30
11-17. The Effect of Air Circulation on a§2 5
Product Condition. As stated previously,
circulation of air in the refrigerated space E 15
is l'
essential to carry the heat from the product to
uj
the evaporator. When air circulation is inade- 5

* Some of the other factors which influence the 5* 10* 15* 20* 25* 30* 35*
space humidity are: air motion, system running Evaporator TD
time, type of system control, amount of exposed
product surface, infiltration, outside air conditions, Fig. 11-21. Variation in evaporator capacity with
etc. evaporator TD.
180 PRINCIPLES OF REFRIGERATION
exposed surface should be given consideration length of the cooler and covering the greater
when determining the desired rate of air circula- portion of the ceiling area are best. As the
tion. Some products, such as flowers and depth of the coil is increased, the coil offers
vegetables, are more easily damaged by excessive greater resistance to the free circulation of air
air circulation than others and must be given and the METD is thereby decreased with a
special consideration. Cut meats, since they resulting decrease in the coil capacity. Since
have more exposed surface, are more susceptible cold air is denser than warm
and tends to
air
to loss of weight and deterioration than are fall to the floor, evaporators should be located
beef quarters or sides, and air velocities should as high above the floor as possible, but care
be lower. On the other hand, where the product should be taken to leave sufficient room between
is in vapor-proof containers, it will not be af- the evaporator and the ceiling to permit the free
fected by high velocities and the rate of air circulation of air over the top of the coil.
circulation should be maintained at a high level For coolers less than 8 ft wide, single, ceiling-
to obtain the maximum cooling effect. mounted evaporators are frequently used. When
Recommended air velocities for product the width of the cooler exceeds 8 ft, two or more
storage are given in Tables 10-10 through 10-13. evaporators should be used. In coolers where
11-18. Natural Convection Evaporators. there not sufficient head room to permit the
is

Natural convection evaporators are frequently use of overhead coils, side-wall evaporators
used in applications where low air velocities may be used. If properly installed, these will
and minimum dehydration of the product are function with approximately the same efficiency
desired. Typical installations are household as overhead coils. Typical overhead and side-
refrigerators, display cases, walk-in coolers, wall installations for large storage rooms are
reach-in refrigerators, and large storage rooms. shown in Figs. 11-22 and 11-23, respectively.
The circulation of air over the cooling coil by In small coolers, baffles are used with natural
natural convection a function of the tempera-
is convection coils to assure good air circulation.
ture differential between the evaporator and the The baffles are installed in such a manner that
space. The greater the difference in temperature, they aid and direct the free flow of air over the
the higher the rate of air circulation. coil and throughout the refrigerated space. The
The by natural convection
circulation of air cold and warm air flues should each have an
is greatly influenced by the shape, size, and area approximately equal to one-sixth or one-
location of the evaporator, the use of baffles, seventh of the floor area of the cooler. Assum-
and the placement of the stored product in the ing that the flues extend the full length of the
refrigerated space. Generally, shallow coils fixture, the width of the flues will then be
(one or two rows deep) extending the entire proportional to the width of the cooler. Since

Fig. 11-22. Overhead installa-


tion of natural convection
evaporator. Evaporator has
cast aluminum fins. (Courtesy
Detroit Ice Machine Company.)
EVAPORATORS III

Ftf. II.23L Sid* will Installa-


tion of naturil tonvtalon
enpantor. (County On role
Ic* Miehmt Company

warm Air has a greater specific volume than 3 (o 4 in. beto* the coil. The horizontal baffles
eoJd air. some manufacturers recommend thai or coil decks should slope ) to 2 in. per foot to
the warm air flue tx a little larger than the cold give direction to the cold ajr flowand lo drain
air flue. In Fig. 1 1-24, the width or the cold air on" the condensate- Also, he coil decks must
I

Due it equal to W/7, whereas the width of the be insulated so that moisture does not condone
warm air flue is equal to W','6. The distance M) on the undersurfcee of the deck and drip off on
from the should be approxi-
coil to the ceiling the product. The dimension (if) is 4 to 7 in.,
maldy equal to the width of the watm air flue whereas (O is usually 1 to 4 in,
and never less than 3 in. Vertical side baffles IMf-Coi1-ajid-Bafr1eAii*mbll«s. The avail-
should extend approximately t in. above and of radory-buill coJI-and-bafrk assemblies
ability
has eliminated the need for the
practically
custom building of baffles on the job. A typkal
ready built coil-and-bafflc assembly is shown
in Fig. 11-25. Since these assemblies are
wide variety or sizes and combina-
available in a
tion (an Table R-l), they can be readily
applied to almost any natural convection
application.
11-20. Hating and Select on uf Natural Con-
I

vection Evaporators, Basic capacity ratings


Tor natural convection evaporators, both prime
surface and finned, arc normally given in Btu/hr/
* F TD. However, in some instances, where it

will simplify evaporator selection, capacity


ratings are given for TD's other than I • F.

For the coil -and- baffle assemblies mentioned


in the preceding section, the ratings given arc
per inch of finned length. For bare pipe evapo-
rators, the ratings given are per square foot of
externa] pipe surface, although in some instances
bare pipe evaporators are rated per lineal foot
of pipe.
Ratings for plate evaporators are given per
Hf. 11-24. Tvpkal baflla amngamaflt for natunl squire foot of plate surface. Both sides of the
conractlon end I. plate arc considered when computing the area
I K. MUNCltt.ES OF REfRlGEIUTlON

Ft|. 11*35, Nitunl «cn»«.

(Court -rty Dunhjm- faith. Int.)

of the plate, Frequently, ratings Tor plate To determine approxi-


evaporators apply to an entire plate or to a mate finned length required
specific group or combination or plats, Over-all length of
Tvp^-.ii eating data f^? hhjcth typp of
cooler (inside) — 17 ft
Allowing I ft on each
mural convection evaporators are given in
end of evaporator for
Tables R-l through R-7. The use or these
working space, the
rating data in the selection of the various types
approximate over-all
of evaporators is best illustrated through the length of the evapo-
use or a series of examples. ratori4(«cFig.Jl-16)
(I7fl-2ft) -|5ft
Enamels J M.i natural convection
Select According to the
cou-and-baflfe assembly (Fig. 1 J -25 and Table manufacturer's speci-
R-l) for the vegetable storage cool? in Example fications {Table ft-1).
KM I. the over-all length of
the evaporator is 7 in
Solution. Since the capacity ratings for this longer than the actual
type of evaporator are given in Btufhr/T TD/in. finned length. Hence,
of finned length, the required evaporator capa- the approximate
city must be reduced to this value before a finned length desired - 14 ft 3 in.
selection can be made from the rating (able, is {15 ft -7 in.) or 171 in.
Abo. recall that a natural convection evapora- To determine the require! capacity in Biu/hr/
tor should extend almost the full length of the
*FTD,in. finned length:
cooler in order to assure adequate air circulation
Required evaporator
around the product. capacity per * F TD
From Example 10-11. Total c* J|H>rutor capacity
inside dimension of cooler — 17 ft x9A DewgiiL...!]«ralorTD
8500 Blu hr
Required evaporator
capacity (average hourly
14* Fin
cooling load) — 8500Biu/hr - e0 7B•u;hr.
l ,
FTD
Required capacity
From Table 10-11. de- {Biu/hr/*FTD/ineh
sired space humidity for Rcquirett o pacity per *FTD
<
mixed vegetable storage *• Approx. 87% m
Dei rcdTtnned length
i

From Table 1 1-2,design *07 Biu.hr/" F TD


evaporator TO required 171 in. tinned length
forS7%RH - 3.35 Btu/hrf F/ineh
EVAPORATORS IS)

Became or the width of the cooler, a two-


tcciicti evaporator will give the best results.
Reference to Tabic R-l indicates that Model
#PK-I6 with two fin* per inch (J -in. fin spacing)
has capacity ofl 65 1
Biu/hr/ F/in.
Using this mode) evaporator, the required
finned length is

- lAfun
J,*5 BtWnr/' F/in.

The overall length of the evaporator is 173


fit. W-tt Arnnf«rn«ni of natural (on**ct»on
in. (166 in. + 7 in.) and. since the overall
,

twtporttori in itortj* COoUr (i*» Example 1 1. 1).


length of the cooler inside is 204 in., the
dc*rancc between the evaporator and the cooler
wall at each end is 15 5 in.
(IS in. x 2), the evaporator is suitable for the
The width of the evaporator should always
be checked a gains the width of the cooler lo be
I
cooler. A logical arrangement of ihc two
evaporator sections in the cooler is shown in
sure that the evaporator can be installed in the
Fig 11-27.
space in accordance with the manufacturer's
To order the evaporator, specify the modet
recommendations. For evaporators of this
number, fin spacing, and finned length, viz:
type, the manufacturer recommends that the
Model #PK- 1 6.3-166 in
side of the evaporator be not less than 6 in. nor
Note. To avoid excessi vely long evaporators,
more than from the cooler wall {installa-
12 in.
which are inconvenient to ship and install, a
tion dimension A of Table R-l) and that the
multiple of evaporators should be used in large
distance between the two sections of the evapo-
coolers. Typical arrangements Tor large coolers
rator (dimension C) be not less than 6 in. nor
are shown in Fig. 11-28. These arrangements
more than 8 ft.
are also suitable For plate banks.
The maximum allowable evaporator width
can be determined by subtracting the minimum ExampJ* 11*2, Using Table R-3, select
of dimensions A and C from the inside width of plate evaporators (banks) for ceiling installation
the cooler, viz in the locker room of Example 10-16, To assure
Maximum width of evaporator •- Inside good air circulation in the locker room, select

width of cooler - {A + C + Q. In this par- evaporators for a 15* FTD,


ticular case, the maximum allowable width of the
Solution, Analysis of room dimensions
evaporator is
(30.5 x 16 ft) indicates that four to six evapo-
ft

108 - +6 in. + 6 - 90 rators (two or three banks installed end-to-


in. (6 in. in.) in.
end over each aisle will be needed to provide
Since the combined width of the two sections good ceiling coverage and adequate air distri-
of evaporator. Model #PK-16, is only 36 in. bution. Reference to Table R-3 will show that
plate banks are available in stock lengths of
inftdt hngei of coota?— r I 108 in. (9ft) and 144 in, (lift). Three banks
108 in. long (a total of 27 ft) or two banks 144
in, long (a total of 24 ft) could be installed
end-to-end over each aisle and allow adequate
working clearance at the ends.
Since the banks are already rated at the design
TD of 15* F, the ratings can be used directly and
the required capacity per bank can be deter-
mined by dividing the total hourly cooling load
by the desired number of plate banks, viz:

Hf> 1 1*1*, Arr»mj«m«ni of mmuni eonvteiion Required capacity _ Total cooling load (Btu/hr)
tviponton In ttorij* coolir (« Exampi* 1 t-|). per bank (Btu/hr) Number of banks desired
IB4 PRINCIPLES OF REFRIGERATION

In this instance, the capacity required per bank


Solution. Inspection of Tabte R-4 will show
is
that the ratings of the p stand assemblies li : ._-

24,650 Btu/hr an based on a J 5° F TD. Since the design TD


-4lWBtWhr/13*FTD only 10 F. it is necessary to
instance
6 buna in this is

or determine (he capacity the plate must have at a

24,650 Btu/hr
IV ITD in order to have the desired design
= 6l62Blu/hr/)5' FTD TD of 10- F. This is accomplished by dividing
4 banks
the average hourly load by the design TD of
10* F and then multiplying by the rating of TD
Referring to Table R-3, we see thai plate 15* F, vis;
bank. Model #5-1 210B-B, has a capacity of 4320
Btu/hr at a 15" F TD when operating below 4590 Btu/hr x 13' F
6885 Btu/hr
32° F (frosted). This will permit good coverage 10" F
of the ceiling and at the same time allow suffi-
Thus, it is determined that an evaporator having
cient working space ui the ends of the banks (see
a capacity of 6885 Btu/hr ai a 15 F TD will '

Fig- ll-29>.
have he desired capacity of 4590 Btu/hr at the
I

In ordering the evaporators, specify refrig-


design TD of ID* F. Th* value can then be
cram and the type of connections desired
used to select the plate stand directly from the
(series or manifold), viz
rating table.
Mode) #6-J2l44-B, series connected for
By referring to Tabic R-4, plate stand,
Refrigerant- 12. Notice {Table R-J) that the
Model #1-72%*-%. which approximately 26 in.
is
manufacturer specifies that two refrigerant flow
wide and 88 in. long (including piping connec-
controls should be used with each bank for
tions), will (it the freeaorr cabinet and has a
Refrigeran 1-12, whereas only one a needed when
ammonia is the refrigerant.
capacity of 7140 Btu/hr ai a 15" F (4760 TD
Btu/hr at 10* F TD>, This provides a small
safety factor and is therefore satisfactory for
Example 11-3, From Table R-4, select a the application.
plate-stand assembly for the freezing cabinet in
Example 10-16. The inside dimensions of (be Alternate Solution. A rule of thumb used in
cabinet are 28 in. x 90 in. and the freezing load selecting plate freezer? for locker plant appli-
is 4590 Btu/hr. Base plate selection on 10 F cations is to allow 0.5 Sq ft of plate surface for
TD. each locker.
Allowing 0.5 sq ft of plate surface per locker
per day, the plate surface required la this
instance is

353 lockers x 0.5 = 1 7fi.S sq ft

V By referring to Table R-2. the plate


flu the cabinet. Model #22S4 (22 in. x 84 in.),
which best

has a surface area (both sides of plate) of 27.24


sq per plate. Therefore, seven pistes of
ft

are required. Seven plates have a total


this size
capacity of 7140 Btu/hr ai a I5 F
5
or 4760 TD
Btu/hr at a 10° F TD.
Example 11-4. Assuming a space tem-
perature of 0* F and a refrigerant temperature
of - T
F (1 7" F evaporalor TD), determine the
lineal feet of 1 i in. iron pipt required to handle
the cooling load on the locker room in Example
10-16.

Solution, By referring to Table R*6, the


capacity per square foot of pipe (outside surface)
at the given conditions is 1.5 Btu/hr/ F TD,
Fig. 11-26. Typical arrange rrmnfi tor natural con- To determine the square feet of pipe surface
vection evaporator* In lirjt cooten. required, divide the average hourly load by the
EVAPORATORS IIS

— 305" *
r
|£ *—^r^ **
" tl IT g* t\
*-r

Flf. 1
1
-It, Inttillillon of pliie
tanks in locker ptint (sm
Eximpl* 11-3.)

1 ^^^^^^^^^ '

capacity per square foot per degree TD and by below its dew point temperature, both the tem-
die design TD, viz: perature and the moisture content of the air an
Opacity required (Btu/hr) reduced (Chapter 5). The temperature reduc-
tion the result of sensible cooling, whereas the
Pipe capacity (Btu/hr/iq ft/' F) x TD is

moisture removed is the result of latent coo ting.


— Pipe surface I sq ft)
Kence. for an evaporator having a total cooling
In this instance, the square feet of pipe surface
capacity of one ton (12,000 Btu/hr) and a
required is
sensible heat ratio of 0.85. the sensible cooling
24,650 Btu/hr capacity of (he evaporator is 10,200 Btu/hr
1.3 x 17 ™>»M" <W% or 12,000). whereas the latent cooling
By referring to Table R-7, 2.1 lineal ft of I in.
capacity is 1800 Btu/hr (12,000 - 10.200).
J
pipe equals I sq ft of external pipe surface. Naturally, the sensible of anyheal ratio
Hence, the lineal feet of I J in- pipe required is evaporator will depend upon ofthe conditions
the application, the design of the evaporate r, and
966 x Z.3 - 2220 ft
the air quantity. An average sensible heat ratio
11*21- Forced Convection Evaporator*. for unit coolers is approximately 0.83.
Forced convection evaporators, commonly As a general rule, the air temperature drop
called "unit coolers" or "blower coils" in com- through a well -designed unit cooler is approxi-
mercial refrigeration, are essentially finned coils mately one- half the difference between the space
in a metal housing and equipped with temperature and the refrigerant temperature.
or more fans to provide air circulation. For example, for an evaporator TD of 10 F.
Some typical unit cookrs on shown in Fig. the air temperature drop through the unit
11-30. u.H)lcr should be .tppri^inuid', f I

The total cooling capacity of any evaporator The constant LOS in Equation 1 1-3 is a con-
is directly related to the air quantity (cfm) version factor involving air density (0J5), air
circulated over the evaporator. The air quantity specific heat (0.24 Btu/lb/° F), and minutes per
required for a given evaporator capacity hour (*0).*
is basically a function of two factors: (i) the
sensible heat ratio and (2} the drop in the
V
£x m en p I b 1 1 -5. 1 (ermine the approximate
quantity of air {cfm) circulated over a unit
temperature of the air passing over the cvapo*
cooler having a capacity of 20.000 Btu/hr if the
rator. vix:
sensible heat ratio is 0,85 and the design evapo-
Cfm - rator TD is 1 3* F.

Total capacity (Btu/hr) x sensible heat ratio x


Solution.Apply- _ 30.000 Btu/hr 0.83
Temperature drop of the air x 1.08 ing Equation 1-5, 1 7.5 x 1.08
(11-5) the air quantity - 2)00 cfm

The sensible heat ratio is (he ratio of the Ai a general rule, air velocities across the face

sensible cooling capacity of the evaporator to of unit coolers are maintained between 300 and
the total cooling capacity. When air is cooled • Sen Section 14-4.
186 PRINCIPLES OF REFRIGERATION

Flf. Il-M. Topical Link tooter deslgni. Noikl thlt eool*r d*ligni iri mch that the air It not dttcharjid
directly on i ho *to red product. (Counety Dunham-Bush, Inc.)

500 ft per minute (Tpm), Although higher cooler capacity resulting from higher air
velocities will result in higher transfer coeffi- velocities. Hence, the net effect in such cases
cients, they are not usually practical since they it to decrease, rather tkm increase, the over-
alio increase horsepower requirements.
fan all capacity of the unit cooler. Too, where
When the fan horsepower is increased beyond a the air 500 fpm there Is a
velocity exceeds
certain point, the additional heal given off by tendency for moisture to he blown from the
the fan motor resulting from the increase it) face of the coil into the space and onto the
horsepower will exceed the increase in unit product.
EVArOWKTORS 117

The iir velocity {fpm) over the evaporator ii Example I J -7. From Table R-8, select a
a function of the air quantity (din) and the face forced convection evaporator (unit cooler)
ana of the evaporator (v\ ft), viz suitable for installation in a beer storage cooler
having a calculated heal load of 16,600 Btu/hr.
Air quantity (cfm)
Velocity (fptro - (11-6)
Since apace humidity is not a factor, use JO F
Face area |sq f i) TD for high system efficiency.

Example 1 1 *. Determine the face am of Solution. From Table R-S, select unit cooler
thcevaporaior in Example -5 if i he face velocity
1 1

is to be maintained at J 50 fpm.
Model #UO!80 having a capacity of 18,000
Btu/hr at a 10* F TD, Since the unit cooler fan
Solution. By rearranging and _ 2100 dm motor operates inside the refrigerated space, the
applying Equation "
11*6, the 350 fpm
motor heat becomes a part of the space cooling
face ana — 6 iq ft
load and must be added to the load calculations.
1 1-22- Rat ng and Saf action of
i Un i t Coolers From Table R-g, the heat given off by the Tan
Basic rating! for unil cooler* are given in Blu/hr/ motor 24,000 Btu/24 hr, Since the fan oper-
is
* F TD. For convenience, sometimes ratings ates continuously to provide air circulation in
are given for 10" F and li* F TDs. Aa in the the refrigerated space, while the average hourly
case Of natural convection evaporators, the cooling load is based on a 16-hr running time,
deafen TD for unit cooler* depends primarily the average Btu per hour load resulting from
on the space humidity requirements. In general, the fan motor heat is
for any given space humidity, the design for TD
unit coolers is about 4* F
lo ** F leas than those 34,000 BtiV?4 hr
used for natural convection evaporator* (we - 1 300 Btu/hr
16 far
Table 1 1-2).
Since the air quantity is usually fixed by (he Hence, when the fan motor beat is considered,
fan selection at the time of manufacture, realiz- the average hourly cooling load for the beer
ation of the rated capacity will depend primarily cooler becomes 18,100 Btu/hr (16,600 + 1500).
upon whether or not the coil is kepi reasonably Since the unit cooler selected has capacity of
free of frost by adequate defrosting When the IS.OOO Btu/hr, it will be adequate for the
space is maintained below 34' F, some means of application.
automatic defrosting musi be used (see Chapter Suggested locations for unit coolers in walk-
20). in refrigerators are shown in Fig. 11-11,

ftg. IIO I. Su(|«ikwii for


location of unit coolin In
walk-in f*frS|tf*ton. (From
tti* Mft£ Dots ftcok, D**Jjn
Volume, 1957-M edition, re-
produced by ptrmmlon of Ux
American Society of Hmlnf.
RvtriftrMint; and Air-Condi*
tlMldf Enjin«rv)
IBS PRINCIPLES Of REFRIGERATION

11-13. Liquid Chilling Evaporators, As with headers and art connected together by re-
air-cooling evaporators, liquid chilling evapo- movable return bends (inset The advantages
I,

rators vary in typeand design according to the claimed for ink unit are rigid construction, the
type of duly for which they art intended. Five elimination of refrigerant [uints, and easy
general types of liquid chiller* are in common of the inner tubes for cleaning.
accessibility

use: (1) the double^pipc cooler. (2) the Baudelot Double-pipe coolers ma;, ix; operated cither
cooler (3) the lank-type cooler. (4) theshell*and- dry-expansion or floodci!. In either cue,
.

coil cooler, and (3) the shell-and-tube cooler. counterflowing of the fluids in the lubes pro*

In all cases, the factors which influence the duces a high heai transfer coefficient.
relatively

performance of liquid chillers are the same as However, ihis type of cooler hits the disadvan-
those which govern the performance of air- more space, particularly head
tage of requiring
cooling evaporators and all other heat transfer room, than some of the oiher cooler design*.

Current conservative design values of heat transfer coefficient C based on


outside surface for bare tube coolers, unless mentioned otherwise, are as
follows: Min |ft«
Flooded shell -and- tube cooler (water to ammonia or R-12) su 150
Flooded ihell-and -finned tube high velocity R-12 water cooler JO 150
Flooded shell-and-tube cooler (brine to ammonia) 45 100
Flooded shell-and-tube cooler (brine to R-I2> j« 90
Dry-expansion shel)-«nd-tube cooler, R-12 in tubes, water in shell 50 115

Baudelot cooler. Hooded (ammonia or R-12 to water) 100 200


Bau deloi cooler, dry-expansion (ammonia to water) 60 150
Baudelot cooler, dry-expansion (R-12 to water) 60 120
Double-pipe cooler (water to ammonia) 50 150
Double-pipe cooler (brine to ammonia) 50 125

Shell -and -coil cooler (water to ammonia) 10 25


Shell-and-coil cooler (water to R-12) 10 25

Spray-type shell-and-tube water coolers (ammonia or R-12) ISO 250


Tank-and-agitator, coil-type water cooler, ammonia, flooded so 125
Tank-and-agiiator, coil-type waler cooler. R-12 Hooded 60 100
Tank, ammonia, brine cooling, coils between can in k* tank 15 40
Tank, high velocity raceway type, brine to ammonia SO 110

Fig. It-XL Ht« tniwftr coeftidants (or vsrfout types of liquid chlllan. (Rtpi-miad from JMJ-5*
ASflE Dots Book, by furmlulon of th* Amtfican SoclKy of H**tlnj. ft«lri{«rsttr-i, «nd AJr-CondltJoiUnt
En|in*tn,)

surfaces. Heat transfer coefficients for average For this reason, the double-pipe cooler is used
designs of tome of the various chiller types are only insome few special appliea lions. A number
listed in the table in Fig. 1 1-32. have been used in the wine-nuking and brewing
11-24. Double-Pipe Coolers, The double-pipe industries to chill wine and wort, and in ihe
cooler consists of two lubes SO arranged that petroleum industry for the shilling of oils.
one tube is inside the other. The chilled fluid 1 -IS. Baudelot Coolers- The Baudelot cooler
1

flows in one direction through the inner tube shown in Hg. l-M contisu of a series of hori-
I

while the refrigerant flows in the opposite zontal pipes which are located One under the
direction through the annular space between the I her and are connected i^geiher to form a
inner and outer tubes. One design of a double- refrigerant circuit or circuit*. For either dry-
pipe cookr is shown in Fig, 1 1-33. In this design expansion or flooded operation, the refrigerant
the outer tubes arc welded to vertical refrigerant is circulated through the inside of the tubes

headers while ihe inner lubes pass through the while the chilled liquid flows in a thin film over
EVAPORATORS IB?

Fij. 11-11. Double pip* cootar. Removable mum


b*ndi (right) ira ditlfnad to make tub* readily
accessible, for clunlnj (Courtey Viltar rltnuficturini Company.)

the outside. The liquid fowl down over the chilling of water Tor carbonation in bottling
tubes by gravity from a distributor foaled at plan is. With
this particular type of chiller it is
the top of the cooler and is collected in a trough possible lo chili liquid to a temperature very
at the bottom. The fact thai the chilled liquid close to the freezing point without the danger
it at atmospheric pressure and is open to the air of damaging the equipment if occasional
nukes the Baudelot cooler ideal for any liquid freezing of the liquid occurs.
chilling application when Aeration u a factor Another advantage of the Baudelot cooler,
The Baudejm chiller has been widely used for the and one which is shared by the double-pipe
cooling of milk, wine, and wort, and for the cooler, is that the refrigerant circuit is readily

Flf- 1 1-14. Bmddoi tooltr


employ *d in milk-cool nj I

application. (Coortu/ Dola


Refriierailn g Company.)
190 PRINCIPLES OF REFRIGERATION

_ Warm fcquWI m

Flf. If-JS. Typlal construc-


tion of tnnk-cjfpe liquid Caol a r.

J?Bfnj(er*n1
fines

split into several parts, a circumstance which a baffle arrangement. As shown in Fig. 11-35.
permits precoohng of the chilled liquid with a motor driven agitator is unlived to circulate
cold wain: before the liquid enter* the direct- the chilled liquid over the cooling coil at
expansion portion of the cooler (see Fig. 7-34>. 1 relatively high velocity, usual h,
between LOO and
11-16. Tank-Type Cooler*. The lank-type 50 ft per minute, the liquet being drawn in at
1

liquid chiller consuls essentially of A bane-lube one end of the coil compart men .ind discharged
I

refrigerant coil installed in the center or si one at the other cod.


side of a large steel lank which contain} the The bare-mbc coils mentioned
spiral-shaped,
chilled liquid. Although! completely Autxnefgctl in Section and the race t-jt -type coil illus-
1 1*4
in the chilled liquid, the refrigerant coil is trated in Fig. 1-36 arc two coO designs fre-
1

separated from the main body of the liquid by quently employed in tank-i>pe chillers. With

M. Flooded rac*w*f colt, (Courttiy Vllltr Hlftufatlu t\ n| Company,}


IVAPOMATOHS 191

either design the coils are operated flooded. a welded steel shell (Fig. 1 1-37), As a genera]
The lee-Cel shown in Fig. 11-11 is another rule, the chiller is operated dry -expansion with
variation or the tank-type chiller. the refrigerant in the coils and ihe chilled liquid
Tank -type chiller*can be Applied to any imhc*hell. In a few cases, the chiller is operated
liquid-chilling application where sanitation is flooded, in which esse the refrigerant is in the
not a primary factor, and arc widely used for shell and the chilled liquid passe* through the
the chilling or water, brine, and other liquids tubes. The former arrangement has ihe ad van*
to be used as secondary refrigerants. Because lage of providing a holdover capacity, thereby

liquxs line

ntVffMlM

tc uuen dfatn

Thtrmal eaptAiion vahw

Flj. I I.JT- Shrll-ind-cdl cootsr, {Courtis/ Acm* tndintriai.)

of their inherent holdover capacity, they are making this type of chiller ideal for small
particularly suitable for applications subject to applications having high but infrequent peak
frequent and seven fluctuations in loading. In toads, tl is used primarily for the chilling of

such cases, a comparatively large chilled-liquid water for drinking and for other purposes where
storage tank is provided in order to minimize sanitation is a prime factor, as in bakeries and
the rise in the temperature of the chilled liquid photographic laboratories.
during periods of peak demand. The advantage When operated Hooded with the refrigerant
gained by precooling is often considerable in in the shell, this type of chiller becomes what
cases where the liquid to be chilled enters the is commonly referred to as an "instantaneous"
cooler at relatively high temperatures. liquid chiller. One of the disadvantages of this
11-27. She -and -Coil Cooler*. The shell-
1
1
arrangement is that there is no holdover
and-coil chiller is usually made up of one or capacity. Since the liquid is not recirculated,
more spiral-shaped, bare-tube coils enclosed in it must be chilled instantaneously as it passes
192 PRINCIPLES OF REFRIGERATION

Liquid Outlet -

MM *c r ^"t
,
£ ti ,
'l

f*l

Bf> tl-3*. Typical iheilmd-tub* chilltn. (0) Hooded typ«- Tuba bundla li ramovablc |b) Dry-txpuuion
Eype (refrigerant in lubm). Not* taffllnf of Wiur circuit Tub* theati ire fixed. (C*ur(t*y WonMnftOII

ihrough the coils. Another disadvantage is that tube chiller it by far the mosi widely used type.
Ihc danger of damaging the chiller in (he event Although individual designs il filer somewhat,
of freeze-up is greatly increased in any chiller depending upon Ihc refrigen-ini used and upon
where the chilled liquid is circulated through whether the chiller is operated dry-expansion
the coils or tubes, rather than over the outside of or flooded, the shcli-and-tube chiller consists
the tubes. For this reason, chillers employing essentially of ft cylindrical Keel shell in which a
this arrangement cannot be recommended for number of Straight tubes are arranged in parallel
any application where it is required to chill the and held in place at the ends by tube sheets.
liquid below 38° F, When the chiller is operated dry-expansion, the
Instantaneous shell -and -coil chillers arc used refrigerant is expanded into the tubes while the
principally for the chilling of beer and oiher chilled liquid is circulated ilmmgh the shell
beverages in "draw-bars," in which case ihc (Fig. 1
1
-38b). When the chiller is operated
beverage is usually precooled to some extent flooded, the chilled liquid is circulated through
before entering the chiller. the 1 iilics .111 J ii contained in the
the refrigerant
11-2& Shell-and-Tvbe Chillers, Shell-and- shell . the levelof the liquid refrigerant in the
tube chillers have a relatively high efficiency, shell being maintained with some type of float
require a minimum of floor space nnd head control {Fig. LlOBd), In either case, the chilled
room, are easily maintained, and are readily liquid is circulated through the chiller and con-
adaptable to almost any type of liquid-chilling necting piping by means of a liquid circulating
application. For these reasons, the shcll-and- pump, usually of the centrifugal type.
EVAPORATORS t?J

Shell diameters for shell~and-tube chillers flooded type me the smaller refrigerant charge
range from approximately 6 to GO in,,and the required and the assurance of positive oil return

number of tubes in the shell varies from fewer to the compressor. Too, aa previously stated,

Lhan SO to several thousand- Tube dkmclem the possibility of damage to the chiller in the

range from I in. through 2 in., and lube lengths event of freeze-up is always considerably less

vary from S to 20 ft. Chillers designed for use when the chilled liquid is circulated over the
with ammonia are equipped with steel tubes, lubes ml her lhan through them. The more
whereas those intended for use with other important construction details of several designs
refrigerant! an usually equipped with copper of dry-cupansion chillers are shown in Figs.
1-39 "and 1-40.
tubes in order to obtain a higher heat transfer 1 1

ooefnaenl. Because of the rela lively low film En order to maintain the liquid velocity

conductance of halocarbon refrigerants, chillers wilhin ihc limits which will produce the most
designed for use with these refrigerants are often effective heat transfer-pressure drop ratio, ihe

equipped with tubes which are fumed on the vctochy of the chilled liquid circulated over the
refrigerant side. In the case of dry-expansion lubes is controlled by varying the length and
chillers, the lubes are finned internally with spacing of the segmental baffles, When the
longitudinal fins of the types shown in Fig. fiow rate and/or liquid viscosity is high, short,
11-12. For flooded operation, the tubes are widely spaced taffies are used to reduce the
finned externally using a very short fin which velocity and minimize the pressure drop through
protrude* from the lube wall only approximately the chiller. When the flow rate and/or liquid
,\thof an inch. viscosity is low. more closely spaced
longer,
As a general rule, dry<*p*nlkm drillers are baffles used in order to increase fluid
are
employed in small and medium tonnage instal- velocity and improve the heat transfer coef-
lations requiring capacities ranging from 2 to ficient (Fig.IMkr).
approximately 250 tons, but ore available in The number and the length of the refrigerant

larger capacities. Flooded chillers, available circuits required to maintain the refrigerant
in capacities ranging from approximately 10 velocity through the chiller tubes within reason-
through several thousand tons, are more able limits depend on the total chiller load and
frequently applied in ihe larger tonnage instal- on the relationship of the chilled liquid flow

lations. rate to the METD. Since the* factors vary


with the individual application, it follows that
Il-M. Dry-Expansion CMEIar*. The principal
advantages of the dry-expansion chiller over the the optimum refrigerant circuit design atso

Flf . I !-«. C u«w*¥ Mrtloit llltMirstini eoimrucrion dtutli of drr-wtpUMlon chllfer with ftxad tub* ihMta,
(Counter Aem* Industrial.)
SM PRINCIPLES OF REFRIGERATION

Flf. I MO.
Dry-expirtKon (hilJ*r with tub* bundl* fnnl*riy rtmovtd IO llMW lutw inwjimint mnif
Tub* bundl*
refrlferani flUtribirtcn,
Fihmr Com piny Inc.)
w
b* rtrnov*d U t unll, (CourtMjf Ktnmrd Cn.non, Amtritln Air
,

varies with the individual application. For this siX'put arrangements are ihe most common,
reason, chillers arc made available with cither more pears are used in many instances.
single or multiple refrigerant circuits of varying As in the case of the dr\ -expansion chiller,
lengths. For the design shown in Tig. 11-39, some Hooded chillers are deu^ned with remov-
the number and length or the refrigerant circuits able tube bundles, whereas other? have fixed
depend on the lube length and on the arrange- lube sheets. In the fixed tube sheet design, the
ment of the baffling in the end-plates or refriger- tube sheets are welded to the shell so that
ant beads which are bolted to the lube sheets the lube bundle is not removable. However,
at the ends of the chilter. The refrigeranl by unbolting the end-plate* the tubes become
circuitarrangement for any one model chiller readily accessible for cleaning and individual
can be changed by -changing the refrigerant lubes can be removed and replaced if necessary.
heads (Fig. 11 -41 A}' The chillers shown in Fig* LI 3Bfr and 11-39
11-30. Flooded Chilian, Standard flooded employ fixed tube sheets, whereas those in Figs.
chiller designs include both single and multipass M->3Bu and MO have tube huiidlcs.
I

lube arrangements, lor single pass flow, the In some flooded chiller designs, the shell
tubes ere so arranged lhat ihe chilled liquid is only partially filled with tubea in order to
passes through all the lubes simultaneously provide a large vapor-disen^igmg area and rela-
and in only one direction. tively low velocity in the spate above the tubes
Multipass circulation of the chilled liquid (Fig. 11*43). Th n design dim i n.j | e<i the possibility
through the ehilkr is accomplished through the of liquid carry-over into the suction line and
use of baffled end-plates or heads which are therefore is particularly well mi i fed to sudden
boiled lo the ends or the chiller (Fig. 1 1-42), heavy increases in loading
The arrangement of the end-plate baffling In those chiller designs where Ihe shell is
determines the number of passes the chilled completely with tubes, a surge drum or
filled
liquid makes from one to the other before accumulator should be used in separate any
leaving the chiller. Although two, four, and entrained liquid from the vapor before the vapor
EVAPORATORS IW
enter* the suction line. Some flooded chiller* The vertical ihcll-and-lubc chiller shown in
re equipped with bull [-in liquid-suction heal Fig. 11-44 has the advantage of requiring a
exchanger* (Fig. 11-45). Although I he prima ry minimum amount of floor space. The chiller
function of the heal exchanger is to insure that is operated flooded. The chilled liquid enters
oniy dry vapor enter* the suction line, it has the chiller at the top and flows by gravity down
the additional effect of increasing the efficiency the inside of the tubes. A circulating pump
of the chiller in that it subcools the liquid draws chilled liquid from the storage tank at the
approaching the chiller and thereby reduces the bottom and delivers it through the connecting
amount of flash gas that enter* the cooler. piping- The return liquid is piped to the dis-
tributor box at the top. from where it again
flown through the lubes. A specially designed
distributor installed at the lop of each tube
(inset) imparts a. swirling motion to the chilled

liquid, which causes the liquid to flow in a. com-


paratively thin film down the inside tube
turfites,
11-31. Spray-Type Chillers. The spray-type
Short cut on-*** ipKir^g chiller is similar in construction to the conven-
tional flooded chiller cuoep! that the liquid
refrigerant sprayed over the outside of the
is

waier tubes from nozzle* located in a spray


header above the tube bundle (Fig. 20-19). The
unevaporated liquid drains from the tube into a
tump bottom of the chiller from where it
at the
is low head
recirculated to the spray nozzles by
liquid pump. A high recirculation rate assures
continuous wetting of the tube surfaces and
results in a high rale of heat transfer.
The advantages of this type of
principal
chiller is its high cflicicncy and relatively small
refrigerant charge. Disadvantages are the high
taint* tm n**cs -intern*! vww installation cost and the need for a liquid
recirculating pump.
1 1 &. Chiller Selection Procedure. Although
4-FHs selection methodssomewhat depending
differ
JOtiHh
upon the type of chiller and the particular manu-
facturer, all are based on the simple funda-
mentals of heat transfer and fluid now which
have already been described. Almost without
a-ataa exception, manufacturers include sample selec-
2 Circuits tion procedure along with the design and
capacity data in their equipment catalogs. The

•^ following
closely
selection
that given
procedure
in of one
manufacturer for the selection of dry-expansion
chillers-*
the
follows
catalog
very

Example II-*, ]i is desired to cool 50 gpm


of wrner from 54° F to 46° F with a refrigemnt
tempera lure ai measured at the cooler outlet of
Flf. I Ml. (o) Baffla ipiCinf in d ry-** pwulon ehil- 40- F usinj; Refrigerant* 2. 1

kfi. (D) Typ+oJ refrliertnt headl far dry-*Kp*nile>n


chiller. (CDurte»r of Acme InduitrkM.) * Acme Industrie*. Inc.
W PRINCIPLES OF REFRIGERATION

Fig. 11*41. Flooded thlller d**F(n«d far multlp»I circulation of chiliad liquid. I",>. circulation i* pm
ttcompliihed by mean* of th* C4ffl«d *fld-£l«*-, or writer heads which ir* bc-lud to Ihe endi of the chiller.
{CourtHr Vilter Minu^ctuhni Company.)

Solution produces the highest pumping had Hence, if


Step 1. Determine ihe total chiller load in space it not problem, ihe must logical choice
tons. would lean to be type 8M U-wcvcr, a check
I

Gpm x 500 x cooling range


1 2.000 Blu/hr/ton
SO x 500 x (54 - 46)
— 16.7 ions
> 2,000

Step 2. Determine the mean effective tem-


perature difference (METD),
Water in minus refrig-
erant temperature 54 — 40 14* F LTD
Water out minus re-
frigerant temperature -Mi - 40 — 6" F STD
From Table II -1, METD - 9.47* F
Step 3. Select trial chiller {shell diameter and
baffles spacing) from Fig, of Table R-9. Enter
I

Fig. 1 at 50 gpm on the lower vertical scale and


move horizontally across the chart to the
diagonal line representing the type unit desired,
The number indicates shell diameter and the
letter indicates baffle spicing, Possible choices
are 10M, I2L, SM, 12K, 10K, and SL, As a
general rule, small diameter chillers are more Fig. I ML Flooded chiller with shell only partially
economical, whereas large diameter chillers are filled with tubst
order to provide i lirgc vapor
In

more compact. Type M


burning produces the d-Hniafln & tree ibovi the tubri. {CourtMy Worth-
lowest pumping head, whereas type L baffling ln|tdn Corporation.)
EVAPORATORS 197

will show that neither 8M nor 8L is available 14 ft DXH chiller has a surface area of 184 sq ft
with sufficient surface area in this instance. (Model No. DXH-1014).
Therefore, select type 10M (8 to 30 tons). From Step 6. Determine the water pressure drop
the point of intersection move vertically upward through the chiller. From the bottom of Fig. 1,
to a diagonal line in the upper portion of Fig. 1 Table R-9, the pressure drop per foot of length
which represents a METD of 9.47° F as found with type Mbaffling is 0.425 ft of water column.

Gas outlet

Water
inlet

Refrigerant
feed
Inset

Fig. 11-44. Vertical shell-and-tube "Spira-Flo" chiller designed for flooded operation. The water flowing
down through the tubes is given a swirling action by specially designed nozzles (inset). (Courtesy Worthington
Corporation.)

in Step 2. From this intersection move hori- Pressure drop = length (feet) x pressure drop/
zontally to the scale at the left margin and read foot
the loading of 1110 Btu/hr/sq ft (loading is the
U value times the METD). 14 feet x 0.425 = 5.95 ft HaO
Step 4.

Surface area -^^


Determine the surface area required.

200.000
-^5--
Loading

180.2 sq ft
11-33. Direct and Indirect Systems.
heat transfer surface into which a volatile liquid
(refrigerant) is expanded and evaporated in
order to produce a cooling effect is
Any

called a
"direct-expansion" evaporator and the liquid
Step 5. Select chiller length from Table R-9 so evaporated is called a "direct-expansion"
to meet surface area requirements. 10 in. x A refrigerant. A direct-expansion or "direct"
198 PRINCIPLES OF REFRIGERATION
WWW/WW/WW/W/W,
Refrigerated
Brine coil "V.
space or

c material

V/s;///;ss///s;;j;ss;;ss;;;//v.
Cold brine
to coil

Warm brine_
"
to chiller

Liquid from /" Refrigerant control

ver
receiver
* —&-i jfVl

Vapor to
I Brine solution!
compressor
suction

?mrm/m/m///??r#mrA Brine pump

Fig. I M5o. Indirect system.

Brine coil

t4- 7~\
Cold air to
refrigerated space

-Air duct
Warm brine
to chiller
Cold brine
Warm air
to coil
Refrigerant control from space

receiver
,^^BZ^mB^.
'**r .

Vapor to rx. s; Brine solution {


compressor z|
suction
^ vM
..
£ "
.
wwiw hw p wbhw/ *
i >7
^ ^-Brine
-Brine pump

Fig. Il-45b. Indirect system —brine coil in communicating duct.

refrigerating system is one wherein the system is required. In such cases, an indirect refrigerat-
evaporator, employing a direct-expansion refrig- ing system is employed. Water or brine (or
erant, is in direct contact with the space or some other suitable liquid) ischilled by a direct-
material being refrigerated, or is located in air expansion refrigerant in a liquid chiller and
ducts communicating with such spaces. Up to then pumped through appropriate piping to the
this point, only direct refrigerating systems have space or product being refrigerated. The
been considered. chilled liquid, called a secondary refrigerant,
Very often it is either inconvenient or un- may be circulated directly around the refriger-
economical to circulate a direct-expansion ated product or vessel or it may be passed
refrigerant to the area or areas where the cooling through an air-cooling coil or some other type
EVAPORATORS 199

of heat transfer surface (Fig. 11-45). In either bottom of the spray unit, from where it is
case, the secondary refrigerant, warmed by the returned to the chiller. Since the air passing
absorption of heat from the refrigerated space through the water spray below its dew
is chilled
or product, is returned to the chiller to be point temperature, a certain amount of water
chilled and recirculated. vapor is condensed from the air and is carried
Indirect refrigerating systems are usually to the basin with the spray water. With either
employed to an advantage in any installation the cooling coil or the spray unit, the amount
where the space or product to be cooled is of cooling and dehumidification can be con-
located a considerable distance from the con- trolled by varying the amount and temperature
densing equipment. The reason for this is that of the chilled water.
long direct-expansion refrigerant lines are Water is also used frequently as a secondary
seldom practical. In the first place, they are refrigerant in small beverage coolersand in
expensive to install and they necessitate a large farm coolers designed for cooling milk cans.
refrigerant charge. Too, long refrigerant lines, In such cases, the water, because of high its
particularly long risers, create oil return prob- conductivity, permits more rapid of the chilling
lems and cause excessive refrigerant pressure product than would be possible with air. Too,
losses which tend to reauce the capacity and the water supplies a holdover capacity which
efficiency of the system. Furthermore, leaks tends to level out load fluctuations resulting
are more serious and are much more likely to from intermittent loading of the cooler.
occur in refrigerant piping than in water or 11-35. Brines. Obviously, water cannot be
brine piping. employed as a secondary refrigerant in any
Indirect refrigeration is required also in many application where the temperature to be main-
industrial process cooling applications where tained is below the freezing point of water. In
it is often impractical to maintain a vapor tight such cases, a brine solution is often used.
seal around the product or vessel being cooled. Brine is the name given to the solution which
Too, indirect systems are used to an advantage results when various salts are dissolved in water.
in any application where the leakage of refriger- If a salt is dissolved in water, the freezing tem-
ant and/or oil from the lines may cause con- perature of the resulting brine will be below the
tamination or other damage to a stored product. freezing temperature of pure water. Up to a
The latter applies particularly to meat packing certain point, the more salt dissolved in the
plants and large cold storage applications when solution, the lower will be the freezing tempera-
ammonia is used as a refrigerant. ture of the brine. However, if the salt concen-
1 1-34. Secondary Refrigerants. Some com- tration is increased beyond a certain point, the
monly used secondary refrigerants are water, freezing temperature of the brine will be raised
calcium chloride and sodium chloride brines, rather than lowered. Hence, a solution of any
ethylene and propylene glycols, Methanol salt in water has a certain concentration at
(methyl alcohol), and glycerin. which the freezing point of the solution is lowest.
Almost without exception, water is used as A solution at the critical concentration is called
the secondary refrigerant in large air condition- a eutectic solution. At any concentration above
ing systems and also in industrial process cooling or below this critical concentration, the freezing
installations where the temperatures maintained temperature of the solution will be higher, that
are above the freezing point of water. Water, is, above the eutectic temperature.* When the

because of its fluidity, high specific heat value, salt content of the brine is less than that which
and high film coefficient, is an excellent is required for a eutectic solution, the excess
secondary refrigerant. It also has the advantage water will begin to precipitate from the solution
of being inexpensive and relatively noncorrosive. in the form of ice crystals at some temperature
In air conditioning applications, the chilled above the eutectic temperature. The exact tem-
water is circulated through an air cooling coil perature at which the ice crystals will begin to
or through a water spray unit. In either case, * At any temperature other than the eutectic
the air is both cooled and dehumidified. In the temperature, the term "freezing temperature" is
water spray unit, the water i. sprayed from used to mean the temperature at which ice or salt
nozzles and collected in a pan or basin at the crystals begin to precipitate from the solution.
200 PRINCIPLES OF REFRIGERATION

form depends upon the degree of the salt con- (2) sodium chloride. The two brines are pre-
centration and upon the relative solubility of the pared from calcium chloride (CaCl^ and sodium
salt in water, the latter factor decreasing as chloride (NaCl) salts, respectively, the latter
the temperature of the solution decreases. The salt being the common table variety.
continued precipitation of ice crystals from the Calcium chloride brine is used primarily in
solution as the temperature is reduced causes a industrial process cooling, in product freezing
progressive increase in the concentration of the and storage, and in other brine applications
remaining brine until, at the eutectic tempera- where temperatures below 0° F are required.
ture,a slush consisting of ice and eutectic brine The lowest freezing temperature which can be
The further removal of heat from this
will exist. obtained with calcium chloride brine (the eutec-
mixture will result in solidification of the eutec- tictemperature) is approximately —67° F. The
tic brine. Solidification of the eutectic brine will saltconcentration in the eutectic solution is
take place at a constant temperature. approximately 30% by weight. The freezing
On the other hand, when the salt content of temperature of various concentrations of cal-
the brine is in excess of the amount required for cium chloride brine are given in Table 11-3,
a eutectic solution, the excess salt will begin to along with some of the other important pro-
precipitate from the solution in the form of salt pertiesof the brine.
crystals at some temperature above the eutectic The principal disadvantage of calcium
temperature. Continued precipitation of salt chloride brineis its dehydrating effect and its

from the mixture as the temperature is reduced tendency to impart a bitter taste to food pro-
will result in a mixture of salt and eutectic brine ducts with which it comes in contact. For this
when the eutectic temperature is reached. The reason, when calcium chloride brine is used in
further removal of heat from the mixture will food freezing applications, the system must be
result in solidification of the eutectic brine at designed so as to prevent the brine from coming
constant temperature. into contact with the refrigerated product.
Two types of brine are commonly used in Sodium chloride brine is employed mainly

refrigeration practice: (1) calcium chloride and in those applications where the possibility of
product contamination prevents the use of
calcium chloride brine. Sodium chloride brine
Air
-
is employed extensively in installations where
the chilling and freezing of meat, fish, and other
products are accomplished by means of a brine
spray or fog.
The lowest temperature obtainable with
sodium chloride brine is approximately —6° F.
For this freezing temperature the salt concen-
tration in the solution is approximately 23%.
The thermal properties of sodium chloride brine
Brine from
at various concentrations is given in Table 11-4.
chiller
of interest to notice that the thermal

iff Mr M Brine to
It is
properties of both calcium chloride and sodium
chloride brines are somewhat less satisfactory
1r
ii '"
Jl concentrator
than those of water.
brines is
As the salt content of the
increased, the fluidity, specific heat
value, and thermal conductance of the brines all
Air
m i Concentrator
decrease. Hence, the stronger the brine solu-
\, Brine from tion, the greater the quantity of brine that must
I concentrator
be circulated in order to produce a given
*" Brine to
refrigerating effect.
chiller
I Brine Since the specific gravity of the brine increases
as the salt concentration increases, the degree of
Fig. 1 1-46. Brine spray cooler. salt concentration and the thermal properties of
EVAPORATORS 201

Air -*

Brine to

M Eliminators 1 ut concentrator

Fig. 1 1-47. Brine spray cooler. Brine WW- To condensing


unit

Direct
expansion MhHAai.'i
evaporator

the brine can be determined by measuring the advantages of glycol solutions, they are being
of the brine with a hydrometer.
specific gravity used to replace brines in a number of installa-
11-36. Antifreeze Solutions. Certain water tions, particularly in the brewing and dairy
soluble compounds, generally described as anti- industries. The change-over from brine to glycol
freeze agents, are often used to depress the can be accomplished with practically no change
freezing point of water. The more widely known in the plant facilities.
antifreeze agents are ethylene glycol, propylene 1 1-37. Brine Spray Units. Like chilled water,
glycol, Methanol (methyl alcohol), and glycerin. the chilled brine (or antifreeze solution)may be
All thesecompounds are soluble in water in all circulated directly around the refrigerated pro-
proportions. The freezing temperature of water duct or container, or it may be used to cool the
in solution with various percentages of each of air in a •refrigerated space. When used to cool
these compounds is given in Table 11-5. air, the chilled brine is circulated through a

Propylene glycol is probably the most exten- serpentine coil or through a brine spray unit.
sively used antifreeze agent in refrigeration Two types of brine spray units which have been
service. In common with ethylene glycol, used extensively are shown in Figs. 11-46 and
propylene glycol has a number of desirable 11-47. In the former unit chilled brine from a
properties. Unlike brine, glycol solutions are brine chiller located outside the refrigerated
noncorrosive. They are also nonelectrolytic and space is- sprayed down from spray nozzles and
therefore may be employed in systems containing collected in the basin of the unit, from where it

dissimilar metals. Being extremely stable com- is returned to the brine chiller. In the latter type
pounds, glycols will not evaporate under normal the brine is chilled by a direct-expansion coil
operating conditions. Because of the many located within the brine spray unit itself.
202 PRINCIPLES OF REFRIGERATION

PROBLEMS Based on an evaporator TD of 10° F, determine


the size and number of individual freezer plates
1. Awalk-in cooler 8 ft by 9 ft by 9 ft high has to be used as shelves in the freezing cabinet.
walls 6 in. thick and is maintained at a tempera-
ture of 35° F. The load on the cooler is 7500
3. The load on a tank-type brine cooler is

Btu/hr.
4500 Btu/hr. The brine is to be maintained at
(a) Select a natural convection cooling coil a temperature of 35° F with a refrigerant tem-
which will produce a perature of 19° F. Assuming little or no
(Plasti-Cooler)
agitation of the brine, determine the lineal feet
relative humidity of approximately 85%
in the cooler.
of | in. pipe required for the evaporator.
(6) Select a Unit Cooler which will produce 4. It is desired to cool 100 gpm of water from
approximately the same conditions in the 56° Fto 46° F with a refrigerant temperature
cooler. 38° Fat the cooler outlet. Select an appro-
2. A freezing cabinet 6 ft high, 25 in. deep, and priate chiller and determine the water pressure
80 in. wide has a freezing load of 3600 Btu/hr. drop through the chiller in psi.
the performance of reciprocating compressors
will apply also to the performance of rotary and
centrifugal compressors.
12-2. The Compression Cycle. Before at-
tempting to analyze the performance of the
compressor, it is necessary to become familiar
with the series of processes which make up

12 the compression cycle of a reciprocating


pressor.
com-

A compressor, with the piston shown at four


points in its travel in the cylinder, is illustrated

Performance in Fig. 12-1.


on the suction
As the piston
stroke, low-pressure
moves downward
vapor from
the suction line drawn
of Reciprocating through the suction valves.
is into the cylinder
On the upstroke of
the piston, the low-pressure vapor com-
Compressors pressed and then discharged as a high-pressure
is first

vapor through the discharge valves into the head


of the compressor.
To prevent the piston from striking the valve
compressors are designed
plate, all reciprocating
with a small amount of clearance between the
12-1. Refrigeration Compressors. Vapor
top of the piston and the valve plate when the
compressors used in refrigeration are of three
principal types:
piston is at the top of its stroke. The volume of
(1) reciprocating, (2) rotary,
this clearance space is called the clearance
and (3) centrifugal. Of the three, the recipro-
volume and is the volume of the cylinder when
cating compressor is by far the" one most
the piston is dead center.
at top
frequently used.
Not all the high-pressure vapor
will pass out
Rotary compressors are limited to use in very
through the discharge valves at the end of the
small fractional horsepower applications, such
compression stroke. A certain amount will
as home refrigerators and freezers and small
remain in the cylinder in the clearance space
commercial applications. Even in this limited
between the piston and the valve plate. The
area, rotary compressors represent only a small
vapor which remains in the clearance space at
fraction of the total number used. Some rotary
the end of each discharge stroke is called the
compressors are used also as booster compres-
clearance vapor.
sors.* Their use for this purpose appears to be
Reference to Figs. 12-2 and 12-3 will help to
increasing.
clarify the operation of the compressor. Figure
Centrifugal compressors are used only on
1 2-2 is a time-pressure diagram in which cylinder
very large applications, usually at least SO tons
pressure is plotted against crank position.
or above. In this area, they are widely accepted
Figure 12-3 a theoretical pressure- volume
is
and are rapidly increasing in number because
diagram of a typical compression cycle. The
the number of large applications is growing
on theTP andPV diagrams corre-
lettered points
steadily.
spond to the piston positions as shown in
Only the performance of reciprocating com-
Fig. 12-1.
pressors will be discussed in this chapter.
At point A, the piston is at the top of its
Reciprocating compressor design, along with
stroke,which is known as top dead center. When
the design and performance of rotary and centri-
the piston is at this position, both the suction
fugal compressors, is discussed elsewhere in the
and discharge valves are closed. The high
text ata more appropriate time and place.
pressure of the vapor trapped in the clearance
However, much that is said in this chapter about
space acts upward on the suction valves and
* Booster compressors are discussed in Chapter holds them closed against the pressure of the
20. suction vapor in the suction line. Because the
203
204 PRINCIPLES OF REFRIGERATION
Discharge^
pressure of the vapor in the head of the com-
pressor is approximately the same as that of the
vapor in the clearance volume, the discharge
valves are held closed either by their own weight
or by light spring loading.
As the piston moves downward on the suction
stroke, the high-pressure vapor trapped in the
clearance space is allowed to expand. The

180
Crank position

Fig. 12-2. Theoretical time-pressure diagram of


compression cycle in which cylinder pressure is
plotted against crank position.

piston reaches the bottom of its stroke at point


C. During the time that the piston is moving
from B to C, the cylinder is filled with suction
vapor and the pressure in the cylinder remains
constant at the suction pressure. At point C,
the suction valves close, usually by spring action,
and the compression stroke begins.
The pressure of the vapor in the cylinder
increases along line C-D as the piston moves
upward on the compression stroke. By the time
the piston reaches point D, the pressure of the
vapor in the cylinder has been increased until
it is higher than the pressure of the vapor in the

head of the compressor and the discharge valves


are forced open; whereupon the high-pressure
vapor passes from the cylinder into the hot gas
line through the discharge valves. The flow of

Fig. 12-1. (a) Piston at top dead center, (b) Suction


valves open, (c) Piston at bottom dead center, (d)

Discharge valves open.

expansion takes place along line A-B so that


the pressure in the cylinder decreases as the
volume of the clearance vapor increases. When
the piston reaches point B, the pressure of the
re-expanded clearance vapor in the cylinder
Clearance
becomes slightly less than the pressure of the
vapor in the suction line; whereupon the suction
Volume of
valves are forced open by the higher pressure in re-expanded • Volume

the suction line and vapor from the suction line clearance vapor

flows into the cylinder. The flow of suction


vapor into the cylinder begins when the suction Fig. 12-3. Pressure-volume diagram of typical com-
valves open at point B and continues until the pression cycle.
PERFORMANCE OF RECIPROCATING COMPRESSORS 205

the vapor through the discharge valves con- Example 12-1. Calculate the piston dis-
tinues as the piston moves from D to A while placement of a two cylinder compressor
the pressure in the cylinder remains constant at rotating at 1450 rpm, if the diameter of the
the discharge pressure. When the piston returns cylinder is 2.S in. and the length of stroke is 2
in.
to point A, the compression cyclecompleted is

and the crankshaft of the compressor has Solution. Substituting in Equation 12-1,
rotated one complete revolution. 3.1416 x (2.5)* x 2 x 1455 x 2
12-3. Piston Displacement. The piston dis- 4 x 1728
placement of a reciprocating compressor is the = 16.52 cu ft/min
total cylinder volume swept through by the 12-4.Theoretical Refrigerating Capacity.
piston in any certain time interval and is usually
The refrigerating capacity of
any compressor
expressed in cubic feet per minute. For any
depends upon the operating conditions' of the
single-acting, reciprocating compressor, the pis-
system and, like system capacity, is determined
ton displacement is computed as follows:
by the weight of refrigerant circulated per unit
of time and by the refrigerating effect of each
v,--
n-Z)* x L x Nxn
(12-1) pound circulated.*
4 x 1728
The weight of refrigerant circulated per
where V„ -» the piston displacement in cubic minute by the compressor is equal to the weight
feet per minute of the suction vapor that the compressor
D = the diameter of the cylinder (bore) compresses per minute. If it is assumed that the
in inches compressor is 100% efficient and that the
L = the length of stroke in inches cylinder of the compressor fills completely with
N = revolutions of the crankshaft per suction vapor at each downstroke of the piston,
minute (rpm) the volume of suction vapor drawn into the
n — thenumber of cylinders compressor cylinder and compressed per minute
will be exactly equal to the piston displacement
The volume of the cylinder which is swept
of the compressor. The weight of this volume
through by the piston each stroke (each revolu-
of vapor, which is the weight of refrigerant
tion of the crankshaft) is the difference between
circulated per minute, can be calculated by
the volume of the cylinder when the piston is
multiplying the piston displacement of the
at the bottom of its stroke and the volume of
compressor by the density of the suction vapor
the cylinderwhen the piston is at the top of its
at the compressor inlet.
stroke. This part of the cylinder volume is
Once the weight of refrigerant compressed
found by multiplying the cross-sectional area of
per minute by the compressor has been deter-
the bore by the length of stroke. Thus:
mined, the theoretical refrigerating capacity of
ttD* the compressor in tons can be found by multi-
Cross-sectional area of the
plying the weight of refrigerant compressed per
bore in square inches
minute by the refrigerating effect per pound and
Volume of cylinder swept then dividing by 200.
through by the piston each
stroke in cubic inches
xL Example 12-2. The compressor in Example
12-1 is operating on a R-12 system at a suction
Once the cylinder volume known, the total
is
temperature of 20° F. If the suction vapor
reaching the compressor inlet is saturated and
cylinder volume swept through by the piston of
if the temperature of the liquid at the refrigerant
a single cylinder compressor each minute in
control is 100° F, determine
cubic inches can be determined by multiplying
(a) the total weight of refrigerant circulated
the cylinder volume by the rpm (N). When the per minute
compressor has more than one cylinder, the (6) the theoretical refrigerating capacity of
cylinder volume must also be multiplied by the the compressor in tons.
number of cylinders («). In either case, dividing * Since it is the compressor which circulates the
the result by 1728 will give the piston displace- refrigerant through the system, compressor capacity
ment in cubic feet per minute. and system capacity are one and the same.
206 PRINCIPLES OF REFRIGERATION

Solution (c) Piston displacement of


(a) From Example 12-1, compressor = 16.52 cu ft/min
piston displacement = 16.52 cu ft/min
Theoretical refriger- _ 16.52
From Table 16-3, den- ating capacity of ~ 4.55
sity of R- 12 saturated
=
compressor in tons = 3.63 tons
vapor at 20° F 0.8921 lb/cu ft
Weight of refrigerant = 16.52 x 0.8921
circulated per minute = 14.74 lb/min 12-5. Actual Refrigerating Capacity. The
From Table 16-3,
actual refrigerating capacity of a compressor is
(b)
enthalpy of R-12 satur- always less than its theoretical capacity as
ated vapor at 20° F = 80.49 Btu/lb calculated in the previous examples. In the
Enthalpy of R-12 satur- preceding examples it has been assumed: (1)
ated liquid at 100° F = 31.16 Btu/lb that at each downstroke of the piston the cylinder
Refrigerating effect = 49.33 Btu/lb of the compressor fills completely with suction
Theoretical refrigerating vapor from the suction line and (2) that the
capacity of com- = 14.74 x 49.33 density of the vapor filling the cylinder is the
pressor = 727.12 Btu/min same as that in the suction line.
Theoretical refriger- 727.12 If these assumptions were correct, the actual
ating capacity in 200 refrigerating capacity would be exactly equal to
tons = 3.63 tons the theoretical capacity. Unfortunately, this is

Since specific volume is the reciprocal of not the case. Because of the compressibility
density, an alternate method of determining of the refrigerant vapor and the mechanical
the weight of refrigerant circulated per minute clearance between the piston and the valve plate
by the compressor is to divide the piston of the compressor, the volume of suction vapor
displacement of the compressor by the specific the cylinder during the suction stroke is
filling

volume of the suction vapor at the compressor always less than the cylinder volume swept
inlet. through by the piston. Too, it will be shown
When the volume of vapor to be circulated later that the density of the vapor filling the
per minute per ton for any given operating cylinder is less than the density of the vapor
conditions is known, the capacity of the com- For these reasons, the actual
in the suction line.
pressor in tons for the operating conditions in volume of suction vapor at suction line condi-
question may be found by dividing the piston tions which is drawn into the cylinder of the
displacement of the compressor by the volume compressor is always less than the piston
of vapor to be compressed per minute per ton. displacement of the compressor and, therefore,
the actual refrigerating capacity of the com-
Example 12-3. For conditions of
the
pressor is always less than its theoretical
Example 12-2, find (a) the weight of refrigerant
circulated per minute per ton ; (6) the volume of capacity.

vapor to be compressed per minute per ton; 12-6.Total Volumetric Efficiency. The
and (c) the theoretical refrigerating capacity of actualvolume of suction vapor compressed per
the compressor in tons. minute is the actual displacement of the com-
Solution
pressor. The ratio of the actual displacement of
the compressor to its piston displacement is
(a) From Example 12-2,
refrigerating effect = 49.33 Btu/lb known as the total or real volumetric efficiency

Weight of refrigerant of the compressor. Thus:


_ 200
circulated per minute ~~
4933
Per ton = 4.05 lb/min E -* x 100 (12-2)
(b) From Table 16-3,
volume of
specific
R-12 saturated vapor where Ev = the total volumetric efficiency
at20°F = 1.121 cu ft/lb Va = actual volume of suction vapor
Volume of vapor to be compressed per minute
compressed per =4.05 x 1.121 Vv — the piston displacement of (he
minute per ton = 4.55 cu ft/min compressor
PERFORMANCE OF RECIPROCATING COMPRESSORS 207

or space after the discharge valves close. The


Actual weight of suction vapor vapor left in the clearance space has been
compressed x 100 compressed to the discharge pressure and, at
Ev = the beginning of the suction stroke, this vapor
Theoretical weight of suction vapor
compressed must be re-expanded to the suction pressure
before the suction valves can open and allow
When com-
the volumetric efficiency of the
vapor from the suction line to flow into the
pressor known, the actual displacement and
is
cylinder. The piston will have completed a part
refrigerating capacity can be found as follows:
of its suction stroke and the cylinder will already
Ev be partially filled with the re-expanded clearance
Vn - K. x (12-3)
vapor before the suction valves can open and
100
and admit suction vapor to the cylinder. Hence,
Actual Theoretical suction vapor from the suction line will fill only
refrigerating = refrigerating x —^- (12-4) that part of the cylinder volume which is not

capacity capacity already filled with the re-expanded clearance


vapor.
Example 12-4. If the volumetric efficiency
In Fig. 12-3, Vc is the total volume of the
of the compressor in Example 12-3 is 76%,
cylinder when is at the bottom of its
the piston
determine: (a) the actual volumetric displace-
ment (b) the actual refrigerating capacity.
stroke. Va
which represents the clearance
,

volume, is the volume occupied by the clearance


Solution vapor at the end of the compression stroke.
(«) From Example 12-1, The difference between Vc and Va then is the
piston displacement 16.52 cu ft/min
volume of the cylinder swept through by the
Actual volumetric 16.52 x 0.76 piston each stroke. On the down stroke of the
displacement 12.66 cu ft/min
piston, the clearance vapor expands from Va to
(b) From Example 12-3, Vb before the suction valves open. Therefore,
theoretical refrigerating
the part of the cylinder volume which is filled
capacity 3.63 tons
with suction vapor during the balance of the
Actual refrigerating 3.63 x 0.76
suction stroke is the difference between Vj,
capacity 2.76 tons
and Ve .

The actual refrigerating capacity of the 12-9. Theoretical Volumetric Efficiency.


compressor may also be determined as in
'
The volumetric a compressor due
efficiency of
Examples 12-2 and 12-3, if actual displacement to the clearance factor alone is known as the
is substituted for piston displacement. theoretical volumetric efficiency. It can be
12-7.Factors Influencing Total Volumetric shown mathematically that the theoretical
Efficiency. The factors which tend to limit the volumetric efficiency varies with the amount of
volume of suction vapor compressed per work- clearance and with the suction and discharge
ing stroke, thereby determining the volumetric pressures. The reason for this is easily explained,
efficiency of the compressor, are the following: 12-10. Effect of Increasing the Clearance.
If the clearance volume of the compressor is
1. Compressor clearance
increased in respect to the piston displacement,
2. Wiredrawing
the percentage of high-pressure vapor remaining
3. Cylinder heating
in the cylinder at the end of the compression
4. Valve and piston leakage
stroke will be increased. When re-expansion
12-8. The Effect of Clearance on Volumetric takes place during the suction stroke, a greater
Efficiency. Because of compressor clearance percentage of the total cylinder volume will be
and the compressibility of the refrigerant vapor, filled with the re-expanded clearance vapor and
the volume of suction vapor flowing into the the volume of suction vapor taken in per stroke
cylinder is less than the volume swept through will be less than when the clearance volume is

by the piston. As previously shown, at the end smaller. To obtain maximum volumetric
of each compression stroke a certain amount of efficiency, the clearance volume of a vapor
vapor remains in the cylinder in the clearance compressor should be kept as small as possible.
208 PRINCIPLES OF REFRIGERATION
It should be noted that this does not hold Examination of Equation 12-3 indicates that
true for a reciprocating liquid pump. Since a the compression ratio is increased by either

liquid is not compressible, the liquid left in the increasing the discharge pressure or lowering
clearance space at the end of the discharge the suction pressure, or both.
stroke has the same specific volume as the liquid In the preceding section it was shown that
at the suction inlet. Therefore, there is no increasing the discharge pressure or lowering
re-expansion of the liquid in the clearance during the suction pressure decreases the volumetric
the suction stroke and the volume of liquid efficiency. It follows, then, that when the suction
taken in each stroke is always equal to the and discharge pressures are varied in such a
volume swept by the piston, regardless of direction that the compression ratio is increased,
clearance. the volumetric efficiency of the compressor
12-1 1. Variation with Suction and Discharge decreases. Likewise, decreasing the compression
Pressures. Increasing the discharge pressure or ratio will increase the volumetric efficiency. For
lowering the suction pressure will have the same a compressor of any given clearance, the volu-
effect on volumetric efficiency as increasing the metric efficiency varies inversely with the
clearance. If the discharge pressure is increased, compression ratio.
the vapor in the clearance will be compressed 12-13. The Effects of Wiredrawing. Wire-
to a higher pressure and a greater amount of drawing is defined as a "restriction of area for a
re-expansion will be required to expand it to flowing fluid, causing a loss in pressure by
the suction pressure. Likewise, if the suction (internal and external) friction without the loss
pressure is lowered, the clearance vapor must of heat or performance of work; throttling."*
experience a greater re-expansion in expanding In order to have a flow of vapor from the
to the lower pressure before the suction valves suction line through the suction valves into the
will open. compressor cylinder, there must be a pressure
On the other hand, for a constant discharge differential across the valves sufficient to over-
pressure, the amount of re-expansion that the come the spring tension of the valves and valve
clearance vapor experiences before the suction weight and inertia. This means that the
valves open diminishes as the suction pressure suction vapor experiences a mild, throttling
rises. It is evident, then, that the volumetric expansion or drop in pressure as it flows
efficiency of the compressor increases as the through the suction valves and passages of the
suction pressure increases and decreases as the compressor. Therefore, the pressure of the
discharge pressure increases. suction vapor filling the cylinder of the com-
12-12. Compression Ratio. The ratio of the pressor is always less than the pressure of the
absolute suction pressure to the absolute vapor in the suction line. As a result of the
discharge pressure is called the compression expanded condition of the vapor filling the
ratio. Thus, cylinder, the volume of suction vapor taken in

Absolute discharge pressure from the suction line each stroke is less than
JK —— r-: :
. (12-3) if the vapor filling the cylinder was at the
Absolute suction pressure
suction line pressure.
where R = the compression ratio. A similar pressure differential is required

Example across the discharge valves in order to cause the


12-5. Calculate the compression
ratioof a R-12 compressor when the suction discharge vapor to flow through the valves
temperature is 20° F and the condensing tem- into the condenser. To provide the necessary
perature is 100° F. pressure differential across the discharge valves,
the vapor in the cylinder must be compressed to
Solution. From Table 16-3,
a pressure somewhat higher than die actual
absolute pressure of R-12 satur-
ated vapor at 20° F = 35.75 psi
condensing pressure. The vapor left in the
Absolute pressure of R-12 clearance space at the end of the di charge
saturated vapor at 100° F = 131.6 psi stroke will be at this higher pressure. To re-
Compression ratio _ 131.6 expand from this higher pressure during the
-
35?75
= 3.69 * Asre Data Book, 1957-58 (page 39-27).
PERFORMANCE OF RECIPROCATING COMPRESSORS 209

suction stroke, the clearance vapor must suffer and other compressor parts so that the transfer
a greater amount of re-expansion than if it had of heat to the suction vapor occurs at a higher
been compressed only to the condensing pres- rate.
sure. As a result of the greater expansion of 12-15. The Effect of Piston and Valve Leak-
the clearance vapor, a larger portion of the age. Any back leakage of gas through either the
cylinder volume is filled with the re-expanded suction or discharge valves or around the piston
clearance vapor during the down stroke of the will decrease the volume of vapor pumped by
piston and the amount of suction vapor drawn the compressor. Because of precision manu-
in from the suction line is reduced. is very little leakage of
facturing processes, there
Unlike the other factors which determine gas around the pistons of a compressor in good
volumetric efficiency, wiredrawing is not directly condition. However, since it is not possible to
affected by the compression ratio. In general, design valves that will close instantaneously,
wiredrawing a function of the velocity of the
is there is always a certain amount of back
refrigerant vapor flowing through the valves and leakage of gas through the suction and discharge
passages of the compressor. As the velocity of valves.
the vapor through the valves is increased, the As the pressure in the cylinder is lowered at
effectof wiredrawing increases. the beginning of the suction stroke, a small
The refrigerant velocity through the valves of amount of high-pressure vapor in the head of
a compressor depends upon the design of the the compressor will leak back into the cylinder
valves, the refrigerant used, and the speed of the before the discharge valves can close tightly.
compressor. Similarly, at the start of the compression stroke,
Wiredrawing is greatest for those refrigerants some of the vapor in the cylinder will flow
having the greatest volumes and the
specific back through the suction valves into the, suction
lowest latent heat values because the volume line before the suction valves can close.
of vapor circulated per ton of refrigerating To assure prompt closing of the valves, both
capacity is greater. This accounts for the large the suction and discharge valves are usually
wiredrawing effect associated with R-12. constructed of lightweight materials and are
Increasing the speed of the compressor slightly spring loaded. However, since the spring
increases the piston displacement. Hence, the tension increases wiredrawing, the amount of
velocity of the vapor through the valves and the spring loading is critical.

effects of wiredrawing are increased as the rpm For any given compressor, the amount of
are increased. backleakage through the valves is a function of
12-14. The Effects of Cylinder Heating. the compression ratio and the speed of the
Another factor which tends to reduce the compressor. The higher the compression ratio,
volumetric efficiency of the compressor is the the greater is the amount of valve leakage.
heating of the suction vapor in the compressor The effect of compressor speed on valve
cylinder. The suction vapor entering the leakage is discussed later.
compressor cylinder is heated by heat con- 12-16. Determining the Total Volumetric
ducted from the hot cylinder walls and by Efficiency. The combined effects of all of the
friction which results from the turbulence of the foregoing factors on the volumetric efficiency of
vapor in the cylinder and from the fact that the the compressor varies with the design of the
refrigerant vapor is not a perfect gas. The compressor and with the refrigerant used.
heating causes the vapor to expand after Furthermore, any one compressor the
for
entering the cylinder so mat a smaller weight of volumetric efficiency is not a constant amount;

vapor will fill the cylinder and thereby still it changes with the operating conditions of the
further reduce the volume of vapor taken in system. Therefore, the total volumetric effi-

from the suction line. ciency of a compressor is difficult to predict


Cylinder heating increases as the compression mathematically and can be determined with
ratio increases. At high compression ratios, accuracy only by actual testing of the compressor
the work of compression is greater and the in a laboratory.
discharge temperature is higher. This causes However, the results of such tests indicate

a rise in the temperature of the cylinder walls that the volumetric efficiency of any one
210 PRINCIPLES OF REFRIGERATION
of the suction vapor, each cubic foot of vapor
compressed by the compressor will represent a
greater weight of refrigerant when the suction
efficiency

temperature is high than when the suction


temperature is low. This means that for any
Volumetric
given position displacement, the weight of
refrigerant circulated by the compressor per
unit of time increases as the suction temperature
increases.
2 3 4 5 6 7 8 9 10 11 12 13 14 The effect of suction temperature on com-
Compression ratio
pressor capacity is best illustrated by an actual
Fig. 12-4. Effect of compression ratio on volumetric example.
efficiency of R-12 compressor.
Example 12-6. Assuming 100% efficiency,
ifthe liquid reaches the refrigerant control at
100° F in each case, determine the weight
compressor is primarily a function of the
compression ratio and, for any given com- of refrigerant circulated per minute and the
theoretical refrigerating capacity of the com-
pression ratio, remains practically constant,
pressor in Example 12-1 when operating at each
regardless of the operating range. It has been
of the following suction temperatures: (a) 10° F
determined also that compressors having the
and (b) 40° F.
same design characteristics will have approxi-
mately the same volumetric efficiencies, regard- Solution
less of the size of the compressor. (a) From Table 16-3,
The relationship between the compression density ofR-12 satur-
ratio and the volumetric efficiency of a typical ated vapor at 10° F 0.7402 lb/cu ft

R-12 compressor is illustrated by the curve in From Example 12-1,


Fig. 12-4. In addition, in order to facilitate piston displace-
future calculations, the average volumetric ment = 16.52 cu ft/min
efficiencies of a group of typical R-12 com- Weight of refrigerant
pressors at various compression ratios are circulated per
given in Table 12-1. The values given are for minute at 10° F = 16.52 x 0.7402
compressors ranging in size from 5 to 25 hp. suction = 12.23 lb/min
Smaller compressors will have slightly lower From Table 16-3,
efficiencies, whereas larger compressors will enthalpy of R-12
have slightly higher efficiencies. saturated vapor at
12-17.Variation in Compressor Capacity 10° F = 79.36 Btu/lb
with Suction Temperature. Compressor per- Enthalpy of R-12
formance and cycle efficiency will vary consider- liquid at 100° F = 31.16 Btu/lb
ably with the operating conditions of the system. Refrigerating effect = 48.20 Btu/lb
The most important factor governing the
Theoretical refriger-
capacity of the compressor is the vaporizing
ating capacity of
temperature of the liquid in the evaporator, compressor at 10° F = 12.23 x 48.20
that is, the suction temperature. The large suction, Btu/min = 589.49 Btu/min
variations in compressor capacity which accom- Theoretical refrigerat- 589.49
pany changes in the operating suction tempera- ing capacity in tons 200
ture are primarily a result of a difference in the = 2.95 tons
density of the suction vapor entering the (b) From Table 16-3,
suction inlet of the compressor. The higher the density of R-12 satur-
vaporizing temperature of the liquid in the ated vapor at 40° F = 1.263 lb/cu ft
evaporator, the higher is the vaporizing pressure From Example 12-1,
and the greater is the density of the suction piston displace-
vapor. Because of the difference in the density ment = 16.52 cu ft/min
PERFORMANCE OF RECIPROCATING COMPRESSORS 211

Weight of refrigerant effect of each pound of refrigerant circulated.


circulated per minute = 16.52 x 1.263 Although the actual gain in refrigerating effect
at 40° F suction = 20.86 lb/min per pound only 6.95 %, when this increase is
is
From Table 16-3, applied to the entire weight of refrigerant
enthalpy of R-12 circulated at the higher suction temperature, the
saturated vapor at
net gain in capacity over the original capacity
40° F = 82.71 Btu/lb
which can be attributed to the greater refriger-
Enthalpy of R-12 ating effect is 11.8% (1.705 x 0.0695 = 1.823
liquid at 100° F = 31.16 Btu/lb
and 1.283 - 1.705 = 0.118 or 11.8%).
Refrigerating effect = 51.55 Btu/lb The actual variation in compressor capacity
Theoretical refriger- with changes in suction temperature is more
ating capacity of pronounced than that indicated by theoretical
compressor at 40° F = 20.86 x 51.55 computations. That is, the change in the actual
suction, Btu/min = 1075.33 Btu/min compressor capacity with variations in suction
Theoretical refriger- 1075.33 temperature is always greater than the change
ating capacity in 200 in the theoretical capacity. The reason for this
tons = 5.38 tons is that the compression ratio changes as the

In analyzing the results of Example 12-6 the suction temperature changes. When the vapor-
izing temperature increases while the condensing
following observations are of interest:
temperature remains constant, the compression
Although the piston displacement of the
1. ratio is decreased and the volumetric efficiency
compressor is the same in each case, the weight of the compressor is improved. Hence, at the
of refrigerant circulated per minute by the higher suction temperature, in addition to
compressor increases from 12.23 lb/min to pumping a greater weight of refrigerant per unit
20.86 lb/min when the operating suction of volume, the volume of vapor pumped by the
temperature is raised from 10° F to 40° F. The compressor is also larger because of the im-
increase in the weight of refrigerant circulated proved efficiency.
results entirely from the greater density of the
suction vapor entering the suction inlet of the Example 12-7. Assuming that the satur-
compressor. In this instance, the percentage ated discharge temperature is 100° F, determine
the actual refrigerating capacity of the com-
increase in the weight of refrigerant circulated is
pressor in Example 12-6 when operating at each
20.86 - 12.23 of the suction temperatures in question.
x 100 =70.5%
12.23
Solution
2. The theoretical refrigerating capacity of (a) From Table 16-3,

the compressor at the 10° F suction temperature absolute pressure corre-


is 2.95 tons, whereas at the 40° F suction sponding to 100° F
saturation temperature 131.6 psi
temperature, the capacity increases to 5.38 tons.
This represents an increase in refrigerating Absolute pressure corre-
sponding to 10° F
capacity of
saturation temperature = 29.35 psi
5.38 - 2.95 Compression ratio 131.6
2^5 x 100 = 82.3%
29.35
= 4.47
Although the increased density of the suction
From Table 12-1,
vapor at the higher suction temperature volumetric efficiency = 76.3%
accounts for the greater part of the increase in
From Example 12-6,
compressor capacity, it is not the only reason theoretical refrigerating
for it. As indicated, the increase in the weight capacity at 10° F
of refrigerant circulated is only 70.5%, whereas suction = 2.95 tons
the total increase in compressor capacity is Actual refrigerating
82.3%. The additional 1 1.8 %gain in capacity is capacity at 10° F = 2.95 x 0.763
brought about by an increase in the refrigerating suction = 2.22 tons
212 PRINCIPLES OF REFRIGERATION
(b) From Table 16-3, Theoretical weight of
absolute pressure corre- refrigerant circulated
sponding to 100° F satu- per minute by corn- = 16.52 x 0.7402
ration temperature = 131.6 psi compressor = 12.23 lb
Absolute pressure Refrigerating effect per
corresponding to 40° F pound at 10° F
saturation temperature = 51.68 psi vaporizing and 120° F
Compression ratio 131.6 condensing = 43.20 Btu/lb
31.68 Theoretical refriger-
= 2.55 ating capacity of = 12.23 x 43.20
From Table 12-1, compressor = 527.34 Btu/min
volumetric efficiency = 85.7% Theoretical refriger- 527.34
From Example 12-6, ating capacity in tons 200
theoretical refrigerating = 2.64 tons
capacity at 40° F From Table 16-3,
suction = 5.38 tons
absolute suction
Actual refrigerating pressure = 29.35 psi
capacity at 40° F = 5.38 x 0.857
Absolute discharge
suction = 4.61 tons
pressure = 171.8 psi

Whereas the theoretical increase in com- Compression ratio 171.8


pressor capacity is only 82.3%, the actual 29.35
increase in refrigerating capacity is = 5.85
From Table 12-1,
4.61 -2.22 volumetric efficiency = 66.5%
x 100 =
2.22 Actual refrigerating 2.645 x 0.665
capacity in tons = 1.76
= 107.7%
(ft) For the 40° F suction temperature.
of Condensing Temperature
12-18. Effect From Example 12-1,

on Compressor Capacity. In general, the piston displacement


refrigerating capacity of the compressor de- of compressor = 16.52 cu ft/min

creases as the condensing temperature increases From Table 16-3, den-

and increases as the condensing temperature sity of R-12 saturated

decreases. The effect that the condensing vapor at 40° F = 1.263 lb/cu ft

temperature has on compressor efficiency and Theoretical weight of


capacity can be evaluated by comparing the refrigerant circulated

results of the following example with those of per minute by = 16.52 x 1.263

Examples 12-6 and 12-7.


compressor = 20.86 lb

Refrigerating effect per


Example 12-8. Determine the theoretical pound at 40° F
and actual refrigerating capacities of the com- evaporating and
pressor in Example 12-1 for each of the two condensing 120° F = 46.55 Btu/lb
vaporizing temperatures given in Examples
Theoretical refriger-
12-6 and 12-7, if the condensing temperature in
ating capacity of = 20.86 x 46.55
each case is 120° F rather than 100° F.
compressor = 971 Btu/min
Solution Theoretical refriger- 971
(a)For the 10° F vaporizing temperature. ating capacity in tons ~200
From Example 12-1, = 4.85
piston displacement From Table 16-3,
of compressor = 16.52 cu ft/min absolute suction
From Table 16-3, den- pressure = 51.68 psi
sity of R-12 saturated Absolute discharge
vapor at 10° F = 0.7402 lb/cu ft pressure = 171.8 psi
PERFORMANCE OF RECIPROCATING COMPRESSORS 213

Compression ratio 171.8 temperature is somewhat greater than that in the


51.68 condensing temperature. Whereas the increase
3.32 in condensing temperature is only 20° F
From Table 12-1, (120° - 100°), the increase in the discharge
volumetric efficiency 78.5% temperature is F (137.5° - 114°). This is
23.5°
Actual refrigerating 4.85 x 0.785 accounted for by the greater work of com-
capacity in tons 3.81 pression at the higher compression ratio. Had
the condensing temperature been increased in
Examining first the 10° F cycle, notice that such a way that the compression ratio does not
raising the condensing temperature from 100° F change (by increasing the suction temperature
to 120° F reduces the theoretical refrigerating in proportion), the increase in the discharge
capacity of the compressor from 2.95 tons to temperature would have been approximately
2.64 tons and the actual capacity from 2.22 tons the same as that in the condensing tempera-
to 1.76 tons. ture.
Since a 100% efficient compressor is assumed High discharge temperatures are undesirable
to displace a theoreticalvolume of vapor equal and are to be avoided whenever possible. The
to its piston displacement and since the density higher the discharge temperature, the higher is

of the suction vapor entering the compressor the average temperature of the cylinder walls
for any one vaporizing temperature is always and the greater is the superheating of the
the same regardless of the condensing tempera- suction vapor in the compressor cylinder. In
ture, the theoretical weight of refrigerant addition to its adverse effect of compressor
displaced by the compressor is the same at all high discharge temperatures tend to
efficiency,
condensing temperatures, and therefore the increase the rate of acid formation in the
theoretical refrigerating capacity of the com- system, cause carbonization of the oil in the
pressor for any condensing temperature depends head of the compressor, and produce other
only upon the refrigerating effect per pound of effects detrimental to the equipment.
refrigerant circulated. Hence, the difference The loss of compressor efficiency and capacity
in the theoretical refrigerating capacity of the resulting from an increase in the condensing
compressor at the two condensing temperatures temperature of the cycle is more serious when
results entirely from the difference in the the suction temperature of the cycle is low than
pound.
refrigerating effect per when the suction temperature is high. The
The reduction in actual compressor capacity desirability of operating a refrigerating system
may be attributed to several factors: (1) a at the lowest practical condensing temperature
reduction in the refrigerating effect per pound has already been pointed out. This is of par-
and (2) a reduction in the volumetric efficiency ticular importance when the suction temperature
of the compressor. of the cycle is low and the compressor is already
Increasing the condensing temperature while operating at a relatively low efficiency.
the suction temperature remains constant When the cycle is operating at a 40° F
increases the compression ratio and reduces vaporizing temperature, increasing the con-
the volumetric efficiency of the compressor so densing temperature from 100° F to 120° F
that the actual volume of vapor displaced by the reduces the theoretical capacity of the com-
compressor per unit of time decreases. There- pressor from 5.38 tons to 4.85 tons and the
fore, even though the density of the vapor actual compressor capacity from 4.61 tons to
entering the compressor remains the same at 3.81 tons. The loss in theoretical capacity is
all condensing temperatures, the actual weight

of refrigerant circulated by the compressor per 5.38 - 4.85


x 100 = 10%
unit of time decreases because of the reduction 5.38
in the quantity of vapor handled.
Increasing the condensing temperature in- The loss in actual compressor capacity amounts
creases the isentropic discharge temperature. In to
this instance, it is interesting to note (Fig. 7-7) 4.61 - 3.81
x 100 = 17.4%
that the increase in the isentropic discharge 4 61
.
214 PRINCIPLES OF REFRIGERATION

For the 10° F cycle, the loss in theoretical efficient compressor operating on an ideal
compressor capacity is compression cycle and does not represent the
actual total horsepower which must be delivered
2.95 - 2.64
x 100 = 10.5% to the shaft of the compressor. In actual
2.95 practice, there are certain losses in power which
accrue because of the mechanical friction in the
and the loss in actual compressor capacity is
compressor and because of the deviation of an
2.55 - 1.76 actual compression cycle from the ideal com-
x 100 = 31%
2.55 pression cycle. Naturally, additional power
must be supplied to the compressor to offset
Note that the loss in theoretical capacity
these losses. Therefore, the actual power
brought about by increasing the condensing
required by a compressor will always be greater
temperature is approximately the same for
than the theoretical computations indicate.
both suction temperatures, whereas the loss in 12-20. Variation in Compressor Horse-
actual compressor capacity is much greater at
power with Suction Temperature. Although
the lower suction temperature. To a great
the horsepower per ton of refrigerating capacity
extent, it is the loss in volumetric efficiency that
diminishes as the suction temperature rises, the
causes the marked decrease in the actual
horsepower required by the compressor may
capacity of the compressor at the higher
either increase or decrease, depending upon
condensing temperature. The change in volu-
whether the work done by the compressor
metric efficiency for a given change in condens-
increases or decreases.
ing temperature becomes greater as the suction
The total amount of work done by the
temperature of the cycle decreases. This
compressor per unit of time in compressing the
accounts for the greater effect that a change in
vapor and, hence, the power required to drive
condensing temperature has on compressor
the compressor, is the function of only two
capacity when the suction temperature is low.
factors (1) the work of compression per pound
:

12-19. Compressor Horsepower. The theore- of vapor compressed and (2) the weight of vapor
tical horsepower required to drive the compres-
compressed per unit of time.
sor may be found by multiplying the actual
The amount of work which is done in
refrigerating capacity of the compressor in tons
compressing the vapor from the suction
by the theoretical horsepower required per ton
pressure to the discharge pressure varies with
for the operating conditions in question.
the compression ratio. The greater the com-
Example Find the theoretical horse-
12-9. pression ratio, the greater is the work of
power required to drive the compressor in compression. Therefore, when the suction
Example 12-4. temperature is raised while the condensing
temperature remains the same, the compression
Solution. From Example
ratio becomes smaller and the work of com-
12-4, actual refrigerating
capacity in tons = 2.76 tons pression per pound is reduced. However, at the
same time, because of the greater density of the
From Fig. 7-9, theoretical
suction vapor, the weight of vapor compressed
horsepower required per ton = 0.965 hp
by the compressor per unit of time increases.
The theoretical horsepower = 2.76 x 0.965
Since the saving in work done resulting from
required to drive the com-
the reduction in the work per pound is seldom
pressor = 2.66 hp
sufficient tooutweigh the increase in the work
Notice that it is the actual refrigerating of the compressor because of the increase in the
capacity of the compressor, rather than the weight of vapor compressed, raising the suction
theoretical refrigerating capacity, which must temperature will usually increase the power
be used in determining the theoretical horse- requirements of the compressor.
power requirements of the compressor. Example 12-10. Compute the theoretical
The horsepower as calculated in
theoretical horsepower required by the compressor in
the preceding example is only an indication of Example 12-7 at each of the suction tempera-
the power which would be required by a 100% tures listed.
PERFORMANCE OF RECIPROCATING COMPRESSORS 215

Solution comparison to the increase in compressor


(a) From Example 12-7, capacity. In this instance, for a 30° F rise in
actual refrigerating
suction temperature, the capacity of the
capacity in tons at 10° F compressor increased 107%, whereas com-
suction temperature = 2.22 tons
pressor horsepower increased only 21 %. The
From Fig. 7-9, theoreti-
average increase in compressor capacity per
cal horsepower per ton
at 10° suction and
degree of rise in suction temperature is 107%/
100° F condensing = 30° F or 3.21 %, whereas the increase in horse-
1.13 hp
Theoretical horsepower %
power amounts to only 0.7 per degree of rise.
of compressor at 10° F = 2.22 x 1.13 The relationship between compressor capacity
suction = 2.51 hp and the horsepower of the compressor at
(6) From Example 12-7, various suction temperatures is shown by the
actual refrigerating curves in Fig. 12-5. The curves are for a typical
capacity in tons at 40° F R-12 compressor operating at a constant con-
suction temperature = 4.61 tons
densing temperature of 100° F.
From Fig. 7-9, theoreti-
As shown by the curve in Fig. 12-5 the
cal horsepower per ton
horsepower required by a R-12 compressor
at 40° F suction and = 0.683 hp
100° F condensing increases as the suction temperature increases

Theoretical horsepower up to a certain point at which the horsepower


of compressor at 40° F = 4.61 x 0.683 required by the compressor is at a maximum.
suction = 3.15 hp On reaching this point, if the suction temperature
is further increased, the horsepower required
Although the horsepower per ton decreases
by the compressor diminishes. This is not true,
39.5 %
as the suction temperature is raised from
however, for compressors using ammonia as a
10° F to 40° F, because of the increase in the
refrigerant. For compressors using ammonia,
refrigerating capacity of the compressor, the
the horsepower does not reach a maximum
horsepower required by the compressor in-
value, but continues to increase indefinitely as
creases from 2.51 hp to 3.15 hp. This represents
the suction temperature increases.
an increase in the power required of
The suction temperature at which the horse-
— 1T-X100=21%
3.15 -2.51
power required by a R-12 compressors reaches
a maximum depends upon the condensing
The increase in compressor horsepower with temperature and increases as the condensing
the suction temperature is relatively small in temperature increases.
7 7

*&* —
Fig. 12-5. Curves illustrate the
-**"
&sf> f& &y
temperature
effects of suction
on the capacity and horse-
power of reciprocating com-
pressors.
A>?
^s

^2**
S^r,w fo»

-40' -30' -20° -10* 0' 10* 20° 30* 40° 50*
Suction temperature
1

216 PRINCIPLES OF REFRIGERATION

42 4.6

3.0
^^
**»
^ 4.5

-.2.8 4-4" 2.6 12-6. Curves illustrate


Fig.
c c
e s the effects of condensing tem-
£2.6
o
^s„s sJS>-
4.3 -S - 2.2
I perature on capacity and
a horsepower of reciprocating
5 2.4 4.2 i.8 i compressors.
vx^^^-J |
&**2- a.
V**™ E o
o
12 4.1 <->
1.4

m
?.o 4.0 1.0
80° 90' 100* 110- 120*
Condensing temperature

12-21. The Effect of Condensing Tempera- 12-22. Brake Horsepower. The total horse-
ture on Compressor Horsepower. The power which must be supplied to the shaft of the
curves in Fig. 12-6 illustrate the relationship compressor is called the brake horsepower and
between the horsepower required per ton of may be computed from the theoretical horse-
refrigerating capacity, the actual refrigerating power by application of a factor called over-all
capacity of the compressor, and the horsepower compressor efficiency. The over-all efficiency is
required by the compressor at various condensing an expression of the relationship of the theoreti-
temperatures when the suction temperature is cal horsepower to the brake horsepower in
keptconstant. Note that, although the theoretical percent. Written as an equation, the relationship
horsepower required per ton increases as the is

condensing temperature increases, the theoretical Thp


E
horsepower required by any one compressor will °=wP xl0° (12-6)

not increase in the same proportion. This is true


Thp
because the decrease in the refrigerating capa- and Bhp = (12-7)
city of the compressor which is coincident with
E l\00
o

an increase in the condensing temperature will where E = the over-all efficiency in percent
offset to some extent the increase in the horse- Thp = the theoretical horsepower
power per ton. Bhp = the brake horsepower
For instance, according to Fig. 7-9, for a cycle
operating at a 10° F vaporizing temperature, Example 12-11. Determine the brake
the theoretical horsepower required per ton horsepower required by the compressor in
increases from 1.1 3 to 1 .52 when the condensing Example 12-14, if the over-all efficiency of the
compressor is 80%.
temperature of the cycle is increased from 100° F
to 120° F. At the same time, Example 12-10 Solution. From Example 12-4,
illustrates that the actual refrigerating capacity Thp = 3.12 hp
of one particular compressor drops from 2.22 Applying Equation 12-7, the Thp
tons to 1.76 tons when the condensing tempera- Bhp E
ture is raised from 100° F to 120° F. The 3.12
theoretical horsepower required by the com- 086
pressor at the 100° F condensing temperature is = 3.9 hp
1.13 x 2.22 = 2.51 hp The over-all efficiency is sometimes broken
down into two components (1) the compression :

For the 120° F


condensing temperature, the efficiency and (2) die mechanical efficiency. In
theoretical horsepower required by the com- such cases, the relationship is
pressor is

1.52 x 1.76 =2.68hp (12-8)


PERFORMANCE OF RECIPROCATING COMPRESSORS 217

where Ee = the compression efficiency in per- A theoretical indicator diagram for an ideal
cent compression cycle is shown in Fig. 12-7. It has
Em = the mechanical efficiency in percent been illustrated previously that the area under
a process diagram on a pressure-volume chart
Thp is a measure of the work of the process. In
so that Bhp = (12-9)
E c X E„ Fig. 12-7, notice that the area dDCd represents
the work done by the piston in compressing the
The compression efficiency of a compressor is vapor during the isentropic process CD, and
a measure of the losses resulting from the devia- that the area aADba represents the work done
tion of the actual compression cycle from the by the piston in discharging the vapor from the
ideal compression cycle, whereas the mechanical cylinder during the constant pressure process
efficiency of the compressor a measure of the
is
DA, whereas the area aABa represents the work
losses resulting from the mechanical friction in done back on the piston by the vapor during
the compressor. The principal factors which the isentropic re-expansion (of the clearance
bring about the deviation of an actual com- vapor) process AB. Since the work of process
pression cycle from the ideal compression cycle AB is work given back to the piston by the
are: (1) wiredrawing, (2) the exchange of heat fluid, work input to the compression
the net
between the vapor and the cylinder walls, and cycle is the the work of processes CD
sum of
(3) fluid friction due to the turbulence of the and DA, less the work of process AB. Therefore,
vapor in the cylinder and to the fact that the the net work of the compression cycle is
refrigerant vapor is not an ideal gas. Notice represented by the area BADCB, the total area
that the factors which determine the compression enclosed by the cycle diagram.
efficiency of the compressor are the same as those The work of the compression cycle as deter-
which influence the volumetric efficiency. As a mined from the indicator diagram is called the
matter of fact, for any one compressor, the indicated work and the horsepower computed
volumetric and compression efficiencies are from the indicated work is called the indicated
roughly the same and they vary with the com- horsepower.
pression ratio in about the same proportions. Since the indicator diagram illustrated in
For this reason, the brake horsepower required Fig. 12-7 is a theoretical indicator diagram of
per ton of refrigerating capacity can be approxi- an compression cycle, the indicated work
ideal
mated with reasonable accuracy by dividing the is the work of an ideal compression cycle and
theoretical horsepower per ton by the volu- the indicated horsepower computed from the
metric efficiency of the compressor and then indicated work would, of course, be exactly
adding about 10% to offset the power loss due
to the mechanical friction in the compressor.
Written as an equation,

=
M(hd -h )e x 1.1
Bhp (12-10)
42.42 x E„

Since the relationship between the various


factors which influence the compression effi-

ciency are difficult to evaluate mathematically,


the compression efficiency of a compressor can
be determined accurately only by actual testing
of the compressor.
12-23. Indicated Horsepower. A device fre-
quently used to determine the compression
efficiency is the indicator diagram. An indicator
diagram is a pressure-volume diagram of the Volume
actual compression cycle of the compressor
which is produced during the actual testing of Fig. 12-7. Theoretical indicator diagram for an ideal
the compressor. compression cycle.
218 PRINCIPLES OF REFRIGERATION
The relationship of the indicated horsepower
to the theoretical horsepower is

l«J D Thp
_4l^ (12-12)

An indicator diagram of an actual com-


Q pression cycle is shown in Fig. 12-8. The area
ABCD, enclosed by the cycle diagram, is, of
course, a measure of the work of the cycle.
"'
An ideal cycle, AB'CD', is drawn in for com-
parison. Pressures P1 and P2 represent the
1
1Q 1

1
1 1
pressure of the vapor entering and leaving the
Vb Vc '

compressor. The areas above line P2 and


Volume below line P x represent the increased work of
the cycle due to wiredrawing. Notice that at
Fig. 12-8. Theoretical indicator diagram for an
actual compression cycle.
the end of the suction and discharge strokes
(points C and A), the piston velocity diminishes
to zero and the pressure of the vapor tends to
return to Px and P2 respectively. The other
,

equal to the theoretical horsepower. However, deviations from the ideal cycle represent the
in actual practice, since the indicator diagram losses resulting from the heating of the vapor
reproduces the true paths
of the various compressor cylinder. BC indicates
in the Line
processes which make up the actual compression the approximate volume of the suction vapor at
cycle, the indicated work of the diagram is an the end of the suction stroke, whereas line BC
accurate measure of the actual work of the represents the approximate volume of this same
compression cycle and, therefore, the ^indicated weight of vapor in the suction line. The
horsepower computed on the basis of the deviation of the actual compression process
indicated work is the actual horsepower from the isentropic can be seen by comparing
required to do the work of the actual com- the actual compression path CD to the isen-
pression cycle. tropic path CD'.
Care should be taken not to confuse indi- The direction of the periodic heat transfer
cated horsepower with brake horsepower. between the vapor and the cylinder walls at
Although the indicated horsepower includes various times and points in the cycle is indi-
the power required to offset the losses resulting
cated by the arrows. The arrows pointing in
from the deviation of an actual compression denote heat transfer from the cylinder walls
to
cycle from the ideal cycle, it does not include the vapor, whereas arrows pointing out indicate
the power required to overcome the losses
heat transfer from the vapor to the cylinder
resulting from the mechanical friction in the
walls.
compressor. In other words, the indicated The temperature of the cylinder walls of the
horsepower takes into account the compression compressor will fluctuate around some mean
efficiency but not the mechanical efficiency.
value which is between the suction and dis-
Hence, brake horsepower differs from indicated charge temperatures of the vapor. During the
horsepower in that the brake horsepower latter part of the re-expansion process, during
includes the power required to overcome the the period in which the vapor is being admitted
mechanical friction in the compressor, whereas to the cylinder, and during the initial part of the
the indicated horsepower does not. The compression stroke, the cylinder wall tempera-
horsepower necessary to overcome the mechani- ture is greater than the vapor temperature and
cal friction in the compressor is sometimes
heat passes from the cylinder walls to the vapor.
referred to as the friction horsepower (Fhp), so
During the latter part of the compression stroke,
that during the discharge period, and during the
Bhp = Ihp + Fhp (12-11) early part of the suction stroke, the temperature
PERFORMANCE OF RECIPROCATING COMPRESSORS 219

of the vapor exceeds the cylinder wall tempera- the ideal cycle is ordinarily not too much in
ture and heat passes from the vapor to the error for an air-cooled compressor.
cylinder walls. Water-jacketing of the compressor cylinder
12-24. Isothermal vs. Isentropic Compres- results in lowering the temperature of the
sion. Reference to Fig. 12-9 will show that if cylinder walls, and cooling of the vapor during
the compression process in the compressor was compression will be greater for the compressor
isothermal rather than isentropic the net work having a water jacket. Too, cylinder heating is
of the compression cycle would be reduced even reduced and the vapor is discharged from the
though the work of the compression process compressor at a lower temperature. All of this
itself is greater for isothermal compression than has the effect of reducing the work of the
for isentropic compression. The reduction in compression cycle. However, the gain is
the work of the cycle which would be realized usually not sufficient to warrant the use of a
through isothermal compression is indicated by water jacket on most compressors, particularly
the crosshatched area in Fig. 12-9. compressors designed for R-12. For the most
Isothermal compression is not practical for a part, water-jacketing of the compressor is
refrigeration compressor since it would result limited to compressors designed for use with
in the discharge of saturated liquid from the refrigerants which have unusually high dis-
compressor. Furthermore, if a cooling medium charge temperatures, such as ammonia. Even
were available at a temperature low enough to then, the purpose of the jacketing is not so
cool the compressor sufficiently to produce much for increasing compressor efficiency as
isothermal compression, the cooling medium it is to reduce the rate of oil carbonization and

could be used directly as the refrigerant and the formation of acids, both of which increase
there would be no need for the refrigeration rapidly as the discharge temperature increases.
cycle. 12-26. Wet Compression. Wet compression
12-25. Water-Jacketing the Compressor occurs when small particles of unvaporized
Cylinder. Any heat which isup by the
given liquid are entrained in the suction vapor
compressor cylinder to some external cooling entering the compressor. However, theoretical
medium represents, in effect, heat given up by computations indicate that wet compression
the vapor during the compression process. will bring about desirable gains in compression
Cooling of the vapor during compression causes efficiency and reduce the work of compression.
the path of the compression process to shift This would be true if the small particles of liquid
from the isentropic path toward an isothermal vaporized during the actual compression of the
path. Of course, the greater the amount of vapor. However, in actual practice, this is not
cooling, the greater will be the shift toward the the case. Since heat transfer is a function of
isothermal. time and since compression of the vapor in a
If the temperature of the air surrounding the modern high-speed compressor takes place very
compressor were exactly the same as the
temperature of the compressor cylinder, there
would be no transfer of heat from the cylinder
to the air and any heat given up by the vapor
to the cylinder would be eventually reabsorbed
by the vapor and the compression process
would be approximately adiabatic. However,
since there is nearly always some transfer of
heat from the compressor to the surrounding
air, compression is usually polytropic rather
than isentropic. For an air-cooled compressor,
the transfer of heat to the air will be slight and,
therefore, the value of the polytropic com-
pression exponent, n, will very nearly approach Volume
the isentropic compression exponent, k. Hence,
the assumption of isentropic compression for Fig. 12-9. Isentropic vs. isothermal compression.
220 PRINCIPLES OF REFRIGERATION

rapidly, there is not sufficient time for the liquid speed changes, and therefore the change in
to completely vaporize during the compression compressor capacity will not be proportional
stroke. Hence, some of the liquid particles to the speed change.
remain in the vapor in the clearance volume The variation in the volumetric efficiency
and vaporize during the early part of the with changes in the speed of rotation is brought

suction stroke. This action reduces the volu- about principally by changes in the -effects of
metric efficiency of the compressor without wiredrawing, cylinder heating, and the back
benefit of the return of work to the piston by leakage of gas through the suction and discharge
the expansion of the vaporizing particles. valves.
A result similar to this is encountered when The amount of back leakage through the
excessive cooling of the cylinder reduces the valves in percent per cubic foot of vapor dis-
temperature of the vapor in the clearance below placed is at a maximum at low compressor
the saturation temperature corresponding to speeds and decreases as the speed of the com-
the discharge. Some of the clearance vapor will pressor is increased. Cylinder heating, too, is

condense and the particles of liquid formed will greatest at low compressor speeds. On the
vaporize during the early part of the suction other hand, the effect of wiredrawing is at a
stroke. minimum at low speeds and increases as the
12-27. The Effect of Compressor Clearance speed increases because of the increase in the
on Horsepower. Theoretically, the clearance vapor passing through the valves.
velocity of the
of the compressor has no effect on the horse- Hence, as the speed of rotation increases, the
power, since the work done by the piston in volumetric efficiency of the compressor due to
compressing the clearance vapor is returned to the cylinder heating and valve leakage factors
the piston as the clearance vapor re-expands increases, while, at the same time, the volumetric
at the start of the suction stroke. However, since efficiency due to the wiredrawing factor de-
the refrigerant vapor is not an ideal gas, there creases. follows, then, that there is one
It

is some loss of power in overcoming the internal critical speed of rotation at which the combined

friction of the fluid so that the power returned effect of these factors are at a minimum and the
to the piston during the re-expansion of the volumetric efficiency is at a maximum. From
clearance vapor will always be less than an efficiency standpoint, this is the speed at
the power required to compress it. Hence, the which the compressor should be operated. At
clearance does have some, although probably speeds higher than this critical speed, the
slight, effecton the power requirements. volumetric efficiency of the compressor diminish
12-28. Compressor Speed. Since the speed of because the loss of efficiency due to the wire-
rotation is one of the factors determining piston* drawing effect will be greater than the gain
displacement (Equation 12-1), the capacity of resulting from the decrease in the effect of
the compressor changes considerably when the cylinder heating and valve leakage. Likewise, at
speed of the compressor is changed. If the speeds below the critical speed, the volumetric
speed of the compressor is increased, the piston efficiency will be lower because the losses
displacement is increased and the compressor accruing from the increase in cylinder heating
displaces a greater volume of vapor per unit of and valve leakage will be greater than the gain
time. Theoretically, based on the. assumption resulting from the decrease in the wiredrawing
that the volumetric efficiency of the compressor losses.
remains constant, the capacity of the compressor The critical speed will vary with the design of
varies in direct proportion to the speed change. the compressor and with the refrigerant used,
That is, if the speed of the compressor is and can best be determined by actual test of the
doubled, the piston displacement and capacity compressor.
of the compressor are also doubled. Likewise, It is general practice in the design of modern
if the speed of the compressor is reduced, the high-speed compressors to use large valve ports
piston displacement and capacity of the in order to reduce the wiredrawing effect to a
compressor are reduced in the same propor- practical minimum. These large openings in the
tion. However, the volumetric efficiency of the valve-plate tend to increase the clearance
compressor does not remain constant during volume and decrease the volumetric efficiency
PERFORMANCE OF RECIPROCATING COMPRESSORS 221

due to the clearance factor, but the advantages useful cooling, there is no offsetting gain in
accruing from the reduction in the wiredrawing capacity and the refrigerating capacity of the
effect more than offsets the loss of efficiency due compressor reduced in inverse proportion to
is

to the greater clearance. This is particularly the increase inthe specific volume of the
true where the power requirements are con- suction vapor at the compressor inlet.
cerned, since the loss of power due to wire- Regardless of whether or not the superheating
drawing is much greater than the loss of power produces useful cooling, the horsepower
due to the clearance factor. required to drive any one compressor is
12-29. Mechanical Efficiency. The mechanical practically the same for a superheated cycle as
friction in thecompressor varies with the speed for a saturated cycle. It was shown in Section
of rotation, but for any one speed, the mechan- 8-4 that, when superheating of the vapor
ical friction, and therefore the friction horse- produces useful cooling, both the horsepower
power, will remain practically the same at all required per ton and the refrigerating capacity
operating conditions. Since the friction horse- of the compressor are the same for the super-
power remains the same, it follows that the heated cycle as for the saturated cycle. It
mechanical efficiency of the compressor depends follows, then, that the horsepower required by
entirely upon the loading of the compressor. any one compressor be the same for both
will
As the total brake horsepower of the compressor cycles. On the other hand, when superheating
increases due to loading of the compressor, the of the vapor produces no useful cooling, the
friction horsepower, being constant, will become horsepower per ton is greater than for the
a smaller and smaller percentage of the total saturated cycle. However, at the same time,
horsepower .and the mechanical efficiency will the refrigerating capacity of the compressor is
increase. It is evident that the mechanical less for the superheated cycle and the increase
efficiency of the compressor will be greatest in the horsepower required per ton is more or
when the compressor is fully loaded. The less offset by the reduction in compressor
mechanical efficiency of the compressor will capacity, so that the horsepower required by the
vary with the design of the compressor and compressor is still approximately the same as
with compressor speed. An average compressor for the saturated cycle. Notice that, although
of good design operating fully loaded at a the horsepower required by any one compressor
standard speed should have a mechanical is not appreciably changed by the superheating

efficiency somewhat above 90%. of the suction vapor, when the superheating
12-30. The Effect of Suction Superheat on does not produce useful cooling, the refriger-
Compressor Performance. It has been shown ating capacity and efficiency of the compressor
that superheating of the suction vapor causes are materially reduced. This is particularly
the vapor to reach the compressor in an true when the compressor is operating at a low
expanded condition. Therefore, when the suction temperature. It should be noted also
vapor reaches the compressor in a superheated that superheating of the suction vapor reduces
condition, the weight of refrigerant circulated the amount of cylinder heating and the efficiency
by the compressor per minute is less than when of the compressor is increased to some extent.
the vapor reaches the compressor saturated. 12-31. The Effect of Subcooling on Com-
Whether or not the reduction in the weight of pressor Performance. When subcooling of
refrigerant circulated by the compressor reduces the liquid refrigerant is accomplished in such a
the refrigerating capacity of the compressor way that the heat given up by the liquid leaves
depends upon whether or not the superheating the system, the specific volume of the suction
produces useful cooling. When the super- vapor at the compressor inlet is unaffected by
heating produces useful cooling, the gain in the subcooling and the weight of refrigerant
refrigerating capacity resulting from the increase circulated per minute by the compressor is the
in the refrigerating effect per pound is usually same as when no subcooling takes place. Since
sufficient to offset the loss in refrigerating the refrigerating effect per pound is increased by
capacity resulting from the reduction in the the subcooling, the capacity of the compressor is
weight of refrigerant circulated. On the other increased by an amount equal to the amount of
hand, when the superheating produces no subcooling. Notice that the increase in the
222 PRINCIPLES OF REFRIGERATION
refrigerating capacity of the compressor result- Table R-10C. Since the compressor ratings
ing from the subcooling is accomplished without given in Table R-10A are in accordance with
increasing the power requirements of the com- ASHRAE standards, it follows that in order to
pressor. Therefore, subcooling improves com- realize the listed ratings, the suction vapor must
pressor efficiency, provided the heat given up enter the suction inlet of the compressor at the
during the subcooling leaves the system. conditions shown in Table R-10C. For example,
When the heat given up during the subcooling for a compressor operating at a saturated
does not leave the system, as when a heat suction of —40° F, the suction vapor should
exchanger is used, the gain in capacity due to enter the compressor at a temperature of 35° F
the subcooling is approximately equal to the (superheated 75° F from -40° F to 35° F), if
loss in capacity due to the superheating, and the listed rating is to be obtained. Likewise, for
the refrigerating capacity of the compressor is a compressor operating at a saturated suction
very little affected. However, there is some of 40° F, the actual temperature of the suction
small gain in compressor efficiency, since the vapor entering the compressor should be 65° F
superheating of the vapor in the heat exchanger (superheated 25° F from 40° F to 65" F).
reduces the effect of cylinder heating. Where the actual temperature of the suction
12-32. Compressor Rating and Selection. vapor is less than that indicated in Table
As previously stated, mathematical evaluation R-10C, the tabulated rating is corrected by
of all the factors which influence compressor using an appropriate multiplier to obtain the
performance is not practical. Hence, com- actual compressor capacity. The multipliers
pressor capacity and horsepower requirements given in Table R-10D correct the ratings to a
are determined accurately only by actual basis of no superheat for the saturated condition
testing of the compressor. Table R-10A is a listed. Where the actual suction vapor tem-
typical compressor rating table supplied by the perature is intermediate between saturation and
compressor manufacturer for use in compressor the temperature shown in Table R-10C, the
selection. The ratings have been determined by multiplier is corrected accordingly.
actual testing of the compressor under operating The superheating is assumed to occur in the
conditions set forth in the compressor testing evaporator, in the suction line inside the re-
and rating standards of the American Society frigerated space, or in a liquid-suction heat
of Heating, Refrigerating, and Air Conditioning exchanger so that the superheat produces useful
Engineers (see Table R-10B). cooling (Section 8-4). Superheating which
It has been shown in the foregoing sections occurs outside the refrigerated space should
that both the refrigerating capacity and the be disregarded with respect to the tabulated
horsepower requirements of a compressor vary ratings.
with the condition of the refrigerant vapor The superheating requirement of the
entering and leaving the compressor. Notice ASHRAE standards at first appears to compli-
in Table R-10A that compressor refrigerating cate unnecessarily the compressor rating and
capacities (Btu/hr) and horsepower require- selection procedure. However, this is not the
ments are listed for various saturated suction case. The superheating requirement is very
and discharge temperatures. The saturated realistic in that the amount of superheating
suction temperature is the saturation tempera- specified in the rating standards very nearly
ture corresponding to the pressure of the vapor approaches that amount which would normally
at the suction inlet of the compressor, and the be expected in a well-designed application.
saturated discharge temperature is the saturation Hence, the effect of the superheating require-
temperature corresponding to the pressure of ment is to cause the compressor to be rated
the vapor at the discharge of the compressor. under conditions similar to those under which
Although compressor ratings are based on the compressor will be operating in the field.

the saturated suction and discharge tempera- For unusual cases, no


this reason, except in
tures, ASHRAE test standards require a certain appreciable error will occur if the compressor
amount of suction superheat and specify that ratings given in Table R-10A are used without
the actual temperature of the suction vapor correction of any kind. Furthermore, com-
entering the compressor be those listed in pressor capacity requirements are not usually
PERFORMANCE OF RECIPROCATING COMPRESSORS 223

critical within certain limits. There are several loss of approximately 3 psi. From Table 16-3,
reasons for this. First of all, the methods of the saturation pressure of Refrigerant- 12 corre-
determining the required compressor capacity sponding to a temperature of 28° F is 41 .59 psia.
(cooling load calculations) are not in themselves Allowing for the 3 psi pressure loss in the
exact. Too, it is seldom possible to select a suction line, the pressure of the vapor at the
compressor which has exactly the required suction inlet of the compressor is 38.39 psia
capacity at the design conditions. Another (41.59 - 3). From Table 16-3, the saturation
reason that compressor capacity is not critical temperature corresponding to a pressure of
within reasonable limits is that the operating 38.59 psia, and, therefore, the saturated suction
conditions of the system do not remain constant temperature, is approximately 24° F.
at all times, but vary from time to time with the The design saturated discharge temperature
loading of the system, the temperature of the depends primarily on the size of the condenser
condensing medium, etc. General practice is to selected and upon the quantity and temperature
select a compressor having a capacity equal to of the available condensing medium. Methods of
or somewhat in excess of the required capacity condenser selection are discussed in Chapter 14.
at the design operating conditions. 12-33. Condensing Unit Rating and Selec-
It was shown in Chapter 8 that subcooling of tion. Since condensing unit capacity depends
the liquid increases the refrigerating effect per upon the capacity of the compressor, methods
pound and thereby increases compressor of rating and selecting condensing units are
capacity. With regard to subcooling, the practically the same as those for rating and
ratings given in Table R-10A are based on selecting compressors. The only difference is
saturated liquid approaching the refrigerant that, whereas compressor capacities are based
control, that is, no subcooling. Where the on the saturated suction and discharge tempera-
liquid is subcooled by external means (Section tures, condensing unit capacities are based on
8-7), the capacity of the compressor may be the saturated suction temperature and on the
increased approximately 2% for each 5°F of quantity and temperature of the condensing
subcooling. Here again, for reasons outlined in medium. Since the size of the condenser is

the preceding paragraph, the effect of subcooling fixed at the time of manufacture for any given
is usually neglected in selecting the compressor. condenser loading, the only variables deter-
To select a compressor for a given application, mining the saturation temperature at the
the following data are needed: discharge of the compressor (and therefore the
capacity of the compressor at any given suction
1. The required refrigerating capacity
temperature) is the quantity and temperature of
(Btu/hr)
the condensing medium. For air-cooled con-
2. The design saturated suction temperature
densing units, when the quantity of the air
3. The design saturated discharge temperature
passing over the condenser is fixed by the fan
Naturally, the required refrigerating capacity selection at the time of manufacture, the only
is the average hourly load as determined by the variable determining the capacity of the con-
cooling load calculations. However, if an densing unit, other than the suction temperature,
evaporator selection is made prior to the is the ambient air temperature (temperature of
compressor compressor should be
selection, the the air entering the condenser). Hence, ratings
selected to match the evaporator capacity rather for air-cooled condensing units are based on
than the calculated load. The reasons for this the saturated suction temperature and the
are discussed in Chapter 13. ambient air temperature.
The design saturated suction temperature Ratings for water-cooled condensing units
depends upon the design conditions of the are based on the saturated suction temperature
application. Specifically, it depends upon the and on the entering and leaving water tempera-
evaporator temperature (the saturation tem- tures.* Typical capacity ratings for air-cooled
perature of the refrigerant at the evaporator *
For any given condenser loading and entering
outlet)and upon the pressure loss in the suction water temperature, the leaving water temperature
line. For instance, assume an evaporator depends only on the quantity of water (gallons per
temperature of 28° F and a suction line pressure minute) flowing through the condenser.
224 PRINCIPLES OF REFRIGERATION
and water-cooled condensing units are given in Approximate capacity
Tables R-ll and R-12, respectively. of compressor at a 23° F
saturated suction = 34,000 + 2780
Example 12-12. A certain refrigeration = 37,380 Btu/hr
application has a calculated cooling load of
33,000 Btu/hr. If the design saturated suction Example 12-15. A certain refrigeration
and discharge temperatures are 20° F and application has a calculated cooling load of
100° F, respectively, select a compressor from 8750 Btu/hr and the ambient temperature is
Table R-10A which will meet the requirements 90° F. If the design saturated suction tempera-
of the application. ture is 20° F, select an air-cooled condensing
unit which will satisfy the requirements of the
Solution. Locate the desired saturated dis- application.
charge temperature in the first column of the
table (100° F). Next, in the second column, Solution. From Table R-ll, select a 1 hp
locate the desired saturated suction tempera- condensing unit having a capacity of 9340
ture and read to the right until a compressor Btu/hr at the prescribed conditions.
having a capacity equal to or somewhat in
excess of the desired capacity is found. Select
compressor, Model #5F20, which has a capacity
PROBLEMS
of 34,000 Btu/hr at 1450 rpm. 1. A four-cylinder reciprocating compressor
having a 2 in.bore and a 2.5 in. stroke is
Example 12-13. Assume that the design
rotating 100 rpm. Compute the piston
at
saturated suction in Example 12-12 is 0° F and
displacement in cubic feet per minute.
make a new compressor selection.
Arts. 18.18 cfm
Solution. Using the procedure outlined in the 2. Assume the compressor in Problem 12-1 is
solution to Example 12-12, select compressor
operating on the cycle described in Problem 7-1
Model #5F30 which has a capacity of 36,000 and compute the theoretical refrigerating
Btu/hr at 1750 rpm.
capacity of the compressor in Btu/hr.
Ans. 2.47 tons
Example 12-14. Determine the capacity of
compressor Model #5F20 when operating at a 3. Using the conditions of Problem 12-2,
23° Fsaturated suction temperature, if the determine:
saturated discharge temperature is 100° F. (a) The volumetric efficiency of the com-
pressor.
Solution. From Table R-10A,
(b) The actual refrigerating capacity of the
Compressor capacity
compressor in tons. Ans. 1.52 tons
at 30° F suction = 44,200 Btu/hr
(c) The brake horsepower required per ton
Compressor capacity (allow 10% for mechanical friction and
at 20° F suction = 34,600 Btu/hr assume the compression efficiency is the
Capacity change per same as the volumetric efficiency).
10° F change in saturated Ans. 2.7 hp/ton
suction temperature = 44,200 - 34,600 (d) The brake horsepower required to
total
= 9600 Btu/hr drive the compressor. Ans. 4.1 hp
Average capacity change
per ° F change in suction
4. From the compressor rating tables, select a
9600 compressor which will satisfy the following
temperature
10 conditions:
= 960 Btu/hr (a) Required capacity 24,900 Btu/hr
Total capacity change (b) Design saturated suction temperature
for 3° F change in suction 10° F
temperature = 960 x 3 (c) Design saturated discharge temperature
= 2780 Btu/hr 105° F
design conditions and the system will not
perform satisfactorily.
In any event, it is important to understand
that, regardless of the equipment selected, the
system will always establish equilibrium at some
conditions such that all the system components
will have equal capacity. Hence, whether or not

13 system equilibrium is established at the sys-


tem design conditions depends entirely upon
whether or not the equipment is selected to have
approximately equal capacities at the system
System Equilibrium design conditions. This concept is best illu-

strated through the use of a series of examples.


and Cycling Example 13-1. A
walk-in cooler, having a
calculated cooling load of 11,000 Btu/hr, is to
Controls be maintained at 35° F. The desired evaporator
TD is 12° F and the ambient temperature is
90° F. Allowing 3° F (equivalent to approxi-
mately 2 lb) for the pressure drop in the suction
line (see Section 12-32, select an air-cooled
condensing unit and a unit cooler from manu-
13-1. System Balance. In the designing of a facturer's catalog data.

refrigerating system, one of the most important Solution. Since the design space temperature
considerations is that of establishing the proper is35° F and the design evaporator is 12° F, TD
relationship or "balance" between the vapor- the design evaporator temperature is 23° F
izing and condensing sections of the system. (35° F -
12° F). When we allow for a 3° F
It isimportant to recognize that whenever an from pressure
loss in the suction line resulting

evaporator and a condensing unit are connected drop, the saturation temperature at the com-
together in a common system, a condition pressor suction is 20° F (23° F - 3° F).

of equilibrium or "balance" is automatically From Table


R-ll, select lj hp condensing

established between the two such that the rate


unit,which has a capacity of 12,630 Btu/hr at a
20° F saturated suction and 90° F ambient air
of vaporization is always equal to the rate of

That is, the rate at which the temperature. Although the condensing unit
condensation.
vapor is removed from the evaporator and
capacity is somewhat in excess of the calculated
load of 11,000 Btu/hr, it is sufficiently close to
condensed by the condensing unit is always
equal to the rate at which the vapor is produced
make the condensing unit acceptable. How-
in the evaporator by the boiling action of the
system balance, the unit
ever, to assure proper
cooler selection must now be based on the
liquid refrigerant. Since all the components in
condensing unit capacity of 12,630 Btu/hr
a refrigerating system are connected together in
series, the refrigerant flow rate through all
rather than on the calculated load of 11,000
Btu/hr. * Hence, a unit cooler having a capacity
components is the same. It follows, therefore,

that the capacity of all the components must be


of approximately 12,630 Btu/hr at a 12° F TD
is required.
of necessity the same.
Obviously, then, where the system com- From Table R-8, unit cooler Model #105 has
ponents are selected to have equal capacities at a capacity of 10,500 Btu/hr at a 10° F TD.
the system design conditions, the point of
Using the procedure outlined in Section 11-23,
it can be determined that this unit cooler will
system equilibrium or balance will occur at the
system design conditions. On the other hand,
have a capacity of 1 2,600 Btu/hr when operating
when the components selected do not have * Either the evaporator or the condensing unit
equal capacities at the system design conditions, may be selected first. However, once either one has
system equilibrium be established at
will been selected, the other must be selected for approxi-
operating conditions other than the system mately the same capacity.
225
226 PRINCIPLES OF REFRIGERATION
instance, the evaporator TD will be 12° F when,
and only when, the suction temperature is 20° F.
Any suction temperature other than 20° F will
result in an evaporator TD either greater or
Nf smaller than 12° F. For example, assume a
^& suction temperature of 25° F. Adding 3° F to
(Compressor; - allow for the suction line loss, the evaporator
temperature is found to be 28° F (25° F + 3° F).
\r
\\ Then subtracting the evaporator temperature
\*w from the space temperature, it is determined
fc
V\>1
that the evaporator TD be 7° F (35° F -
will
28° F) when the suction temperature is 25° F.
0' 5° 10° 15° 20° 25° Using the same procedure, it can be shown that
Suction temperature if the suction temperature is reduced to 15° F,
J I
I I
I L the evaporator TD will increase to 17° F, and
32° 27° 22° XT 12° V when the suction temperature is 10° F, the
Evaporator TD evaporator TD
will be 22° F, and so on.
Fig. 13-1. Graphic analysis of system balance. Apparently, then, raising or lowering the
suction temperature always brings about a
corresponding adjustment in the evaporator TD.
at a 12° F TD. Since this is very close to the Provided that the space temperature is kept
condensing unit capacity, the unit cooler is constant, raising the suction temperature
ideally suited for the application. reduces the evaporator TD, whereas lowering
13-2. Graphical Analysis of System Equi- the suction temperature increases the evaporator
librium. For any particular evaporator and TD.
condensing unit connected together in a With regard to Fig. 13-1, the following
common system, the relationship established procedure is used in making a graphical analysis
between the two, that is, the point of system of the system equilibrium conditions:
balance, can be evaluated graphically by
plotting evaporator capacity against condensing 1. On graph paper, lay out suitable scales
unit capacity on a common graph. Using data for capacity (Btu/hr), suction temperature (° F),

taken from the manufacturers' rating tables, and evaporator TD (° F). The horizontal lines
condensing unit capacity is plotted against are used to represent capacity, whereas the
suction temperature, whereas evaporator capa- vertical lines are given dual values, representing
city is plotted against evaporator TD. A both suction temperature and evaporator TD.
graphical analysis of the system described in The latter is meaningful, however, only when
Example 13-1 is shown in Fig. 13-1. the suction temperature and evaporator TD
In order to understand the graphical analysis scales are so correlated that the two conditions
of system equilibrium in Fig. 1 3-1 , it is important which identify any one vertical line are condi-
to recognize that, for any given space tempera- tions which, at the design space temperature,
a fixed relationship between the
ture, there is actually represent conditions that will occur
evaporator TD and the compressor suction simultaneously in the system. The procedure
temperature. That is, for any given space for correlating the suction temperature and
temperature, once the evaporator TD is evaporator TD scales was discussed in the
selected, there only one possible suction
is preceding paragraphs.
temperature which will satisfy the design 2. Using manufacturer's catalog data, plot
conditions of the system. the capacity curve for the condensing unit.
Notice that, in Example 13-1, for the design Since condensing unit capacity is not exactly
space temperature of 35° F and assuming a proportional to suction temperature, the con-
3° F loss in the suction line, the only possible densing unit capacity curve will ordinarily have
suction temperature that can coexist with the a slight curvature. Hence, for accuracy, a
design evaporator TD
of 12° F is 20° F. In this capacity point is plotted for each of the suction
SYSTEM EQUILIBRIUM AND CYCLING CONTROLS 227

temperatures listed in the table and these points occur at conditions other than the design
are connected with the "best-fitting" curve. conditions. For instance, assume that unit
3. From theevaporator manufacturer's cata- cooler Model #UC-120, rather than Model
log data, plot the evaporator capacity curve. #UC-105, is selected in the foregoing example.
Since evaporator capacity is assumed to be At the design evaporator TD of 12° F, this unit
proportional to the evaporator TD, the evapo- cooler has capacity of 14,000 Btu/hr, whereas
rator capacity curve isa straight line, the position the condensing unit selected has a capacity of
and direction of which is adequately established only 12,630 Btu/hr at the design suction
by plotting the evaporator capacity at any two temperature of 20° F. Consequently, at the
selected TDs. The evaporator capacity at any design conditions, evaporator capacity will be
other TD will fall somewhere along a straight greater than condensing unit capacity, that is,
line drawn through these two points. In Fig. vapor be produced in the evaporator at a
will
13-1, evaporator TDs of 7°F and 12° F are greater rate than it is removed from the evapo-
used in plotting the two capacity points required rator and condensed by the condensing unit.
to establish the evaporator capacity curve. Therefore, the system will not be in equilibrium
at these conditions. Rather, the excess vapor
Notice in Fig. 13-1, that as the suction willaccumulate in the evaporator and cause an
temperature increases, the evaporator TD increase in the evaporator temperature and
decreases. This means, in effect, that as the pressure. Since raising the evaporator tempera-
suction temperature increases the capacity of ture increases the suction temperature and, at
the evaporator decreases while the capacity of the same time, reduces the evaporator TD, the
the condensing unit (compressor) increases. condensing unit capacity will increase and the
Likewise, as the suction temperature decreases, evaporator capacity will decrease. System
the capacity of the evaporator increases while equilibrium will be established when the
the capacity of the condensing unit decreases. evaporator temperature rises to some point
The intersection of the two capacity curves where the suction temperature and evaporator
indicates the point of system equilibrium. In TD are such that the condensing unit capacity
this instance, because the evaporator and and the evaporator are equal. In this instance,
condensing unit have been selected to have system equilibrium, as determined graphically
equal capacities at the system design conditions, (point A in Fig. 13-2), is established at a
the point of system equilibrium occurs at the suction temperature of approximately 23° F.
system design conditions (12° F and 20° FTD The evaporator TD
is approximately 9° F,
suction temperature). Although the total which is 3° F less than the design evaporator
system capacity is somewhat greater than the TD of 12° F and which will result in a space
calculated load, the difference is not sufficiently humidity somewhat higher than the design
great to be of any particular consequence, and condition. The total system capacity is approxi-
means only that the system will operate fewer mately 13,500 Btu/hr, which is about 23%
hours out of each 24 than was originally greater than the calculated hourly load of
anticipated.* The relationship between system 11,000 Btu/hr. This means that the system
capacity and the calculated load is discussed running time will be considerably shorter than
more fully later in the chapter. originally calculated. For instance, if the
Ithas already been pointed out that where the original load calculation based on a 16-hr
is
evaporator and condensing unit selected do not running time, the system will operate only
have equal capacities at the system design about 13 hr out of each 24.
conditions, the point of system equilibrium will The question immediately arises as to whether
or not this system will perform satisfactorily.
* For simplicity, the heat given off by the evapo-
Although this would depend somewhat on the
rator fan motor has been neglected. If this heat is
particular application, the answer is that it
added to the cooling load, the total system capacity
would be almost exactly equal to the calculated load. probably would not in the majority of cases.
It is not often that equipment can be found which so There are several reasons for this. First, the
nearly meets the requirements of an application as in evaporator TD of 9° F is considerably less than
this instance. the design TD of 12° F and would probably
228 PRINCIPLES OF REFRIGERATION
28 logical corrective measures prescribe either an
26 increase in the condensing unit capacity or a
24
22
\ X <"' reduction in the evaporator capacity in order to
re-establish the point of system equilibrium at
| ^\B
§20 '
conditions nearer to the design conditions.
1
1 \ Which of these two measures will produce the
| 18 \
8 most depends upon the rela-
satisfactory results
16
I"l4
II z —™ — \ ^p tionship between the over-all system capacity
and the calculated load. Whereas increasing
&12 I —
either the condensing unit capacity or the evapo-
10

8 ^r\
l\
1

i
\
\\
1

1 \
rator capacity will always bring about an in-
crease in the over-all system capacity, reducing
0* 5* 10* 15* 20* 25' either the condensing unit capacity or the
Suction temperature evaporator capacity will always bring about a
1 1 1 1 1 1
reduction in the over-all system capacity.
32* 27* 22* 17* 12* 7*
Referring to Fig. 13-2 for the system under con-
Evaporator TD sideration, if the condensing unit capacity is
Fig. 13-Z increased to the evaporator capacity at the
design conditions, system equilibrium will shift
result in a space humidity too high for the appli- from point A to point B. On the other hand, if
cation. Ordinarily, the humidity in the refrig- the evaporator capacity is reduced to the con-
erated space must be maintained within certain densing unit capacity at the design conditions,
fixed limits. Assuming that the design TD is the point of system equilibrium will shift from
selected to produce the median condition within A to C. Notice that, although the system is

these limits, a one degree deviation from the balanced at the design conditions at either
design TD in either direction is usually the points B or C, the over-all system capacity at
maximum which can be allowed if the space point B is considerably above the calculated
humidity is to be maintained within the limits load, whereas at point C the over-all system
specified for the application. capacity very nearly approaches the calculated
Another consideration is the fact that the load. Hence, in this instance, it is evident that
system capacity is some 23% greater than the increasing the condensing unit capacity as a
calculated load so that the system operating means of bringing the system into balance at the
time will be relatively short. Although the sys- design conditions cannot be recommended,
tem capacity exceeds the calculated load by a since it would also increase the over-all system
larger margin than good practice prescribes, this capacity and therefore tend to aggravate the
in itself would not ordinarily cause any serious already existing problem of excessive system
problem in a majority of applications. How- capacity with relation to the calculated load.
ever, since the shorter running time will also On the other hand, in addition to bringing the
tend to aggravate the already existing problem system into balance at the design conditions,
of high humidity, especially in the wintertime, reducing the evaporator capacity will also have
when the two conditions are taken together, it the beneficial effect of reducing the over-all
seems unlikely that the system would produce system capacity and thereby bringing it more
satisfactory results in any application where the into line with the calculated load.
space humidity is an important factor. 13-3. Decreasing or Increasing Evaporator
In the event that the equipment in question Capacity. Reducing the evaporator capacity
represents the best available selection, the ques- can be accomplished in several ways. One is to
tion arises as towhat can be done to bring the "starve" the evaporator, that is, to reduce the
system into balance at conditions more in amount of liquid refrigerant in the evaporator
keeping with the design conditions. In this by adjusting the refrigerant flow control so that
instance, since the problem is one of excessive the evaporator only partially flooded with
is

evaporator capacity with relation to the con- liquid. This effectively reduces the size of the
densing unit capacity at the design conditions, evaporator, since that part of the evaporator
SYSTEM EQUILIBRIUM AND CYCLING CONTROLS 229

which is not filled with liquid becomes, in effect, Note. Where the compressor driver is a
a part of the suction line. four-pole, alternating-current motor operating
Another method of reducing the capacity of on 60 cycle power, the approximate driver speed
the evaporator is to reduce the air velocity over is 1750 rpm. For a two-pole, alternating-current

the evaporator by slowing the evaporator fan or motor, the approximate speed is 3500 rpm.
blower. However, method has its limita-
this
tions in that the air velocity must be maintained
Example 13-2. A refrigeration compressor
having a 10 flywheel is driven by a four-pole,
in.
at a level sufficient to assure adequate air
alternating-current motor. If the diameter of
circulation in the refrigerated space. Too, re- the motor pulley is 4 in., determine the speed
ducing the change in the
air quantity causes a of the compressor.
sensible heat ratio of the evaporator. Depending
Solution. Rearranging and
upon the particular application, this may or may
applying Equation 13-1,
not be desirable.
Rpmi =R x D
As a general rule, there is little, if anything, ^D,
that can be done to increase the capacity of an _ 1750 x 4
undersized evaporator. Occasionally, the evapo- 10
rator capacity can be increased by increasing = 700
the air quantity. However, since increasing the Example 13-3. Determine the diameter of
air quantity also increases air velocity and fan the motor pulley required to reduce the speed
horsepower requirements, this method has its of the compressor in Example 13-2 from 700 to
limitations for reasons already discussed in 600 rpm.
Section 11-22.
Solution. Rearranging and
In some cases, the evaporator surface area applying Equation 13-1, Ds _ Rpm J
x D t

can be increased somewhat by using a length of Rpm^


eitherjjare tubing or finned tubing as a "drier 600 x 10
loop" or as additional evaporator surface. How- 1750
ever, this too has its limitations because of the
= 3.5 in.

pressure drop accruing in the tubing. Another method of reducing condensing unit
13-4. Decreasing or Increasing Condensing capacity is to reduce the volumetric efficiency of
Unit Capacity. Decreasing the condensing the compressor by increasing the clearance
unit capacity can be accomplished in several volume. This increase is accomplished by in-

ways, all of which involve decreasing the com- stalling a thicker gasket between the cylinder
pressor displacement. Probably the simplest housing and the valve-plate.
and most common method of reducing the con- In some cases, small increases in condensing
densing unit capacity is to reduce the speed of unit capacity can be obtained by merely in-
the compressor by reducing the size of the pulley creasing the speed of the compressor. However,
on the compressor driver. The speed reduction when the capacity increase needed is substan-
required is approximately proportional to the tial, it is usually more practical and more
desired capacity reduction. economical to use a larger size condensing unit
The relationship between the speed of the and reduce the capacity as necessary. The
compressor and the speed of the compressor reasons for this are several.
driver is expressed in the following equation: compressor capa-
First, since the increase in
be accompanied by an increase in the
city will
Rpm1 x D = Rpmt
x x D2 (13-1)
horsepower requirements, any substantial in-
where Rpmx = the speed of the compressor crease in the compressor capacity will tend to
(rpm) overload the compressor driver and necessitate
D = 1 the diameter of the compressor the use of a larger size. Too, some thought must
flywheel (inches) be given to the condenser capacity. Here again,
Rpmt = the speed of the compressor any increase in compressor capacity will tend to
driver (rpm) place a heavier load on
the condenser. If the
Z)2 = the diameter of the driver pulley size of the condenser
not increased in propor-
is

(inches) tion to the increase in the condenser load,


230 PRINCIPLES OF REFRIGERATION
On the other hand, if the flow rate of the water
entering the tank varies from time to time, it is

1 evident that if the level of the water


maintained within fixed limits, the pumping
is to be

system must be selected to have a capacity equal


to or somewhat in excess of the highest sustained
flow rate of the water entering the tank. It is

evident also that some means of cycling the


pump "off" and "on" must be provided. Other-
wise, during periods when the flow rate of the
water entering the tank is less than maximum,
the pumping rate will be excessive and the level
Fig. 13-3 of the water in the tank will be reduced below
the desired levei. One convenient and practical
means of cycling the pump is to install a float
excessive compressor discharge temperature and control in the tank (Fig. 1 The float control
3-4).

pressure will result.Not only will this materially is arranged to close the electrical contacts and
reduce the of the equipment and increase
life start the pump when the water in the tank rises
maintenance and operating costs, but it will also to a predetermined maximum level. When the
tend to nullify to some extent the gain in capa- water level in the tank falls to a predetermined
city originally accruing from the increase in lower limit, the float control acts to close the
compressor speed. electrical contacts and stop the pump. In this

It is apparent from the foregoing that, in most way, intermittent operation of the pump will
cases, increasing the capacity of either the eva- maintain a relatively constant water level in
porator or the condensing unit is something the tank.
which not easily accomplished. Therefore, it
is The latter principle is readily applied to the

is usually more practical and more economical refrigerating system. Since the cooling load on a
to select oversized equipment rather than under- refrigerating system varies from time to time,
sized equipment. When the evaporator or the the system is usually designed to have a capacity
condensing unit is and capacity reduc-
oversized equal to or somewhat in excess of the average
tion is required to bring the system components maximum cooling load. This is done so that the
into balance at the desired conditions, the capa- temperature of the space or product can be
city reduction can readily be made with little, if maintained at the desired low level even under
any, loss in system efficiency. peak load conditions. As in the case of the water
13-5. System Capacity vs. Calculated Load.
The relationship between system capacityand
system load one which warrants careful con-
is

sideration and which can be best explained by


To pump motor
comparing the refrigerating system to a water
pumping system. For example, assume that it
is desired to maintain a constant water level in

the tank shown in Fig. 13-3. If the water flows


into the tank at a fixed and constant rate which
is readily computable, the water in the tank can
be maintained at a fixed level simply by installing
a pumping system which has a capacity exactly
equal to the flow rate of the water into the tank.
Since the flow rate of the water entering the tank
is constant and since the pumping rate is equal
to the water flow rate, the pump will operate
continuously and no other water level control of
any kind will be needed. Fig. 13-4
SYSTEM EQUILIBRIUM AND CYCLING CONTROLS 231

pumping system, since the system refrigerating


capacity will always exceed the actual cooling
load, some means of cycling the system "off"
and "on" is needed in order to maintain the Stationary
contact
temperature of the space or product at a con-
stant level within reasonable limits and to pre-
vent the temperature of the space or product
from being reduced below the desirable
minimum. Bellows

For any refrigerating system, the relative


length of the "off" and "on" cycles will vary
with the loading of the system. During periods
of peak loading, the "running" or "on" cycles
will be long and the "off" cycles will be short,
whereas during periods of minimum loading the
"on" cycles will be short and the "off" cycles
will be long.* Fig. 13-4. Schematic diagram of simplified pressure

13-6. Cycling Controls. The controls used to control.

cycle a refrigerating system "on" and "off" are


of two principal types: (1) temperature actuated mostats. Whereas float controls are sensitive to
(thermostatic) and (2) pressure actuated. Each and are actuated by changes in liquid level,
of these types is discussed in the following
thermostats are sensitive to and are actuated by
sections.
changes in temperature. Thermostats are used
13-7. Temperature Actuated Controls. to control the temperature level of a refrigerated
Temperature actuated controls are called ther-
space or product by cycling the compressor
Bellows (starting and stopping the compressor driving
motor) in the same way that float controls are

^m used to control liquid level by cycling the pump


(starting
13-8.
and stopping the pump motor).
Temperature Sensing Elements. Two
<a)
types of elements are commonly used in thermo-
stats to sense and relay temperature changes to
the electrical contacts or other actuating mecha-
nisms. One is a fluid-filled tube or bulb which is
Diaphragm ~>k . connected to a bellows or diaphragm and filled
with a gas, a liquid, or a saturated mixture of
the two (Figs. 13-5aand 13-56).t Increasing the
temperature of the bulb or tube increases the
pressure of the confined fluid which acts through
(b)
the bellows or diaphragm and a system of levers
to close electrical contacts or to actuate other
Fig. 13-5. Bulb-type temperature sensing element.
compensating mechanisms (Fig. 13-6). Decreas-
ing the temperature of the tube or bulb will
* Unlike the water pumping system, refrigerating
have the opposite effect.
systems are designed to have sufficient capacity to
permit "off" cycles even during periods of peak f The thermostat described here is called a
loading. This is necessary in order to allow time for remote-bulb thermostat. Although there are a
defrosting of the evaporator. However, allowances number of different types of thermostats, this is the
are made for defrosting time in the load calculations type most frequently used in commercial refriger-
(the 24-hr load, is divided by the desired running ation applications. Thermostats are used for many
time to obtain the average hourly load) and need purposes other than controlling a compressor
not be further considered when selecting the equip- driving motor, as, for example, opening and closing
ment. valves, starting and stopping damper motors, etc.
232 PRINCIPLES OF REFRIGERATION
Dissimilar
metals

/
Invar-

___ Brass
"or steel
Fig. 13-7. Bimetal-t/pe tem-
perature sensing element.
_Bimetal
"element

Normal

M
Another and entirely different temperature of the temperature sensing element. Where the
sensing element is the compound bar, commonly temperature sensing element of the thermostat is
a bimetal element.
called The compound bar is located in or on the product and controls the
made up of two dissimilar metals (usually Invar product temperature directly, the differential is
and brass or Invar and steel) bonded into a flat usually small (2° F or 3° F). On the other hand,
strip (Fig. 13-7a). Invar is an alloy which has a where the sensing element is located in the space
very low coefficient of expansion, whereas brass and controls the space temperature, the differen-
and steel have relatively high coefficients of tial is ordinarily about 6° F or 7° F. In many
expansions. Since the change in the length of instances, the sensing element of the thermostat
the Invar per degree of temperature change will is clamped to the evaporator so that the space
always be less than that of the brass or steel, or product temperature is controlled indirectly
increasing the temperature of the bimetal ele- by controlling the evaporator temperature, in
ment causes the bimetal to warp in the direction which case the differential used must be larger
of the Invar (the inactive metal) as shown in (15° F to 20° F or more) in order to avoid short-
Fig. 13-76, whereas decreasing the temperature cycling of the equipment.
of the bimetal element causes the bimetal to When the thermostat controls the space or
warp in the direction of the brass or steel (the product temperature directly, the average space
active metal) as shown in Fig. 13-7c. This or product temperature is approximately the
change in the configuration of the bimetal ele- median of the cut-in and cut-out temperatures.
ment with changes in temperature can be utilized Therefore, to maintain an average space tem-
directly or indirectly to open and close electrical perature of 35° F, the thermostat can be adjusted
contacts or to actuate other compensating for a cut-in temperature of approximately 38° F
mechanisms. and a cut-out temperature of approximately
13-9. Differential Adjustment. Like float 32° F.
controls, thermostats have definite "cut-in" and On the other hand, when the space tempera-
"cut-out" points. That is, the thermostat is ture is controlled indirectly by controlling the
adjusted to start the compressor when the tem- evaporator temperature, an allowance must be
perature of the space or product rises to some made in the cut-out setting to compensate for
predetermined maximum (the cut-in tempera- the evaporator TD. For example, for an
ture) and to stop the compressor when the average space temperature of 35° F and assum-
temperature of the space or product is reduced ing an evaporator TD
of 12° F, to compensate
to some predetermined minimum (the cut-out for the evaporator TD, the cut-out temperature
temperature). would be set at 20° F (32° F - 12°) rather than
The difference between the cut-in and cut- at 32° F. Notice that the cut-in temperature is
out temperatures is called the differential. In set at 38° F in either case. This is because the
general, the size of the differential depends upon space temperature and the evaporator tempera-
the particular application and upon the location ture are the same at the time that the system
SYSTEM EQUILIBRIUM AND CYCLING CONTROLS 233

cycles on. After the compressor cycles off the in many cases it is necessary to use trial-and-
evaporator continues to absorb heat from the error methods to determine the optimum settings
space and warms up to the space temperature for a specific installation.
during the ofT cycle (Fig. 13-8). Therefore, when 13-10. Range Adjustment. In addition to the
the space temperature rises to the cut-in tem- differential, cycling controls have another adjust-
perature of 38° F, the evaporator will also be at ment, called the "range," which is also asso-
the cut-in temperature of 38° F. As soon as the ciated with the cut-in and cut-out temperatures.
compressor is started, the evaporator tempera- Although, like the differential, the range can be
ture is quickly reduced below the space tempera- defined as the difference between the cut-in and
ture by an amount approximately equal to the cut-out temperatures, the two are not the same.
design evaporator TD. Therefore, in this in- For example, assume that a thermostat is
stance, when the space temperature is reduced adjusted for a cut-in temperature of 30° F and a
to 32° F (the desired minimum), the evaporator cut-out temperature of 20° F. Whereas the
temperature (which the thermostat is controlling) differential is said to be 10° F (30° - 20°), the
willbe approximately 20° F (12° F less than the range is said to be between 30° F and 20° F.
space temperature). Although it is possible to change the range
Regardless of whether the thermostat controls without changing the differential, it is not
the space temperature directly or indirectly, possible to change the differential without
proper adjustment of the cut-in and cut-out changing the range. For instance, suppose that
temperatures is essential to good operation. If the thermostat previously mentioned is re-
the cut-in and cut-out temperatures are set too adjusted so that the cut-in temperature is raised
close together (differential too small) the system to 35° F and the cut-out temperature is raised to
will have a tendency to short-cycle (start and 25° F. Although the differential is still 10° F
stop too frequently). This will materially reduce (35° — 25°), the operating range of the control
the life of the equipment and may result in other is 5° F higher than it was originally, that is, the
unsatisfactory conditions. On the other hand, operating range is now between 35° F and 25° F,
if the cut-in and cut-out temperatures are set too whereas previously it was between 30° F and
far apart (differential too large), the on and off 20° F. In this instance, the range of the control
cycles will be too long and unnecessarily large is changed, but the differential remains the same.
fluctuations in the average space temperature Under the new control setting the average space
will result. Naturally, this too is undesirable. temperature will be maintained approximately
Although approximate cut-in and cut-out 5° F higher than under the old setting.
temperature settings for various types of appli- Suppose now that the differential is increased
cations have been determined by field experience, 5° F by raising the cut-in temperature from 30° F

-Average evaporator temperature


-Minimum evaporator temperature (cut-out temperature)

Fig. 13-8. Notice that when the unit cycles "on," the evaporator temperature is the same as the space
temperature, whereas when the unit cycles "off," the evaporator temperature is lower than the space
temperature by an amount equal to the design evaporator TD.
234 PRINCIPLES OF REFRIGERATION

, Differential adjustment

Fig. 13-9. Schematic diagram


of thermostatic motor control
illustrating range and differen-
tial adjustments.

Range
adjustment

to 35° F while the cut-out temperature is left at Turning the differential-adjusting screw clock-
the original setting of 20° F. Notice that both wise causes the limit bar A to move toward the
the differential and the range are changed. The screw head, thereby increasing the travel of the
differential, originally 10° F, is now 15° F and pin B in the slot. This has the effect of in-
the range, originally between 30° F and 20° F, is creasing the differential by lowering the cut-out
now between 35° F and 20° F. With this control temperature. Turning the differential adjusting
setting, the running cycle will be somewhat screw counterclockwise raises the cut-out tem-
longer because the differential is larger. Too, By manipu-
perature and reduces the differential.
the average space temperature will be 2 or 3 latingboth range and differential adjustments,
degrees higher because the cut-in temperature is the thermostat can be adjusted for any desired
higher. If the differential had been increased by cut-in and cut-out temperatures.
lowering the cut-out temperature 5° F rather The arrangement shown in Fig. 13-9 repre-
than by raising the cut-in temperature 5° F, the sents only one of a number of methods which
operating range of the control would have can be employed to adjust the cut-in and cut-out
and the average
shifted to the opposite direction temperatures. The particular method used in
space temperature would have been 2 or 3 any one control depends on the type of control
degrees lower than the original space tempera- and on the manufacturer. For example, for the
ture. controlshown in Fig. 13-9, changing the range
Typical range and differential adjustments adjustment changes both the cut-in and cut-out
are shown in Fig. 13-9. Turning the range- temperatures simultaneously, whereas for an-
adjusting screw clockwise increases the spring other type of control changing the range adjust-
tension which the bellows pressure must over- ment changes only the cut-in temperature. For
come in order to close the contacts and, stillanother type of control, changing the range
therefore, raises both the cut-in and cut-out adjustment changes only the cut-out tempera-
temperatures. Turning the range-adjusting ture. However, whatever the method of adjust-
screw counterclockwise decreases the spring ment, the principles involved are similar and the
tension and lowers both the cut-in and cut- exact method of adjustment is readily deter-
out temperatures. mined by examining the control . In many cases,
SYSTEM EQUILIBRIUM AND CYCLING CONTROLS 235

instructions for adjusting the control are given directly through evaporator temperature con-
on the control itself. trol. Which of these two methods of control is
If electrical contacts are permitted to open or the most suitable for any given application
close slowly, arcing will occur between the con- depends upon the requirements of the applica-
tacts, and burning or .welding together of the tion itself.
contacts will result. Therefore, cycling controls For applications where close control of the
which employ electrical contacts must all be space or product temperature is desired, a
equipped with some means of causing the con- thermostat which controls the space or product
open and close rapidly in order to avoid
tacts to temperature directly will ordinarily give the best
arcing.In Fig. 13-9, the armature and per- results. On the other hand, for applications
manent magnet serve this purpose. As the where off-cycle defrosting is required and where
pressure in the bellows increases and the minor fluctuations in the space or product tem-
movable contact moves toward the stationary perature are not objectionable, indirect control
contact, the strength of the magnetic field of the space temperature by evaporator tempera-
between the armature and the horseshoe magnet ture controlis probably the better method.
increases rapidly. When the movable contact In order to assure complete defrosting of the
approaches to within a certain, predetermined, evaporator, the evaporator must be allowed to
minimum distance of the stationary contact,the warm up to a temperature of approximately
strength of the magnetic field becomes great 37° or 38° F during each off cycle. When the
enough to overcome the opposing spring tension thermostat controls the evaporator temperature,
so that the armature is pulled into the magnet the cut-in temperature of the thermostat can be
and the contacts are closed rapidly with a snap set for 38° F. Since the evaporator must warm
action. up to this temperature before the compressor
As the pressure in the bellows decreases, can be cycled on, complete defrosting of the
spring tension acts to open the contacts. How- evaporator during each off cycle is almost
ever, since the force of the spring is opposed certain. On the other hand, when the thermo-
somewhat by the force of magnetic attraction, stat controls the space temperature, there is no
the contacts will not separate until a consider- assurance that the evaporator will always warm
able force is developed in the spring. This up sufficiently during the off cycle to permit
causes the contacts to snap open quickly so that adequate defrosting.
arcing is again avoided. If we assume that the thermostat is properly
Toggle mechanisms are also frequently used adjusted, if the load on the system is relatively
as a means of causing the contacts to open and constant and the capacity of the system is
close with a snap action. Too, some controls handle the load, no defrosting prob-
sufficient to
employ a mercury switch as a means of over- lems are likely to arise with either method of
coming the arcing problem. A typical mercury temperature control. However, if the system
switch is illustrated in Fig. 13-10. As the glass is subject to considerable changes in load,
tube is tilted to the right, the pool of mercury defrosting problems are sometimes experienced
enclosed in the tube make contact between the in applications where the thermostat controls
two electrodes. As the bulb is tilted back to the the space temperature. When the system is
left, contact is broken. The surface tension of
the mercury provides the snap action necessary Glass tube

to prevent arcing.
13-11. Space Control vs. Evaporator Con-
trol. When the sensing element of the thermo-
stat is located in the space or in the product, the
thermostat controls the space temperature or
product temperature directly. Likewise, when Pool of
the sensing element is clamped to the evaporator, mercury
the thermostat controls the evaporator tempera-
Contacts
ture directly. In such cases, control of the space
or product temperature is accomplished in- Fig. 13-10. Mercury contacts.
236 PRINCIPLES OF REFRIGERATION

operating tinder peak load conditions, the tem- the system rises above a certain, predetermined
perature of the space tends to remain above the pressure, the high-pressure control acts to break
cut-out temperature of the thermostat for the circuit and stop the compressor. When the
extended periods so that running cycles are long pressure on the high-pressure side of the system
and frost accumulation on the evaporator is returns to normal, the high-pressure control
heavy. Too, under heavy load conditions, the and start the compressor.
acts to close the circuit
space temperature warms up to the cut-in However, some high-pressure controls are
temperature of the thermostat very quickly equipped with "lock-out" devices which require
during the off cycle so that the off cycles are that the control be reset manually before the
usually short. Frequently, the off cycles are too compressor can be started again. Although
short to allow adequate defrosting of the eva- high-pressure controls are desirable on all

porator. In such cases, when the compressor systems, because of the possibility of a water
cycles on again, the partially melted frost is supply failure, they are essential on systems
caught on the evaporator and frozen into ice. utilizing water-cooled condensers.
Eventually the evaporator will be completely Since the condensing pressures of the various
frozen over with ice, air flow over the evaporator refrigerants are different, the cut-out and cut-in
will be severely restricted, and the system will settings of the high-pressure control depend on
become inoperative. the refrigerant used.
13-12. Pressure Actuated Cycling Controls. 13-14. Low-Pressure Controls. Low-pressure
Pressure actuated cycling controls are of two and as
controls are used both as safety controls
types: (1) low-pressure actuated and (2) high- temperature controls. When used as a safety
pressure actuated. Low-pressure controls are control, the low-pressure control acts to break
connected to the low-pressure side of the system the circuit and stop the compressor when the
(usually at the compressor suction) and are low-side pressure becomes excessively low and
actuated by the low-side pressure. High- to close the circuit and start the compressor
pressure controls, on the other hand, are con- when the low-side pressure returns to normal.
nected to the high-pressure side of the system Like high-pressure controls, some low-pressure
(usually at the compressor discharge) and are controls are equipped with a lock-out device
actuated by the high-side pressure. which must be manually reset before the com-
The design of both the low-pressure and the pressor can be started.
high-pressure controls is similar to that of the Low-pressure controls are frequently used as
remote-bulb thermostat. The principal differ- temperature controls in commercial refrigeration
ence between remote-bulb thermostat and the applications. Since the pressure at the suction
pressure controls is the source of the pressure inlet of the compressor is governed by the
which actuates the bellows or diaphragm. saturation temperature of the refrigerant in the
Whereas the pressure actuating the bellows of evaporator, changes in evaporator temperature
the thermostat is the pressure of the fluid con- are reflected by changes in the suction pressure.
fined in the bulb, the pressures actuating the Therefore, a cycling control actuated by changes
bellows of the low-and-high-pressure controls in the suction pressure can be utilized to control
are the suction and discharge pressures of the space temperature indirectly by controlling the
compressor, respectively. Like the thermostat, evaporator temperature in the same way that
both controls have cut-in and cut-out points the remote-bulb thermostat is used for this
which are usually adjustable in the field. purpose. In such cases, the cut-in and cut-out
13-13. High-Pressure Controls. High- pressures of the low-pressure control are the
pressure controls are used only as safety controls. saturation pressures corresponding to the cut-in
Connected to the discharge of the compressor, and cut-out temperatures of a remote-bulb
the purpose of the high-pressure control is to thermostat employed in the same application.
cycle the compressor off in the event that the For example, assume that for a certain appli-
pressure on the high-pressure side of the system cation the cut-in and cut-out temperature
becomes excessive. This is done in order to settings for a remote-bulb thermostat are 38° F
prevent possible damaging of the equipment. and 20° F, respectively. If a low-pressure con-
When the pressure on the high-pressure side of trol is used in place of the thermostat, the cut-in
SYSTEM EQUILIBRIUM AND CYCLING CONTHOLS 237

pressure setting for the lew-pressure control wj|] line would cause the control to cycle the com-
be SQpsia (the saturation pressure or R-12 pressor when the pressure in the evaporator was
corresponding to a temperature of 38 V} and reduced to only 39 psia rather than the desired
the cut-out pressure selling will be 36 psia (the 36 psia. The system would have a tendency to
saturation pressure of R-12 corresponding to a short cycle because the differential is too small
temperature or 20= F)-* and unsatisfactory operation would result.
As thie evaporator warms up during the off Pressure loss in the suction line in no way
cycle, the pressure in the evaporator increases ajfects the cut-in selling of the control. Since
accordingly. When the pressure in the evapora- pressure drop a function of velocity or how,
is
tor rises to the cut-in pressure setting of the low- there is no pressure drop in the suction line
pressure control, the low-pressure control acts when the system is idle. As soon as the com-
to close the circuit and start the compressor. pressor cycles off, the pressure at the suction of
Very soon after the compressor start*. the compressor evaporator pressure so
rises to the
temperature and pressure of ihe evaporator are that at the time the compressor cycles on the
reduced to approximately the design evaporator pressure at the compressor inlet is the same
temperature and pressure and they remain nt as the evaporator pressure. Hence, the cut-in
this condition throughout most of the running pressure setting of the control is made without
cycle (see Fig. 13-8}, Near
or the the end regard for the pressure drop in the suction
running cycle the evaporator temperature and line.
pressure are gradually reduced below the design Since the low-pressure control controls eva-
conditions. When the evaporator pressure is porator temperature rather than space tempera-
reduced to the cut-out pressure setting of the ture, it is an ideal temperature control for
low-pressure control, the control Ads to break applications requiring off-cycle defrosting. This
the circuit and stop the compressor. is particularly true for '"remote" installations
Since the refrigerant vapor undergoes drop where the compressor is located some distance
in pressure while flowing through the suction from the evaporator, tn such installations, low-
line, thepressure of the vapor at the suction pressure temperature control has a distinct
inletof the compressor is usually 2 or 3 lb less advantage over thermostatic temperature con-
than the evaporator pressure. This is particu- trol in that it ordinarily results in a considerable
larly when the compressor is located
true saving in wiring.
electrical Because of the
some distance from the evaporator. Since ihe remote bulb, the thermostat must always be
low-pressure control is actuated by the pressure located near the evaporator or space whose
at the suction inlet of the compressor, the temperature is being controlled. This requires
pressure drop accruing in the suction line must that a pair of electrical conductors be installed
be taken into account when the pressure control between the fk)it are and the condensing unit. On
Bettings are made. To compensate for Ihe the other hand, the low-pressure control is
pressure loss in the suction line, the cut-out located at the compressor near the power source
pressure setting is lowered by an amount equal so that the amount of control wiring needed is

to the pressure loss in the lino. For example, much less.

assuming a 3-1 b pressure loss in the suction line, 13-15. Dual-Praiiur* Controls, A dual-pres-
when the pressure in the evaporator is 36 psia, sure control Is a combination or the low-and-
the pressure at the suction inlet of the com- high- pressure controls in a single control.
pressor will be 33 psia. Hence, if it is desired to Ordinarily, only one set of electrical contact
cycle the compressor off when the pressure in points are used in the control, although a
the evaporator is reduced to 36 psia, the cut-out separate bellows assembly is employed for each
pressure of the low-pressure control is set for of the two pressures. A typical dual pressure
33 psia. In this instance, failure to make an control is shown in Fig. 13-11. This type of
nltowuiicc for the pressure loss in the Miction pressure control is frequently supplied as Stan*
* When refrigerants other than R-12 are raed ui dard equipment on condensing units.
the system, the pressure setting! will be the nar- m 13*16. The Pump-Down Cycle, A commonly

ration praam es of those refrigerants corresponding used method of cycling the condensing unit,
to the desired cut-in and cut-out temperature* known as a "pump-down cycle," employs both a
33S PRINCIPLES OF REFRIGERATION

Rg. 11-11. Du*l pnmirt control, {Covrtmj 9< P«nn Contrail. Inc.)

thermostat and a low-pressure control. In 1 the compressor. When the temperature of the
pump-dawn or evaporator tccrh
cycle, the space space or evaporator rises to tile cut-in tem-
perature is controlled directly by the thermostat. perature of the thermostat, the thermostat
However, instead of starting and stopping the closes the solenoid circuitand energizes the
compressor driver, the thermostat nets 10 open solenoid, thereby opening the liquid line and
and clou a solenoid valve installed in the liquid permitting liquid refrigerant to enter the
line, usually near the refrigerant flow control evaporator. Since the c^ipumtar is warm,
(Fig. 13-12). As the space or evaporator tem- the liquid entering the .-..ipuraujr vaporises
perature is reduced to the cut-out temperature rapidly so that the evaporator pressure rises
of the thermostat, the thermostat breaks the immediately to the cut-in pressure of the
solenoid circuit, thereby de-energizing the low-pressure control, whereupon the low-
solenoid and interrupting the Gow of liquid pressure control closes the compressor driver
refrigerant to the evaporator- Continued circuit and start* the compressor.
operation of the compressor causes evacuation The advantages of the purip-down cycle are
of the refrigerant from that portion of the many. One of the most important ones being
system beyond the point where the refrigerant that the amount of refrigerant absorbed by the
flow is interrupted by the solenoid. When the oil in the cranfceasc of the compressor during the
pressure in the evacuated portion of the system olf cycle is substantially reduced. The problem
is reduced to [he cut-out pressure of the of crankcase oil dilution by refrigerant absorp-
low pressure control, the low-pressure control tion during the off cycle is fully discussed in

breaks the compressor driver circuit and stops Chapter IS.


STSTEM EQUILIBRIUM AND CYCLING CONTROLS 239

13-17. Variation i in System Capacity. Si is well-designed system will operate very nearly
worth while lo notice that balh the operating at the design conditions,
conditions and the capacity ol" a refrigerating 13-10, Capacity Control. The importance of
system change as the load on lie system changes.
I balancing the system capacity with the system
When the load on the system is heavy and heI load cannot be overemphasized. Any time the
space temperature is high, I he evaporator TD System capacity deviates considerably from the
will be somewhat larger than the design evapo- system load, unsatisfactory operating conditions
rator TD and the capacity of the evaporator will will result. It has already been pointed out that
be greater than the design evaporator capacity. good practice requires that the system be
Because of the higher evaporator capacity, the designed to have n capacity equal to or slightly
suction temperature will also be higher so that in excess of the average maximum sustained
equilibrium is maintained between the vaporiz- load. This is done so that the system will have
ing and condensing sections of the system. sufficient capacity to maintain the temperature
Hence, under heavy load conditions, the system and humidity at the desired level during
operating conditions arc Somewhat higher than periods of peak loading. Obviously, as the
the average design conditions and the system cooling load decreases, there is a lendency for
capacity is somewhat greater than the average the system to become oversized in relation to
design capacity. Obviously, the horsepower the load.
requirements of the compressor are greatest In applications where the changes in the
at this peak load condition and the compressor average system load are not great, capacity
driver must be selected lo have sufficient horse- control is adequately accomplished by cycling
power lo meet these requirements. the system on and oft as described in the
Conversely, when the load on the system is preceding sections. In such cases, assuming
light, the space temperature wilt be lower (Kan that the cycling controls are property adjusted,
the average design space temperature, the the relative length of the on and off cycles wilt
evaporator TD will
than the design TD,
be less vary with the load on the system. During
and the suction temperature will be lower than periods when the load is heavy, on cycles will
the design suction temperature. Therefore, the be long and off cycles will be short, whereas
system operaiing conditions wilt be somewhat during periods when the load is light, on cycles
tower than the average design operating con* will be short and off cycles will be long. Natur-
ditiofls and the system capacity will be some- ally, the degree of variation in the length of
what less than the average design capacity the on and off cycles will depend on the degree
During each running cycle the system passes of load fluctuation. However, since the system
through a complete series of operating condi- must always be designed to have sufficient
tions and capacities, the operaiing conditions capacity to handle the maximum load, when
and capacity being highest at the beginning of the changes in the system load are substantial,
the running cycle when the space temperature it is evident that the system will be considerably
is and lowest at the end of the running
highest, oversized when the load is at a minimum. A
cycle when the space temperature Is lowest. system which is oversized for the load will
However, during most of the running cycle, a usually prove lo be as unsatisfactory as one that

!,„
V ftiwef JinT"

flew antral

c
I EvwofHw
Flj. I S>-|1. Pump-down cycle.

c
hmmh i. :i i. VPii
nwnmiii
140 PRINCIPLES OF MFMGERATtON
reduction) and latent (nvuistun reduction)
loads.* In applications where the latent load
b too large a percentage of the total load,
satisfaction of the latent load will result in
ovcrcooling of the space utile** sensible heat is

artificially introduced into ihe space. In such


cases, the sensible heat is usually added to the
space in the form of relic, u The air is first
paased across a cooling coil and cooled to the
temperature necessary to reduce the moisture
content to the desired level ,md the air is then
reheated to ihe required dr\ hulb temperature.
Fif, I J- 13. Evaporator iplit Into two lefmcnti lor
capacity control. Clining tht wl«noid rtlv» In (hi The reheating is accomplished with steam or
llqwW tto* Of iff rtwfii A rtndtrt ihlt portion of hot water coils, with electric Mrip-heatcrs, or
th* •viporvcor irwpemiva. Th* cipicn, reduction with hot gas from the compressor discharge,
obulnad U proportion*) to th* iiiriac* iru cydwl In some installations, iIk- refrigerating capa-
city of the system is adequately controlled by
controlling ihe capacity of the compressor only.
Since the flow rate of the refrigerant must be
is undersized for the load. When the system is the same in all component*, any change in the
undersized for the load, the running: »"« will capacity of any one component will automati-
be excessive, the space temperature will be high cally result in a similar .iiljustment in the
Tor emended periods, and the off cycles will be capaci ly of al the o ther
I com portents. Therefore,
too short to permit adequate dcfrosiing of ihe increasing or decreasing the capacity of the
evaporator. On the other hand, where the compressor will, in effect, increase or decrease
system is oversized for the load, the off cycles the capacity of the entire sisicm, However,
will be loo long and the equipment running it important to notice that with this method
is
time wtll he insufficient to remove the required of capacity control the operating conditions of
amount of moisture from the space. This will the system will change as the capacity of the
result in unsatisfactory {higher than normal) system changes.
humidity conditions in the refrigerated space. Where it is desired to vnr\ ihe Capacity of the
For this reason, when changes in the system System without allowing the operating condi-
toad arc substantial, it is usually necessary to tions of the system to change it is necessary lo
provide some means of automatically (or
manually) varying the capacity of the system
other than by cycling Ihe system on and off.
This b true also of large installations when the
size of the equipment renders cycling the system

on and off impractical.


There are many methods of bringing the
refrigerating capacity Into balance with the
refrigerating load. Naturally, the most suitable
method in any one case will depend upon the
conditions and requirements of the installation
itself.Some installations require only one or
two step* of capacity control, whereas others
require a number of steps. Frequently, several
methods are employed simultaneously in order Fig- 11-14. Cell drtultad (or lac* control. (Court**?
Kifimrd Diviiion. Am»He»n Air Filter Company,
to obtain the desired flexibility. Too, in some
Inc.)
cases, it is necessary to impose an artificial
load on the equipment to achieve the proper * This problem is usually rr,- -
< < .it utc in the winter'
balance between the sensible (temperature lime when the transmission i» .11 gAin) load is light.
SYSTEM EQUILIBRIUM AND CYCLING CONTROLS 341

control both the evaporator capacity and the


compressor capacity directly.*
Some of the more common methods of con-
trollingevaporator and compressor capacities
are considered in the following sections.
13-1?. Evaporator Capacity Control. Prob-
Fact damper
ably the most effective method of providing
evaporator capacity control it lo divide the Fig. It 14, Evaporator equipped with * (ace damper
evaporator into several separate sections or co vary the quantity otalrpasjin g ever chc evaporator.
which are individually controlled so that
circuits Al the dam par movn toward the cloud paiition. the
one or more sections or circuit* an be cycled rwiiunea aialrut which the Mower muat wsrk is

out as the land decreases (Fig. 13-13}. Using Increased so that the total quantity of air circulated
this method, any percentage of the evaporator

capacity con be cycled out in any desired


number of sieps. The number and size of ofTected.tOn the other hand, "depth" control
the individual evaporator sections depend on always changes the sensible heat ratio. As a
the number of steps of capacity desired and the general rule, the more depth the evaporator has
percent change in capacity required per step, the greater is its (moisture
Jatcnt cooling
respectively. The arrangement of the evaporator removal) capacity. Hence, as one or more rows
sections or circuiting depends on the relation* of the evaporator are cycled out, the sensible

ship of the sensible load to ihe total load at 11k heat ratio increases.
various load conditions- Basically, two circuit Another common method of varying the
arrangements are possible- Evaporator circuit- evaporator capacity is to vary the amount of
ing can be arranged lo provide cither "face" air circulated over (he evaporator through the

control of "depth" control, or both (Fig. 13-14 use of "face" or "face-ond-by-pass" dampers.
and 11-15). When "face" control is used, the (Fig*. 1 3- 16 and 13-17) Muflispeed blowers can
"sensible heat ratio" of the evaporator also be used for this purpose. Too, is some in-
is not
siances, mul lis peed blowers and dampers are u%d
together if) order to provide the desired balance.
In nearly all cases, application of any of Ihe
foregoing methods of evaporator capacity
control will necessitate simultaneous control of
compressor capacity.
13-M, Compressor Capacity Control. There
in a number of different methods of controlling
the capacity of reciprocating compressors. One
method, already mentioned, is to vary the speed
of the compressor by varying the speed of the
compressor driver. When an engine or turbine
a employed to drive the compressor. Ihe
compressor capacity can he modulated over a
wide range by governor control of the com-
pressor driver.
When an electric motor drives the compressor.
Hb> I J- IS, Coll circuit Ml for dapih control. (Cour*
Only two speeds are usually available SO that the
teijr Ksrmind DMtl&n, American Air Filter Company,
compressor operates either at full capacity or at
Inc.).
30% capacity. When more than two speeds ore
desired, it is necessary to use two separate
* Any reduction in ivucjii loud unj/tit lyalcni
windings in the motor, in which case four speeds
capacity will also have some elf«t on the capacity
will be possible.
of the condenser and on the sin of the refrigerant
lines. These topics lie d taunted in Chapter* J 4 ; Ratio of the acntlbic coding capacity to the
and 17, respectively. total cooling capacity.
MI PWKCJPLES OF KEFRlGc RATION

By- pass damp** the cut-in pressure of the prcuure control, the
control energizes the solenoid valve and admits
i

Mt
! - high-pressure gas from the condenser to the
unloadcr piston which act* to depress ihe
suction valves and bold them open. When
the suction pressure rises to the cut-out pressure

I - of the pressure control, the solenoid valve


energized and the unloadcr piston is
is

relumed
de-
to
\Fk» dime** the normal position.
In addition to providing; opacity control,
Pit- fMT. Evaporator aqulpped with fit* and by- cylinder unloadcr* of all t;,pes art used to
pus dimptn lor dpicicy control. D*mp*r* if* unload the compressor cylinders during com-
JrttereoAnectad w
tint by-pui damper opum wider
pressor start-up so that the compressor starts
a* ft« d»mp*r It tlowd eft. With thii iirinjerrmnt
in an unloaded condition, thereby reducing (he
tht quantity of air puling e*»r traporstor can til*
inrush current demand.
be rtfulated by allowing mart or leu »lr to by-pua
iht avipcrator . Hcwtver, ragirdilMJ of tba petition When any of the capacity control methods
of thi dim para, tht total quaintly of air circulated described thus far are used. Ihe horsepower
remain* practically tht aim*. requirements of the compressor decrease as the
capacity decreases, although not in the same
proportions.
Capacity control of multicylinder corn- Another method of controlling compressor
pretwrt it frequently obtained by "unloading" capacity is to throttle the cm if pressor suction.
one or more cylinders that they become» However, since it reduces compressor capacity
ineffective.One method of accomplishing this uiihout reducing compressor horsepower, this
is by-pas the discharge from one or more
to method it seldom used.
cylinder] buck into the suction tine as shown Still another meant of con trolling compressor
in Fig. 13-18. When the suction pressure drop* capacity which it employed with good results is
to a certain predetermined value a solenoid to operate two or more compressors in parallel
valve in the by-pass line, actuated by a pressure (Fig. 13-20). Individual low.. pressure controls
control, opens and alky** ihe discharge from ore used to cycle the compressor*. The cut-in
one or more cylinders to flow through the and cut-out pressures of the mili vidua] controls
by-put tine back into the suction line where it are so adjusted that the corn pressors cycle off
mixes with the incoming suction vapor. At in sequence as the suction pressure decreases
long as (he suction pressure remains below the and cycle on in sequence when the suction
cut-in setting of the conirol. the discharge from pressure rises. Very often these compressors
the unloaded cylinders continues to by-pus to are equipped wilh cylinder unio;iders to provide
the suction line. When Ihe suction pressure additional steps of control Multiple com-
rises to the cut-out setting of the pressure- pressor systems are discussed in detail in
con trot, the solenoid valve ,% dc-energixed and Chapter 20.
the by-pass tine is closed so that the compressor
it returned to full capacity operation.
Another method of unloading compressor
cylinders is to depress the suction valves of the
cylinder or cylinders to be unloaded so that they
remain open during the compression (up)
stroke. With the auction v.thes held open, die
suction vapor drawn into the cylinder during
the suction stroke is returned to the suction line
during the compnuvDn stroke. A typical
unloadcr of this type it shown in Fig. 13-19,
The operation of the unloadcr mechanism is
as follows: when the suction pressure falls (o He. I ML Schauta** diagram of cyilAoV by-pan.
SYSTEM EQUILIBRIUM AND CYCUNG CONTROLS 243

Typical
solenoid valve
(shown energized)

Connect to
discharge side
of compressor

Fig. 13-19. Condenser pres-


sure actuated cylinder un-
loader mechanism. (Courtesy
Dunham-Bush, Inc.)

Valve plate

Cylinder

13-21. Multiple-System Capacity Control. This method of capacity control is well


Another method of controlling capacity is to where only two steps of
suited to installations
employ two or more separate systems. The capacity control are required, as in chilling or
evaporators for the separate systems may be in combination chilling and storage applications.
the same housing and air stream or they may The use of two or more separate systems has the
be in separate housings and air streams. In added advantage of providing a certain amount
either case, separate compressors and condensers of insurance against losses accruing from
are used, although in some instances the con- equipment failure. Should one system become
densers may be in a common housing. inoperative, the other can usually hold the load
until repairs can be made.

PROBLEMS
1. Assuming a 2° F loss in saturated suction
temperature due to refrigerant pressure drop
in the suction line.
(1) Select an air-cooled condensing unit to
operate in conjunction with the natural
convection evaporator in Problem 12-1.
(2) Plot the evaporator and condensing unit
Fig. 13-20. Two compressors installed in parallel as capacities on a common graph and
a means of controlling compressor capacity. As the determine:
load diminishes, one compressor is cycled out to (a) The saturated suction temperature at
reduce the compressor capacity. Often, one com- the point of system balance
pressor is equipped with cylinder unloaders to pro- (b) The capacity of the system in Btu/hr at
vide additional steps of control. the point of system balance.
: .

densing medium includes both the heat absorbed


in the evaporator and the heat of compres-
sion, the heat load on the condenser always
exceeds that on the evaporator by an amount
equal to the heat of compression. Since the

14 work (heat) of compression per unit of refrig-


erating capacity depends upon the compression
ratio, the heat load on the condenser per unit of
evaporator load varies with the operating

Condensers and conditions of the system.


The quantity of heat liberated at the con-

Cooling Towers denser


capacity
per minute per
at various
ton
and condensing
suction
of evaporator

temperatures can be estimated from Charts 14-1,


14-2, and 14-3. Chart 14-1 applies to R-12
systems, whereas Charts 14-2 and 14-3 apply to
R-22 and R-717 (ammonia) systems, respec-
14-1. Condensers. Like the evaporator, the tively. The values given are based on a simple

condenser is a heat transfer surface. Heat from saturated cycle.


the hot refrigerant vapor passes through the
Example 14-1. An R-12 system, operating
wails of the condenser to the condensing at a 15° F
suction temperature, has a condens-
medium. As the result of losing heat to the ing temperature of 100° F. Determine the load
condensing medium, the refrigerant vapor is on the condenser in Btu per minute per ton.
first cooled to saturation and then condensed
Solution. In Chart 14-1, locate the 15° F
into the liquid state. suction temperature line at the base of the graph.
Although brine or direct expansion refrig- Follow the line until it intersects the 100° F
erants are sometimes used as condensing condensing temperature curve. The value on
mediums in low temperature applications, in the left-hand index corresponding to this point
the great majority of cases the condensing is approximately 245 Btu per minute per ton.

medium employed is either air or water, or a any given set of operating


It is evident that for
combination of both. conditions there a fixed relationship between
is
Condensers are of three general types: the condenser load and the evaporator load.
(1) air-cooled, (2) water-cooled, and (3) evapo- For instance, for the R-12 system described in
rative. Air-cooled condensers employ air as the Example 14-1, the relationship between the
condensing medium, whereas water-cooled condenser load and the evaporator load is such
condensers utilize water to condense the that 245 Btu are liberated at the condenser for
refrigerant. In both the air-cooled and water- each 200 Btu taken in at the evaporator.
cooled condensers, the heat given off by the Once the relationship between the condenser
condensing refrigerant increases the temperature load and the evaporator load has been estab-
of the air or water used as the condensing lished for any given set of operating conditions,
medium. the total condenser load corresponding to any
Evaporative condensers employ both air and given total evaporator load can be easily
water. Although there is some increase in computed. The following equation may be used
the temperature of the air passing through the
condenser, the cooling of the refrigerant in the ax* He
(14-1)
condenser results initially from the evaporation
of the water from the surface of the condenser. where Qc = the condenser load in Btu/hr
The function of the air is to increase the rate of Qe = the evaporator load in Btu/hr
evaporation by carrying away the water vapor qe = the condenser load in Btu/min/ton
which results from the evaporating process. (from Fig. 14-15)
14-2. The Condenser Load. Since the heat
qe
= the evaporator load in Btu/min/ton
given up by the refrigerant vapor to the con- (always 200 Btu)
244
CONDENSERS AND COOLING TOWERS 245

Note: Q may also be in Btu/min or in tons,


t Since the condenser capacity must always be
in which case Qe will be in Btu/min or in tons, equal to the condenser load, it is evident from

respectively. the foregoing that, for any given condensing


load, the larger the surface area of the condenser,
Example For the system described in
14-2. the smaller will be the required temperature
Example 14-1, determine the load on the con- differential and the lower will be the condensing
denser if the load on the evaporator is 35,000
temperature. Too, since the load on the
Btu/hr.
condenser is always proportional to the load on
Solution. Applying the evaporator (system), any increase or decrease
Equation 14-1, the load = 35,000 x 245/200 in the load on the evaporator will be reflected by
on the condenser, Qc = 42,875 Btu/hr an increase or a decrease, respectively, in the
condensing temperature.
It is important to notice that any increase or 14-4. Quantity and Temperature Rise of
decrease in the load on the evaporator (system) Condensing Medium. In both the air-cooled
will result in a proportional increase or decrease
and water-cooled condensers, all the heat given
in the load on the condenser.
off by the condensing refrigerant increases the
14-3. Condenser Capacity. Since heat trans-
temperature of the condensing medium. There-
ferthrough the condenser walls is by conduction,
fore, in accordance with Equation 2-8, the
condenser capacity is a function of the funda-
temperature rise experienced by the condensing
mental heat transfer equation: medium in passing through the condenser is

directly proportional to the condenser load and


Q=A x U x D (14-2)
inversely proportional to the quantity and
where Q= the condenser capacity (Btu/hr) specific heat of the condensing medium, viz:
A = the surface area of the condenser
ft
(sqft) (Ta - Tx) =
U= the transfer coefficient of the con- MxC (14-3)

denser walls (Btu/hr/sq ft/° F) where Tx = the temperature of the air or water
D— the log mean temperature difference entering the condenser (7",)
between the condensing refrigerant T2 = the temperature of the air or water
and the condensing medium leaving the condenser (Tx)
Examination of the factors in Equation 14-2 (r2 — Tj) = the temperature rise (AT) experi-
will show that for any fixed value of U enced in the condenser
the capacity of the condenser depends on the = the load on the condenser in Btu
Q,
surface area of the condenser and on the per hour
temperature difference between the condensing M= the weight of air or water circulated
refrigerant and the condensing medium. It is through the condenser in pounds
evident also that for any one condenser of per hour
specific size and design, wherein the surface C= the specific heat at constant
area and the U factor are both fixed at the time pressure of the air or water
of manufacture, the capacity of the condenser Assuming that C has a constant value, for
depends only on the temperature differential any given condenser load (Q,), Equation 14-2
between the refrigerant and the condensing contains only two variables, and AT, the M
medium. Therefore, for any one specific value of each being inversely proportional
condenser, the capacity of the condenser is to the value of the other, viz:
increased or decreased only by increasing or
decreasing the temperature differential. Further-
more, if it is assumed that the average tempera-
M = C xQ.AT (14-4)

ture of the condensing medium is constant, it


AT =
follows that an increase or a decrease in the
capacity of the condenser is brought about only
C xM (14-5)

by an increase or a decrease, respectively, in the Therefore, for any given condenser load, if
condensing temperature. the temperature rise of the condensing medium
246 PRINCIPLES OF REFRIGERATION

is known, the quantity of condensing medium convert from pounds per minute to cubic feet
circulated through the condenser in pounds per per minute, viz:
hour can be determined by applying Equation
cfm = M (lb/hr)
rz —
x t>(cu ft/lb)
:

14-4. Likewise, if the quantity circulated is


60 min
known, the temperature rise through the con-
Assuming the specific volume of the air to be
denser can be computed by applying Equation
the specific volume of standard air (13.34 cu ft/
14-5.
lb), incorporation of these conversion factors
Average specific heat values for air and water
into Equation 14-7 results in the following:
are 0.24 Btu/lb and 1 Btu/lb, respectively. By
substituting the appropriate value for C, Q, x 13.34 cu ft/lb
cfm =
Equations 14-4 and 14-5 can be written to apply 0.24 x 60 x AT
specifically to either water or air, viz: or, combining constants (13.34/0.24 x 60 =

for water -£ (14-6)


1/1.08),

cfm = Qs
1.08.x AT
(14-11)

»-% (14-7) Example 14-3. If the load on a water-


cooled condenser is 150,000 Btu/hr and the
Q'
and for air M
"'
0.24 x AT
(14-8) temperature rise of the water in the condenser
is 10° F. What is the quantity of water cir-
culated in gpm ?
Qs
IT
*"
0.24 x M (14-9)
Solution. Applying Equation 150,000
14-10, the water quantity in gpm
Since general practice is to express air and 500 x 10
water quantities in cubic feet per minute (cfm)
= 30 gpm
and in gallons per minute (gpm), respectively, Example 14-4. TheJoad on a water-cooled
it is usually desirable to compute condensing condenser is 90,000 Btu/hr. If the quantity of
medium quantities in these units rather than in water circulated through the condenser is 15
pounds per hour. gallons per minute, determine the temperature
To convert pounds of water per hour into rise of the water in the condenser.

gallons per minute, divide by 60 min to reduce Solution. Rearranging and 90,000
pounds per hour to pounds per minute, and applying Equation 14-10, AT ~ 500 x 15
then divide by 8.33 lb per gallon to convert
= 12°F
pounds per minute to gallons per minute, viz:
Example 14-5. Thirty-six gallons of water
M(lb/hr)
gpm per minute are circulated through a water-
60 min x 8.33 lb/gal cooled condenser. If the temperature rise of the
water in the condenser is 12° F, compute the
If these conversion factors are incorporated
load on the condenser in Btu/hr.
into Equation 14-6, the water quantity can be
computed directly in gpm. The following equa- Solution. Rearrang-
tion results: ing and applying Equa- = 500 x Ar x gpm
tion 14-10, the load = 500 x 12 x 36
Q.
8pm = on Q, the condenser, = 216,000 Btu/hr
60 x 8.33 x Ar
Example 14-6. The load on an air-cooled
or, combining constants (60 x 8.33 500), condenser is 121,500 Btu/hr. If the desired
temperature rise of the air in the condenser is
G, 25° F, determine the air quantity in cfm which
gpm (14-10)
500 X AT must be circulated over the condenser.
Toreduce pounds of air per hour to cubic Solution. Applying Equa-
feet per minute, dividepounds per hour by 60 tion 14-11, the air quantity in 121,500
min to determine pounds per minute and then cfm 1.08 x 25
multiply by the specific volume of the air to 4500 cfm
CONDENSERS AND COOLING TOWERS 247

92°-».Water out
Fig. 14-1. Water temperature
rise through condenser.

Example 14-7. Three thousand cfm of air greater the flow rate of the condensing medium
are circulated over an air-cooled condenser. If the lower will be the condensing temperature.
the load on the condenser is 64,800 Btu/hr, 14-5. Condenser Application. As a general
compute the temperature rise of the air passing rule, forany given condenser load, the size of
over the condenser.
the condenser and the quantity of condensing
Solution. Rearranging and medium circulated will depend upon the
applying Equation 14-1 1, AT = -^64,800
= 20°F
--
^ entering temperature of the condensing
and upon the desired condensing temperature.
A careful analysis of the
medium

data in Sections 14-3


For any given condenser and condenser and 14-4 will show that the condensing tempera-
loading, the condensing temperature of the ture of the refrigerant in the condenser is a

refrigerant in the condenser will depend only function of three variables: (1) the entering

upon the average temperature of the con- temperature of the condensing medium, (2)
densing medium flowing through the condenser. the temperature rise in the condenser, and (3) the

The lower the average temperature of the con- temperature difference between the refrigerant
densing medium the lower is the condensing and the condensing medium. This relationship
temperature. For example, assume that the is illustrated in Fig. 14-1.

size and loading of a condenser are such that


Recalling that the temperature rise in the

the required mean temperature differential condenser varies inversely with the flow rate of
between the refrigerant and the condensing the condensing medium and that the tempera-

medium is 15° F. If the average temperature of ture differential between the refrigerant and the
the condensing medium is 90° F, the condensing condensing medium varies inversely with the

temperature will be 105° F (90 + 15), whereas if size (surface area) of the condenser,* it is

the average temperature of the condensing evident that:

medium is 85° F, the condensing temperature


1. For any given condensing surface and
will be 100° F (85 + 15).
flow rate, die condensing temperature will
The average temperature of the condensing
increase or decrease as the entering temperature
medium flowing through the condenser depends of the condensing medium increases or de-
upon both the initial temperature of the creases.
condensing medium entering the condenser and
2. For any given entering temperature, the
the temperature rise experienced in the con-
and the higher the
larger the condensing surface
denser. Since the temperature rise of the
flow rate, the lower will be the condensing
condensing medium decreases as the flow rate
temperature.
increases, the greater the quantity of condensing
3. For any given entering temperature, the
medium circulated the lower is the average
temperature of the condensing medium. There- * Assuming the transfer coefficient to be con-
fore, for any given condenser loading, the stant.
248 PRINCIPLES OF REFRIGERATION
amount of condensing surface required for a increases. If the flow rate is increased beyond a
given condensing temperature decreases as the certain point, the increase in the power required
flow rate of the condensing medium increases. to circulate the condensing medium will more
than power require-
offset the reduction in the
With regard to the latter statement, this ments of the compressor accruing from the
means in effect that the same condensing increased flow rate. Therefore, the quantity of
temperature can be maintained with either a condensing medium which can be economically
small condensing surface and a high flow rate circulated is limited by the power requirements

or a large condensing surface and a low flow of the fan, blower, or pump.
rate. However, it should be recognized that the Obviously, the optimum flow rate for the
flow rate of the condensing medium is fixed condensing medium is the one which will result
within certain limits by the size and design of in the lowest over-all operating costs for the
the condenser. If the flow rate through the system. This will vary somewhat with the
condenser is too low, flow will be streamlined conditions of the individual installation, being
rather than turbulent and a low transfer coeffi- influenced by the type of application, the size
cient will result. On the other hand, if the flow and type of condenser used, fouling rates, and
rate is too high, the pressure drop through the the design conditions for the region, along with
condenser becomes excessive, with the result such practical considerations as the cost and
that the power required to circulate the con- availability of water, utility costs, local codes
densing medium also becomes excessive. and restrictions, etc. For example, since good
Since the design entering temperature of the system efficiency prescribes lower condensing
condensing medium is usually fixed by condi- temperatures for low temperature applications
tions beyond the control of the system designer, than for high temperature applications, it
it follows that the size and design of the con- follows that for the same condenser load the
denser and the flow rate of toe condensing optimum condensing medium flow rate will
medium are determined almost entirely by the usually be higher for a low temperature appli-
design condensing temperature. cation than for a high temperature application.
Although low condensing temperatures are Too, where the entering temperature of the
desirable in that they result in high compressor condensing medium is relatively high, larger
efficiency and low horsepower requirements for condensing surfaces and higher flow rates
the compressor, this does not necessarily mean are required to provide reasonable condensing
that the use of a large condensing surfaceand a temperatures than where the entering tempera-
high flow rate in order to provide a low con- ture of the condensing medium is lower.
densing temperature will always result in the 14-6. Air-Cooled Condensers. The circula-
most practical and economical installation. tion of air over an air-cooled condenser may be
Other factors which must be taken into account either by natural convection or by action of a
and which tend to limit the size of the condenser fan or blower. Where air circulation is by
and/or the quantity of condensing medium natural convection, the air quantity circulated
circulated are initial cost, available space, and over the condenser is low and a relatively large
the power requirements of the fan, blower, or condensing surface is required. Because of
pump circulating the condensing medium. Too, their limited capacity natural convection con-
where water is used as the condensing medium densers are used only on small applications,
and the water leaving the condenser is wasted principally domestic refrigerators and freezers.
to the sewer (see Section 14-9), the availability Natural convection condensers employed on
and cost of the water must also be considered. domestic refrigerators are usually either plate
The limitations imposed on condenser size by surface or finned tubing. When finned tubing
the factors of initial cost and available space is used, the fins are widely spaced so that little
are self-evident. As for the power requirements or no resistance is offered to the free circulation
of the fan, blower, or pump circulating the of air. Too, wide fin spacing reduces the
condensing medium, it has already been stated possibility of the condenser being fouled with
that the power required to circulate the con- dirt and lint.
densing medium increases as the flow rate The plate-type condenser is mounted on the
CONDENSERS AND COOLING TOWERS 249

back of the refrigerator in such a way that an limitation in physical size, chassis-mounted
formed to increase air circulation.
air flue is condensers of the type shown in Fig. 6-12 are
Finned tube condensers are mounted either on rarelyfound in capacities larger than 2 tons.*
the back of the refrigerator or at an angle Another disadvantage of the chassis-mounted
underneath the refrigerator. Regardless of air-cooled condenser is their susceptibility to
condenser type or location, it is essential that the fouling. Since most condensing units are
refrigerator be so located that air is permitted mounted on the condenser air tends
floor, the
to circulate freely through the condenser at all to sweep across the floor so that dirt, lint, and
times. Too, warm locations, such as one other foreign materials are picked up from the
adjacent to an oven, should be avoided when- floor and carried to the surface of the condenser,
ever possible. thereby "clogging" the condenser and restricting
A number of domestic freezer manufacturers the air flow.
utililize the outer shell of the freezer (outside Too, on "open-type" air-cooled condensing
wall surface) as a condensing surface. This is units the condenser fan is usually mounted on
accomplished by bonding bare tubing to the the shaft of the compressor driver (Fig. 6-12).
inside surface of the outer shell so that the Naturally this limits both the size and the
entire outer shellbecomes a plate-type heat location of the fan so that the quantity of air
transfer surface. The use of these "wrap- circulated over the condenser is always less
around" condensers permits a considerable than that which would produce maximum
reduction in the size of the freezer (6 to 8 in. on efficiency at full load conditions.Notice also
both length and width), not only because it that, because of the fan location, the distri-
eliminates the space ordinarily required for the bution of the air over the condenser surface is
condenser but also because it allows the use of very uneven, being much greater on the end of
3 to 4 in. of insulation in the walls where the condenser directly in front of the fan.
normally 6 to 8 in. is required in order to prevent Remote air-cooled condensers are used in
moisture from condensing on the outside surface sizesfrom 1 ton up to 100 tons or more and may
of the freezer. The slightly higher operating be mounted either indoors or outdoors. When
costs which accrue as a result of reducing the located indoors, provisions must be made for an
amount of wall insulation is more than offset adequate supply of outside air to the condenser
by the savings in space that this practice makes (Fig. 14-2). If the condenser is installed in a
possible. warm location, such as in an attic or boiler
Air-cooled condensers employing fans or room, ducts should be used to carry the air into
blowers to provide "forced-air" circulation can the condenser and back to the outside. Because
be divided into two groups according to the of the large quantity of air required, only the
location of the condenser: (1) chassis-mounted smaller sizes are mounted indoors.
and (2) remote. When located outdoors, the air-cooled con-
A chassis-mounted air-cooled condenser is denser may be mounted on the ground, on the
one that is mounted on a common chassis with roof, or on the side of a wall, with roof locations
the compressor and compressor driver so that being the most popular. Typical wall and roof
it is an integral part of the air-cooled "con- installations are shown in Figures 14-3 and
densing unit" (Fig. 6-12). Although chassis- 14-4, respectively. In all cases, the condenser
mounting of the air-cooled condenser makes should be so oriented that the prevailing winds
possible a very compact, completely self- for the area in the summertime will aid rather
contained condensing unit which is ideally than retard the action of the fan. In the event
suited for use on small commercial fixtures, that such orientation is not possible, wind
this arrangement has certain inherent disadvan- deflectors should be installed on the discharge
tages which make chassis-mounting impractical side of the condenser (Fig. 14-5).
in larger applications.
* This is the approximate condenser capacity
The principal disadvantage of chassis-
required on a 3 hp, commercial, air-cooled con-
mounted air-cooled condensers is that the densing. Approximately 25 %
of the motor horse-
physical size of the condenser is limited to power is required to drive the fan. Naturally, this
the dimensions of the chassis. Because of the reduces the horsepower available to the compressor.
250 PRINCIPLES OF REFRIGERATION

Ceiling Purge valve

Fig. 10-2. Indoor installation of


air-cooled condenser. (Cour-
tesy Kramer Trenton Com-
pany.)

/ Compressor''
* Locate receiver below unicon outlet

One significant outgrowth of the remote air- type is rapidly gaining in popularity and is now
cooled condenser has been the development of available in almost any desired capacity.
a new type of air-cooled condensing unit which 14-7. Air Quantity and Velocity. For an
is designed specifically for remote installation. air-cooled condenser there is a definite relation-
The air-cooled condensing unit illustrated in ship between the size (face area) of the con-
Fig. 14-6 is typical of these newer designs. This denser and the quantity of air circulated in that
the velocity of the air through the condenser is

critical within certain limits. Good design pre-


scribes the minimum air velocity that will pro-
duce turbulent flow and a high transfer coefficient.
Increasing the air velocity beyond this point
causes an excessive pressure drop through the
condenser and results in an unnecessary increase
in the power requirements of the fan or blower
circulating the air.
The velocity of the air passing through an air-
cooled condenser is a function of the free face
area of the condenser and the quantity of air
circulated. The relationship is given in the
following equation:

Air quantity (cfm)


Air velocity (fpm) =
Free face area (sq ft)

The free face area of the condenser is the area


air spaces between the tubes and fins.
of the free
The actual free area per unit of face area varies
with the design of the condenser, being depend-
Fig. 14-3. Remote air-cooled condensers installed on ent upon the size, number, and arrangement of
outside wall. (Courtesy Kramer Trenton Company.) the tubes and fins.
1

CONDENSERS AND COOLING TOWERS 25

Fig. 14-4. Remote air-cooled condensers mounted on roof. (Courtesy Dunham-Bush, Inc.)

Normally, air velocities over air-cooled con- manufacturer's recommendations as to the air
densers are between 500 and 1000 fpm. How- quantities should be carefully followed.
ever, because of the many variables involved, 14-8. Rating and Selection of Air-Cooled

the optimum air velocity for a given condenser Condensers. Capacity ratings for air-cooled
design is best determined by experiment. For condensers are usually given in Btu/hr for
this reason, most air-cooled condensers come various operating conditions. It has already
from the factory already equipped with fans or been shown that since the surface area and the
blowers so that the air quantity and velocity value of U are fixed at the time of manufacture,
over the condenser are fixed by the manu- the capacity of any one condenser depends only
facturer. In all cases, to realize peak perform- on the mean temperature difference between
ance from an air-cooled condenser, the the air and the condensing refrigerant. Since

Fig. 14-5. Remote air-cooled


condensers equipped with
wind deflectors. (Courtesy
Kramer Trenton Company.)
252 PRINCIPLES OF REFRIGERATION

Fig. 14-6. Air-cooled condensing unit designed for remote installation. Notice generous size of condenser.
(Courtesy Kramer Trenton Company.)

most air-cooled condensers come equipped with In order to select a condenser from the rating
fans or blowers, the quantity of air circulated tables, the following design data must be known :

over the condenser is also fixed so that the (1) The design suction and condensing tem-
average temperature of the air passing over the peratures
condenser depends only on the dry bulb tem- (2) The compressor capacity in Btu/hr
perature of the entering air and the load on the (3) The design outdoor dry bulb temperature
condenser. Obviously, then, in such cases, the (use values in Table 10-6A. Round off to
capacity of the condenser is directly proportional next highest multiple of 5)
to the temperature difference between the dry
Example 14-8. From Table R-13, select
bulb temperature of the entering air and the an air-cooled condenser for a compressor
condensing temperature. This temperature having a capacity of 75,000 Btu/hr if the design
differential is often referred to as the "tempera- evaporating and condensing temperatures are
ture split" in order to distinguish it from the 20° F and 110° F, respectively, and the outdoor
mean effective temperature differential.* design dry bulb for the region is 90° F.
Table R-13 is a typical manufacturer's rating Solution. From Table
table for air-cooled The basic
condensers. R-l 3B, the correction fac-
ratings given in Table R-13A are based on 90° F tor for 20° F suction tem-
ambient air temperature, 120° F condensing perature = 0.95
temperature, and 40° F evaporating tempera- From Table R-12C, the
ture. For other design conditions multiply the correction factor for con-
densing temperature of
basic rating from Table R-13 A by the correction 110° F and entering air
factors for variation in evaporating temperatures
temperature of 90° F = 0.665
(Table R-13B) and for variation in entering air
Required capacity of
and condensing temperatures (Table R-13Q. condenser at basic rating 75 000
* The temperature split is always proportional to
conditions - 0.95 x 0.665
the METD. = 11 8,700 Btu/hr
CONDENSERS AND COOLING TOWERS 253

From Table R-13A, select condenser Model shown that, in general, a water flow rate of
#BD1000 which has a capacity of 120,000 between 2.5 and 3 gal per minute per ton usually
Btu/hr at the basic rating conditions. provides the most economical balance between
compressor horsepower and pump horsepower.
Experience has shown that as a general rule In some instances, the water supply for a
selecting an air-cooled condenser on the basis waste-water system is taken from a well or from
of a condensing temperature of 1 10° F will some nearby body of water, such as a river,
most economical condenser
result in the size. lake, pond, etc., in which case both the cost of
Hence, the actual size of the condenser selected the water and the pumping horsepower must be
will depend upon the outdoor design dry bulb considered in determining the optimum water
temperature for the region in question. The flow rate.
higher the dry bulb temperature, the larger the To a large extent, the quantity of water cir-

condenser required. For example, for a con- culated through the condenser determines the
densing temperature of 1 10° F, if the dry bulb design of the water circuit in the condenser.
temperature is 85° F, the condenser can be Since heat transfer is a function of time, it

selected for a 25° F temperature split, whereas follows that where low water quantities necessi-
if the dry bulb temperature is 90° F, the con- tate a high temperature rise in the condenser,

denser must be selected for a 20° F temperature the water must remain in contact with the con-
split, which will require a larger size. densing refrigerant for a longer period than
14-9. Water-Cooled Condenser Systems. when the water flow rate is high and the tem-
Systems employing water-cooled condensers can perature rise required is smaller. Hence, where
be divided into two general categories: (1) the water flow rate is number of water
low, the
waste-water systems and (2) recirculated water circuitsthrough the condenser are few and the
systems. In waste-water systems the water long so that the water will remain in
circuits are

supply for the condenser is usually taken from the condenser for enough time to permit the
the city main and wasted to the sewer after required amount of heat to be absorbed. On
passing through the condenser (Fig. 14-7). In the other hand, when the flow rate is high and
recirculated water systems the water leaving the the temperature rise low, more circuits are used
condenser is piped to a water cooling tower and the circuits are shorter in order to reduce
where its temperature is reduced to the entering the pressure drop to a minimum. This is illu-
temperature, after which the water is recircu- strated in Figs. 14-9a and 14-96. In Fig. 14-9a,

lated through the condenser (Fig. 14-8). the two water circuits through the condenser are
Naturally, where the condenser water is connected in series for a low flow rate and a high
wasted to the sewer, the availability and cost of temperature rise. The water enters through
the water are important factors in determining opening A and leaves through opening C.
the quantity of water circulated per unit of con- Opening B is capped. In Fig. 14-9ft, the two
denser load. As a general rule, an economical water circuits are connected in parallel for a high
balance between water and power costs pre- flow rate and a low temperature rise. The water
scribes a water flow rate of approximately 1.5 enters through opening B and leaves through
gal per minute per ton of capacity. openings A and C.
The high cost of water, along with limited In designing the condenser water circuit
sewer facilities and recurring water shortages in must be given to the water
particular attention
many regions, has tended to limit waste-water
/Water regulating valve
systems to very small sizes. Too, many cities

have placed severe restrictions on waste-water Suction Warm mater out _^.
systems, particularly where the water supply is
taken from the city main and wasted to the
sewer.
When the condenser water is recirculated the
power required to circulate the water through
the water system must be taken into account in
determining the water flow rate. Experience has Fig. 14-7. Waste water system.
254 PRINCIPLES OF REFRIGERATION

Fig. 14-8. Recirculating water


system.
Hot gas in

Pump-^

velocity and pressure drop through the con- the water is forced through the condenser by
denser. In all cases the minimum permissible city main pressure, the pressure drop through
velocity is that which will produce turbulent the condenser is not critical as long as it is

flow and a high transfer coefficient. Since within the limits of turbulent flow and the avail-
pressure drop is a function of velocity, the able city main pressure. In such cases, high
pressure drop through the condenser increases velocities are recommended in order to take
as the water velocity increases. For this reason, advantage of the higher transfer coefficient. On
the maximum permissible velocity in any one the other hand, when the water is circulated by
case is usually determined by the allowable action of a pump, a high pressure drop through
pressure drop. * For waste-water systems, where the condenser will increase the pumping head
and the power required to circulate the water.
Therefore, for recirculating water systems, the
optimum water velocity is one which will pro-
vide the most economical balance between a
high transfer coefficient and a low pumping
head.
In Figs. 14-9a and 14-9A, it is of interest to
to)
notice that for the same flow rate the velocity
and pressure drop through the circuit arrange-
ment in Fig. 14-9a are approximately four times
as great as that through the circuit arrangement
//t\rtsJDhJjt\ C Water in
in Fig. 14-96. Too, because of the higher velo-
Water out
|
** >* ** '*'
city, the transfer coefficient
somewhat higher
is
B ~U A\ A\ A\ A\
\y \y \y_\y w A Water in
for the condensing surface in Fig. 14-9a and less
condensing surface is required for the same heat
transfer capacity.
<b)
14-10. Fouling Rates. Another factor which
Fig. 14-9. (o) Water circuit connected for series must be considered in selecting a water-cooled
flow, (b) Water circuit connected for parallel flow. condenser is fouling of the tube surface on the
* Excessive velocity will usually cause erosion of water side. The fouling is caused primarily by
the water tubes, particularly at points where the mineral solids which precipitate out of the water
water changes direction. The maximum velocity and adhere to the tube surface. The scale thus
recommended by Air Conditioning and Refrigeration formed on the tube not only reduces the water
Institute (ART) is 8 fps. side transfer coefficient, but it also tends to
CONDENSERS AND COOLING TOWERS 255

restrict the water tube and reduce the quantity this arrangement, some air-cooling of the refrig-
of water circulated, both of which will cause erant is provided in addition to the water-
serious increases in the condensing pressure. cooling. Counterflowing of the fluids in any
In general, the rate of tube fouling is in- type of heat exchanger is always desirable since
fluenced by: (1) the quality of the water used it results in the greatest mean temperature
with regard to the amount of impurities con- difference between the fluids and, therefore, the
tained therein, (2) the condensing temperature, highest rate of heat transfer.
and (3) the frequency of tube cleaning with Several types of double-tube condensers are
relation to the total operating time. shown in Figs. 14-11 and 14-12. The type
Most manufacturers of water-cooled con- shown in Fig. 14-11 can be cleaned mechan-
densers give condenser ratings for clean tubes ically by removing the end-plates (inset). The
and for four stages of tube fouling in accordance type shown in Fig. 14-12 is cleaned by circu-
with the scale factors given in Table 14-1 for lating approved chemicals through the water
various types of water. These scale factors are tubes (see Section 14-23).
an index of the reduction in the tube transfer Equipped with water-regulating valves (Sec-
coefficient resulting from the scale deposit. In tion 14-20), double-tube condensers make ex-
selecting a water-cooled condenser, a minimum cellent "booster" condensers for use with

Refrigerant
"
vapor in

Water out
Fig. 14-10. Double -tube
water-cooled condenser.

Water in^

Condensed
refrigerant out
P
scale factor of 0.0005 should always be used. chassis-mounted air-cooled condensers during
Under no circumstances should a condenser be periods of peak loading. Since the water valve
selected on the basis of clean tubes. However, can be adjusted to open and allow water to flow
when the condensing temperature is low (leaving through the condenser only when the con-
water temperature less than 100° F) and the densing pressure rises to some predetermined

condenser tubes are to be cleaned frequently, level, the amount of water used is relatively
the fouling factor from Table 14-1 may be small in comparison to the savings in power
reduced to the next lowest value. The use of afforded by the increased compressor effi-

scale factors will be illustrated later in the ciency.


chapter. The shell-and-coil condenser is made up of
14-11. Water-Cooled Condensers. Water- one or more bare-tube or finned-tube coils
cooled condensers are of three basic types: enclosed in a welded steel shell (Fig. 14-9). The
(1) double-tube, (2) shell-and-coil, and (3) shell- condensing water circulates through the coils
and-tube. while the refrigerant is contained in the shell

As its name implies, the double-tube con- surrounding the coils. Hot refrigerant vapor
denser consists of two tubes so arranged that enters at the top of the shell and condenses as it
one is inside of the other (Fig. 14-10). Water is comes in contact with the water coils. The con-
piped through the inner tube while the refrig- densed liquid drains off the coils into the bottom
erant flows in the opposite direction in the of the shell, which often serves also as the
space between the inner and outer tubes. With receiver tank. Care should be taken not to
256 PRINCIPLES OF REFRIGERATION

Fig. 14-11, Double-pipe condenser* wlih mechanically deanable tube*. {Courtwy Hilnead and Mitchell.)

overcharge the system with refrigerant since an from one end to the other before leaving the
excessive accumulation of liquid in the ton- condenser. The number of passes may be as few
denser will tend to cover too much of the con- as two or as many as twenty.
densing surface and cause an increase in Frit For any given total number of lubes, the
discharge temperature and pressure. number of tubes per pass varies inversely with
Most sheli-and-coil condensers are equipped the number of passes. For example, assuming
with a split water circuit. The two parts of the that a condenser has a total of forty tubes, if
circuit are connected in series for waste-water there are two passes, the number of tubes per
systems (Fig. J 4-96) and in parallel for recircu- pass is twenty, whereas if there are four passes,
lating systems (Fig. i4-9a). As a general rule, the number of tubes per pass is ten.

shcll-and-coil condensers arc used only for small It is important to notice that for the same
installations up to approximately 10 tons total number of tubes and the same water
capacity, quantity, the velocity of the water and the
Shelt-and-coil condensers are cleaned by cir- pressure drop through the condenser will be four
culating an approved chemical through the times as great for a four-pass condenser as for a
water coils. two-pass condenser. Because of the higher velo-
The sbcll-and-tube condenser consists of a city the transfer coefficient will be higher for the
cylindrical steel shell in which a number of four-pass condenser and a smaller condensing
straight tubes are arranged in parallel and held surface will be required for a given heat transfer
in place at the ends by tube sheets. Construc- capacity. However, on the other hand, because
tion is almost identical to that of the flooded- of the high pressure drop, the power required to
type shell-and-tube liquid chiller. The con- circulate the water will be greater. Hence, for a
densing water is circulated through the tubes, waste- water system, the- four-pass condenser is
which may be either steel or copper, bare or probably the best selection, whereas for a
extended surface. The refrigerant is contained recirculating system, the two-pass condenser is

in the steel shell between the tube sheets. Water probably the better of the two.*
circulates in the annular spaces between the tube Shcil-and-fube condensers are available in
sheets and the end-piates, the end-plates being capacities ranging from 2 tons up to several
baffled to act as manifolds to guide the water * This example intended only In illustrate the
is
flow through the tubes. The arrangement of the principles of design and should not be construed
end-plate bathing determines the number of to mean that four-pass condensers are undesirable
passes the water makes through the condenser for recirculating systems.
CONDENSERS AND COOLING TOWERS 157

Hg. 14-12. Typical double-pipe condenser cortfigur«iofl*. (fl) Trombone configuration, (t) HeUlt configura-
tion. (Courtesy Edwardi Engineering Corporation.)

hundred tons or more. Shell diameters range Table R-I4 are based on condensing tempera-
from approximately 4 in, up to 60 in,, whereas tures of 102° and 105^ F, 20" and 10" water rise
tube length varies from approximately j ft to 20 and 0.0005 scale factor which is the minimum
ft. The number and the diameter or the tubes recommended in ARf standards.
depend on the diameter of the shell. Tube Where other conditions exist,the following
diameters of through 2 in. are common,
g in. procedure should be followed in selecting the
whereas the number of Lubes in the condenser proper condenser.
varies from as few as six or eight to as many as a Condensers must not be selected for less than
thousand or more. The end-plates of the con- 0.5gpm per tube below which streamline instead
denser are removable to permit mechanical of turbulent water flow occurs. ART standards
cleaning of the water tubes. indicate that the water velocity should not
Single-pass, vertical sheH-and-tube condensers exceed E fps which is 5.75 gpm per tube for
are sometimes employed on large ammonia Acme STF and SRF condensers.
installations. The construction of the vertical It is necessary to have the following informa-
she! -and- lube
I condenser is similar to that of the tion to select a proper condenser
vertical shcll-and-tubc chiller illustrated in Fig.
1. Total tons (low side).
1 1 -44. The vertical condenser is equipped * i th a
2. Evaporator temperature.
water box at the top to distribute the water to 3. Condensing temperature.
the tubes and a drain at the bottom to carry the
4. Water temperature "in."
water away. Bach tube is equipped at the top 5. Water temperature "out," or gpm avail-
with a distributor fitting which imparts a rotating
able.
motion to the water to assure adequate wetting 6. Type of water or required scale factor.
of the lube. The hot refrigerant vapor usually
enters at the side of the shell near the middle of Then proceed as follows

the condenser and the liquid leaves the con- 1 Determine the corrected tons to be used in
denser at the side of the shell near the bottom. selecting the proper condenser by reference to
The height of vertical shell-and-tube condensers Fig. 2, Table R-I4. The factor obtained for the
ranges from 12 ft to IE ft. The tubes are desired evaporator temperature and condensing
mechanically clcanabk. temperature is multiplied by the actual tons to
14-12- and Selection of Water'
Rating obtain corrected tons.
Cooled Condensen.* The ratings shown in 2. Determine the water temperature rise and
* The maLcriiil in this section is reprinted directly gpm per ton. Knowing cither factor, the other

from the manufacturer's catalog, the only alter* may be obtained by reference to Fig. 3, Table
ation being the. table designations. Courtesy of R-14. Use corrected tons to determine the total
Acme Industries, Inc. gpm required.

258 PRINCIPLES OF REFRIGERATION

3. Determine the temperature differences Refrigeration load 30 tons


between the condensing temperature and the Condensing temperature 100° F
"water in" and "water out" temperatures and
Suction temperature 30° F
Water available 2 gpm/ton
find the METD by referring to Table 11-1.
river water reasonably clean at 78°F
4. Make preliminary selection of condenser
Maximum tube length 12 ft
shell diameter by reference to Table R-14, Maximum water pressure drop 7.5 psi
basing the selection on the corrected tons found
in step 1 . Find the number of tubes per pass and Solution
then by referring to step 2, find the gpm per 1. From Fig. 2, the correction factor for
tube.
30° F temperature and 100° F con-
suction

5. Select the desired scale factor by reference


densing temperature is 1.013.
Corrected tons 30 x 1.013 = 30.4 tons
to Table 14-1 which suggests scale factors for
2. From Fig. 3, for 2 gpm/ton the water
various types of water. The most commonly
temperature rise is found to be 14.4°.
used factor 0.0005 and it should be borne in
30.4 x 2 = 60.8
is
Total gpm
mind when selecting a factor that a determina- Water "out" temperature 78 plus 14.4
tion is being made of the frequency of cleaning = 92.4° F
which will be required. 3. GTD 100 - 78 = 22°
6. Referring to Fig. 1, Table R-14, determine LTD 100 - 92.4 = 7.6°
the rate of heat transfer "£/" for the gpm per From Table 11-1, METD = 13.55° F
tube in step 4 and the scale factor in step 5. 4. Refer to Table R-14. Use of four passes
will usually give an economical selection for
7. Calculate the surface required by use of the
75° F water in and 95° F water out which
following formula.
approximates the required water conditions.
Square feet of surface Note that a lOf shell will probably be needed.
This shell has sixty tubes.
Corrected tons x 14,400
=
U x METD gpm
or per
r tube — —^ — — f
Total gpm x number of passes
-
;
Number of tubes in condenser
:

8. Select a condenser having at least the _ 60.8 x 4


required surface from Table R-14. Be sure to 60
use the shell diameter determined in the pre- = 4.05 gpm per tube
liminary selection of step 4.
5.Referring to Table 14-1, for clean river
9. Make final checks on selection.
water and over 3 fpm velocity, the suggested
Using the gpm per tube from step 4 and
a. scale factor is 0.001.
the nominal tube length shown in Table R-14 6. 1, the V factor for 4.05 gpm per
From Fig.
for the model selected in step 8, refer to Fig. tube and 0.001 scale factor is 121.5 Btu per hour
4 of Table R-14 to obtain water pressure drop per square foot of extended surface per °F
through condenser. METD.
b. Obtain nominal operating charge from 7. Square feet required

the last column of Table R-14. This is the Corrected tons x 14,400
maximum weight of liquid refrigerant which U factor x METD
can be allowed in the shell during the operat- _ 30.4 x 14,400
~ = 266 sq ft
ing period covering some of the lower tubes. 121.5 x 13.55
Larger shell diameters or separate receivers
8. Referring to Table R-14, a Model STF-
may be used where greater storage capacity is 1010 has 289 sq ft external tube surface and
needed during operation. should be selected. When installed the water
c. Determine the pump down capacity connection should be made for four-pass
from Table R-14. If less than the total weight operation.
of refrigerant to be used in the system and 9. (a). For water pressure drop, refer to Fig.

provision for complete pump-down are re- 4 and note that the pressure drop for 4.05 gpm
per tube in an STF-1010 condenser connected
quired, an additional receiver should be used.
for four passes is 7.1 psi. (b). Table R-14 shows
Example 14-9. Select an R-12 condenser a nominal operating charge of 38 lb of R-12,
to meet the following conditions: which will normally be sufficient for a 30-ton
CONDENSERS AND COOLING TOWERS 259

installation. However, if more operating exposed water surface and the length (time) of
storage needed, a separate receiver may be
is exposure, (3) the velocity of the air passing
chosen, or alternately a different condenser through the tower, and (4) the direction of the
selection may be made if more economical, (c).
air flow with relation to the exposed water
Table R-14 also shows pump-down capacity
surface (parallel, transverse, or counter).
which is 252 lb of R-12. Usually this will be
sufficient, but if greater pump-down capacity is
For any given water temperature entering the
required, a separate receiver tank must be used. tower, the vapor pressure difference is essentially
a function of the wet bulb temperature of the
14-13. Simplified Ratings. Simplified ratings, entering air. In general, the lower the entering
based on the horsepower of the compressor wet bulb temperature, the greater the vapor
driver, are available for most air-cooled and pressure differential and the greater the tower
water-cooled condensers, particularly in smaller capacity.
sizes. Since the power required by the com- The exposed water surface includes: (1) the
pressor varies with both the evaporator load and surface of the water in the tower basin, (2) all
the compression ratio, it provides a reasonable wetted surfaces in the tower, and (3) the com-
index of the condenser load at all operating bined surface of the water droplets falling
conditions. Table R-l 5, which applies to double- through the tower.
tube condensers of the type shown in Fig. 14-12, Theoretically, the lowest temperature to
isa typical simplified condenser rating table. which the water can be cooled in a cooling
14-14. Cooling Towers. Cooling towers are tower is the wet bulb temperature of the entering
essentiallywater conservation or recovery de- air, in which case the water vapor in the leaving
vices. Warm
water from the condenser is air will be saturated. In actual practice, it is not
pumped over the top of the cooling tower from possible to cool the water to the wet bulb tem-
where it falls or is sprayed down to the tower perature of the air. In most cases, the tem-
basin. The temperature of the water is reduced perature of the water leaving the tower will be
as it up heat to the air circulating through
gives 7° to 10° F above the wet bulb temperature of
the tower. the entering air. Too, the air leaving the tower
Although there is some sensible heat transfer will always be somewhat less than saturated.
from the water to the air, the cooling effect in a The temperature difference between the tem-
cooling tower results almost entirely from the perature of the water leaving the tower and the
evaporation of a portion of the water as the wet bulb temperature of the entering air is
water falls through the tower. The heat to called the tower "approach." As a general rule,
vaporize the portion of water that evaporates is all other conditions being equal, the greater the
drawn from the remaining mass of the water so quantity of water circulated over the tower the
that the temperature of the mass is reduced. closer the leaving water temperature approaches
The vapor resulting from the evaporating pro- the wet bulb temperature of the air. However,
cess is carried away by the air circulating the quantity of water which can be economically
through the tower. Since both the temperature circulated over the tower is somewhat limited by

and the moisture content of the air are increased the power requirements of the pump.
as the air passes through the tower, it is evident The temperature reduction experienced by the
that the effectiveness of the cooling tower water in passing through the tower (the differ-
depends to a large degree on the wet bulb tem- ence between the entering and leaving water
perature of the entering air. The lower the wet temperatures) is called the "range" of the tower.
bulb temperature of the entering air, the more Naturally, to maintain equilibrium in the con-
effective is the cooling tower. denser water system, the tower "range" must
The efficiency of a cooling tower is influenced always be equal to the temperature rise of the
by all the factors governing the rate at which the water in the condenser.*
water will evaporate into the air (see Section The load on a cooling tower can be approxi-
4-8). Some of the factors which determine mated by measuring the water flow rate over the
cooling tower efficiency are: (1) the mean
difference in vapor pressure between the air and * Except where a condenser by-pass is used. See
the water in the tower, (2) the amount of Section 14-17.
:

260 PRINCIPLES OF REFRIGERATION

tower and the entering and leaving water tem- depends on the design of the tower and the wind
peratures. The following equation is applied velocity.
"Bleed-off" is the continuous or intermittent
Tower load(Btu/min) =
flow rate(gpm)
wasting of a certain percentage of the circulated
x 8.33 x (entering water temperature
— water in order to avoid a build-up in the con-
leaving water temperature) (14-12)
centration of dissolved mineral solids and other
Example 14-10. Determine the approxi- impurities in the condenser water. Without
mate load on a cooling tower if the entering and bleed-off the concentration of dissolved mineral
leaving water temperatures are 96° F and 88° F, solids in the condenser will build up quite rapidly
respectively, and the flow rate of the water over as a result of the evaporation taking place in the
the tower is 30 gpm. cooling tower. Since the scaling rate is propor-
tional to the quality of the water, as the concen-
Solution. Applying
14-12, the tower load tration of mineral solids in the water increases

(Btu/min) = 30 x 8.33 the scaling rate also increases.


x (96 - 88) The amount of bleed-off required to maintain
= 2000 Btu/min the concentration of dissolved mineral solids at
a reasonable level depends upon the cooling
Since the load on the tower is equal to the
range, the water flow rate, and the initial water
load on the condenser, the approximate refrig-
conditions. Suggested bleed-off rates for various
erating capacity of the system can be computed
by dividing the tower load by the condenser load cooling ranges are given in Table 14-2. To
in Btu/min/ton corresponding to the operating determine the quantity of water loss by bleed-
conditions of the system. off, multiply the water flow rate over the tower
by the factor obtained from Table 14-2.
Example 14-1 1. Compute
the refrigerating
capacity of an R-13 system operating on the Example 14-12. Determine the quantity of
cooling tower of Example 14-10, if the evaporat- water lost by bleed-off if the water flow rate over
ing and condensing temperatures are 20° F and the tower is 30 gpm and the range is 10° F.
110° F, respectively.
Solution. From
Solution. From Fig. 14-1, Table 14-1, the percent
the load on the condenser bleed-off required = 0.33 %
= 247 Btu/min/ton The quantity of
water lost by bleed-off = 30 gpm x 0.0033
The approximately refrig- = 0.099 gpm
erating capacity of the
system The bleed-off line should be located in the hot
Tower load (Btu/min) water return line near the top of the tower so
Condenser load (Btu/min/ton) that water is wasted only when the pump is
2000 running (Fig. 14-8).
247 Make-up water, to replace that lost by evapo-
= 8.1 tons ration, drift, and bleed-off, is piped to the
Since the heat absorbed per pound of water tower basin through a float valve which tends to
evaporated is approximately 1000 Btu, assuming maintain a constant water level in the basin.
a condenser load of 250 Btu/min/ton, the 14-15. Cooling Tower Design. According to

quantity of water evaporated per ton of refrig- the method of air circulation, cooling towers are
eration (evaporator) is approximately 0.25 lb classified as either natural draft or mechanical

per minute or 2 gal per hour. draft. When air circulation through the tower
In addition to the water lost by evaporation, is by natural convection, the tower is called a

water is lost from the cooling tower by "drift" natural draft or atmospheric tower. When air
and by "bleed-off." A
small amount of water in circulation through the tower is by action of a
the form of small droplets is entrained and fan or blower, the tower is called a mechanical
carried away by the air passing through the draft tower. Mechanical draft towers may be
tower. Water lost in this manner is called the further classified as either induced draft or
drift loss. The amount of drift loss from a tower forced draft, depending on whether the fan or
CONDENSERS AND COOLING TOWERS 261

amount of wetted surface in the tower and to


break up the water into droplets and slow its
Hot water in
fallto the bottom of the tower. Atmospheric
towers containing decking are called "splash-
deck." Often, in splash-deck towers, no spray
nozzles are used and the water is broken up into

droplets by the "splash-impact" method.


The quantity and velocity of the air passing
through a natural draft cooling tower depend
on the wind velocity. Hence, the capacity of a
natural draft tower varies with the wind velocity,
Cold water out
as does the amount of "drift" experienced. Too,
Make-up water
natural draft towers must always be located out-
from city main
of-doors in places where the wind can blow
Fig. 14-13. Natural draft-cooling tower. freely through the tower. In commercial appli-
cations, roof installations are common.
Since air through mechanical
circulation
blower draws the air through the tower or forces draft towers is by action of a fan or blower,
(blows) it through. A schematic diagram of a small mechanical draft towers can be installed
spray-type natural draft tower is shown in Fig. indoors as well as out-of-doors, provided that
14-13. Schematic diagrams of induced draft and an adequate amount of outside air is ducted into
forced draft towers are shown in Figs. 14-14 and and out of the indoor location. Too, since
14-15, respectively. larger air quantities and higher velocities can be
In the spray-type atmospheric tower, the used, the capacity of a mechanical draft tower
warm water from the condenser is pumped to per unit of physical size is considerably greater
the top of the tower where it is sprayed down than that of the natural draft tower. In addition,
through the tower through a series of spray most mechanical draft towers contain some sort
nozzles. Since the amount of exposed water of decking or fill to improve further the effi-

surface depends primarily on the spray pattern, ciency. Spray eliminators must be used in
a good spray pattern is essential to high effi- mechanical draft towers to prevent excessive
ciency. The type of spray pattern obtained drift losses.
depends on the design of the nozzles. For most 14-16. Cooling Tower Rating and Selection.
nozzle designs, a water pressure drop of 7 to Table R.-16 contains rating data for the spray-
101b per square inch will produce a suitable type, natural draft cooling tower illustrated in
spray pattern. Fig. 14-13 and is a typical cooling tower rating
Some natural draft towers contain decking or table. Notice that the tower ratings are given
filling (usually of redwood) to increase the in tons, based on a heat transfer capacity of 250

Water in Water
distributor

"Air out

Fig. 14-14. Small Induced


draft-cooling tower.
.

262 PRINCIPLES OF REFRIGERATION


Air out

\ \ M f t f t t t f,
I Spray
>Teliminators

Wood fill

Fig. 14-15. Forced draft-


cooling tower.

Air in

Water out

Btu/min/ton. Nominal tower ratings are based Exam pie 4- 4. It is desired to cool 90 gpm
1 1

on a 3 mi per hour wind velocity, and 80° F F to 86° F when the design wet bulb is
from 96°
design wet bulb temperature, and a water flow
78° F. Select the proper tower from Table
R-16.
rate over the tower of 4 gpm per ton. Tower
performance at conditions other than those
listed in the table can be determined by using
Solution
Tower range = 96 - 86 = 10°
the rating correction chart that accompanies the
table. Tower approach = 86 - 78 = 8°

To proper tower from the rating


select the From rating correc-
table, the following data must be known: tion chart, range-ap-
proach factor = 1.1
1 Desired tower capacity in tons (compressor
From wet bulb cor-
capacity)
rection chart, wet bulb
Design wet bulb temperature
2.
= 1.04
factor
Desired leaving water temperature (con-
3.
Nominal gpm = 90 x 1.1 x 1.04
denser entering water temperature or tower
= 103 gpm
approach)
From Table R-16, for 103 gpm nominal,
or select tower, Model #SA-68
1. Desired flow rate over the tower (gpm)
2. Design wet bulb temperature Example 14-15. It is required to cool water
3. Desired entering and leaving water tem- for 30 tons at 5 gpm/ton to a 5° F approach of
peratures (tower cooling range and tower an 80° F wet bulb. Select the proper tower from
approach) Table R-16.

Example 14-13. From Table R-16, select a


Solution
cooling tower to meet the following conditions:
1. Required tower capacity =20 tons Total gpm required
for 30 tons at 5 gpm/
2. Design wet bulb
= ton 30 x 5 = 150 gpm
temperature 78° F
3. Desired leaving water From rating cor-
temperature = 86° F don chart, rating cor-
rection factor for 5
Solution. From Table
R-16, select tower,
gpm/ton and 5° ap-
Model #CSA-66, which has a capacity of 20.7 proach = 1.15
tons at the desired conditions when the flow
rate over the tower is 3 gpm per ton. Hence, for From wet bulb
20-tons capacity, a total of 60 gpm (20 x 3) correction chart, wet
must be circulated over the tower. As shown in bulb correction factor 1.0

the table, the entering water temperature will be Nominal gpm 150 x 1.15 x 1.0
approximately 96° F. 172.6 gpm
CONDENSERS AND COOLING TOWERS 263

From Table R-16, for 172.6 gpm nominal, Tower gpm x tower range x 500
select tower Model #SA-612. = condenser gpm x condenser rise x 500
14-17. Condenser By-Pass. For any given Eliminating the constant,
tower range and approach, the entering and
Tower gpm x tower range
leaving water temperatures will depend only on
the wet bulb temperature of the air. Hence, in
= condenser gpm x condenser rise (14-13)

regions (particularly coastal areas) where the


outdoor wet bulb temperature is relatively high,
Example 14-16. A compressor on a refrig-
erating system has a capacity of 25 tons. The
a closer approach to the wet bulb temperature is design wet bulb temperature is 80° F. The
required in order to maintain a reasonable con- desired condenser water entering temperature is
densing temperature with an economical con- 87° F and the desired temperature rise through
denser size than in areas where the wet-bulb the condenser is 10° F. Select a cooling tower
temperature is lower. It has already been shown from Table R-16 and determine:
that, in general, the greater the quantity of water 1. The total gpm circulated over the tower
circulated over the tower per unit of capacity 2. The temperature of the water entering the
the closer the leaving water temperature will tower
approach the wet bulb temperature. Therefore, 3. The tower cooling range
in regions having a high wet bulb temperature, 4. The temperature of the water leaving the
it is usually desirable to circulate a greater condenser
quantity of water over the tower than can be
5. The gpm circulated through the condenser
6. The gpm circulated through the by-pass
economically circulated through the condenser
because of the excessive pumping head encoun-
Solution. From Table
R-16, tower, Model
tered. This can be accomplished by installing #SA-58 has a capacity of 25 tons at an 80° F
a condenser by-pass line as shown in Fig. 14-8. wet bulb temperature and a 7° approach. This
Through the use of a condenser by-pass, a capacity is based on a water flow rate of 4
certain, predetermined portion of the water gpm/ton and on a cooling range of 7.5°
circulated over the tower is permitted to by-pass (94.5 - 87).

the condenser, thereby reducing the over-all Total gpm over the tower
pumping head. for 25 tons

The advantage of the condenser by-pass is


= 25 tons x 4 gpm/ton
= 100 gpm
that it makes possible the maintenance of
reasonable condensing temperatures with mod- From Table R-16, the
erate condenser and tower sizes without greatly tower entering water tem-
perature
increasing the pumping head. The quantity of
= 94.5° F
water flowing through the by-pass is regulated
Tower range
by the hand valve in the by-pass line. Once the = 94.5 - 87 - 7.5°
hand valve has been adjusted for the proper flow
Water temperature leaving
rate through the by-pass, the handle should be
condenser
removed from the valve so that the valve adjust- = 87 + 10 = 97° F
ment cannot be changed indiscriminately. An Rearranging and applying
excessive amount of water flowing through the Equation 14-13, condenser
by-pass will not only tend to starve the con- gpm
denser and raise the condensing pressure, but it _ Tower gpm x tower range
may also cause the pump motor to become Condenser rise
overloaded, thereby rendering the entire system _ 100 x 7.5
inoperative. The desired flow rate through the 10
by-pass determined by subtracting the flow
is = 75 gpm
rate through the condenser from the flow rate
Gpm circulated through
over the tower. This will be illustrated presently. by-pass
Since the cooling tower capacity must of = Tower gpm — condenser gpm
necessity be equal to the condenser capacity at = 100-75
the design conditions, it follows that: = 25 gpm
364 PRINCIPLES OF REFRIGERATION
of the condenser into fhe air, the source of the
vaporising heat being the condensing refrigerant
in the condenser coil.

The cooling produced is approximately 1000


\^J|/ / Eliminators titu per pound of water evaporated. All the heat

given up by the refrigerant in the condenser


eventually leaves the condenser as either sensible
Spray heat or latent heat (moisture) in the discharge
air. Since both the temperature and the mois-
Refrigerant
vapor in
i ,, I >:
ture content of the air are increased as the air
1 passes through the condenser, the effectiveness
Refrigerant c of the condenser depends, in part, on the wet
liquid Out
3 bulb temperature of the entering air. The lower
Condensing coil
the wet bulb temperature of the entering air the
more effective is the evaporative condenser.
Air in
To facilitate cleaning and scale removal, the
condensing coil is usually made up of bare
M-thL'-ut: rather than fumed tubing. The amount of coil
!;-~.-I Water tank -_-_-_-_-:
•.'iL.'.'J surface used per ton of capacity varies with the

?•.. m Ti
manufacturer and depends to a large extent on
the amount of air and water circulated.
Fig. 14-14. Schematic diagram of evaporative con- Generally, the capacity of the evaporative
denser.
condenser increases as the quantity of air cir-
culated through the condenser increases. As a
practical matter, the maximum quantity of air
14-18. Evaporative Condeneers. An evapo-
rative condenser is essentially a water Conser-
vation deviceand is, in effect, a condenser and
a cooling tower combined into a single unit. A
diagram of a typical evaporative condenser is

shown in Fig. 14-16,


As previously stated, both air and water are
employed The
in the evaporative condenser.
water, pumped from the sump up to the spray
header, sprays down over the refrigerant coils
and returns to the sump. The air is drawn in
From the outside at the bottom of the condenser
by action of the blower and is discharged back
to the outside at the top of the condenser. In
some cases, both pump and biower are driven
by the same motor. In others, separate motors
are used. The eliminators installed in the air
Stream above the spray header arc to prevent
entrained water from being carried over into the
blower. Art alternate arrangement, with the
blower iocated on the entering air side oF the
condenser, is shown in Fig. 14-17.
Although the actual thermodynamic pro- Fig. 14-17. Cutaway view of "Dri-Fan" evaporative
cesses taking place in the evaporative condenser
condenser. Funnel-shaped overflow drain provide]
are somewhat complex., the fundamental process au (ornate bleed-off. (Courtesy/ Refrigeration Engln-
is that of evaporative cooling. Water is evapo- Bering, Inc. A proprietary design of Refrigeration
rated from the spray and from the wetted surface Engineering, Inc.)
CONDENSERS AND COOLING TOWERS 265

which can be circulated through the condenser 6-ton evaporator load (Refrigerant- 12)
islimited by the horsepower requirements of the 20° evaporator temperature
fan and by the maximum air velocity that can be 78° entering wet bulb temperature
105° F condensing temperature
permitted through the eliminators without the
carry over of water particles. Solution. Since the rating table is in terms of
The quantity of water circulated over the evaporator load at 40° F, it is necessary to
condenser should be sufficient to keep the tube correct for other evaporator temperatures by
surface thoroughly wetted in order to obtain using a correction factor from R-17B as follows:
maximum efficiency from the tube surface and to Tons x evaporator correction factor
•= Rating table tons
minimize the rate of scale formation. However,
Therefore, 6 x 1.05 = 6.3 tons.
a water flow rate in excess of the amount
Referring to Table R-17A, the E-135F has a
required for adequate wetting of the tubes will capacity of only 5.6 tons at 78° F entering wet
only increase the power requirements of the bulb and 105° F condensing temperature. It
pump without materially increasing the con- does, however, have the required capacity of 6.3
denser capacity. tons at between 105° F and 110° F condensing
Assuming a condenser load of 15,000 Btu per temperature.
hour per ton, the water lost by evaporation is The compressor ratings should then be
approximately 15 lb (2 gal) per hour per ton checked to see if the compressor originally
selected has the required capacity at between
(15,000/1000). In addition to the water lost by
105° F and 110° F condensing temperature. If
evaporation, a certain amount of water is lost by
not, it will be necessary to select the next larger
drift and by bleed-off. The amount of water
size evaporative condenser or compressor to do
lost by drift and by bleed-off is approximately
the job.
l.S to 2.5 gal per hour per ton, depending upon The next larger size evaporative condenser,
the design of the condenser and the quality of the E-270F, has a capacity of 11.2 tons at the
water used. Hence, total water consumption given conditions; however, the required capa-
for an evaporative condenser is between 3 and 4 city of 6.3 tons will be obtained at a condensing

gal per hour per ton. temperature between 90 and 95° F. The com-
Some evaporative condensers are available pressor selection should then be made for these
conditions.
equipped with desuperheating coils, which are
usually installed in the leaving air stream. The 14-20. Water Regulating Valves. The water
hot gas leaving the compressor passes first flow rate through a water-cooled condenser on a
through the desuperheating coils where its tem- waste water system is automatically controlled
perature is reduced before it enters the con- by a water regulating valve (Fig. 14-18). The
densing coils. The desuperheating coils tend to valve is on the water line at the inlet of
installed
increase the over-all capacity of the condenser the condenser and is actuated by the compressor
and reduce the scaling rate by lowering the discharge (Fig. 14-7). When the compressor
temperature of the wetted tubes. Too, often the is in operation, the valve acts to modulate the
receiver tank is located in the sump of the flow of water through the condenser in response
evaporative condenser in order to increase the to changes in the condensing pressure. An
amount of liquid subcooling. increase in the condensing pressure tends to
14-19. Rating and Selection of Evaporative collapse the bellows further and open the valve
Condensers. Table.R-17 is a typical evapo- wider against the tension of the range spring,
rative condenser rating table. Notice that the thereby increasing the water flow rate through
ratings are based on the temperature difference the condenser. Likewise, as the condensing
between the condensing temperature and the pressure decreases, the valve moves toward the
design wet bulb temperature. The following closed position so that the flow rate through the
sample selection is reprinted directly from the condenser is reduced accordingly. Although
manufacturer's catalog data:* the regulating valve tends to maintain the con-
densing pressure constant within reasonable
Example 14-17. Select an evaporative con- limits, the condensing pressure will usually be
denser for the following conditions:
considerably higher during periods of peak
* McQuay Products. loading than during those of light loading.
Hi PRINCIPLES OF REFRIGERATION

Flf. 14-lfl. Typical threaded-type water regulating valve. Larger sixes are available with, flange connection*.
{a) Cross-sectional view showing principal parts, (b) Exterior view, (Courtesy Penn Contrail, Inc.)-

When the compressor cycles off, the water erant in the condenser can never be lower than
valve remains open and water continues la flow the ambient temperature at the condenser, the
through ihc condenser until the pressure in the shut-off point of the water valve should be set at
condenser is reduced to a certain predetermined a saturation pressure corresponding to the maxi-
minimum, which time the valve closes off
at mum ambient temperature in the summertime
completely and shuts o FT the water flow. When at the condenser location.Too, the shut-off
the compressor cycles on again, ihc water valve pressure of the valve must be high enough so
remains closed until the pressure in the con- that the minimum condensing temperature in
denser builds up to the valve opening pressure, the wintertime is sufficiently high to provide a
at which time the valve opens and permits water pressure differential across the refrigerant con-
to flow Lh rough the condenser. The opening trollarge enough to assure its proper operation.
pressure of the valve is approximately 7 psi The capacity of water regu Sating valves varies
above the shut-oil pressure. with the size of the valve and the pressure drop
The water valve is set for the desired shut-off across the valve orifice. The available pressure
pressure by adjusting the tension of the range drop across the valve orifice is determined by
spring. The minimum operating pressure for subtracting the pressure drop through the con-
the valve, that is, the shut-off pressure, must be denser and water piping from the total pressure
set high enough so that the valve will not remain drop available at the water main.
open and permit water to flow through the con- Water regulating valves are usually selected
denser when the compressor is on the off cycle. from flow charts (Table fl-lS), In order to
Since the saturation temperature of the refrig- select the proper valve from the flow chart, the

CONDENSERS AND COOLING TOWERS 267

following data must be known: (1) the desired pressure is 40 psig and manufacturer's table

water quantity in gpm; (2) the maximum gives drop through condenser and accompany-
ambient temperature in the summertime; (3) ing piping and valves as 15 psi. Drop through
installed piping approximately 4 psi. Select
the desired condensing temperature; and (4)
proper size of water regulating valve from Table
the available water pressure drop across the
R-18.
valve.
The following selection procedure and sample Solution
selection are reprinted directly from the litera- Draw a line through 27 gpm see dotted
1. —
ture of the manufacturer:* line,upper half of Flow Chart (Table R-18).
2. Closing point of valve is pressure of R-12
1. Draw horizontal line across upper half of corresponding to 86° F ambient = 93 psig.
Flow Chart (Table R-18) through the required 3. Opening point of valve is 93 + 7 = 100

flow rate. psig.

2. Determine refrigerant condensing pressure 4. Condensing pressure rise = 125 — 100 =


rise above valve opening point.
25 psi.
5. Draw line through 25 psi—see dotted line,
a. Valve closing point (to assure closure
lower half of Flow Chart.
under all conditions) must be the refrigerant
6. Available water pressure drop through
condensing pressure equivalent to the highest
valve = 40 - 19 = 21 psi.
ambient air temperature expected at time of 7. Interpolate just over the 20 psi curve
maximum load. Read this in psig from "Satu- circle on lower half of Flow Chart.
rated Vapor Table" for refrigerant selected. 8. Draw vertical line upward from this point
b. Read from the same table the operating —
to flow line circle on Flow Chart marks this
condensing pressure corresponding to selected intersection.

condensing temperature. 9. This intersection falls between curves for


c. Valve opening point will be about 7 psi 1 in. and \\ in. valves. The 1J in. valve is re-

quired.
above closing point.
d. Subtract opening pressure from operat-
14-21. Condenser Controls. For reasons of
ing pressure. This gives the condensing
economy, the condensing medium is circulated
pressure rise.
through the condenser only when the com-
3. Draw horizontal line across lower half of
pressor is operating. Hence, common practice
Flow Chart through this value.
is to cycle the condenser fan and/or pump on
4. Determine the water pressure drop through
and off with the compressor. This is usually
the valve — this is the pressure actually available
accomplished by electrically interlocking the
to force the water through the valve.
fan and/or pump circuit with the compressor
Determine the minimum water pressure
a.
driver circuit. Method of interlocking electrical
available from city mains or other source.
circuits are discussed in Chapter 21.
b. From condensing unit manufacturer's
Whereas high pressure controls are always
tables read pressure drop through condenser
desirable as safety devices on any type of system,
corresponding to required flow.
they are absolutely essential on all equipment
c. Add to this estimated or calculated drop
employing water as the condensing medium in
through piping, between water valve and
etc.,
order to protect the equipment against damage
condenser, and from condenser to drain (or
from high condensing pressures and tempera-
sump of cooling tower).
tures in the event that the water supply becomes
Subtract total condenser and piping drop
d.
restricted or is shut-off completely. The high
from available water pressure. This is the pressure control has already been discussed in
available pressure drop through the valve.
Section 13-13.
Example. 14-18. The required flow for an a refrigerating system is to function pro-
If

R-12 system is found to be 27 gpm. Condensing perlyand efficiently, the condensing temperature
pressure is 125 psig and the maximum ambient must be maintained within certain limits. As
temperature estimated at 86° F. City water previously described, high condensing tempera-

Controls, Inc., Goshen, tures cause losses in compressor capacity and


» By permission of Penn
Indiana. efficiency, excessive power consumption, and,
268 PRINCIPLES OF REFRIGERATION
in some cases, overloading of the compressor capacity of the condenser during periods when
driver and/or serious damage to the compressor the ambient temperature low and/or the refrig-
is

itself. erating load is light. Although the methods


An
abnormally low condensing temperature, employed to control the capacity of the con-
on the other hand, will cause an insufficient denser vary somewhat with the type of con-
pressure differential across the refrigerant con- denser used, all involve reducing either the
trol (condensing pressure to vaporizing pres- quantity of condensing medium circulated or
sure), which reduces the capacity of the control the amount of effective condensing surface.
and results in starving of the evaporator and Condenser capacity control devices are usually
general unbalancing of the system. actuated by pressure or temperature controls
As a general rule, low condensing tempera- which respond to condensing pressure or
tures result from eitherone or both of two temperature.
principal causes: (1) low ambient temperatures With regard to air-cooled condensers, the
and (2) light refrigerating loads. Naturally, the condensing temperature is maintained within
problem of low condensing temperatures is more the desired limits by varying the air quantity
acute in the wintertime when the ambient through the condenser or by causing a portion
temperature and the refrigerating load are both of the condenser to become filled with liquid so
apt to be low. as to reduce the amount of effective condensing
To maintain the condensing temperature at a surface.
sufficiently high level, it is necessary to make The air quantity through the condenser is

some provision for reducing or controlling the varied by cycling the fan or blower or by the use

Modulating
control valve
(open on drop
in pressure)

Fig. 14-19. Winterstat control


of air-cooled condensers, (a)

Loop Winterstat may be used


wherever 3 feet of head room
is above the top of the
available
condenser. This type is the
simplest and lowest in cost.
(b) No-loop Winterstat is
employed where head room
Constant is not available above con-
inlet pressure denser. Valves and W
are
throttling valve
supplied as an integral unit and
must be mounted at the level
of the liquid outlet of the
Air-cooled
condenser condenser. (The Winterstat is
a proprietary design of the
Kramer Trenton Company and
is manufactured under the
following patent numbers:
2,564,310; 2,761,287; and
2,869,330.)

From
condenser
outlet

To receiver
(b)
CONDENSERS AND COOLING TOWERS 269

Fig. 14-20. Pressure stabilizer.


(A proprietary design of
Dunham-Bush, Inc.) (Courtesy
Dunham-Bush, Inc.)

of dampers placed in the air stream. Because it iscontrolled by the regulating valve installed
tends to cause large fluctuations in the con- between the condenser and the receiver. This
densing temperature, cycling of the fan cannot valve is and
set at the desired operating pressure,

be recommended as a means of controlling the throttles from the open position to the closed
capacity of air-cooled condensers. Modulating position as the condensing pressure drops. The
dampers installed in the air stream provide up the liquid in the con-
throttling action backs
satisfactory control of the air quantity in many denser, thus reducing the amount of effective
cases, although some difficulty is experienced condensing surface. The subcooled liquid
with dampers when the condenser is exposed to coming from the condenser is forced through
high wind velocities. the heat exchanger portion of the pressure
A more satisfactory method of controlling stabilizer and receives enough heat from the hot

the capacity of air-cooled condensers is to vary gas to satisfactorily establish the balanced
the amount of effective condensing surface by pressure temperature relationship in the receiver.
causing the liquid refrigerant to back up into This assures satisfactory condensing pressure
the lower portion of the condenser whenever the and a solid column of liquid at the refrigerant

condensing pressure drops below the desired control.

minimum. To accomplish this, one design of The pressure stabilizer is designed with a pre-
capacity control employs a modulating valve determined pressure drop to insure against
installed in a by-pass line between the inlet and liquid refrigerant reheating during warm weather

outlet of the condenser (Fig. 14-19). As the operations. During high ambient air tempera-
receiver pressure falls, the modulating valve tures,where the condensing temperature is above
opens and allows high-pressure vapor from the the setting of the regulating valve, the liquid
compressor discharge to flow through the by- flows through the valve, which is fully open, and
pass line, thereby restricting the flow of liquid thereby by-passes the heat exchanger section
refrigerant from the condenser and causing the (Fig. 14-21a). In Fig. 14-216, as the ambient

liquid to back up into the lower portion of the temperature drops to 50° F the condensing tem-
unit. The amount of discharge vapor by-passed, perature drops below the setting of the regulat-
and therefore the amount of liquid refrigerant ing valve. The valve then modulates toward
retained in the lower portion of the condenser, the closed position, and this action limits the

is automatically controlled by the modulating flow of liquid through the regulating valve.
valve and depends upon the receiver tank Consequently, the liquid backs up in the con-
pressure. denser until the condensing surface is reduced
Another device used to restrict the amount of approximately 60%. The liquid which is forced
effective condensing surface is called a "pressure to pass through the heat exchanger section is
stabilizer" (Fig. 14-20). The following descrip- then heated up to the saturation temperature.
tion of the operation of the pressure stabilizer When the ambient temperature drops to 0° F
is reprinted directly from the manufacturer's (Fig. 14-2 lc), the regulating valve throttles to

engineering data.* hold 120 psi in the condenser. The liquid logs
The pressure stabilizer is a heat transfer in the condenser so that approximately 10% of

surface which transfers the heat from the hot gas the surface is utilized to condense the hot gas.
discharge of the compressor to the subcooled With regard to evaporative condensers, capa-
liquid leaving the condenser. This heat exchange city control is best obtained through regulation
of the air quantity through the condenser, which
* Courtesy Dunham-Bush, Inc. can be accomplished either by cycling the blower
270 PRINCIPLES OF REFRIGERATION

Cycling of the pump as a means of controlling


90° Amb., R-12
110* Cond. temp. the capacity of an evaporative condenser cannot
' '" vr
r' i
-'
be recommended. Each time the pump cycles
I
*' -;' 'v
136 g^ ' ' ) off a thin film of scale is formed on the con-

denser tubes. Consequently, frequent cycling of


PTTX. ........ ....'.j
the condenser pump greatly increases the scaling
rate, which reduces the
efficiency of the con-
denser and greatly increases maintenance costs.
With reference to water-cooled condensers,
recall that for a given load and condensing
surface, the condensing temperature varies with
the quantity and temperature of the water
entering the condenser. Where waste water is
used, the modulating action
of the water-
50* Amb., R-12 regulating valve controls the water flow rate
102* Cond. temp.
through the condenser and maintains the con-
;
[
•—
l..."..'.'..-"l^J,»l densing temperature above the desired minimum
,II'I.H'..I| ^T"f J
so that low condensing temperatures are not
usually a problem with waste water systems. On
the other hand, since the flow rate of the water
through the condenser on a recirculating water
system is maintained constant, the condensing

temperature decreases as the temperature of the


water leaving the tower decreases. Therefore,
when the ambient air temperature is low, the
condensing temperature will also be low unless
some means is provided for restricting the flow
rate through the condenser or for increasing the
0* Amb., R-12
102* Cond. temp.
temperature of the water leaving the tower.
-
ffi'A'.",;.',..',-.'.'M One method of controlling the condensing
temperature in a recirculating water system is to
installa water-regulating valve in the water line
at the inlet to the condenser. The modulating
action of the water valve will restrict the water
flow rate through the condenser in response to a
drop in the condensing pressure. When a water-
regulating valveis used in a recirculating water

system, the pressure drop through the valve


must be taken into account in computing the
pumping head.*
total
Where mechanical draft cooling towers are
used, the condensing temperature can be main-
tained at the desired level through regulation of
Fig. 14-21. Air-cooled condenser control employing the tower leaving water temperature. As in the
pressure stabilizer. (Courtesy Dunham-Bush, Inc.) case of the evaporative condenser, this can be
accomplished by cycling the tower fan or by
installing dampers in the air stream.
or by installing dampers in the air stream. Of
* Except in those cases where they have a specific
the two methods, the latter is usually the most
function, water-regulating valves should never be
satisfactory, especially where modulating dam-
used in recirculating water systems, since they tend
pers are used and the air quantity can be varied to restrict the water flow and increase the pumping
through a wide range. head unnecessarily.
CONDENSERS AND COOLING TOWERS 271

14-22. Winter Operation. When the com-


pressor and/or condenser are so located that they
are exposed to low ambient temperatures, the
pressure in these parts may fall considerably
4 fa
below that in the evaporator during the com- ^Modulating
dampers
pressor off-cycle. In such cases, the liquid
refrigerant, which otherwise would remain in

the evaporator, very often tends to migrate to Water bleeds

the area of lower pressure in the compressor and


condenser. With no liquid refrigerant in the
evaporator, an increase in evaporator tempera-
E Drain-
a
ture not reflected by a corresponding increase
is

in the evaporator pressure, and, where the sys- Float valve


tem is controlled by a low pressure motor
control, the rise in evaporator pressure may not
be sufficient to actuate the control and cycle the
system on in response to an increase in the
Overflow rm\•k
1 Sump tank
Pump
evaporator temperature.
Fig. 14-23. Evaporative condenser equipped with
Corrective measures are several. One is to
modulating dampers for capacity control. Protected
installa thermostatic motor control in series
auxiliarypump is designed to prevent freezing during
with the low pressure control. The thermostat
winter operation. (Courtesy Refrigeration Engineer-
is adjusted to cycle the system on and off,
ing Inc.)
whereas the low pressure control serves only as a
safety device. Another, and usually more prac-
tical, solution is to isolate the condenser during
denser during the off-cycle. The (R) valve,

the off-cycle. One method of isolating the con-


which closes on drop of pressure at the valve
inlet, closes the compressor stops, pre-
when
denser during the off-cycle is illustrated in Fig.
14-22. The check valve (Q in the condenser
venting the flow of refrigerant from the evapo-
from boiling rator, through the compressor valves and
liquid line prevents the refrigerant
and backflowing to the con- discharge line, into the condenser. With the
off in the receiver
condenser isolated, the evaporator pressure can
build up and start the compressor regardless of
the ambient temperature at the condenser.
Air-cooled Modulating valve
Another and rather obvious problem con-
condenser (open on rise of cerning the operation of evaporative condensers
inlet pressure)
and cooling towers in the wintertime is the
ggB%g
danger of freezing when the equipment is
exposed to freezing temperatures. In general,
Check valve
the measures employed to prevent freezing are
To liquid_ similar to those used to prevent low condensing
receiver temperatures, that is, controlling the air quantity
through the tower by the use of dampers or by
Modulating by-pass From _y cycling the fan. In addition, an auxiliary sump
valve-open of drop discharge
must be installed in a warm
and thelocation
in outlet pressure
piping arranged so that the water drains by
Fig. 14-22. Sure-start WintersUt provides normal gravity into the auxiliary sump and does not
head and receiver pressures when the compressor remain in the tower or condenser sump (Figs.
starts by allowing the compressor to impose its full 14-23 and 14-24).
discharge pressure on the liquid through the open 14-23. Condenser and Tower Maintenance.
(W) valve. When the receiver pressure is up to condensers require
As a general rule, air-cooled
normal, the (R) valve opens and allows the discharge
littlemaintenance other than regular lubrication
gas to flow to the condenser. (Courtesy Kramer
Trenton Company.) of the fan and motor bearings. However, the
272 PRINCIPLES OF REFRIGERATION

tubes by applying an approved inhibited acid


j
^ ** ^^^^^^™
compound, many of which are available in
either liquid or powder form. After the tower

Tower static or condenser sump has been drained, cleaned,


head and filled with fresh water, the cleaning com-
pound can be added directly to the sump water.
The pump is then started and the cleaner is
I circulated through the system until the system is
clean, atwhich time the sump is again drained,
flushed, and filled with clean water before the
Additional
static head
system is placed in normal operation.
Indoor It should be pointed out that descaling com-
tank
pounds have an acid base and should not be
allowed to contact grass, shrubs, or painted
surfaces. Therefore, it is usually advisable to
remove the cooling tower spray nozzles, if any,
in order to minimize the danger of damaging
shrubs or painted surfaces with drift from the
tower.
When rapid descaling of the condenser tubes
is required, an inhibited solution (18%) of
muriatic acid may be used. However, muriatic
on the condenser tubes.
acid should be used only
The system pump should not be used to circu-
Fig. 14-24. Protected indoor tank. late the acid. A small pump having an acid
resistant impeller (brass or nylon) may be used
for this purpose (see Fig. 14-25). After the
fan blades and condensing surface should be condenser is clean, it should be flushed with
inspected occasionally for the accumulation of clean water or with an acid neutralizer as
dust and other foreign materials. These parts recommended by the manufacturer.
should be kept clean in order to obtain high Corrosion is usually greatest in areas near
efficiency from the condenser. salt water or in industrial areas where relative
Any type of condenser employing water is
subject to scaling of the condenser tubes, corro-
Condenser -

sion, and the growth of algae and bacterial


slime on all wetted surfaces. The latter is con-
by frequent cleaning of the infected parts
trolled
and by the use of various algaecides which are
available commercially.
As previously stated, the scaling rate depends
primarily upon the condensing temperature and
the quality of water used. The scaling rate will
be relatively low where the condenser leaving
water temperature is below 100° F. Too, the
importance of providing for the recommended
amount of bleed-off cannot be overemphasized
with regard to keeping the scaling rate at a
minimum. In addition, a number of chemical
companies have products which when added to
the sump water considerably reduce the scaling
rate.
Scale can be removed from the condenser Fig. 14-25. Apparatus for descaling condenser.
:

CONDENSERS AND COOLING TOWERS 273

large concentrations of sulfur and other indus- 9. From Table R-12, select an air-cooled con-
trialfumes are found in the atmosphere. Corro- denser for a compressor having a capacity of
sion damage is minimized by regular cleaning 42,000 Btu/hr if the design suction and discharge
and painting of the affected parts and by temperatures are 40° F and 130° F, respectively,
application of protective coatings of various and the outdoor design dry bulb temperature
for the region is 95° F.
types.

PROBLEMS 10. Select a shell-and-tube water-cooled con-


denser for an R-12 system to meet the following
1. An R-12 system is operating at an evaporator conditions
temperature of 0° F and a condensing tempera-
Refrigeration load and eva-
ture of 100° F. From Chart 14-1, determine the
porator 60 tons
heat load on the condenser in Btu per minute
Evaporator temperature 40° F
per ton of refrigeration. Ans. 257 Btu/min/ton
Condensing temperature 1 10° F
2. AnR-22 system operating with a 40° F Water quantity 2.5 gpm/ton
evaporator and a 110° F condenser has an Untreated cooling tower water enters con-
evaporator load of 10 tons. Determine the heat denser at 85° F.
load on the condenser in Btu/hr.
11. Rework Problem 10 using a condensing
Ans. 141,000 Btu/hr
temperature of 120° F.
3. The heat rejected to a water-cooled con-
denser is120,000 Btu/hr. How many square 12. A
cooling tower and a water-cooled con-
feet of effective tube surface must this condenser denser (with by-pass) are operating with a
have if the U factor of the condenser is 100 condenser load of 240,000 Btu/hr. Forty-eight
Btu/hr/sq ft/° F and the METD is 5° F at the gpm are circulated through the condenser and
desired gpm? Ans. 240 sq ft 32 gpm are by-passed. The ambient wet bulb
temperature is 78° F and the tower approach is
4. The heat load on the evaporator of an air
7° F. Determine:
conditioning system is 60,000 Btu/hr. If the
(a) The temperature of the water entering the
coefficient of performance of the system is 4 : 1,
what is the heat load on the condenser in Btu/hr ?
condenser. Ans. 85° F
(6) The temperature of the water leaving the
Ans. 75,000 Btu/hr
condenser. Ans. 95° F
5. An R-12 waste water system operating at a (c) The temperature of the water entering the
40° F suction temperature and a 105° F con- cooling tower. Ans. 91° F
densing temperature has an evaporator load of (</) The tower range. Ans. 6° F
5 tons. If the condenser is selected for a 12° F
water temperature rise, how many gpm must be 13. Acompressor on a Refrigerant- 12 system
circulated through the condenser? has a capacity of 50 tons. The design wet bulb
Ans. 11.5 gpm temperature is 78° F. The desired condenser
water entering temperature is 85° F and the
6. Seventy-two gallons of water per minute are desired temperature rise through the condenser
circulated through a water-cooled condenser.
is 12° F. Select a cooling tower from Table R-15
If the temperature rise of the water in the con-
and determine:
denser is 14° F, what is the heat load on the (a) The total gpm circulated over the tower
condenser? Ans. 504,000 Btu/hr (b) The temperature of the water entering the
7. AnR-12 air conditioning system operating tower
with an evaporator temperature of 40° F and a (c) The temperature of the water leaving the
condensing temperature of 120° F has an condenser
evaporator load of 60,000 Btu/hr. 4500 cfm of (d) The tower range
air are circulated over the condenser. If the (e) The gpm circulated through the condenser
temperature of the air entering the condenser is (/) The gpm by-passed
90° F, compute: (a) the leaving air temperature
14. Select an evaporative condenser for the
and (b) the METD.
following conditions:
Ans. (a) 104.6° F (b) 21.89° F Refrigerant- 12 system
8. If the air-cooled condenser in Problem 7 has —
Evaporator load 10 tons
a free face area of 5.5 sq ft, what is the velocity Evaporator temperature 40° F—
of the air through the condenser? Wet bulb temperature of entering air 78° F —
Ans. 818 fpm —
Condensing temperature 105° F
gravitational force or pressure is exerted in a
downward direction only. However, because of
the loose molecular structure of fluids, the gravi-
tation force or pressure exerted at any point in a
body of fluid acts equally in all directions up, —
down, and sideways, and always at right angles
to any containing surfaces. When no force
other than the force of gravity is acting on the
15 fluid, the pressure at any depth in a body of fluid

is proportional to the weight of fluid above that


depth. When an external force in addition to

Fluid Flow, the force of gravity


pressure at any depth in the fluid
is applied to the liquid, the
is proportional
to the weight of the fluid above that depth, plus
Centrifugal Liquid the pressure caused by the external force.
For example, assume that a flat-bottomed
Pumps, Water and container 1 sq ft in cross section and 10 ft high
is filled to the top with water at a temperature of
Brine Piping 60° F (Fig. 15-1). Since water at 60° F has a
density of 62.4 lb per cubic foot, if the pressure
of the atmosphere on the surface of the water is

neglected, the total force acting on the bottom


of the tank due to the weight of the water alone
is 624 lb (10 x 62.4). Since the base area of the
15-1. Fluid Pressure. The total pressure
tank is 1 sq ft, the pressure exerted on the
exerted by any fluid is the sum of the static and
bottom of the tank is 624 psf or 4.33 psi
velocity pressures of the fluid, viz:
(624/144). Since this pressure acts equally in all
Pt =P* +Pv (15-1) directions, on the sides of the tank
it is exerted
where p t = the total pressure at the base as well as on the bottom of the tank.
p, = the static pressure Assume now that level A in the water column
pv = the velocity pressure is exactly 1 ft below the surface of the water.

All flowing fluids possess kinetic energy and The volume and weight of water above this level
therefore exert a force or pressure in the direc- are 1 cu ft and 62.4 lb, respectively. Since this
tion of flow. The pressure exerted by a fluid weight of water is also evenly distributed over an
which is the direct result of fluid motion or velo- area of 1 sq ft, the fluid pressure acting in all

city is called the velocity pressure of the fluid. directions from any point at level A is 62.4 psf
Any pressure exerted by a fluid which is not the or 0.433 psi. Similarly, the volume and weight
direct result of fluid motion or velocity, regard- of water above level B, which is located 5 ft

less of the force causing the pressure, is called below the surface of the water, are 5 cu ft and
the static pressure of the fluid. For fluids at rest 3121b (5 x 62.4), respectively, and the fluid
(static), the velocity pressure is equal to zero pressure at this level is 312 fsf or 2.165 psi.

and the total pressure is equal to the static If the force exerted on the top of the water by
pressure. Whereas velocity pressure acts only the pressure of the atmosphere is taken into
in the direction of flow, static pressure acts account, the pressure of the water at any level
equally in all directions. This is easily demon- in the tank will be increased by an amount equal
strated through the use of an example employing to the pressure of the atmosphere. Assuming
a gravitational column. normal sea level pressure, the fluid pressures at
It was shown in Chapter 1 that the action of levels A and B are 15.129 psi (0.433 + 14.696)
gravity on any body causes the body to exert and 16.861 (2.165 + 14.696), respectively, while
a force which is commonly referred to as the the pressure at the base of the tank is 19.026 psi
weight of the body. For a solid material, (4.33 + 14.696). However, it should be recog-
because of the rigid molecular structure, the nized that since the pressure of the atmosphere
274

FLUID FLOW, CENTRIFUGAL LIQUID PUMPS, WATER AND BRINE PIPING 275

is on the outside of the tank the


exerted also
P=
pressure tending to burst the tank is still only
that resulting from the gravitational effect on
the water alone.
For any noncompressible fluid (liquid), the
pressure exerted by the fluid at any level in a
fluid column is directly proportional to the
depth of the fluid at that level.* Hence, the
pressure of a liquid at any level in a column of
liquid can be determined by multiplying the
depth at that level times the density of the fluid,
viz:

Pressure (psf) = depth (ft) x density (lb/cu ft)

(15-2)

depth (ft) x density (lb/cu ft)


Pressure (psi)
144
(15-3)

15-2. Head-Pressure Relationship. The


vertical distance between any two levels in a
Fig. 15-1. Illustrating head-pressure relationship.
column of "head" of the
liquid is called the
liquid at the lower level with respect to the upper
level. For example, with respect to level B in

Fig. 1 5- 1 , the head of the water at the base of the to 1 For mercury, the head-pressure ratio
psi.

column is With respect to the top of the


5 ft. is 2.04 in. to1 psi. This means that a pressure

column, the head of the water at the base of of 1 psi is equivalent, to head of 2.31 ft of water
the column is 10 ft. Similarly, with respect to the column or 2.04 in. of mercury column. Con-
top, the water heads at levels A and £ are 1 ft versely, a 1 ft column of water (1 ft water head)

and 5 ft, respectively. is equivalent to 0.433 psi, whereas a 1 ft column

Since the depth of the liquid at any level in a of mercury (1 ft mercury head) is equivalent to
liquid column is equal to the head of the liquid 24.48 psi.
at that level with respect to the top
of the column, With respect to the head-pressure relationship,
the head can be substituted for depth in the following general statements can be made:
Equation 15-3 and the following relationship 1. For any liquid of given and uniform
between head and pressure is established: density, the pressure exerted by the liquid is
directly proportional to the head of the liquid.
Head (ft) x density (3/cu ft)
Pressure (psi) = 2. At any given head, the pressure exerted by
144 any liquid is directly proportional to the density
(15-4)
of the liquid. Liquids having different densities
„T
Head
,. ,, .
(ft) = — .*, /,
Density (lb/cu
-——
Pressure (psi) x 144
ft)
(15-5)
K
'
will exert different pressures at the same head.
15-3. Static and Velocity Heads. The total
It is evident from the foregoing that there is a head of any sum of the static and
fluid is the

definite and fixed relationship between the head velocity heads of the fluid, viz:
and the pressure of any liquid, the head-pressure ht =h + h v (15-6)
s
ratio for any given liquid being dependent upon
where, h t = the total head in feet
the density of the liquid. For example, in the
h, = the static head in feet
case of water, the head-pressure ratio is 2.31 ft
hv = the velocity head in feet
* This not true of a compressible fluid because
is The head of any liquid is expressed as
static
the density of a compressible fluid varies with the the height in feet (or inches) of a gravitational
depth. column of that liquid which would be required
276 PRINCIPLES OF REFRIGERATION
To convert velocity head to velocity,

V= y/2g x h v (15-10)

n
Static
pr ure To convert velocity pressure to velocity,
T |-t
y f\ fa * /». * 144
(15-11)
V P

15-4. Head-Energy Relationship. Although


the term "head" itself is entirely independent of
weight or density, should be recognized that
it

the head of any fluid is numerically equal to the


energy per pound of fluid. For this reason, head
is often used to express energy per pound of
Fig. 15-2. Illustrating relationship between the
fluid.
static, velocity, and total pressures of a fluid flowing
The basic relationship of head to energy or
in a circuit.
work is shown in the following equation :

to produce a base pressure equal to the static Energy or work (ft-lb) = mass (lb) x head (ft)

the head in feet (15-12)


pressure of the liquid. That is,

of liquid column equivalent to the static pressure Since velocity head (h v) is equal to V*l7g
of the liquid is called the static head of the (Equation 15-7), it follows that the total velocity
liquid. Likewise, the head in feet of liquid (kinetic) energy (E„) of any given mass (Af) of
column equivalent to the velocity pressure of a fluid flowing at any given velocity (V) can be
liquid is called the velocity head of the liquid. expressed as
The fundamental relationship between velo-
city and velocity head is established by Galileo's
Ek =Mx —
law, which states in effect that all falling bodies,
regardless of weight, accelerate at equal rates The fact that the preceding equation is iden-

and that the final velocity of any falling body, tical to Equation 1-7 indicates that the velocity

neglecting friction, depends only upon the height head of a an expression of the kinetic
fluid is
energy per pound of fluid. Similarly, it can be
from which the body falls. Hence, the height in
feet from which a body must fall in order to
shown also that the static head of a fluid is an
attain a given velocity is the velocity head corre- expression of the potential energy per pound of
fluid.
sponding to that velocity. The velocity head
corresponding to any given velocity can be In any fluid column of uniform and constant
determined by applying the following equation: density, the potential energy per pound of fluid
is the same at all levels in the column. However,
-*" the potential energy at various levels is differently
h (15-7)
divided between the energy of position and the
where, h v = the velocity head in feet energy of pressure (head) depending upon the
V = the velocity in feet per second (fps) elevation. For example, in Fig. 15-1, 1 lb of
g = the acceleration due to gravity (32.2 water at the uppermost level in the tank has a
ft/sec/sec) potential energy of position with relation to the
By combining and/or rearranging Equations base of 10 ft-lb (1 lb x 10 ft) in accordance with
15-7 and 15-4, the following relationships are Equation 1-8. Since the head at this level is

established: zero, the potential energy of pressure (head) is


To convert velocity head to velocity pressure, also zero. On the other hand, 1 lb of water at

JLxi the base of the tank has no potential energy of

* - "14T (15 " 8) positions, but has pressure or head energy of


10 ft-lb (1 lb x 10 ft), according to Equation
To convert velocity to velocity pressure,
15-12. Likewise, 1 lb of water at a level midway
V* x P in the water column also has potential energy in
Pv = (15-9)
Ig x 144 the amount 10 ft-lb, the energy being evenly
FLUID FLOW, CENTRIFUGAL LIQUID PUMPS, WATER AND BRINE PIPING 277

divided between the energy of position and the For any given flow rate (quantity of flow), the
energy of pressure. velocity of the fluid flowing in a conduit varies
15-5. Static Head-Velocity Head Relation- inversely with the cross-sectional area of the
ship in Flowing Fluids. The fact, that the conduit. This relationship is expressed by the
static pressure of a fluid is exerted equally in all basic equation
directions, whereas the velocity pressure of the *-\ (15-13)
fluid is exerted only in the direction of flow,
makes it relatively simple to measure the static where V= the velocity in feet per second
and velocity pressures (or heads) of a fluid Q= the flow rate in cubic feet per second
flowing in a conduit. This is illustrated in Fig. A = the cross-sectional area of the con-
15-2. Notice that tube A is so connected to the duit in square feet
conduit that the opening of the tube is exactly Note. When Q is in cubic feet per minute,
perpendicular to the line of flow. Since only the V will be in feet per minute.
static pressure of the fluid will act in this direc- In accordance with Equation 15-13, the fluid
tion, the height of the fluid column in tube A is velocity (and velocity head) in section B of the
a measure of the static pressure or static head of conduit in Fig. 15-3 is greater than that in
the fluid in the conduit. On the other hand, sections A and C, since the cross-sectional area
tube B is so arranged in the conduit that the of section B is less than that of sections A and C.
opening of the tube is directly in the line of flow. Assuming that the total head of the fluid is the
Since both the static pressure and the velocity same at all points in the conduit, it follows then
pressure of the flowing fluid act on the opening that the static head-velocity head ratio in section
of tube B, the height of the liquid column in tube B is different from that in sections A and C. As
B is a measure of the total pressure or total head the fluid flows through the reducer between
of the fluid. Since the total pressure or head of a sections A and B, static head is converted to
fluid is the sum of the static and velocity velocity head (pressure is converted to velocity).
pressures or heads, it follows that the difference Conversely, as the fluid flows through the
in the heights of the two fluid columns is a increaser between sections Band C, velocity
measure of the velocity pressure or velocity head head is converted back into static head (velocity
of the fluid in the conduit. is converted to pressure).
If losses because of friction are neglected, the In view of the head-energy relationship, it is

total pressure or head of a flowing fluid will be evident that the conversion of static head to
the same at all points along the conduit. How- velocityhead is in fact a conversion of potential
ever, the total head may be differently divided energy (pressure) into kinetic energy (velocity).
between static head and velocity head at the Likewise, the conversion of velocity head to
several points, depending upon the velocity of static head represents a conversion of kinetic
the fluid at these points. energy (velocity) to potential energy (pressure).

Fig. 15-3. Illustrating changes in static-velocity pressure ratio resulting from changes in conduit area.
278 PRINCIPLES OF REFRIGERATION
15-6. Friction Head. It has already been result is called the "total equivalent length."
established that a fluid flowing in a conduit will This valueis then applied in Equation 15-14 to

suffer losses in energy (converted into heat) as a determine the total friction loss through the
result of thework of overcoming friction. These piping.
energy losses are frequently expressed in terms
Example 15-1. A
water piping system con-
of pressure drop or head loss. The pressure drop
sists of 128 ft of 2 in. straight pipe, 6 standard
in psi or the head loss in feet experienced by a elbows, and 2 gate valves (full open). Using
fluid flowing between any two points in a con- fairly rough pipe, if the flow rate thrdugh the
duit is known as the friction head or friction loss system is 40 gpm, determine:
between these two points. (a) The total equivalent length of straight pipe
The amount of pressure drop or head loss (b) The total friction loss through the piping

suffered by a fluid due to friction in flowing in psi and in feet of water column.

through a conduit varies with a number of Solution. From Table 15-1, the equivalent
factors: (1) the viscosity and specific gravity of lengths of 2 in. standard elbows and 2 in. gate
the fluid, (2) the velocity of the fluid, (3) the valves (full open) are 5 ft and 1 .2 ft, respectively.
hydraulic radius (ratio of perimeter to diameter) From Chart 15-2, for a flow rate of 40 gpm, the
of the conduit, (4) the roughness of the internal friction loss per hundred feet of 2 in. nominal
surface of the conduit, and (5) the length of the pipe is 3 psi. From Table 1-1, a pressure of
1 psi is equivalent to 2.31 ft of water column.
conduit.
Obviously, the mathematical evaluation of all (a) Total equivalent length
these factors is too laborious for most practical
Straight pipe = 128.0 ft

purposes. As a general rule, the friction loss in


Six 2 in. elbows @5 ft = 30.0

piping is determined from charts and tables.


Six 2 in. gate valves @
1.2 ft 2.4
The pressure (friction) loss in psi per hundred
160.4 ft
feet of straight pipe is given in Charts 15-1 and
(b) Applying Equation
15-2 for various flow rates in various sizes of
15-14, the total friction
pipe. Chart 15-1 applies to smooth copper tube, loss through the piping
XJ
loo
whereas Chart 15-2 applies to fairly rough pipe.
= 4.8 psi
Since the pressure loss for a given pipe size and
Converting to ft H4 = 4.8 x 2.31
flow rate is proportional to the length of the = 11.09 ft H2
pipe, the pressure loss through any given length
of straight pipe is determined by the following Although the pressure loss determined from
equation: Charts 15-1 and 15-2 apply only to water, the
charts can be used for other fluids by multiplying
Total pressure loss (ft)
the water pressure loss obtained from these
Total length of pipe (ft)
" charts by the correction factors listed in Table
100 15-2.
x pressure loss/100 ft (psi) (15-14) 15-7. Centrifugal Pumps. Liquid pumps used
Pipe fittings, such as elbows, tees, valves, etc., in the refrigerating industry to circulate chilled
offer a greater resistance to flow than does water or brine, and the condenser water are
straight pipe and therefore must be taken into usually of the centrifugal type.
account in determining the total friction loss A centrifugal pump consists mainly of a
through the piping. For convenience, this is rotating vane-type impeller that is enclosed
done by considering the fittings as having a in a stationary casing. The liquid being pumped
resistance equal to a certain length of straight is drawn in through the "eye" of the impeller
pipe called the "equivalent length." Table 15-1 and is thrown to the outer edge or periphery of
lists the equivalent length of straight pipe for the impeller by centrifugal force. Considerable
various types of fittings and valves. Notice that velocity and pressure are imparted to the liquid
the equivalent length varies with the size of the in the process. The liquid leaving the periphery
Pipe- of the impeller is collected in die casing and
When the equivalent length of the fittings is directed through the discharge opening (Fig.
added to the actual length of straight pipe, the 15-4).
FLUID FLOW, CENTRIFUGAL LIQUID PUMPS, WATER AND BRINE PIPING 279

Frequently, the impeller of the pump is


mounted directly on the shaft of the pump-
driving motor so that the pump and motor are
an integral unit (Fig. 1 5-5). In other cases, the
pump and motor are separate units and are
connected together by a. flexible coupling.
In general, the capacity of a centrifugal pump
depends on the design and size of the pump and
on the speed of the motor. For a pump of
specific size, design, and speed, the volume of
liquid handled varies with the pumping head

Fig. 15-5, Typical centrifugal pump and motor


asig m bl y . (Gourteiy Bell & Getsttt Co m pany ,)

pump ratings are available in table form, more


frequently they are taken from head-capacity
curves (see Chart R-19). In either case, before
the proper pump can be selected from the manu-
facturer's ratings, it is necessary to know the
required gpm and the total pumping head
against which the pump must operate.
15-8. Total Pumping Head. The total pump-
ing head is the sum of the static head and the
friction head.
The static head is the vertical distance be-
1
tween the "free liquid level' and the highest
point to which the liquid must be lifted by the

pump. For the condenser-water circulating sys-


tem in Fig. 15-7, the static bead, measured in
feet ofwater column. Is the vertical distance in
feet between the free water level in the tower
basin and the tower spray header. Because of
the water head in the tower basin, the water in
the discharge pipe will stand to the level of the
Flg> 15-4. Fluid flow through centrifugal pump.
(Courtejy Ingereoll-iUfld Company.)

against which the pump must wort. A charac-


teristic head-capacity curve for a typical centri-
fugal pump is shown in Fig. 5-6. Notice that
1

the pumping head is maximum when the valve


on the discharge of the pump is closed, at which
time the pump delivery is zero. As the valve is
opened, the pumping head decreases and the
deliver rate increases.
Centrifugal pumps are rated in gpm of Gpm
delivery pumping heads, that is,
at various
Fig. 15-4. Centrifugal pump dalivery i apacity in-
centrifugal pumps are rated to deliver a certain erewei h
the pumping h«ad dttrn*«. {CourtMy
gpm against a certain pumping head. Although IngsrsoH-Rind Company.)
280 PRINCIPLES OF REFRIGERATION
and cooling towers, are found in the manu-
facturers' rating tables.
When more than one condenser (or chiller,
etc.) isused in the system, the condensers are
Static piped in parallel and only the condenser with the
head
largest pressure drop is considered in computing
thepumping head.
The pressure loss through the cooling tower,
as given by the tower manufacturer, is the total
head and includes both the tower static and
friction heads. Therefore, the static head of the
tower should not be considered separately in
determining the total pumping head. When the
Total length of tower static head is the only static head in the
straight pipe system, the static head should be disregarded
80 ft
entirely. However, in the event that an auxiliary
indoor storage tank employed, as shown in
is

Fig. 14-24, the vertical distance between the


level of the water in the tank and the normal

Gate valve >


water level in the tower basin must be treated
as a separate static head.
Globe
Since pump manufacturers always express the
/^valve
Condenser pumping head in "feet of water column," it is
necessary to compute the pumping head in these
units. When the pressure loss through the
several system components is given in psi or in
other units of pressure, it must be converted to
Pump-^ feet of water column before it can be used in
computing the pumping head. The required
Fig. 15-7. Condenser-water circulating system. conversion factors are found in Table 1-1.

*
water in the tower basin of its own accord.
1

Therefore, the distance the water is actually


lifted by the pump is only the distance from the
water level in the tower basin up to the spray
header. Contrast this with the pumping system
shown in Fig. 15-8. Static
head
When the piping system is a closed circuit, as
in Fig. 1 no static head on the pump,
5-9, there is
since the fluid on one side of the piping system
will exactly balance the fluid on the other side.
* [|Purnp|
A typical piping system curve in which gpm |

is plotted against total head is shown in Fig.


15-10. Notice that the total head increases as
the flow rate through the system increases and
that the increase in the total head results
entirely from an increase in the friction head,
the static head being constant.
Lower
15-9. Determining the Total Pumping tank
Head. The pressure loss through the various
system components, such as condensers, chillers, Fig. 15-8
FLUID FLOW, CENTRIFUGAL LIQUID PUMPS, WATER AND BRINE PIPING 281

Example 15-2. The recirculating water sys-


tem shown in Fig. 15-7 for a 10-ton refrig-
is
erating system. The flow rate over the tower is
40 gpm (4 gpm/ton).The flow rate through the
condenser is 30 gpm (3 gpm/ton), with 10 gpm Pipe system
curve Friction loss
(1 gpm/ton) flowing through the condenser by-
pass. From the manufacturers' rating tables,
the tower head based on 4 gpm/ton is 24 ft of
water column, whereas toe pressure drop s
through the condenser for 30 gpm is 11.2 psi or Static head
25.9 ft of water column (11.2 x 2.31). If the (or elevation)
size of the piping is 2 in. nominal, determine the
total pumping head and select the proper pump
from Chart R-19. Gpm
Fig. 15-10. Friction head of piping system increases
Solution. Total equiva- as flow rate through system increases. (Courtesy
lent length of pipe: Ingersoll-Rand Company.)
Straight pipe 80.0 ft

3 2 in. standard el-
bows at 5 ft 15.0 Converting to ft HaO = 3.71 x 2.31
2—2 in. tees (side out- = 8.58 ft H 2
let) at 12 ft 24.0 Total pumping head
4—2 in. gate valves Piping = 8.58 ft

(open) at 1.2 ft 4.8 Condenser = 25.90


123.8 ft
Tower = 24.00
From Chart 15-2, the = 57.58 ft H2
pressure loss per 100 ft of From Chart R-19, select pump Model
pipe (40 gpm and 2 in. pipe) 3 psi #1531-28, which has a delivery capacity of 40
Applying Equation 15-14, 123.8 gpm at a 57-ft head.
x 3
the total pressure loss 100 Example 15-3. At the required flow rate of
through the piping 3.71 psi 100 gpm, a certain water system has a pumping
head of 60 ft of water column. Select the proper
pump from Chart R-19.
Open balance
""
Solution. Reference to Chart R-19 shows that
tank
pump Model #1531-30 is the smallest pump
Chilled water
air-cooling
which can be used. However, since this pump
coil will deliver 125 gpm at a 60-ft head, to obtain
the desired flow rate of 100 gpm, the pumping
head must be increased to 73 ft of water. This is
accomplished by throttling the pump with a
globe valve installed on the discharge side of the
pump. (The pump should never be throttled on
the suction side.)
15-10. Power Requirements. The power
required to drive thepump depends upon the
Water chiller
delivery rate in pounds per minute, the total
pumping head, and the efficiency of the pump,
viz:
Pounds per minute x total head in feet
"
33,000 x pump efficiency

Since the flow rate is usually in gpm, a more


practical equation is
Fig. 15-9. Closed chilled water (or brine) circulating
system. To compute pumping head
having greatest friction loss. There is no
use circuit
static head.
Bhp =Gpm x33,000 head
x
x
total 8.33 lb/gal
efficiency
282 PRINCIPLES OF REFRIGERATION
Combining constants, lines should be kept as short as possible and a
minimum amount of fittings should be used.
Gpm x total head in feet Standard weight steel pipe or Type "L"
Bhp " (15 - 15)
3960 x efficiency copper tubing are usually employed for con-
denser water piping. Pipe sizes which will
Equation IS- IS applies to water. When a
provide water velocities in the neighborhood of
liquid other than water is handled, the specific
5 to 8 fps at the required flow rate will usually
gravity of the liquid must be taken into account,
prove to be the most economical. For example,
viz:
assume that 150 gpm of water are to be circu-
Gpm x total head x specific gravity
Bhp = lated through 100 equivalent feet of piping. The
3960 x efficiency following approximate values of velocity and
(15-16)
friction loss are shown in Chart 15-2 for a flow
rate of 150 gpm through various sizes of pipe:
From Equation 15-16, it is evident that the
power required by the pump increases as the
Pipe Size Velocity Friction Loss per 100 ft
delivery rate, total head, or specific gravity
(inches) (fps) (psi) (ftHaO)
increases,and decreases as the pump increases.
Typical pump horsepower and efficiency 2 15.5 31.5 72.8

curves are shown in Fig. 15-11. Notice that 2\ 10.0 10.5 24.3

pump horsepower is lowest at no delivery and 3 7.1 4.8 11.1

increases progressively as the delivery rate 3| 5.2 2.0 4.6

increases. Hence, any decrease in the pumping 4 3.9 1.1 2.5

head will cause an increase in both the delivery


Notice that whereas increasing the pipe size
rate and the power requirements of the pump.
from 2 to 3 in. results in a considerable reduction
Pump efficiency, also lowest at no delivery,
in the friction loss (from 72.8 to 11.1ft), a
increases to a maximum as the flow rate is

increased and then decreases as the flow rate


further increase in the pipe sizefrom 3 to 4 in.
is
reduces the friction loss by only an additional
further increased. The pump efficiency curve in
9.4 ft of water column (11.1 to 2.54 ft), which
Fig. 15-11 indicates that the highest efficiency is
will not ordinarily justify the increase in the cost
obtained when the pump is selected to deliver
of the pipe. Depending upon the characteristics
the desired gpm when operating at some point
of the available pump, either 3 in. or 3 J in. pipe
near the midpoint of its head-capacity curve.
15-11. WaterPiping Design. In general, the
should be used. For instance, assume two
water piping should be designed for the mini-
separate systems having pumping heads, ex-
clusive of tiie friction loss in the piping, of 55 ft
mum friction loss consistent with reasonable
of water column and 65 ft of water column,
initial costs so that the pumping requirements
respectively. Reference to Chart R-19 indicates
are maintained at a practical minimum. Water
that the only suitable pump for either of the
systems is Model 31531-32, which has a delivery
rate of 150 gpm at a 70-ft head. Therefore, for
the system having the 55-ft head, the permissible
friction loss in the piping is 15 ft (70 — 55),
whereas for the system having the 65-ft head,
the permissible friction loss in the piping is only
5 ft (70 — 65). For the latter system, 3J in. pipe
must be used, since the use of 3 in. pipe would
result in a total pumping head in excess of the
allowable 70 ft and necessitate the use of the
next larger size pump. On the other hand, for
Gpm the former system, 3 in. pipe is the most prac-
Fig. 15-1 1. Variations in pump horsepower and tical size. The use of 3 J in. pipe in this instance
efficiency with delivery rate. (Courtesy Ingersoll- would result in a total pumping head of only
Rand Company.) 61 ft and would necessitate throttling of the

FLUID FLOW. CENTRIFUGAL LIQUID PUMPS, WATER AND BRINE PIPING 283

pump discharge in order to raise the pumping If the flow rate through the pipe is 60 gpm,
head to 70 ft and obtain the desired flow rate of determine:
ISO gpm. (a) The total equivalent length of straight

In designing the piping system, care should be pipe. Ans. 297.5 equivalent ft
(b) The total friction loss through the pipe is
taken to include all and fittings necessary
valves
for the proper operation and maintenance of the
feet of water column. Ans. 9.28 ft 2 H
2. Rework Problem 1 using fairly rough pipe.
water circulating system. It is good practice to
install a globe valve on the discharge side of the
Ans. 12.02 ft 2 H
pump to regulate the water flow rate when the
3. A recirculating condenser water system con-
sists of 100 ft of straight pipe, and 6 standard
Too, where the piping is long
latter is critical.
90° elbows.At the desired flow rate the pressure
and/or the quantity of water in the system is drop through the condenser is 7.5 psi and the
large, shut-off valves installed on the inlet and pressure drop over the tower is 10 ft of water
outlet of both the pump and the condenser will column. If 60 gpm are circulated through the
permit repairs to these pieces of equipment system, determine:
without the necessity of draining the tower. A (a) The total equivalent length of pipe.

drain connection should be installed at the Ans. 1 30 equivalent ft

lowest point in the piping and the piping should (b) The total head against which the pump

be pitched downward so as to assure complete


must operate. Ans. 42.92 ft 2 H
drainage during winter shut down. 4. From the manufacturer's rating curves,
select a pump to fit the conditions of Problem 3.
The pump must always be located at some
point below the level of the water in the tower 5. For a Refrigerant- 12 system, select a water
regulating valve to meet the following con-
basin in order to assure positive and continuous
ditions:
priming of the pump. When quiet operation is (a) Desired condensing temperature range
required, the pump may be isolated from the 90° to 105°.
piping with short lengths of rubber hose. Auto- (6) Maximum entering water temperature
mobile radiator hose is suitable for this purpose. 85° F.
(c) Desired water quantity through condenser

PROBLEMS at maximum loading 9 gpm. —


(d) Pressure available at city main during
1. A
water piping system consists of 13S ft of period of peak loading 50 psi. —
nominal Type L smooth copper tube, 5
2.5 in. (e) Pressure loss through condenser and water
standard elbows, and 2 globe valves (full open). —
piping 12 psi.
extensively as refrigerants in the past, have been
discarded as more suitable fluids were developed.
Others, still in the development stage, show

promise for the future. Tables 16-2 through


16-6- list the thermodynamic properties of some
of the refrigerants in common use at the present
time. The use of these tables has already been,

16 described in an earlier chapter.


16-2. Safe Properties. Ordinarily, the safe
properties of the refrigerant are the prime con-
sideration in the selection of a refrigerant. It is

Refrigerants for this reason that some fluids, which otherwise


are highly desirable as refrigerants, find only
limited use as such. The more prominent of
these are ammonia and some of the straight
hydrocarbons.
To be suitable for use as a refrigerant, a fluid
16-1. The Ideal Refrigerant. Generally speak-
should be chemically inert to the extent that it is
ing, a refrigerant is any body or substance which nonflammable, nonexplosive, and nontoxic both
acts as a cooling agent by absorbing heat from
in the pure state and when mixed in any propor-
another body or substance. With regard to the
tion with air. Too, the fluid should not react
vapor-compression cycle, the refrigerant is the unfavorably with the lubricating oil or with any
working fluid of the cycle which alternately material normally used in the construction of
vaporizes and condenses as it absorbs and gives Nor should
refrigerating equipment. it react
off heat, respectively. To be suitable for use as a unfavorably with moisture which despite strin-
refrigerant in the vapor-compression cycle, a
gent precautions is usually present at least to
fluidshould possess certain chemical, physical,
some degree in all refrigerating systems. Fur-
and thermodynamic properties that make it both thermore, it is desirable that the fluid be of such a
safe and economical to use.
nature that it will not contaminate in any way
Itshould be recognized at the onset that there
foodstuff or other stored products in the event
is no "ideal" refrigerant and that, because of the that a leak develops in the system.
wide differences in the conditions and require- 16-3. Toxicity. Since all fluids other than air
ments of the various applications, there is no one are toxic in the sense that they will cause
refrigerant that is universally suitable for all
suffocation when in concentrations large enough
applications. Hence, a refrigerant approaches
to preclude sufficient oxygen to sustain life,
the "ideal" only to the extent that its properties
toxicity a relative term which becomes mean-
is
meet the conditions and requirements of the ingful only when the degree of concentration
application for which it is to be used.
and the time of exposure required to produce
Table 16-1 lists a number of fluids having harmful effects are specified.
properties which render them suitable for use as
The toxicity of most commonly used refrig-
refrigerants.* However, it will be shown erants has been tested by National Fire Under-
presently that only a few of the more desirable
writers. As a result, the various refrigerants are
ones are actually employed as such. Some, used
separated into six groups according to their
* Since some of the fluorocarbon refrigerants, degree of toxicity, the groups being arranged in
first introduced to the industry under the trade descending order (Column 1 of Table 16-7).
name "Freon," are now produced under several Those falling into Group 1 are highly toxic and
different trade marks, the ASRE, in order to avoid
are capable of causing death or serious injury
the confusion inherent in the use of either pro-
in relatively small concentrations and/or short
prietary or chemical names, has adopted a number-
ing system for the identification of the various
exposure periods. On the other hand, those

refrigerants. Table 16-1 lists the ASRE number classified in Group 6 are only mildly toxic, being
designation, along with the chemical name and capable of causing harmful effects only in rela-
formula for each of the compounds listed. tively large concentrations. Since injury from
284
REFRIGERANTS 285

the latter group is caused more by oxygen odor of the refrigerant, and whether or not
deficiency than by any harmful effects of the experienced personnel are on duty to attend the
fluids themselves, for all practical purposes the equipment. For example, a small quantity of
fluids in Group 6 are considered to be nontoxic. even a highly toxic refrigerant presents little
However, it should be pointed out that some hazard when used in relatively large spaces in
refrigerants, although nontoxic when mixed with that it is not possible in the event of a leak for
air in their normal state, are subject to decom- the concentration to reach a harmful level.
position when they come in contact with an open Too, the danger inherent in the use of toxic
flame or an electrical heating element. The refrigerants is somewhat tempered by the fact

products of decomposition thus formed are that toxic refrigerants (including decomposition
highly toxic and capable of causing harmful products) have very noticeable odors which tend
effects in small concentrations and on short to serve as a warning of their presence. Hence,
exposure. This is true of all the fluorocarbon toxic refrigerants are usually a hazard only to
refrigerants (see Column 3 of Table 16-7). infantsand others who, by reason of infirmity or
16-4. Flammability and Explosiveness. confinement, are unable to escape the fumes.
With regard to flammability and explosiveness, At the present time, ammonia is the only toxic
most of the refrigerants in common use are refrigerant that is used to any great extent, and
entirely nonflammable and nonexplosive its use is ordinarily limited to packing plants,
(Column 2 of Table 16-7). Notable exceptions ice plants, and large cold storage facilities where
to this are ammonia and the straight hydro- experienced personnel are usually on duty.
carbons. Ammonia is slightly flammable and 16-5. Economic and Other Considerations.
explosive when mixed in rather exact propor- Naturally, from the viewpoint of economical
tions with air. However with reasonable operation, it is desirable that the refrigerant
precautions, the hazard involved in using have physical and thermal characteristics which
ammonia as a refrigerant is negligible. will result in the minimum power requirements
Straight hydrocarbons,on the other hand, per unit of refrigerating capacity, that is, a high
are highly flammable and explosive, and their coefficient of performance. For the most part,

use as refrigerants except in special applications the properties of the refrigerant which influence
and under the surveillance of experienced the coefficient of performance are: (1) the
operating personnel not usually permissible.
is latent heat of vaporization, (2) the specific
Because of their excellent thermal properties, volume of the vapor, (3) the compression ratio,
the straight hydrocarbons are frequently and (4) the specific heat of the refrigerant in
employed in ultra-low temperature applications. both the liquid and vapor states.
In such installations, the hazard incurred by Except in very small systems, a high latent
their use is minimized by the fact that the heat value is desirable in that the weight of
equipment is constantly attended by operating refrigerant circulated per unit of capacity is

personnel experienced in the use and handling less. When a high latent heat value is accom-
of flammable and explosive materials. panied by a low specific volume in the vapor
The "American Standard Safety Code for state, the efficiency and capacity of the com-

Mechanical Refrigeration" sets forth in detail pressor are greatly increased. This tends not
the conditions and circumstances under which only to decrease the power consumption but
the various refrigerants can be safely used. also to reduce the compressor displacement
Most and ordinances governing
local codes required,which permits the use of smaller, more
on
the use of refrigerating equipment are based compact equipment. However, in small systems,
this code, which is sponsored jointly by the if the latent heat value of the refrigerant is too
ASRE and ASA. high, the amount of refrigerant circulated will
The degree of hazard incurred by the use of be insufficient for accurate control of the liquid.
toxic refrigerants depends upon a number of A low specific heat for the liquid and a high
factors, such as the quantity of refrigerant used vapor are desirable in that
specific heat for the

with relation to the size of the space into which both tend to increase the refrigerating effect
the refrigerant may leak, the type of occupancy, per pound, the former by increasing the sub-
whether or not open flames are present, the cooling effect and the latter by decreasing the
286 PRINCIPLES OF REFRIGERATION

Refrigerent-50
Methane
(CH4) -259°F

Refrigerant-40 Refrigerant-30 Refrigerant-20 Refrigerant-10


Methyl Chloride Methylene Chloride Chloroform Carbontetrach loride
(CH3CI) -11*F (CH2CI2) 104"F (CHCI3) 142-F (CCI4) 169°F

Refrigerant-21 Refrigerant-22
Dichloromonofluoromethane Monochlorodifluoromethane
(CHCI2F) 48'F (CHCIF2) -41°F

Refrigerant-11 Refrigerant- 12 Refrigerant- 13 Refrigerant- 14


Trichloromonofluoromethane Dichlorodifluoromethane Monochlorotrifluoromethane Carbontetrafluoride
(CClaF) 75'F (CCI2F2) -22°F (CCIF3) — 115*F (CF4) -198°F

Fig. 16-1. Methane series refrigerants.


REFRIGERANTS 287

superheating effect. When both are found in a initial cost of the refrigerating equipment and
single fluid, the efficiency of a liquid-suction which permit automatic operation and a
heat exchanger is much improved. minimum of maintenance.
The effect of compression ratio on the work 16-6. Early Refrigerants. In earlier days, when
of compression and, consequently, on the mechanical refrigeration was limited to a
coefficient of performance, has already been few large applications, ammonia and carbon
discussed in a previous chapter. Naturally, all dioxide were practically the only refrigerants
other factors being equal, the refrigerant giving available. Later, with the development of
the lowest compression ratio is the most small, automatic domestic and commercial
desirable.Low compression ratios result in such as sulfur dioxide and
units, refrigerants

low power consumption and high volumetric methyl chloride came into use, along with
efficiency, the latter being more important in methylene chloride, which was developed for
smaller systems since it permits the use of use with centrifugal compressors. Methylene
small compressors. chloride and carbon dioxide, because of their
Too, it is desirable that the pressure-tempera- safe properties, were extensively used in large
ture relationship of the refrigerant is such air conditioning applications.

that the pressure in the evaporator is always With the exception of ammonia, all these

above atmospheric. In the event of a leak on the refrigerants have fallen into disuse and are
low pressure side of the system, if the pressure found only in some of the older installations,
in the low side is below atmospheric, consider- having been discarded in favor of the more
able amounts of air and moisture may be drawn suitable fluorocarbon refrigerants as the latter
into the system, whereas if the vaporizing were developed. The fluorocarbons are practi-
pressure above atmospheric, the possibility of
is cally the only refrigerants in extensive use at the

drawing in air and moisture in the event of a present time. Again, an exception to this is

leak is minimized. ammonia which, because of its excellent thermal


Reasonably low condensing pressures under properties, is still widely used in such instal-
normal atmospheric conditions are also desirable lations as ice plants, skating rinks, etc. A few
in that they allow of lightweight
the use other refrigerants also find limited use in special
materials in the construction of the condensing applications.
equipment, thereby reducing the size, weight, 16-7. Development of the Fluorocarbons.

and cost of the equipment. The search for a completely safe refrigerant
Naturally, the critical temperature and with good thermal properties led to the develop-
pressure of the refrigerant must be above the ment of the fluorocarbon refrigerants in the late
maximum temperature and pressure which will 1920's. The fluorocarbons (fluoronated hydro-

be encountered in the system. Likewise, the carbons) are one group of a family of compounds
freezing point of the refrigerant must be safely known as the halocarbons (halogenated hydro-
below the minimum temperature to be obtained carbons). The halocarbon family of compounds
in the cycle. These factors are particularly are synthesized by replacing one or more of the
important in selecting a refrigerant for a low hydrogen atoms in methane (CH*) or ethane
temperature application. (CgHg) molecules, both of which are pure
In Table 16-8, a comparison is given of the hydrocarbons, with atoms of chlorine, fluorine,
performance of the various refrigerants at and/or bromine, the latter group comprising the
standard ton conditions (5° F evaporator and halogen family. Halocarbons developed from
86° F condensing). Notice particularly that, the methane molecule are known as "methane
with the exception of air, carbon dioxide, and series halocarbons." Likewise, those developed

ethane, the horsepower required per ton of from the ethane molecule are referred to as
refrigeration is very nearly the same for all the "ethane series halocarbons."
refrigerants listed. For this reason, efficiency The composition of the methane series halo-
and economy of operation are not usually carbons is shown in Fig. 16-1. Notice that the
deciding factors in the selection of the refrig- basic methane molecule consists of one atom of
erant. More important are those properties carbon (Q and four atoms of hydrogen (H).
which tend to reduce the size, weight, and If the hydrogen atoms are replaced progressively
288 PRINCIPLES OF REFRIGERATION

Refrigerant- 170
Ethane

(CH3CH3) -127.5-F

-113
114
Trichlorotrifluoroethane
Dichlortetrafluofoethane
(CCI2FCCIF2) 117.6'F (CClFjjCClFii) 38.4°F

Fig. 16-2. Ethane series refrigerants.

with chlorine (CI) atoms, the resulting com- carbons of the ethane series in common use.
pounds are methyl chloride (CH3 C1), methylene The presence of the two carbon atoms identifies
chloride (CHaClg), chloroform (CHCI3), and the basic molecule as ethane, rather than
carbontetrachloride (CC1 4), respectively, the methane, which has only one carbon atom.
lasttwo being the base molecules for the more The individual characteristics of these and
popular fiuorocarbons of the methane series. other refrigerants are discussed in the following
If the chlorine atoms in the carbontetra- sections.
chloride molecule are now replaced progressively 16-8. The Effect of Moisture. It is a well-
with fluorine atoms, the resulting compounds established fact that moisture will combine in
are trichloromonofluoromethane (CC1 S F), varying degrees with most of the commonly
dichlorodifluoromethane (CCl a F2), mono- used refrigerants, causing the formation of
chlorotrifluoromethane (CCIF3), and carbon- highly corrosive compounds (usually acids)
tetrafiuoride (CF^, respectively. In the same which will react with the lubricating oil and
order, the ASRE refrigerant standard number with other materials in the system, including
designations for these compounds are Refrig- metals. This chemical action often results in
erants-11, 12, 13, and 14, the last figure in the pitting and other damage to valves, seals,
numbers being an indication of the number of bearing journals, cylinder walls, and other
fluorine atoms in the molecule. polished surfaces. It may also cause deteriora-
The molecular structure of Refrigerants-21 tion of the lubricating oiland the formation of
and 22, which are also fiuorocarbons of the metallic and other sludges which tend to clog
methane series, is shown in Fig. 16-1. Notice valves and oil passages, score bearing surfaces,
the presence of the hydrogen atom in each of and otherwise reduce the life of the equipment.
these two compounds, an indication that they Moisture corrosion also contributes to com-
are derivatives of the chloroform molecule pressor valve failure and, in hermetic motor-
rather than the carbontetrachloride molecule. compressors, often causes breakdown of the
Figure 16-2 shows the molecular structure of motor winding insulation, which results in
Refrigerants-113 and 114, the only two fiuoro- shorting or grounding of the motor.
REFRIGERANTS 289

Although a completely moisture-free refrig- to hold moisture in solution decreases as the


erating system is not possible, good refrig- temperature decreases, it follows that the
erating practice demands that the moisture moisture content in low temperature systems
content of the system be maintained below the must be maintained at a very low level in order to
level which will produce harmful effects in the avoid freeze-ups. Hence, moisture corrosion in
system. The minimum moisture level which low temperature systems is usually at a minimum.
will produce harmful effects in a refrigerating The various refrigerants differ greatly both as
system is not clearly defined and will vary to the amount of moisture they will hold in
considerably, depending upon the nature of the solution and as to the effect that the moisture
refrigerant, the quality of the lubricating oil, and has upon them. For example, the straight
the operating temperatures of the system, par- hydrocarbons, such as propane, butane, ethane,
ticularly the compressor discharge temperature. etc.,absorb little if any moisture. Therefore,
Moisture in a refrigerating system may exist any moisture contained in such systems will be
as "free water" or it may be in solution with the in the form of free water and will make its
refrigerant. When moisture is present in the presence known by freezing out in the refrig-
system in the form of free water, it will freeze erant control. Since this moisture must be
into ice in the refrigerant control and/or in the removed immediately in order to keep the system
evaporator, provided that the temperature of operative, moisture corrosion will not usually
the evaporator maintained below the freezing
is be a problem when these refrigerants are used.
point of the water. Naturally, the formation of Ammonia and sulfur dioxide, on the other
ice in the refrigerant control orifice will prevent hand, have an affinity for water and therefore
the flow of liquid refrigerant through that part are capable of absorbing moisture in such large
and render the system inoperative until such quantities that free water is seldom found
time that the ice melts and flow through the in systems employing these two refrigerants.
control is restored. In such cases, refrigeration However, the effectsproduced by the combina-
is usually intermittent as the flow of liquid is tion of the water and the refrigerant are entirely
started and stopped by alternate melting and different for the two refrigerants.
freezing of the ice in the control orifice. In ammonia systems, the combination of
Since free water exists in the system only water and ammonia produces aqua ammonia, a
when the amount of moisture in the system strong alkali, which attracts nonferrous metals,
exceeds the amount that the refrigerant can such as copper and brass, but has little if any
hold in solution, freeze-ups are nearly always effect on iron or steel or any other materials in
an indication that the moisture content of the the system. For this reason, ammonia systems
system is above the minimum level that will can be operated successfully even when rela-
produce corrosion. On the other hand, the tively large amounts of moisture are present in
mere absence of freeze-ups cannot be taken to the system.
mean that the moisture content of the system In the case of sulfur dioxide, the moisture
is necessarily below the level which will cause and sulfur dioxide combine to form sulfurous
corrosion, since corrosion can occur with some acid (H2SO3), which is highly corrosive. In
refrigerants at levels well below those which view of the high solubility of water in S02 , the
will result in free water. Too, it must be amount of acid formed can be quite large.
recognized that freeze-ups do not occur in air Hence, corrosion in sulfur dioxide systems can
conditioning systems or in any other system be very heavy.
where the evaporator temperature is above The halocarbon refrigerants hydrolyze only
the freezing point of water. For this reason, slightly and therefore form only small amounts
high temperature systems are often more sub- of acids or other corrosive compounds. As a
ject to moisture corrosion than are systems general rule, corrosion will not occur in systems
operating at lower evaporator temperatures, employing halocarbon refrigerants when the
since relatively large quantities of moisture can moisture content is maintained below the level
go unnoticed in such systems for relatively long which will cause freeze-ups, provided that high
periods of time. quality lubricating oils are used and that dis-
Since the ability of an individual refrigerant charge temperatures are reasonably low.
290 PRINCIPLES OF REFRIGERATION
16-9. Refrigerant-Oil Relationship. With a copper plating is not found in ammonia systems.
few exceptions, the oil required for lubrication However, neither is it found in sulfur dioxide
of the compressor is contained in the crankcase systems, although copper has been employed
of the compressor where it is subject to contact extensively with this refrigerant.
with the refrigerant. Hence, as already stated, In any event, regardless of the nature of
the refrigerant must be chemically and physically and/or the cause of unfavorable reactions
stable in the presence of oil, so that neither the between the refrigerant and the lubricating oil,
refrigerant nor the oil is adversely affected by these disadvantages can be greatly minimized
the relationship. or eliminated by the use of high quality lubri-
Although some refrigerants, particularly sul- cating oils, having low "pour" and/or "floe"
fur dioxide and the halocarbons, react with points (see Section 18-16), by maintaining the
the lubricating oil to some extent, under normal system relatively free of contaminants, such as
operating conditions the reaction is usually air and moisture, and by designing the system so

slight and therefore of little consequence, that discharge temperatures are reasonably low.
provided that a high quality lubricating oil is 16-10. Oil Miscibility. With regard to the
used and that the system is relatively clean and refrigerant-oil relationship, one important
dry. However, when contaminants, such as air characteristic which differs for the various
and moisture, are present in the system in any refrigerants is oil miscibility, that is, the ability
appreciable amount, chemical reactions involv- of the refrigerant to be dissolved into the oil and
ing the contaminants, the refrigerant, and the vice versa.
lubricating oil often occur which can result in With reference to oil miscibility, refrigerants
decomposition of the oil, the formation of may be divided into three groups: (1) those
corrosive acids and sludges, copper plating, which are miscible with oil in all proportions
and/or serious corrosion of polished metal under conditions found in the refrigerating
surfaces. High discharge temperatures greatly system, (2) those which are miscible under
accelerate these processes, particularly oil de- conditions normally found in the condensing
composition, and often result in the formation of section, but separate from the oil under the
carbonaceous deposits on discharge valves and conditions normally found in the evaporator
pistons and in the compressor head and dis- section, and (3) those which are not miscible
charge line. This condition is aggravated by the with oil at all (or only very slightly so) under
use of poorly refined lubricating oils containing conditions found in the system.
a high percentage of unsaturated hydrocarbons, As to whether or not oil miscibility is a
the latter being very unstable chemically. desirable property in a refrigerant there is some
Because of the naturally high discharge disagreement. In any event, the fact of oil

temperature of Refrigerant-22 (see Table 16-8), miscibility, or the lack of it, has little if any
breakdown of the lubricating oil, accompanied significance insofar as the selection of the
by motor burnouts, is a common problem with refrigerant is However, since it
concerned.
hermetic motor-compressor units employing greatly influences the design of the compressor
this refrigerant, particularly when used in con- and other system components, including the
junction with air-cooled condensers and long refrigerant piping, the degree of oil miscibility
suction lines. is an important refrigerant characteristic and
Copper plating of various compressor parts therefore should be considered in some detail.

is often found in systems employing halocarbon With regard to the oil, one of the principal
refrigerants. The parts usually affected are the effects of an oil miscible refrigerant is to dilute
highly polished metal surfaces which generate the oil in the crankcase of the compressor,
heat, such as seals, pistons, cylinder walls, thereby lowering the viscosity (thinning) of the
bearing surfaces, and valves. The exact cause of oil and reducing its lubricating qualities. To
copper plating has not been definitely deter- compensate for refrigerant dilution, the com-
mined, but considerable evidence does exist that pressor lubricating oil used in conjunction with
moisture and poor quality lubricating oils oil-miscible refrigerants should have a higher
are contributing factors. initial viscosity than that used for similar duty
Because copper is never used with ammonia, with nonmiscible refrigerants.
REFRIGERANTS 291

Viscosity may be defined as a measure of ing about the return of oil to the crankcase
fluid friction or as a measure of the resistance depends primarily on three factors: (1) the
that a fluid offers to flow. Hence, thin, low oil miscibility of the refrigerant, (2) the type of
viscosity fluids will flow more readily than evaporator used, and (3) the evaporator tem-
thicker, more viscous fluids. To provide perature.
adequate lubrication for the compressor, the When an oil-miscible refrigerant is employed,
viscosity of the lubricating oil must be main- the problem of oil return is greatly simplified
tained within certain limits.
If the viscosity of by the fact that the oil remains in solution with
the oil is not have sufficient
too low, the oil will the refrigerant. This permits the oil to be
body to form a protective film between the carried along through the system by the
various rubbing surfaces and keep them refrigerant and, subsequently, to be returned to
separated. On the other hand, if the viscosity the crankcase through the suction line, provided
of the oil is too high, the oil will not have that the evaporator and the refrigerant piping
sufficient fluidity to penetrate between the are properly designed.
rubbing surfaces, particularly where tolerances Unfortunately, when nonmiscible refriger-
are close. In either case, lubrication of the ants are used, once the oil passes into the
compressor not be adequate.
will condenser, the return of theoil to the crankcase
Any through the system with
oil circulating is not so easily accomplished. The reason for
the refrigerant will have an adverse affect on this is that, except for a small amount of
the efficiency and capacity of the system, the mechanical mixing, the refrigerant and the oil
principal reason being that the oil tends to will remain separate, so that only a small
adhere to and to form a film on the surface of portion of the oil is actually carried along with
the condenser and evaporator tubes, thereby the refrigerant. For example, in the case of
lowering the heat transfer capacity of these two ammonia, which is lighter than oil, a large
units. Since the oil becomes more viscous and percentage of the oil will separate from the
tends to congeal as the temperature is reduced, liquid ammonia and settle out at various low
the problem with oil is greatest in the evaporator points in the system. For this reason, oil
and becomes more acute as the temperature of drains should be provided at the bottom of all

the evaporator is lowered. receivers, evaporators, accumulators, and other


Since the only reason for the presence of oil vessels containing liquid ammonia, and pro-
in the refrigerating system is to lubricate the visions should be made for draining the oil
compressor, it is evident that the oil will best from these points, either continuously or
serve its function when confined to the com- periodically, and returning it to the crankcase.
pressor and not allowed to circulate with the This may be accomplished manually or auto-
refrigerant through other parts of the system. matically.
However, since, with few exceptions, the system When flooded-type evaporators are used, the
refrigerant unavoidably comes into contact with refrigerant velocity will not usually be sufficient
the oil in the compressor, a certain amount of to permit the refrigerant vapor to entrain the
oil in the form of small particles will be en- oil and carry it over into the suction line and

trained in the refrigerant vapor and carried over back to the crankcase. Hence, even with oil
through the discharge valves into the discharge miscible refrigerants, where flooded-type evap-
line. If the oil is not removed from the vapor orators are employed, it is often necessary to
at this point, it will pass into the condenser and make special provisions for oil return. The
liquid receiver from where it will be carried methods used to insure the continuous return
to the evaporator by the liquid refrigerant. of the oil from the evaporator to the crankcase
Obviously, in the interest of system efficiency in such cases is described in Chapter 19.
and in order to maintain the oil in the crank- Since the oil acts to lubricate the refrigerant
case at a constant level, some provision must be flow control and other valves which may be in
made for removing this oil from the system and the system, the circulation of a small amount of
returning it to the crankcase where it can per- oil with the refrigerant is not ordinarily objec-
form its lubricating function. tionable. However, because of the adverse
The degree of difficulty experienced in bring- effect on system capacity, the amount of oil
292 PRINCIPLES OF REFRIGERATION

should be kept to a practical minimum. Too, depending on whether the pressure in the
comes initially from
since the oil in circulation system at the point of leakage is above or below
the compressor crankcase, an excessive amount atmospheric pressure. When the pressure in
in circulation may cause the oil level in the the system is above atmospheric at the point of
crankcase to below the minimum level
fall leakage, the refrigerant will leak from the
required for adequate lubrication of the com- system to the outside. On the other hand,
pressor parts. when the pressure in the system is below
In order to minimize the circulation of oil, an atmospheric, there isno leakage of refrigerant
oil separator or trap is sometimes installed to the outside, but air and moisture will be
in the discharge line between the compressor drawn into the system. In either case, the
and the condenser (see Section 19-12). system will usually become inoperative in a
As a general rule, discharge line oil separators very short time. However, as a general rule,
should be employed in any system where oil outward leaks are less serious than inward ones,
return is likely to be inadequate and/or where usually requiring only that the leak be found and
the amount of oil in circulation is apt to be repaired and that the system be recharged with
excessive or to cause an undue loss in system the proper amount of refrigerant. In the case of
capacity and efficiency. Specifically, discharge inward leaks, the air and moisture drawn into
line oil separators are recommended for all the system increase the discharge pressure and
systems employing nonmiscible refrigerants temperature and accelerates the rate of corro-
(or refrigerants which are not oil miscible at the sion. The presence of moisture in the system
evaporator conditions), not only because of the may also cause freeze-upof the refrigerant
difficulty experienced in returning the oil from control. Furthermore, after the leak has been
the evaporator to the crankcase but also because located and repaired, the system must be
the presence of even small amounts of oil in the completely evacuated and dehydrated before it
evaporators of such systems will usually cause can be placed in operation. A refrigerant drier
considerable loss of evaporator efficiency and should also be installed in the system.
capacity. The necessity of maintaining the system free
The same thing is usually true for systems of leaks demands some convenient means for
employing miscible refrigerants when the checking a new system for leaks and for
evaporator temperature is below 0° F. Oil detecting leaks if and when theyoccur in
separators are recommended also for all systems already in operation. New systems
systems using flooded evaporators, since oil should be checked for leaks under both vacuum
return from this typeof evaporator is apt to and pressure.
be inadequate because of low refrigerant One method of leak detection universally
velocities. used with all refrigerants employs a relatively
Although oil separators are very effective in viscous soap solution which is relatively free of
removing oil from the refrigerant vapor, they bubbles. The soap solution is first applied to
are not 100% efficient. Therefore, even though the pipe joint or other suspected area and then
an oil separator is used, some means must still examined with the help of a strong light. The
be provided for returning to the crankcase the formation of bubbles in the soap solution
small amount of oil which will always pass indicates the presence of a leak. For adequate
through the separator and find its way into testing with a soap solution, the pressure in the
other parts of the system. Too, since oil system should be 50 psig or higher.
separators can often cause serious problems in The fact that sulfur and ammonia vapors
the system if they are not properly installed, produce a dense white smoke (ammonia
the use of oil separators should ordinarily be sulfite) when they come into contact with one
limited to those systems where the nature of the another provides a convenient means of
refrigerant or the particular design of the checking for leaks in both sulfur dioxide and
system requires their use. Oil separators are ammonia systems. To check for leaks in a
discussed in more Chapter 19.
detail in sulfur dioxide system, a cloth swab saturated
16-11. Leak Detection. Leaks in a refrigerat- with stronger ammonia (approximately 28%
ing system may be either inward or outward, available in any drug store) is held near, but
REFRIGERANTS 293

not in contact with, all pipe joints and other high suction superheats should be avoided in
suspected areas. A leak is indicated when the ammonia systems.
ammonia swab gives off a white smoke. Although pure anhydrous ammonia is non-
Ammonia systems are checked in the same corrosive to metals normally used in refrig-
all
way except that a sulfur candle is substituted for erating systems, in the presence of moisture,
the ammonia swab. Dampened phenophthalein ammonia becomes corrosive to nonferrous
paper, which turns red on contact with am- metals, such as copper and brass. Obviously,
monia vapor, may also be used to detect these metals should never be used in ammonia
ammonia leaks. systems.
A halide torch is often used to detect leaks in Ammonia is not oil miscible and therefore
systems employing any of the halocarbon refrig- will not dilute the oil in the compressor crank-
erants. The halide torch consists of a copper case. However, provisions must be made for
element which is heated by a flame. Air to the removal of oil from the evaporator and an
support combustion is drawn in through a oil separator should be used in the discharge line
rubber tube, one end of which is attached to the of allammonia systems.
torch. The free end of the tube is passed around Ammonia systems may be tested for leaks
all suspected areas.The presence of a halo- with sulfur candles, which give off a dense white
carbon vapor is indicated when the flame smoke in the presence of ammonia vapor, or by
changes from its normal color to a bright green applying a thick soap solution around the pipe
or purple. The halide torch should be used only which case a leak is indicated by the
joints, in
in well-ventilated spaces. appearance of bubbles in the solution.
For carbon dioxide and the straight hydro- 16-13. Sulfur Dioxide. Sulfur dioxide (SO
s)
carbons, the only method of leak detection is is produced from the combustion of sulfur. It
the soap solution previously mentioned. is highly toxic, but nonflammable and
16-12. Ammonia. Ammonia is the only refrig- nonexplosive. In the 1920s and 1930s, sulfur
erant outside of the fluorocarbon group that is dioxide was widely used in domestic refrigerators
being used to any great extent at the present and in small commercial fixtures. Today, it is
time. Although ammonia is toxic and also found only in a few of the older commercial
somewhat flammable and explosive under cer- units, having been replaced first by methyl
tain conditions, its excellent thermal properties chloride and later by the more desirable fluoro-
make it an ideal refrigerant for ice plants, carbon refrigerants.
packing plants, skating rinks, large cold storage The boiling point of sulfur dioxide at atmos-
facilities, etc., where experienced operating pheric pressure is approximately 14° F. Satur-
personnel are usually on duty and where its ation pressures at standard ton conditions of
toxic nature is of little consequence. 5°F and 86° F are 5.9 in. Hg and 51.8 psig,
Ammonia has the highest refrigerating effect respectively.
per pound of any refrigerant. This, together Sulfur dioxide is not oil miscible. However,
with a moderately low specific volume in the unlike ammonia and carbon dioxide, liquid sul-
vapor state, makes possible a high refrigerating fur dioxide is heavier than oil so that the oil
capacity with a relatively small piston dis- floats on top of the refrigerant. Since this
placement. characteristic simplifies the problem of oil
The boiling point of ammonia at standard return, accounts for the popularity enjoyed by
it

atmospheric pressure is -28° F. The evapora- sulfur dioxide in the past for small automatic
tor and condenser pressures at standard ton equipment.
conditions of 5° F and 86° F are 19.6 psig and Like most common refrigerants, sulfur dioxide
154.5 psig, respectively, which are moderate, so in the pure state is noncorrosive to metals
that lightweight materials can be used in the normally used in the refrigerating system. How-
construction of the refrigerating equipment. ever, it combines with moisture to form
However, the adiabatic discharge temperature sulfurous acid (H2SOg) and sulfuric acid
is relatively high, being 210° F at standard ton (HgSO^, both of which are highly corrosive.
conditions, which makes water cooling of the 16-14. Carbon Dioxide. Carbon dioxide (COj)
compressor head and cylinders desirable. Too, is one of the first refrigerants used in mechanical
294 PRINCIPLES OF REFRIGERATION

refrigerating systems. It is odorless, nontoxic, refrigerant, which accounts for its wide use in
nonflammable, nonexplosive, and noncorrosive. the past in both domestic and commercial appli-
Because of its safe properties, it has been widely cations. Its boiling at atmospheric
point
used in the past for marine service and for air pressure is —10.65° F.
Evaporator and con-
conditioning in hospitals, theaters, hotels, and denser pressures at standard ton conditions are
in other places where safety is the prime con- 6.5 psig and 80 psig, respectively.
sideration. Although a few of these older Although methyl chloride is considered non-
installations are still in service, at the present toxic, in large concentrations it has an anesthetic
time the use of carbon dioxide as a refrigerant effect similar to that of chloroform, a compound

is limited for the most part to extremely low to which it is closely related. Methyl chloride is
temperature applications, particularly in the moderately flammable and is explosive when
production of solid C02 (dry ice). mixed with air in concentrations between 8.1
One of the chief disadvantages of carbon and 17.2% by volume. The hazard resulting
dioxide is its high operating pressures, which from these properties is the principal reason for
under standard ton conditions of 5° F and 86° F the discarding of methyl chloride in favor of the
are 317.5 psig and 1031 psig, respectively. safer fluorocarbon refrigerants.
Naturally, this requires the use of extra heavy Methyl chloride is corrosive to aluminum,
piping and equipment. However, because of the zinc, and magnesium, and the compounds
high vapor density of COa the volume of vapor
, formed in combination with these materials are
handled by the compressor is only 0.96 cu ft per both flammable and explosive. Hence, these
minute per ton at 5° F, so that compressor sizes metals should not be used in methyl chloride
are small. systems. In the presence of moisture, methyl
Another disadvantage of carbon dioxide is chloride forms a weak hydrochloric acid, which
that the horsepower required per ton is approxi- is corrosive to both ferrous and nonferrous

mately twice that of any of the commonly used metals. Too, since natural rubber and the
refrigerants. For carbon dioxide, the theoretical synthetic, Neoprene, are dissolved by methyl
horsepower required per ton at standard con- chloride, neither is suitable gasket material for
ditions is 1.84, whereas for ammonia, the horse- use in methyl chloride systems.
power required per ton is only 0.989, the latter Oil return in methyl chloride systems is

value being typical for most refrigerants. simplified by the fact that methyl chloride is oil
Since its boiling temperature at atmospheric miscible. However, in selecting the compressor
pressure ( — 109.3° F) is below its freezing tem- lubricating oil, crankcase dilution must be taken
perature (— 69.9° F) at this pressure, carbon into account.
dioxide cannot exist in the liquid state at atmos- Leaks in a methyl chloride system are found
pheric pressure nor at any pressure below its with the aid of a soap solution which is applied
triple point pressure of 75.1 psia. At any to the suspected joints. The presence of methyl
pressure under 75.1 psia, solid carbon dioxide chloride vapor may be detected with a halide
sublimes directly into the vapor state and there- leak detector. However, this method is not
fore below this pressure is found only in the recommended because of the flammability of
solid and vapor states. Because of the low methyl chloride.
critical temperature of COa (87.8° F), relatively 16-16. Methylene Chloride (Carrene I).

low condensing temperatures are required for Methylene chloride (CH2 C1 2 ), another halo-
liquefaction.Carbon dioxide is nonmiscible in carbon of the methane series, has a boiling point
oiland therefore will not dilute the oil in the of 103.5° F at atmospheric pressure, a charac-
crankcase of the compressor. Like ammonia, teristic which permits the refrigerant to be stored

it is lighter than oil. Hence, oil return problems in sealed cans rather than in compressed gas
are similar to those encountered in an ammonia cylinders.Under standard ton conditions, the
system. evaporator and condenser pressures are both
Leak detection is by soap solution only. below atmospheric pressure, being 27.6 in. Hg
16-15. Methyl Chloride. Methyl chloride and 9.5 in. Hg, respectively. Since the volume
(CH 3C1) is a halocarbon of the methane series. of the vapor handled per ton of refrigerating
It has many of the properties desirable in a capacity is quite large (74.3 cu ft/min/ton at
3

REFRIGERANTS 295

5° F), centrifugal compressors, which are par- Along with its safe properties, the fact that
ticularly suited to handling large volumes of low Refrigerant- 12 condenses at moderate pressures
pressure vapor, are required. under normal atmospheric conditions and has a
Although it dissolves natural rubber, methy- boiling temperature of — 21 °F at atmospheric
lene chloride is noncorrosive even in the presence pressure makes it a suitable refrigerant for use
of moisture. It is also nontoxic and nonflam- in high, medium, and low temperature appli-
mable. Because of its safe properties, it has been cations and with all three types of compressors.
widely used in large air conditioning installations. When employed in conjunction with multistage
The fact that methylene chloride is oil miscible centrifugaltype compressors, Refrigerant-12
is of little consequence, since in centrifugal com- has been used to cool brine to temperatures as
pressors the oiland refrigerant do not ordinarily low as -110° F.
come one another.
in contact with The fact that Refrigerant-12 is oil miscible
A halide torch or soap solution may be used under all operating conditions not only sim-
to detect leaks. However, the pressure in the plifies the problem of oil return but also tends
system must be built up above atmospheric in to increase the efficiency and capacity of the
either case. system in that the solvant action of the refrig-
16-17. Refrigerant-ll. Refrigerant- 1
1 (CC1 3 F) erant maintains the evaporator and condenser
is a fluorocarbon of the methane series and tubes relatively free of oil films which otherwise
has a boiling point at atmospheric pressure of would tend to reduce the heat transfer capacity
74.7° F. Operating pressures at standard ton of these two units.
conditions are 24 in. Hg and 3.6 psig, respec- Although the refrigerating effect per pound for
tively,which is very similar to those of methylene Refrigerant-12 is relatively small as compared to
chloride. Although the theoretical horsepower that of some of the other popular refrigerants,
required at standard ton conditions (0.927) is this is not necessarily a serious disadvantage.
approximately the same as that for methylene In fact, in small systems, the greater weight of
chloride, the compressor displacement required Refrigerant-12 which must be circulated is a
at these conditions (36.32 cu ft/min/ton)is only decided advantage in that it permits closer
approximately one-half that required for methy- control of the liquid. In larger systems, the
lene chloride. disadvantage of the low latent heat value is

Like other fluorocarbon refrigerants, Refrig- offset somewhat by a high vapor density, so that
erant- 11 dissolves natural rubber. However, the compressor displacement required per ton of
it is noncorrosive, nontoxic, and nonflammable. refrigeration is not much greater than that
The low operating pressures and the relatively required for Refrigerants-22, 500, and 717. The
high compressor displacement required necessi- horsepower required per ton of capacity com-
tate the use of a centrifugal compressor. pares favorably with that required for other
Refrigerant-ll is used mainly in the air commonly used refrigerants.
conditioning of small office buildings, factories, A halide torch is used for leak detection.
department stores, theaters, etc. A
halide torch 16-19. Refrigerant- 1 3. Refrigerant-13(CC1F,)
may be used for leak detection. was developed for and is being used in ultra-low

16-18. Refrigerant- 1 2. Although its suprem- temperature applications, usually in the low
acy is being seriously challenged in some areas stage of a two or three stage cascade system. It
by Refrigerant-22, Refrigerant- 12 (CCl 8 Fg) is by is also being used to replace Refrigerant-22 in
far the most widely used refrigerant at the some low temperature applications.
present time. It is a completely safe refrigerant The boiling temperature of Refrigerant- 13 is

in that it is nontoxic, nonflammable, and non- — 144.5° F at atmospheric pressure. Evaporator


explosive. Furthermore, it is a highly stable temperatures down to — 150°F are practical.
compound which is difficult to break down even The critical temperature is 83.9° F. Since con-
under extreme operating conditions. However, densing pressures and the compressor displace-
if brought into contact with an open flame or ment required are both moderate, Refrigerant-1
with an electrical heating element, Refrigerant- is suitable for use with all three types of
12 will decompose into products which are compressors.
highly toxic (see Section 16-3). Refrigerant-13 is a safe refrigerant. It is not
296 PRINCIPLES OF REFRIGERATION

miscible with oil. A halide torch may be used evaporator temperatures between —20 and
for leak detection. —40° F, still another advantage added to
16-20. Refrigerant-22. Refrigerant-22 Refrigerant-22 is that the evaporator pressures
(CHClFj) has a boiling point at atmospheric for Refrigerant-22 at these temperatures are
pressure of —41.4° F. Developed primarily as a above atmospheric, whereas for Refrigerant-12
low temperature refrigerant, it is used exten- the evaporator pressures will be below atmos-
sively in domestic and farm freezers and in pheric. However, all this should not be taken to
commercial and industrial low temperature mean that Refrigerant-22 is superior to Refrig-
systems down to evaporator temperatures as erant-12 in all applications. As a matter of fact,
low as — 125°F. It also finds wide use in except in those applications where space limita-
packaged air conditioners, where, because of tions necessitate the use of the smallest possible
space limitations, the relatively small compressor equipment and/or where the evaporator tem-
displacement required is a decided advantage. perature is between -20° F and -40° F,
Both the operating pressures and the adiabatic Refrigerant-12, because of its lower discharge
discharge temperature are higher for Refrig- temperatures and greater miscibility with oil, is

erant-22 than for Refrigerant-12. Horsepower probably the more desirable of the two
requirements are approximately the same. refrigerants.
Because of the high discharge temperatures The ability of Refrigerant-22 to absorb mois-
experienced with Refrigerant-22, suction super- ture is considerably greater than that of Refrig-
heat should be kept to a minimum, particularly erant-12 and therefore less trouble is experi-
where hermetic motor-compressors are em- enced with freeze-ups in Refrigerant-22 systems.
ployed. In low temperature applications, where Although some consider this to be an advantage,
compression ratios are likely to be high, water the advantage gained is questionable, since any
cooling of the compressor head and cylinders is amount of moisture in a refrigerating system is
recommended in order to avoid overheating of undesirable.
the compressor. Air-cooled condensers used Being a fluorocarbon, Refrigerant-22 is a safe
with Refrigerant-22 should be generously sized. refrigerant. A halide torch may be used for leak
Although miscible with oil at temperatures detection.
found in the condensing section, Refrigerant-22 16-21. Refrigerant-1 13. Refrigerant-113
will often separate from the oil in the evaporator. (CCljjFCClFa) boils at 117.6° F under atmos-
The exact temperature at which separation pheric pressure. Operating pressures at standard
occurs varies considerably with the type of oil ton conditions are 27.9 in. Hg and 13.9 in. Hg,

and the amount of oil mixed with the refrigerant. respectively. Although the compressor displace-
However, no difficulty is usually experienced ment per ton is somewhat high ( 1 00.76 cu ft/min/
with oil return from the evaporator when a ton at standard ton conditions), the horsepower
properly designed serpentine evaporator is used required per ton compares favorably with other
and when the suction piping is properly designed. common refrigerants. The low operating
When flooded evaporators are employed, oil pressures and the large displacement required
separators should be used and special pro- necessitate the use of a centrifugal type
visions should be made to insure the return of compressor.
oilfrom the evaporator. Oil separators should Although used mainly in comfort air con-
always be used on low temperature applica- ditioning applications, it also employed in
is

tions. industrial process water and brine chilling down


The principal advantage of Refrigerant-22 to 0° F.
over Refrigerant-12 is the smaller compressor Refrigerant-1 13 is a safe refrigerant. halide A
displacement required, being approximately torch may be used for leak detection.
60% of that required for Refrigerant-12. Hence, 16-22. Refrigerant-1 14. Refrigerant- 114
for a given compressor displacement, the refrig- (CClgCClFj) has a boiling point of 38.4° F under
erating capacity is approximately 60% greater atmospheric pressure. Evaporating and con-
with Refrigerant-22 man with Refrigerant-12. densing pressures at standard ton conditions are
Too, refrigerant pipe sizes are usually smaller 16.1 in. Hg and 22 psig, respectively. The com-
for Refrigerant-22 than for Refrigerant-12. For pressor displacement required is relatively low
REFRIGERANTS 297

for a low pressure refrigerant (19.59 cu ft/min/ weight) and Refrigerant- 152a (26.2%). It has a
ton at standard conditions) and the horsepower boiling point at atmospheric pressure of —28° F.
required compares favorably with that required Evaporator and condenser pressures at standard
by other common refrigerants. ton conditions are 16.4 psig and 113.4 psig,
Refrigerant- 1 14 is used with centrifugal com- respectively. Although the horsepower require-
pressors in large commercial and industrial air ments of Refrigerant-500 are approximately the
conditioning installations and for industrial same as those for Refrigerants- 12 and 22, the
process water chilling down to —70° F. It is compressor displacement required is greater
also used with vane-type rotary compressors in than that required for Refrigerant-22, but some-
domestic refrigerators and in small drinking what less than that required for Refrigerant- 12.
water coolers. The principal advantage of Refrigerant-500
Like Refrigerant-22, Refrigerant- 1 14 is oil lies in the fact that its substitution for Refrig-
miscible under conditions found in the con- erant- 12 results inan increase in compressor
densing section, but separates from oil in the capacity of approximately 18%. This makes it
evaporator. However, because of the type of possible to use the same direct connected com-
equipment used with Refrigerant- 1 14 and the pressor (as in a hermetic motor-compressor
conditions under which it is used, oil return is unit) on either 50 or 60 cycle power with little or
not usually a problem. no change in the refrigerating capacity or in the
Refrigerant-1 14 is a safe refrigerant. A halide power requirements.
torch may be used for leak detection. It will be shown in Chapter 21 that the speed
16-23. Straight Hydrocarbons. The straight of an alternating current motor varies in direct
hydrocarbons are a group of fluids composed in proportion to the cycle frequency. Therefore,
various proportions of the two elements hydro- an electric motor operating on 50 cycle power
gen and carbon. Those having significance as will have only five-sixths of the speed it has when
refrigerants are methane, ethane, butane, pro- operating on 60 cycle power. For this reason,
pane, ethylene, and isobutane. All are extremely the displacement of a direct connected com-
flammable and explosive. Too, since all act as pressor reduced approximately 18% when a
is
anesthetics in varying degrees, they are con- change is made from 60 to 50 cycle power.
sidered mildly toxic. Although none of these Since the increase in capacity per unit of dis-
compounds absorb moisture to any appre-
will placement accruing from the substitution of
ciable extent, all are extremely miscible with oil Refrigerant-500 for Refrigerant- 12 is almost
under all conditions. exactly equal to the loss of displacement suffered
Although a few of the straight hydrocarbons when changing for 60 to 50 cycle power, the
(butane, propane, and isobutane) have been same motor-compressor assembly is made
used in small quantities for domestic refrigera- suitable for use with both frequencies by the
tion, their use is ordinarily limited to special simple expedient of changing refrigerants.
applications where an experienced attendent is 16-25. Refrigerant Drying Agents. Refrig-
on duty. Ethane, methane, and ethylene are erant drying agents, called desiccants are fre-
employed to some extent in ultra-low tempera- quently employed in refrigerating systems to
ture applications, usually in the lower stage of remove moisture from the refrigerant. Some
two and three stage cascade systems. However, of the most commonly used desiccants are silica
even in these applications, it is likely that they gel (silicon dioxide), activated alumina (alu-
will be replaced in the future by Refrigerants- 13 minum oxide), and Drierite (anhydrous calcium
and 14, the latter being used only in pilot plants sulfate).Silica gel and activated alumina are
at the present time. adsorption-type desiccants and are available in
Leak detection is by soap solution only. granular form. Drierite is an absorption type
I6-Z4. Refrigerant-500. Refrigerant-500, desiccant and is available in granular form and
commonly known as Carrene 7*, isan azeo- in cast sticks.
tropic mixturet of Refrigerant- 12 (73.8% by more when mixed in precise pro-
liquids, which,
* A proprietary refrigerant of the Carrier Corpor- portions, form a compound having a boiling
ation. temperature which is independent of the boiling
f An azeotropic mixture is a mixture of two or temperatures of the individual liquids.
starving or overfeeding of the evaporator,
depending upon the direction of the load shift.
Too, the valve must be opened and closed
manually each time the compressor is cycled on
and off.

Obviously the hand expansion valve is suitable

17 for use only on large systems where an operator


is on duty and where the load on the system is

relatively constant. When automatic control is

desired and/or when the system is subject to

Refrigerant Flow frequent load fluctuations, some other type of


refrigerant flow control is required.

Controls At the present time, the principal use of the


hand expansion valve is as an auxiliary refrig-
erant control installed in a by-pass line (Fig.
17-29). It is also frequently used to control the
flow rate through oil bleeder lines (Fig. 19-12).
17-3. Automatic Expansion Valves. A
17-1. Types and Function. There are six schematic diagram of an automatic expansion
basic types of refrigerant flow controls: (1) the valve is shown in Fig. 17-2. The valve consists
hand expansion valve, (2) the automatic expan- mainly of a needle and seat, a pressure bellows
sion valve, (3) the thermostatic expansion valve, or diaphragm, and a spring, the tension of the
(4) the capillary tube, (5) the low pressure float, latter being variable by means of an adjusting
and (6) the high pressure float. screw. A screen or strainer is usually installed
Regardless of type, the function of any refrig- at the liquid inlet of the valve in order to pre-
erant flow control is twofold: (1) to meter the vent the entrance of foreign materials which may
liquid refrigerant from the liquid line into the cause stoppage of the valve. The construction
evaporator at a rate commensurate with the rate of a typical automatic expansion valve is shown
at which vaporization of the liquid is occurring in Fig. 17-3.
in the latter unit, and (2) to maintain a pressure The automatic expansion valve functions to
differential between the high and low pressure maintain a constant pressure in the evaporator
sides of the system in order to permit the refrig- by flooding more or less of the evaporator sur-
erant to vaporize under the desired low pres- face in response to changes in the evaporator
sure in the evaporator while at the same time load. The constant pressure characteristic of the
condensing at a high pressure in the condenser. valve results from the interaction of two opposing
17-2. Hand Expansion Valves. Hand ex- forces: (1) the evaporator pressure and (2) the
pansion valves are hand-operated needle spring pressure. The evaporator pressure,
valves (Fig.17-1). The rate of liquid flow exerted on one side of the bellows or diaphragm,
through the valve depends on the pressure acts to move the valve in a closing direction,
differential across the valve orifice and on the whereas the spring pressure, acting on the
degree of valve opening, the latter being manu- opposite side of the bellows or diaphragm, acts
ally adjustable. Assuming that the pressure to move the valve in an opening direction. When
differential across the valve remains the same, the compressor is running, the valve functions
the flow rate through a hand expansion valve to maintain the evaporator pressure in equi-
will remain constant at all times without regard librium with the spring pressure.
for either the evaporator pressure or the As the name implies, the operation of the
evaporator loading. valve automatic and, once the tension of the
is

The principal disadvantage of the hand spring adjusted for the desired evaporator
is

expansion valve is that it is unresponsive to pressure, the valve will operate automatically
changes in the system load and therefore must be to regulate the flow of liquid refrigerant into the
manually readjusted each time the load on the evaporator so that the desired evaporator
system changes in order to prevent either pressure is maintained, regardless of evaporator
298
REFRIGERANT FLOW CONTROLS 299

loading. For example, assume that the tension Adjusting


~
screw
of the spring is adjusted to maintain a constant
pressure in the evaporator of lOpsig. There-
after, any time the evaporator pressure tends to
fall below lOpsig, the spring pressure will Bellows or
diaphragm""?"^.
exceed the evaporator pressure causing the valve
to move in the opening direction, thereby
Needle Evaporator pressure
increasing the flow of liquid to the evaporator
and flooding more of the evaporator surface.
As more of the evaporator surface becomes
effective, the rate of vaporization increases and
Strainer

Fig. 17-2. Schematic diagram of automatic expansion


valve.

It is important to notice that the operating


characteristics of the automatic expansion valve
are such that the valve will close off tightly when
the compressor cycles off and remain closed
until the compressor cycles on again. As pre-
viously described, vaporization continues in the
i" Flare
evaporator for a short time after the com-
pressor cycles off and, since the resulting vapor
is not removed by the compressor, the pressure
in the evaporator rises. Hence, during the off
cycle, the evaporator pressure will always exceed
the spring pressure and the valve will be tightly
closed. When the compressor cycles on, the
evaporator pressure will be immediately reduced

0.078" Orifice
below the spring pressure, at which time the
valve will open and admit sufficient liquid to the
evaporator to establish operating equilibrium
f Flare between the evaporator and spring pressures.

Fig. 17-1. Small capacity hand-expansion valve.


(Courtesy Mueller Brass Company.)

the evaporator pressure rises until equilibrium


is established with the spring pressure. Should
the evaporator pressure tend to rise above the
desired 10 psig, immediately override the
it will
pressure of the spring and cause the valve to
move in the closing direction, thereby throttling
the flow of liquid into the evaporator and
reducing the amount of effective evaporator
surface. Naturally, this decreases the rate of
vaporization and lowers the evaporator pressure
until equilibrium is again established with the Fig. 17-3. Typical automatic expansion valve. (Cour-
spring pressure. tesy Controls Company of America.)
300 PRINCIPLES OF REFRIGERATION

Automatic fact, if the load on the evaporator is permitted to


- expansion fallbelow a certain level, the automatic expan-
valve
sion valve, in an attempt to keep the evaporator
pressure up, will overfeed the evaporator to the
Liquid to extent that liquid will enter the suction line and
here be carried to the compressor where it may cause
serious damage. However, in a properly
Liquid designed system, overfeeding is not likely to
from
occur, since the thermostat will usually cycle the
receiver
compressor off before the space or product
temperature is reduced to a level such that the
Vapor! load on the evaporator will fall below the
compressor critical point.
(a)
Obviously, since it permits only a small
Automatic portion of the evaporator to be filled with liquid
-expansion during periods when the load on the system is
valve
heavy, the constant pressure characteristic of the
automatic expansion valve severely limits the
capacity and efficiency of the refrigerating
system at a time when high capacity and high
efficiency are most desired. Too, because the
receiver evaporator pressure is maintained constant
throughout the entire running cycle of the com-
Liquid to
here
pressor, the valve must be adjusted for a
pressure corresponding to the lowest evaporator

Vapor 1
temperature required during the entire running
compressor cycle (see Fig. 17-5). This results in a consider-
(b)
able loss in compressor capacity and efficiency,

Fig. Operating characteristics of the auto-


17-4. since advantage cannot be taken of the higher
matic expansion valve under varying load conditions. suction temperatures which would ordinarily
(a) Heavy load conditions, (b) Minimum load exist with a full-flooded evaporator during the
conditions. early part of the running cycle.
Another disadvantage of the automatic ex-
pansion valve, which can also be attributed to
The chief disadvantage of the automatic its constant pressure characteristic, is that it

expansion valve is its relatively poor efficiency cannot be used in conjunction with a low
as compared to that of other refrigerant flow pressure motor control, since proper operation
controls. In view of the evaporator-compressor of the latter part depends on a rather substantial
relationship, it is evident that maintaining a change in the evaporator pressure during the
constant pressure in the evaporator requires that running cycle, a condition which obviously
the rate of vaporization in the evaporator be
kept constant. To accomplish this necessitates 30
severe throttling of the liquid in order to limit
the amount of effective evaporator surface when
the load on the evaporator is heavy and the heat &|15 ^
transfer capacity per unit of evaporator surface >
LU
(A
0)

is high (Fig. 17-4a). As the load on the evapora- o. «-0ff ->fr -0n->j.^0ff 0n-»f«— Off
5
tor decreases and the heat transfer capacity per
unit of evaporator surface is reduced, more and 5 10 15 20 25 30 35 40 45 50 55 60
Time in minutes
more of the evaporator surface must be flooded
with liquid if a constant rate of vaporization is Fig. 17-5. Operating characteristics of the automatic
to be maintained (Fig. 17-46). As a matter of expansion valve.
REFRIGERANT FLOW CONTROLS 301

cannot be met when an automatic expansion evaporator pressure. In this respect, the con-
valve is used as the refrigerant flow control. denser by-pass serves the same function as the
In view of its poor efficiency under heavy load cylinder by-pass type of compressor capacity
conditions, the automatic expansion valve is control. * However, unlike the cylinder by-pass,
best applied only to small equipment having the condenser by-pass does not unload the
relatively constant loads, such as domestic refrig- compressor in any way. Hence, with the con-
erators and and small, retail ice-cream
freezers denser by-pass, thereis no reduction in the work

storage cabinets. However, even in these appli- of compression or in the power requirements of
cations the automatic expansion valve is seldom the compressor. For this reason, the condenser
used at the present time, having given way to by-pass is not generally recommended as a
other types of refrigerant flow controls which means of controlling the capacity of the
are more efficient and sometimes lower in compressor.
cost. Care should be taken to connect the by-pass
Some automatic expansion valves are now condenser at a point low enough on
line to the

being employed as "condenser by-pass valves." the condenser to insure that slightly "wet"

Thermostatic

^^"""expansion valve

J
C
Fig. 17-6. Automatic expansion
valve employed as condenser
by-pass valve.
Automatic expansion valve
adjusted for minimum desired
_ Condenser
"by-pass line
evaporator pressure

^> r r\

c ")

c
As such thay are installed in a by-pass line vapor, rather than superheated vapor, is by-
between the condenser and the suction line (Fig. passed to the suction line. Superheated vapor
17-6) where they serve to regulate the flow of hot directly from the discharge of the compressor, if

gas which is by-passed from the condenser by-passed to the suction line, will cause excessive
directly into the suction line in order to prevent discharge temperatures and result in overheating
the evaporator pressure from dropping below a of the compressor and possible carbonization of
predetermined desired minimum. In such cases, the lubricating oil. On the other hand, wet
the valve is set for theminimum desired eva- vapor, that is, vapor containing small particles

porator pressure. As long as the pressure in the of liquid, will tend to reduce the operating
evaporator remains above the desired minimum, temperature of the compressor through the
the valve will remain closed and no gas is by- cooling effect produced by vaporization of the
passed from the condenser into the suction line. * The use of the condenser by-pass is common on
However, any time the evaporator tends to fall some types of automobile air conditioning units.
below the desired minimum, the by-pass valve In such cases, the condenser by-pass serves to offset
opens and permits hot gas from the condenser the high compressor capacity which accrues as a
to pass directly into the suction line in an result of the increased piston displacement at high
amount just sufficient to maintain the minimum speeds.
302 PRINCIPLES OF REFRIGERATION

Bulb pressure (Pa)

A ^- Evap, pres. (PQ 20'-21,05 psig

Spring pressure (Prf


Fig. 17-7. Illustrating operat-
ing principle of conventional

20 , -21.05psig liquid-charged thermostatic


expansion valve.

20 , -21.05psig

Remote bulb

30"-21.05psig* f 20°-21 05 psig

liquid particles in the compressor cylinder. Too, diaphragm through a capillary tube, and (4) a
vaporization of the liquid particles in the com- spring, the tension of which is usually adjustable
pressor reduces the volumetric efficiency of the by an adjusting screw. As in the case of the
compressor, which, under the circumstances, is automatic expansion valve and all other
also beneficial since it will reduce the amount of refrigerant controls, a screen or strainer is

vapor which must be by-passed from the usually installed at the liquid inlet of the valve
condenser in order to maintain the minimum to prevent the entrance of foreign material
evaporator pressure. which may cause stoppage of the valve.
17-4. Thermostatic Expansion Valves. The characteristic operation of the thermo-
Because of its high efficiency and its ready staticexpansion valve results from the inter-
adaptability to any type of refrigeration appli- action of three independent forces, viz: (1) the
cation, the thermostatic expansion valve is evaporator pressure, (2) the spring pressure, and
the most widely used refrigerant control at the (3) the pressure exerted by the saturated liquid-
present time. Whereas the operation of the vapor mixture in the remote bulb.*
automatic expansion valve is based on main- As shown in Fig. 17-7, the remote bulb of the
taining a constant pressure in the evaporator, expansion valve is clamped firmly to the
the operation of the thermostatic expansion suction line at the outlet of the evaporator,
valve based on maintaining a constant degree
is where it is responsive to changes in the tem-
of suction superheat at the evaporator outlet, a perature of the refrigerant vapor at this point.
circumstance which permits the latter control to Although there is a slight temperature differen-
keep the evaporator completely filled with refrig- tial between the temperature of the refrigerant

erant under all conditions of system loading, vapor in the suction line and the temperature
without the danger of liquid slopover into the of the saturated liquid-vapor mixture in the
suction line. Because of its ability to provide remote bulb, for all practical purposes the
full and effective use of all the evaporator sur- temperature of the two are the same and
face under all load conditions, the thermostatic therefore it may
be assumed that the pressure
expansion valve is a particularly suitable refrig- exerted by the fluid in the bulb is always the

erant control for systems which are subject to saturation pressure of the liquid-vapor mixture
wide and frequent variations in loading. in the bulb corresponding to the temperature
Figure 17-7 is a schematic diagram of a
* With some exceptions which are discussed
thermostatic expansion valve showing the
later,the fluid in the remote bulb is the refrigerant
principal parts of the valve, which are: (1) a
used in the system. Hence, the remote bulb of a
needle and seat, (2) a pressure bellows or thermostatic expansion valve employed on a Re-
diaphragm, (3) a fluid-charged remote bulb frigerant-12 system would ordinarily be charged with
which is open to one side of the bellows or Refrigerant-12.
REFRIGERANT FLOW CONTROLS 303

of the vapor in the suction line at the point of tending to open the valve is exactly equal to the
bulb contact. force tending to close the valve (P x + P2 Ps) =
Notice that the pressure of the fluid in the and the valve will be in equilibrium. The valve
remote bulb acts on one side of the bellows or willremain in equilibrium until such time that
diaphragm through the capillary tube and tends a change in the degree of suction superheat
to move the valve in the opening direction, unbalances the forces and causes the valve to
whereas the evaporator pressure and the spring move in one direction or 'the other.

pressure act together on the other side of the By careful analysis of the foregoing example
bellows or diaphragm and tend to move the itcan be seen that for the conditions described
valve in a closing direction. The operating the valve will be in equilibrium when and only
principles of the thermostatic expansion valve when the degree of superheat of the suction
are best described through the use of an vapor at the remote bulb location is 10° F,
example. which is exactly the amount required to offset
With reference to Fig. 17-7, assume that the pressure exerted by the spring. Any change
Refrigerant- 12 vaporizing in the
liquid is in the degree of suction superheat will cause the

evaporator at a temperature of 20° F so that valve to move in a compensating direction in


the evaporator pressure (PJ is 21.05 psig, the order to restore the required amount of supers
saturation pressure of Refrigerant- 12 corre- heat and reestablish equilibrium. For instance,
sponding to a temperature of 20° F. Assume if the degree of suction superheat becomes less

further that the tension of the spring is adjusted than 10° F, the pressure in the remote bulb will
to exert a pressure (Pj) of 7.41 psi, so that the be less than the combined evaporator and
total pressure tending to move the valve in the spring pressures and the valve will move toward
closing direction is 28.46 psi, the sum of Pj and the closed position, thereby throttling the flow
P% (21.05 + 7.41). If the pressure drop in the of liquid into the evaporator until the superheat
evaporator is ignored, it can be assumed that is increased to the required 10° F. On the other

the temperature and pressure of the refrigerant hand, if the superheat becomes greater than
are the same throughout all parts of the 10° F, the pressure in the remote bulb will
evaporator where a liquid-vapor mixture of the exceed the combined evaporator and spring
refrigerant is present. However, at some point pressures and the valve will move toward the
B near the evaporator outlet all the liquid will open position, thereby increasing the flow of
have vaporized from the mixture and the liquid into the evaporator until the superheat is

refrigerant at this point will be in the form of reduced to the required 10° F.
a saturated vapor at the vaporizing temperature In all cases, the amount of superheat required
and pressure. As the refrigerant vapor travels to bring a thermostatic expansion valve into
from point B through the remaining portion of equilibrium depends upon the pressure setting
the evaporator, it will continue to absorb heat of the spring. It is for this reason that the
from the surroundings, thereby becoming spring adjustment is called the "superheat
superheated so that its temperature is increased adjustment." Increasing the tension of the
while its pressure remains constant. In this spring increases the amount of superheat
instance, assume that the refrigerant vapor is required to offset the spring pressure and bring
superheated 10° from 20 to 30° F during its the valve into equilibrium. A
high degree of

travel from point B to the remote bulb location superheat is usually undesirable in that it tends

at point C. The saturated liquid-vapor mixture to reduce the amount of effective evaporator
in the remote bulb, being at the same tempera- surface. On the other hand, decreasing the
ture as the superheated vapor in the line, will spring tension reduces the amount of superheat
then have a pressure (Ps) of 28.46 psig, the required to maintain the valve in a condition of
saturation pressure of Refrigerant- 12 at 30° F, equilibrium and therefore tends to increase the
which is exerted on the diaphragm through the amount of effective surface. However, if the

capillary tube and which constitutes the total valve superheat is set too low, the valve will
force tending to move the valve in the opening lose control of the refrigerant to the extent that

direction. itwill alternately "starve" and "overfeed" the

Under the conditions just described, the force evaporator, a condition often called "hunting."
304 PRINCIPLES OF REFRIGERATION

is again established, the evaporator temperature


and pressure will be higher than before because
of the increased rate of vaporization. Further-
more, since the valve will maintain a constant
superheat of approximately 10° F, the tempera-
ture of the vapor at point C will also be higher
because of the increase in the evaporator tem-
perature, as will the temperature and pressure
of the fluid in the remote bulb.
Obviously, then, unlike the automatic expan-
sion valve, the thermostatic expansion valve
cannot be set to maintain a certain evaporator
temperature and pressure, only a constant
superheat. When a thermostatic expansion
valve used as a refrigerant control, the evapo-
is

rator temperature and pressure will vary with the


loading of the system, as described in Chapter
13. A typical internally equalized thermostatic
expansion valve is shown in Fig. 17-8.
17-5. Externally Equalized Valves. Since
Fig. 17-8. Conventional liquid-charged, internally
the refrigerant undergoes a drop in pressure
equalized thermostatic expansion valve. (Courtesy
General Controls.) because of friction as it flows through the
evaporator, the saturation temperature of the
refrigerant is always lower at the evaporator
As a general rule,thermostatic expansion valves outlet than at the evaporator inlet. When the
are adjusted for a superheat of 7° to 10° by the refrigerant pressure drop through the evaporator
manufacturer. Since this superheat setting is is relatively small, the drop in saturation tem-
ordinarily satisfactory for most applications, it perature is and therefore of little
also small
should not be changed except when absolutely consequence. However, when the pressure
necessary. drop experienced by the refrigerant in the
Once the valve is adjusted for a certain evaporator is of appreciable size, the saturation
superheat, the valve will maintain approximately temperature of the refrigerant at the evaporator
that superheat under all load conditions, outlet will be considerably lower than that
regardless of the evaporator temperature and at the evaporator inlet, a circumstance which
pressure, provided that the capacity and adversely affects the operation of the expansion
operating range of the valve are not exceeded. valve in that it necessitates a higher degree of
For instance, in the preceding example, assume suction superheat in order to bring the valve
that because of an increase in system loading into equilibrium. Since more of the evaporator
the rate of vaporization in the evaporator surface will be needed to satisfy the higher
increases to the extent that all the liquid is superheat requirement, the net effect of the
vaporized by the time the refrigerant leaves evaporator pressure drop, unless compensated
point B', rather than point B„m Fig. 17-7. The for through the use of an external equalizer,
greater travel of the vapor before reaching will be to reduce seriously the amount of
point C will cause the superheat to exceed 10° F, evaporator surface which can be used for
in which case the increased bulb pressure effective cooling.
resulting from the higher vapor temperature at For example, assume that a Refrigerant- 12
point C will cause the valve to open wider and evaporator is fed by a standard, internally
increase the flow of liquid to the evaporator, equalized thermostatic expansion valve and that
whereupon more of the evaporator surface will the saturation pressure and temperature of the
be filled with liquid so that the superheat is refrigerant at the evaporator inlet (point A) is
again reduced to the required 10° F. However, 21.05 psig and 20° F, respectively, the former
it is important to notice that when equilibrium being the evaporator pressure (P ) exerted on
x
REFRIGERANT FLOW CONTROLS 305

the diaphragm of the valve. If the valve spring reduce the over-all capacity and efficiency of the
is adjusted for a pressure (iy of 7.41 psi, a bulb system.
pressure (P3) of 28.46 psig (21.05 + 7.41) will Although an external equalizer does not
be required for valve equilibrium. reduce the evaporator pressure drop in any way,
If it is assumed that the refrigerant pressure it does compensate for it so that full and

drop in the evaporator is negligible, as in the use of all the evaporator surface can
effective
previous example, the saturation pressure and be obtained. Notice in Fig. 17-9 that the
temperature of the refrigerant at the evaporator externally equalized valve is so constructed that
outlet will be approximately the same as those the evaporator pressure (Px ) which acts on the
at the evaporator inlet, 21.05 psig and 20° F, valve diaphragm is the evaporator outlet
and the amount of suction superheat required pressure rather than theevaporator inlet
for operation of the valve will be only 10° pressure. This is accomplished by completely
(30° - 20°), as shown in Fig. 17-7. On the isolating the valve diaphragm from the evapora-
other hand, assume now that in flowing through tor inlet pressure, while at the same time per-
the evaporator, the refrigerant experiences a mitting the evaporator outlet pressure to be
drop in pressure of 10 psi, in which case the exerted on the diaphragm through a small
saturation pressure at the evaporator outlet diameter tube which is connected to the
will be approximately 11 psig (21 — 10), 10 psi evaporator outlet or to the suction line 6 to 8 in.
less than the inlet pressure. Since the saturation beyond the remote bulb location on the com-
temperature corresponding to 1 1 psig is approxi- pressor side, as shown in Fig. 17-9. An
mately 4° F, it is evident that a suction super- expansion valve equipped with an external
heat of approximately 26° F will be required to equalizer connection is shown in Fig. 17-10.
provide the 30° F suction vapor temperature Since the evaporator pressure (Pa ) exerted on
which is necessary at the point of bulb contact in the diaphragm of the externally equalized valve
order to bring the valve into equilibrium. is the evaporator outlet pressure rather than
In order to satisfy the greater superheat the evaporator inlet pressure, the effect of the
requirement, vaporization of the liquid must be evaporator pressure drop is nullified to the
completed prematurely in the evaporator extent that the degree of suction superheat
(point B' in Fig. 17-7) so that a considerable required to operate the valve is approximately
portion of the evaporator surface becomes the same as when the evaporator pressure drop
relatively ineffective. Naturally, the loss of is Notice in Fig. 17-9 that the
negligible.
effective evaporator surface will materially evaporator (outlet) pressure (Pj) exerted on the

18.38 psig

|A 21.05 psig-20'F (sat)

16psjg-15*F (sat)

2
11 psig— 16*F (saturation temp— 11*F)

1*- Super heat


C
Fig. 17-9. Schematic diagram of externally equalized thermostatic expansion valve.
306 PRINCIPLES OF REFRIGERATION

Fig. 17-10. Externally equal-


ized thermostatic expansion
valve. (Courtesy Sporlan Valve
Company.)

diaphragm is 11 psig, which, when added to Another disadvantage of the conventional


the spring pressure (Pg) of 7.41 psi, constitutes a thermostatic expansion valve is its tendency to
total pressure of 18.41 psi, tending to move the open wide and overfeed the evaporator when
valve in the closing direction. Hence, a bulb the compressor cycles on, which in many cases
pressure of 18.41 psig, corresponding to a permits liquid to enter the suction line with
saturation temperature of approximately 16° F, possible damage to the compressor. Over-
is required for equilibrium. Since the saturation feeding at start-up is caused by the fact that
temperature corresponding to the suction the evaporator pressure drops rapidly when the

vapor pressure of 11 psig is 4°F, a suction compressor is started and the bulb pressure
superheat of only 12° F (16° - 4°) is necessary remains high until the temperature of the bulb
to provide valve equilibrium. is cooled to the normal operating temperature

17-6. Pressure Limiting Valves. The pro- by the suction vapor. Naturally, because of the
pensity of a conventional liquid charged high bulb pressure, the valve will be unbalanced
thermostatic expansion valve for keeping the in the open direction during this period and
evaporator completely filled with refrigerant, overfeeding of the evaporator will occur until
without regard for the evaporator temperature the bulb pressure is reduced.
and pressure, has some disadvantages as well as Fortunately, these operating difficulties can
advantages. Although this characteristic is be overcome through the use of thermostatic
desirable in that it insures full and efficient use expansion valves which have built-in pressure
of all the evaporator surface under all condi- limiting devices. The pressure limiting devices
tions of loading, it is, at the same time, unde- act to throttle the flow of liquid to the evaporator
sirable in that it also permits overloading of by taking control of the valve away from the
the compressor because of excessive
driver remote bulb when the evaporator pressure
evaporator pressures and temperatures during rises to some predetermined maximum. Not
periods of heavy loading. only does this protect the compressor driver
REFRIGERANT FLOW CONTROLS 307

from overload during periods of heavy loading, The maximum evaporator pressure, called
it also tends to eliminate liquid flood-back to the the maximum (MOP) of the
operating pressure
compressor because of overfeeding at start-up. valve, depends on the pressure of the gas in the
The maximum operating pressure (MOP) of cartridge and can be changed simply by chang-
the expansion valve can be limited either by ing the valve cartridge. Cartridges are available
mechanical means or by the use of a gas charged for almost* any desired maximum operating
remote bulb. The former is accomplished by pressure.
placing a spring or a collapsible cartridge The operation of the spring-type pressure
between the diaphragm and the valve stem or limiting valve (Fig. 17-12) is similar to that of the
push-rods which actuate the valve pin. In the collapsible cartridge type in that the spring
collapsible cartridge-type (Fig. 17-11), the acts as a solid link between the valve diaphragm
cartridge, which with a noncondensible
is filled and the valve stem or push-rods whenever the
gas, acts as a solid link between the diaphragm pressure in the evaporator is less than the
and the valve stem as long as the evaporator spring tension. When the evaporator pressure
pressure than the pressure of the gas in
is less a point where it exceeds the tension for
rises to
the cartridge. Hence, control of the valve is which the spring is adjusted, the spring collapses
vested in the remote bulb and the valve operates and the flow of refrigerant to the evaporator is
as a conventional thermostatic expansion valve throttled until the evaporator pressure is again
as long as the evaporator pressure is less than reduced below the spring tension. Obviously,
the pressure of the gas in the cartridge. How- the maximum operating pressure of the valve
ever, when the evaporator pressure exceeds depends on the degree of spring tension, which
the cartridge pressure, the cartridge collapses, in some cases is adjustable in the field.
thereby taking control of the valve away from In addition to the overload protection
the bulb and allowing the superheat spring to afforded by pressure limiting valves, they also
throttle the valve until the evaporator pressure tend to eliminate the possibility of liquid
isreduced below the cartridge pressure, at which flood-back to the compressor at start-up. The
time the cartridge will again act as a solid link, fact that the evaporator pressure must be
thereby returning control of the valve to the reduced below the MOP of the valve before the
remote bulb. valve can open delays the valve opening

Pressure
charged
mechanical
cartridge

Fig. 17-11. Cartridge-type


pressure limiting valve. (Cour-
tesy Alco Valve Company.)
308 PRINCIPLES OF REFRIGERATION

Ft j, 17-12. Spring-type pressure limiting valve, (Courtesy Detroit Contrail Division, American Rldiltor
and Standard Sanitary Corporation.)

sufficiently to permit the suction vapor lo cool However, whereas in the liquid charged valve,
the remote bulb and reduce the bulb pressure the remote bulb charge is sufficiently large to
before the valve opens. Hence, the valve does assure that a certain amount of liquid is always
not open wide and overfeed the evaporator present in the remote bulb, in the gas charged
urtien the compressor is started. valve the bulb charge is limited so that at
The "pull-down" characteristics of a pressure some predetermined bulb temperature all the
limiting expansion valve as compared to those liquid will have vaporised and the bulb charge
of a conventional expansion valve is shown in will be in the form of a saturated vapor. Once
Fig. 17-13. the bulb charge is in the form of a saturated
17-7. G^s-Charged Expansion Valves. The vapor, further increases in the bulb temperature
gas-charged thermostatic expansion valve is (additional superheat) will have very little effect

essentially a pressure limiting valve, the pressure on the bulb pressure. Hence, by limiting the
limiting characteristic of the valve being a amount of charge in the remote bulb, the
function of its limited bulb charge. maximum pressure wiiich can be exerted by the
The remote bulb of the gas-charged expansion remote bulb is also limited. As in the case of
valve, like that of the liquid charged valve. Is mechanical pressure limiting devices, limiting
always charged with the system refrigerant. the pressure exerted by the remote bulb also
REFRIGERANT FLOW CONTROLS 309

limits the evaporator pressure, since valve the gas charged thermostatic expansion valve
equilibrium is established only when the bulb provides the same compressor overload and
pressure (P3) is equal to the sum of the evapora- flood-back protection as do mechanical pressure
tor pressure (Pj) and the spring pressure CP2)> limiting valves.
the latter pressure being constant. Therefore, Since the evaporator pressure is limited
any time the evaporator pressure exceeds the by limiting the bulb pressure (charge),
indirectly
maximum operating pressure of the valve, the any change in the superheat setting (spring
sum of the evaporator and spring pressures pressure) will cause the maximum evaporator
will always exceed the bulb pressure and the pressure (the MOP
of the valve) to change.
valve will be closed. Since the bulb pressure (P3) is always equal to
For example, suppose the system illustrated the evaporator pressure (Px) plus the spring
in Fig. 17-7 is equipped with a gas charged pressure (Pa), increasing the superheat setting
thermostatic expansion valve having a MOP (Pa) will decrease the MOP (Px ). Likewise,
of 25 psig with a superheat setting of 10° F, in decreasing the superheat setting wiH increase
which case the bulb charge will be so limited the maximum operating pressure of the valve.
that it will become 100% saturated vapor when In view of the limited bulb charge, some
the bulb temperature reaches the saturation precautions must be observed when installing a
temperature corresponding to 32.41 psig, the gas charged expansion valve. The valve body
latter being the sum of the maximum evaporator must be in a warmer location than the remote
pressure (25 psig) and the spring pressure bulb and the tube connecting the remote bulb
equivalent to 10° F of superheat (7.41 psi). to the power head must not be allowed to
Once the bulb reaches this temperature, touch a surface colder than the remote bulb;
additional superheating of the suction vapor will otherwise the bulb charge will condense at the
have very on the bulb pressure and
little effect coldest point and the valve will become inopera-
therefore will not cause the valve to open wider. tive because of the lack of liquid in the remote
In this instance, any time the evaporator bulb (Fig. 17-14). Too, care should be taken to
pressure exceeds 25 psig, the sum of the so locate the remote bulb that the liquid does not
evaporator and spring pressures will exceed the drain from the bulb by gravity.
maximum bulb pressure of 32.41 psig and the 17-8. Importance of Pressure Limiting
valve will be closed. On the other hand, any Valves. The importance of pressure limiting
time the evaporator pressure is below 25 psig, valves is readily understood when it is recognized
thesum of the evaporator and spring pressures that many refrigeration systems are subject to
will be less than the maximum bulb pressure occasional "pull-down" loads which are sub-
and the bulb will be in control of the valve. stantially greater than the average system load
The valve will then respond normally to any under normal operation. Since evaporator
changes in the suction superheat. pressures and temperatures are abnormally
Because of its pressure limiting characteristics, high during these pull-down periods, the

1 1

Low side pressure after


long shut down period

Fig. Chart showing


17-13.
comparative performance of
the gas charged and liquid -— Liquid charged valve
charged valves during "pull-
Motor
down." (Courtesy Detroit
overloaded
Controls Division, American
Gas char
Radiator and Standard Sanitary
Corporation.) /
/ jed v alve

\ A 1
K A A <Y A !\

\ ,''
w \6 ^pW \
310 PRINCIPLES OF REFRIGERATION
is not required, it is usually more practical to
Remote bulb loses limit the evaporator pressure to some maximum
control if liquid reasonably near the average evaporator pressure
condenses at
these points under normal operating conditions and then
motor the compressor accordingly. This will
ordinarily result in the use of a smaller size
motor, thereby effecting a saving in both the
initial and operating costs of the system.

Qrop of liquid must


For this reason, pressure limiting expansion
1 be in remote bulb valves of all types are widely used at the present
for proper control time, particularly in air conditioning appli-
cations. As a general rule, a pressure limiting
Fig. 17-14. Gas-charged thermo expansion valve.
(Courtesy Alco Valve Company.)
expansion valve is selected to have a MOP
approximately 5 to 10 psi above the average
evaporator pressure encountered at normal
capacity and horsepower requirements of the loading of the system. In ordering such a valve,
compressor are greatly increased, which often the desired MOP must be stated.
results in overloading of the compressor driver. 17-9. Cross-Charged Expansion Valves.
Obviously, two solutions to the problem are Although expansion valves having a bulb
possible. One is to increase the size of the charged with the system refrigerant are suitable
compressor driver so that it has sufficient for most medium and high temperature
power to carry the load during the overload applications, they are not ordinarily satisfactory
period. The other is to limit the maximum for low temperature applications. The reason
evaporator pressure in order to avoid excessive for this becomes evident upon examination of
compressor loading. Which is the better the pressure-temperature relationship of any
solution depends upon the requirements of the common refrigerant.
particular installation. In applications where A pressure-temperature curve for Refrigerant-
rapid reduction of the space or product tem- 12 is shown in Fig. 17-15. Notice that the
perature is former solution is the
desirable, the change in pressure per degree of temperature
one recommended. However, since motoring change decreases considerably as the tempera-
the compressor for an occasional peak load ture of the refrigerant decreases. Therefore,
condition unnecessarily increases both the the amount of superheat required to cause a
initial and operating costs of the system, in given increase in remote bulb pressure is much
applications where rapid reduction of the load greater at low temperatures than at high

/o 1 /
65 -5.5 • /

60
r*
{5 psi
55 i

S
5
50
45
40
M
<%
ys

Fig. 17-15.The effect of tem- $\


35
perature range on valve super- 1
lao %if\
heat.
a 25 &f\
t*-13*-»l3
20
I ' 5 psi
15
10
5

-40* -30* -20* -10* 10 20 30 40 50 60


Temperature (*F)
REFRIGERANT FLOW CONTROLS 311

temperatures. For example, notice in Fig. valve into equilibrium. Curve C in Fig. 17-16
17-15, that at a temperature of 45° F, a tem- indicates the bulb temperature necessary to
perature change of only approximately 5.5° F bring the valve into equilibrium if the superheat
will cause a pressure change of 5 psi, whereas spring adjusted for a pressure of 5 psi.
is
at —20° F, a temperature change of approxi- Because of the higher superheat requirement at
mately 13° F is required for a 5 psi change in high evaporator temperatures, the cross-charged
pressure. Obviously, then, when the expansion valve has a pressure limiting effect which affords
valve bulb is charged with the system refrigerant, a certain amount of protection against motor
the amount of suction superheat necessary to overload and compressor flood-back at start-up.
actuate the valve becomes excessive at low Cross-charged expansion valves are by no
temperatures, with the result that much of the means limited to low temperature applications.
evaporator surface becomes ineffective. Hence, They are also used extensively in commercial
for low temperature applications, good practice applications where pressure limiting is desired
prescribes the use of an expansion valve having and where their varying superheat characteristic
a bulb charged with some fluid other than the is not objectionable. In such applications, they

70
65
60
y
s/
55 A 1>•

50
9;

ST 45 i&rtx
I 40 £&)>&* 9

Fig. 17-16. Operating charac-


teristics of cross-charged ther-
S 35
rl <*
S >f\r
10'
mostatic expansion valve.
30
25
m ^0 / Apc
btt 8«

20 /fl"
cut
^ Pr^^6'
xgi

*~2*
15
10

-40* -30* -20* -10*



10 20 30 40 50 60
Temperature (*F)

system refrigerant, usually one which has a are often preferable to the gas-charged valve
boiling point somewhat below that of the system because they have tendency to "hunt" and
less
refrigerant, so that the pressure change in the because the remote bulb location is not so
bulb per degree of suction superheat is more critical. However, cross-charged valves are not
substantial at the desired operating temperature suitable for air conditioning installations since
of the valve. This will permit operation of the these systems require a constant superheat for
valve with a normal amount of suction super- optimum performance.
heat. a cross-charged valve will perform
Since
Expansion valves whose bulbs are charged satisfactorily only within a given temperature
with fluids other than the system refrigerant range, several different types of cross charges
are called "cross-charged" valves because the are required for the various temperature ranges.
pressure-temperature curve of the fluid crosses Naturally, in ordering a cross-charged valve,
the pressure-temperature curve of the system the desired operating temperature range must
refrigerant. Notice in Fig. 17-16 that the be stated so that a valve with the proper cross
pressure-temperature curve of the bulb fluid charge can be selected.
(curve B) is somewhat flatter than that of the 17-10. Multioutlet Valves and Refrigerant
system refrigerant (curve A), so that as the Distributors. When an evaporator has more
evaporator pressure increases, a greater amount than one refrigerant circuit, the refrigerant
of suction superheat is required to bring the from the expansion valve is delivered to the
312 PRINCIPLES OF REFRIGERATION

Fig. 17-17. Multioutlet thermo-


static expansion valve. (Cour-
tesy Alco Valve Company.)

various evaporator circuits through a refrigerant


distributor. In some instances the refrigerant
distributor is an integral part of the valve itself.
In others, it is a completely separate unit. In
either case, important that the design of the
it is

distributor such that the liquid-vapor mixture


is

leaving the valve be evenly distributed to all of


the evaporator circuits, if peak evaporator

—I performance is to be expected.
A multioutlet expansion valve incorporating
a refrigerant distributor is shown in Fig. 17-17.
Pressure datum line Since expansion and distribution of the refrig-
erant occur simultaneously within the valve
itself, this, along with the carefully proportioned
passages through the radial distributor, assures
even distribution of a homogeneous mixture of
liquid and vapor to each of the several evapora-
tor circuits.
Four different types of refrigerant distributors
are in common use at the present time: (1) the
venturi type, (2) the pressure drop type, (3) the
centrifugal type, and (4) the manifold type.
Any of these distributors can be used with any
A ) Smooth contoured standard, single-outlet expansion valve.
approech
A venturi-type distributor, along with its
Divergent section
Gradual reduction in
flow characteristics, is shown in Fig. 17-18.
velocity-contour flow
Little turbulence
This type of distributor utilizes the venturi
Pressure recovery-low pressure drop principle of a large percentage of pressure
Fig. 17-18. Flow through a venturi-type distributor. recovery, and depends on contour flow for
(Courtesy Alco Valve Company.) equal distribution of the liquid-vapor mixture
REFRIGERANT FLOW CONTROLS 313

Fig. 17-19. Pressure drop-type


Retainer Nozzle Body Tubing
ring
distributor. (Courtesy Sporlan
Exploded View
Valve Company.)
(a)

(b)

to each of the evaporator circuits. This distri- the capacity of the distributor, and the pressure
butor provides a minimum of turbulence and a drop prevents separation of flash gas from the
minimum over-all pressure loss, the pressure liquid, causing a homogeneous mixture of
loss being confined only to wall frictional losses. liquid and vapor to pass through the distributor.
It may be mounted in any position. A centrifugal-type distributor is illustrated in
A pressure drop-type distributor and its flow This type of distributor depends
Fig. 17-20.
pattern is shown in Fig. 17-19. The following upon a high entrance velocity to create a
description is condensed from the manufacturer's
catalog data.* External equalizer line
The distributor consists of a body or housing,
Centrifugal
the outlet end of which is drilled to receive the
distributor*
tubes connecting the distributor to the evapora-
tor. The inlet end is recessed to receive an
interchangeable nozzle which is held in place by
a snap ring. The refrigerant, after leaving the
expansion valve, enters the distributor inlet and
passes through the nozzle. The nozzle orifice is
sized to produce a pressure drop which increases
the velocity of the liquid, thereby homogen-
eously mixing the liquid and vapor and elimin-
ating the effect of gravity. The nozzle orifice
centers the flow of refrigerant so that it impinges
squarely on the center of the conical button
inside the distributor body. The outlet passage
holes are accurately spaced around the base of
the conical button so that the mixture coming -Evaporator
off the button divides evenly as it enters these
holes. The orifice size of the nozzle determines Fig. 17-20. Single outlet thermostatic expansion
valve and centrifugal-type distributor. (Courtesy
* Sporlan Valve Company. Alco Valve Company.)
314 PRINCIPLES OF REFRIGERATION

Since there is enough liquid in a liquid


charged expansion valve to insure that control
of the valve will remain with the bulb under
all conditions, a liquid charged thermostatic

expansion valve can be installed in any position


(power head up, down, or sideways), either
inside or outside of the refrigerated space,
without particular concern for the relative
temperatures of the valve body and remote bulb.
On the other hand, gas charged valves must be
o a o o installed so that the valve body is always warmer
than the remote bulb, preferably with the power
head up.
With the exception of the manifold-type

W. t=d/ distributor, when a refrigerant distributor


used, the valve should be installed as close to
the distributor as possible.
is

Fig. 17-21. Single outlet thermostatic expansion


valve and manifold-type distributor. (Courtesy Alco
Valve Company.)

swirling effect which maintains a homogeneous


mixture of the liquid and flash gas and which
distributes the mixture evenly to each of the
evaporator tubes.
A manifold or Weir type distributor is

illustrated in Fig. 17-21. This type of distri-

butor depends upon level mounting and low


entrance velocities to insure even distribution
of the refrigerant to the evaporator circuits.
A baffle is often installed in the header in order
to minimize the tendency to overfeed the
evaporator circuits directly in front of the
heater inlet connection. Too, an elbow installed
between the expansion valve and the header
inlet will usually reduce the refrigerant velocity
and help prevent unequal distribution of the
refrigerant to the evaporator circuits.
Some typical distributor applications are
shown in Fig. 17-22.
17-1 1. Expansion Valve Location. For best
performance, the thermostatic expansion valve
should be installed as close to the evaporator as
possible. With the exception of a refrigerant
distributor, where one is used, there should be
no restrictions of any kind between the eva-
porator and the expension valve. When it is
necessary to locate a hand valve on the outlet
side of the valve, the hand valve should have a Fig. 17-22. Illustrating applications of refrigerant
full sized port. distributors. (Courtesy Sporlan Valve Company.)
REFRIGERANT FLOW CONTROLS 315

17-12. Remote Bulb Location. To a large liquid will flood back to the compressor when
extent, the performance of the thermostatic the compressor starts. When the remote bulb
expansion valve depends upon
the proper is located inside the refrigerated space, the
location and remote bulu.
installation of the temperature between the fixture
difference
When an external remote bulb is used (mounted temperature and the evaporator temperature is
on the outside, rather than the inside of the not usually large enough to affect adversely
refrigerant piping) as is normally the case, the expansion valve operation. However, when it is
bulb should be clamped firmly (with metal necessary to locate the bulb outside the refrig-
clamps) to a horizontal section of the suction erated space, both the bulb and the suction
line near the evaporator outlet, preferably linemust be well insulated from the surround-
inside the refrigerated space. ings. The insulation must be nonhydroscapoic
Since the remote bulb must respond to the and must extend at least 1 ft or more beyond
temperature of the refrigerant vapor in the the bulb location on both sides of the bulb.
suction line, it is essential that the entire length Care must be taken also to locate the thermal
of the remote bulb be in good thermal contact bulb at least 1$ ft from the point where an

"E=
External bulb on small suction line

Fig. 17-23. Usual location of


expansion valve remote bulb.
(Courtesy Alco Valve Com-
pany.)

External bulb on large suction line

with the suction line. When an iron pipe or uninsulated suction line leaves a refrigerated
steel suction line is used, the suction line should fixture. When the bulb is located on the
be cleaned thoroughly at the point of bulb suction line too close to the point where the line
location and painted with aluminum paint in leaves the refrigerated space, heat conducted
order to minimize corrosion. On suction lines along the suction line from the outside may
under f in. OD, the remote bulb is usually cause the bulb pressure to increase to the
installed on top of the line. For suction lines extent that the valve will open and permit

f in. OD and above, a remote bulb located in liquid to fill the evaporator during the off cycle.
a position of 4 or 8 o'clock (Fig. 17-23) will On air conditioning applications, when suit-

normally give satisfactory control of the valve. able pressure limiting valves are employed, the
However, since this is not true in all cases, the remote bulb may be located either outside or
optimum bulb location is often best determined inside the air duct, but always out of die direct

by trial and error. air stream. On brine tanks or water coolers,


important also that the remote bulb be
It is the bulb should always be located below the
so located that it is not unduly influenced by liquid level at the coldest point.
temperatures other than the suction line tem- Whenever the bulb location is such that there
perature, particularly during the compressor off isa possibility that the valve may open on the off
cycle. If the temperature of the bulb is per- cycle, a solenoid valve should be installed in the

mitted to rise substantially above that of the liquid line directly in front of the expansion
evaporator during the off cycle, the valve will valve so that positive shut-off of the liquid
open allowing the evaporator to become filled during the off cycle is assured. The system then
with liquid refrigerant, with the result that operates on a pump-down cycle.
316 PRINCIPLES OF REFRIGERATION

the remote bulb must be located on the evapora-


tor side of a liquid-suction heat exchanger.
Several of the more common incorrect
remote bulb applications are shown in Figs.
17-24 through 17-26, along with recommended
corrections for piping and remote bulb locations
to avoid these conditions. In Fig. 17-24, liquid
can trap in the suction evaporator
line at the
outlet, causing the loss of operating superheat
and resulting in irregular operation of the valve
due to alternate drying and filling of the trap.
If valve operation becomes too irregular, liquid
may be blown back to the compressor by the gas
which forms in the evaporator behind the trap.
Evaporator Figure 17-25 illustrates the proper remote
Multi-outlet bulb location to avoid trapped oil or liquid from
thermo
affecting the operation of the expansion valve
expansion
valve when the suction line must rise at the evaporator
outlet. Liquid or oil accumulating in the trap
during the off cycle will not affect the remote
bulb and can evaporate without "slugging" to
the compressor when the compressor is started.
This piping arrangement is often used deliber-
ately on
large installations to avoid the possi-
bilityof liquid slugging to the compressor.
Figure 17-26 illustrates the incorrect appli-
cation of the remote bulb on the suction header
of an evaporator. With poor air circulation
through the evaporator, liquid refrigerant can
pass through some of the evaporator circuits
without being evaporated and without affecting
the remote bulb, a condition which can cause
flood back to the compressor. The correct
remote bulb location is shown by the dotted
lines. However, correcting the remote bulb
Multi-outlet
location in this instance will do nothing to
thermo
expansion
, Thermo expansion valve
valve
External equalizer line
Correct

(b)

Fig. 17-24. (a) Remote bulb location shown trapped,


(b) Remote bulb location shown free draining. (Cour-
tesy Alco Valve Company.)

Under no circumstances should a remote bulb Evaporator


ever be located where the suction line is trapped. Liquid trap-
As short as possible-
Any accumulation of liquid in the suction line
at the point where the remote bulb is located
Fig* 17-25. Recommended remote bulb location and
will cause irregular operation (hunting) of the
schematic piping for rising suction line. (Courtesy
expansion valve. Except in a few special cases, Alco Valve Company.)
REFRIGERANT FLOW CONTROLS 317

improve the poor air distribution, but will only


prevent flood back to the compressor. The air
distribution must be approached as a separate
problem.
Since a trapped or partially trapped suction
line at the remote bulb location will cause poor
expansion valve performance, care should
always be taken to arrange the suction piping
from the evaporator so that oil and liquid will
be drained away from the remote bulb location
by gravity.
Location of the remote bulb on a vertical
section of suction line should be avoided when- Fig. 17-27. Remote bulb well and suggM&ad piping.
ever possible. However, in the event that no (Courtesy Alco Valve Company.)
other location is possible, the bulb should be
installed well above the liquid trap on a suction
riser. because of the pressure-temperature relationship
In some instances, it is necessary or desirable of the refrigerant, the maximum permissible
to employ a remote bulb well, so that the remote pressure drop will vary with the individual
bulb is in effect installed inside of the suction line refrigerant and with the operating temperature
(see Fig. 1 7-27). As a general rule, a remote bulb range. For example, with regard to Refrigerant-
well should be used when low superheats are 12, the permissible pressure drop is approxi-
required or when the remote bulb is likely to be mately 2.5 psi when the evaporator temperature
influenced by heat conducted down the suction is about 40° F, whereas for evaporator tempera-

line from a warm space. It is desirable also in tures below 0°F, the permissible evaporator
installations where the suction line is very short pressure drop is only approximately 0.5 psi.

or where the size of the suction line exceeds External equalizers are required also whenever
2\ in. OD. multioutlet expansion valves or refrigerant dis-
17-13. External Equalizer Location. In tributors are employed, regardless of whether or
general, an external equalizer should be used in not the pressure drop through the evaporator is
any case where the pressure drop through the excessive. In such installations, the external
evaporator is sufficient to cause a drop in the equalizer is required in order to compensate for
saturation temperature of the refrigerant in the pressure drop suffered by the refrigerant in
excess of 2° F at evaporator temperatures above passing through the distributor.
0°F or in excess of approximately 1°F at It should be emphasized at this point that the
evaporator temperatures below 0° F. Naturally, refrigerant pressure loss in a refrigerant distri-
butor in no way affects the capacity or efficiency
.Thermo expansion valve of the system, provided that the expansion valve
Equalizer line has been properly selected (see Section 17-14).
In any system, with or without a distributor, the
refrigerant must undergo a drop in pressure
Faulty remote
bulb location
between the high and low pressure side of the
system. On systems without distributors, all of
this pressure drop is taken across the valve.
When a distributor is employed, a portion of the
pressure drop occurs in the distributor and the
remainder across the valve. The total pressure
Evaporator Correct remote drop and refrigerating effect are the same in
bulb location
either case.

Fig. 17-26. Correct remote bulb location on "short This is not true, however, when the pressure
circuiting" evaporator to prevent "flood back." loss is in the evaporator itself. As described in
(Courtesy Alco Valve Company.) Section 8-9, any refrigerant pressure drop in the
318 PRINCIPLES OF REFRIGERATION

evaporator tends to reduce the capacity and the first two factors determine the required
efficiency of the system. Hence, evaporators liquid flow rate through the valve, whereas the
having excessive pressure drops should be 'latter determines the size orifice required to
avoided whenever possible. deliver the desired flow rate, the flow rate
As a general rule, the external equalizer through the orifice being proportional to the
connection is made on the suction line 6 to 8 in. pressure differential across the valve.
beyond the expansion valve bulb on the com- The pressure difference across the valve can
pressor side.However, in applications where never be taken as the difference between the
the external equalizer is used to offset pressure suction and discharge pressures as measured at
drop through a refrigerant distributor and the the compressor. When these two pressures are
pressure drop through the evaporator is not used as a basis for determining the pressure
excessive, the external equalizer may be con- difference across the expansion valve, an allow-
nected either to one of the feeder tubes or to one ance must always be made for the pressure losses
of the evaporator return bends at approximately which accrue between the expansion valve and
the midpoint of the evaporator. When the the compressor on both the low and high
external equalizer connected to a horizontal
is pressure sides of the system. This includes the
line, it should be installed on top of the line in refrigerant distributor when one is used.
order to avoid the drainage of oil or liquid into When the available pressure difference across
the equalizer tube. the expansion valve has been determined, a
17-14. Expansion Valve Rating and Selec- valve should be selected from the manufacturer's
tion. Before the proper size valve can be rating table which has a capacity equal to or
selected, a decision must be made as to the exact slightly in excess of the system capacity at the
type of valve desired with respect to bulb charge, system design operating conditions.
pressure limiting, the possible need for an
external equalizer, and the size of the valve inlet Example 17-1. From Table R-20, select the
proper size expansion valve for a Refrigerant- 1
and outlet connections. Obviously, the nature
system, if the desired capacity is 8 tons at a 20° F
and conditions of the application will determine
eviporator temperature and the available
the type of bulb charge and also whether or not a
pressure drop across the expansion valve is
pressure limiting valve is needed. An externally approximately 65 psi.
equalized valve should be used whenever the
pressure drop through the evaporator is sub- Solution. From Table R-20, select expansion
and/or when a refrigerant distributor is
stantial valve Model #TJL1100F, which has a capacity
employed. The size of the inlet and outlet of 8.1 tons at a 20° F evaporator temperature
connections of the valve should be equal to
when the pressure drop across the valve is 60
psi. The letters in the model number indicate
those of the liquid line and evaporator, respec-
valve type and refrigerant.
tively. A slight reduction in size at the evapora-
tor inlet is permissible. Thermostatic expansion valves will not
Once a decision has been made on all of the operate satisfactorily at less than 50% of their
foregoing, the proper size valve can be selected rated capacity. Therefore, when the load on the
from the manufacturer's catalog ratings. Table system is likely to fall below 50% of the design
R-20 is a typical thermostatic expansion valve load, the evaporator should be split intotwo or
rating table. The expansion valves are rated in more separate circuits, with each circuit being
tons of refrigerating capacity (or Btu/hr) at fed by an individual expansion valve. With this
various operating conditions. Normally, valve arrangement it is possible to cycle out portions
ratings are based on a condensing temperature of the evaporator as the load on the system fall
of 100° F with zero degrees of subcooling, but off so that the load on any one expansion valve
with solid liquid approaching the valve. never drops below 50 %
of the design capacity of
In order to select the proper size valve from the valve.
the rating table, the following data should be 17-15. Capillary Tubes. The capillary tube
known: (1) the evaporator temperature, (2) the is the simplest of all refrigerant flow controls,
system capacity in tons, and (3) the available consisting merely of a fixed length of small
pressure difference across the valve. In general, diameter tubing installed between the condenser
-

REFRIGERANT FLOW CONTROLS 319

Accumulator
and the evaporator in place of the conventional
liquid line (Fig. 17-28). Because of the high
from its length and
ft!
frictional resistance resulting
small bore, the capillary tube acts to restrict or
D
to meter the flow of liquid from the condenser c
to the evaporator and also to maintain the J
required operating pressure differential between
thesetwo units.
C
For any given tube length and bore, the
resistance of the tube is fixed or constant so that
Capillary tube bonded
the liquid flow rate through the tube at any one to suction line for

time is always proportional to the pressure heat exchange

differential across the tube, said pressure differ-


ential being the difference between the vaporizing
and condensing pressures of the system. Like-
wise, the greater the frictional resistance of the
tube (the longer the tube and/or the smaller the
bore), the greater is the pressure differential
required for a given flow rate.
Since the capillary tube and the compressor
are connected in series in the system, it is
evident that the flow capacity of the tube must of Strainer
Fig. 17-28. Capillary tube system.
necessity be equal to the pumping capacity of
the compressor when the latter is in operation.
Consequently, if the system is to perform
efficiently and balance out at the design operat- perature. Hence, the net effect of too much
and bore of the tube restriction in the capillary tube is to lower the
ing conditions, the length
must be such that the flow capacity of the tube suction pressure and raise the condensing

at the design vaporizing and condensing pres- pressure. Since both these conditions tend to

sures is exactly equal to the pumping capacity increase the flow capacity of the tube and, at the

of the compressor at these same conditions. same time, decrease the pumping capacity of the
In the event that the resistance of the tube is compressor, it is evident that the system will

such that the flow capacity of the tube is either eventually establish equilibrium at some operat-
greater than or less than the pumping capacity ing conditions where the capacity of the tube
of the compressor at the design conditions, a and the capacity of the compressor are exactly
balance will be established between these two the same. In this instance, the point of balance
components at some operating conditions other will be at a lower suction pressure and a higher

than the system design conditions. For example, condensing pressure than the system design
if the resistance of the tube is too great (tube too pressures. Too, since the capacity of the com-
pressor is reduced at these conditions, the over-
long and/or bore too small), the capacity of the
tube to pass liquid refrigerant from the con- all system capacity will be less than the design
denser to the evaporator will be less than the capacity.

pumping capacity of the compressor at the On the other hand, when the tube does not
design conditions, in which case the evaporator have enough resistance (tube too short and/or
will become starved while the excess liquid will bore too large), the flow capacity of the tube
back-up in the lower portion of the condenser at will be greater than the pumping capacity of the

the entrance to the capillary tube. Naturally, compressor at the design conditions, in which
starving of the evaporator will result in lowering case overfeeding of the evaporator will result
the suction pressure, whereas the build-up of with the danger of possible liquid flood-back to
liquid in the condenser will result in a reduction the compressor. Also, there will be no liquid
of the effective condensing surface and, con- seal in thecondenser at the entrance tc the tube
sequently, an increase in the condensing tem- and, therefore, uncondensed gas will be allowed
320 PRINCIPLES OF REFRIGERATION
to enter the tube along with the liquid. Ob- the compressor driver may also result. How-
viously, the introduction of latent heat into the ever, of more importance is the fact that all of
evaporator in the form of uncondensed gas will the excess liquid in the condenser will pass to
have the effect of reducing the system capacity. the evaporator during the off cycle. Being at the
Furthermore, because of the excessive flow rate condensing temperature, a substantial amount
through the tube, the compressor will not be of such liquid will cause the evaporator to warm
able to reduce the evaporator pressure to the up rapidly, thereby causing defrosting of the
desired low level. evaporator and/or short cycling of the com-
From the foregoing it is evident that the pressor. Moreover, where a considerable
design of the capillary tube must be such that amount of liquid enters the evaporator during
the flow capacity of the tube is identical to the the off cycle, flood-back to the compressor is
pumping capacity of the compressor at the likely tooccur when the compressor cycles on.
system design conditions. It is evident also that Other than its simple construction and low
a system employing a capillary tube will operate cost, the capillary tube has the additional
at maximum efficiency only at one set of operat- advantage of permitting certain simplifications
ing conditions. At all other operating con- in the refrigerating system which further reduces
ditions, the efficiency of the system will be some- manufacturing costs. Because the high and low
what less than maximum. However, it should pressure equalize through the capillary tube
be pointed out that the capillary tube is self- during the off cycle, the compressor starts in an
compensating to some extent and, if properly "unloaded" condition. This allows the use of a
designed and applied, will give satisfactory low starting torque motor to drive the com-
service over a reasonable range of operating pressor; otherwise a more expensive type of
conditions. Normally, as the load on the system motor would be required. Furthermore, the
increases or decreases, the flow capacity of the small and critical refrigerant charge required by
capillary tube increases or decreases, respec- the capillary tube system results not only in
tively, because of the change in condensing reducing the cost of the refrigerant but also in
pressure which ordinarily accompanies these eliminating the need for a receiver tank. Natu-
changes in system loading. rally, all these things represent a substantial
The capillary tube differs from other types of savings in the manufacturing costs. Thus,
refrigerant flow controls in that it does not close capillary tubes are employed almost universally
off and stop the flow of liquid to the evaporator on all types of domestic refrigeration units, such
during the off cycle. When the compressor as refrigerators, freezers, and room coolers.
cycles off, the high and low side pressures Many used also on small commercial
are
equalize through the open capillary tube and packaged units, particularly packaged air con-
any residual liquid in the condenser passes to ditioners.
the low pressure evaporator where it remains Capillary tubes should be employed only on
until the compressor cycles on again. For this those systems which are especially designed for
reason, the refrigerant charge in a capillary tube their use. They are best applied to close-coupled,
system is critical and no receiver tank is em- packaged systems having relatively constant
ployed between the condenser and the capillary loads and employing hermetic motor-com-
tube. In all cases, the refrigerant charge should pressors. Specifically, a capillary tube should
be the minimum which will satisfy the require- not be used in conjunction with an open type
ments of the evaporator and at die same time compressor. Because of the critical refrigerant
maintain a liquid seal in the condenser at the charge, an open type compressor may lose suffi-
entrance to the capillary tube during the latter cient refrigerant by seepage around the shaft
part of the operating cycle. Any refrigerant in seal to make the system inoperative in only a
excess of this amount will only back up in the very short time.
condenser, thereby increasing the condensing The use of capillary tubes on remote systems
pressure which, in turn, reduces the system (compressor located some distance from the
efficiency and tends to unbalance the system by evaporator) should also be avoided as a general
increasing the flow capacity of the tube. If the rule. Such systems are very difficult to charge
overcharge is sufficiently large, overloading of accurately. Furthermore, because of the long
REFRIGERANT FLOW CONTROLS 321

liquid and suction lines, a large charge of refrig- gradual expansion of the liquid as its pressure is
erant is required, all of which concentrates in reduced, seriously reduces the flow capacity of
the evaporator during the off cycle. Serious the tube. When the tube is not bonded to the
flood-back to the compressor is likely to occur suction line, the tube must be shortened suffi-
at start-up unless an adequate designed accumu- ciently to offset the throttling action of the vapor
lator is installed in the suction line. in the tube.
Condensers designed for use with capillary 17-16. Flooded Evaporator Control. Re-
tubes should be so constructed that liquid drains frigerant flow controlsemployed with flooded
freelyfrom the condenser into the capillary tube evaporators are usually of the float type. The
in order to prevent the trapping of liquid in the float control consists of a buoyant member
condenser during the off cycle. Any liquid (hollow metal or pan) which is
ball, cylinder
trapped in the condenser during the off cycle responsive to refrigerant liquid level and which
will evaporate and pass through the tube to the acts to open and close a valve assembly to admit
evaporator in the vapor state rather than in the more or less refrigerant into the evaporator in
liquid state. The subsequent condensation of accordance with changes in the liquid level in
this vapor in the evaporator will unnecessarily the float chamber. The float chamber may be
add latent heat to the evaporator, thereby located on either the low pressure side or high
reducing the capacity of the system. pressure side of the system. When the float is
Too, the diameter of the condenser tubes located on the low pressure side of the system,
should be kept as small as is practical so that a the float control is called a low pressure float
minimum amount of liquid backed-up in the control. When the float is locatedon the high
condenser at the tube inlet will cause a maxi- pressure side of the system, the float control is
mum increase in the condensing pressure and known as a high pressure float control.
therefore a maximum increase in the flow The principal advantage of the flooded evap-
capacity of the tube. orator lies in the higher evaporator capacity
Evaporators intended for use with capillary and efficiency which is obtained therefrom.
tubes should provide for liquid accumulation at With flooded operation, the refrigerant in all
the evaporator outlet in order to prevent liquid parts of the evaporator is predominately in the
flood-back to the compressor at start-up (Fig. liquid state and a high refrigerant side tube
17-28). The function of the accumulator is to coefficient isproduced, as compared to that
absorb the initial surge of liquid from the obtained with the dry-expansion type evaporator
evaporator as the compressor starts. The liquid wherein the refrigerant in the evaporator is
then vaporizes in the accumulator and returns predominately in the vapor state, especially in
to the compressor as a vapor. To expedite the the latter part of the evaporator. For this
return of oil to the compressor crankcase, liquid reason, float controls (flooded evaporators) are
from the evaporator usually enters at the bottom used extensively in large liquid chilling installa-
of the accumulator, whereas the suction to the tions where advantage can be taken of the high
compressor is taken from the top. refrigerant side conductance coefficient. On the
In most cases, best performance is obtained other hand, because of their bulk and the rela-
when the capillary tube is connected directly tively large refrigerant charge required, float
between the condenser and the evaporator controls are seldom employed on small appli-
without an intervening liquid line. When the cations, having been discarded in this area in
condenser and evaporator are too far apart to favor of the smaller, more versatile thermostatic
make direct connection practical, some other expansion valve or the simpler, more economical
type of refrigerant control should ordinarily be capillary tube.
used. 17-17. Low Pressure Float Control. The
Bonding (soldering) the capillary tube to the low pressure float control (low side float) acts
suction line for some distance in order to to maintain a constant level of liquid in the
provide a heat transfer relationship between the evaporator by regulating the flow of liquid
two is usually desirable in that it tends to mini- refrigerant into that unit in accordance with the
mize the formation of flash gas in the tube. rate at which the supply of liquid is being
Flash gas, formed in the tube because of the depleted by vaporization. It is responsive only
322 PRINCIPLES OF REFRIGERATION

Vapor to
compressor

Fig. 17-29. Low side float


valve controlling liquid level in
accumulator. Note liquid pump
used to recirculate refrigerant
through evaporator.

Hand expansion valve

to the level of liquid in the evaporator and will toward the open or closed position to feed more
maintain the evaporator filled with liquid refrig- or less liquid into the evaporator in direct res-
erant to the desired level under all conditions ponse to minor changes in the evaporator liquid
of loading without regard for the evaporator level. For intermittent operation, the valve is so
temperature and pressure. it responds only to minimum and
designed that
Operation of the low pressure float valve may maximum liquid levels, at which points the valve
be either continuous or intermittent. With con- is either fully open or fully closed as the result
tinuous operation, the low pressure float valve of a toggle arrangement built into the valve
has a throttling action in that it modulates mechanism.

Point A should be above


liquid level in float chamber

Trap permits gravity return

rof suction
to
oil rich liquid refrigerant
line ahead
, of interchanger
i

Metering valve
Strainer and solenoid
~*y v valve. Valve normally Do not use
closed except when operated
pilot
Primary flow
compressor is operating solenoid valve

Fig. 17-30. Flooded evaporator (recirculating injector circulation). (Courtesy General Electric.)
REFRIGERANT FLOW CONTROLS 323

The low pressure float may be installed valve is a liquid level actuated refrigerant flow
directly in the evaporator or accumulator in control which regulates the flow of liquid to the
which it is controlling the liquid level (Fig. 11-1), evaporator in accordance with the rate at which
or it may be installed external to these units in a the liquid is being vaporized. However, whereas
separate float chamber (see Fig. 17-29). the low pressure float valve controls the evapora-
On large capacity systems, a by-pass line tor liquid level directly, the high pressure float
equipped with a hand expansion valve is usually valve is located on the high pressure side of the
installed around the float valve in order to system and controls the amount of liquid in the
provide refrigeration in the event of float valve evaporator indirectly by maintaining a constant
failure. Too, hand stop valves are usually in- liquid level in the high pressure float chamber
stalled on both sides of the float valve so that (Fig. 17-31).
the latter can be isolated for servicing without The operating principle of the high pressure
the necessity for evacuating the large refrigerant float valve is relatively simple. The refrigerant
charge from the evaporator (Fig. 17-29). Notice vapor from the evaporator condenses into the
also in Fig. 17-29 the liquid pump employed to liquid state in the condenser and passes into the

Weighted
valve pin

c
c
Fig. 17-31. High pressure float Intermediate valve
valve. (pressure reducing valve)

provide forced circulation of the refrigerant floatchamber and raises the liquid level in that
through the evaporator tubes. It is of interest component, thereby causing the float ball to rise
to compare this method of recirculation to the and open the valve port so that a proportional
injectionmethod of recirculation shown in Fig. amount of liquid is released from the float
17-30, and with the gravity recirculation method chamber to replenish the supply of liquid in the
shown in Fig. 11-1. The hand expansion valve evaporator. Since vapor is always condensed in
in the by-pass line around the liquid pump in the condenser at the same rate that the liquid is
Fig. 17-29 is to provide refrigeration in the event vaporized in the evaporator, the high pressure
of pump failure. float valve will continuously and automatically
Low pressure float valves may be used in feed the liquid back to the evaporator at a rate
multiple or in parallel with thermostatic expan- commensurate with the rate of vaporization,
sion valves. In many instances, a single low regardless of the system load. When the com-
pressure float valve can be used to control the pressor stops, the liquid level in the float chamber
liquid flow into several different evaporators. drops, causing the float valve to close and
17-18. High Pressure Float Valves. Like the remain closed until the compressor is started
low pressure float valve, the high pressure float again.
324 PRINCIPLES OF REFRIGERATION

'Service valve
Service valve

Bunker
type coil^

Fig. 17-32. Typical high pres-


sure float valve application.
(Courtesy Alco Valve Com-
pany.)

Liquid

header"

Oil drain

Since the high pressure float valve permits evaporator through the drop leg pipe attached
only a small and fixed amount of refrigerant to to the bottom of the surge drum. The suction
remain in the high pressure side of the system, vapor is drawn off at the top of the surge drum,
itfollows that the bulk of the refrigerant charge as is the flash gas resulting from the expansion
is always in the evaporator, and that the re- of the liquid asit passes through the float valve.

frigerant charge is critical. An overcharge of To prevent liquid flood-back during changes in


refrigerant will cause the float valve to overfeed loading, the surge drum should have a volume
the evaporator with the result that liquid re- equal to at least 25% of the evaporator
frigerant will flood back to the compressor. volume.
Moreover, if the system is seriously overcharged, The construction of a typical high pressure
the float valve will not throttle the liquid flow float valve is illustrated in Fig. 17-33. Notice
sufficiently to allow the compressor to reduce the that the float valve opens on a rising liquid level

evaporator pressure to the desired low level. and that the construction of the valve pin and
On the other hand, if the system is under- float arm pivot are such that the weight of the
charged, operation of the float will be erratic and float ball will move the valve pin in a closing
the evaporator will be starved. direction as the liquid refrigerant recedes in the
The high pressure float valve may be used float chamber. Notice also that the float ball is

with a dry-expansion type evaporator as shown so positioned that the valve seat is always sub-
in Fig. 17-31, or with a flooded-type evaporator merged in the liquid refrigerant in order to

as shown in Fig. 17-32. With the latter, liquid eliminate the possibility of wire-drawing by high
refrigerant is expanded into the surge drum (low velocity gas passing through the valve pin and
pressure receiver) from where it flows into the seat. Too, the high pressure float assembly

Body^. Float arm

Head

Pivots

Fig. 17-33. High pressure float


valve. (Courtesy Alco Valve
Company.) Valve pin

Valve seat

Outlet

'
Inlet
REFRIGERANT FLOW CONTROLS 325
contains a vent tube to prevent the float systems, a pilot valve is ordinarily employed for
chamber from becoming gas bound by non- this purpose (see Section 17-21).
condensible gases which may otherwise collect Because of their operating characteristics,
in thechamber and build up a pressure, thereby high pressure float controls cannot be employed
preventing liquid refrigerant from entering the in multiple or in parallel with other types of
chamber. The use of the vent tube makes refrigerant flow controls.
possible the installation of the high pressure 17-19. Float Switch. As shown in Fig. 17-34,
float valve at a point above or below the con- a float switch can be employed to control the
denser without the danger of gas binding. level of liquid in the evaporator. The float
Unlike the low pressure float control, the high switch consists of two principal parts: (1) a
pressure float control, being independent of the float chamber equipped with a ball float which
evaporator liquid level, may be installed either rises and falls with the liquid level in the
above or below that unit. However, the float evaporator and float chamber and (2) a mercury

Line Fuse
-\=r-
Evaporator pressure Solenoid pilot valve
To holding regulator
contact on motor
Strainer Heat exchanger
starter relay

a Start-stop
switch

Surge drum-

Water cooled section

\ ]
^Strainer

fc Solenoid liquid valve

"* Float switch

Liquid level

Refrigerant cooled
Globe valve
section

Fig. 17-34. Baudelot cooler with float switch, solenoid liquid valve, evaporator pressure regulator, and
solenoid pilot valve. (Courtesy Alco Valve Company.)

valve should be located as close to the evapora- switch which is actuated by the ball float to open
tor as possible and always in a horizontal line in and close a liquid line solenoid valve when the
order to insure free action of the float ball and level of liquid in the evaporator falls and rises,
valve assembly. When the float is located some respectively. A
hand expansion valve is in-
distance from the evaporator, it is usually stalled in the liquid line between the solenoid
necessary to provide some means of maintaining valve and the evaporator to throttle the liquid
a high liquid pressure in the line between the refrigerant and prevent surging in the evaporator
float valve and the evaporator in order to from the sudden inrush of liquid when the
prevent premature expansion of the liquid before liquid line solenoid opens and to eliminate short
it reaches the evaporator. In small systems, this cycling of the solenoid valve.
is accomplished by installing an "intermediate" Float switches have many applications for
valve in the liquid line at the entrance to the operating electrical devices associated with the
evaporator (see inset of Fig. 17-31). In larger refrigerating system. They may be arranged for
326 PRINCIPLES OF REFRIGERATION

Suction line

Return from

Thermostatic
expansion valve Insert bulb
of level
master
element
ooooo
mm"" Fig. 17-35. Thermostatic
pansion valve used as liquid
ex-
Electrical

connections levelcontrol. (Courtesy Spor-


2" pipe or larger lan Valve Company.)

reverse action (close on rise) by employing a The specially designed thermal element is an
reverse acting switch. insert bulb consisting of a low wattage electric

Liquid Level Control with Thermo-


17-20. heating element (approximately 15 watts) and a
static Expansion Valve. A thermostatic reservoir for the thermostatic charge. The
expansion valve with a specially designed heating element of the thermal bulb is a means
thermal element can also be used to control the of providing an artificial superheat to the
liquid level in flooded-type evaporators. Figure thermostatic charge, which increases the bulb
17-35 illustrates a typical installation on a pressure and results in opening the port of the
vertical surge drum. expansion valve allowing more refrigerant to be

Manual opening
stem

Pilot line

Bleed orfice "B"

Cage spring "C"


Fig. 17-36. Pilot-operated ex-

pansion valve main regulator.
(Courtesy Alco Valve Com-
pany.)

Main port
REFRIGERANT FLOW CONTROLS 327

fed to the evaporator. As the liquid level in the top of the piston in response to changes in the
evaporator rises and more liquid comes in con- temperature and pressure of the suction vapor.
tact with the bulb the effect of the heater When the superheat in the suction vapor
element is overcome, thereby decreasing the
increases, indicating the need of greater refrig-
superheat and allowing the thermostatic expan- erant flow, the pilot thermo valve moves in an
sion valve to throttle to the point of equilibrium opening direction and, supplies a greater pres-
or eventual shut-off. sure to the top of the piston thereby moving
The thermostatic expansion valve is installed the piston in an opening direction and providing
in the liquid lineand may be arranged to feed a greater flow of refrigerant. Conversely, when
liquid directly into the evaporator or accumula- the suction vapor superheat decreases, indicating
tor (surge tank), into an accumulator drop leg, the need of a reduction in refrigerant flow, the
or into a coil header. pilot valve moves in a closing direction. This
17-21. Pilot Control Valves. Pilot operated provides less pressure on top of the piston, per-
liquid control valves are employed on large mitting the piston to move in a closing direction

External
Remote equalizer line
bulb well

Pilot thermo
expansion valve

Pilot solenoid
liquid valve

.Main line
strainer

Note: When suction line rises dimension


"A" should be as short as possible

Fig. 17-37. Pilot-operated thermo expansion valve on shell-and-tube-water cooler with refrigerant in the
tubes. (Courtesy Alco Valve Company.)

tonnage installations. The pilot valve actuating and provide a smaller flow. In operation, the
the liquid control valve is usually a thermostatic pilot valveand the main piston assume inter-
expansion valve, a low pressure float valve, or a mediate or throttling positions depending on
high pressure float valve. A liquid control valve the load.
designed for use with a thermostatic expansion A liquid control valve designed for use with a
valve pilot is illustrated in Fig. 17-36. The high pressure float valve as a pilot is illustrated
liquid control valve opens when pressure is in Fig. 17-38. A system employing this type of
supplied to the top of piston "A" from the pilot refrigerant control is illustrated in Fig. 17-39.
line. The small bleeder port "B" in the top of Operation of the high pressure float pilot is
the piston vents this pressure to the outlet similar to that of the thermo expansion valve
(evaporator) side of the liquid control valve. pilot. On a rise in the level in the pilot receiver,
When the pressure supply to the top of the the float opens and admits high pressure liquid
piston cut off, the cage spring "C" closes the
is
through the pilot line to the liquid control valve
liquid control valve. piston. This pressure acts against a spring and
Figure 1 7-37 illustrates a pilot operated expan- opens the valve stem to admit liquid to the
sion valve installed on a direct expansion shell evaporator. As the level in the pilot receiver
and tube chiller. The externally equalized descends, the pilot valve closes and the high
thermostatic pilot valve supplies pressure to the pressure in the pilot line is bled off to the low
328 PRINCIPLES OF REFRIGERATION
adjusting the pressure on the spring above the
Pilot line^
control piston. A gage is installed in the pilot
connection-J'fpt
line to aid in the adjustment of the valve at
Strainer screen initial start-up.
17-22. Solenoid Valves. Solenoid valves are
widely used in refrigerant, water, and brine
lines in place of manual stop valves in order to
provide automatic operation. few of their A
many functions in the refrigerating system are
described at appropriate places in this book.
Asolenoid valve is simply an electrically
operated valve which consists essentially of a
coil of insulated copper wire and an iron core
or armature (sometimes called a plunger) which
is drawn into the center of the coil magnetic
field when the coil is energized. By attaching a
valve stem and pin to the coil armature, a valve
port can be opened and closed as the coil is
Hand operating energized and de-energized, respectively.
stem Although there are a number of mechanical
variations, solenoid valves are of two principal
Caution-for normal
system operation types: (1) direct acting and (2) pilot operated.
stem should be Small solenoid valves are usually direct acting
screwed in until flats whereas the larger valves are pilot
(Fig. 17-42),
only are exposed.
operated (Fig. 17-43). In the direct acting valve,
Fig. 17-38. Liquid control valve — high pressure. the valve stem attached to the coil armature
(Courtesy York Corporation.) controls the main valve port directly. In the
pilot operated type, the coilarmature controls
only the pilot port rather than the main valve
side through an adjustable internal bleeder port port. When the coil is energized, the armature
in the liquid control valve. A pressure gage is drawn into the coil magnetic field and the

should be installed in the pilot line to facilitate pilot port A is opened. This releases the
adjusting the control valve during the initial pressure on top of the floating main piston B
start-up. through the open pilot port, thereby causing a
A liquid control valve designed for use with pressure unbalance across the piston. The
a low pressure float pilot is shown in Fig. 17-40. higher pressure under the piston forces the
A typical application of a low pressure float piston to move upward, opening the main
pilot is shown in Fig. 17-41. As the liquid level valve port C. When the coil is de-energized, the
in the cooler drops, the low pressure float pilot armature drops out of the coil magnetic field
opens and allows the pressure in the pilot line and closes the pilot port. The pressure immedi-
to be relieved to the cooler so that the high ately builds up on top of the main piston,
pressure liquid acting on the bottom of the causing the piston to drop and close off the
liquid control valve piston can lift the piston and main valve port.
admit more refrigerant to the cooler. As the Except where the solenoid valve is specially
level in the cooler builds up, the pilot float closes designed for horizontal installation, the solenoid
and high pressure liquid is bled into the area valve must always be mounted in a vertical
above the control piston through an internal position with the coil on top.
bleeder in the bottom of the piston. The In selecting a solenoid valve, the size of the
resulting high pressure on top of the piston valve is determined by the desired flow rate
causes the piston to drop down and close the through the valve and never by the size of the
main valve port. The latter bleeder is not line in which the valve is to be installed.
adjustable, but modulation can be obtained by Consideration must also be given to the
REFRIGERANT FLOW CONTROLS 329*

C
L.
o

j2
>s
8
t

2
I
C
s
s

8
a

o
c
o

I
Q.
<
330 PRINCIPLES OF REFRIGERATION

o
>-

s
£
o
o

f>

a.

'I
— o
a.
*o"
IL U

§ e
u o
a.

o
>-

MO
REFRIGERANT FLOW CONTROLS 331

maximum allowable pressure difference across


the valve and to the pressure drop through the
valve.
17-23. Suction Line Controls. Suction line
controls are of two general types-: (1) evaporator
pressure regulators and (2) suction pressure
regulators.
The function ot tne evaporator pressure
regulator is to prevent the evaporator pressure,
and therefore the evaporator temperature, from
dropping below a certain predetermined mini-
mum, regardless of how low the pressure in the

\W
f\\\\

LW\\\^ !S^^-

Fig. 17-43. Pilot-operated solenoid valve of the


floating piston type. (Courtesy Sporlan Valve
Company.)

Fig. 17-42. Small, direct-acting solenoid valve.


The throttling-type evaporator pressure
(Courtesy Sporlan Valve Company.)
regulator (Fig. 17-44) is never fully closed
while the compressor is operating. As the load
suction line may drop because of the action of on the evaporator decreases and the evaporator
the compressor. It is important to recognize
that the evaporator pressure regulator does not
maintain a constant pressure in the evaporator
but merely limits the minimum evaporator
pressure. Evaporator pressure regulators are
available with either throttling action (modu-
lating) or snap-action (fully open or fully
closed).The differential between the closing
and opening points of the snap-action control
not only gives close control of the product
temperature but also provides for automatic
defrosting of air-cooling evaporators when the
temperatures in the space are sufficiently high Fig. 17-44. Throttling-type evaporator pressure
to permit off-cycle defrosting. regulator. (Courtesy Controls Company of America.)
332 PRINCIPLES OF REFRIGERATION

External
pilot circuit
connection

Fig. 17-45. Pilot-operated


evaporator pressure regulator.
(Courtesy Alco Valve Com-
pany.)

''Main port

.Is

Fig. 17-46. Crankcase pressure


^W^sgp
regulator. (Courtesy Sporlan
Valve Company.)
REFRIGERANT FLOW CONTROLS 333

pressure tends to fall below the preset minimum operation of the regulator is similar to that of
regulator pressure, the regulator modulates the pilot solenoid described in the previous
toward the closed position to throttle the section. With external pilot control, operation
suction vapor to the compressor, thereby of the regulator is similar to that of the pilot
maintaining the evaporator pressure above the operated liquid control valve described in
desired minimum. As the load on the evaporator Section 17-21.
increases and the evaporator pressure rises It is of interest to notice that the solenoid
above the regulator setting, the regulator pilot shown in Fig. 17-34 has nothing to do
modulates toward the open position so that at with the pressure regulating function of the
full load the regulator is in the full open evaporator pressure regulator. The use of the
position. solenoid pilot permits the evaporator pressure
Evaporator pressure regulators can be used to serve also as a suction stop valve.
in any installation when the evaporator pressure The function of the suction pressure regulator
or temperature must be maintained above a (Fig. 17-46), sometimes called a "crankcase
certain minimum. They are widely used with pressure regulator" or a "suction pressure
water and brine chillers in order to prevent hold-back valve," is to limit the suction pressure
freeze-ups during periods of minimum loading. at the compressor inlet to a predetermined
They are also used frequently in air-cooling maximum, regardless of how high the pressure
applications where proper humidity control in the evaporator rises because of an increase
prescribes a minimum evaporator temperature. in the evaporator load. The purpose of the
In multiple evaporator systems where the suction pressure regulator is to protect the
evaporators are all operated at approximately compressor driver from overload during periods
the same temperature, a single evaporator when the evaporator pressure is above the
pressure regulator can be installed in the normal operating pressure for which the
suction main to control the pressure in all the compressor driver was selected. Suction
evaporators. On the where a
other hand, pressure regulators are recommended for
multiple of evaporators connected to a single use on any where motor protec-
installation

compressor are operated at different tempera- tion is desired because the system is subject
tures, a separate evaporator pressure regulator to:
must be installed in the suction line of each of
1. High starting loads.
the higher temperature evaporators (see Section
2. Surges in suction pressure.
20-16). This arrangement prevents the pressures
3. High suction pressure caused by hot gas
in warmer evaporators from dropping
the
defrosting or reverse cycle (heat pump)
below the desired minimum while the com-
operation.
pressor continues to operate to satisfy the
4. Prolonged operation at excessive suction
coldest evaporator.
pressures.
In large sizes, evaporator pressure regulators
are pilot operated. The valve shown in Fig. Like evaporator pressure regulators, suction
17-45 is designed for either internal or external pressure regulators in large sizes are pilot
pilot control. With internal pilot control, operated.
Refrigerants 12, 22, 500 (Carrene 7), and 717
(ammonia).
As a general rule, because of limited valve
areas, reciprocating compressors cannot be
employed economically with low pressure
refrigerants which require a large volumetric
displacement per ton of capacity. Although
18 best applied to
pressures above one atmosphere
systems having
and
evaporator
relatively
high condensing pressures, reciprocating com-
pressors have also been used very successfully
Compressor in both low temperature and ultra-low tem-
perature installations.
Construction Reciprocating compressors are available in
sizes ranging from i hp in small domestic

and Lubrication units up through 100 hp or more in large


The fact that recipro-
industrial installations.
cating compressors can be manufactured
economically in a wide range of sizes and
designs, when considered along with
its dura-

bility and high efficiency under a wide


its
18-1. Types of Compressors. Three types variety of operating conditions, accounts for
of compressors are commonly used for refrig- its widespread popularity in the refrigeration
eration duty: (1) reciprocating, (2) rotary, field.
and (3) centrifugal. The reciprocating and Reciprocating compressors are of two basic
rotary types are positive displacement com- types: (1) single-acting, vertical, enclosed
pressors, compression of the vapor being compressors and (2) double-acting, horizontal
accomplished mechanically by means of a compressors employing crossheads and piston
compressing member. In the reciprocating rods. In single-acting compressors, compression
compressor, the compressing member is a of the vapor occurs only on one side of the
reciprocating piston, whereas in the rotary piston and only once during each revolution
compressor, the compressing member takes the of the crankshaft, whereas in double-acting
form of a blade, vane, or roller. The centrifugal compressors, compression of the vapor occurs
compressor, on the other hand, has no com- alternately on both sides of the piston so that
pressing member, compression of the vapor compression occurs twice during each revolution
being accomplished primarily by action of the of the crankshaft.
centrifugal force which is developed as the vapor Vertical compressors are usually of the
is rotated by a high speed impeller. enclosed type wherein the piston is driven
All three compressor types have certain directly by a connecting rod working off the
advantages in their own field of use. For th^ crankshaft, both connecting rod and crankshaft
most part, the type of compressor employed in being enclosed in a crankcase which is pressure
any individual application depends on the size tight to the outside, but open to contact with
and nature of the installation and on the the system refrigerant (Fig. 18-1). Horizontal
refrigerant used. compressors, on the other hand, usually employ
18-2. Reciprocating Compressors. The re- crankcases which are open (vented) to the
ciprocating compressor is by far the most outside, but isolated from the system refrigerant,
widely used type, being employed in all fields in which case the piston is driven by a piston
of refrigeration. It is especially adaptable for rod connected to a crosshead, which in turn, is
use with refrigerants requiring relatively small actuated by a connecting rod working off the
displacement and condensing at relatively crankshaft (Fig. 18-2).
high pressures. Among the refrigerants used Because of its design, the horizontal com-
extensively with reciprocating compressors are pressor is obviously not practical in small sizes
334
COMPRESSOR CONSTRUCTION AND LUBRICATION US

Bg- IE. I. Large capacity,


reciprocating compressor
showing details of lubrication

t/lttm. {Courtesy Vik*r


Manufacturing Company.}

and therefore is limited to the larger industrial crankcase not exposed to the system refrig-
is

applications. As compared to the vertical type, erant. The principal disadvantage of the
the horizontal compressor requires more floor horizontal compressor is that the packing or
space, but Jess head room. Also, while it is seal around the piston rod is subject to both

more expensive than ihe vertical type, it is also the suction and discharge pressures, whereas in

more accessible for maintenance since the the vertical type compressor, the packing or

Oil KKkel-IKWJ luhicaLcn Ufie .war.

Hew oil |ig.lu (rama crasshfrf nin Md GUite taction wlw

Flirt

dHflllM pockets
piste j«ket
IK mi hens

Frame bwt full langjh Law *™»


reducing OirtHutfiS discharge hvc

Fig. 18-2, Donblt-aeting. horizontal sOmpr«40r, Cinder clearance Can bi adjusted msmJally to obtain
capacity control. {Courtesy WorthlngtOn Corporation .)
33S PRINCIPLES OF REFRIGERATION

a.

"5

g
COMPRESSOR CONDUCTION AND LUBRICATION 337

seal around (he crankshaft is subject only to the most part, the type of piston used depends on
suction pressure. This disadvantage is made the method of suction gas intake and on the
more serious because it is usually more difficult location of the suction valves. Automotive-type.
to maintain a pressure Light sea) around the pistons are used when the suction gas enters the
reciprocating piston nod of the horizontal cylinder through suction valves located in the
compressor than it is around the rotating shaft cylinder head (valve plate) as shown in Fig, 8-3. 1

of the vertical type compressor. Double-trunk pistons arc ordinarily used in


Vertical, single-acting reciprocating com- medium and large compressors, in which case
pressors differ considerably in design according the suction gas enters through ports in the
to the type of duly for which they are intended. cylinder wall and in the side of the piston and
Numerous combinations of the following passes into the cylinder through suction valves
design Features are used in order to obtain the located in the top of the piston (Fig. 18-1),
desired flexibility: (I) the number and arrange- Notice that the bottom of the piston contains
ment of the cylinders, (2) type of pistons, (3) a bulkhead that seals off the hollow portion of
type and arrangement of valves, (4) crank and the piston from the crankcasc.
piston speeds, (5) bore and stroke, (6) type of Because of small piston clearances (approxi-
crankshaft, (7) method of lubrication, etc. mately 0.003 in. per inch of cylinder diameter),
ISO. Cylinders. The number of cylinders the oi! on the cylinder wall is usually
film
varies from few as one to as many as sixteen.
as sufficient to prevent gas blow-by around the
In multicylinder compressors, the cylinders may pistons in small compressors. For this reason,
be arranged in line, radially, or at an angle to rings are seldom used on pistons less than 2 in.
each other to form a V or W
pattern. For two in diameter, However, these pistons arc
and three cylinder compressors the cylinders provided with oil grooves to facilitate lubri-
are usually arranged in line. Where four or cation of the cylinder waits. Automotive- type
more cylinders are employed, F, W, or radial pistons having diameters above 2 in. arc usually
arrangements are ordinarily used. In-line equipped with two compression rings and one
arrangements have the advantage of requiring oil ring, the latter sometimes being located at
only a single valve plate, whereas F, IF, and the bottom of the piston. Double- trunk
radial arrangements provide better running pistons are equipped with from one to three
balance and permit the cylinders to be stag- compression rings at the top and one or two
gered so that the over-all compressor length is oil rings at !hc bottom.
less. As a general rule, pistons are manufactured
Compressor cylinders are usually constructed from close-grained cast iron, as are the rings,
of close-grained cast iron which [5 easily However, a number of aluminum pistons are in
machined and not subject to warping. For use. The use of cast iron permits closer toler-
small compressors, the cylinders and crankcasc ances. When aluminum
pistons are used, they
housing are often cast in one piece, a practice are usually equipped with at least one com-
which permits very close alignment of the pression ring,
working parts. For larger compressors, the 18-S. Suction and Discharge Valves, Since
cylinders and crankcasc housing are usually cast it influences to a greater or lesser degree all
separately and flanged and bolted together. the which determine both the volu-
factors
As a general rule, the cylinders of the larger metric and compression efficiencies of the
compressors are usually equipped with re- compressor, the design of the compressor
placeable liners or sleeves. suction and discharge valves is one of the most
Small compressors often have fins cast important considerations in compressor design.
integral with the cylinders and cylinder head Furthermore, it will be shown later that valve
to increase cylinder cooling, whereas cylinder design determines to a large extent the over-all
castings for larger compressors frequently design of the compressor.
contain water jackets far this purpose. The friction loss (wiredrawing effect) suffered
ItM, Pistons. Pistons employed in refrigera- by the vapor in flowing through the compressor
tion compressors are of two common types: valves and passages is primarily a function of
(1) automotive and (2) double-trunk . For the vapor velocity and increases as the velocity of
339 PRINCIPLES OF REFRIGERATION

the vapor increases. Therefore, in order to ptale,Q) the flexing or reed. All three types
minimize the wiredrawing losses, the valves operate automatically, opening and closing in
provide the largest response to pressure differentials caused by
should be designed to
and to open changes in the cylinder pressure. To facilitate
possible restricted Area (opening)
Too, whenever rapid closing of the valve, most discharge
with the least possible effort.
practical the valves should be so located as to valves and some suction valves are spring
provide Tor straight-line flow (unifiow) of the loaded.

through the compressor valves and 18-6. Poppet Valves. The poppel valve is
vapor
passages- In all cases, the valve openings must similar to the automotive valve, except that the
valve stem is much shorter. The valve is
be sufficiently large to maintain vapor velocities
within the maximum limits. The maximum enclosed in a cage which serves both as a valve

permissible vapor velocity may be defined as seat and valve stem guide and also as a retainer

that velocity beyond which ihe increase in the for the valve spring. A spring, dasbpot, or
bleeder arrangement also included in the
effect will produce a marked
is
wiredrawing
reduction in the volumetric efficiency of the assembly to cushion and limit the valve travel-

compressor and/or p material increase in the Except for minor differences, the design of the
power requirements of the compressor. suction and discharge poppet valves is essen-
tially the same, the principal difference being
To minimize back, leakage of the vapor
through the valves, the valves should be that the suction poppet valve is beveled on the

designed to close quickly and tightly. Id order stem side of the valve face, whereas the dis-
to open easily and close quickly, the valves charge poppet valve is beveled on the opposite
should be constructed of lightweight material side.
The poppet valve, one of the first types used
and be designed for a low lift. They should be
refrigeration compressors, is essentially a
strong and durable and they should operate in
Furthermore, they slow speed valve and as such is limited at the
quietly and automatically.
should be so designed and placed that they do present time to a few types of slow speed
compressors. Jn high speed machines, the
not increase the clearance volume of the
compressor. poppet valve has been discarded in favor of
Although there are numerous modifications either the ring plate valve or the flexing valve,

within any one type, valves employed in both of which are more adaptable to high speed
refrigeration compressors can be grouped into operation than is the poppet valve. The
three basic types: (I) the poppet, (2) the ring principal advantage of the poppet valve is that

Fig. 1*-4, Rlnj piate ntot uiemblj- Outer ring pflte l» the suction rtlvt. The two inn*f ring! ccmstitut*

the discharge v&k*. (Courtesy Frkk Company,)


COMPfcE&SOft CONSTRUCTION AND LUBWCATION 339

Fig 18-5. Discharge valve


assembly (diie valve), (from
the ASRE &ota Book, Design
Volume, 1957-58 Edition.
Reproduced by permission of
the American Society of Melt- Hole for soft
Shoulder screw
ing. Refrigerating anrf Air- ^Sy_ brass wire
Conditioning Engineers) for locking
Coil spring,
screws
Discharge vatae
retainer
Disc valve _
Flat spring

it car be mounted flush and therefore does not seat,a series of ribbon steel strips, and a valve
increase the clearance volume of the com- guard or retainer, The flexible metal strips fit
pressor, over slots in the valve seat and are held in place
J«-7- fting Plate Valves. The ring plate valve by the valve guard. The operation of the
(Fig. 18-4) consists of a valve seat, one or more Feather valve is illustrated in Fig. 18-7. It is
ring plates, one or more valve springs, and a important to notice that in order to aliow the
retainer. The ring plates are held firmly against valve reeds to flex under pressure, they are riot
the valve seat by the valve springs, which also tightly secured at cither end. The principal
help to provide rapid closure of the valves. The advantage of the Feather valve is that the reeds
function Df the retainer is to hold the valve are Ugh t weight and easily opened and are so
springs in place and to limit the valve lift. designed that they provide a large restricted
The ring plate valve is suitable for use in both area, ail of which tend to reduce the wiredrawing
slow speed and high speed compressors and it effect to a minimum.
may be used as cither the suction or the dis- One disadvantage of all flexing valves, and
charge ^alve. In fact, when both suction and one that is shared by the ring plate type, is that
discharge valves are located in the head, ihey they cannot be mounted can the poppet
flush as
are usual ty contained in the same ring plate vaj ve. Because of the presence of the valve port
assembly. For example, in Fig, 18-4, the outer spaces, the clearance volume is necessarily
ring serves as the suction valve, whereas (he increased in compressors employing either
all
two smaller rings serve as [he discharge valve. ring plate or Rising valves of any design.
One modification of the ring plate valve is A flexing valve design widely used in smaller
the disc valve (Fig. 18-5), which is simply a thin compressors is the flapper valve, of which there
metal disc held in place on the valve seat by a are enumerable variations. The Rapper valve
retainer, is a thin steel reed, which is usually fastened
18-8. Flexing Valves, Flexing valves vary in securely at one end while the opposite unfas-
individual design to a much greater extent than tened end rests on the valve seat over the valve
do either the poppet or ring plate-types. One port. The free end of the reed flexes or "flaps"
popular type of flexing valve suitable for use in to cover and uncover the valve port (Fig, 8-8), 1

medium and large compressors is the Feather A dapper valve design frequently employed
valve* (Fig. 18-6), which consists of a valve in discharge valves, called a "beam" valve, is
* A proprietary design of the Worth ington Pump shown in Fig, 18-9. The valve reed is held in
and Machinery Company, place over the valve port by a spring-loaded
340 PRINCIPLES OF REFRIGERATION

consists of a flexible metal disc which is held


down on the valve seat by a screw or bolt
through the center of the disc. The disc flexes
up and down to uncover and cover the valve
port. A diaphragm valve used as a suction valve
mounted in the crown of a piston is illustrated

in Fig. 18-10.
18-*. Valve Location. As previously de-
scribed, the discharge valves are usually located
in the cylinder head, whereas the suction valves
may be located either in the head, in which case
the suction vapor enters the cylinder through the
crown of the piston, in
cylinder head, or in the
which case the suction vapor enters through
the side of the cylinder. As a general rule,
with larger compressors, the suction valves are
located in the piston and the suction vapor
enters through the cylinder wall. With small

Valve

Fig. tB-4. One popular design of flexing vaNe.


{Courwiy Worihlnjtom Corporation.)

beam which is arched in the center to permit


the reed to flex upward at this point. The ends
of the reed are slotted and are held down by
only the tension of the coil springs in order to
allow ibc ends of the reed (o move as the reed
flexes up and down at the center. The spring-
loaded beam also act.'; as a safety device to
protect the compressor against damage in the

event that a slug of liquid refrigerant or oil

enters the valve port. Since the valves are


designed to handle vapor, there is not usually
of liquid of
sufficient clearance to pass a slug
any bind. However, with the arrangement in
Fig. 18-9, the whole valve assembly will lift to
pass liquid slugs. Under ordinary discharge
pressures, the tension of the springs is ample to
hold the beam down firmly on the ends of the
reed. Fig, 1B-7. llluitrailng the operation of the Worth I nj-
Another type of flexing valve in common use tan Feither valve. (Courtesy Worthirjto n Corpora-
is the diaphragm valve. The diaphragm valve tion.)
:

COMPHESSOft CONSTRUCTION AND LUBRICATION 341

and medium compressors, the suction valves


are usually located in the cylinder head. When
both valves are placed in the cylinder head, ihe
head must be partitioned to permit separation
of the suction and discharge vapors.
Most large compressors are equipped with
secondary safely heads which are located al
the end of the cylinder and held in place bv
heavy coil springs (Fig. 18-1), Under normal
discharge pressures, the safety head is held
(irmly in place by the springs. However, in the Flf. tM. Compressor vilv* plate aisembly. (Cour-
event that a slug of liquid or some other nety Tccumseh Products Com piny.)
noncompressible material enters the cylinder,
the safety head will rise under the increased
pressure and permit the material to pass into are being operated successfully at speeds up to
the cylinder head, thereby preventing damage 3500 rpm. The maximum rotational speed of
to the compressor. In smaller compressors., the compressor is more or less limited by the
the discharge valve is usually designed to maximum allowable piston velocity.
provide this protection. Theoretically, there is no limit to piston
In large compressors, the valves and seats speeds. However, as a practical matter, piston
arc removable for replacement, fn small speeds are limited to 9maximum of approxi-
compressors, the suction and discharge valves mately 800 fpm, the limiting factor being the
are usually incorporated into a valve plate available valve area.
assembly, which is removed and replaced as a Since considerable difficulty is experienced in
unit (see Fig. 18-9). finding sufficient space in the compressor for
IB-IO. Crank and Piston Speeds, tn an effort valve arrangements, valve areas tend to be
to reduce the size and weight of the compressor, somewhat limited. Hence, when piston veloci-
the trend in modern compressor design is tiesare increased beyond 900 fpm. the velocity
toward higher rotational speeds. Since single- of the vapor through the valves will usually
cylinder piston displacement is a function of become excessive, with the result that the
bone, stroke, and rpm, it follows that as the volumetric efficiency of the compressor is
rpm is increased, the bore and stroke can be decreased while the power required by the
decreased proportionally without loss of dis- compressor is increased.
placement, provided that the volumetric effi- Piston velocity is a function of compressor
ciency of the compressor remains the same. rpm and the length of the piston stroke. The
Rotative speeds between 500 and 1750 rpm following relationship exists
are quite common, whereas some compressors Piston velocity (fpm) **
rpm a stroke (ft)

x 2 strokes per revolution

For example, a compressor ha ving a 4 in. stroke


and rotating at 1200 rpm will have a piston
speed of
1200 x
—4 in. x 2
« 800 fpm

If the rotational speed of the compressor is

increased to 3600 rpm, in order to maintain a


piston velocity of 300 fpm, the length of stroke
(*> will have to be reduced to

Fig. I6.-8. Flapper-type fltxtfng valve, (a) Port open.


(b) Pon d™*d.
12
tttz
x S00 fpm
3600 rpm x 2
~- 1.33 in.
3*2 PRINCIPLES OF REFRIGERATION

Fig. IB- 10, Dttphnam-iyp* suction *il*e. TTil* »<n* type of vtlve If often mounted In * valve plate and used
u i discharge vike, (Courtesy York Corporation.)

Obviously, then, the maximum speed at which large bore is in order to


usually required
an individual compressor can be rotated without provide sufficient valve area. Too, since the
exceeding allowable piston velocities depends piston stroke and the compressor rpm are both
upon the length or stroke. The shorter the limited somewhat by the maximum allowable
stroke, the higher is the maximum permissible piston velocity, it follows that the only practical
[pax This accounts for the fact that the means of increasing single-cylinder piston
volumetric efficiency of a compressor will displacement is to increase the size of the bore.
usually remain constant or increase slightly as The increase in piston displacement accruing
the speed of the compressor is increased up to a from an increase in the size of the bore need
certain point beyond which, if the speed is not increase the vapor velocity through the
further increased, [he efficiency of the conv compressor valves since the increase in the bore
piessor will decrease while the power required also increases the available valve area.
per ton of refrigerating capacity will be greater However, since the amount of blow-by
(see Seel ion 12-28). around the piston increases as the size of the
M-ll, Bore and Stroke, The relationship of bore is increased with relation to the stroke,
the bore to the stroke differssomewhat with good design practice limits the bore dimension
the individual compressor. Although the bore to approximately 125% of thai of the stroke
dimension may be either less than or greater If the bore is increased beyond this point,
than that of the stroke, the general trend in the blow-by around the piston becomes exces-
high speed compressors is toward a large bore sive.

and a short stroke. When the suction and Since compressors working on low tempera-
discharge valves are both located in the head, a ture or "boosier" applications (see Section
COMPRESSOR CONSTRUCTION AND LUBRICATION M3
20-11} must handle relatively large volumes of (Fig. 18-2). The stuffing box is a cylindrical
vapor pet ton of capacity, they are usually housing which is cast as an integral part of the
designed with a large bore and a short stroke in crankcase where the shaft emerges, and which is

order to obtain the maximum piston displace- bored to an inside diameter somewhat larger
ment per cylinder. than the diameter of the crankshaft. A series
Cylinder bores range from approximately 1 of packing rings, placed over the shaft and
in. in smalt domestic compressors up to inserted into the stuffing box, fills the space in
approximately 18 in. in some of the large the stuffing box between the shaft and the
industrial types, stuffing box housing. The packing is held in
18-12. Cranks, Rods, and Bearings. Crank- place by a threaded gland nut which, when
shafts employed in large compressors are of the tightened, causes the packing rings to swell
crank-throw type and are usually constructed and press tightly against the shaft and housing,
of forged stccf or alloy east iron. All bearing- thereby affecting a vapor tight seal between the
journals are highly polished and are usually two. Because of the pressure of the rings
ease-hardened, particularly where brass or against the rotating shaft, the rings will even-
aluminum bearings are used. As a general rule, tually wear and permit refrigerant to seep
the crankshaft has a standard taper on the around the shaft, whereupon the packing gland
flywheel end, the flywheel being fastened lo the nut must be tightened again to reestablish a
crankshaft with one or more woodruff keys tight seal- Although the stuffing hex seal is
and a locknut arrangement. Crankshaft satisfactory for large ammonia installations

bearings are usually of the sleeve type, although where an operator is on duty to tighten the
antifriction (roller or bad) bearings are packing gland as the occasion requires, they
sometimes used for the mains. Common are not suitable for small compressors or for
hearing materials are bronze, aluminum, and large compressors designed for automatic
babbit. operation.
The eccentric-type shaft, which consists of A crankshaft seal suitable for use on auto-
a cast iron eccentric mounted on a straight matic equipment must be self-adjusting to
steel shaft (see Fig. IB-3) is often used in smaller compensate for wear and for varying crankcase
compressors. The eccentric is counterbalanced pressures. It must not leak under pressure or
and is fastened to the shaft by a kcy-and-lock vacuum when the shaft is rotating or idle. It

screw arrangement. Since the bearing of the must be self-lubricating, have a reasonably
connecting rod completely encircles the eccen- long life, and be easily replaceable in the field.
tric, the entire eccentric acts as a bearing sur- Although there are a number of different seal
face. designs which meet these qualifications and
Connecting rods are constructed of bronze, which are in use at the present time, one rela-
aluminum, forged steel, or cast iron. Wrist pins tively simple design of crankshaft sjeal, which is

are usually case-hardened steel. Wrist-pins rapidly gaining in popularity. Is shown in Fig-

bearings are generally of the sleeve type made 13-11. The seal consists essentially of a spring-
of bronze and pressed into the rod- Bronze, loaded bronze or hard carbon seal nose which
aluminum, and cast iron arc often used without is sealed to the crankshaft with a synthetic
bearings, in which case the shaft is usually rubber gasket. The spring holds the scat nose
case-hardened. firmly against a highly polished steel seal face
IB- 1 3. Crankshaft Seals. In order to prevent which is a part of the seal plate. An oil film
the leakage of refrigerant and oil from the between these two smooth surfaces form an
crankcasc (or [be leakage of air into the effective vapor-tight seal. Notice that sealing
crankcase in the event that the pressure in occurs in three places: (I) at the rubber gasket
the crankcase is below atmospheric), a seal or between the seal nose and the crankshaft,
packing must be provided at the point where the (2) between the seal nyse and seal face, and
crankshaft passes through the crankcase. One (3) at the gasket between the seal plate and the
of (he oldest methods of sealing the crankshaft, crankcase housing.
still employed on some large ammonia com- The compressor shown in Fig. 1S-1 employs
pressors, is through the use of a stuffing Box a double shaft seal. Notice that the seal
1

344 PRINCIPLES OF REFRIGERATION


Mitel
retaining
rinj
Oil ^ -•-*

Sea! ulult-

Crankshaft

tMarifiE

Fig. 16-11. Crankshaft jc

Gasket
Seal spring

remains completely submerged in oil during especially important when selecting a lubri-
both the running and off cycles. cating oil for these units.
18-14. Compressor Lubricating Oils. The For the most part, the chemical stability of
fact that the compressor lubricating oil usually an oil is closely related to the amount of
comes n o contact \w ith, a nd often mi xes w th he
i ! i , i unsaturated hydrocarbons present in the oil.
system refrigerant make it necessary that the oil The smaller the percentage of unsaturated
used to lubricate refrigeration compressors be hydrocarbons contained in (he oil, the more
specially prepared for that purpose. Same of stable is the oil. For refrigeration service, a
the more important properties of the oil which high quality oil with a very low percentage of
must be considered when selecting the com- unsaturated hydrocarbons is desired. These
pressor lubricating oil are: <1) chemical oils are usually light in color, being just off
stability, (2) ponr and/or floe point, (3) dielectric from a water-white.
strength, and (4) viscosity, in evaluating these IB* 4.
1 Pour, Cloud, and Floe Points. The
oil properties with relation to an individual pour point of an oil is the lowest temperature at
compressor, all the following factors should be which the oil will flow, or "pour," when tested
taken into account: (1) the type and design of under certain specified conditions. Of two oils
th* compressor, (2) the nature of the refrigerant having the same viscosity, one may have a
to be used,
(3) the evaporator temperature, and higher pour point than the other because of a
(4) thecompressor discharge temperature. greater wa * content. Pour point an importan 1
is
IB- 5. Chemical Stability. The importance consideration in selecting an low tem-
oil for
of chemical stability is emphasized by the fact perature systems. Naturally, the pour point
that it is necessary for the compressor lubri- of the oil should be well above the lowest
cating oil to perform its lubricating function temperature to be obtained in the evaporator.
continuously and effectively without under- If the pour point of the oil is loo high, the oil
going change for long periods of lime. Since tends to congeal on the surface of the evaporator
changing the oil in a hermetic motor-compressor tubes, causing a loss in evaporator efficiency.
is not usually practical, the same oil frequently Since this oil is not returned to the compressor,
remains in these units throughout the life of the inadequate lubrication of the compressor may
unit, which is often ten years or more. Because also result.
of the high discharge temperatures encountered Since all lubricating oils contain a certain
in hermetic motor -compressor units, particularly amount of paraffin, wax will precipitate from
where air-cooled condensers are used, the any oil if the temperature of the oil is reduced
ability of the oil to remain stable and resist to a sufficiently low level. Because the oil
decomposition under high temperature is becomes cloudy at this point, the temperature
COMPRESSOR CONrTRUCTLON AND LUBRICATION 345

at which the wax begins to precipitate from the cating oil, viscosity may also be defined as a
oil is called the cloud point of the oil, Jf the measure of the "body" of the oil or of the
cloud point of the oil is too high, wax will ability of the oil to perform its lubricating
precipitate from the oil in the evaporator and function by forming a protective him or coat-
in the refrigerant control. Although a small ing between the various moving parts of the
amount of wax in the evaporator does little compressor, thus keeping the parts separated
harm, a small amount of wax in ihe refrigerant and preventing wear. In order to provide
control will cause stoppage of thai part, with adequate lubrication for the compressor, the
the result that the system will become Inopera- viscosity of the oilmust be maintained within
tive, reasonable limits. of the oil is
If the viscosity
The Roc point of the oil is the temperature at too low, the oil wilt not have sufficient body to
which wax will start tofrom a precipitate keep the moving parts separated and thin film
mixture of 90% Refrigerant- 12 and 16% oil by lubrication will result, accompanied by exces-
volume. Since [he use of an oil soluble refrig- sive wearing of the rubbi ng surfaces. Too, si nee,
erant lowers the viscosity of the oil and affects in addition to its lubricating function, the oil
both the pour and cloud points, where oil frequently must serve as a sealing agent between
miscible refrigerantsemployed, the floe
are the low and high pressures in the compressor,
point of the oil is a more important property excessive blow-by of vapor around the pistons
than the pour or cloud points. The use of 10% (in a reciprocating compressor! or vanes (in a
oil in the oil-refrigerant mixture to determine rotary compressor) may occur when the
the floe point seems quite realistic, since the viscosity of the oil is low. On the other hand,
tendency of an oil-refrigerant mixture to when the viscosity of [he oil is loo high, fluid
separate wax increases as the amount of oil in friction willbe excessive and ihc power con-
the mixture increases and since the amount of sumption of the compressor will be increased.
oil circulating with the refrigerant seldom Furthermore, in extreme case, a high viscosity
exceeds 10% and is usually much less. oil may not have sufficient fluidity to penetrate

Because floe point of the oil is a measure of between the various rubbing surfaces, particu-
the relative tendency of the oil to separate wax larly when tolerances are close, with the result
when mixed with an oil soluble refrigerant, it that the lubrication of the compressor parts
is an important consideration when selecting an will be inadequate.
oil for use with an oil miscible refrigerant at The viscosity of a lubricating oil is usually
evaporator temperatures below 0° F. However, measured in Saybolt Seconds Universal (SSU)T
hoc point has no significance when a non- which is an index of the time in seconds required
miscihte refrigerant is used, for a given quantity of oil (60 mm) at a con-
ra-17. Dielectric Strength. The dielectric trolled temperature (usually 100° F) to flow by
strength of an oil is a measure of the resistance gravity from a reservoir into a flask through a
that the oil offers to the flow of electric current. capillary tube of specified internal diameter
It is expressed in terms of the voltage required (0.1765 cm) and length (1.225 cm). An oil
to cause an electric current to arc across a gap having a temperature of 100" F and requiring
one-tenth of an inch wide between two poles 300 sec to pass through the tube is said to have
immersed in the oil. Since any moislure, a viscosity of J0O SSU at 100" F.
dissolved metals, or other impurities contained The viscosity of the lubricating oil changes
in the oil will lower its dielectric strength, a high considerably with the temperature, increasing as
dielectric strength is an indication that tbc oil is the temperature decreases. The effect of tem-
relatively free of contaminants. This is especi- perature on the viscosity of a typical lubricating
ally important in oils used with hermetic motor- oil is shown graphically in Fig. 18-12 (see lop
compressor units, since an oil of low dielectric line —0% refrigerant dilution). Notice that the
strength may contribute to grounding or oil has a viscosity at 100° F of approximately
shorting of the motor windings, 175 SSU, but increases to approximately I BOO
1 0-1 A. Viscosity, Viscosity has already been SSU when the temperature of the oil is reduced
defined in Section 16-10 as the resistance that a to 40" P.
fluid offers to flow. With regard to the lubri- Shown also in Fig, 18-12 is the effect of
H* PRINCIPLES OF REFRIGERATION

2000f

Fig. 18-12. ViKWlty tem-


perature curves of solution
of Refrigerant- ( 2 in o\\.

(Front ASfl£ Dfflo Book, Design


Volume, f957-iS Edition. Re-
prod ut*d by p*cm)Hipfl of the
Amerldll Soviet/ of Heating,
Refrigerating and Air-Condi-
tioning Engineers.)

-20 20 40 60 SO 100 ffiD 140 ISO ISO 210


TemparatUfa F

refrigerant dilution on the viscosity of the with the bottom of the main crank bearings,
Imbricating Notice, for example, that pure
oil- With each revolution of the crankshaft, the
oil having a viscosity of 175 SSU at 100° F connecting rod and crankshaft (or eccentric)
has a viscosity of about 60 SSU at this same dip into the oil causing the oil to be splashed
temperature when diluted with 1 5 % Refrigerant- up on the cylinder walk, bearings, and other
12. rubbing surfaces. Usually, small cavities or oil
It is evident from the foregoing that both the reservoirs are located at each end of the crank-
operating temperature range and the effect of case housing immediately over the main
refrigerant dilution must be taken into account bearings. These cavities collect oil which feed
in selecting proper viscosity oil. In all
the by gravity down into the main bearings and
cases, the compressor manufacturer's recom- shaft seal (Fi£. 1S-3), In some instances,
mendations should be followed when Ihcy are connecting rods are rifle-drilled to carry oil to

available. When such data are not available. the wrist-pin bearings. Too,
scoops or oil

(he values given in Fig. 13-13 may be used as a dippers arc sometimes installed on the end of the
guide. connecting rods to increase splashing and/or to
18-19. Methods of Lubrication. Methods of aid in forcing oil through rifle-drilled oil
lubricating the compressor vary somewhat passages.
depending upon the type and size of the A modified type of splash lubrication, some-
compressor and upon the individual manu- times called flooded lubrication, employs
facturer. However, for the most part, lubri- slinger rings, discs, screws, or similar devices to
cation, methods can be grouped into two general above the crankshaft or
raise the oil to a level
types (1) splash and (2) forced feed. Although
i main bearings, from where it is allowed to
forced feed lubrication can be found even in flood over the bearings, and/or feed through oil
very small compressors, as a general rule, small, channels to the various rubbing surfaces (Fig.
vertical, enclosed compressors up through 6-14). This method is particularly suitable for
approximately 15 hp are splash lubricated. small, high speed compressors where the
Above this size, most compressors employ conventional splash system may result in
some type of forced feed lubrication. Often, a excessive oil carryover because of violent
combination of the splash and forced feed splashing of the oil in the crankcasc.
methods is found in a single compressor, In the forced feed method of lubrication, the
Jn the splash method of lubrication, the oil is forced under pressure through oil tubes
compressor crankcasc acts as an oil sump and and/or rifle-drilled passages in the crankshaft
is filled with oil to a level approximately even and connecting rods to the various rubbing
: 1

COMPRESSOR CONSTRUCTION AND LUBRICATION U7


surfaces. After performing its lubricating A. Small Systems
function, the oil drains by gravity back into a Type of SU at 100 F
sump located in the crankcase or the com- Refrigerant Compressor Oil Viscosity
pressor. The oil is circulated under pressure Ammonia Reciprocating J 50-300

developed by a small oil sump located in the Carbon dioxide Reciprocating 280-300
crankcase of the compressor, usually at the end Sulfur dioxide Reciprocating 70-200
of the crankshaft (Fig. 18-1). Since most oil Sulfur dioxide Rotary 2SO-300
pumps are automatically reversible, the direc- Methyl chloride Reciprocating 2S0-3O0
tion of crank rotation is not usually critical Refrigerant-30 Centrifugal 280-300
with regard to compressor lubrication. How- Refrigerant-30 Rotary 150-300
ever, this is not true or all compressors, particu- Refrigerant- 1 Centrifugal 28G-30G
larly those employing oil dippers with splash Refrigerant- 12 Centrifugal and
lubrication.When rotation is critical, an arrow reciprocating 2SO-300
denoting the proper direction of rotation is Refrigerant- 21 Reciprocating 280-300
usually embossed on the flywheel or crankcase Refrigerant-] 13 Centrifugal 280-300
housing. Refrigerant- 1 14 Rotary 28G-3O0
Oil strainers are always placed at the suction
B. Industrial Refrigeration
inlet of the oil pump to prevent the entrance of (Ammonia and carbon dioxide compressors
foreign material info the pump or bearings.
with Splash, force- feed, or gravity
Although not required, oil filters are worthwhile circulating systems)
in all forced feed lubrication systems to elimi-
SU Viscosity
nate the possibility of plugged oil line resulting
Type of Compressor Range
from the accumulation of sludges or other
Where oiJ may enter refrig-
residue, An oil pressure failure safety switch
eration system or
(Section Zl-20) should be employed in con-
compressor cylinders 1 50-300 at 100 F
junction with all forced feed lubricatfonsystems.
Where oil is prevented from
In some large compressors, cylinders are
entering system or
lubricated by mechanical forced feed lubricators
cylinders:
which are located external to the compressor
In force feed or gravity
crankshaft. In such cases, the cylinder lubri-
systems 506-600 at 100 F
cation system is entirely separate from the
In splash systems 150-160 at 100 F
internal pressure lubricating system (Fig. 18-14).
Steam-driven compressor
The bearings and cross-heads of horizontal,
cylinders when conden-
open crankcase compressors are usually splash
sate is reclaimed for ice
lubricated (Fig. 18-2), Oil from the crankcase
making 140-165 at 210 F
is carried to the cross-head by splash-fed
troughs. Cylinders and piston rod packing C. Miscellaneous Equipment
glands arc lubricated by mechanical forced feed SU Viscosity at
lubricators similar to those used to lubricate Type of Requirement 00 F Range
1

the cylinders of large vertical compressors, Bearings


18-10, Liquid Refrigerant in the Com- king oiled, normal
pressor Crankcase, The presence of liquid temperature 280-300
refrigerant in the compressor crankcase is Ring oiled, low temperature 100-115
always undesirable for a number of reasons. Chain oiled 280-300
In the first place, excessive dilution of the Sail and roller bearings:
crankcase by liquid refrigerant can result in
oil Oil lubricated 280-300
inadequate lubrication of the compressor parts. Grease lubricated —
More important, however, is that fad that the Wick oilers 280-600
liquid refrigerant will vaporize in the crankcase Fig. IB- 1 3. Lubrication recommendation:. {From
and cause foaming of the oil, with the result ASKE Ou<rt took, Design Volume, I957-S& Edition, by
that the amount of oil carried over into the permission of the American Society of Heating, Re.
discharge line is materially increased. Under frige rating and Air-Conditioning Engineers.)
PRINCIPLES OF REFRIGERATION

f i g, 1 a- 1 *- Sifigle-aKifig, verti-
cal com pressor with enclosed
erankctse. (Courtesy Vllrer
Manufacturing Company. )

more likely to occur during start-up than at any


certain, conditions, oil foaming may become so
In any event, it is easily prevented
the oil is pumped out of the other lime.
severe that all
or corrected by proper application
and adjust-
crankcase. Mot only will this leave the com-
the ment of the refrigerant control and/or by
pressor without lubrication but there is also
noncompressible slugs of liquid properly designed suction piping,
possibility thai
2, Liquid refrigerant may
drain by gravity
refrigerant and oil wil! enter the cylinder and
into the crankcase from the
compressor
cause serious damage 10 the compressor in the
evaporator and/or suction piping during the off
form of broken valves and pistons and bent or
cycle. This condition is also caused by
faulty
broken rods and shafts. Too, where consider- the
system design, particularly with reference to
able oil foaming occurs in compressors employ-
feed lubrication, lubrication will evaporator and suction piping. A leaking
ing forced
refrigerant control also be a contributing
may
often be inadequate because the oil pump is
factor- Here, again, the condition is readily
unable to develop sufficient pressure to deliver
corrected or prevented by proper design.
the oil to the various rubbing surfaces.
3. Any time the temperature at the com-
Furthermore, vaporization of liquid refrig-
pressor crankcase falls below that of the
erant in the crankcase tends to reduce the
the
evaporator, liquid refrigerant will boil off in
capacity and efficiency or the compressor in
evaporator and condense in the compressor
that the resulting vapor is drawn into the
crankcase. Naturally, this can occur only
cylinder and displaces vapor which would
during the off cycle and only when the com-
otherwise be taken from the suction line.
pressor is so located that the ambient tempera-
Liquid refrigerant may gain entrance into the
ture at the crankcase can fall below that of the
crankcase in a number of ways:
evaporator. It is prevalent in tbe wintertime in
installations where the compressor is
located
I . Improper application or adjustment of the
cold
flow control will often cause outside or in a basement or some other
refrigerant
location. The only solution, of course, is to
continuous Or intermittent overfeeding of the
maintain tbe temperature of the crankcase
evaporator, in which case liquid refrigerant will
slop-over from the evaporator into the suction
above the saturation lemperaturt of the
refrigerant vapor. This can be accomplished
by
line and be carried to live compressor
crankcase.
installing an electrical heating element in tbe
As described in Chapter 17, this condition is
1

COMPRESSOR CONSTRUCTION AND LUBRICATION 3«

crankcase or by moving the compressor to a pressure in the evaporator and in the crankcase
warmer location. is immediately reduced Co 25 psig. At this lower
pressure, the maximum percentage of refrig-
Because of the tendency of the lubrication erant that can be present in the oil -refrigerant
oil to absorb oil miscible refrigerant vapors, a
mixture is only approximately 1 3 %. Therefore,
certain amount of liquid refrigerant will always
in a matter of only a few seconds approximately
be dissolved into the lubricating oil in any
system employing an oil miscible refrigerant,
one-third of the refrigerant by weight of0%
the totaJ mixture) must of necessity vaporize
assuming that the refrigerant vapor and the oil new
out of the mixture in order to establish the
are permitted to come in contact with one
percentage. Naturally, the vaporization of this
another, as is usually the case. For the most
much refrigerant out of the mixture in such a
part the percentage refrigerant that can be
short time will cause severe foaming of the oil
dissolved into the crankcase oil depends on
with the result that a considerate amount of
three factors : (I) the degree of miscibility of
oil is drawn into the compressor.
the refrigerant, (2) the pressure of the refrigerant foaming
There are several ways to reduce oil
vapor, and (3) the temperature of the lubri-
and the loss of oil from the crankcase during
cating oil. For any one refrigerant, the per-
compressor start-up. One common method is
centage refrigerant that will be dissolved into
to equip the compressor with an oil check valve,
the oil depends only on the pressure of the
which is installed in the oil passage between the
refrigerant vapor, the temperature of the oil,
suction inlet of the compressor and the crank-
and the length of time that the two are in contact
case (Fig. 8-16). With oil miscible refrigerants,
i

under steady conditions. pumped


the oil over into the system ordinarily
The of Refrigerant- 1 in oil under
solubility
returns to the compressor with the suction
various conditions of tempera tu re and pressure
vapor. On entering the suction inlet, the oil is
is shown graphically in Fig. 13-15. Notice that
separated from the vapor by impingement before
the percentage of Refrigerant- 12 which can be
the vapor enters the cylinder. The separated oil
dissolved in the oil increases considerably as
the temperature of the oil decreases and as the
pressure of the refrigerant
For example, when the temperature of the oi!
is 100" ¥ and the refrigerant vapor pressure is
vapor increases. 140

120 fl 7 1 f V i

maximum percentage of Rcfrigeram-

X^
20 psi, the
12 which can be present in the oi [-refrigerant
mixture
8%
in the crankcase
by weight. However,
is

if
only approximately
the oil is cooled to 1
h
j*
SO F while the refrigerant vapor pressure is I ^qV
increased to 60 psi, the percentage refrigerant
in

as
the oil-refrigerant mixture could be as high
42%. In other words, under the latter
/)
/ / si-
conditions, the soiled lubricating oil in the
crankcasc could actually be 42% Refrigerant- 12.
The practical significance of the foregoing can
be il lustiatcd by the use of an
that during the
esam pie Suppose
compressor off cycle, the pressure
.
20
y'
on the low side of a Refrigerant- 2 system 1
w//
10 20 30 40 SO 60
rises to 38 psig, whereas the crankcase cools to faunt by weight of R-12
a temperature of BO" F- Assuming that the
off cycle is of sufficient length to permit equi-
Flf. I6-)S. Temperature-pTeMure relationship of
librium to be established, trie percentage pf
Refrigerant- 12 mixture! (pressure it f»i()- (From
oi! I

liquid refrigerant in the oil-refrigerant mixture ASRE Data book, Design Volume, 19S7-5B Ed linn. i

in the crankcase will be approximately


by 20% Reproduced by permission of the American Society
weight, as determined from Fig. 18-15. Suppose of Heating, Refrigerating and Air-Conditioning
now that the compressor cycles on and that the Engineers.)
iSO PRINCIPLES OF REFRIGERATION

pressure produced by the throttling action of


the refrigerant control will supply the pressure
differential necessary to
cause the oil to flow
through the check valve into the crankcase.
Suction However, the suction inlet chamber of the
BCTM4
compressor must be large enough to serve as a
reservoir for ail the oil that returns to the
compressor during the time that the crankcase
pressure is too high to permit oil drainage into
the crankcase.
Oil check
valve
Another method of reducing the amount of
oil foaming at start-up, and one which is
Bleed pert rapidly growing in popularity, is to install a
small wattage heating element in the compressor
crankcase.The crankcase heater is wired to
come on when the compressor cycles off and
serves to keep the oil in the crankcase warm
during the officycle so that the amount of
refrigerant which can be dissolved into the oil
is However, care should be
relatively small.
taken to wire the heater to the secondary of Ihe
Ffg, I ft- 16. Illustrating oil ehtsk value and bind port.
main disconnect so that It cannot be turned off
unless the main disconnect is pulled.
Still another method of reducing oii foaming

drains from the inlet chamber to the crank case at start-up is to operate the system on a pump-
through an oil passage provided for this down cycle, in which case the evaporator is

purpose. Since this oil passage also serves to completely evacuated and the crankcase pres-
equalize the crankcase pressure to the com- sure reduced to a low level before the compressor
pressor suction, a cheek valve installed in the cycles off. The resulting low pressure in the
oil passage will prevent the crankcase pressure crankcase limits the amount of refrigerant
from venting to the suction, thereby eliminating absorbed by the oil. The pump-down cycle
the sudden reduction in crankcase pressure used alone or in conjunction with either the
which produces oil foaming at start-up. oil check valve or (he crankcase heater is very
However, since no oil can drain through the effective In reducing oil foaming.
passage to the crankcase until the crankcase 18-21. Rotary Compressors. Rotary com-
pressure reduced 10 the suction pressure, a
is pressors in common use are of two general
smalt bleed port must be provided around the designs. One employs a cylindrical steel
check for through it) to permit the crankcase roller which revolves on an eccentric shaft, the
pressure to bleed off slowly into the compressor latter being mounted concentrically in a
suction after the compressor cycles on. The cylinder (Fig. 18-17). Because of the shaft
bleed port is required also to relieve cylinder eccentric, Ihe cylindrical roller is eccentric
blow-by gases back to the suction of the with the cylinder and touches the cylinder wall
compressor. If hlow-hy gases arc not vented at the point of minimum clearance. As the
to the suction, the crankcase pressure will build shaft turns, the roller rolls around the cylinder
up to the discharge pressure. Not only would wall in the direction of shaft rotation, always
this prevent the oil from returning to the maintaining contact with the cylinder wail.
crankcase, it would also cause a material With relation to (he camshaft, the inside surface
increase in the power requirements of the of the cylinder roller moves counter to the
compressor. direction of shaft rotation in the manner of a
During the normal running cycle, the crank- crankpin bearing. A spring-loaded blade,
case pressure is approximately the same as the mounted in a slot in the cylinder wall, bears
suction, and minor fluctuation in the suction firmly against the roller at all times. The blade
COMPRES5QK CONSTRUCTION AND LUBRICATION 351

moves in and out of the cylinder slot to follow low pressure vapor will be in the cylinder.
the roller as the latter rolls around the cylinder The manner in which the vapor is compressed
wall. by the roller is illustrated by the sequence of
Cylinder heads or end-plates are used to close drawings in Fig. I S-1T.
the cylinder at each end and to serve as supports The whole cylinder assembly is enclosed in a
for the camshaft- Both the roller and blade housing and operates submerged in a bath of
extend the full length of the cylinder with only oil. Notice that the high pressure vapor is

working clearance being allowed between discharged into the space above the oil level in

these parts and the end-plates. Suction and the housing from where it passes into the
discharge ports are located in the cylinder wall discharge line. Ail rubbing surfaces in the
near the blade slol, but on opposite sides. The compressor including the end-plates are highly
flow of vapor through both the suction and polished and closely fitted. Although no
discharge ports is continuous , except for the suction valves are needed, a check of flapper
instant that the roller covers one or the other valve is installed in the discharge passage to
of the ports. The suction and discharge vapors eliminate back-feeding of the discharge vapor
are separated in the cylinder al the point of into the cylinder. When the compressor is
contact between the blade and roller on one operating, an oil film forms a seal between the
side and between the roller and cylinder wall high and low pressure areas. However, when
on the other side. the compressor stops, the oil seal is lost and

The poin! on the cylinder wall in contact with the high and low pressures equalize in the
the roller changes continuously as the roller compressor. A check valve must be placed in
travels around the cylinder. At one point the suction line (or discharge line) to prevent
during each compression cycle the roller will the high pressure discharge gas from backing
cover the discharge ports, at which time only up through the compressor and suction line

Dnch*rE«

Shift

Hg- I B.I 7. Blade-type rotary m


compressor.

to m
3S1 PRINCIPLES OF REFRIGERATION

To suction line to the evaporator when the com-


r.on fiercer pressor cycles off.
Hscl .;.::.
Although rotary compressors are- positive
port
displacement machines, because of their rotary
motion and the smoother, more constant flow
Discharge
of the suction and discharge gases, they are
reed
ROtor much less subject to mechanical vibration and
slot
to the pronounced discharge pulsations associ-
ated with the reciprocating compressor. How-
Cjhnder ever, like reciprocating compressors, rotary
compressors experience volumetric and com-
pression losses resulting from blow-by around
Suction/ the compressing element, back leakage through
valves, cylinder heating, clearance, and wire-
Fig, 18-18. Vant-ijHpfl row? ee-mprewor. drawing. As a general rule, the efficiency of
rotary compressors is relatively high, being
about 65 to 80%, depending on the individual
into !he evaporator when the compressor design and the operating conditions.
cycles off. Rotary compressors are particularly suitable
Another design of rotary compressor employs for applications requiring a relatively large
a series of rotating vanes or blades which are compressor displacement at moderate operating
installed equidistant around the periphery of a pressures. Under these conditions, the rotary
slotted rotor (Fig. 18-18). The rolor shaft is compressors will usually have a distinct dis-

mounted eccentrically in a steel cylinder so that placement advantage over reciprocating types
the rotor nearly touches the cylinder wall on of comparable size. For this reason, large
one side, the two being separated only by an rotary compressors of the rotating vane design
oil film at this point. Directly Opposite this are extensively used in industrial low tempera-
point the clearance between the rotor and the ture applications (Fig. 18-19), being employed
cylinder wall is maximum. Heads or end-plates as booster compressors in the low stages of
are installed on the ends of the cylinder to seal two- and three-stage cascade systems.
the cylinder and to hold the rotor shaft. The Small rotary compressors have been used
vanes move back and forth radially in the rotor successfully in domestic units for a number of
slots as they follow the contour of the cylinder years. Although a few rotary compressors have
wall when the rotor is turning. The vanes are been utilized in commercial installations, the
held firmly against the cylinder wall by action difficulty encountered in the manufacture of the
of the centrifugal force developed by the rotating larger sizes tends to limit their use in this area.
rolor. In some instances, the blades are spring- I(>11. Centrifugal Compressors- The centri-
loaded to obtain a more positive seal against the fugal compressor consists essentially of a scries
cylinder wall. of impeller wheels mounted on a steel shaft and
The suction vapor drawn into the cylinder enclosed in a cast iron casing (Fig. 18-20),
through suction ports in the cylinder wall is The number of impeller wheels employed
entrapped between adjacent rotating vanes, The depends primarily on the magnitude of the
vapor is compressed as the vanes rotate from thermodynamic head which the compressor
the point of maximum rotor clearance to the must develop during the compression process.
point of minimum rotor clearance. The com- Compressors employing two. three, and four
pressed vapor is discharged from the cylinder wheels (stages of compression) are common.
through ports located in the cylinder wall near More wheels may be used when the required
the point of minimum rotor clearance. increase in head is sufficiently large to demand
This type of rotary compressors also requires it, As many as twelve wheels have been used
the use of a check valve in the suction or in some individual cases.
discharge line to prevent the discharge gas from As shown in Fig. 13-21,
the impeller wheel of
leaking back through the compressor and a centrifugal compressor consists of two discs,
COMPRESSOR CONSTRUCTION AND LUBRICATION 353

a hub disc and a cover disc, with a number of and at increased temperature and pressure, The
blades or vanes mounted radially between them. high-pressure, high-velocity vapor discharged
To resist corrosion and erosion, the impeller from the periphery of the wheel is collected in
blades are usually constructed either of stainless specially designed passages, in the casing which
steel or of high carbon steel with a lead coating. reduce the velocity of the vapor and direct the
A typical two-stage rotor is shown in Fig. vapor to the inlet of the next stage impeller or,
18-32, in the caseof the last stage impeller, to a dis-
The operating principles of the centrifugal charge chamber, from where the vapor passes
compressor arc similar to those of the centrifu- through the discharge line to the condenser.
gal fan or pump. Low-pressure, tow-velocity The refrigerant flow path through a two-Stage
vapor from the suction line is drawn in the centrifugal compressor is shown diagrammatic
inter cavity or *'eye'* of the impeller wheel cally in Fig.
|£23,
along the axis of the rotor shaft. On entering The routing impeller wheels are essentially
the impeller wheel, the vapor is forced radially the only moving parts of ihe centrifugal com-
outward between the impeller blades by action pressor and as such arc the source of all the
of the centrifugal force developed by the energy imparted to the vapor during the com-
rotating wheel; and; is discharged from the blade pression process. The action of the impeller is
tips into the compressor housing at high velocity such that both the static and velocity heads of

Equalize*
check valve

Cyllnd*f-)«tttt
nil ll*SI

Capacity unlove?
by-pass line

Capacity unloaded
automatic con mi'
valve

Compressor
sub- base

Capacity unKMtter
expander poduet

Fig. I ft- 1 9. Large capacity, rotating vana-type rotary compressor. (Courtesy Freezing Equipment Sales, Inc.}
PRINCIPLES REfFtlGE RATION

Fig, IB-M. Four-sta^e centrifugal compressor wuh upper half of housing removed to show construction of
detakli. (Courtesy York Corporation,)

the vapor are increased by the energy so be produced by an equivalent gravitational


imparted to the vapor. The centrifugal forte column (Section 15-3).
exerted on the vapor confined between, and In addition to thehead which is
static
rotated with, the blades of the impeller wheels produced head is also
cenlrifiigaliy, a velocity
cause* self-compression of the vapor in much developed within the impeller wheel because of
the same manner that the force of gravity causes the increase in the velocity of the vapor as the
the upper layers of a gas column compress the
to vapor passes from the eye to the periphery of the
lower layers of the column. Hence, the static wheel. As the mass of refrigerant vapor passes
head produced cenlrifugally within the impeller through, and is rotated by the impeller wheel, it

wheels is equal to the static head which would attains a rotational velocity approaching that
of the wheel. Since the greater portion of this
velocity hend is subsequently converted to
statichead within the casing surrounding the
wheels, the total increase in pressure developed
by a single wheel is the sunt of the increases in
both the static and velocity pressures of the
vapor,
Hy assuming radial blades, the total head
developed by a single impeller wheel is directly
proportional to the square of the peripheral
velocity of the wheel, viz;

y%
Fly, 14-21.
Impeller wheel.
Cutaway view of centrifugal COmfsrfcHOr
(Courtesy York Corporation.)
N=—
i
COMPRESSOR CONSTRUCTION AND LUBRICATION 355

where H= the total head in feet small. As a general rule, two or more impeller
V- the periphiat velocity of the wheel wheels must be used in order to obtain the
in fps necessary pressure increase, in which case com-
g = the gravitational constant pression of the vapor occurs in stages as the
The total increase in pressure produced per vapor passes from one wheel to the next.
wheel is Assuming equal vapor velocities at the inlet
H x P V* x p and outlet of the compressor, the total Increase
fi
144 " 144 *£ it) pressure in the compressor is the sum of the

where p = [he pressure in psi pressure increases produced by the individual


p = the mean density of the vapor in wheels. Notice that in any series of wheels the
Ib'cu ft wheels are made progressively smaller in size in
From the foregoing it is evident that for a the direction of vapor flow in order to com-
refrigerant of given density, the total increase pensate for the reduced volume of the vapor
in pressure developed by a single wheel depends resulting from prior compression in the pre-
only on the tip velocity of the impeller blades, ceding wheel or wheels (Fig. IB -20).
this tip velocity in turn being proportional to Since the head of a fluid is an expression of
the rotational speed of the rotor shaft and to the energy per pound of flu id, it follows that the
the diameter of the impel let wheel.
1
However, head developed by the compressor during the
since the maximum tip velocity is limited by the compression process is numerically equal to the
strength or materials and by the sonic speed of work done in foot-pounds per pound of vapor
the refrigerant, it follows that the maximum compressed, and that the magnitude of the
increase in pressure which can be obtained with head which must be produced by the com-
a single impeller wheel is also limited. For pressor depends on the refrigerant used and on
this reason, single-stage centrifugal compressors, the difference between the saturated suction and
such as the one shown in Fig. S-24, can be used 1 discharge temperatures. 1 herefote, it is evident
only in those few applications where, because any given set of operating conditions,
that, for

of a small temperature head (difference between the head which must be produced by the
vaporizing and condensing temperatures), the compressor, that is, the diameter, speed, and
increase in pressure (head) required is relatively number of wheels required, will be the same

Pj. 18-22. Two-siage rotor centrifugal Cornpr**»r nowr Huembljr, (CaurtBiy York Corporation ,)
3S* PRINCIPLES OF REFRIGERATION

Fig. It'll* Diagram of jas flow


thfoujh centrifugal eemprei-
jor. {Court «y York Corpora-
tion.)

for small capacity compressors as for large discharges to the condenser approximately 500
capacity compressors. For this reason, centri- cu ft of vapor per minute. The exact tonnage
fugal compressors arc not practical in small this represents depends upon the refrigerant
sizes. used and on the operating conditions. At the
For good wheel performance, the diameter, present time, centrifugal compressors are
width, and eye dimensions of the impeller wheel available in sizes ranging from approximately
must be maintained within certain ratio limits. 35 tons to well over 2000 tons.
Since the width of the impeller
must be reduced as the volume of
vapor handled is reduced in order
to Insure stable operation at low-
gas volumes, the wheel width could
become very narrow, resulting in
high friction losses and poor
wheel performance. Therefore, to
keep the wheels in proportion.
It becomes necessary to reduce the
diameter of the wheels as the
width of the wheels is reduced.
At the same time, the speed of
rotation and/or the number of
wheels must be increased in order
to maintain the required head.
Since this tends to increase manu-
facturing and other costs, most
manufacturers agree that the
smallest practical size of centri-
fugal compressor is one which

Fig. 18-14, Slnjle-s.ta|*«nirl-


fug»l oompnatsor. {Ceurteiy
York CorpnsrKion,)
COMPRESSOR. CONSTRUCTION AND LUBRICATION 3S7

Centrifugal compressors sure essentially high often installed in the casing passages which
speed machines. Rotative speeds of between convey the vapor from the discharge of one
3000 and 8000 rpin arc quite common, with wheel to the inlet of the nest. The diffuser vanes
much higher speeds being used in some indivi- are curved in a direction opposite to that of
dual cases. Because of their high rotative vapor discharge from the impeller wheels and
speeds, centrifugal compressors arc capable of are so designed (area increasing in the direction
handling large volumes of vapor in relatively of vapor flow) that velocity reduction and the
small sizes. Although especially suited for use accompanying increase in static pressure take
with low pressure refrigeranls requiring a large place gradually and smoothly and with a mini-
compressor displacement at moderate com- mum loss of energy. When diffuser vanes are
pression ratios, they have been applied success- not employed, gradual velocity reduction is
fully in all temperature ranges with both low obtained by discharging the vapor directly into
and high pressure refrigerants. scroll- or volute-shaped passages which guide
Some of the more common refrigerants the vapor from one wheel to the next. A com-
employed with centrifugal compressors arc pressor of volute design is shown in Fig. 13-25,
Refrigerants-ll, -12, -113, and ammonia. The In some instances, diffuser vanes and volutes are
high displacement required per ton of refrig- used together in a single compressor.
eration with Refrigerants- 1 and 113 make 1 The back leakage of refrigerant between the
these refrigerants idea) for use with centrifugal several wheels or stages is limited to a practical
compressors in high temperature applications minimum by the use of labyrinth- type seals
where the displacement required per ton of which are arranged between the rotor and the
capacity is relatively low. Their use in such stationary partitions, The labyrinth seal con-
applications permits small refrigerating capaci- sists essentially of a series of thin steel strips

ties without requiring small compressor frames which are fastened to the rotor and which match
and wheel sizes. On the other hand, when the lands and grooves in the stationary partitions
required refrigerating capacity is large and/or (Fig. S-26), The labyrinth of passages provided
1

the evaporator design temperature is low, by this type of seal causes a drop in the pressure
refrigerants which require a relatively small of the refrigerant gas as it passes through each
displacement per ton capacity, such as Refrig- restricted area formed by the shaft sealing strips
erants- 1 1 and ammonia, will ordinarily allow and housing. As the pressure drops, the velo-
the use of smaller compressors to produce the city of the gas increases. However, on entering
same tonnage. In any event, because of the the next pocket, the gas encounters a large
difference in the operating pressures, head quantity of gas at rest and the increased velocity
requirements, and other characteristics of the acquired during passage through the restriction
several refrigerants, the compressor must be is dissipated by the production of turbulence in
designed to fit the refrigerant as well as the the pockets. The leakage through the labyrinth
application. seal is proportional to the clearance between the
Centrifugal Compressor efficiencies are rela- shaft sealing strips and the compressor housing
tive high in all sizes and over a wide range of and is also a function of the number of restrictions
operating conditions, being about 70 to 80% or pockets provided.
as a general rule, although values well over 30% The fact that the vapor pressure on the dis-
are obtained n i many instances. Efficiency losses charge side of the impeller wheels is always
in a centrifugal compressor are due primarily to greater than the pressure on the suction or inlet
irreversible changes resulting from turbulence side of the wheels causes the rotor assembly to
and fluid friction. develop an axial thrust toward the suction inlet
18-23. Centrifugal Compressor Construc- of the compressor. To offset this thrust, a
tion and Lubrication. For maximum- balance disc is usually installed on the rotor
pressor efficiency, the conversion of velocity shaft on the discharge side of the high stage
pressure into static pressure in the casing must impeller wheel (Fig. 13-20). This disc, equipped
occur gradually and smoothly and without an with a labyrinth seal, acts as a floating partition
appreciable loss in the total pressure head- To at the end of the discharge space. The pressure
accomplish this, a series of diffuser vanes is on the outboard side of the balance disc is
.

358 PRINCIPLES OF REFRIGERATION

Fif. 18-lSd. VoltitE-type t&mpreHQr. (Courtesy Worthinjton Corporation.}

of the Sow stage The rotor assembly is supported radially in


equalized to the suction inlet
impeller through an equalizer line (Fig. I&-23), the housing by two main bearings, one localcd
whereas the inboard side of the disc is subject at each end of the rotor shaft (Fig. (S-2G). A
to the discharge pressure of the high stage Kingsbury-type thrust bearing mounted on the
wheel. When the balance disc is properly sized, discharge end of the shaft positions the rotor

the pressure differential across the disc will assembly axially in the casing. Since the axial
thrust of the rotoris usually neutralised by one
exactly balance the natural thrust of the rotor
assembly (Fig. 18-27). means or another, the load on the thrust bearing
In one design of a three-stage compressor is ordinarily very light. As in the case of the

(Fig. 18-256), ibe impellers are so positioned on open-type reciprocating and rotary compressors,
the shaft that the axial thrust developed by the a shaft seal is employed between Ihe compressor

third-stage impeller opposes the thrust developed housing and the rotor shaft in order to prevent
by the first- and second-stage impellers. Since inward or oulward leakage at ihe point where
the third-stage impeller is the highest pressure the shaft protrudes from the compressor

impeller, the thrust produced by it is sufficient housing


to counter substantially the combined thrust of
Centrifugal compressors are pressure lubri-

the other two impellers. cating cither by a submerged type oil pump
COMPRESSOR CONSTRUCTION AND LUBRICATION 359

c
o
I
o
a.
t_
O
y

t
o

t)

o
u

Q.
£
Q

U
M
s

</> o E W^: o ifl w


360 PRINCIPLES OF REFRIGERATION

Second-stage
impeller at
cond. press.

Fig. 18-26. Labyrinth seal


between impellers. (Courtesy
York Corporation.)

Labyrinth seal
between housing
and shaft
driven directly from the rotor shaft or by a a relatively high efficiency over a wide range of
separate, externally mounted, motor-driven oil load conditions, and its high volumetric dis-
pump with an external oil reservoir. The prin- placement per unit of size, there are certain other
cipal parts of the compressor requiring lubri- desirable performance characteristics inherent
cation are the two main bearings, the Kingsbury in the of a centrifugal compressor.
design
thrust bearing, and the shaft seal. Since these Principal among
these is its relatively flat head-
parts are so located that they do not come into capacity characteristic as compared to that of
direct contact with the system refrigerant during positive displacement compressors. This, along
normal operation, lubrication is simplified in with an extreme sensitivity to changes in speed,
that there is little or no contamination of the greatly simplifies the problem of capacity con-
refrigerant by the compressor lubricating oil. trol and tends to give the centrifugal compressor
The leakage of oil along the rotor shaft from a decided advantage over the reciprocating type
the main bearings into the refrigerant spaces is in any large tonnage installation where the
minimized by the use of oil seal labyrinth glands evaporator temperature must be maintained
which are installed on the shaft on the inboard relatively constant despite wide variations in
side of each of the main bearings (Fig. 18-20). evaporator loading.
Oil coolers are employed to maintain oil tem-
perature during normal operation. Oil heaters
are usually installed in the oil reservoir to
Discharge
prevent excessive refrigerant dilution of the oil outlet
Suction
during periods of shut-down. Oil filters are
inlet
standard equipment on all centrifugal com-
pressors. Compressors employing a shaft-driven
oil pump must also be equipped with an aux-
iliary oil pump to supply oil pressure during 1 Arrows indicate equal
pressures acting
start-ups and at other times when the shaft
assembly
on rotor
driven pump cannot supply adequate lubrication — :.")
,

a..^»«>-:-. -*
for the compressor parts.
External equalizing line-'
18-24. Performance of Centrifugal Com- Fig. 18-27. Diagrammatic sketch of thrust balance.
pressors. In addition to its ability to maintain (Courtesy York Corporation.)
COMPRESSOR CONSTRUCTION AND LUBRICATION 361

Like the centrifugal pump or blower, the


delivery capacity (in cfm or in tons refrigeration)
of an individual centrifugal compressor will
decrease as the thermodynamic head produced
by the compressor increases. Conversely, it is
true also that as the delivery rate of the com-
pressor reduced the head produced by the
is

compressor must increase. Therefore, since the


maximum head which the compressor is capable
of developing is limited by the peripheral speed
of the impeller wheels, it follows that the mini-
mum delivery capacity of the compressor is also
limited. If the load on the evaporator becomes
too small, the thermodynamic head necessary
to handle the reduced volume of vapor will

50 1

100* F
45 B
condensing
temperature
*&\ D
.,40

|35
A
^ y\
100 120 140 160 180
Tons refrigeration
200

Fig. 18-29. Centrifugal versus reciprocating perfor-


220

r>5 mance. (Courtesy York Corporation.)


E Cons tent
«30 rp m "
~2
reciprocating type in Figs. 18-28 through 18-31.
& 25 Some of the more important differences in the
^20 performance of the two compressors become
apparent on careful examination of these data.
15
Notice in Fig. 18-28 that a reduction in refrig-
V
10 erating capacity from 240 to 100 tons is accom-
100 120 140 160 180 200 220 240 plished by the centrifugal compressor with a
Tons refrigeration

Fig. 18-28. Centrifugal versus reciprocating perfor- 108


40* F Ih
mance. (Courtesy York Corporation.) evaporator 1
E
106 1
con stant rpm

exceed the maximum head which the compressor 104


i
can produce. When this point is reached,
com-
pressor operation becomes unstable and the 102 ^ o

compressor begins to "surge" or "hunt."


Centrifugal _\ /
compressor '

However, with proper capacity control methods,


the load on a centrifugal compressor can be
A
/
reduced to as little as 10% of the design load 98

without exceeding the pumping limit of the


compressor. i

As in the case of the reciprocating compressor, i


the capacity and the power requirements of the 94 i
{
'

F
centrifugal vary with the vaporizing and con- 9? Gl
densing temperatures of the cycle and with the 130 140 150 160 170 180 190
speed of the compressor. With reference to Brake horsepower
these variables, the performance of the centri- Fig. 18-30. Centrifugal versus reciprocating perfor-
fugal compressor is compared to that of the mance. (Courtesy York Corporation.)
362 PRINCIPLES OF REFRIGERATION

Capacity characteristics Horsepower characteristics


Constant evaporating temperature Constant evaporating temperature
Constant condensing temperature Constant condensing temperature
Varying speed Varying speed

100

90
V \\ s
1 80 \ \ \ Worth-
\ \ Fig. 18-31. (Courtesy
i V \ \
> ^Reciprocating N Reciprocating
ington Corporation.)
|eo \ \
\
k
\
50
Identrif ugal
\
.
\\ Centrifugal
\
\
40

100 90 80 70 60 50 40 30 20 100 90 80 70 60 50 40 30 20
Per cent speed Per cent speed

corresponding change in evaporator tempera- A-B illustrates that the centrifugal compressor
ture of only 10° F as compared to a 29° F change experiences a rapid reduction in capacity as the
required by the reciprocating compressor to condensing temperature increases. This charac-
effect the same tonnage reduction. This means teristic of a centrifugal compressor makes it

in effect that a centrifugal compressor will main- possible to control compressor capacity by
tain a more constant evaporator temperature varying the quantity and temperature of the con-
over a much wider range of loading than will the denser water. The capacity of the compressor
reciprocating type. Naturally, this is an impor- can be reduced by this means until point A is
tant advantage in any installation requiring the reached, beyond which a further increase in the
maintenance of a constant evaporator tempera- condensing temperature will cause the required
ture under varying load conditions. thermodynamic head to exceed the developed
Too, the fact that a rather substantial change head of the compressor for the given speed and
in capacity is brought about by only a small tons capacity, with the result that hunting will
change in the suction temperature makes prac- occur.
tical the use of suction throttling devices as a The reduction in the capacity of the recipro-
means of controlling the capacity of a centri- cating compressor with a rise in the condensing
fugal compressor, a practice which cannot be temperature is relatively small as compared to
recommended for reciprocating compressors. that experienced by the centrifugal compressor.
It is of interest to notice also that the oper- Regardless of the increase in condensing temper-
ating range of the centrifugal compressor is ature, the reciprocating compressor will continue
definitely limited by the "surging" or "hunting" to have a positive displacement and produce
characteristic of the compressor. In Fig. 18-30, a refrigerating effect.
the centrifugal evaporator temperature cannot Figure 18-29 compares the power require-
fall below 35° F regardless of the reduction of ments of the centrifugal and reciprocating com-
the evaporator loading. A further decrease in pressors under conditions of varying condensing
evaporator would cause the compressor to reach temperature. Whereas the centrifugal shows a
its "pumping limit" and a rise in the evaporator reduction in power requirements with an in-
temperature would occur. By comparison, the crease in the condensing temperature to corre-
positive displacement reciprocating Compressor spond with the rapid fall off in capacity shown
will continue to reduce the evaporator tempera- in Fig. 18-31, the reciprocating compressor
ture and pressure as the evaporator load is shows a small increase in power requirements
reduced until a capacity balance is obtained to correspond with the small change in refrig-
between the evaporator loading and the com- eration tonnage shown in Fig. 18-29 for that
pressor capacity. machine.
Figure 18-31 shows a performance comparison Figure 18-30 also illustrates the nonover-
between centrifugal compressors operating at loading characteristic of the centrifugal com-
constant speed and evaporator temperature but pressor. Notice that an increase in condensing
with varying condensing temperature. Curve temperature causes a reduction in both the
COMPRESSOR CONSTRUCTION AND LUBRICATION 363

and the power require-


refrigerating capacity Because of its extreme
sensitivity to changes
ments of the compressor, although the horse- in speed, the centrifugal compressor is ideally
power required per ton increases. suited for capacity regulation by means of
With regard to compressor speed, the centri- variable speed drives, such as steam turbines
fugal compressor is much more sensitive to and wound-rotor induction motors. When con-
speed changes than is the reciprocating type. stant speed drives, such as synchronous or
Whereas the change in the capacity of the squirrel cage motors, are employed, speed con-
reciprocating compressor is approximately pro- trol can be obtained through the use of a
portional to the speed change, according hydraulic or magnetic clutch installed between
to the performance corves in 18-31, a Fig. the drive and the step-up gear.
speed change of only 12% will cause a 50% 18-26. Centrifugal Refrigeration Machines.
reduction in the capacity of the centrifugal Centrifugal compressors are available for re-
compressor. frigeration duty only as an integral part of a
18-25. Capacity Control. Capacity control centrifugal refrigerating machine. Because of
of centrifugal compressors is usually accom- the relatively flat head-capacity characteristic of
plished by one of the following three methods: the centrifugal compressor, and the resulting
(1) varying the speed of the compressor, (2) limitation in the operating range, the component
varying the suction pressure by means of a parts of a centrifugal refrigerating system must
suction throttling damper, or (3) varying the be very carefully balanced. Too, since very
condensing temperature through control of the marked changes in capacity accrue with only
condenser water. Often, some combination of minor changes in the suction or condensing
these methods is used. temperatures, the centrifugal refrigerating system

Fig. 18-32. Centrifugal refrigerating machine. (Courtesy Worthington Corporation)


364 PRINCIPLES OF REFRIGERATION

Condenser
water in JSp
* » » i " '«
»
— -\\
Condenser » )j

CondensenP^^^^
water out

Economizer
Carrier
centrifugal
compressor
Fig. 18-33. Flow diagram for
typical centrifugal refrigerating
machine. (Courtesy Carrier
Corporation.)

Chilled
brine out

Chilled •
brine in
B Refrigerant liquid

EH Refrigerant vapor

must be close-coupled, as shown in Fig. mediate chamber. From the intermediate cham-
18-32, in order to reduce the refrigerant line ber the cool liquid passes through the inter-
pressure losses to an absolute minimum. mediate float valve into the evaporator, at which
A schematic diagram of a centrifugal refrig- time the temperature of the liquid is reduced
erating system is shown in Fig. 18-33. Except to the evaporator temperature by additional
for the introduction of a flash intercooler flashing. Hence, the .effect of the intercooler is

(Section 20-12) between the condenser and to increase the refrigerating effect per pound and
evaporator, the centrifugal refrigerating system to reduce the amount of flash gas in the evapora-
operates on a conventional vapor-compression tor. Since the flash vapor from the intermediate
cycle. High pressure liquid drains from the chamber is taken into the suction of the second-
bottom of the condenser into the high pressure stage impeller, the pressure of this vapor will be
chamber of the intercooler, from where it passes above the evaporator pressure and therefore the
through a high pressure float valve into the power required to compress it to the condensing
intermediate chamber of the intercooler. In pressure will be less. Too, the cool vapor from
passing through the float valve, a portion of the the intercooler reduces the temperature of the
liquid flashes into the vapor state, thereby discharge vapor from the first-stage impeller
cooling the balance of the liquid to the tempera- with the result that the capacity and efficiency
ture corresponding to the pressure in the inter- of the system are increased.
Steel pipe should be of either the seamless or
lap-welded types, except that butt-welded pipe
may be used in sizes up to 2 in. All steel pipe
1 in. or smaller should be Schedule-80 (extra
heavy). Above this size, Schedule-40 (standard
weight) pipe may be used, except that liquid lines

19 up to 1 J in. should be Schedule-80.


Copper tubing is available in either hard or
soft temper. The hard drawn tubing comes in
20 ft straight lengths, whereas the soft temper is
Refrigerant Piping usually packaged in 25 and 50 ft coils. Only
types K and L are suitable for refrigerant

and Accessories lines.

Soft temper copper tubing may be used for


refrigerant lines up to J in. OD, and is recom-
mended for use where bending is required, where
the tubing is hidden, and/or where flare connec-
tions are used. Hard temper tubing should be
19-1. Piping Materials. In general, the type used for all sizes above $ in. OD and for smaller
of piping material employed for refrigeration sizes when rigidity is desired.
piping depends upon the size and nature of the 19-2. Pipe Joints. Depending on the type and
installation, the refrigerant used, and the cost of sizeof the piping, joints for refrigerant piping
materials and labor. Specific minimum require- may be screwed, flanged, flared, welded, brazed,
ments for refrigerant piping, with regard to type or soldered. When refrigerant pressures are
and weight of piping materials, methods of below 250 psi, screwed joints may be used on
joining, etc., are set forth in the American pipe sizes up to 3 in. For higher pressures,
Standard Safety Code for Mechanical Refrig- screwed joints are limited to pipe sizes 1
J in.
eration (ASA Standard B9.1). Since the speci- and smaller. Above these sizes, flanged joints of
fications in this standard represent good, safe, the tongue and groove type should be used.
piping practice, they should be closely followed. Screwed-on flanges are limited to the pipe sizes
Too, in all cases, local codes and ordinances listedabove. For larger sizes, welded-neck
must be taken into account. flanges are required. A joint compound, suit-
The materials most frequently used for refrig- able for refrigerant piping and applied to the
erant piping are black steel, wrought iron, male threads only, should be used with all screw
copper, and brass. All these are suitable for use connections.
with all the common refrigerants, except that Welding is probably the most commonly used
copper and brass may not be used with ammonia, method of joining iron and steel piping. Pipes
since, in the presence of moisture, ammonia 2 in. and over are usually butt-welded, whereas
attacks nonferrous metals. those l£in. and smaller are generally socket-
Copper tubing has the advantage of being welded. Branch connections should be re-
lighter in weight, more resistant to corrosion, inforced.
and easier to install than either wrought iron Flared compression fittings may be used for
or black steel. With all refrigerants except connecting soft temper copper tubing up to size
ammonia, refrigerant lines up to 4& in. OD may f in. OD. Above this size and for hard temper
be either copper or steel. All lines above this copper tubing, joints should be made with
size should be steel. However, general practice sweat fittings using a hard solder. Hard solders
is to use all steel pipe in any installation where are silver brazing alloys with melting tempera-
a considerable amount of piping exceeds 2 in. tures above 1000° F. Soft solder (95 % tin and
in size. Wrought iron pipe, although more 5% antimony), having a melting point below
expensive than black steel, is sometimes used in 500° F, may be used for tubing \ in. OD and
place of the latter because of its greater resistance smaller. A suitable noncorrosive soldering flux
to corrosion. should be used with both types of solder.
365
366 PRINCIPLES OF REFRIGERATION

Flare fittings should be forged brass, and usually amounts to approximately fin. per
sweat fittings may be either wrought copper or hundred feet of piping. This is not ordinarily a
forged brass. Cast sweat fittings #re not suitable serious problem, since refrigerant piping is
for refrigeration duty. As a general rule, better usually three dimensional and therefore suffi-
fitting joints will result when tubing and fittings absorb the small changes in
ciently flexible to
are obtained from the same manufacturer. length.However, care should be taken not to
19-3. Location. In general, refrigerant piping anchor rigidly both ends of a long straight
should be located so that it does not present a length of pipe.
safety hazard, obstruct the normal operation 19-4. Vibration and Noise. In most cases,
and maintenance of the equipment, or restrict the vibration and noise in refrigerant piping
the use of adjoining spaces. When the require- originates not in the piping itself but in the
ments of refrigerant flow will permit, piping connected equipment. However, regardless of
would be at least 7£ ft above the floor, unless the source, vibration, and the objectional noise
installed against the wall or ceiling. The piping associated with it, is greatly reduced by proper
code prohibits refrigerant piping in public hall- piping design. Often, relatively small vibrations
ways, lobbies, stairways, elevator shafts, etc., transmitted to the piping from the connected
except that it may be placed across a hallway equipment are amplified by improperly designed
provided that there are no joints in the hallway piping to the extent that serious damage to
and that nonferrous pipe 1 in. and smaller is the piping and/or the connected equipment
encased in rigid metal conduit. results.
The arrangement of the piping should be such For the most part, vibration in refrigerant
that it is easily installed and readily accessible piping caused by the rigid connection of the
is

for inspection and maintenance. In all cases, piping to a reciprocating compressor, by gas
the piping should present a neat appearance. pulsations resulting from the opening and
All lines should be run plumb and straight, and closing of the valves in a reciprocating com-
parallel to walls, except that horizontal suction pressor, and by turbulence in the refrigerant gas
lines, discharge lines, and condenser to receiver due to high velocity. When centrifugal and
lines should be pitched in the direction of rotary compressors are used, vibration and noise
flow. in the refrigerant piping
is not usually a serious
All piping should be supported by suitable problem, being caused only by the latter of the
ceiling hangers or wall brackets. The supports above three factors. The reason lies in the
should be close enough together to prevent the rotary motion of the centrifugal and rotary
pipe from sagging between the supports. As a compressors and in the smooth flow of the gas
general rule, supports should not be more than into and out of these units, as compared to the
8 to 10 ft apart. A
support should be placed not pulsating flow through the reciprocating-type
more than 2 ft away from each change in compressor.
direction, preferably on the side of the longest Since a small amount of vibration is inherent
run. All valves in horizontal piping should be in the design of certain types of equipment, such
installed with the valve stems in a horizontal as reciprocating compressors, not possible
it is

position whenever possible. All valves in copper to eliminate vibration completely. However, if
tubing smaller than 1 in. OD
should be sup- the piping immediately adjacent to such equip-
ported independently of the piping. Risers may ment is designed with sufficient flexibility, the
be supported either from the floor or from the vibration will be absorbed and dampened by the
ceiling. piping rather than transmitted and amplified by
When piping must pass through floors, walls, it. On small units piped with soft temper copper

or ceilings, sleeves made of pipe or formed tubing, the desired flexibility is obtained by
galvanized steel should be placed in the openings. forming vibration loops in the suction and
The pipe sleeves should extend 1 in. beyond each discharge lines near the point where these lines
side of the openings and curbs should be used are connected to the compressor. If properly
around pipe sleeves installed in floors. designed and placed, these loops will act as
Provisions must be made also for the thermal springs to absorb and dampen compressor
expansion and contraction of the piping which vibration and prevent its transmission through
REFRIGERANT PIPING AND ACCESSORIES 367

the piping to other parts of the system. Where


the compressor is piped with rigid piping,
vibration eliminators (Fig. 19-1) installed in the
suction and discharge lines near the compressor WELDED

are usually effective in dampening compressor


vibration. Vibration eliminators should be
placed in a vertical line for best results.
On larger systems, adequate flexibility is ordin- SEAMLESS
FLEXIBLE
arilyobtained by running the suction and dis-
TIN BRONZE
charge piping approximately 30 pipe diameters (MHXCOPHt,
114% TM)
in each of two or three directions before
TUBING
anchoring the pipe. In all cases, isolation type
hangers and brackets should be used when
piping is supported by or anchored to building
construction which may act as a sounding board
to amplify and transmit vibrations and noise in HIGH TENSILE

the piping. KONZE


WIRE
Although vibration and noise resulting from MAIDING
gas pulsations can occur in both the suction and man conm.
1.71% Hi mix

discharge lines of reciprocating compressors,


it is much more frequent and more intense in the
discharge line. As a general rule, these gas
pulsations do not cause sufficient vibration and
noise to be of any consequence. Occasionally,
however, the frequency of the pulsations and the COPPER
FERRULE
design of the piping are such that resonance is
established, with the result that the pulsations
are amplified and sympathetic vibration (as with COPPER

a tuning fork) is set up in the piping. In some TUBE END

instances, vibration can become so severe that


the piping is torn loose from its supports.
Fig. 19-1. Vibration eliminator. (Courtesy Anaconda
Fortunately, the condition can be remedied by
Metal Hose Division, The American Brass Company.)
changing the speed of the compressor, by in-
stalling a discharge muffler, and/or by changing
the size of length of the discharge line. Since
changing the speed of the compressor is not general, refrigerant piping should be so designed
usually practical, the latter two methods and installed as to:
are better solutions, particularly when used
together. 1. Assure an adequate supply of refrigerant
When and noise are caused by gas
vibration to all evaporators
turbulence resulting from high velocity, the 2. Assure positive and continuous return of
usual remedy is to reduce the gas velocity by oil to the compressor crankcase
increasing the size of the pipe. Sometimes this 3. Avoid excessive refrigerant pressure losses
can be accomplished by installing a supple- which unnecessarily reduce the capacity and
mentary pipe. efficiencyof the system
19-5. General Design Considerations. Since 4. Prevent liquid refrigerant from entering
many of the operational problems encountered the compressor during either the running or off
in refrigerationapplications can be traced cycles, or during compressor start-up
directly to improper design and/or installation 5. Avoid thetrapping of oil in the evaporator
of the refrigerant piping and accessories, the or suction line which may subsequently return
importance of proper design and installation to the compressor in the form of a large "slug"
procedures cannot be overemphasized. In with possible damage to the compressor.
368 PRINCIPLES OF REFRIGERATION
19-6. Suction Line Size. Because of its flow rate in cfm by the internal area of the pipe
relative location in the system, the size of the in square feet. The refrigerant flow rate in
suction piping is usually more critical than that pounds per minute per ton at various operating
of the other refrigerant lines. Undersizing of conditions can be determined from Charts
the suction piping will cause an excessive refrig- 19-2A, B, and C for Refrigerant-12, 22, and
erant pressure drop in the suction line and ammonia, respectively. Internal areas for
result in a considerable loss in system capacity various pipe sizes are listed in Table 19-1.
and efficiency. On the other hand, oversizing of
the suction piping will often result in refrigerant Example 19-1. A Refrigerant-12 system,
velocities which are too low to permit adequate with a capacity of 40 tons, is operating at a
20° F evaporator temperature and a 1 10° F con-
oil return from the evaporator to the compressor
densing temperature. Compute the refrigerant
crankcase. Therefore, the optimum size for the
velocity in the suction line, if the line is 3£ in.
suction piping is one that will provide the mini-
OD copper tube.
mum practical refrigerant pressure drop com-
mensurate with maintaining sufficient vapor Solution. From
velocity to insure adequate oil return. Chart 19-2A, the flow
Most systems employing oil miscible refrig- rate in pounds per min-
erants are so designed that oil return from the ute per ton 4.26
evaporator to the compressor through theis From Table 16-3, the
suction line, either by gravity flow or by entrain- specific of R-12 satu-

ment in the suction vapor. When the evaporator rated vapor at 20° F = 1.097 cu ft/lb

is located above the compressor and the suction


Refrigerant flow rate
line can be installed without risers or traps, the
in cfm for 40 tons = 4.26 x 1.097
x 40
oil will drain by gravity from the evaporator to
= 187 cfm
the compressor crankcase, provided that all
From Table 19-1, the
horizontal piping is pitched downward in the
internal area of 3 j in.
direction of the compressor. In such cases, the OD copper tube = 6.81 sqin.
minimum vapor velocity in the suction line is of Applying Equation
little importance and the suction piping can be 15-13, the refrigerant
sized to provide the minimum practical pressure velocity in the suction 187 cfm x 144
drop without regard for the velocity of the vapor. pipe 6.81 sq in.
This holds true also for any system employing 3850 fpm
a nonmiscible refrigerant and for any other
system where special provisions are made for oil In the interest of high system efficiency, good
return. design requires that the suction piping be sized
On the other hand, when the location of the so that the over-all refrigerant pressure drop in
evaporator and/or other conditions are such the line does not cause a drop in the saturated
that a riser is required in the suction line, the suction temperature of more than one or two
riser must be sized small enough so that the degrees for Refrigerants- 12 and 22, or more than
resulting vapor velocity in the riser under mini- one degree for ammonia. Since the pressure-
mum load conditions will be sufficiently high to temperature relationship of all refrigerants
entrain the oil and carry it up the riser and back changes with the temperature range, the maxi-
to the compressor. The minimum vapor velo- mum permissible pressure drop in the suction
city required for oil entrainment in suction risers piping varies with the evaporator temperature,
for various suction temperatures and pipe sizes decreasing as the evaporator temperature de-
are given in Charts 19-1 A and B for Refrigerants- creases. For instance, for Refrigerant-12 vapor
12 and 22, respectively. These velocities should at 40° F, the maximum
permissible pressure
be increased by 25 % to determine the minimum drop in the suction piping (equivalent to a 2° F
design velocity for a suction riser. drop in saturation temperature) is 1.8 psi,
In accordance with Equation IS- 13, for any whereas for Refrigerant-12 vapor at —40° F,
given flow rate and pipe size, the refrigerant the maximum permissible pressure drop in the
velocity in the pipe can be calculated by dividing suction line is only 0.4 psi.
REFRIGERANT PIPING AND ACCESSORIES 369

Tonnage capacities of various sizes of iron Suction line pres-


pipe and type L copper tubing at various suction sure loss in °F _ Actual equiv. length
temperatures are listed in Tables 19-2, 19-3, 50
and 19-4 for Refrigerants-12, 22, and ammonia, / Actual tons V'*
respectively. The values listed in the tables are X
\ Table tons/
based on a suction line pressure loss equivalent 18
_ 52J / 40 \
to 2° F per 100 ft of pipe for Refrigerants-12 and X
50 \30.6/
22, and 1°F for ammonia. The condensing = x (1.1 13)1 *
1.55
temperatureis taken as 100° F for Refrigerant-
=
1.88°F
12 and as 105° F for Refrigerant-22 and am- From the chart at the bottom of Table 19-2,
monia. In all cases, tonnage capacities at other the pressure loss in psi corresponding to 1 .88° F
condensing temperatures can be determined by at a 20° F suction temperature is approximately
applying the correction factors given at the 1.3 psi.

bottom of each table. Equations are also given When the suction piping is sized on the basis
at the bottom of the tables for correcting of a one or two degree drop in the saturated
tonnages for other pressure losses and equivalent suction temperature, as in the preceding
lengths. The following example will serve to example, the resulting vapor velocity will ordin-
illustrate the use of the tables. arily be sufficiently high to insure the return of
oil up a suction riser during periods of minimum

Example 19-2. A 40-ton, Refrigerant- 12 loading. However, exceptions to this are likely
system has an evaporator temperature of 20° F to occur in any system where the evaporator
and a condensing temperature of 1 10° F. If a temperature is low, where the suction line is
suction pipe 30 ft long containing six standard excessively long, and/or where the minimum
elbows is required, determine: system loading is less than 50% of the design
(a) the size of type L copper tubing required and
load. When any of the above conditions exist,
(b) the over-all pressure drop in the suction line
the vapor velocity should be checked for the
in psi.
minimum load condition to be sure that it will
be above the minimum required for successful
Solution. Adding 50% to the straight length oil entrainment in risers.
of pipe as a allowance establishes a trial
fitting
The widespread use of automatic capacity
equivalent length of 45 ft (30 ft x 1.5). From
control on modern compressors, in order to vary
Table 19-2, 3| in. OD
copper tubing has a
the capacity of the compressor to conform to
capacity of 34 tons based on a condensing
temperature of 100° F and a suction line changes in the system load, tends to complicate
pressure loss equivalent to 2° F per 100 ft of the design of all of the refrigerant piping.
pipe. Since the pressure loss is proportional to Through the use of automatic capacity control,
the length of pipe and since the equivalent single compressors are capable of unloading
length of pipe is only 45 ft in this instance, this down to as little as 25 % of the maximum design
pipe size may be sufficient and a trial calculation capacity. When two or more such compressors
should be made. From Table 15-1, 3 J in. OD are connected in parallel, the system can be
(3 in. nominal) standard elbows have an equiva-
designed to unload down to as little as 10% of
lent length of 3.8 ft.
Actual equivalent length of suction piping:
the combined maximum design capacity of the

Straight pipe
compressors. Obviously, when the system
length = 30 ft capacity varied over such a wide range, any
is

6 ells at 3.8 ft = 22.8 ft suction piping sized small enough to insure

Total equivalent vapor velocities sufficiently high to carry oil up


length = 52.8 ft a riser during periods of minimum loading will
cause a prohibitively high refrigerant pressure
Correction factor
from Table 19-3 to drop during periods of maximum loading. On
correct tonnage for the other hand, sizing the pipe for a low
1 10° F condensing pressure drop at maximum loading will result
temperature is 0.9. = 34 x 0.9 in riser velocities too low to return oil. For-
Corrected tonnage = 30.6 tons tunately, the vapor velocity in horizontal piping
370 PRINCIPLES OF REFRIGERATION

yTo compressor minimum loading will be too low for oil return
up the riser.
Therefore, the size of the riser must be
reduced. Try the next smaller pipe size, which
Pitch
is 2| in. OD. From Chart 19-1 A, the minimum
velocity for oil entrainment with 20° F vapor
_ Vertical riser sized in a 2§ in. OD
suction riser is 1300 fpm. By
for minimum loading increasing the table value by 25 %, the minimum
design velocity is 1625 fpm. From Table 19-1,
the internal area of 2f in. OD
copper tubing is
4.76 sq in. At the minimum load of 10 tons, the
vapor velocity in the riser will be

46.8 x 144
Eccentric = 1415 fpm
reducer 4776

Since this is still too low for adequate oil


entrainment, try the next smaller pipe size,
Pitch
From which is 2J in. OD. Using the same procedure,
evaporator
it is found that the minimum design velocity for

oil entrainment with 20° F vapor in a 2\ in. OD


Fig. 19-2. Illustrating method of reducing the size of suction riser is 1500 fpm, and that the vapor
a vertical suction riser. (Courtesy York Corporation.) velocity at the minimum load of 10 tons is 2180
fpm. Therefore, use 2\ in. OD
copper tube for
the riser and 3| in. OD
copper tube for the
is not critical and the problem is mainly one of balance of the suction piping.
riser design. In most cases, when the minimum The refrigerant pressure loss in the riser in
system load not less than 25
is of the design % degrees is (see Note 3 at the bottom of Table
load, the problem can be solved by reducing the 19-2)
size of the riser only, with the balance of the 10 40 y- a
/ °
suction piping being sized for a low pressure
50
X
((i2T)) =°- 248 F
drop at maximum loading.
When this is added to the pressure loss in the
Example 19-3. Assume that the minimum
balance of the suction piping, the over-all
load for the system described in Example 19-2
pressure loss will still be well within the
is 25 %of the design load, or 10 tons. Compute
acceptable limits.
the velocity of the vapor in the suction piping
under the minimum load condition and check Figure 19-2 illustrates the proper method of
in Table 19-1 to determine if it is sufficiently high reducing the line size at a vertical riser. An
to carry oil of the 10 ft riser. eccentric reducer with its flat side down should
be used at the bottom connection before entry
Solution. From Example 19-1, the flow rate
to the elbow. This is done to prevent forming an
in pounds per minute per ton is 4.26 and the
specific volume of the vapor is 1.097 cu ft/lb, so area of low gas velocity which could trap a layer
that the flow rate through the suction piping in of oil extending the length of the horizontal line.
cfm for 10 tons is 46.8 (4.26 x 1.097 x 10). At the top of the riser, the line size is increased
From Table 19-1, the internal area of 3 J in. OD beyond the elbow with a standard reducer, so
copper tubing is 6.81 sq in. Therefore, the velo- that any oil reaching this point cannot drain
city of the vapor in the suction piping is back into the riser.
46.8 x 144 19-7. Double-Pipe Risers. As a general rule,
= 993 fpm when the suction riser reasonably short and
is
6.81
the minimum system load does not fall below
From Chart 19-1 A, the minimum velocity for
oil entrainment with 20° F vapor in a 3| in. OD 25 % of the maximum design load, undersizing

suction riser is approximately 1430 fpm. In-


of the riser to provide adequate vapor velocity
creasing the table value by 25 %, the minimum during periods of minimum loading will not
design velocity is found to be 1775 fpm (1430 x cause a significant increase in the over-all suction
1.25). It is evident that the vapor velocity at line pressure drop, particularly if the horizontal
REFRIGERANT PIPING AND ACCESSORIES 371

portion of the piping is liberally sized. On the pressor during either the running or off cycles,
when the suction riser is quite long
other hand, or during compressor start-up. Generally,
and/or when the minimum system loading is unless the system is operated on a pump-down
less than 25 %
of the design loading, undersizing cycle, it is good practice to install a liquid-
of the riser to conform to the requirements of suction heat exchanger in the suction line of all
minimum loading will ordinarily result in an systems employing dry-expansion evaporators.
excessive pressure loss in the suction piping The reason for this is that thermostatic expan-
during periods of maximum loading, especially sion valves frequently do not close off tightly
in low temperature installations. In such cases, during the compressor off cycle, thereby per-
shown in Fig. 19-3, should
the double-pipe riser, mitting off cycle leakage of liquid refrigerant
be employed. into the evaporator from the liquid line. When
The small diameter riser is sized for the mini- the compressor starts, the excess liquid often
mum load condition, whereas the combined slops over into the suction line and is carried to
capacity of the two pipes is designed for the the compressor unless a liquid-suction heat
maximum load condition. The larger riser is exchanger is employed to trap the liquid and
trapped slightly below the horizontal line at the vaporize it before it reaches the compressor.
bottom. During periods of minimum loading, The liquid-suction heat exchanger also serves to
oil will settle in the trap and block the flow of trap and vaporize any liquid which may carry
vapor through the larger riser, thereby increasing over into the suction line because of overfeeding
the flow rate and velocity in the smaller of the expansion valve during start-up or during
riser to a level high enough to insure oil return sudden changes in the evaporator loading.
up the riser. As the system load increases, the Ordinarily, the liquid-suction heat exchanger
velocity increases in the small riser until the can be safely omitted if the system is operated
pressure drop across the riser is sufficient to on a pump-down cycle, in which case the liquid
clean the oil out of the trap and permit flow refrigerant will be pumped out of the evaporator
through both pipes. before the compressor cycles off, and the liquid
Notice that the trap at the bottom of the large line solenoid installed ahead of the refrigerant
riser is made up of two 45° elbows and one 90° control will prevent liquid refrigerant from
elbow. This is done to keep the volume of the entering the evaporator from the liquid line,
trapped oil as small as possible. Notice also even though the expansion valve itself may not
that inverted loops are used to connect both close off tightly.
risers to the top of the upper horizontal line, When the evaporator is located above the
so that oil reaching the upper line cannot drain compressor, and the system not operated on a
is

back into the risers. pump-down cycle, the suction line should be
19-8. General Design of Suction Piping. trapped immediately beyond the expansion
The suction piping should always be so arranged valve bulb, as shown in Fig. 19-4, so that liquid
as to eliminate the possibility of liquid refrig- refrigerant cannot drain by gravity from the
erant (or large slugs of oil) entering the com- evaporator to the compressor during the off

Fig. 19-3. Double suction


riser construction. (Courtesy
Carrier Corporation.)

U-bend or
2 ells

Method A Method B
'

372 PRINCIPLES OF REFRIGERATION


below 50% of the design capacity of the valve,
the flow rate through the individual risers should
never drop below 50% of the design flow rate.
Therefore, the use of individual risers for each
Without pump-down '

control —«. evaporator (or separately fed segment) should


eliminate the problem of oil return at minimum
With pump-down loading. Figures 19-7 through 19-9 illustrate
control some of the various methods of connecting
multiple evaporators to a common suction
main when it is not practical to use individual
risers.
Fig. 19-4. Evaporator located above compressor.
The suction piping at the compressor should
be brought in above the level of the compressor

cycle. If the system is operated on a pump-down


cycle, the trap may be omitted and the piping
arranged for free draining (dotted lines in Fig.
19-4), since all the liquid is pumped from the
evaporator before the compressor cycles off.

When the evaporator is located below the


compressor and the suction riser is installed
immediately adjacent to the evaporator, the
riser should be trapped as shown in Fig. 19-5 to
prevent liquid refrigerant from trapping at the
thermal bulb location. In the event that trapping
of the line is not practical, the trap may be

omitted and the thermal bulb moved to a posi-


tion on the vertical riser approximately 12 to 18
in. above the horizontal header, as shown by the
dotted line in Fig. 19-5.
When a multiple of evaporators is to be
connected to a common suction main, each
evaporator (or separately fed evaporator seg-
ment) should be connected to the main with an
Fig. 19-6. Multiple evaporators, individual suction
individual riser, as shown in Fig. 19-6. Since
lines.
thermostatic expansion valves do not perform
properly when the load on the evaporator falls

suction inlet. The piping should be designed


Preferred
without liquid traps and so arranged that the oil
will drain by gravity from the suction line into
the compressor. When multiple compressors
are connected to a common suction header, the
piping should be designed so that oil return to
the several compressors is as nearly equal as
possible. The lines to the individual compressors
should always be connected to the side of the
header. Some typical piping arrangements for
multiple compressors are shown in Figs. 19-10
and 19-11.
19-9. Discharge Piping. Sizing of the dis-
charge piping is similar to that of the suction
Fig. 19-5. Evaporator below compressor. piping. Since any refrigerant pressure drop in
REFRIGERANT PIPING AND ACCESSORIES 373

the discharge piping tends to increase the com-


pressor discharge pressure and reduce the capa-
cityand efficiency of the system, the discharge
piping should be sized to provide the minimum
practical refrigerant pressure drop. Tonnage
capacities for various sizes of discharge pipes
are given in Tables 19-2, 19-3, and 19-4. The
values listed in the tables are based on an over-
all refrigerant pressure drop per 100 ft of equiva-
lent length corresponding to a 2° F drop in the
saturation temperature of Refrigerants- 12 and

Fig. 19-8. Evaporators at different levels connected


to a common suction riser.

vapor velocity in the riser under minimum load


conditions is sufficiently high to entrain the oil
and carry it up the riser. Minimum vapor velo-

citiesfor oil entrainment in discharge risers are


given in Charts 19-1C and D
for Refrigerants- 12
and 22, respectively. The table values should be
increased by 25% to determine the minimum
design velocity. When the
system capacity
varies over a wide range, a double-pipe riser may
be necessary, unless a discharge line oil sepa-
rator is used. When an oil separator is installed
Fig. 19-7. Multiple evaporators, common suction
line.

22,and for a 1° F loss in saturation temperature


forammonia. The procedure for sizing the dis-
charge piping is the same as that used for the
suction piping.
All horizontal discharge piping should be
pitcheddownward in the direction of the refrig-
erant flow so that any oil pumped over from
the compressor into the discharge line will drain
toward the condenser and not back into the com-
pressor head. Although the minimum vapor
velocity in horizontal discharge piping is not
ordinarily critical, special attention must be
given to the vapor velocity in discharge line
risers. As in the case of a suction riser, the Fig. 19-9. Evaporators at different levels connected
discharge line riser must be designed so that the to a double suction riser.
374 PRINCIPLES OF REFRIGERATION
draining from the vertical riser during the off
cycle will drain into the oil separator. How-
Alternate
ever, certain precautions must be taken to
approaches
eliminate the possibility of liquid refrigerant
passing from the oil separator to the com-
pressor crankcase during the off cycle (see
Section 19-12).
A purge valve, to permit purging of non-
condensible gases from the system, should be
installed at the highest point in the discharge
piping or condenser.
When two or more compressors are connected
together for parallel operation, the discharge
piping at the compressors must be arranged so
that the oil pumped over from an active com-
pressor does not drain into an idle one. Under
no circumstances should the piping be arranged
so that the compressors discharge directly into
one another. As a general rule, it is good
Fig. 19-10. Suction piping for compressors connected
practice to carry the discharge from each com-
in parallel.
pressor nearly to the floor before connecting to a
i

common discharge main (Fig. 19-14). With this


in the discharge line, the vapor velocity in the piping arrangement, a discharge line trap is not
discharge riser is not critical and the riser should required at the riser, since the lower horizontal
header serves this purpose.
be sized for a low pressure drop, since any oil
which is not carried up the riser during periods
Alternate
of minimum loading will drain back into the
approach~N i 1
,
separator (Fig. 19-12).
When the compressor is not operating, the oil
adhering to the inside surface of a discharge
riser tends to drain by gravity to the bottom of
is more than 8 to 10 ft long,
the riser. If the riser
the amount of oil draining from the riser may be
quite large. Therefore, the discharge line from
the compressor should be looped to the floor to
form a trap so that oil cannot drain from the
discharge piping into the head of the com-
pressor. Since this trap will also collect any
liquid refrigerant which may condense in the
discharge riser during the compressor off cycle,
it is especially important when the discharge

piping is in a cooler location than the liquid


receiver and/or condenser. Additional traps,
of vertical should be installed Oil equalizer
one for 25 ft rise,
line
in the discharge riser when the vertical rise
exceeds 25 ft, as shown in Fig. 19-13. The
horizontal width of the traps should be held to a
minimum and can be constructed of two stan- Crankcase y
pressure /
dard 90° elbows. The depth of the traps should
equalizer
be approximately 18 in.
The traps may be omitted if an oil separator Fig. 19-11. Suction piping for com pressors connected
is used, since any oil or liquid refrigerant in parallel.
REFRIGERANT PIPING AND ACCESSORIES 375

Fig. 19-12. Arrange for pre- Pitch down to

venting liquid return to com-


separator N
pressor crankcase. 3__r
r. Oil
separator

^Sight^Oil return line


glass
Solenoid *J
for free draining, either in a horizontal line or
in a down-comer, as shown in Fig. 19-14, but
never in a riser.
19-10. Liquid Lines. The function of the
liquid line is to deliver a solid stream of sub-
cooled liquid refrigerant from the receiver tank
to the refrigerant flow control at a sufficiently
high pressure to permit the latter unit to operate
efficiently. Since the refrigerant is in the liquid
state, any oil entering the liquid line is readily
carried along by the refrigerant to the evapora-
no problem with oil return in
tor, so that there is
liquid lines. For this reason, the design of the
liquid piping is somewhat less critical than that
of the other refrigerant lines, the problem
encountered being mainly one of preventing the
liquid from flashing before it reaches the refrig-
erant control.

Fig. 19-13. Piping of discharge riser.

In the event that the discharge header must be


located above the compressors, the discharge
from the individual compressors should be
connected to the top of the header, as shown in
Fig. 19-15, so that oil cannot drain from the
header into the head of an idle compressor.
In order to reduce the noise and vibration
created by the compressor discharge pulsations, Pitch down
discharge mufflers arerecommended for all
multiple compressor installations and for a
single compressor installation where the noise
of the discharge pulsations may become objec- Fig. 19-14. Discharge piping of multiple compressors
tionable. Discharge mufflers must be installed connected in parallel.
376 PRINCIPLES OF REFRIGERATION
19-2, 19-3, and 19-4. The pressure drop per foot
of vertical lift is found in Table 19-6.

Example 19-4. A
Refrigerant- 12 system
has a capacity of 35 tons. The equivalent length
of liquid line including fittings and accessories
is 60 ft. If the line contains a 20 ft riser,
determine:
(a) the size of the liquid line required
(b) the over-all pressure drop in the line
(c) the amount of subcooling (°F) required to
prevent flashing of the liquid.

Solution
(a) From Table 19-2, If in. OD
copper tubing
has a capacity of 3S.1 tons based on a 1.8 psi
pressure drop per 100 equivalent feet of pipe.
(6) For 60 ft equivalent
length, the friction loss
in the pipe = 1.8 psi x 0.6
= 1.00 psi
Pressure loss due to 20 ft
Fig. 19-15. Discharge piping for multiple compressors lift = 0.55 psi x 20
connected in parallel. = 11.00 psi
Over-all pressure drop = 1 +11.0
= 12.0 psi

Flash gas in the liquid line reduces the capa- (c) Assuming the condensing temperature to be
city of the refrigerant control, causes erosion of 100° F, the pressure at the condenser is 131.6
the valve pin and seat, and often results in psia. The pressure at the refrigerant control
erratic control of the liquid refrigerant to the is 119.6 psia, which corresponds to a satura-
evaporator. To avoid flashing of the liquid in tion temperature of approximately 93° F.
the liquid line, the pressure of the liquid in the The amount of subcooling required is approxi-
line must be maintained above the saturation mately 7° F (100° - 93°).

pressure corresponding to the temperature of 19-11. Condenser to Receiver Piping. Since


the liquid. the amount of refrigerant in the evaporator and
Since the liquid leaving the condenser is condenser varies with the loading of the system,
usually subcooled 5 to 10° F, flashing of the a liquid receiver tank is required on all systems
liquid will not ordinarily occur if the over-all employing hand expansion valves, automatic
pressure drop does not exceed 5 to 10 psi. How-
ever, if the pressure drop is much in excess of
10 psi, it is very likely that some form of liquid
subcooling will be required if flashing of the
liquid is to be prevented. In most cases, a
liquid-suction heat exchanger and/or a water-
cooled subcooler will supply the necessary sub-
cooling. In extreme cases, a direct-expansion
subcooler may be required. (Fig. 19-16.)
The amount of subcooling required in any
individual installation can be determined by
computing the liquid line pressure drop.
Pressure drop in the liquid line results not only
from friction losses but also from the loss of
head due to vertical lift. Tonnage capacities of
various sizes of liquid pipes are listed in Tables Fig. 19-16.
REFRIGERANT PIPING AND ACCESSORIES 377

expansion valves, thermostatic expansion valves,


Inlet Equalizer Outlet
or low pressure float valves. Some exceptions to
i

T connection
this are installations using water-cooled con-
densers, wherein the water-cooled condenser
also serves as the liquid receiver. In addition to
accommodating fluctuations in the refrigerant
charge, the receiver tends to keep the condenser
drained of liquid, thereby preventing the liquid
level from building up in the condenser and
reducing the amount of effective condenser
surface. The liquid receiver serves also as a Equalizer i Outlet
connection "
pump-down storage tank for the
refrigerant.
liquid
H
In general, the condenser to receiver piping
must be so designed and sized as to allow free
draining of the liquid from the condenser at all
times. If the pressure in the receiver is permitted
to rise above that in the condenser, vapor
binding of the receiver will occur and the liquid
refrigerant will not drain freelyfrom the con- Equalizer
denser. Vapor binding of the receiver is likely connection

to occur in any installation where the receiver _Q_


is so located that it can become warmer than

the condenser. The problem of vapor binding


is more acute in the wintertime and during

periods of reduced loading.


Inlet and
Although the exact preventative measures outlet
which can be taken to eliminate vapor binding
of the receiver depends somewhat on the type of Fig. 19-17. (a) Top inlet through-flow receiver, (b)
condenser, in every instance it involves proper Bottom inlet through-flow receiver, (c) Surge-t/pe
receiver.
equalization of the receiver pressure to the
condenser.
Basically, there are two types of liquid
receivers: the through-flow and the surge (Fig. 19-18). All horizontal piping should be pitched
19-17). The through-flow type may be either toward the receiver at least \ in. per foot. When
bottom inlet or top inlet. With the through-flow a stop valve is placed in the line, it should be
type receiver, all the liquid from the condenser located a minimum distance of 8 in. below the
drains into the receiver before passing into the liquid outlet of the condenser and should be
liquid line. The surge type differs from the installed so that the valve stem is in a horizontal
through-flow in that only a part of the liquid position. In the event that a trap in the line is
from the condenser, that part not required in unavoidable, a separate equalizing line must be
the evaporator, enters the receiver. With the installed from the top of the receiver to the con-
surge-type receiver, the refrigerant liquid enters denser, as shown in Fig. 19-19. When an
and leaves the receiver through the same equalizing line is used, the condenser to receiver
opening. piping may be sized for a refrigerant velocity of
When a top-inlet, through-flow type receiver 150 fpm.
is used, equalization of the receiver pressure to All bottom inlet receivers of both the through-
the condenser can be accomplished directly flow and surge types must be equipped with
through the condenser to receiver piping, pro- equalizing lines. The minimum vertical distance
vided that the piping is sized so that the refrig- (Aa of Fig. 19-19) between the outlet of the con-
erant velocity does not exceed 100 fpm and denser and the maximum liquid level in the
that the line is not trapped at any point (Fig. receiver to prevent the back-up of liquid in the
378 PRINCIPLES OF REFRIGERATION

^Purge valve

High pressure liquid


to expansion valve Fig. 19-18. Top inlet through
type receiver hookup. (Cour-
tesy York Corporation.)

This line sized so that at full


load, the velocity does not
exceed 100 ft/min. slope
horizontal line toward receiver
H" per ft. or more.

Purge
\\/~ valve

-Gas inlet ^Equalizer

Highest expected
/operating liquid level

H ^/condenser s^ s"
Surge type
receiver
jr k
%!

nU R" ^X^H. P. liquid


to exp. valve

Fig. 19-19. Surge-type receiver


h2
JmL P^L *^'B"
\y$i&mBr%'
hookup. (Courtesy York
Corporation.)
^3\ Size this line so the
^velocity at max. load
does not exceed 1507min.

Maximum Velocity of Type Valve between h% Required


Drain line, Ibs/min Condenser and Receiver inches

150 None 14
150 Angle 16
150 Globe 28
100 None, Angl e, or Globe 14

Size drain line to receiver for maximum velocity of 150 ft/min.


If a valve is located in this line, the trapping height limitation may
require a larger size line to minimize the pressure drop.
REFRIGERANT PIPING AND ACCESSORIES 379

condenser is listed at the bottom of Fig. 19-19.


Gas inlet
This value increases as the pressure loss in the ^-Equalizer
condenser to receiver line increases, but should £ 3*
never be less than 12 in. (I I Condenser

Figure 19-20 illustrates a satisfactory piping


Detail "A"
arrangement for multiple condensers connected
A
in parallel to a top inlet receiver.
arrangement for multiple condensers with a
piping
-« — Gas inlet

bottom inlet receiver is shown in Fig. 19-21.


The vertical distance h2 is determined from the
bottom of Fig. 19-19.
19-12. Oil Separators. As a general rule, Equalizer
discharge line oil separators should be employed
Relief valve
in any system when oil return is likely to be
inadequate or difficult to accomplish and/or
when the amount of oil in circulation is apt to be
excessive or to cause an undue loss in the effi-
High \ "quid level
pressure i

ciency of the various heat transfer surfaces. . receiver

Specifically, discharge line oil separators are


recommended for: (1) all systems employing Valve elevations should To evaporator
nonmiscible refrigerants, (2) low temperature -be below liquid level

systems, (3) all systems employing nonoil- in the receiver

returning evaporators, such as flooded liquid Fig. 19-21. Parallel shell-and-tube condensers with

chillers, when oil bleeder lines or other special bottom inlet receiver. (Courtesy York Corporation.)

provisions must be made for oil return, and (4)


any system where capacity control and/or long
suction or discharge risers cause serious piping On entering the separator, the velocity of the
design problems. refrigerant vapor is considerably reduced
Discharge line oil separators are of two basic because of the larger area of the separator with
types: (1) impingement and (2) chiller. The relation to that of the discharge line, whereupon
impingement-type separator (Fig. 19-22) con- the oil particles, having a greater momentum
sists of a series of screens or baffles through than that of the refrigerant vapor, are caused to
which the oil laden refrigerant vapor must pass. impinge on the surface of the screens or baffling.
The oil then drains by gravity from the screens
or baffles into the bottom of the separator, from
Gas inlet
where it is returned through a float valve to the
compressor crankcase or, preferably, to the
suction inlet of the compressor (Fig. 19-12).
The water-cooled, chiller-type separator,
sometimes called an oil chiller, is similar in
construction to the water-cooled condenser.
Water is circulated through the tubes while the
Size for lOO'/min
velocity at max discharge vapor passes through the shell. The
flow through oil is separated from the vapor by precipitation
each condenser
on the cold water tubes, from where it drains
into a drop-leg sump. The oil may be drained
- Slope header towards
receiver
manually from the sump or returned auto-
J* per ft or more.
Size entire header for matically to the compressor through a float
lOO'/min velocity at the valve. (Fig. 19-23.) The water flow rate through
total design flow rate. the separator must be carefully controlled so
Fig. 19-20. Parallel shell-and-tube condensers with that the refrigerant vapor is not cooled below
top inlet receiver. (Courtesy York Corporation.) its condensing temperature, in which case liquid

380 PRINCIPLES OF REFRIGERATION

Fig. 19-22. Impingement-type


oilseparator with float drainer.
(Courtesy York Corporation.)

refrigerant could be condensed in the separator parts of the system. Some of the various methods
and passed to the compressor crankcase through of accomplishing this are described in the
the float valve. following sections.
In some low temperature applications, a The principal hazard associated with the use
direct-expansion, chiller-type separator is in- of a discharge line oil separator is the possibility

stalled in the liquid line. The operation and of liquid refrigerant passing from the oil sepa-
installation of the direct-expansion oil separator rator to the compressor crankcase when the
are similar to those of the direct-expansion compressor is idle. The liquid refrigerant may
liquid subcooler described in Section 19-10. The drain into the separator from the discharge
liquid refrigerant passing through the separator piping or it may condense in the separator itself
is below the pour point
chilled to a temperature during the off cycle.
of the oil, thereby causing the o" to congeal on While the compressor is operating, the tem-
the chiller tubes. The oil is drained from the perature of the oil separator is relatively high

separator periodically by taking the separator and the of liquid refrigerant con-
possibility
out of service and allowing it to warm up to a densing in the oil separator is rather remote,
temperature above the pour point of the oil. particularly if the separator is located reasonably
Although properly applied oil separators are close to the compressor. However, after the
usually very effective in removing oil from the compressor cycles off, the separator tends to
refrigerant vapor, they are not 100% efficient. cool to the condensing temperature, at which
Therefore, even when an oil separator is used, time some of the high pressure refrigerant vapor
some means should be provided for removing is likely to condense in the separator. This
the small amount of oil which will always pass raises the liquid level in the separator and causes
through the separator and find its way into other the float to open and pass a mixture of oil and

Discharge from compressor To


-*- -»
£_ condenser

Fig. 19-23.
chiller-type
Application
oil
of
separator.
(Courtesy York Corporation.)
a Trap'

Oil return
<
Oil sight glass-
I
Water
Her,connections

compressor ^Manual drain valve


crankcase
High pressure float
REFRIGERANT PIPING AND ACCESSORIES 381

liquid refrigerant to the compressor crankcase. into the discharge line from the compressor is
Condensation of the refrigerant in the separator not readily carried along through the system by
is most apt to occur when the oil separator is the refrigerant. Therefore, an oil separator
installed in a cooler location than the condenser, should be installed in the discharge line of all
in which case the liquid will boil off in the con- ammonia systems to reduce to a minimum the
denser and condense in the separator. amount of oil that passes into the system. Pro-
To eliminate the possibility of liquid refrig- visions must be made also to remove from the
erant draining from the oil separator into the system and return to the crankcase the small
compressor crankcase during the off cycle, the amount of oil that gets by the separator.
oil drain line from the separator should be Oil, being heavier than liquid ammonia, tends
connected to the suction inlet of the compressor, to separate from the ammonia and settle out at
rather than to the crankcase, and the line should various low points in the system. For this
be equipped with a solenoid valve, a sight glass, reason, oil sumps are provided at the bottom of
a hand expansion valve, and a manual shut-off all receivers, evaporators, accumulators, and
valve (see Fig. 19-12). With the help of the other vessels in the system containing liquid
sight glass, the hand throttling valve can be ammonia, and provisions are made for draining
adjusted so that the liquid (oil and refrigerant) the oil from these points either continuously or
from the separator is bled slowly into the suction periodically. Since the amount of oil involved is
inlet of the compressor. The solenoid valve is small, when an operator is on duty the draining
interlocked with the compressor motor starter is usually done manually and the oil discarded.

so that it is energized (open) only when the com- Of course, this requires that the crankcase oil
pressor is operating. This arrangement prevents be replenished periodically.
the liquid refrigerant and oil in the separator Since the lubricating oil is not returned to the
from draining to the compressor during the off compressor through the refrigerant piping, the
cycle, but permits slow draining into the suction minimum vapor velocity in ammonia piping is of
inlet of the compressor when the compressor is no consequence and the piping is sized for a low
operating. pressure drop without regard for the minimum
To minimize the condensation of refrigerant vapor velocity.
vapor in the oil separator during the off cycle, 19-14. Nonoil Returning Halocarbon Eva-
oil separators should be installed near the com- porators. The design of some halocarbon
pressor in as warm a location as possible. The evaporators is such that the oil reaching the
separators should also be well insulated in order evaporator cannot be entrained by the refrig-
to retard the loss of heat from the separator erant vapor and carried over into the suction
after the compressor cycles off. line and back to the compressor. The more
In some instances, the oil from the separator common of these evaporators are flooded liquid
is drained into an oil receiver where it is stored chillersand certain types of air-cooling evapora-
untilneeded in the compressor crankcase (Fig. tors that are operated semiflooded by bottom-
17-39). The oil receiver contains a heating feeding with a thermostatic expansion valve.
element which boils off the liquid refrigerant to In both cases, the problem with oil return results
the suction line. Oil from the oil receiver is from the lack of sufficient refrigerant velocity
admitted to the compressor crankcase as needed and turbulence in the evaporator to permit
through a float valve located in the compressor entraining the oil and carrying it over into the
crankcase. suction line.
Oil receivers cannot be used with compressors With the semiflooded evaporator, oil return
equipped with crankcase oil check valves. Since from the evaporator is usually accompanied by
the oil receiver is at the suction pressure, the adjusting the expansion valve for a low super-
higher crankcase pressure could force all the oil heat and slightly overfeeding the evaporator so
out of the crankcase into the oil receiver. When that a small amount of the oil-rich liquid
oil receivers are employed, oil check valves must refrigerant in the evaporator is continuously
be removed and crankcase heaters installed. carried over into the suction line. This arrange-
19-13. Ammonia Piping. Since ammonia is ment will ordinarily keep the oil concentration
a nonmiscible refrigerant, the oil pumped over in the evaporator within reasonable limits. As
382 PRINCIPLES OF REFRIGERATION

shown in Fig. 19-24, a liquid-suction heat directly to the suction line, a liquid-suction heat
exchanger is installed in the suction line to exchanger is required in order to evaporate the
evaporate the liquid refrigerant from the oil- liquid refrigerant from the bleed mixture before
refrigerant mixture before the latter reaches the it reaches the compressor. Figures 17-39 and
compressor. 17-41 illustrate typical piping arrangements'
Flooded liquid equipped
chillers are usually when bleed off is to an oil receiver.
with oil bleeder lines which permit a measured 19-15. Crankcase Piping for Parallel Com-
amount of the oil-rich liquid in the chiller to be pressors. When two or more compressors
bled off into the suction line or, in some are operating in parallel off a common suction
instances, into an oil receiver. A throttling line, it is very unlikely that the oil returning
valve is installed in the bleeder line so that the through the suction line will be evenly distri-
flow rate through the line can be adjusted. The buted to the several compressors. Too, it is
bleeder line also contains a solenoid valve which very unlikely that the amount of oil pumped
is wired to open only when the compressor is over by any two compressors will be exactly the
operating, so that flow through the bleed line same even when the compressors are alike in
does not occur when the compressor is idle. both design and size. For these reasons, when
Since Refrigerant- 12 is completely oil miscible compressors are piped for parallel operation,
at all temperatures above the pour point of the it is necessary to interconnect the crankcases of

oil, the oil bleeder connection on Refrigerant- 12 the several compressors both above and below
chillers may be located at any point below the the crankcase oil level, as shown in Fig. 19-11.
liquid level in the chiller. Refrigerant-22, on the This requires that the compressors be so
other hand, is partially oil miscible at evaporator placed on their respective foundations that the
temperatures. Therefore, when refrigerant tur- oil tappings of the individual compressors are
bulence in the evaporator is relatively low, there all at exactly the level. The crankcase oil
same
isa tendency for the oil-refrigerant mixture in equalizing line may
be installed either level with
the evaporator to separate into two layers, with or, preferably, below the level of the crankcase

the upper layer containing the greater concen- oil tappings. Under no circumstances should
tration of oil. For this reason, the oil bleeder the oil equalizing line be allowed to rise above
connection of Refrigerant-22 chillers should be the level of the crankcase tappings.
located just above the midpoint of the liquid Since any small difference in the crankcase
level in the chiller. pressure of the several compressors will cause
As shown in Fig. 17-30, when the bleed off is a difference in the crankcase oil levels, it is

Liquid-suction
interchanger

T ^ T
Hand
-expansion
valve
Fig. 19-24. Forced circulation
air cooler with direct expan-
Strainer-*' \ Fan
sion feed of flooded-type coil.
Thermal \
(Courtesy Carrier Corpora-
expansion'
tion.)
valve

Size suction riser for


oil return at lowest

possible loading
S .
Coil

Base pan
COMPRESSOR CONSTRUCTION ANO LUBRICATION 3*3

refrigerating capacity an J the power require- Because of it* extreme sensitivity to changes
ments of the compressor, although (he horse* in speed, the centrifugal compressor is ideally
power required per ton increases. suited for capacity regulation by means of
With regard to compressor speed, the centri- variable speed drives, such as steam turbines
fugal compressor is much more sensitive 10 and wound-rotor induction motors. When con-
speed changes than is the reciprocating type. stant speed such as synchronous or
drives,
Whereas the change in the capacity of the squirrel cage motors, are employed, speed con-
reciprocating compressor is approximately pro- trol can be obtained through the use of a

portional to the speed change, according hydraulic or magnetic clutch installed between
to the performance curves in IB-3J, a Fig. the drive and the step- up gear.
speed change of only 12% will cause a 50% 11-24. Centrifugal Refrigeration Machine*.
reduction in he capacity of the centrifugal
I Centrifugal compressors arc available for re-
compressor. frigeration duly only as an integral part of a
18-25. Capacity Control, Capacity control centrifugal refrigerating machine. Because of
of centrifugal compressors is usually accom- the relatively flat head-capacity characteristic of
plished by one of the following three methods; the centrifugal compressor, and the resulting

0) varying the speed of the compressor, (2} limitation in the operating range, the component
varying the suction pressure by means of n parts of a centrifugal refrigerating system must
suction throttling damper, or (3J varying he I be very carefully balanced. Too, since very
condensing temperature through control of the marked change* in capacity accrue with only
condenser water. Often, some combination of minor changed in the suction or condensing
these methods » used. temperatures, the centrifugal refrigerating system

Fig. ti-12. Ctmrifui»l rifrig«r»tlnj mithtaa. (Courteiy Wonhinpofl CorppraUon)


REFRIGERANT PIPING AND ACCESSORIES »7

the piping Co other parts of the system. Where


the compressor is piped with rigid piping,
vibration eliminators (Fig. 19-1) installed in the
lines near the compressor WELHD
suction and discharge
are usually effective in dampening compressor
vibration.
placed
Vi bra lion eliminators should be
in a vertical line for besl result*.
. I
SiiMUSS
On larger systems, adequ ate neslhil ity is ord in-
HEIItLI
arily obtained by running I he suction and dis-
TIN WWII
charge piping approximately 30 pipe diameters
in each of two or three directions before TDIlMi
anchoring the pipe. In all cases, Isolation type
hangers and brackets should he used when
piping is supported by or anchored to building
construction which may act as a sounding board
to amplify and transmit vibrations and noise In men husks
the piping.
mm
Wilt
Although vibration and noise resulting from
gas pulsations can occur in both the suction and mi mwl
discharge lines of reciprocating compressors,
it is much more Frequent and more intense in the

discharge tine. As a general rule, these gas


pulsations do not cause sufficient vibration and
noise to be of any consequence, Occasionally,
however, the frequency of the pulsations and tile (OFftl
FttMl
design or the piping are such that resonance is
established, with the result that the pulsation*
arc amplified and sympathetic vibration (as with MM
a tuning fork is set up in the piping,
I In tome M
instances, vibration can become so severe that
the piping is torn loose from its supports.
Flj. If- 1. Vibnttoneilmiftlior. (Cou rtuf Aruttndl
Fortunately, the condition can be remedied by
M*tii Hoh Di»liien, The Amtrioii Bra* QoaipMtiy.)
changing the speed of the compressor, by in-
stalling a discharge mulTler and/or by changing
,

the size of length of the discharge line- Since


changing the speed of the compressor is not general, refrigerant piping should be so designed
usually practical, (he latier two tnelhods and installed as to:
are better solutions, particularly when used
together. 1, Assure an adequate supply of refrigerant
When and noise are caused by gas
vibration to ail evaporators

turbulence resulting from high velocity, the 2, Assure positive and continuous return of
usual remedy is to reduce the gas velocity by oil to the compressor crankcase
increasing the sire of (he pipe, Sometimes Ihts 3, Avoid excessive refrigerant pressure losses

can be accomplished by installing n supple- which unnecessarily reduce the capacity and
mentary pipe. efficiency of the system

19-5. General Design Consideration*. Since 4, Prevent liquid refrigerant from entering

many of the operational problems encountered the compressor during either the running or off
in refrigeration applications can be traced cycles, or during compressor start-up

directly to improper design and/or installation !. Avoid the trapping of oil in the evaporator

of the refrigerant piping and accessories, the or suction line which may subsequendy return
importance of proper design and installation to the compressor in the form, of a large "slug"

procedures cannot be overemphastied. In with possible damage to the compressor.


REFRIGERANT PIPING AND ACCESSORIES 3S3

may be installed level with, or above, the crank-


ease tappings of the compressors. The crank-
case pressure equalizing line must not be
allowed lo drop below the level of the crankcase
tappings and it must nol contain any liquid
traps of any kind.
Both the oil equalizing line and the crankcase
pressure equalizing line should be the same size
as the crankcase tappings. Manual shut-off
valves should be installed in both lints between
the compressors so that individual compressors
can be valvcd oil for maintenance or repairs
without the necessity of shutting down the entire
system.
19-14. Liquid Indicators {Sight Glasses).
A liquid indicator or sight glass installed in the
liquid line of a refrigerating system provides a
means of determining visually whether or not
the system has a sufficient charge of refrigerant.
Jf the system is short of refrigerant, the vapor
hubbies appearing in the liquid stream will be
easily visible in the sight gjass. The sight glass
should be installed as close to the liquid receiver
as possible, but far enough downstream from
any valves w
that ihc resulting disturbance foes
nol appear in the sight glass. Wnen liquid hoes

are long, an additional sight glass is frequently


installed in front of the refrigerant control (or
liquid line solenoid, when one is used) to deter-
mine a solid stream of liquid is reaching the
if

refrigerant control. Bubbles appearing in the


sight glass at this point indicate that the liquid is
flashing in the liquid line as a result of excessive
pressure drop, in which case the bubbles can be
eliminated only by reducing the liquid line
pressure drop or by further subcooling of the

liquid refrigerant. Typical sight glasses are


shown in Fig. 19-15.
19-17. Refrigerant De hydra tors. Refriger-

ant driers (Fig. 19-26) are recommended for


all refrigerating systems employing a halo-
Fl£. 19,3$. Typial liquid indicators or light glmei.
carbon refrigerant. In small systems the drier
Notice moisture indicator IrKOrpprited In iln|le pert
is usually installed directly in the liquid line.
sight gla». Th« coler of tka molllur* mdlMtor
In larger systems the by-pass arrangement
danotf* the reinlve maistuft concent otthn syitom..
(a) Double pore iijht glass, (b) Slnglt port light glisi. Shown in Fig. 19-27 is employed. With the
(Courtesy Mueller Bnus Company,) latter method of installation, the drier cartridge

can be removed and reinstalled without inter-


rupting he operation of the system. Too, the
I

necessary also lo equali/e the crankcase drier can beusedintcrmittenily as needed. When

pressures. This is done by interconnecting the the drier is not being used valves A and B are
crankca&es above the crankco.se oil level with B open and valve C is closed. When the drier is
crankcase pressure equalizing line. This line In service valves B and C are open and valve A
iW PRINCIPLES Of REFRIGERATION

valves, may be used. In all cases, the strainer


shouM be imply tired so that the accumulation
of foreign material in the strainer will not cause
an excessive refrigerant pressure drop.
Most refrigerant compressor! come equipped
with a strainer in the suction inlet chamber-
When installing the refrigerant piping, cue
should be taken to arrange the suction piping at
the compressor to at lo permit servicing of this
strainer.
If -If. Pressure Relief Valve*. Pressure
relief valve* are safety valves designed to relieve
the pressure in the system to the atmosphere,
or in the oul-of-doors through a vent line, in the
event that the pressure in the system rises to an
unsafe level for any reason. Most refrigerating
systems have at lost one pressure relief valve
(or fusible plug) mounted on the receiver tank
or water-cooled condenser. In many instances,
additional relief valves are required at other
points in the system. The exact number, loca-
tion, and type of relief devices required ire set
forth in the American Standard Safety Code for
Mcchanical Refrigeration, and depends for the
most part on the type and size of the system.
Since local codes vary somewhat in this respect,
they should always be considered when designing
an installation. A typical pressure relief valve is

illustrated in Fig. 19-1S.


A fusible plug is sometimes substituted for
A fusi ble plug is limply
the pressure re! ief val ve.
a pipe plug which has been drilled and DIM
with a mctat alloy designed to melt at some pre-
determined fixed temperature (Fig. \%29}. The
design melting temperature of the fusible plug
depends on the pressure-temperature relation-
F4f. rM*. Sirurhl- through typ*. nonrtflllibhi ship of the refrigerant employed in the system.
drier. (Couth? Hutllir Bras Cwnpany.) lf-20. Receiver Tank Valve*. Receiver tank
valve* are usually of the packed type, equipped

isdosed Under no circu msiances should valves


.

A and C ever be closed at the same lime, except


when the drier cartridge is being changed With
valves B and C both closed, cold liquid could
be (rapped in the drier, which, upon warming,
could create tremendous hydra u Ik pressures, and
burst the drier casing.
If* 1, Strainers.
1 Strainers should be installed
immediately in front of automatic valves in
all

all refrigerant lines. When twoor more auto-


matic valves arc installed close together, a single
strainer, placed immediately upstream of the Fla. lf-21. Sidt outl« fli-nr mttilltc in bf-pux Una.
Flf. It>2t, Typltil prmura
filli(«h(t. (C6unaa>Mual!«T
Brut Compin^)

I. Valve body

3 Otsehoktor
4. tint*
5 Sprint itWw
6- Spnng
7. Oudt) conwMA
6 Laid h*I and locking wi
(pnnwrti ilmtion of factory

Pig. !?-». ftacalvar tank *»Jvai.(o) An(f* typ* with pranura r*4l«f outlst (nontnckaaatinf). (b) An(ta tvpa
with dip lutu (noftbackitadng). (Court**? Mutlhv Brw Company.)

Ml
1U PRINCIPLES OF REFRIGERATION
How between live refrigerant lines and ihe com-
pressor, and he "hack
i seat" controls Ihe gage
port of the valve, When the valve stem is" hack-
tea led," the gage port is closed and the refrig-
erant line it open to the compressor. When the
valve » "front-seated." the gage port is open lo
the compressor and the refrigerant line ti closed
to the compressor and gage port- With the
valve stem in nn intermedia le position between
the watt, both the refrigerant tine and the gage
port are open lo the compressor and, of course,
to each other,
19-22. Manual Valval. Manual valves used
for refrigeration duty may be of the globe,
angle, or gate type. Since the piping code pro-
hi hi is the use of gate valves in refrigerant lines,
except in large installations where an operating
attendant is on duty, they are used primarily
in wa ler and Gate vn Ives have a very
brine ines. I

low pressure drop, but do not permit throttling


and therefore can be employed only where full-
flow or no-flow conditions are desired. Both
globe and angle valves are suitable for thro tiling.
Since the angle valve ofTen the least resistance
to flow, its use is recommended whenever
practical,
Either the "packed" or "packless" type valve
is suitable for refrigeration duty, provided the
valve has been designed for thai purpose. Packed
valves should be of (he back -seating type in
order to permit packing under pressure and to

m
Fl|. It-Jt. Compr«wr ttrtif vtlvt, {4) Deck-
mud. (*) lni*rm«di»t« (e). f rontWittd
poilflort.

with a cap seal and designed for direct installa-


tion on the receiver tank. (Fig. 19-30.) When
designed for installation on top of ihe receiver
tank, the valves must be provided with dip
tubes so that Ihe liquid refrigerant can be drawn
from ihe bottom of the receiver. Some valve*
alio have tappings to accommodate a relief valve
or Itaaible plug,
19-21. Compressor S*rvle* Valves. Com-
pressor service valves are usually designed lo
bolt directly to the compressor bousing. As
shown in Fig. 19-31, they have both "front" and Mf. 19*32. Pick«j trp* ininuil nlvi. (Country
teats. The "front seat" controls the Vlltar Mtftubcturlng Company.)
REFRIGERANT PIPING AND ACCESSORIES 387

reduce the possibility of leakage through the The refrigerant velocity in the suction pipe
(c)
packing in the full-open position (Fig. 19-32.) at design capacity. Ans. 4200 fpm
Many packed valves are equipped with cap (d) The refrigerant velocity in the suction pipe

seals which completely cover and seal the at 50% of design capacity. Ans. 2100 fpm
valve stem, thereby eliminating the possibility
(e) The discharge pipe size. Ans. 2f in. OD

of leakage when the valve (/) The refrigerant velocity in the discharge
is not in use.
pipe at 50% of design capacity. Ans. 1 100 fpm
(g) Liquid line size. Ans. If in. OD
PROBLEMS (h) The over-all pressure drop in the liquid
line at design capacity. Ans. 4 psi
1. A Refrigerant- 12 system with a design capa- (i) The condenser to receiver pipe size.
city of 65 tons is operating with a suction
temperature of 40° F and a condensing tempera-
Ans. 2| in: OD
ture of 1 10° F. The suction line is 40 ft long and
2. Determine the size suction and discharge
contains 2 elbows and a 10 ft riser. The dis-
risers required to insure adequate oil return

charge line is 60 ft long and has 5 elbows and a


when the system in Problem 1 is operating at
IS ft riser. The condenser to receiver liquid line
25 % of design capacity.
is 50 ft long with 3 elbows. The liquid line is 50
Ans. (a) 2f in. OD (b) 2\ in. OD
ft long with 4 elbows and a 30 ft riser. Using 3. Rework Problem 1 using Refrigerant-22.
Type L copper tube for the refrigerant piping, Ans. (a) 2f in. OD (b) 1.23° F (c) 4000
determine: fpm (d) 2000 fpm
(c)2iin. (/) OD
(a) The suction pipe size. Ans. 3£ in. OD 2000 fpm ig) If in. OD
(A) 7 psi (/) 2f in, OD
(b) The over-all pressure drop (°F) in the 4. Rework Problem 2 using Refrigerant 22.
suction pipe at design capacity. Ans. 2° F Ans. (a) 2i in. OD
(b) 1 \ in. OD
As a general rule, the more frequently the
evaporator is defrosted the smaller is the frost
accumulation and the shorter is the defrost
period required.
20-2. Methods of Defrosting. Defrosting of
the evaporator accomplished in a number of
is

different ways, all of which can be classified as

20 either "natural defrosting" or "supplementary-


heat defrosting" according to
heat used to melt off the
the-
frost.
source of the
Natural
defrosting, sometimes called "shut-down" or

Defrost Methods, "off-cycle" defrosting, utilizes the heat of the


air in the refrigerated space to melt the frost
from the evaporator, whereas supplementary-
Low-Tern perat u re heat defrosting is accomplished with heat
supplied from sources other than the space air.
Systems, Some common sources of supplementary heat
are Water, brine, electric heating elements, and
and Multiple hot gas from the discharge of the compressor.
All methods of natural defrosting require

Temperature that the system (or evaporator) be shut down


for a period of time long enough to permit the
evaporator temperature to rise to a level well
Installations above the melting point of the frost. The exact
temperature rise required and the exact length
of time the evaporator must remain shut down
in order to complete the defrosting vary with the
individual installation and with the frequency
20-1. Defrosting Intervals. The necessity of defrosting. However, in every case, since
for periodically defrosting air-cooling evapora- the heat to melt the frost comes from the space
tors which operate at temperatures low enough air, the temperature in the space must be
to cause frost to collect on the evaporator allowed to rise to whatever level is necessary to
surface has already been established. How melt off the evaporator frost, which is usually
often the evaporator should be defrosted about 37° to 40° F. For this reason, natural
depends on the type of evaporator, the nature defrosting is not ordinarily practical in any
of the installation, and the method of defrosting. installation when the design space temperature
Large, bare-tube evaporators, such as those is below 34° F.

employed in breweries, cold storage plants, etc., The simplest method of defrosting is to shut
are usually defrosted only once or twice a the system down manually until the evaporator
month. On the other hand, finned blower coils warms up enough to melt off the frost, after
are frequently defrosted as often as once or which the system is started up again manually.
twice each hour. In some low temperature When several evaporators connected to the
installations defrosting of the evaporator is same condensing unit are located in different
continuous by brine spray or by some antifreeze spaces or fixtures, the evaporators can be taken
solution. out of service and defrosted one at a time by
In general, the length of the defrost period is manually closing a shut-off valve located in the
determined by the degree of frost accumulation liquid line of the evaporator being defrosted.
on the evaporator and by the rate at which heat When defrosting is completed, the evaporator is
can be applied to melt off the frost. For the put back into service by opening the shut-off
most part, the degree of frost accumulation will valve.
depend on the type of installation, the season If automatic defrosting is desired, a clock
of the year, and the frequency of defrosting. timer can be used to shut the system down for a
388
DEFROST METHODS AND LOW-TEMPERATURE SYSTEMS 389

fixed period of time at regular intervals. Both The most common method of natural de-
the number and the length of the defrost frosting is the "off-cycle" defrost. As described
periods can be adjusted to suit the individual in an earlier chapter, off-cycle defrosting is
installation. As a general rule, natural con- accomplished by adjusting the cycling control
vection evaporators are defrosted only once a so that the evaporator temperature rises to
day, in which case the defrost cycle is usually 37° F or 38° F during every off cycle. If the
started around midnight and lasts for several system has been properly designed, the evapo-
hours. On the other hand, unit coolers should rator will be maintained relatively free of frost
be defrosted at least once every 3 to 6 hr. Since it will be completely defrosted
at all times, since
it is usually undesirable to keep the system out during each off cycle.
of service for any longer than is necessary, the 20-3. Water Defrosting. For evaporator
length of the defrost period should be carefully temperatures down to approximately minus
adjusted so that the system is placed back in 40° F, defrosting can be accomplished by spray-
service as soon as possible after defrosting. ing water over the surface of the evaporator
In one variation of the time defrost, the coils. For evaporator temperatures below
defrost cycle is initiated by the defrost timer minus 40° F, brine or some antifreeze solution
and terminated by a temperature or pressure should be substitued for the water. A typical
control that is actuated by the evaporator water defrost system is illustrated in Fig. 20-1.
temperature or pressure. With this method, Although water defrosting can be made
the defrost period is automatically adjusted to automatic, it is often designed for manual
the required length, since the evaporator operation. Ordinarily, the following procedure
temperature (or pressure) will rise to the cut-in is used to carry out the defrosting:
setting of the control as soon as defrosting is 1 . A stop valve in the liquid line is closed and
completed. the refrigerant is evacuated from the evaporator,

Defrost position
Normal position Drain-off position
mmsfiaavsssssa
'/mmmmssmm

Defrost valve cut-away detail Defrost valve cut-away detail


normal position drain-off position
Defrost valve cut-away detail
defrost position

Fig. 20-1. Typical water defrost system. (Courtesy Dunham-Bush, Inc.)


390 PRINCIPLES OF REFRIGERATION

Fig. 20-2. Evaporator equipped for electric defrosting. Heater elements are installed through the center of
the tubes. Inset shows details of mechanical sealing of the heater elements. (Courtesy Dunham-Bush, Inc.)

after which the compressor is stopped and the returned to a reservoir and recirculated, rather
evaporator fans are turned off so that the water than wasted. Unless the reservoir is large
spray is not blown out into the refrigerated enough so that the addition of heat is not
space. If the evaporator is equippped with required, some means of reheating the solution
louvers, these are closed to isolate further the in the reservoir may be necessary. Since the
evaporator and prevent fogging of the refrig- water from the melting frost will weaken the
erated space. solution, the defrost system is equipped with a
2. The water sprays are turned on until the "concentrator" to boil off the excess water and
evaporator is defrosted, which requires approxi- return the solution to its initial concentration.
mately 4 to 5 minutes. After the sprays are One manufacturer circulates a heated glycol
turned off, several minutes are allowed for solution through the inner tube of a double-tube
draining of the water from the evaporator coils evaporator coil. The principal advantage
and drain pan before the evaporator fans are gained is that the glycol is not diluted by the
started and the system put back in operation. melting frost.
To eliminate the possibility of water freezing KM. Electric Defrosting. Electric resistance
in the drain line, the evaporator should be heaters are frequently employed for the de-
located close to an outside wall and the drain frosting of finned blower coils. An evaporator
line should be amply sized and so arranged equipped with defrost heaters is shown in Fig.
that the water is. drained from the space as 20-2. Ordinarily, the drain pan and drain line
rapidly as possible. A trap is installed in the are also heated electrically to prevent refreezing
drain line outside the refrigerated space to of the melted frost in these parts.
prevent warm being drawn into the space
air The electric defrost cycle can be started and
through the drain line during normal operation. stopped manually or a defrost timer may be
In some instances, a float valve is employed in used to make defrosting completely automatic.
the drain pan to shut off the water spray and In either case, the defrosting procedure is the
prevent overflowing into the space in the event same. The defrost cycle is initiated by closing
that the drain line becomes plugged with ice. a solenoid valve in the liquid line causing the
When brine or an antifreeze solution replaces evaporator to be evacuated, after which the
the water spray, the defrosting solution is compressor cycles off on low pressure control.
DEFROST METHODS AND LOW-TEMPERATURE SYSTEMS 391

At the same time, the heating elements in the


evaporator are energized and the evaporator
fans turned off so that the heat is not blown ( y
out into the refrigerated space. After the )
evaporator is defrosted, the heaters are de-
energized and the system put back in operation
( /
by opening the liquid line solenoid and starting
L- )
the evaporator fans. Hot gas
20-5. Hot Gas Defrosting. Hot gas defrost- /solenoid

ing has many variations, all of which in some ^ By-pass line


way hot gas discharged from the
utilize the
compressor as a source of heat to defrost the
evaporator. One of the simplest methods of
hot gas defrosting is illustrated in Fig. 20-3. A
->
by-pass equipped with a solenoid valve is C r -i

installed between the compressor discharge and


the evaporator. When the solenoid valve is
tt (
-J !

opened, thehot gas from the compressor


discharge by-passes the condenser and enters Fig. 20-3. .Simple hot gas defrost system.

the evaporator at a point just beyond the


refrigerant control. Defrosting is accomplished
as the hot gas gives up its heat to the cold 20-6. Re-evaporator Coils. One common
evaporator and condenses into the liquid state. method of hot gas defrosting employs a re-
Some of the condensed refrigerant stays in the evaporator coil in the suction line to re-evaporate
evaporator while the remainder returns to the the liquid, as shown in Fig. 20-4. During the
compressor where it is evaporated by the com- normal running cycle the solenoid valve in the
pressor heat and recirculated to the evaporator. suction line is open and the suction vapor from
This method of hot gas defrosting has several the evaporator by-passes the re-evaporator coil
disadvantages. Since no liquid is vaporized in in order to avoid an excessive suction line
the evaporator during the defrost cycle, the pressure loss. At regular intervals (usually 3 to
amount of hot gas available from the com- 6 hr) the defrost timer starts the defrost cycle by
pressor will be limited. As defrosting progresses, opening the solenoid in the hot gas line and
more liquid remains in the evaporator and less closing the solenoid in the suction by-pass line.
refrigerant is returned to the compressor for At the same time, the evaporator fans are
recirculation, with the result that the system stopped and the re-evaporator fan is started.
tends to run out of heat before the evaporator The liquid condensed in the evaporator is
is completely defrosted. re-evaporated in the re-evaporator coil and
Another, and more serious, disadvantage of returned as a vapor to the compressor, where it is
this method is the possibility that a large slug compressed and recirculated to the evaporator.
of liquid refrigerant will return to the com- When defrosting is completed, the defrost cycle
pressor and cause damage to that unit. This is may be terminated by the defrost timer or by an
most likely to occur either at the beginning of evaporator-temperature actuated temperature
the defrost cycle or immediately after defrosting control. In either case, the system is placed
is completed. back in operation by closing the hot gas
Fortunately, both these weaknesses can be solenoid, opening the suction solenoid, stopping
overcome by providing some means of re- the re-evaporator fan and starting the evapora-
evaporating the liquid which condenses in the tor fans.
evaporator before it is returned to the com- 20-7.Defrosting Multiple Evaporator Sys-
pressor. The particular means used to re- tems. When two or more evaporators are
evaporate the liquid is the principal factor connected to a common condensing unit, the
distinguishing one method of hot gas defrosting evaporators may be defrosted individually, in
from another. which case the operating evaporator can serve as
392 PRINCIPLES OF REFRIGERATION

Suction line

Thermostatic
® exp. valve

Liquid
solenoid

Suction line from Liquid line to


other evaporators other evaporators
(if required) (if required)

Fig. 20-4. Hot gas defrost system employing re-evaporator coll. (Courtesy Kramer-Trenton Company.)

a re-evaporator for the refrigerant condensed in cycle. An automatic expansion valve is used to
the evaporator being defrosted. A flow diagram meter the liquid refrigerant into the condenser
of this arrangement is illustrated in Fig. 20-5. for re-evaporation. Flow diagrams for normal
20-8. Reverse Cycle Defrosting. By em- operation and defrosting are shown in Figs.
ploying the reverse cycle (heat pump) principle, 20-6(a) and 20-6(fc), respectively, Modern
the condenser can be utilized as a re-evaporator practice replaces valves A, B, C, and D of Fig.
coil to re-evaporate the refrigerant that con- 20-6 with a single four-way valve as illustrated
denses in the evaporator during the defrost in Fig. 20-7.

Normal Operation
Open: B-2-3-6-7
Close: A-l-4-5-8

Defrost "X"
Open: A-l-3-5-7
Close: B-2-4-6-8

Defrost "Y"
Open: A-2-4-6-8
Close: B-l-3-5-7

Crankcase
pressure'
regulator

Fig. 20-5. Hot gas defrost —multiple evaporator system.


DEFROST METHODS AND LOW-TEMPERATURE SYSTEMS 393

20-9.Heat Bank Defrosting. The Thermo- order to avoid unnecessary suction line pressure
bank* method of hot gas defrosting employs loss and superheating of the suction vapor by
a water bank to store a portion of the heat the bank water. Also, to control the maximum
ordinarily discarded at the condenser when the water bank temperature, a by-pass is built into
evaporator is being refrigerated. During the the water bank heating coil. The by-pass is so
defrost cycle, the heat stored in the water bank sized that a greater portion of the discharge gas

Fig. 20-6. (a) Reverse cycle


hot gas defrost system (defrost
cycle), (b) Reverse cycle hot
gas defrost system (normal
operation).

is used to re-evaporate the refrigerant con- by-passes the heater coil and flows directly to
densed in the defrosting evaporator. the condenser as the temperature of the bank
During normal operation (Fig. 20-8a), the water increases.
discharge gas from the compressor passes When the frost reaches a predetermined
through the heating coil in the water bank first thickness, the defrost cycle (Fig. 20-86) is
and then goes to the condenser, so that a portion initiated by an electric timer which opens the
of the heat ordinarily discarded at the con- hot gas solenoid valve, closes the suction
denser is stored in the bank water. Notice solenoid valve, and stops the evaporator fans.
that the suction vapor by-passes the holdback Hot gas is discharged into the evaporator where
valve and bank during the refrigerating cycle in itcondenses and defrosts the coil. The con-
• A proprietary design of the Kramer-Trenton densed refrigerant flows to the holdback valve
Company. which acts as a constant pressure expansion
"

394 PRINCIPLES OF REFRIGERATION

valve and feeds liquid to the re-evaporator coil liquid refrigerant in the coil and suction line is

immersed in the bank water. In this process, re-evaporated. The timer then returns the
the bank water actually freezes on the outside of system to normal operation. When normal
the re-evaporator coil. The heat stored in the operation is resumed, the bank water is promptly
bank is transferred to the refrigerant which restored to its original temperature by the hot
evaporates completely in the re-evaporator coil. gas passing through the heating coil.

Expansion Evaporator
valve

Compressor
Fig. 20-7. (a) Reverse cycle

hot gas defrost normal opera-
tion, (b) Reverse cycle hot gas

Receiver
defrost —defrost cycle.

Expansion-,
Evaporator
valve

Check valve

Four way
valve

Expansion
valve," Condenser

Check valve

Iver '
Receiver

Thus both sensible and latent heat are abstracted 20-10. Vapot Defrosting. A schematic dia-
from the bank water, making available vast gram of the Vapomatic defrosting system is
heat quantities for fast defrost and the refrig- shown in Fig. 20-9, along with an enlarged view
erant returns to the suction inlet of the of the Vapot,* which is the heart of this hot gas

compressor completely evaporated. defrost system. The Vapot, which is actually a

Defrost is completed in approximately 6 to 8 specially designed suction line accumulator,


min. This is followed by a post-defrost period traps the liquid refrigerant condensed in the
lasting a few minutes after the closing of the hot * Proprietary designs of Refrigeration Engineer-

gas solenoid valve. During post-defrost any ing, Inc.


DEFROST METHODS AND LOW-TEMPERATURE SYSTEMS 395

Re-evaporator coil' Re-evaporator coil

Thermobank Thermobank

Receiver- Compressor Receiver' "•Compressor

Normal operation Defrost

(a) (b)

Fig. 20-8. Thermobank hot gas defrosting. (Courtesy Kramer-Trenton Company.)

evaporator and, by means of a carefully sized The defrost cycle is initiated by a defrost
bleed tube, continuously feeds a measured timer which opens the hot gas solenoid valve
amount of the liquid back to the compressor and stops the evaporator fans. An evaporator
with the suction vapor. The small amount of temperature control terminates the defrost
liquid feeding compressor is
back to the and normal operation.
restores the system to
vaporized by the heat of compression and 20-11. Multistage (Booster) Compression.
returned to the evaporator. In this way, the It has already been established that the capacity
Vapot provides a continuous source of latent and efficiency of any refrigerating system

heat for defrosting the evaporator and at the diminish rapidly as the difference between the
same time eliminates the possibility of large suction and condensing temperatures is increased
slugs of liquid returning to the compressor. by a reduction in the evaporator temperature.
The heat exchanger in the Vapot has no The losses experienced are due partially to the
significance in the defrost cycle. rarification of the suction vapors at the lower

Suction Suction to
Defrost limit from coil compressor
tat—~*p
thermostat Suction line

Pitch defrost line


continuously H in.

per foot minimum


Liquid line
between solenoid
connections'
and compressor

Attach defrost line to


drain-pitch 3$ in. or more -
per foot while in cold area

Trap drain outside


cold area
fl
mQ
Fig. 20-9. (Left) Typical application of Vapot. (Right) Enlarged view of Vapot. (Courtesy Recold
Corporation.)
396 PRINCIPLES OF REFRIGERATION

f . <
[C
1
I

^- )

Fig. 20-10. Three-stage, direct


staged compression system.

evaporator temperatures and partially to the refrigerants having progressively lower boiling
increase in the compression ratio. Since any The compressed refrigerant
points (Fig. 20-1 1).
increase in the ratio of compression is accom- vapor from the lower stage is condensed in
panied by a rise in the discharge temperature, a heat exchanger, usually called a cascade
discharge temperatures also tend to become condenser, which is also the evaporator of the
excessive as the evaporator temperature is next higher stage refrigerant.
reduced. Both methods of multistaging have relative
Whereas conventional single-stage systems advantages and disadvantages. The particular
will usually give satisfactory results with method which will produce the best results in a
evaporator temperatures down to -40° F, given installation depends for the most part on
provided that condensing temperatures are the size of the installation and on the degree of
reasonably low, for evaporator temperatures low temperature which must be attained. In
below minus 40° F, some form of multistage some instances, a combination of the cascade
compression must be employed in order to and direct staging methods can be used to an
avoid excessive discharge temperatures and to advantage. In these cases, the compound
maintain reasonable operating efficiencies. In compression (direct staging) is usually applied
larger installations, multistage operation should to the lower stage of the cascade.
be considered for any evaporator temperature 20-12. Intercoolers. With direct staging,
below 0° F. cooling of the refrigerant vapor between the
All methods of accomplishing multistage several stages of compression (desuperheating)
compression can be grouped into two basic is necessary in order to avoid overheating of
types: direct staging and (2) cascade
(1) the higher stage compressors. Since the refrig-
staging.The direct staging method employs erant gas is superheated during the compres-
two or more compressors connected in series sion process, if the gas is not cooled before it
to compress a single refrigerant in successive enters the next stage compressor, excessive dis-
stages. A flow diagram of a simple, three-stage, charge temperatures will result with subsequent
direct staged multicompression system is shown overheating of the higher stage machines.
in Fig. 20-10. Notice that the pressure of the Because of the large temperature differential
refrigerant vapor is raised from the evaporator between the condenser and the evaporator,
pressure to the condenser pressure in three cooling of the liquid refrigerant is also desirable
increments, the discharge vapor from the lower in order to avoid heavy losses in refrigerating
stage compressors being piped to the suction of effect because of excessive flashing of the liquid
the next higher stage compressor. and the accompanying
in the refrigerant control,
Cascade staging involves the use of two or increase in thevolume of vapor which must be
more separate refrigerant circuits which employ handled by the low stage compressor.
DEFROST METHODS AND LOW-TEMPERATURE SYSTEMS 397

Three common methods of gas desuper- pressure of the liquid going to the evaporator is
heating and liquid cooling for direct staged reduced to the intermediate pressure in the
systems are illustrated in Fig. 20-12. The intercooler. This reduces the pressure drop
intercooler shown in Fig. 20- 12a an "open"
is available at the expansion valve
and necessitates
or "flash" type intercooler. The liquid from oversizing of the valve, which often results in
the condenser is expanded into the intercooler sluggish operation. Too, since the low-
where its temperature is reduced by flashing to temperature, low-pressure liquid leaving the
the saturation temperature corresponding^to intercooler is saturated, there is a tendency for
the intercooler pressure. Since suction from the the liquid to flash in the liquid line between the
intercooler is taken into the high stage com- intercooler and the evaporator. For this
pressor, the temperature of the liquid leaving reason, the liquid line should be designed for the
the intercooler to go to the low temperature minimum possible pressure drop.
evaporator is the saturation temperature corre- A shell-and-coil intercooler, sometimes called
sponding to the intermediate pressure (pressure a "closed type" intercooler, is illustrated in
between stages). Since the refrigeration expended Fig. 20-126. This type of intercooler differs
in cooling the liquid to the intermediate temper- from the flash type in that only a portion of the
ature is accomplished much more economically liquid from the condenser is expanded into the
at the level of the high stage suction than at that intercooler, whereas the balance, that ^rtion
of the low stage suction, cooling of the liquid going to the evaporator, passes through the coil
in the intercooler has the effect of reducing the submerged in the intercooler liquid. Therefore,
horsepower per ton as well as the displacement with the shell-and-coil type intercooler, the
required for the low stage compressor. pressure of the liquid is not reduced to the
The discharge gas from the low stage com- intermediate pressure, that is, the -liquid cooling
pressor desuperheated by causing it to bubble
is occurs in the form of subcooling rather than as
up through the liquid in the intercooler, after a reduction in the saturation temperature. The
which the discharge gas passes to the suction of advantages gained by this method, of course,
the high stage compressor along with the flash are the higher liquid pressures made available at
gas from the intercooler. the expansion valve and the elimination of flash
The principal advantages of the flash type gas in the liquid line. With good intercooler
intercooler are in its simplicity and low cost, design the liquid can be cooled to within 10 to
and the fact that the temperature of the 20 degrees of the saturation temperature
liquid refrigerant is reduced to the saturation corresponding to the intermediate pressure.
temperature corresponding to the intermediate Vapor velocities in both types of flooded
pressure. The chief disadvantage is that the intercoolers should be limited to a maximum of

Hot gas

Evaporator
H Low stage
condenser
(refrig. cooled)

Low stage
(booster)
comp.
High stage
oomp.

Fig. 20-11. Cascade system (two-stage). (Courtesy Carrier Corporation.)


398 PRINCIPLES OF REFRIGERATION

Open type
intercooler

(a)

(b)

Fig. 20-12. Various types of gas and liquid intercoolers. (o) Di re« staged system —open, flash-type intercooler.

(b) Direct staged system—closed, shell-and-coil type intercooler. (c) Direct staged system —dry-expansion
intercooler. (Courtesy Carrier Corporation.)
DEFROST METHODS AND LOW-TEMPERATURE SYSTEMS 399

200 fpm and ample separation area should be To evaporator Liquid from receiver

allowed above the liquid level in the inter-


coolers in order to prevent liquid carryover into
Hand expansion valve
the high stage compressor.
Another arrangement, employing a dry-
expansion intercooler, is shown in Fig. 20- 12c.
This type of intercooler is not suitable for
ammonia systems, but is widely used with
Refrigerants-12 and 22. The liquid from the \i
From tow stage compressor ^- _--€- T° high stage compressor^

condenser is subcooled as it passes through the


Fig. 20-13. Liquid injection gas intercooling.
coil in the intercooler. Desuperheating is
accomplished by overfeeding of the intercooler
so that a small amount of liquid is carried over be attained by the direct staging method. The
into the desuperheating area where it is vapor- practical low limit is approximately — 125° F
ized by the hot gas from the discharge of the with either Refrigerants-12 or 22 and -90° F
low stage compressor. The gas is cooled by with ammonia. Below these temperatures
vaporizing the liquid and by mixing with the cascade staging is ordinarily required, with
cold vapor from the intercooler. some high-pressure, low boiling point refriger-
Since ammonia has a very high latent heat ant, such as ethane, ethylene, R-13, R-13B1, or
value, liquid cooling is not as important in R-14, being used in the lower stage. Because
ammonia systems as in systems employing of their extremely high pressures at normal
fluorocarbon refrigerants. For this reason, condensing temperatures and/or their relatively
liquid coolingsometimes neglected in
is low critical temperatures, these high pressure
ammonia systems, in which case the discharge refrigerants must be condensed at rather low
vapor from the low stage compressor is usually temperatures and therefore are cascaded with
desuperheated by injecting a small amount of R-12, R-22, or propane. A three-stage cascade
liquid ammonia directly into the line connecting system employing ethylene, methane, and pro-
the low and high stage compressors (Fig. 20-13). pane in the low, intermediate, and high stages
The vaporization of the liquid ammonia in this respectively is illustrated in Fig. 20-14.
line provides the necessary gas cooling. The chief disadvantage of cascade staging is

In some ammonia systems, the discharge gas the overlap of refrigerant temperatures in the
is cooled in a water-cooled intercooler that is cascade condenser, which tends to reduce the
similar in design to the shell-and-tube or shell- thermal efficiency of the system somewhat below
and-coil water-cooled condensers. The effec- that of the direct staged system. On the other
tiveness of this type of intercooler depends on hand, cascade staging makes possible the use of
the temperature of the available water and high density, high pressure refrigerants in the
increases as the available water temperature lower stages, which will usually result in a
decreases. As a general rule, water-cooled considerable reduction in the displacement
intercoolers will not produce enough gas cooling required for the low stage compressor. The use
to lower the power requirements, but will of high pressure refrigerants also simplifies the
usually provide sufficient cooling to keep the design of the low stage evaporator in that higher
discharge temperature within the maximum refrigerant pressure losses through the evapora-
limit and thereby prevent overheating of the tor can be permitted without incurring excessive
high stage compressor. losses in system capacity and efficiency. Too,
20-13. Direct Staging vs. Cascade Staging. since the refrigerants in the several stages do not
Direct staging requires the use of refrigerants intermingle, and each stage is a separate
that have boiling points low enough to provide system within itself, the problem of oil return
the low temperatures desired in the evaporator to the compressors is somewhat less critical than
and which, at the same time, are condensable in the direct staged system.
under reasonable pressures with air or water at A single stage Refrigerant- 12 system and a
normal temperatures. This requirement tends two-stage, direct staged Refrigerant- 12 system
to limit the degree of low temperature that can operating between the same temperature limits
400 PRINCIPLES OF REFRIGERATION
DEFROST METHODS AND LOW-TEMPERATURE SYSTEMS 401

are compared on pressure-enthalpy coordinates method of oil equalization requires that the
in Fig. 20-15. If the volumetric and compression crankcase pressures in the several compressors
efficiencies of the compressors were considered, be exactly the same (Section 19*15) and therefore
the difference between the single and multistage is not practical when the compressors are
compression systems would be approximately connected in series, since the higher pressures
twice as great as that indicated by the values existing in the crankcase of the high stage
because of the difference in the compression compressors would force the oil through the
ratios. equalizing lines into the lower pressures existing
20-14. Oil Return in Multistage Systems. in the crankcases of the low stage compressors.
Since the several stages of the cascade system One common method of equalizing the oil
are actually separate and independent systems, levels in the crankcases of compressors con-
oil return is accomplished in the individual nected in series is shown in Fig. 20-12. An oil
stages in the same manner as in any other single separator installed in the discharge line of the
stage system operating under the same condi- high stage compressor separates the oil from
tions. This is not true of the direct staged the discharge gas and returns it to the suction
system. Whenever two or more compressors inletof that machine. High side float valves
are interconnected, either in parallel or in series, maintain the desired oil level in the high stage
there is no assurance that oil return to the compressors by continuously draining the
individual compressors will be in equal amounts. excess oil returning to these compressors to the
Therefore, some means of insuring equal next lower stage compressor through the oil
distribution of the oil among the several transfer lines. For manual operation, hand
compressors must be provided. When the stop valves (normally closed) can be substituted
compressors are connected in parallel, the oil for the float drainers, in which case the hand
can be maintained at the same level in all the valves are opened periodically to adjust the oil
compressors by interconnecting the crankcases levels by bleeding oil from the higher stage
as shown in Fig. 19-11. However, this simple compressors to the lower stage compressors.

Desuperheattng of
disclutje vapor from
first-stage compressor

r
M0*F Condenser pressure
131.6

Line of single

stage compression

Interstage pressure
30.56
(0.43 lb of R-12 evaporated in

intercooier per pound of R-12


circulated)

First-

compression

Evaporator pressure
7.125

Enthalpy (Btu/lb)

Fig. 20-15. Mi diagram of two-stage direct-staged R-12 system with flash intercooier. First-stage compressor
displacement —16.19 cfm/ton. Second-stage compressor displacement 6.24 cfm/ton including vapor from —
intercooier. Compression ratio for each stage is approximately 4.3 to I.
402 PRINCIPLES OF REFRIGERATION

Electric power

Evaporator

Check j_
valve |

Low pressure Condenser


motor control

Fig. 20-16. Three-evaporator multiple temperature installation. Manual stop valves in suction and liquid
lines permit isolation of the individual evaporators for maintenance.

20-15. Multiple Temperature System. A and therefore the saturation temperature of the
multiple temperature system is one wherein refrigerant, in these units at the desired high
two or more evaporators operating at different level. A check valve is installed in the suction

temperatures and located in different spaces or line of the lowest temperature evaporator to
fixtures (or sometimes in the same space or prevent the higher pressure from the warmer
fixture) are connected to the same compressor or evaporators from backing up into the cold
condensing unit. The chief advantages gained evaporator when the former are calling for
by this type of operation are a savings in space refrigeration.Since the check valve will remain
and a reduction in the initial cost of the equip- closed as long as the pressure in the suction
ment. However, since higher operating costs main is above the pressure in the low tempera-
will usually more than offset the initial cost ture evaporator, it is evident that the low
advantage, a multiple temperature system is temperature evaporator will receive little, if any,

economically justifiable only in small capacity refrigeration until the refrigeration demands of
installations where operating- costs in any case the high temperature evaporators are satisfied.
are relatively small. One obvious disadvantage For this reason, if a multiple temperature
of the multiple temperature system is that in system is to perform satisfactorily, the load on
the event of compressor breakdown all spaces the lowest temperature evaporator must account
served by the compressor will be without for at least 60%, and preferably more, of the
refrigeration, thereby causing the possible loss total system load. When the high temperature
of product which otherwise would not occur. evaporator(s) constitute more than 40% of the
A
typical three-evaporator multiple tempera- total load, the refrigeration demands of the
ture system is illustrated in Fig. 20-16. An high temperature evaporator(s) will cause the
evaporator pressure regulator valve is installed compressor to operate a greater portion of the
in the suction line of each of the warmer time at suction pressures too high to permit
evaporators in order to maintain the pressure, adequate refrigeration of the low temperature
DEFROST METHODS AND LOW-TEMPERATURE SYSTEMS 403

evaporator, with the result that temperature temperature evaporators to the suction main,
control in that unit will be erratic. the suction pressure will immediately rise above
The evaporator pressure regulators installed the cut-in setting of the low pressure control
in the suction line of the warmer evaporators and the compressor will cycle on.
may be either the throttling or snap-action The fact that the pressure in the high tem-
types, the latter being employed whenever perature evaporators is always above the cut-in

"off-cycle" defrosting of the high temperature setting of the low pressure control poses
evaporator is desired (Section 17-23). somewhat of a problem when throttling-type
In multiple temperature systems, a low evaporator pressure regulators are employed on
pressure motor control is ordinarily used to the high temperature evaporators. Since this
cycle the compressor on and off, the cut-in type of regulator is open any time the evapora-
and cut-out pressures of the control being tor pressure is above the pressure setting of the
adjusted to suit the conditions required in the regulator, it will often cause the compressor to
low temperature evaporator. cycle on when the evaporator it is controlling
When the compressor is operating, the does not actually require refrigeration. As
pressure at the suction inlet of the compressor soon as the compressor starts, the pressure in
will depend on the rate at which vapor is being the evaporator is immediately reduced below
generated in all the evaporators. If the com- the regulator setting and the regulator closes,
bined load on the several evaporators is high, causing the compressor to cycle off again on the
the suction pressure will also be high. When low pressure control.
the refrigeration demands of one of the higher To prevent short cycling of the compressor
temperature evaporators is satisfied, the eva- on the low pressure control when throttling

porator pressure regulator will close (or throttle) type evaporator pressure regulators are em-
so that little or no vapor enters the suction main ployed, it is usually necessary to install a
from that unit, thereby causing a reduction in solenoid stop valve in the suctions line of the
the suction pressure. As previously mentioned, high temperature evaporators in order to obtain
whether or not the low temperature evaporator positive shut-down of these evaporators during
is being refrigerated at any given time depends the compressor off-cycle. With pilot operated

on whether the suction pressure is below or evaporator pressure regulators, positive close-
above the pressure in that evaporator. However, off of the regulator during the compressor off
the low temperature evaporator will always be cycle can be obtained by controlling the regu-
open to the compressor (refrigerated) at any lator with a temperature or solenoid pilot.
time that the demands of the high temperature In small systems, a surge tank installed in the
evaporators are satisfied and the regulator suction main will usually prevent short cycling
valves are closed (or throttled). When the of the compressor. Because of the relatively
demands of the low temperature evaporator are large volume of the surge tank, it is capable of

also satisfied, the suction pressure will drop absorbing reasonable pressure increases occur-
below the cut-out setting of the low pressure ring in the high temperature evaporator and
control and the compressor will cycle off. thereby preventing the suction pressure from
With the compressor on the off cycle, any one rising prematurely to the cut-in pressure of the
of the evaporators is capable of cycling the low pressure control. The size of the surge tank
compressor on again. For the low temperature required depends on the ratio of the high tem-
evaporator, a gradual rise in pressure in that perature load to the low temperature load and
unit to the cut-in pressure of the low pressure on the temperature differential between the
control will start the compressor, whether or not high and low temperature, the size of the tank
either of the high temperature evaporators also required increasing as each of these factors
requires refrigeration. increases. Obviously, this method of preventing
Since the pressure maintained in the high short cycling is best applied to systems where
temperature evaporators, even at the lower limit, the high temperature load is only a small
is always above the cut-in setting of the low portion of the total load and/or where the
pressure control, if either of the evaporator difference in temperature between the high and
pressure regulators opens one of the high low temperature evaporators is relatively small.
404 PRINCIPLES OF REFRIGERATION

Fig. 20-17. Multiple unit installation employing thermostat-solenoid


control. Manual stop valves in suction
lines permit Isolation of individual evaporators for maintenance.

Low stage
'evaporator Low side
float

— Low
float
side JJuction from high
stage evaporator

High stage
evaporator

Suction from low stage Subcooled liquid to


6
evaporator low stage evaporator

Discharge from low Discharge from high


/* stage compressor
stage compressor Fig. 20-18. A multiple
temperature system employing
two-staged direct staged com-
pression.
Suction gas from
high stage
evaporator mixes
with and
desuperheats
discharge gas
from low stage
compressor
DEFROST METHODS AND LOW-TEMPERATURE SYSTEMS 405

20-16.Solenoid Controlled Multiple Tem-


perature Systems. Thermostatically con-
trolled solenoid stop valves, installed in either
the liquid or the suction lines of the high tem-
perature evaporators, are frequently used to
obtain multiple temperature operation. A
typical installation employing solenoid valves
in the liquid line is shown in Fig. 20-17. The
operation of this type of system is similar to
that of systems employing snap-action regula-
tors in the suction line, except that there is no
control of the evaporator pressure and tem-
perature. A
space-temperature actuated thermo-
stat controls the solenoid valves. When the
space temperature the thermostat contacts
rises,
close, energizing the solenoid coil and opening
the liquid line to the evaporator. The pressure
in the evaporator rises as the liquid enters the
evaporator, causing the low pressure control to
cycle the compressor on, if the latter is not al-
ready running. If the compressor is already run-
ning, the entrance of liquid into the evaporator
will cause a rise in the operating suction pressure.
When the temperature in the space is re-
duced to the desired low level, the thermostat
contacts open, de-energizing the solenoid coil
and closing the liquid line, whereupon the
evaporator pumps down to the operating
suction pressure, or to the cut-out pressure in
the event that none of the other evaporators is
calling for refrigeration.Since all the evapo-
rators pump-down as they are cycled out, the
system receiver tank must be large enough to
hold the entire system refrigerant charge. This
isnot true, however, if the solenoids are installed
in the suction line rather than in the liquid line.
Placing the solenoid valves in the suction
line has the disadvantage of requiring larger,
more expensive valves. Too, in the event of a
leaky refrigerant control, there is always the
danger that liquid will accumulate in the
evaporator while the suction solenoid is closed
and flood back to the compressor when the
solenoid is opened.
When solenoids are employed to obtain
Fig. 20-19. Three-stage, multiple temperature sys- multiple temperature operation, there is no
tem employing Refrigerant- 21 with centrifugal direct control of the evaporator pressure and
compressors. (Courtesy York Corporation.) temperature, since the pressure in the evaporator
406 PRINCIPLES OF REFRIGERATION

at any given time depend upon the number


will With either suction line or liquid line sole-

of evaporators open to the compressor. noids, check valves should be installed in the
Obviously, with this type of operation it is very suction lines of the lower temperature eva-
difficult to maintain a balanced relationship porators to avoid excessive pressures and tem-
between the space and evaporator tempera- peratures in these units when the warmer
tures, and therefore humidity control in the evaporators are open to the suction line.

refrigerated space becomes very indefinite. 20-17. Multiple Temperature Operation


For this reason, when humidity control is in Staged Systems. A multiple temperature
important, it is usually necessary to install a system employing direct staged compression is
throttling-typeevaporator pressure regulator illustrated in Fig. 20-18. Notice that the high

in the suction lines of the high temperature temperature evaporator serves also as the gas
evaporators in order to maintain the pressure and liquid cooler for the lower stage. The high
and temperature in these evaporators at the stage compressor must be selected to handle the

necessary high level. However, evaporator high temperature load in addition to the load
pressure regulators should be used only with passed along by the low stage compressor.
suction line solenoids. Short cycling of the com- A three-stage, multiple temperature, cascade-

pressor may result if they are employed in con- staged system employing Refrigerant- 12 in all
junction with liquid line solenoids. three stages is shown in Fig. 20-19.
transformers are delta (A) connected as shown
in Fig. 21-1 A, in which case the supply voltages
are 115 V
and 230 V single-phase and 230 V
three-phase. With the open delta arrangement
shown in Fig. 21-lc, three-phase power for
isolated users can be supplied with only two
transformers.

21 Power is

establishments at 460
frequently delivered to commercial
V and is available to large
industrial users at much higher voltages by
special arrangement with the power company.
ElectricMotors Naturally, all motors must be selected to
conform to the characteristics of the available
and Control power supply. Power companies guarantee to
maintain voltages within plus or minus 10% of

Circuits the design voltage. Most motors will operate


satisfactorily within these voltage limits. Many
motors are designed so that they can be operated
on either low or high voltage by reconnecting
external motor leads.
In addition to the type of power supply
21-1. Electric Motors. Single-phase and available, some of the other more important
three-phase alternating current motors of factors that must be taken into account in
various types are employed in the refrigerating order to select the proper type motor are:
industry as drives for compressors, pumps, and
1. The conditions prevailing at the point of
fans. Afew two-phase and direct current
installation with respect to the ambient tem-
motors are also used on occasion.
perature and to the presence of dust, moisture,
Single-phase motors range in size from
or explosive materials.
approximately ^
hp up through 10 hp,
2. Starting torque requirements (loaded or
whereas three-phase motors are available in
unloaded starting).
sizes ranging from approximately £ hp on up,
3. Starting current limitations.
although the latter are seldom employed in
4. Single or multispeed operation.
sizes below 1 hp. When three-phase power is
5. Continuous or intermittent operation.
available, the three-phase motor is usually
6. Efficiency and power factor (not important
preferred to the single-phase type in integral
for small motors).
horsepower sizes because of its greater simpli-
cityand lower cost. All motors generate a certain amount of heat
Practically all power in the United States is due to power losses in the windings. If this
generated as 60-cycle, three-phase alternating heat is not dissipated to the surroundings, motor
current and is supplied at the point of use as temperature will become excessive and break-
single-phase and/or three-phase alternating down of the winding insulation will result.
current. Voltages available at the point of use Open type (ventilated) motors aredesigned to
depend somewhat on the type of transformer operate at temperatures approximately 40° C
connection. In areas where power consumption (72° F) above the ambient temperature under
is predominantly single-phase low voltage, fullload, whereas totally enclosed motors are
transformers are "1"' connected so that the designed for a 55° C
temperature rise at full
low voltage load can be distributed evenly load. Both types are guaranteed by the manu-
among the three transformers. As shown in facturer to operate continuously under full
Fig. 21-la, voltages supplied from a "Y" load conditions without overheating when the
connected transformer bank are 120 V and 208 ambient temperature does not exceed 40° C.
V single-phase and 208 V three-phase. When When higher ambient temperatures are encoun-
the power load is predominantly three-phase, the tered, it is sometimes necessary to employ a
407
408 PRINCIPLES OF REFRIGERATION
Motors may be classified according to the
type of enclosure as: (1) open, (2) totally
enclosed, (3) splash-proof, and (4) explosion-
proof. Open-type motors are designed so that
air is circulated directly over the windings to
carry away the motor heat. This type of motor
can be used in any application where the air
is relatively free of dust and moisture, the motor

is not subject to wetting, and the hazards of

fire or explosion do not exist. Splash-proof


motors are designed for installation out-of-
doors or in any other location where the motor
may be subject to wetting. Totally enclosed
motors are designed for use where dust and
moisture conditions are severe. These motors
are unventilated and must dissipate their heat
to the surrounding air through the motor
housing. Explosion-proof motors are designed
for installation in hazardous locations, as when
explosive dusts or gases are contained in the
air.

For the most part, motor starting torque


requirements depend on the load characteristics
of the driven machine. Low starting torque
motors may be used with any machine that
starts unloaded. On the other hand, high
starting torque motors must be used when the
driven machine starts under load. Since
motor starting (locked rotor) currents often
exceed five to six times the full load current
of the motor, where large motors are employed,
230 V a low starting current characteristic is desirable
in order to reduce the starting load on the
wiring, transformers, and generating equip-
ment.
Alternating current motors may be classified
according to their principle of operation as
either induction motors or synchronous motors.
Fig. 21-1. (a) Delta connected transformers, (6) An induction motor is one wherein the magnetic
"Y" connected transformers, (c) Transformers field of the rotor is induced by currents flowing
connected open delta. in the stator windings.A synchronous motor
is one wherein the rotor magnetic field is
motor designed to operate
at a proportionally produced by energizing the rotor directly from
lower temperature rise. an external source.
Most motors can be operated with small 21-2. Three-Phase Induction Motors. Three-
overloads for reasonable periods of time phase induction motors are of two general
without damage. However, since the heat types: (1) squirrel cage, and (2) wound rotor
generated in the motor increases as the load on (slip ring). The two types are similar in con-
the motor increases, continuous operation of struction except for rotor design. As shown in
the motor under overload conditions will cause Fig. 21-2, each
has three separate stator
excessive winding temperatures and materially windings, one for each phase, which are evenly
shorten the life of the insulation. and alternately distributed around the stator
ELECTRIC MOTORS AND CONTROL CIRCUITS 409

cote to establish the desired number of poles. The rotor of an induction motor always
A four-pole, three-phase motor will have twelve somewhat less than that of the
rotates at a speed
poles, four poles for each of the three phases. rotating statorfield. If the speed of the rotor

When the stator is energized, three separate were the same as that of the field, the conductors
currents, each 120 electrical degrees out-of- of the rotor winding would be standing still
phase with the other two, flow in the stator with respect to the rotating stator field rather
windings and produce a rotating magnetic field than cutting across it, in which case no voltage
in the stator. At the same time, the currents would be induced in the rotor and the rotor
induced in the rotor windings establish a would have no magnetic polarity. Therefore,
magnetic field in the rotor. The magnetic poles it is necessary that the rotor turn at a speed
of the rotor field are attracted by, and tend to slightly less than that of the stator field so that
follow, the poles of the rotating stator field, the conductors of the rotor winding continuously
causing the rotor to rotate as shown in Fig. 21-3. cut the flux of the stator field as the latter

Line

Squirrel cage
rotor

Fig. 21-2. (a) Squirrel cage


polyphase motor, (b) Wound
rotor (slip ring) polyphase
motor.

Wound
rotor

(6)
410 PRINCIPLES OF REFRIGERATION

Three phase current


A 2
a
=5 •
« .2 b
t5 s I
"5s 60
o
oc
,?, id"
c C
. M E CO
> <•
o a 8 E
j £
oS $
u iil
£«
F*S x: c
i V w
*-»

c 3
5,1 DC
.2 «
Q H
.e
£M 01
U
«
V -c 3
£ .2 E o 2
« l- 1c e 0)
c h- MIC «> 4J
O 0)
r t _
S a
ja u id
rs £u V>
F .2 01
f a
3 X to c
* 00
C « vt «
m c
?S E
E
M 0)
c 3
?l —c 8 E
C c s
L. M »-»
M E
« £ f C
L-
1
si M U ,0 3 C
t-
0) -o* L.
c u 3
s
<0
H 8
§ 8
C 01 •!"

C
» "2 8
« &
C
l£ 8
i a *
« c
i> « j= -I
Q- S *»
c
8
f=
8 1 13
ii
o v e «!
fc* £ 8 £
3
'2
$ 2
U 32
— *>
=
•5 — <c

HI E S
C
15
o -5
id
« To »
> *• Kl 3 —.E
O"
« " » c
J- .C ! a 2

h 3
=
«
4-*
:e
=5

01
§
|
V
\>
"S Jo g Is
c
g i 1*3
cd

5 •»
1
M fc F c «
M r- to

F ••
ELECTRIC MOTORS AND CONTROL CIRCUITS 411

o
8
£
3
-C
a.
i,

M U

Eg.
412 PRINCIPLES OF REFRIGERATION
slips by. The difference between the speed of the in a laminated iron core and connected together
rotor and that of the stator field is called the (short-circuited) at the ends with heavy end-
"magnetic slip" or "rotor slip." The greater rings, giving the winding the appearance of a
the load on the motor, the greater is the amount squirrel cage, from which the motor derives its

of rotor slip. However, since the amount of name.


slip changes only slightly as the load increases or The squirrel cage induction motor is by far
decreases, and is very small even at full load, the most common type of three-phase motor and
three-phase induction motors are usually is available in a number of designs which pro-
considered to be constant speed motors. vide a variety of starting torque-starting current
Neglecting rotor slip, the speed of any characteristics. Two designs frequently em-
alternating current motor is a function of the ployed with refrigerating equipment are designs
frequency and the number of stator poles. The B and C. Design B motors develop a locked
synchronous speed of an alternating current rotor or starting torque between 125% and
motor can be determined by the following 275% of full load torque with relatively low
equation: starting currents. The normal torque-low

Motor speed (rpm)


r =— -
Frequency x 120
Number
t
'— —
of poles
starting current characteristic
makes it ideal for use as a drive for blowers,
of this design

(21-1) fans, and pumps, and for compressors which


are started unloaded. Design C motors have a
For a four-pole alternating current motor high-starting torque-low starting current charac-
operating on 60-cycle power, the synchronous teristic which makes them suitable as drives for
speed is
compressors which must start under load.
60 x 120 Design C motors develop a starting torque
= 1800 rpm
between 225% to 275% of full load torque, but
Rotor slip is usually expressed as a percentage are slightly less efficient than the design B
of the synchronous speed. For instance, for a motor.
motor having a synchronous speed of 1800 rpm Multispeed operation of squirrel cage induc-
tion motors can be obtained by proper design
and operating at 17S0 rpm, the percentage slip
is
of the stator windings. Since motor speed

1800 - 1750 decreases as the number of poles increase, it


°
2/78 /o follows that by doubling the number of stator
1800
motor can be reduced by
poles, the speed of the
Rotor a measure of the power
slip is also one-half. For a single-winding motor, the
losses in the motor. In this instance, 2.78 of % number of poles can be changed in a two to one
the total power input to the motor is converted ratio by bringing extra leads outside of the
to heat in the motor. Hence, there is a definite motor. When more than two speeds are desired,
relationship between the amount of slip and the the motor is wound with two or more separate
efficiency of the motor. The higher the slip, windings for each phase. Two speeds are
the lower is the efficiency of the motor. available with each separate winding.
The motor is the turning
starting torque of a 21-4. Wound Rotor (Slip Ring) Motors.
effort ortorque that the motor develops at the Wound rotor motors are employed in appli-
instant of starting when full voltage is applied cations where the excessive starting current of a
to the motor terminals. Starting torque is large squirrel cage motor would be objectionable
usually expressed as a percentage of full load and/or where a number of operating speeds are
torque and depends to some extent on the desired in the range between one-half and
resistance of the rotor winding. An increase in maximum speed.
rotor resistance increases the starting torque, A slip ring or wound rotor motor induction
but also increases the amount of rotor slip and motor differs from the squirrel cage type only
decreases motor efficiency. in the rotor The rotor winding
winding.
21-3. Three-Phase Squirrel Cage Motors. consists of insulated coils, grouped to form
The rotor winding of a squirrel cage motor definite pole areas so that the rotor has the same
consists of bar-type copper conductors embedded number of poles as the stator. The terminal
ELECTRIC MOTORS AND CONTROL CIRCUITS 413

connections of the rotor windings are brought causes the rotor to rotate. Since the motor
out to slip rings. The leads from the brushes starts as a squirrel cage motor, the speed will be
on the slip rings are connected to external slightly less than synchronous speed. After
resistors, as shown in Fig. 21-26. The operating the motor comes up to speed, direct current is
principle of the slip ring motor isthe same as applied to the field windings. This produces
that of the squirrel cage motor, except that by alternate north and south poles on the rotor
inserting external resistance in the rotor circuit which lock the rotor into synchronization with
when starting high-starting torque can be the rotating armature field.
developed with low values of starting current. By adjusting the flow of direct current to the
As the motor accelerates, the resistance is field coils, the synchronous motor can be
gradually cut out of the rotor circuit until at operated at unity power factor. Therefore,
speed the rotor windings are short-circuited.
full synchronous motors can be applied to an
With the rotor winding short-circuited, the advantage in any large installation where
motor operates with low slip and high effi- constant speed and high efficiency are desired.
ciency. However, the big advantage of the synchronous
The speed of the wound rotor motor can be motor is that it can be used to correct the low
maximum down to approximately
varied from power factor that results from heavily inductive
50% of maximum by inserting resistance in the loads. By increasing the flow of direct current
rotor circuit. The starting resistors can be used through the field winding (overexerting the
for this purpose provided they are designed for field), the synchronous motor is operated with

a large enough current capacity to prevent a leading power factor which can be adjusted to
excessive heating in continuous service. At offset exactly the lagging power factor produced
reduced speeds, the wound rotor tends to lose by inductive loads. Power factor correction
its constant speed characteristic and the speed will in no way affect the load-carrying capacity
of the motor will vary somewhat with the load. of the synchronous motor.
21-5. Synchronous Motors. The synchro- 21-6. Single-Phase Motors. Single-phase
nous motor is so named because the field motors commonly used in the refrigerating
(rotor) poles are synchronized with the rotating industry are of the following types: (1) split-
poles of armature (stator) windings.
the phase, (2) capacitor start, (3) capacitor start
Therefore, the speed of the synchronous motor and run, (4) permanent capacitor, and (5)
depends only on the frequency of the power shaded pole. AH these motors are induction
supply and the number of poles, and is indepen- motors and all employ a squirrel cage rotor.
dent of motor load. The armature winding of The principal factor which distinguishes one
the synchronous motor is similar to those of type from another is the particular method used
the squirrel cage and wound rotor motors. The to produce a starting torque.
field (rotor) winding consists of a series of coils When a single-phase stator winding is
which make up the field poles. The field coils energized, current flow is simultaneous in all
are connected through slip rings to a direct the stator poles and no rotating stator field is

current power source and are so connected produced. Furthermore, the current induced
together that alternate north and south poles are in the squirrel cage rotor winding is such that
formed when the field winding is energized with the magnetic field set up in the rotor is exactly
direct current. The direct current is usually in line with the magnetic field of the stator. The
supplied by a small direct current generator, condition occurring can be compared to the
called an exciter, mounted on the
which is "dead center" condition of a single piston
motor shaft. The rotor is also equipped with a engine. Therefore, there is no tendency for the
squirrel cage winding, called the "damper" rotor to However, if the rotor is
rotate.
winding, which is used to start the motor. The started to rotate by some means, the current
damper winding can be designed for a variety of induced in the rotor winding will lag slightly
starting torque-starting current characteristics. behind the current in the stator winding. This
Whenpolyphase power is applied to the causes the rotor field to lag the stator field and
armature winding, the rotating magnetic field produces a torque that keeps the rotor turning.
acts on the squirrel cage damper winding and Hence, once the rotor of a single-phase motor is
414 PRINCIPLES OF REFRIGERATION

Centrifugal
alone. Since the starting winding is wound
switch with relative small wire, it will heat very quickly
and, if allowed to remain in the circuit for an
appreciable length of time, will be destroyed
by overheating.
Since the maximum phase split that can be
achieved with the split-phase motor is approxi-
mately 30 electrical degrees, the split-phase
Squirrel cage
motor has a relatively low starting torque and
motor
can be used only with machines which start
Fig. 21-4. Split-phase motor.
unloaded. These motors are generally available
in sizes ranging from ii> to a n p for both 115 V
and 230 V operation. They are used primarily
started, a rotating field is produced and the as drives for small fans, blowers, and pumps.
motor operates in a manner similar to that of the 21-8. Capacitor Start Motors. The capaci-
three-phase squirrel cage motor. tor start motor is identical to the split-phase
21-7. Split-Phase Motors. In order to motor in both construction and operation,
produce a starting torque in the single-phase except that a capacitor is installed in series with
motor and make the motor self-starting, a the starting winding, as shown in Fig. 21-5.
second stator winding, called the "starting" Too, the starting winding of the capacitor
or "auxiliary" winding, is employed in addition start motor is usually wound with larger wire
to the phase winding, the latter winding being than that used for the starting winding of the
referred to as the "main" or "running" winding. split-phase motor. The use of a capacitor in
The relative position of the two windings in the series with the starting winding causes the
stator of a four-pole, single-phase motor is current in this winding to lead the voltage,
shown in Fig. 21-4. Notice that the starting and whereas the current in the running winding lags
running windings are connected in parallel the voltage by virtue of the high inductance of
directly across the single-phase line. In the that winding. With this arrangement, the phase
split-phase type motor, the starting winding is displacement between the two windings can be
wound with small wire so that the winding has made to approach 90 electrical degrees so that
a high resistance and low inductance, whereas true two-phase starting is achieved. For this
the running winding is wound with large wire reason, the starting torque of the capacitor
and a high inductance.
to have a low resistance start motor is very high, a circumstance which
Both windings are energized at the instant of makes it an ideal drive for small compressors
starting. However, because of the higher that must be started under full load.
inductance of the running winding with relation As in the case of the split-phase motor, the
to that of the starting winding, the current flow starting winding of the capacitor start motor
in the running winding lags the current flow in" is taken out of the circuit when the rotor
the starting winding by approximately 30
electrical degrees. Since the currents flowing Capacitor
in the two windings are 30 degrees out-of-phase
with each other, the single phase is "split" to Centrifugal
switch
give the effect of two phases and a rotating
field is set up in the stator which produces a
starting torque and causes the rotor to rotate.
When the rotor has accelerated to approxi-
mately 70% of maximum speed, which is a
matter of a second or two, a centrifugal mecha-
nism mounted on the rotor shaft opens a Squirrel cage
switch in the starting winding. With the rotor

starting winding disconnected, the motor


continues to operate on the running winding Fig. 21-5. Capacitor start motor.
ELECTRIC MOTORS AND CONTROL CIRCUITS 415

approaches approximately 70% of maximum


speed, and motor operates on the
thereafter the
running winding alone. Capacitor start motors
are generally available in sizes ranging from £
through f hp for both 1 15V and 230V operation.
21-9. Capacitor Start and Run Motors.
Construction of the capacitor start and run
motor is identical to that of the capacitor start
motor with the exception that a second capaci-
tor, called a "running" capacitor, is installed in
series with the starting winding but in parallel Fig. 21-7. Permanent capacitor motor.
with the starting capacitor and starting switch,
as shown in Fig. 21-6. The operation of the
similar to that of the capacitor start-and-run
capacitor start-and-run motor differs from that
of the capacitor start-and-split-phase motors is
motor, except that no starting capacitor or

that the starting or auxiliary winding remains


starting switch is used. The capacitor shown in

At the instant of series with the auxiliary winding in Fig. 21-7


in the circuit at all times.
starting, the starting-and-running capacitors
remains in the circuit continuously. The
capacitor is sized for power factor correction
Running capacitor but used also as a starting capacitor. How-
is
Starting capacitor
too small to provide
ever, since the capacitor is
a large degree of phase displacement, the
Centrifugal switch starting torque of the permanent capacitor is
very low. These motors are available only in
small fractional horsepower sizes. They are
used mainly as drives for small fans which are
mounted directly on the motor shaft. The
chief advantage of this type of motor is that it
lends itself readily to speed control down to
Squirrel cage SO % of rated speed. Also, it does not require a
rotor
starting switch.
Fig. 21-6. Capacitor start and run motor. 21-11. Shaded Pole Motors. Construction
of the shaded pole motor somewhat
differs

are both in the circuit in series with the auxiliary from that of the other single-phase motors in
winding so that the capacity of both capacitors that the main stator winding is arranged to form

is utilized during the starting period. As the salient poles, as shown in Fig. 21-8. The auxil-

rotor approaches 70% of rated speed, the iary winding consists of a shading coil, which

centrifugal mechanism opens the starting switch surrounds a portion of one side of each stator
and removes the starting capacitor from the pole. The shading coil usually consists of a
and the motor continues to operate with
circuit, single turn of heavy copper wire which is

both main and auxiliary windings in the circuit. Stator winding


The function of the running capacitor in series
Shading coil
with the auxiliary winding is to correct power
(starting winding)
factor. As a result, the capacitor start-and
run-motor not only has a high starting torque
but also an excellent running efficiency. These
motors are generally available in sizes ranging Squirrel cage
rotor
from approximately f through 10 hp, and are
widely used as drives for refrigeration com-
pressors in single-phase applications.
21-10. Permanent Capacitor Motors. Con-
struction of the permanent capacitor motor is Fig. 214. Shaded pole motor.
416 PRINCIPLES OF REFRIGERATION

short-circuited and carries only induced current. currents heats the wire and causes it to expand
In operation, the flux produced by the induced sufficiently to pull contacts "S" open and
current in the shading coil distorts the magnetic remove the starting winding from the circuit.
field of the stator poles and thereby produces a After the starting winding is out of the circuit,
small starting torque. Shaded pole motors are the normal running current through the
widely used as drives for small fans which are running winding will generate enough heat to
mounted directly on the motor shaft. They are maintain the "S" contacts in the open position,
available in sizes ranging from j%j through but not enough to cause additional expansion
approximately £> hp. In addition to its ready of the wire and open contacts "M" (Fig. 21-96).
adaptability to speed control, the main advan- However, if for any reason the motor draws
tages of the shaded pole motor are its simple a sustained overcurrent, the wire will expand
construction and low cost. further and pull open contacts "M," removing
21-12. Hermetic Motors. Motors frequently the running winding from the circuit (Fig.
employed motor-compressor units
in hermetic 21 -9c). Contacts "M" are actually overload
are three-phase squirrel cage motors and split- contacts which act as overcurrent protection for
phase, capacitor start, and capacitor start-and- the motor. The mechanical arrangement of
run single-phase motors. Whereas the split- the two sets of contacts is such that contacts
phase and capacitor start motors are limited to "M" cannot open without also opening contacts
small fractional horsepower units, the capacitor "S."
start-and-run motor isused in sizes from $ Since the action of the hot wire relay depends
through 10 hp. Three-phase squirrel cage on the amount of current flow through the alloy
motors are employed from 3 hp up. wire, these relays must be sized to fit the current
Although air, water, oil, and liquid refrig- characteristics of the motor. They are best
erant are sometimes used as cooling mediums applied to the split-phase type motor.
to carry away the heat of hermetic motors, the 21-14. Current Coil Relays. The current
large majority of hermetic motors are suction coil relay is used primarily with capacitor start
vapor cooled. For this reason, hermetic motor- motors. It is a magnetic type relay and is

compressor units should never be operated for actuated by the change in the current flow in
any appreciable length of time without a the running winding during the starting and
continuous flow of suction vapor through the running periods. The coil of the relay, which is
unit. made up of a relatively few turns of large wire,
In the single-phase hermetic motor, a spe- is connected in series with the running winding.
cially designed starting relay replaces the shaft- The relay contacts, which are normally open,
mounted centrifugal mechanism as a means of are connected in series with the starting winding,
disconnecting the starting winding (or starting as shown in Fig. 21-10.
capacitor) from the circuit after the motor When the motor is energized, the high locked
starts. Three types of starting relays have been rotor current through the running
passing
used, namely: (1) the hot wire or timing relay, winding and through the relay coil produces a
(2) the current coil relay, and (3) the voltage relatively strong magnet around the coil and
coil or potential relay. causes the relay armature to "pull-in" and close
21-13. Hot Wire Relays. The hot wire the starting contacts energizing the starting
relay depends on the heating effect of the high- winding. With the starting winding energized,
starting current to cause the thermal expansion the rotor begins to rotate and a counter emf is
of a special alloy wire, which in turn acts to induced in the stator windings which opposes the
open the starting contacts and remove the and reduces the current through the
line voltage
starting winding from the circuit. As shown in windings and relay coil. As the current flow
Fig. 21-9, the hot wire relay contains two sets through the relay coil diminishes, the coil field
of contacts, "S" and "M," which are in series becomes too weak to hold the armature,
with the starting and running windings, respec- whereupon the armature falls out of the coil
tively. Both sets of contacts are closed at the field by gravity (or by spring action) and opens
instant of starting so that both windings are con- the starting contacts. The motor then runs on
nected to the line (Fig. 21-9a). The high-starting the running winding alone.
ELECTRIC MOTORS AND CONTROL CIRCUITS 417

Line

Thermal element

Thermal element

Thermal element

Fig. 21-9. Hot wire starting relay, (a) Starting position, (b) Run position, (c) Overload position.

21-15. Potential Relays. Potential or voltage the motor is energized, both the starting and
coil relays are employed with capacitor start running windings are in the circuit. As the
and capacitor start-and-run motors. The motor starts and comes up to speed, the voltage
potential relay differs from the current coil type in the starting winding increases to a value
in that the coil is wound with many turns of considerably above that of the line voltage
small wire and is connected in parallel with (approximately 150%), as a result of the
(across) the starting winding, rather than in action of the capacitor(s) in series with this
series with the running winding, as shown in winding.*
Fig. 21-11. The relay contacts are connected * The vector sum of the voltages across the starting
in series with the starting capacitor and are winding and capacitor(s) is equal to the line
closed when the motor is not running. When voltage.
418 PRINCIPLES OF REFRIGERATION

lays contain built-in overcurrent protection,


Relay (starting) these are usually sensitive only to motor over-
contacts current and do not provide protection against
overheating from other causes.
21-17. Motor Starting Devices. For frac-
tional horsepower motors the motor starting
equipment sometimes consists only of a direct
acting (line voltage) manual switch, thermostat,

w Relay or low pressure control installed in the motor


coil
circuit between the motor and the power
Fig. 21-10. Current-coil type starting relay.
source (Fig. 21-12a). The control acts to open
and close the motor circuit to stop and start

The high voltage generated in the starting


winding produces a relatively high current
115 volt power
flow through the relay coil and causes the coil
armature to pull in and open the starting
contacts. With the capacitor motor, start -Neutral

opening the relay contacts disconnects both the


starting winding and starting capacitor from
the circuit. With the capacitor start-and-run
motor, only the starting capacitor is disengaged. Fused
With either type or motor, the starting winding disconnect or
circuit breaker
voltage decreases somewhat when the starting I

contacts open, but remains high enough to hold L.


the coil armature in the field and keep the
starting contacts open until the motor is Operating
stopped. control

Thermal Overload Protection for


21-16. I

Hermetic Motors. All hermetic motor-com-


pressors should be equipped with some type of
thermal device which will protect the motor
(Motor)
against overheating regardless of the cause.
Thermal overload devices of this type are usu-
ally designed to be fastened directly to, and in Fig. 2I-I2<i. Direct acting, line voltage controls with

good thermal contact with, the motor-com- 1 15 V single-phase motor.

pressor housing, so that they are sensitive not


only to motor overcurrent but also to over- the motor, respectively. Safety controls, such
heating resulting from high discharge tempera- as high pressure cut-outs, overcurrent protective
tures and other such causes. devices, etc., are connected in series with the
Although some types of motor starting re- operating or "cycling" control, as shown in Fig.
21-126. The contacts of the safety controls are
Running normally closed and do not open to break the
capacitor
circuit unless called on to perform their pro-
tective function.
With low voltage, single-phase power, the
line voltage controls are installed in the "hot"
line,never in the neutral (Fig. 21-12a). With
high voltage, single-phase power, the controls
may be installed in either one or both of the
-Relay coil
power In the case of three-
lines (Fig. 21-126).
Fig. 21-1 1. Potential type suiting relay. phase power, at least two of the three power
ELECTRIC MOTORS AND CONTROL CIRCUITS 419

linesmust be opened to disconnect the motor


from the power source. This requires the use
of double-pole controls, as illustrated in Fig.
21-12c. However, in all cases, regardless of the
type of power supplied, all "hot" lines must be
protected individually with a properly sized
fuse or circuit breaker.
Since the contacts of direct acting controls
must be heavy enough to carry the full load
Fused disconnect
current of the motor(s) they are controlling, or circuit breaker
these controls tend to become unwieldy when
the full load current of the motor exceeds 15 or
20 amperes. Therefore, general practice is
to control larger motors indirectly through a
magnetic contactor.
A magnetic contactor or motor starter is
essentially an electrical relay which in its
simplest form consists of a coil of insulated wire,
Hi-Lo
called a holding coil, and an armature to which pressure
control

230 volt single-phase power

Overcurrent
protectors

I Fused
I
disconnect
I or circuit
I breaker

Fig. 2I-I2c. Direct acting, line voltage controls used


with 230 V three-phase power.
r
Operating I

control I

the electrical contacts are attached (Fig. 21-13).


The operation of the magnetic contactor is
Hi-Lo similar to that of the solenoid valve described
pressure
(safety
in Chapter 17. When the holding coil is
control) energized, the armature is pulled into the coil

magnetic thereby closing the electrical


field,

contacts and connecting the motor to the power


source. When the holding coil is de-energized,
Overcurrent the armature drops out of the coil
field, causing
protector the contacts to open and disconnect the motor
from the power source. When a magnetic con-
tactor is employed, the motor is controlled
indirectly by controlling the contactor holding
coil. Therefore, the operating control is
installed in series with the holding coil in the
Fig. 21-126. Direct acting, line voltage controls used holding coil circuit rather than directly in the
with 230 V single-phase power. motor circuit.
420 PRINCIPLES OF REFRIGERATION

Fig. 21-13. Motor controlled


indirectly through magnetic
Magnetic^ contactor.
contactor

Armature
(insulated J
ir---
from ^Operating or
contactors) "cycling" control

The advantages gained by employing mag- section is called an "across-the-line" starter, and
motors to the power
netic contactors to connect is so named because it connects the motor
source are several. First, since the current directly across the line at full voltage immedi-
required to energize the holding coil is small, ately when the holding coil is energized. This
the contacts of the operating and safety controls type of motor starter is suitable for motors up
can be of relatively light construction, which to 20 or 25 hp and is more widely used than any
results in a reduction in both the size and the other type. However, in order to prevent
cost of the controls. Second, since the holding excessive current surges in the power lines

coil circuit is electrically independent of the during the starting period, general practice is to
motor circuit, the holding coil circuit voltage start squirrel cage motors above 25 hp under
may be different from that of the motor circuit. reduced voltage. Reduced voltage starting is
This permits the use of low voltage (usually accomplished through the use of reduced
24 V) control circuits, which are safer and voltage starters which introduce resistors or
generally less expensive to buy and install. A auto-transformers into the motor circuit during
magnetic contactor employing a low voltage the starting period. A resistance type reduced

control circuit is shown in Fig. 21-14. voltage starter is illustrated in Fig. 21-15. When
Holding coils for magnetic contactors are the operating control closes, the #1 holding
manufactured for all standard voltages and coil is energized and the main contacts (#1)
frequencies and are readily interchangeable in close, thereby connecting the motor to the
the field. The holding coil voltages most power source through the resistors. This
commonly used are 24, 115, 230, and 460. allows the motor to start under reduced voltage
21-18. Reduced Voltage Starters. The mag- and, at the same time, energizes the timing
netic contactor described in the preceding relay. After a predetermined time interval, the
ELECTRIC MOTORS AND CONTROL CIRCUITS 421

Fig. 21-14. Magnetic contactor Transformer


with low voltage control
circuit.

24 volts
holding coil Operating
control

Relay heater )_ . , ,
0verload rela *
Relay contacts
j

Timing
relay Fig. '21-15. Resistance type
contacts
reduced voltage starter.
422 PRINCIPLES OF REFRIGERATION
motor is subjected to a sustained overcurrent,

the temperature of the heater element increases


above normal and the excess heat given off by
the heater causes warping of a bimetal element
(or melting of a special alloy metal) which opens
the overload contacts in the holding coil

circuit. This de-energizes the holding coil

which in turn disengages the motor from the


power source. A
time delay action built into
the overload relay prevents tripping of the
overload during the motor starting period and
during momentary overloads.
21-20. Oil Pressure Failure Control.
Another safety control frequently encountered
in the control circuits of refrigeration equipment
is the oil pressure failure control. The function
of this control is to cycle the compressor off

when the useful oil pressure developed by


the oil pump falls below a predetermined
minimum, or in the event that the oil pressure
fails to build up to the minimum safe level
within a predetermined time interval after the
compressor is started. An external view of
the oil pressure failure control is shown in Fig.

21-16.
In studying the operating characteristics of
the oil pressure failure control it is important to
recognize that the total oil pressure, as measured
Fig. 21-16. External view of oil pressure failure by an oil pressure gage, is the sum of the crank-
control. (Courtesy Penn Controls, Inc., Goshen, case (suction) pressure and the pressure
Indiana.)
developed by the oil pump, and therefore is not
the true or useful oil pressure. To determine
the useful oil pressure, the suction pressure

timing relay closes and energizes the #2 holding must be subtracted from the total oil pres-
the difference between the two being
coil, which in turn closes the #2 contactors
sure,

and shunts the resistors out of the motor the useful oil pressure developed by the oil

circuit.
pump.
Motor Overcurrent Protection. It To be effective, the oil pressure failure switch
21-19.
and must be actuated by the useful oil pressure
is important to recognize that line fuses
rather than by the total oil pressure. This is
circuit breakers are designed to protect the
circuit only and do not provide overcurrent pro-
accomplished by using two pressure bellows
tection for the motor. Therefore, unless the opposed to each other, as shown in Fig. 21-17.

One bellows connected to the crankcase and


motor is equipped with a built-in thermal over- is

reflects crankcase pressure, whereas the other


load, separate overcurrent protection must be
provided in the circuit of each motor. To bellows is connected to the discharge of the oil
pump and reflects total oil pressure. The
satisfy the need for overcurrent protection,
many magnetic motor starters come equipped pressure differential between the two bellows

with overload relays. pressures is equal to the useful oil pressure


and utilized to actuate the pressure differ-
The "overload relay" consists essentially of is

ential switch of the oil pressure failure con-


two parts: (1) a heater element installed in the
motor circuit and (2) a set of contacts installed trol.

in the holding coil circuit. In the event that the A time delay relay incorporated into the oil
ELECTRIC MOTORS AND CONTROL CIRCUITS 423

pressure failure control allows the compressor willbe exactly equal to the crankcase pressure
to operate 90 to 120 sec with the oil pressure during the compressor off cycle, and both the
below the safe level. This permits the com- timing relay heater and the holding coil are
pressor to start with zero oil pressure and also energized. If, after the compressor starts, the
prevents unnecessary shut-down of the com- useful oil pressure builds up to the cut-in
pressor in the event that the oil pressure momen- pressure of the oil pressure safety control, the
tarily below the minimum safe limit.
falls differential pressure switch will open and remove
However, if the oil pressure does not rise the relay heater from the circuit. This action
to the safe level within the alloted time, the will allow the compressor to continue normal
oil pressure failure control will shut-down the operation. On the other hand, if the useful
compressor. Before the compressor can be oil pressure does not build up to the cut-in
restarted, the oil pressure failure control must pressure of the control within the alloted time,
be reset manually. the differential pressure switch will not open
Referring to Fig. 21-17, notice that the timing and the heater is left in the circuit. Continued
relay consists of a timing switch and a heater operation of the relay heater will cause the
element. The timing switch is connected in bimetal of the timing switch to warp and open
series with the holding coil of the magnetic the timing contacts. This breaks the holding
starter, and the heater is connected in parallel coil circuit and stops the compressor.
with the holding coil. The pressure differential If the useful oil pressure falls below the cut-
switch is connected in series with, and controls out point of the oil pressure failure control while
the operation of, the relay heater. The resistor the compressor is operating, the differential
in series with the relay heater limits the current pressure switch closes and energizes the relay
flow through the heater and makes the oil heater. If the oil pressure does not build up to
pressure failure control adaptable to both 1 IS V the cut-in pressure again within the alloted time
and 230 V control circuits. interval, continued operation of the heater will
Since the oil pump operates only when the open the timing switch and stop the com-
compressor is operating, the total oil pressure pressor.

To
motor

To discharge
of oil pump

Fig. 21-17. Oil pressure failure control.


424 PRINCIPLES OF REFRIGERATION

a
3
O
p

8
ELECTRIC MOTORS AND CONTROL CIRCUITS «5
As indicated in the foregoing, the oil pressure interlocked that the compressor cannot operate
both a cut-in pressure and a
failure control has unless the evaporator blower and the condenser
cut-out pressure. These should be set in pump are operating. One
fan or of the more
accordance with the compressor manufacturer's common methods of achieving the desired
instructions whenever such data are available. interlocking is illustrated in Fig. 21-18. In this
In the absence of these data, general practice is instance, the evaporator is permitted to operate
to set the cut-in point of the control for a continuously and is controlled with a manual
pressure approximately 5 psi below the useful off-on switch. With this particular control
oil pressure when the compressor is in operation. arrangement, the fan control is the lead control
The cut-out point is usually set for a pressure and as such may be used to start and stop the
approximately 5 psi below the cut-in pressure. entire system.
For example, assume that the crankcase pressure The holding coil of the condenser starter is
is 37 psig and the total oil pressure is 72 psig, so wired through an auxiliary contact in the
that the useful oil pressure is 35 psi (72 -
37). evaporator blower starter. Since the auxiliary
The cut-in pressure should be set at approxi- contact will be closed only when the holding coil
mately 30 psi (35 - 5) and the cut-out pressure of the blower starter is energized, the condenser
at approximately 25 psi (30 — 5). fan or pump cannot be started without first
21-21. Interlocking Controls. As a general starting the evaporator blower. Likewise, the
rule,a refrigerating system employs at least holding coil of the compressor starter is con-
three motors: the compressor motor, the nected through an auxiliary contact in the
evaporator blower motor, and the condenser condenser starter so that the compressor starts
fan (or pump) motor. Good design practice unless the condenser and evaporator starters are
requires that the controls of these motors be so energized. Notice also that the cycling control

Fig. 21-19. Two motors oper-


ating through one magnetic
contactor.

V V
Overcurrent
protection for
condenser fan
\j \<*MH

230 v
3 phase

Compressor Condenser fan


426 PRINCIPLES OF REFRIGERATION

8K

1
01

E .2
v
*» t.

*• >
c °
* -°
§ t

<2 S-
. >
ELECTRIC MOTORS AND CONTROL CIRCUITS 427

(thermostat) the condenser starter


controls is usually confined to small, packaged equip-
rather than compressor starter.
the This ment and requires that separate overcurrent
arrangement permits the condenser fan or pump protection be provided for each motor.
to cycle off and on with the compressor. A
wiring diagram- for a simple pump-down
Another common method of accomplishing system with interlocking control is illustrated in
the same result is to operate the compressor and Fig. 21-20. Notice particularly the method
condenser motors through the same magnetic of interlocking low voltage and high voltage
contactor, as shown in Fig. 21-19. This method control circuits.
Tables
and
Charts
430 PRINCIPLES OF REFRIGERATION

TABLE l-l. Pressure Conversion Factors


Multiply By To Obtain

Atmosphere 29.92 Inches of mercury


Atmosphere 33.93 Feet of water
Atmosphere 14.70 Pounds per square inch
Atmosphere 1.058 Tons per square foot
Feet of water 0.881 Inches of mercury
(at 32° F)

Feet of water 62.37 Pounds per square foot


Feet of water 0.4335 Pounds per square inch
Feet of water 0.02950 Atmospheres
Inches of mercury (at 62° F) 13.57 Inches of water (at 62° F)
Inches of mercury (at 62° F) 1.131 Feet of water (at 62° F)
Inches of mercury (at 62° F) 70.73 Pounds per square foot
Inches of mercury (at 62° F) 0.4912 Pounds per square inch
Inches of water (at 62° F) 0.07355 Inches of mercury
Inches of water (at 62° F) 0.03613 Pounds per square inch
Inches of water (at 62° F) 5.202 Pounds per square foot
Inches of water (at 62° F) 0.002458 Atmospheres

TABLE 3-1. Properties of Gases


Gas C„ C„ k i

Air 0.2375 0.169 1.406 53.3


Ammonia 0.508 0.399 1.273 90.5
Carbon
dioxide 0.207 0.162 1.28 35.1
Carbon
monoxide 0.243 0.173 1.403 55.1
Hydrogen 3.41 2.42 1.41 765.9
Nitrogen 0.244 0.173 1,41 55.1
Oxygen 0.218 0.156 1.40 48.3
Sulfur
dioxide 0.154 0.123 1.26 24.1
TABLES 431

TABLE 4-1. Properties of Saturated Steam

Absolute Pressure Specific Volume Enthalpy Entropy


Temp.,
°F, Psi, In. Hg, Sat. Evap., Sat. Sat. Evap., Sat. Sat. Evap., Sat.
t P P liquid, vapor, liquid, hfll vapor, liquid, St. vapor,
Of "t h, K s, s,
(1) (2) (3) (4) (5) (6) (7) (8) (9) 00) (11) (12)

32 0.08854 0.1803 0.01602 3306 3306 0.00 1075.8 1075.8 0.0000 2.1877 2.1877
33 0.09223 0.1878 0.01602 3180 3180 1.01 1075.2 1076.2 0.0020 2.1821 2.1841
34 0.09603 0.1955 0.01602 3061 3061 2.02 1074.7 1076.7 0.0041 2.1764 2.1805

35 0.09995 0.2035 0.01602 2947 2947 3.02 1074.1 1077.1 0.0061 2.1709 2.1770
36 0.10401 0.2118 0.01602 2837 2837 4.03 1073.6 1077.6 0.0081 2.1654 2.1735
37 0.10821 0.2203 0.01602 2732 2732 5.04 1073.0 1078.0 0.0102 2.1598 2.1700
38 0.11256 0.2292 0.01602 2632 2632 6.04 1072.4 1078.4 0.0122 2.1544 2.1666
39 0.11705 0.2383 0.01602 2536 2536 7.04 1071.9 1078.9 0.0142 2.1489 2.1631

40 0.12170 0.2478 0.01602 2444 2444 8.05 1071.3 1079.3 C.0162 2.1435 2.1597
41 0.12652 0.2576 0.01602 2356 2356 9.05 1070.7 1079.7 0.0182 2.1381 2.1563
42 0.13150 0.2677 0.01602 2271 2271 10.05 1070.1 1080.2 0.0202 2.1327 2.1529
43 0.13665 0.2782 0.01602 2190 2190 11.06 1069.5 1080.6 0.0222 2.1274 2.1496
44 0.14199 0.2891 0.01602 2112 2112 12.06 1068.9 1081.0 0.0242 2.1220 2.1462

45 0.14752 0.3004 0.01602 2036.4 2036.4 13.06 1068.4 1081.5 0.0262 2.1167 2.1429
46 0.15323 0.3120 0.01602 1964.3 1964.3 14.06 1067.8 1081.9 0.0282 2.1113 2.1395
47 0.15914 0.3240 0.01603 1895.1 1895.1 15.07 1067.3 1082.4 0.0302 2.1060 2.1362
48 0.16525 0.3364 0.01603 1828.6 1828.6 16.07 1066.7 1082.8 0.0321 2.1008 2.1329
49 0.17157 0.3493 0.01603 1764.7 1764.7 17.07 1066.1 1083.2 0.0341 2.0956 2.1297

50 0.17811 0.3626 0.01603 1703.2 1703.2 18.07 1065.6 1083.7 0.0361 2.0903 2.1264
51 0.18486 0.3764 0.01603 1644.2 1644.2 19.07 1065.0 1084.1 0.0380 2.0852 2.1232
52 0.19182 0.3906 0.01603 1587.6 1587.6 20.07 1064.4 1084.5 0.0400 2.0799 2.1199
53 0.19900 0.4052 0.01603 1533.3 1533.3 21.07 1063.9 1085.0 0.0420 2.0747 2.1167
54 0.20642 0.4203 0.01603 1481.0 1481.0 22.07 1063.3 1085.4 0.0439 2.0697 2.1136

55 0.2141 0.4359 0.01603 1430.7 1430.7 23.07 1062.7 1085.8 0.0459 2.0645 2.1104
56 0.2220 0.4520 0.01603 1382.4 1382.4 24.06 1062.2 1086.3 0.0478 2.0594 2.1072
57 0.2302 0.4686 0.01603 1335.9 1335.9 25.06 1061.6 1086.7 0.0497 2.0544 2.1041
58 0.2386 0.4858 0.01604 1291.1 1291.1 26.06 1061.0 1087.1 0.0517 2.0493 2.1010
59 0.2473 0.5035 0.01604 1248.1 1248.1 27.06 1060.5 1087.6 0.0536 2.0443 2.0979

Reproduced with permission from Thermodynamic Properties of Steam by Keenan and Keyes, published by John
Wiley and Sons.
432 PRINCIPLES OF REFRIGERATION

•all as tests s^^SiSfc fcPNsa n«ohB efSSKoo 5


Isssr S&SK5 §5fes52 ssiss iisss s?
ujioioio^J * •*! •* oo co eo C4 N oi si HtHHHO ooodo o
I I I I I I I I I I I I I I I I 111+++
I I I I

a
I
O^OCOOO NNHOO U5 O) * 00 CO t>C4ISHia ©TfOicOOO tN
8 3 S S a! ujujujiguj ufigujigi !§;?<-**« ;- f- «_ ** ** «
•a
JSJ f§§§ §§j OO oc ooooo
I
CJO^Hi-HrH i-<t
3°,S§
OMQ! IS3S
E
eoioujirjio •* * Tt< * eo connnn c^n-hf-h— i i-<oooo o
I I I I I I I I I I I I I I I I I I I I I I I I I

*
.3

3 o»o»ooo OriT-i^rt tNtNeSSco *0 CO eo 5 * -5* * 3 3! lO «5


OO^H^H^H i-H ^H *H ,H F-f t-| ^ i-l i-l ^H ^H^H^H^H^H i-i i-i r-i ,-H ^H i-H

<

*
Ms
OS
>a sS' a §£883 &S222 8S3S88 88899 9SSSSS 8§

i -b
5"°
O O »H ^H r* i-4 ^H ^H ^H *-l *H »-t r-H f« »-t »H -H -* rH »-« i-t ^H ^H ^H t-( i-<

NQO)a> oatocoo
t-iO»C0 00K3
eo^ustot. N^XOOXN
I o < N tN tN nnMNco Dsncococo »* >* ^< * io io

&t fo 88338 83§2§ 38^23 SS8882 gfeiSSS S3


2
a
ft 38883 SSSSS? 38839 39S3S5 SSfeSS R
(A

a OOOOO OOOOO OOOOO OOOOO OOOOO


s
as- 88§f 8i3c?85 ggSSg 8KS28 2
waon SHioftg i-HObt-iOU} O00-Ht~O
COU3<Ot»A i-HtN^<cOOO OMiOlOO t-TfcOM*'
<N ifl 00 f-h tJ( )>.
s HHHNN N«NNIM eOOOOOCOM * * U5 lOKSWiJOtO <0 •»»( t(<

ao to Sous oo»3>obo t»53S H OOOOOCO USOtOcOKJ P5

ft 52.
CRtpiOTflOO ison
00 05 05 0>iCT(Nro THu5"5tDt- 00

OB
to
w S2S iNO.eOSO" 'f^SSS ©352359 00«5(Nt^<5 eo
OS «PCOi-H*C
<o:tfc;SSE
3t~ooe»o S3S9'* <2«oc<<<3io5
Ncs^iiSt^ oooncoSS t~
o.
rtHHH i-H t-l t-< iH i-l rli-l-H-HfNNNlNNlN (N CO P5 CO CO CO

SSiSSS SSSJh 50 "o *" «*> "-1 O0500i>-«0 «5 1< CO IN -h o


I I I I I I I I I
I I I I I I I I I I I I I I I I
TABLES 433

-8.1 S8S9S
1-11$ l^feSS
««soO'H* t^-i^t^o
o5-h*«o8>
8S3SS5I
wt^oosS
"8 O'-K-i'H'H MMMCOCO PJ^i^rjiio uj «5 ddd
a
& NNHtDO lOOS^OOCO NNBhd O^OSCOOO
1 * 1
1 ^ o>o>oo^ •i'i?3S52 SSS'S'S dddt^t^
§&»1h 5.
jsw.g §§§§§ §§§§1 liils iii§§
S^sbSS Sf2S«
SHH09H q5s«'§
OtNI't** *<O00i-Hc5
SSkSS
iQogonio
5 ooooo -h ih n in ei eo si i-i i-i esj eo co * •* •*

-8+3-1 NUlNOn
W 9CJ4INO
u5«5tOtot- t^wt^oooo
>QCQi-<CQ«p
ooaoosaSoS
<£-''*(><*
Soooo
i-H i-H -H i-H »-H »-4 i-H -H -H i-H -H »^ -H IN O) IN IN
L. 1-4 t-t 1-4 1M 1—4 l-H -H l-( l-H 1-4 HHHlHH
i-H i-H i-H
-H i-H -H
1-H 1-4

"i
88888 SSKfSS 88883 S88S8
s
_ rtrt —c— I ririHriri
t—4 H -Hrfrt—l-l
i-Hr-ltNtNtN
HHrtHH HHvHHiH
& i-H i-H i-H i-H i-H i—1 i-H i-H

>
nh^n noaon
S«omn i-iiScorat-
ooioiotpp
i-kc-hSin
t>eano
t»co£cc>eo
ujuj<eco<s t-^t^oooo da>o©.-i
HMH HMnn^
HHI"
I o 4 1—4 1-4

o iNio-*a>-H
iijn-hnn
Hi>f)a* hsBcSo t»<Hb.<et> riaone ei«eiat»n
eoeo^io® qe»«T**
1~ QQQO oohhS
3 2°23fc::2$ SwooebQ
§t»9B9CCS
*H 1H1H1H1HN
-H i— -H
1 a 8888 88888 88888 88888
4 i-H i-H l-H •-! l-H 1-4

1 e
ooooo ooooo ooooo ooooo
(A 3
OOMWO tOTt«'«tOOS
UJOU5NO Q^t^O^i
r»Q(N«J0p NHiiSn «h«nu;
^35io55ui 3oi«r<N t^ooaSoo«& SoShh
06 CO 00 CO ao

O gs OOOOO OOOOO OOOOO


8 o OrH^r-4-H
a
•3
s HagijjS
roco 00606 SssO?
onsass 2SSB*o
SnooH 53 «©.•*"?
<oeo9it«ro
n^isoSn r~-Hioo>e oont>4 ©t~Tii-Hc6
2 i "^t2t~h»gp coaaoo i-n-itNiNeo
1-4 l-H l-H -H l-H
9-4<iqco«o
l-H i-l l-H —H 1-H

&l
P4
OOOOO ooooo ddddd ooooo
eoco«p<oe» St-OOOH NrtOOOH COQOCQIN
S
3
-*H
PL|

O ddddd ddddd ddddd ddddd


Mm A
o « niooaio «^<in<noo
r>t^ooQ-jh
nooojas n$SN»
SS^i-iosSS S H S9^* S?2*SS oi
§^§335
oooo 3s2sS ooooo sii>88Sj
ooooo sslsfejl ooooo
ddddd ddddd ddddd ododd
Oi-HtNCO-* U5«Ot-00<3> Oi-HNeO'* U5«OW00O»
SI* 3
434 PRINCIPLES OF REFRIGERATION

00 CM CD CO
"ft
cSSScSm
CNcNCSItfit*
aSBhiK CO CM CO C5 C©
SStNS>i5
Oi c» lO IQ CO ** t* . „
33<00>-H>ft OS&SSOTfl OOCOI^-^iS fcQSSSJ2
CS^jOOCMt^ 3S59Sr3
*4Cg<-H<OrH
t-^t^oooo ooosoioo o^-HcieN cNtoco-*'* wjococor-.

a S3
& CMt^-«COO lOO5-**«00CO t*-NC0~*»O O "* Ci CO 00 NSHtDO
& a
&?• 3 _
P f'oooioJo oohhw r~r^r--t~t- c-t>.t^r^t>-
ujto'iococo t^h-ooooos
c<i co ro ^i Tii
cococoti
ooooo ooooo ooooo t~t-t^f~t
oooo FSKt^r-t- ooooo
I
rococo coco cpc©
» a, =
000000(3000 oot^t^r-b- i^-t^t*t*t^
SonEn 05CM'*t-05 H^IOOH COCOOO-HCO lO CO O CO i5
^H to iO »0 iO to CO CO CO CO t^- t^> t— 1~ 00 00 00 00 OS 05 0505000

°+3-- CO 35 CM-* t^ OSCM«O00*H


CM CM CO CO CO lO CO CO CO COt-f-t^OO
CMCMCMCMCM
_(*^,—|Wt-H CMCNCMINCM
»-) IHHH
i—
CM CM IN CM IN
i-lHrtHH CM CM IN
t-4
CM IN
CM CM
i—I .—I
N
^H >—
CM CM CM CM CM
i— t-H *-H i—I •-(
I

35 oo-Hcocooo nncoco^ eocooo**co cooo^hcoco osfH^coo*


>a =2' 3 a?
S^ySS cmcMNCMCo eococo-*-* <*i|iioioio incocoeoeo
INININININ CMINCMININ CM CM CM CM IN CM CM IN CM IN CM O. CM CM CM

"8 o
•3
-» O CO CO
Ol-iiOCoJ-l
t>- <Ol^'*C005 N-iONlO
O 05 05 05 05 O «5 CO
CM. Tt<
COt^cOCNt—
CO O CO CO O CO Q—
CO 00
CO 00 CO 00 3<

J •at •O LO CO '_
t~ 00 05 O -H CM lO CO fi 00 C35 — CM CO * CD t~ 05 CM O
^ t^ S S CJ5
XiincMCMCM
Tt<
cS CM N. CM CM
<
CM CO CO CO CO CO CO CO •* •*

O
T3 <p-ic CM 00 00 CO CO
COMHOOi *—IC0005
s. C .3 _ —
<iOCOt ... - IN IOCS CO 00 "*
3! &
— cm co ^ »o
.
111
3 r- oo os t-h cm CM^COOOOJ
3 S* ^ ^ ^
i

-I
(0 T3
O
a o '
cmincMcm£!
2JS323S222
83838 88888 58888 ooooo 88888 OOOOO 1

2
-S
&
OOOOO OOOOO OOOOO OOOOO OOOOO
3
CMO-HC005 oo-s^—ii: sco-^cooo coocooo
NOIOIOCD
.6^ n<C5COIN00 >O<NO00C ^COCMOOO
H •—
CM CM CO <* * ")3SN< iO^HCMCM CO •** iQ CO b-

O .-I CM CM IN CM CM IN CM IN IN CN CM CO CO CO
jB
M NcoW'HO
* "* r- 05
^3 o
t~ocoi<co _icMtocn-*i
) CO -^ CO IO
itHOCOCO
3 t-05 0")CM co 10 co r^ cs >cm-*i35n.
i ^pooo CM 6} CO CO CO
1
CO CO CO coco
35,
o 10000
>oooo
ooooo 00000 ooooo 00000 00000
"b o _. CM
Soo ,. -^
CM1S0O-H
cort
QrlWOH coocooo
ioo-*ogt-
w CO
Q-* 00 CMC
»"5 ><? ^
'

O-H CO CO — t^iOiOt-O
-HCO-1COCM
<coe
"~
SCO'
CO-*oOoooOC -

o* SU S 0000 to»o »o
oooc CM CM CO

OOOOO OOOOO OOOOO OOOOO ooooo


ii SIOOIOCM
OS IO CM -^
00 15 CO CM —
i^> 10 r^ CO ^
<

C^I^COOO-*
OSHHCi ©COCOON *CMph-'0 «HOOnO
O-HNClfl t^t~^00O5
•*tOCO^00
_ -H -H -J -H COCO-*"5U3
,-H»H-<rt*H
cOt~.0000Ci
—(rt-H^rt CMcMCNCMCM CMCMCMCMCM
OOOOO OOOOO OOOOO OOOOO ooooo
a O«-HCflC0^ iOCONOOO O — W ?C rtt iO ffl h> 00 53 OhNW^i
Eg* «
TABLES 435

3232°. 88838 SMS S?8S9S 8E332 . 888SS 99999


,h ,_,_,_,,-< 88n<38 88e$38 S&eSsSS §§eoco8 3eoeo8eo Seo95<SS

usos^ooeo t>M«Hio o^oseooo mnhoo «o oi-* oo co t»e««oi-no o^onoo

0>CQNH1Q
83882 COCoS*HCO 88©-33 gS&Sg SS5SRS
OHHHrt NNNNM CO 00 CO CO *4* -**«*< lO IQ kQlftCOCOCO cor» >>»» t-ooooooao

aboodSoSo) 8S8S2 ssass 00 CO 00 ^* ^* 93JS83 SS88S8 00 o» cS So


nnnnn 9^ CO CO CO CO 00 00 00 CO CO CO 00 00 CO CO 00 CO CO CO 00 CO 00 00 CQ CO CO CO 00 CO ^*

SSS8S3 3883c* 88338 83322 33883 315993 59S3S


eqctcttcNC* C4C*eqeic« ej €4 co eo co co'cocicoco cocooococo cococoooco cococococg

"8
3
.C
*3
C 3S58S989838 SSK889 38S82 982oo* n«n«« **««mm
39983 SSfeSS 388ciS 1:8888 88888 23288 33998
m ^•h cocoa
ui lli 28329
98! O 33883!
§3 HHHC4M o«sast»oB
-I
a ooooo ooooo ggSsS
ooooo?»HtH W »H t^
>oo ooooo
*- 1>H »* *H t-H fH *H iH *H *-4(-l vH

2
ooooo ooooo ooooo ooooo ooooo ooooo ooooo

COCOCOCOCO
mn
* * * * •*
ggortrij
-*>OU5U5>0
H8feg SIIS8
IQIQCSCOCO ONISNS
>eo»ooo>-i
00000000©
1§S8S
0505000

12829 }fc3S 905


mi J3929 SNHt
©g*«c
)QCj I5*C>
NNC )0©S-H CO CO ^* *

88888 88888 >*8888


OOOOO OOOOO OOOOO OOOOO OOOOO OOOOO ooooo
NM0D4H
•* ui us 35 r- £131! 82832
38383 ill;
5SKc
Scocoe
2S82S
CO CO 83
ooooo ooooo ooooo ooooo ooooo ooooo ooooo
pes ooooo
>3eo<-i 3&38S 22J lis
Seocoeoco
s»8S<
3COCO ^* ^ §9! t»b.t->ooab
ooooo ooooo odood ddddd ddooo ooooo

39988 83333 385588 88883 38S88 2CBRR 22fc22


436 PRINCIPLES OF REFRIGERATION

•all 3S888S SSSSS S8S8S 83883 5SSS8


S52
£ 333$$ 5S3S26 !855SS§S SSSSfeS SSSSS

NNHSO m as * oo to t*C*<Si-UO Nhneo


©«t<oseooo
lOU5«P«Ot» OSQ© »H ^ ggg§§ SS'SSS
I: £3-3 88888 liiii
39&SS3 83S533 SS8SSS SSSSS 83388
-s
~ 22228 88833 SSSSSSi 88833 3333.3
fi

— a 89S38 838^(9 S38SS SJSSSSS 53SS3


© go> <4< -<t< <4< * -*< *. -^i •* •«( ^i ^i -^i r|i •* -^i -«ji *•**(.' io loujioujxj

sl"

88S&8 SSfcgS SS58S13 «SSSfe S33J58


eo eo eo eo eo eoeoeoeoeo eoeoeoeoeo eo eo ** * * ^* »* •*
>*< Tt< -^t
i-H?H^HtH^* i-i i-H «-» ft iH rH »H t^ iH 1-H ^H
t-4 t-( ^H
f-H fHi-H i-H i-l

1c
a <OOSTt..-lO ©COt»CO»< iH •* 00 U5 U9 eoM>» >acO^<00>0

sssbs aods8o>-i HHO


>t>.ogo>
©00©>-l
o-*«co'*<
3 HHrtNN
I i-HQpt»OJC
823; )NOO
£3 5* t— lO '
1

HI
_l I is ooo oo ;s;
•IP
a i ooooo ©oooo ©ooo© ©ooo© ©ooo©
s Oh
2 s
10.10a ^ OS ^* ^ OS
5
OS
OONCOoS os co oo eo i- SS CO 00 CO
rt
^H
_H_ NC<l
»-H i-i »-H i-4
n co eo * •*
tlfl^Hilil
*ious»«o
il
il il ^H *1
t»r»ooooos
t^ 00 00 OS
!*•
i-< il »1 —t t-t ^ W HH
»S N C* fl
M
I
is KtNiSeooo eoes3>-it» coostocoos ^>?:2§[t- H29^oo6
*H »-4 i—I «-H
1* i-H

38888
ooooo ooooo ooooo ooooo ooooo
8SSSS
OS n CO »6 .V
iSSS 3§
'J? § us us to to to to 5 ooo >*?8Si
;Os« 3 OS
ooooo ooooo ooooo ooooo OOHHH
* SSSfcSS
SqOcon 35 338e 38833 2g§83
Ohhh

8S3SS 3SS588S 8S8SS 38.588 SoSSS


TABLES 437

§8283 SSggg S2S88 SS&SS gSS&S S83S8 feSSSS


SSSgg 3SS22 28888 g|g§S 23S8B fcSfcg'S
SS8SS5S

lOWTtjoeeo h-N»-<io o^oseooo 0»t~»-i5O© ioo>-*ooe<s noohio ©^©cooo

83SS2 53S2S 83SS8S S3S-3S 8S$f28 83885 3882S;


88888 888SS; SS888 88888 88888 355338 S888S

£8838 82383 SSSfcSS 885S8 S388S; 28888


JoHSfcS!
22222 22222 22222 22222 25555 55555 22222

"8
3
,C
<00000>CO eo^-^iOO O>"*C0©00 iO^t«-*^fe0 OO»OQ0 OS»HO«» 0>CO^*05^
C4 ^f tQ t^ O CO CD O^A CO A iA C4 00 CO tH CO C9 ^1 C9 «S^* a m
ifS tto mtt **k *** »*J kJ h~_* «
J 88S8§ sills SIslS iliil §8gSg Silig islii
1
A ICOO MJHOO eoootoejus 00
:S°
l^t» OrtMOOJ K32S * ^**Q5©^
^^SKS!? gnttiri 4t>3?N t^coOOOt^
OtHooSio r^o»e«t^M
SmSSr2 1

CO >00 OOOOO OOOOO leoSo


>SS S&fcfcfe SSSSS ©©.h-hm S3238
rtrt^JnS 888SJ5*
SSSSS
00000 00000 0000© 00000 00000 00000 00000
2

OU90C4M 0)O>©CO~h NOOnptO iO


SPfc«S hohho 3SF-8S is«snm
9^«ne ^^i**©©
gr'iJ'JS
feteSSS
ss?9!*h?«p
SSfifSl!
£&£;8S2
e?88S3
S2f8£8?
S3<2S5!
ss N 22 Q^*&~*oo
SSSSS
SSS88
SfeffiS'* «SH5?SS° hSSsS

©oooo oo'oo'o 00 000 ©'©'©'©© ©'©'©©'© o'o'ooo odeido


§0j5Ht;
^^SSS S
Et^K&QQW
® N !2S2!2
— S23S K^wQ'* 2322B98S
©sooottoo
-hwosusm
*o5^*oa5
*-«.*e9«©
MootHSs <ocae9w*<
«sok.3Z
-h-h-hRs* e*eoe«5«-* •*u5u55S5 coKKaSSj SSSSS
,
S8eS*5< S8BSS
HHHHH -H-Hrtrt»H rtrt-H^-H rt^rt^^ -H-HC^eiiei NNNNN IN IN IN WIN

2!3!S£2?£
38cSs2 SSSSSiSS!
S82.2S8SE
SSSSPS!
SSPx'is; 2S3!SHS
3Ss?js ©oooooo©
^-h-hESm ©-<»«©
S-*©®oo ojt-est-eo
55o>©mcS
^©t^^*'-<
e!|»eo-*>o
©CO^TCSO
>o35So6*
OSffilQ^
SSSS5* §858°i H«mK.AM
^*^^4^tv
5SSS* ACfiNISa 8558© eA/feShnriX]
285Sn
nnnnn eicJNeiN eieoroeort »»»»» co * -« ^ uj uiuiiotoio
rji Tji Tji -* <*

|§§§! 2S222 22522 8S88S 88&8S 83883 88SS88


I i

438 PRINCIPLES OF REFRIGERATION

a Is ~ gSSSS g£§3§ 2SS33 cS 3© §& ggSSSI , ,

KSS28 gE
S22SS
SNNMN S55SSJ5S
C* CN NNN«M 288S3
S538SS CO <N CNl CN CO CO CO CO CO CO CO •* •* <S g
"5? 'Sb
a S q
(|t.H«0 O -* 00 CO i(3 !>-IN«0-HiO O •# O CO 00 FINHlOO gj

e* N eo co ^
S n -* us us to
n <n •*< to 1^ 1- 06 06 o os ©© -h—icnmco
w 5 •=
g
<M
H K cn
-H ~4 W in
csl
—I *H ^H ^*
esi
»-<
<N CM. <N CM <N
»H i-H -H — (N N. CM CO CO
-H rH *-* — »M
CO CO CO CO
~H ~H »-H .^ i-H Vg
»H »H
i-H l-H
t-H
^HfHFH»MrH
*-H t-H^Hf-)r-l^H
r-<
I

r-*»H »^
<
<-< ^* r-< W
~H —< fH <»
ff
,S

a
5
fe3
•h uj as eo t-
toooocoio
KUjoeor*
ooowflir- onioso nsojoo
noq^oo oii<t-o-H
co t^
Tftoosi-ico
—< m os KGo
jg

m #> *? *&
eococococo
-4* tiJ'us«9 «> «o
co co co co co
ooooiD
cococoeoco
(^i-^t-t-oo
co co co co co
060606 as os
cococococo £V
.b

60 S+9.1S
OS ^< © CO CO © 00 W >C O N-COSSC©'* CO CO lO 00 CO
|CAHSi< St-OS-HCO O CO CO OS CO
So g^> 0000030)0 OSOSCSO© ©»©-<— -H-HCNNN
*>H i-H fH i—( «— rHt— »-t< CN CN CM N CM N CM. CO. CM CN CM CN1

•a 22228 838S3 £8333 9S3S3 S88g£


in iij ui us us tainitiisitii uj us ui 10 »o isioujioo 10 w 10 >o us
5^
•1 h-0^>0.0 3>N<OSO> ICISSS ©OSO>t»CO OS CO US t-~ CM.

pNieoiiiJ coeoujoos cocoon© fqcoiotp^


rijioi'ON'^
5oscoK-h
OOhh« i553coi^<n
NNMM* nnsni* *ogno Q_'"i ei.™.
^usiocOC© 1^36 00OS_©_ t<*«5» e, >,

"S
o rt-r-T-H-H" -i^r-T^-H" T-r 1-r-H'-Hrt' ^-h-h'thn' c^csr<Ne*r n~

> HONlOO r-OOO-Ht-


MSMNCC
IOOQNN QNNHOO
00-*©^CO ©t~iftCN©
— OS US O
$^<0®ES OtOl^OJ-- <
t-~

a 3 to
S5SSK KoSaosO
SSSStn UiS^h-hS S-Hgjcogi -*u5ot^ee ojonSa
o « e» e* ««««« cococococo e*.

CM ooooo o'oooo ooooo ooooo ooooo


SS23S3 SSSSS 38353 £8333 33883
SSiO^c© «(OOVi OBSasoBOO
o
§ (OCOte^CO
SrirtNS
QOtSONO
^*Srt5 *®NN01 Olt^t^'*'*
O0Nt-eMt~ MQtiJa
° ™< 2^'S
3

SS'Ssl'^
* 9. -HcoioKos
OOOpOOOpOO
SRiooo*
03OS 03 OS O
NintjON !5»g>3S
OOO'H^H ?H fH N C* Cfl
SJ3i2SS3
C<l CO CO so -^

55555 SSSoo ooooo ooooo ooooo


(2
ooooo o'oooo ooooo ooooo ooooo
rfiS-ysmm OOCO^COCO ~*0000 OOhhN
r>.eNO»oooo SF22SS
Qcooso«o coosairto ^!25*te co^Ot>-00»-t l-g
oScSO^S SS53S F-oBosS-h nc5-)<"50 t^ooosoeN
2 S CNcicococo cococococo eoeoco-*-* ****!* r)<-*-*i«io

Ml SXS4INH
k.ak.^^h
H^NMIO 00-HlOQCO
oor^eooso u5u3o6Sr~ SSSSS
cousooopp Sfc^-.^
oooos^hqo
B !^3oo^Jh
SoSc<55
S5§os»
cooooS-icO
SSs^*!?
BSr-ios-irt
rfcfit-^Q
u t~: m.'1"* •t22-
c**"1 c<5
,
S5

^
loococoo cocot^-^i^" t^r»^oooo oooooooso c^c^ooo

§595^! §«^^5 3383S SSfeSS SSS8S


e5°

h
TABLES 439

TABLE 7-1. Temperature and Enthalpy of Discharge Vapor after


Isentropic Compression

Condensing Temperature

Saturated 80° 90° 100°


Suction
Temperature / h t h t h

-40° 111.0° 91.6 121.0° 92.3 132.0° 93.9


-30° 105.0° 90.5 116.0° 92.0 127.5° 93.2
-20° 102.0° 90.2 112.5° 91.4 124.0° 92.6
-10° 97.5° 89.5 108.5° 90.7 119.9° 91.9
0° 95.0° 89.2 106.0° 90.3 117.0° 91.5
10° 92.0° 88.7 103.5° 89.9 114.0° 90.9
20° 90.0° 88.4 102.0° 89.6 112.0° 90.6
30° 88.0° 88.1 99.0° 89.1 110.8° 90.4
40° 86.0° 87.7 97.0° 88.8 109.5° 90.2
50° 84.0° 87.4 95.5° 88.6 107.0° 89.8

Condensing Temperature

Saturated 110° 120° 130°


Suction
Temperature / h t h / h

-40° 143.0° 95.1 155.0° 96.3 166.5° 97.3


-30° 138.0° 94.3 150.5° 95.5 161.5° 96.6
-20° 135.5° 93.7 147.0° 94.8 157.5° 95.8
-10° 131.6° 93.1 143.0° 94.2 154.0° 95.2
0° 128.5° 92.6 141.0° 93.7 152.0° 94.8
10° 126.5° 92.1 137.5° 93.2 148.5° 94.3
20° 124.0° 91.7 136.0° 92.8 147.2° 93.9
30° 122.0° 91.4 133.5° 92.5 146.0° 93.6
40° 120.0° 91.1 132.5° 92.2 143.5° 93.2
50° 118.0° 90.8 131.0° 92.0 142.0° 92.9
440 PRINCIPLES OF REFRIGERATION

TABLE 10-1. Heat Transmission Coefficients (U) for Cold Storage Rooms
Btu per hour per square foot per degree F difference between air on the two sides.
Wind velocity 15 mph.

t
Vapor seal on warm side Thickness of Insulation, V Inches
Wall Thickness
-Corkboard X Inches

Concrete block 8 0.12 0.085 0.066 0.054 0.046 0.040 0.035


Concrete block 12 0.12 0.083 0.065 0.053 0.045 0.039 0.035

Cinder block 8 0.11 0.081 0.064 0.052 0.045 0.039 0.034


Cinder block 12 0.11 0.079 0.063 0.052 0.044 0.039 0.034

(Vapor seal on warm side

Corkboard
Common brick 8 0.11 0.081 0.064 0.053 0.045 0.039 0,034

Common brick 12 0.10 0.076 0.061 0.050 0.043 0.038 0.034

, Vapor seal on warm side


Corkboard

Clay tile 4 0.12 0.085 0.066 0.054 0.046 0.040 0.035


Clay tile 6 0.11 0.081 0.064 0.053 0.045 0.039 0.035
Clay tile 8 0.11 0.081 0.064 0.052 0.045 0.039 0.034

,Vapor seal on warm side

Corkboard

.. . J:'<E-*!?I
Concrete 6 0.13 0.089 0.069 0.056 0.047 0.041 0.036
Concrete 8 0.12 0.087 0.068 0.055 0.047 0.040 0.036
:Sv£.*4*l^v.fl Concrete 10 0.12 0.086 0.067 0.055 0.046 0.040 0.035
Concrete 12 0.12 0.085 0.066 0.054 0.046 0.040 0.035

From Carrier Design Data. Reproduced by permission of Carrier Corporation.


TABLES 441

TABLE 10-2. Heat Transmission Coefficients (U) for Cold Storage Rooms
Btu per hour per square foot per degree F difference between the air on the two sides.
Outside wind velocity 15 mph.

Insulating
Thickness of Insulation (Inches)
Type of Construction
Material
3% 5%

Granulated
0.079 0.055
cork

Rock or
0.072 0.050
palco wool

Sawdust 0.097 0.069

1* board on
Corkboard ----- 0.11 0.084 0.067 0.055 0.047
both sides of studs*

Glass or rock
0.084 0.055 0.100 0.077 0.062 0.052
wool fill

Sheet steel on
"
both sides of studs

Thickness of Insulation
Insulating (Inches)
Material
8 10 12
Insulation

Granulated .040 0.033 0.027


cork

Palco or
0.036 0.029 0.025
rock wool

1- board _ b
both sides-2" x 4" studs-16" £ Sawdust 0.051 0.042 0.035

NOTES:
'Coefficients corrected for2x4or2x6 studs, on 16 in. centers.
b
Coefficients corrected for 2 x 4 studs
* Actual thickness *% in.

From Carrier Design Data. Reproduced by permission of Carrier Corporation.


TABLE 10-3. Heat Transmission Coefficients (U) for Cold Storage Rooms
Btu per hour per square foot per degree F difference between air on the two sides.

Wind velocity 15 mph.

Wall, Floor or
Thickness of Insulation, Y Inches
Ceiling Thickness
X (Inches)
Self-supporting partition*

I
Vapor seal on warm side

• Corkboard
Cork partition 0.13 0.089 0.069 0.056 0.047 0.041 0.036

Cement plaster/
on both sides

Floor' Corkboard*
Slab 2 0.046 0.035
0.12 0.087 0.067 0.055 0.040
Finish 2
Slab 5
Finish concrete 0.12 0.084 0.066 0.054 0.046 0.040 0.035
Insulation Finish 3
Slab 6
0.11 0.083 0.065 0.054 0.045 0.039 0.035
Finish 4
a
Foamglas
Floors Slab 2
0.15 0.11 0.087 0.071 0.060 0.053 0.046
Finish 2
'
Vapor seal on warm side
Slab 5
0.15 0.11 0.084 0.070 0.059 0.052 0.046
Finish 3
Slab 6 0.14 0.10 0.083 0.069 0.059 0.051 0.045
Finish 4
Ceiling*
, Vapor seal on warm side

'Concrete slab
Concrete 4 0.12 0.089 0.069 0.056 0.048 0.042 0.036
Concrete 8 0.12 0.086 0.067 0.055 0.047 0.041 0.036

sleeper Corkboard

Ceiling*
>Ceiling joists or wall studs

_ /Sheathing
\
Wood
(actual)
% 0.11 0.082 0.064 0.053 0.045 0.039 0.035

iPaper and vapor ^-Corkboard


seal on warm side

Ceiling*

0.13 0.092 0.072 0.059 0.050 0.043 0.038

,Tee iron
construction
Corkboard

These values may also be used for floors on ground.


* Surface conductance for soil air, 1.65, used on both sides

From Carrier Design Data.. Reproduced by permission of Carrier Corporation.


442
TABLES 443

TABLE 10-4. Thermal Conductivity of Materials Used in Cold Storage Rooms


Practical*
Thermal Thermal Thermal
Conductivity Conductance Conductivity

*
(*) (Q (*)
(Btu per hour (Btu per hour (Btu per hour
per sq ft per sq ft Authority per sq ft

per°F per °F for per °F


per inch per test 2nd and 3rd per inch
Material thickness) thickness) Columns thickness)

Brick, common 5.0 — 1 —


Cement plaster 8.0 — 1 —
Concrete 12.0 — 1 —
Cinder aggregate block 8* — 0.60 1 —
Cinder aggregate block 12* — 0.53 1 —
Gravel aggregate block 8* — 1.0 1 —
Gravel aggregate block 12* — 0.80 1 —
Corkboard 0.28 — 2 0.30
Cork, granulated coarse 0.31 — 2 0.34
Foamglas 0.40 — 4 0.40
Glass wool, density 1 .5 lb per cu ft 0.27 — 1 0.30
Mineral wool board 0.33 — 3 0.36
Redwood bark, palco wool 0.26 — 1 0.29
Rock wool, density 10.0 lb per cu ft 0.27 — 1 0.30
Sawdust, various woods 0.41 — 2 0.45
Tile, hollow clay 4* — 1.0 1 —
Tile, hollow clay 6* — 0.64 1 —
Tile, hollow clay 8* — 0.60 1 —
Wood, yellow pine, or fir 0.80 — 1 —
Authorities: (X) ASHVE
Guide 1945. {2) AS RE
Data Book, Vol. 1—1943. (I) ASRE Data Book,
Vol. 2 —
1942. (4) Pittsburgh Corning Corporation.
* These conductivities were used for insulating materials in calculation of heat transmission
coefficients. %
Most of these values have been increased 10 above laboratory test values to allow for
the effect of moisture gain in the insulating material and for imperfect workmanship. This also
assumes adequate vapor sealing. When no vapor sealing is applied or where the workmanship is
poor the value of the insulation is largely destroyed. It is extremely difficult to get a good vapor seal
with loose fill type insulation.
Foamglas. If a combination of corkboard and Foamglas is used, 1 in. of Foamglas is equivalent
to | in. of corkboard.
Mineral Wool Board. For estimating purposes use heat transmission coefficients for corkboard
increased by 15%.
From Carrier Design Data. Reproduced by permission of Carrier Corporation.
TABLE 10-5 A. Surface Conductance (f)

for Building Structures


TABLE 10-5. V Factors for Glass
Surface Conductance
Number of (Btu per hour per
Panes Btu/hr/sq ft/° F square foot per ° F)

1 1.13 Surface Exposure Winter Summer


2 0.46
3 0.29 Ceilings Inside 1.65 1.20
4 0.21 Roofs Outside 6.00* 4.00f
Walls Inside 1.65 1.65
From ASRE Data Book, Design Volume, Walls Outside 6.00* 4.00t
1949 Edition, by permission of the American
Society of Heating, Refrigerating, and Air- * Average wind velocity 15 mph.
Conditioning Engineers. t Average wind velocity 8 mph.
From Carrier Design Data. Reproduced
by permission of Carrier Corporation.

TABLE 10-6. Refrigeration Design Ambient Temperature Guide*


Average Maximum Average Maximum
Ambient Ambient Ambient Ambient
Location Temp. Temp. Location Temp. Temp.

Alabama Delaware
Birmingham 88 99 Dover 87 96
Mobile 88 97 Milford 87 98
Arizona Wilmington 87 94
Flagstaff 75 90
113
Columbia
District of
Phoenix 100
Washington 89 98
Tucson 84 98
Arkansas Florida
Fort Smith 91 103
Jacksonville 88 96
Little Rock 90 100
Miami 88 90
California Orlando 88 97
Bakersfield 96 114 Tallahassee 88 100
Fresno 94 111 Tampa 88 95
Los Angeles 83 94
Oakland 75 89 Georgia
Sacramento 90 108 Atlanta 87 95
San Diego 75 80 Savannah 89 99
San Francisco 75 83
Idaho
Colorado
Boise 89 105
Colorado Springs 83 94
Pocatello 83 100
Denver 83 98
Grand Junction 88 102 Illinois
Pueblo 83 100 Cairo 89 101
Connecticut Chicago 87 98
Hartford 83 94 Peoria 88 100
New Haven 83 95 Quincy 90 103
New London 83 93 Rockford -87 101
Norwalk 83 96 Springfield 90 102
TABLE 10-6 (Continued)

Average Maximum Average Maximum


Ambient Ambient Ambient Ambient
Location Temp. Temp. Location Temp. Temp.
Indiana Minnesota
Evansville 90 100 Duluth 79 92
Fort Wayne 87 100 Minneapolis 90 102
Indianapolis 89 99 St. Cloud 88 101
South Bend 87 101
Mississippi
Terre Haute 90 100
Jackson 90 99
Iowa
Vicksburg 9P 96
Burlington 90 101
Davenport 90 100 Missouri
Des Moines 90 102 Hannibal 90 102
Dubuque 90 99 Kansas City 92 103
Keokuk 90 101 St. Joseph 92 103
Mason City 86 97 St. Louis 92 103
Sioux City 90 102 Springfield 88 98
Kansas Montana
Concordia 93 108 Billings 85 104
Dodge City 92 106 Butte 75 96
Hutchinson 92 108 Havre 82 99
Salina 95 111
Helena 82 102
Topeka 92 105
Wichita 91 104 Nebraska
Kentucky Lincoln 94 106
Lexington 86 North Platte 89 103
98
Louisville 88 99
Omaha 92 104

Louisiana Nevada
Baton Rouge 88 98 Reno 84 101
New Orleans 89 98 Tonopah 84 96
Shreveport 92 102
New Hampshire
Maine Concord 81 92
Eastport 70 81
Portland 81 93
New Jersey
Atlantic City 83 92
Maryland
Paterson 85 95
Baltimore 89 99
Trenton 85 96
Cumberland 87 102
Massachusetts New Mexico
Boston 84 94 Albuquerque 83 99
Fall River 81 90 Santa Fe 81 90
Lawrence 81 94 New York
Worcester 81 92 Albany 83 96
Michigan Binghamton 83 94
Alpena 82 95 Buffalo 80 89
Detroit 86 99 Elmira 83 97
Grand Rapids 86 98 New York 85 93
Jackson 86 99 Poughkeepsie 83 95
Lansing 86 96 Rochester 83 95
Marquette 81 96 Syracuse 83 96
Saginaw 88 101 Watertown 83 93
•445
446 PRINCIPLES OF REFRIGERATION

TABLE 10-6 (Continued)

Average Maximum Average Maximum


Ambient Ambient Ambient Ambient
Location Temp. Temp. Location Temp. Temp,

North Carolina Tennessee


Asheville 81 93 Chattanooga 87 98
Charlotte 86 98 Knoxville 87 98
Raleigh 86 98 Memphis 89 99
Wilmington 86 95 Nashville 87 98
Winston-Salem 86 97
Texas
North Dakota Dallas 92 102
Bismarck 87 103 El Paso 92 102
Devils Lake 84 100 Fort Worth 92 104
Houston 92 99
Ohio
San Antonio 92 102
Akron 86 98
Canton 86 97 Utah
Cincinnati 88 100 Modena 80 97
Cleveland 83 95 Salt Lake City 88 101
Columbus 88 98
Vermont
Dayton 88 99
Burlington 80 91
Toledo 87 99
Virginia
Youngstown 86 97
Lynchburg 87 99
Oklahoma Norfolk 87 95
Oklahoma City 92 104
Richmond 87 98
Tulsa 92 105
Washington
Oregon Olympia 75 90
Portland 81 95
Seattle 75 86
Pennsylvania Spokane 75 102
Altoona 82 96 Walla Walla 87 105
Erie 83 92
West Virginia
Harrisburg 85 97
Charleston 87 102
Philadelphia 87 97 Clarksburg 84 97
Pittsburgh 85 96 87
Huntington 100
Scranton 82 95 86 98
Parkersburg
Rhode Island Wheeling 86 101
Providence 83 94
Wisconsin
South Carolina Green Bay 85 97
Charleston 88 98 La Crosse 87 99
Columbia 88 99 Madison 87 96
Milwaukee 87 99
South Dakota
Huron 93 107 Wyoming
Pierre 94 110 Cheyenne 79 94
Rapid City 87 103 Lander 80 98
Sioux Falls 88. 102 Sheridan 86 102

* Do not use these temperatures for air conditioning design.


From ASRE Data Book, Design Volume, 1949 Edition, by permission of the American Society
of Heating, Refrigerating, and Air-Conditioning Engineers.
TABLES 447

TABLE 10-6 A. Design Ground Temperatures


Ground Ground
Location Temperature Location Temperature

Alabama Idaho
Birmingham 70° F Boise 60°
Mobile 75 Pocatello 60
Arizona Illinois 60
Flagstaff 60 Cairo
Phoenix 80 Chicago 60
Tucson 80 Peoria 60
Arkansas Quincy 60
Fort Smith 70 Rockford 60
Little Rock 70 Springfield 60

California Indiana
Bakersfield Evansville 65
75
Fresno 80
Fort Wayne 60
Los Angeles Indianapolis 60
75
Oakland South Bend 60
65
Sacramento Terre Haute 65
80
San Diego 65 Iowa
San Francisco 65 Burlington 60
Colorado Davenport 60
Colorado Springs 60
Des Moines 60
Denver 60
Bubuque 60
Grand Junction 60 Keokuk 60
Pueblo 55
Mason City 60
Sioux City 60
Connecticut
Hartford 65
Kansas
New Haven 65
Concordia 60
New London 65
Dodge City 60
Norwalk Hutchinson 60
65
Salina 60
Delaware Topeka 60
Dover 65 Wichita 60
Milford 65
Wilmington 65
Kentucky
Lexington 65
Columbia
District of Louisville 65
Washington 65
Louisiana
Florida Baton Rouge 75
Jacksonville 80 New Orleans 75
Miami 80 Shreveport 70
Orlando 80
Maine
Tallahassee 80
Eastport 60
Tampa 80
Portland 60
Georgia Maryland
Atlanta 70 Baltimore 65
Savannah 75 Cumberland 65
448 PRINCIPLES OF REFRIGERATION
TABLE I0-6A (Continued)
Ground Ground
Location Temperature Location Temperature

Massachusetts New Mexico


Boston 65° F Albuquerque 70° F
Fall River 60 Santa Fe 65
Lawrence 60 New York
Worcester 60 Albany 60
Michigan Binghamton 60
Alpena 60 Buffalo 65
Detroit 60 Elmira 60
Grand Rapids 60 New York 65
Jackson 60 Poughkeepsie 60
Lansing 60 Rochester 60
Marquette 60 Syracuse 60
Saginaw 60 Watertown 60

Minnesota North Carolina


Duluth 50 Asheville 70
Minneapolis 55 Charlotte 70
St. Cloud 55 Raleigh 70
Wilmington 75
Mississippi
Winston-Salem 75
Jackson 75
Vicksburg 75 North Dakota
Bismarck 50
Missouri
Devils Lake 50
Hannibal 60
Kansas City 60 Ohio
St. Joseph 60 Akron 65
St. Louis 60 Canton 65
Springfield 60 Cincinnati 65
Cleveland 65
Montana
Columbus 60
Billings 55
Dayton 65
Butte 55
Toledo 60
Havre 50
Youngstown 60
Helena 55
Oklahoma
Nebraska Oklahoma City 65
Lincoln 60 Tulsa 65
North Platte 55
Omaha 60 Oregon
Portland 70
Nevada
Pennsylvania
Reno 65
Altoona 65
Tonopah 70
Erie 65
New Hampshire Harrisburg 70
Qoncord 55 Philadelphia 70
New Jersey Pittsburgh 65
Scranton 65
Atlantic City 70
Paterson 70 Rhode Island
Trenton 70 Providence 65
TABLES 449

TABLE I0-6A (Continued)


Ground Ground
Location Temperature Location Temperature

South Carolina Virginia


Charleston 75° F Lynchburg 75'
Columbia 75 Norfork 75
South Dakota Richmond 70
Huron 55 Washington
Pierre 55 Olympia 60
Rapid City 55 Seattle 75
Sioux Falls 55 Spokane 60
Tennessee Walla Walla 60
Chattanooga 70 West Virginia
Knoxville 70 Charleston 65
Memphis 70 Clarksburg 65
Nashville 70 Huntington 65
Texas Parkersburg 65
Dallas 70 Wheeling 65
El Paso 70 Wisconsin
Fort Worth 70 Green Bay 55
Houston 75 La Crosse 55
San Antonio 75 Madison 55
Utah Milwaukee 55
Modena 60
Wyoming
Salt Lake City 60 Cheyenne 55
Vermont Lander 55
Burlington 60 Sheridan 55
:

450 PRINCIPLES OF REFRIGERATION

TABLE 10-7. Allowance for Solar


Radiation
(Degrees Fahrenheit to be added to the normal
temperature difference for heat leakage calcu-

lations to compensate for sun effect not to be
used for air-conditioning design)

East South West Flat


Type of Surface Wall Wall Wall Roof

Dark-colored sur-
faces such as
Slate roofing
Tar roofing 8 5 8 20
Black paints
Medium-colored sui
faces, such as:
Unpainted wood
Brick
Red tile 6 4 6 15
Dark cement
Red, gray, or
green paint
Light-colored sur-
faces, such as:
White stone
Light-colored
cement 4 2 4 9
White paint

From ASRE Data Book, Design Volume,


1957-1958 Edition, by permission of the Ameri-
can-Society of Heating, Refrigerating, and Air-
Coriditioning Engineers.
TABLES 451

TABLE 1 0-8 A. Btu per Cubic Foot of Air Removed in Cooling to


Storage Conditions above 30°
Inlet Air Temperature, ° F
Storage 85 90 95 100

Temp., Inter. Air Relative Humidity, %


°F 50 60 70 50 60 70 50 60 50 60

65 0.65 0.85 1.12 0.93 1.17 1.44 1.24 1.54 1.58 1.95
60 0.85 1.03 1.26 1.13 1.37 1.64 1.44 1.74 1.78 2.15
55 1.12 1.34 1.57 1.41 1.66 1.93 1.72 2.01 2.06 2.44
50 1.32 1.54 1.78 1.62 1.87 2.15 1.93 2.22 2.28 2.65
45 1.50 1.73 1.97 1.80 2.06 2.34 2.12 .2.42 2.47 2.85
40 1.69 1.92 2.16 2.00 2.26 2.54 2.31 2.62 2.67 3.06
35 1.86 2.09 2.34 2.17 2.43 2.72 2.49 2.79 2.85 3.24
30 2.00 2.24 2.49 2.26 2.53 2.82 2.64 2.94 2.95 3.35

Reprinted from Refrigeration Engineering Data Book by courtesy of American Society of


Refrigerating Engineers.

TABLE I0-8B. Btu per Cubic Foot Removed in Cooling to


Storage Conditions below 30°
Inlet Air Temperature, ° F
Storage 40 50 80 90 100

Temp., Inter.Air Relative Humidity, %


°F 70 80 70 80 50 60 50 60 50 60

30 0.24 0.29 0.58 0.66 1.69 1.87 2.26 2.53 2.95 3.35
25. 0.41 0.45 0.75 0.83 1.86 2.05 2.44 2.71 3.14 3.54
20 0.56 0.61 0.91 0.99 2.04 2.22 2.62 2.90 3.33 3.73
15 0.71 0.75 1.06 1.14 2.20 2.39 2.80 3.07 3.51 3.92
10 0.85 0.89 1.19 1.27 2.38 2.52 2.93 3.20 3.64 4.04
5 0.98 1.03 1.34 1.42 2.51 2.71 3.12 3.40 3.84 4.27
1.12 1.17 1.48 1.56 2.68 2.86 3.28 3.56 4.01 4.43
-5 1.23 1.28 1.59 1.67 2.79 2.98 3.41 3.69 4.15 4.57
-10 1.35 1.41 1.73 1.81 2.93 3.13 3.56 3.85 4.31 4.74
-15 1.50 1.53 1.85 1.93 3.05 3.25 3.67 3.96 4.42 4.86
-20 1.63 1.68 2.01 2.09 3.24 3.44 3.88 4.18 4.66 5.10
-25 1.77 1.80 2.12 2.21 3.38 3.56 4.00 4.30 4.78 5.21
-30 1.90 1.95 2.29 2.38 3.55 3.76 4.21 4.51 5.00 5.44

Reprinted from Refrigeration Engineering Data Book by courtesy of American Society of


Refrigerating Engineers.
452 PRINCIPLES OF REFRIGERATION

TABLE 1 0-9 A. Average Air Changes per 24 Hours for Storage Rooms above 32° F
due to Door Opening and Infiltration
(Does not apply to rooms using ventilating ducts or grilles)

Air Air Air Air


Volume Changes Volume Changes Volume Changes Volume Changes
cuft per 24 hr cuft per 24 hr cuft per 24 hr cuft per 24 hr

250 38.0 1,000 17.5 6,000 6.5 30,000 2.7


300 34.5 1,500 14.0 8,000 5.5 40,000 2.3
400 29.5 2,000 12.0 10,000 4.9 50,000 2.0
500 26.0 3,000 9.5 15,000 3.9 75,000 1.6
600 23.0 4,000 8.2 20,000 3.5 100,000 1.4
800 20.0 5,000 7.2 25,000 3,0

Note: For storage room with anterooms, reduce air changes to 50% of values in table.
For heavy duty usage, add 50% to values given in table.
From ASRE Data Book, Design Volume, 1949 Edition, by permission of the American Society of
Heating,. Refrigerating, and Air-Conditioning Engineers.

TABLE I0-9B. Average Air Changes per 24 Hours for Storage RoOms below 32° F
due to Door Opening and Infiltration
(Does not apply to rooms using ventilating ducts or grilles)

Air Air Air Air


Volume Changes Volume Changes Volume Changes Volume Changes
cuft per 24 hr cuft per 24 hr cuft per 24 hr cuft per 24 hr

250 29.0 1,000 13.5 5,000 5.6 25,000 2.3


300 26.2 1,500 11.0 6,000 5.0 30,000 2.1
400 22.5 2,000 9.3 8,000 4.3 40,000 1.8
500 20.0 2,500 8.1 10,000 3.8 50,000 1.6
600 18.0 3,000 7.4 15,000 3.0 75,000 1.3
800 15.3 4,000 6.3 20,000 2.6 100,000 1.1

Note: For storage rooms with anterooms, reduce air changes to 50% of values in table.
(1)
For heavy duty usage, add 50% to values given in table.
(2) For locker plant rooms, double the above table values.
From ASRE Data Book, Design Volume, 1949 Edition, by permission of the American Society
of Heating, Refrigerating, and Air-Conditioning Engineers.
TABLES 453

TABLE 10-10. Design Data for Fruit Storage

DtSISN «OOM SPECIFIC


CONDITIONS at
Tamparatara •rod HEAT
Ralatlva CjratM Lataat
TYPE OF HamMhV par lb Mail- Product lataut Otu/lb/
HtUITS Haat Air
STORA6E Air at mum Tamp. Haat DagF
Rac- Rac- TTma Otu/lb at
Racom- Storaga D< F aa- Aftor
om- FurmH om-
maad- Ibla maad tibia
ad Raaga ad Raaga Condi-
Fartad
SUrt In
la.
Hr Fac-
tor 24 Hr
IElm
Natal)
fora Fraac
Fraai- lag
lag
Fiavoa
Ola/lb t dS"f
tloa
la

DagF DagF % % tion Ft./Mln

Applai Skart Sk 3S-40 07 0540 244 44 041 0.41 122 0« a.i •9


Loag lot 10-32 07b 0540 204 40 Mo, 0.2 to
CMII Start 49 OS 314 09 32 24 047 2441 IM
CMII ROM 03 20.4 04 4M
Aprlcoh 35 3M0 03 oms 254 44 0.12 040 122 « a.i W
n 31-32 03a 0M0 22J 7-14 Dayi 0.1 N
CMII Start • OS 314 10 33 a 047 2041 ia
CkillRahk 05 22J 0.3 Md
Avocados 4041 05b M
Skort 40k
Kk
05-10 314 44 0.11 041 IM 274 W
CUII Start 40
1741 05b
05
os-a 104 lODayl 0.3 a
314 00 11 22 047 2241 2H
CMUFhtt SI OS 214 04 Old
OOMPH 1 Mpaahg 70 12-70 K 10-05 104.7 HaaHag Sf-70* 24 0.10 101 75 2440 a
(SaaDac- CMII Start It 15 104.7 40 Si It II4F
0.1 no
CMII HMaai Si 10 to 10 Dan 14 Md
2044) Holdlag Oraoi H Si-W 12 •MS ilJ 14 a
Haldtagklpa St suo 07 00-10 504 14 a
Oorrlaa as SM0 05 1045 254 4.3 0.N 0.41 ia 04 2040 a
(C—orol) Long 32 31-33 05b •MS 22.3 1-10 Da.,
04 40
CMII Start m 05 314 OS 34 a 047 2041 IM
CMII Rah* 32 05 224 04 aid
Craa- Start M 3441 05 05-fO 2»4 54 0.11 047 122 a 84 a
Si Si-40 05b 05-*) 24.4 1-3 Ma 04 a
CMII Start 40 05 314 70 30 a 047 1041 ia
CMIIFtaha 3i OS 24.2 04 Wd
Dataa
fCn<|
Short 30k 3S-40 70c tS-75 204 0.10 045 a 10 -4 IM
20k 20-32 70c 10-71 IS.4 3-4 mo, 040 IM
Driad

ss 35-40 70c 70-75 204 0.10 047 041 41 a IM
32L 32-Si 70c 70-70 104 1-12 Ma 047 IM

Ua?
40 43-SO
34-M
70 t0-70 274 54 041 044 Mi a 144 a
(fcaa« 34 70 4S-7S 204 IS Dan 0.4 a
•ropac Skort a 30-40 05 0040 254 54 040 041 112 77 204 »
(AiimHcm 31 31-32 05b 0045 21.3 14 VA,
CkM Start
04 a
Saltan)
CMII Rat*
40
33
00
05
314 70 34 a 040 M4f
224 04 ad
Skort
[VlaHara
3S
10
3S-40
30-31 05b
05-10 254 54 045 041 112 71 244 w
05-10 204 1-iMa 04 a
CoMonlo) CMII Start 40 00 314 70 34 a 040
Chill Rata*
1441 2M
32 05 224 0.4 Md
wapafnHt Skart « 4MB a 05-10 324 2.0 0.11 041 ia a a4 a
31-34 05b OS-N 224 44 Wk 04 a
CMII Start 40 05 114 75 14 22 0.70 1141 2M
CMIIRaM 32 OS 22J 04 ad
laraamj Skart 0540 Ctb
05b
05-S0 044 14 0.11 041 ia a 20.1 a
. SS St-40 05-10 544 1-4 Mo 04 a
CMII Start at 00 SS4 75 P a 14 1041 2a
CkMHaJ* B 05 544 04 ad
Uomo Skort 4E 4540 05b
Mb
0540 374 44 Ml 041 ia « 214 OS
4S 40-40 0540 1*4 44 Wk 04 M
CkM Start SO 05 454 75 47 a 040 1441
CMII Rat*
IM
4S 01 374 04 Md
Ol««|i Skort 40 40-01 05 0040 314
12 SM4 05b 0540
44 0.11 044 ia 01 204 a
224 0-10 VA,
CMII Start
04 M
40 OS 114 75 n 22 040 1141 2M
CkM Hal* a 01 224 04 Wrf
454 PRINCIPLES OF REFRIGERATION

TABLE 10-10 (Continued)

DESION ROOM SPECIFIC


CONDITIONS Ert
HEAT Maal-
Prod
Temperature leletlve trrelns
Latant Ota/lb/ Latant Wa-
FRUITS HnmldHv par lb Maxl- Product Haat Freez- Alr
type of Air at mum Haat DaaF ter
ing Mo-
STORAGE Rec- Ik. Storage Dot F Tlma Rata Btu/lb I.. Altar
ot Con-
Point tion
om- om- Nnnb-
Permit- Recom- Hr Fac- 24 Hr Fusion tent
mend- lible m d tibia mended Period Hn- tor (Ei.iaa
tora Freez-
Btu/lb % DogF
Condi- Start lih Freex- ing
•d Range i3 Rang* Notef) Ing
tion Ft./Mln
DogF DogF %
Patches Short B K-40 osa •MS 2U 5.1 0.91 0.41 128 •0 29.2 to
Long 12 31-13 •n> 01-05 224 2-4 Wk, 0.1 to
Chid Start 40 05 11.0 05 M 24 o.« 2141 ISO
CbUIRnbh 32 OS 224 04 Md
Peon Short K-40 00a as-to 2*4 040 44 0.01 0.49 122 M 27-20 to
Long Ilk 20-11 90b 05-M 22i7 1-7 Mo, 0.1 to
Chill Start « K 11 a 70 M 24 17.01 ISO
Chill Rail* » K 22.1 0.3 Md
naeeppfoa Short 40 40-4S 01 OS-W 114 34 0.90 040 128 n ISO
Long Mpe 40 40-41 Kb 05-10 11.0 24 Wk 0.1 21.0 ISO
SO S0-M ttb OS-W 41.0 1-4 Wk 0.1 20.1 ISO
Chill Start
Chill Hnl*
45
n
5
IS
17.5
10.0
m 40 1 047 244f
0.1
w
ISOd
Ptamsaad Short IS 35-40 05 00-05 2U 44 040 0.40 lie 00 204 N
fmMt Uag 12 31-32 Kb 10-05 22.1 1-0 Wk, 0.1 00
CMa Start 40 M 20.1 10 14 20 047 204f 200
ChlUFtafah » 10 21.1 04 «0d
Qstoe«t Short K 3M0 05 10-05 2SJ 44 0.90 M0 122 OS 204 40
Loog n 31-12 05b U4S 224 2-1 Mo 04 to
CMII Start 40 05 11.0 M 12 24 047 244f ICO
ChlnFlnbh n •5 22.3 04 Md
From Carrier Design Data. Reproduced by permission of Carrier Corporation,
TABLES -455

TABLE 10-11. Design Data for Vegetable Storage

DESISN ROOM SPECIFIC


CONDITIONS Est
HEAT Mari-
Tamparatwa alatlvo Prod
Grains
Mail- Latant Ota/lb/ Lataat Wa.
VME- Product Fraaa- Alr
TYPf Of Humidity par lb
mom Tamp. Haat DagF Haat tor
tng
TAOUS STORA6E Rac- Roc- Air at
Storaga Doif Timo
Rata
Ofu/lb
of Con Point
Raeom- Fac- la- Aftar Fusion lion
om- Pormh- om- Pormls- Hr tout
% DagF
Parted 24 Hr te*. Frosa- Itu/lb
mand- slbla mond •Iblu fin- tor
Start (Ex. son Froas- t"g
ad Ranga ad Ranga Coadl- ish
Notof) Ing Ft./MIn
DagF DogF % % rlon

Asparagus Short 40 4*45 •3 05-K 32.0 4.0 0.11 041 IX M.0 214 X
n 32-31 Ka 05-K 23.7 JOOay. 0.5 to
Chill Start 40 K 31.0 40 M 24 0.M 13.01 IM
Chill Rahh 33 k 23.2 0.5 40d
loans. Skort 40 40-4S to OS-X 32.0 34 047 0.47 111 X.0 21.7 X
Oroaa Long 11 32-40 Ka 05-X 24.5 XDays 0.7 40
Chill Start 40 05 31.0 X X M 047 IS.0f 150
Chill Flalih » X 23.2 0.7 tOd

hM Short 40 40-45 » 05-M 32.0 IS Days 3.0 0.70 0.M X 41.5 ».4 X
Shallad
(Urn*) Long 33 32-40 Ma 05-X 24.4 XDayt 04 40
Uaahollad

Mi, Short 4) 40-45 X OS-W 32.0 2.0 040 040 121 M4 24.1 X
Taps Off Ua« 31 32-34 X K-X 254 1-3 Mo 0.1 M
•Mb, Short 40 40-45 00 K-K 32.0 3.0 0.M 0.41 121 M4 314 X
Toot On Loag 32 32-3* 10a 05-K 23.7 10-14 Day 0.4 40
ChHI Start 40 » 324 70 34 24 0.00 1741 IX
CMII Rahh n 10 23.7 0.4 40d
Broccoli Short 41 4045 10 K-K 324 44 0.X 0.X IX X4 21.2 X
32 32-X to K-K 23.7 7-10 Days 0.5 to
Chill Start 40 to 324 I44f IX
Chill Rahh 33 10 244 X 34 24 0.30 04 Kd
irasMf Short 40 40-45 X X-K 34.5 54 0.11 0.41 IM 14.5 31.0 X
Sprouts Loag 33 32-X Kb X-K 24.0 3-4 Wk 0.5 to
CMII Start 40 X 32.0 X 34 24 0.00 14.01 IX
Chill Rahh 33 X 244 0.5 Kd
Cabbago Short X 35-40 X K-K 20.2 7.0 0.13 0.47 132 114 31.2 H
32 32-M Kb K-K 25.0 3-4>Mo 0.5 to
CMII Start 43 n 32.0 X 34 24 040 I74f IX
Chill Hah* 32 to 23.7 04 aOd
Carrots. Short 4) 40-45 w K-K 32.0 2.0 0.13 0.45 124 X.0 X.4 X
Topi Off Loag 32 32-34 K K-K 254 4-5 Mo 0.3 to

Carrots, Short 40 40-45 X 05-K 32.0 4.0 0.14 0.45 124 X.0 314 40
Tope On Loag 32 32-34 Xb OS-K 23.7 10-14 Day 0.5 40
ChHI Start 40 X 324 X 34 24 040 17.01 IX
Chill Rahh 32 X 23.7 04 tOd
Ca.H- Short X 35-40 X K-K 244 44 0.X 0.44 in •24 W.I M
flowor Long 32 32-M Xa K-K 23.7 2-3 Wk 0.3 M
Chill Start 40 X 324 m 34 24 O.00 17.01 IX
CMII Rahh 32 X 23.7 0.3 tOd

Cahry p Short 31 35-40 X K-K 244 44 0.11 0.44 m 144 21.7 X


Long (Wattod) 32 31-32 Wa K-K 23.7 2-4 Mo 14 M
Cora Short X 35-40 X K-X 244 7.0 044 040 IK ns 20.1 X
(Croon) Loag 32 31-32 Ma K-X 23.7 44 Dap 04 40
CMII Start 43 X 314 X 34 24 040 I74f IX
Chill Rahh 32 X 22J 04 tOd
Cacons- Short a SO-K X K-K 45J 34 0.X 0.41 137 K.S X4 X
bon Loag 45 4540 X Ot-K 374 10-14 Dayl 04 X
CMII Start M a 41.7 m 52 24 14 I34f 2K
CMII Rahh N N 424 0J I50d

tadb. p Short X 31-40 X X-K M4 44 0.X 044 IM i».o 30.1 X


Loag (lead) X 32-M Xa K-X 244 2-3 Wk 14 X
456 PRINCIPLES OF REFRIGERATION

TABLE 10-11 (Continued)

DESIGN >OOM
Est Mail-.
S PMPITION? Prod
HEAT
Tamporatura Ralatlva Gralnt Sta/IW Latent Wa-
Mail- Product Lataat Fraai- Air
VEGE- TYPE OF Humidity par lb DaoF Haat tar
TAM.ES STORAGE Air at mum Haat of Con Ing Mo-
Rac- Rac- Staraga Do Tlma Rata •ta/lb •a- Attar Point tion
om- Pafmli- om- Parmts- Racom- Hr Fac- Fusion taut
Parted ntf- 24 Hr foro DaoF
mand- tibia mand •Iblt rnandad
Condl- Start lih
Rn- tar
IE*, taa Fran-
Fraai-
lag
Stu/lb % In
Room
ad Ranga ad Ranga NotoO lag R./MIn
Dag_F DagF % % ttoo

LoHuca p Start IS 35-40 •J •MS 24.1 74 0.10 0.44 134 81.0 31.2 10
Loo« 0«dl 35 32-34 10a 10-15 24.5 2-3 Wk 1.0 40

Mator* Short 45 45-50 OS 7545 37.5 34 0.11 0.44 115 »5.0 21.0 •J
Wator-
fjjfi?
Long M 34-40 OSa 75-5S 24.2 2-4 Wk 0.2 ISO

dm"
bah. Long 32 32-35 Day
lOVPW 05 7S-70 22.3 7-10 0J 0.11 0.47 128 814 21.0 10
Cklll Start 40 05 31.0 •0 34 24 0.10 I4.0T 250
Chill Half* n 05 22.3 .2 ISM
OntOMJ Short 10 5*40 75 70-75 4H0 24 0.11 •41 130 •14 30.1 ISO
Long 32 32-tt 75 70-75 11.0 44 Mo 0J ISO
Chill Start 40 75 27.5 70 34 24 0.00 I0.0f ISO
Chill flnlsh » 75 11.5 0J ISOd
Paranlpt Short X 35-40 IS 10-15 24.2 44 0.U 0.44 111 •3.0 2*.1 40
Long 32 32-34 Kb •0-tJ 254 2-4 Mo 04 40
Chill Start 40 W 32J 70 34 24 0.00 I74f 150
Chill Hnhh 32 10 23.7 0J Md
Pom Short 35 35-40 M 05-10 244 M 0J2 0.45 107 00.0 2».7 N
(•ma) Long a
«
32-34 Nb •5-10 23.7 1-2 Wk M N
CMII Start 05 31.0 H 34 20 fc47 1441 ISO
Chill Halih 33 05 23.2 0.5 lOd
Potatooi
Itatlng)
SOr SO-70 55 os-w 45.2 3.0 044 0.47 113 75J 2M ISO
Potatooi
(Sood Urn 31-St 05a 55-10 24.4 0.5 2».» ISO
Stock)

Saaorkravt Short 45 45-50 00 7540 35.3 3.0 0.12 0.52 121 010 24.0 ISO
l«IM Long 39 30-32 50c 7540 I1J 5 Mo 0.2 10
Spinach Short 3S 35-40 TSa 10-15 25.2 7.0 0.12 0.51 I2» 104 30.3 10
Long 32 32-U ISa 10-15 25.0 10-14 Dai > 0.5 40
Swoat Short SS 55-40 m 00-55 54.5 3.0 0J4 0.42 102 7>.0 254 ISO
Potato. Long ss 5S-40 (Sa 50-tS 54.5 4-4 Mo 0.4 It)
Tomatoas Short
(Groan) Long
SS
ss
SS-aO
•540
05 55-10 54.5
Wk
3.0 0.12 6.44 132 ist 30.4 N
55 OS-W 54.5 3-5 0.4 40
Mpaatng as
Chill Start 70
iI-70 55
*S
•5-10 70J 24 N
13J •0 52 34 1.0 I44f 15*
Chill Rnllh so 05 ma 0.4 ltd
(Woo) Long .41 4041 OSa •5-10 irs 7-10 Dar 3.0 10
Tarnlpa 3S 3S-40 K H-M 20.2 4.0 0.10 0.45 121 ors 30.S *)
Long 32 32-34 15a 15-M 25.0 4-6 Mo 40
Chill Start 40 15 34.5 70 34 14 •JO 1741 IS«
Chill Rnllh 32 •1$ 254 0.5 ltd
Short 40 40-4S •5b ts-w 31.0
•ablat 5.0 o.to 0.45 130 104 304 It
fWattad. Long
Mraad)
3S 31-40 •7b •5-10 24.0 2-4 Mo IJ It
Chill Start 10 W 410 •0 35 10 0.70 2341 ISO
CMII Rnllh K •J 24.0 1.2 ltd

From Carrier Design Data. Reproduced by permission of Carrier Corporation.


TABLES 457

TABLE 10-12. Design Data for Meat Storage


DESIGN ROOM
CONDITIONS Eit
SPECIFIC
Tempera'tere Relative Grains Prod
MEATS TYPE OF Humidity par lb Mail- Product Latant tu/lb/ Latent Wa-
Heat Freez- Air
STORAGE Ree- Rec- Air at mum Tamp.
Rata
Haat DaoF tar
ing Mo-
Recom- Staragi «f lima Itu/lb af Con-
om- Permit- om- Permlt- Fac- la- After Fution Point tion
mend- tibia mend tibia Farted Hr 24 Hr tent
tor tere Fram-
Otu/lb % DegF In
ad Raage ad Range Caadl- Start (E».iao Freez-
hi'
Notaf)
ing Room
DagF DagF % % tlan ing Ft./Mtn

Start 55 JO-M 55 55-it 41.7 IS Day. 2.5 0.50 0.30 1 20


rlerdenlng 2(t 20-30 35 70-00 14.4
1.2
Slicing Raom 50 5045 40 3S-40 21.3
to

Cerabfaed CMII Start 38 05b 20.0 100 44 24 0.5*


~
18.01 0.75 0.. J It 72 31.3 250
CMIIaad
HaMlag CMII Rahh 33 Kb 23.2
5.0 Wd
Laag 55 5540 <5 45-70 41.7 4 Ma 0.1 J2-.34 .11-24 7-22 US ISO

Short 35-40 57b 05-10 244


Laag 30 15-32 12b 05-10 20.0 3Wk
5.0 0.75 0.40 W 72 31.3 to
1.7
45 40
Chill Start 07 38.3 100 44 II 0.47 2241 250
CMII Raha 30 17 20.1
1.7
ISOd
•riaad Shart 40 40-45 05 004S 31.0 1.0
Maat 0.75 ISO
Laag 31 31-32 05 IMS 21.3 4 Mo 0.1 ISO
CatMaat Shaft 34 34-n 07a 05-W 24.0
Rah Frozen Laag (-S)-O 05c 004S 4*5 4 Mo 0.1 0.74 0.41 101 70 21 250
lead Shart 34 MOO 15c 0045 24.3
Laag N 30-32 05a 0045 20.4 IS Days
5.7 W
0.4 W
Short 34 34-M K 05-07 24.3
3.4 0.40 0.30 014 52
LotMFresh Laag 20 21-10 05b 21.3 40
1547 I5J 3Wk 1.0
Shart 55 50-M a 5545 41.7
to
Smoked Chill Starts U 70 53.7 105 57 1 140
1.3

541
0.40 0.32 57 ISO

CMII Habht 55 ISO


70 44.1
.3
Wd
Chill Start 45 OS 37.5 105 35 II 047 24.01
CMIIIng Chill Ralih 35
0.45 0.30 044 (0 27 250
05 20.4
1.1
14 Hn Chill Start 31 N 10.1 105 15 14 047 23.0f
ISOd
Chill Raha 21 W 11.7
1.1
250
ISOd
Short 34 14-30 00 05-W 254
Laag 20
3.4 047 0.10 •1.5 50 21 40
20-30 10b 05.10 11.7 2Wk 1.3
Chill Start 45 40 40
314 100 40 5 0.75 1141
CMII Raiih 30 •J 21.4
250
1.3
Wd
40
(Linn. Chill Start 05 114 W s II 0.70 21.01 0.75 0.42 103 72 ISO
Haarti,
ale.) Chill Flnha 32 05'
22.1
1.3
Wd
Shart 15 1540 Wc 05-W 244
Long 32 32-11 10c 05-W 21.7 ISDari
4.2 043 0.44 lit N.4 27 W
Tab 35 35-40 70 70-75 204
0.5
W
Laag 32 32-11
2.3 0.W 0.44 ITS 07 27 ISO
70 70-75 lit 10 Dan 0.2
ISO
(*•*) Shart 34 M-N 05 05-M 24.3 IS Days 34 0.40 040 014 40 21 w
Freth Laag 21 25-10 07b 05-10 114 10 Davt 0.4 0.71 0.37 IN 74 27 40
Laag (-5M 05 05-W 4.46 10 Mo
Wet Picked
Chill Starts 45 OS
0.2
ISO
374 05 41 5 1.00 17.01
Chill Rahh a. 150
Wd
458 PRINCIPLES OF REFRIGERATION

TABLE 10-12 (Continued)

DESIGN ROOM Eit


SPECIFIC
Mail-
CONDITIONS Prod HEAT
Tamparatura Ralatlva Grain! Latant OWIb/ Latant Wn-
Mail- Product tar
Fraai- Alr
OF Humidity par lb DagF Haat Mo-
MEATS TYPE mum Haat
of Con- Ing
STORAGE RaC- Rac- Air at
Storaga Dot F Tima Rata Otu/lb Bo- Attar tant Point tion
Raeom- Fac- Fulion
om- om- Parmis- Hr 24 Hr DagF In
mand-
'armli-
ilbl. nand Ibla mandad Parlod fin-
Start lib
tor (E». 1M
fora Fram- Btu/lb
Fram- ing
% Room
ad Ranga ad Ranga Condi- Notaf) ing Ft./Mln
tion
DagF DagF % %
SauMga
Caung» Short 40 404 80c 75-00 29.1 0.2 0.40 ISO
(Saltad) Long 31 31-32 00c 75-00 20.1 4 Mo 0.0 iso

And
Frank! Short 35 35-40 15a 80-90 25.2 «Hr 4.S 0.U 156 u at H u
Smokad Chill Start 42 80 31.4 70 35 2 1.00 9.0f 150

Chill Flnlih 32 M 21.1 tod


Fran Short 35 35-40 05a 05-90 25.2 7 Dayi 4.3 0.09 0.56 93 45 26 to
Chill Start 42 05 33.4 70 35 2 1.00 9.0f ISO

Chill Flnlih 32 OS 22.3 tOd


55 5S-M 40 35-40 25.5 0.0 to
Smokad Short 40 35 40 05 00-90 31.0 4 Mo 3.2 0.06 0.56 86 60 25 to
Summar Drying 50 41-56 70 45:80 37.2 5.0 to
Long 32 32 34 70 70-75 18.4 6-8 Mo 2.0 40
Wrapping 45 45-50 05 80-8! 37.5 0.0 40
Room
Vaal Short 34 34-31 07b 85-90 24.8 3.6 0.71 0.39 91 43 29 to
Long 2B 21-30 87b 85-90 19.0 15 Dayi 1.3 40
Chill Start 45 to 39.6 100 40 6 0.75 2IJH to
Chill Flnlih 30 n 21.4 1.3 tOd

From Carrier Design Data. Reproduced by permission of Carrier Corporation.


TABLES 459

TABLE 10-13. Design Data for Miscellaneous Storage

DESIOM tOOM
CONDITIONS SPECIFIC
Est
Fred
HEAT Mas*.
MUCH, TYPE OF Urn •Why pari? Mail- •redact Latairi 0ta/lb/ Lataat Wa-
Haat Fraai- Alr
Rec Rec- Air at mum Tamp.
Rata
Haat DagF ter
Storage DagF lima Otg/lb ol Con.
Parmts- om- permis- Racom- Hr Fac- la- Attar Fasten than
Period 24 Hr rant

iearlWhale-
ahrl
meed- dbla
ad
DagF DagF
ad
% Tsible aiaadad
Condi-
tion
Start i*
tor
(Exsaa
Notef)
tare
Fran-
•«
Free* Otv/lb
ag % DagF In

Ft./Mtn

WoedaaKeg Short 3C 3540 05 0045 254 4 Mo 74u 1.0 (24 20 ISO


Metal Keg 36 354 74c 45-70 204 4 Ma 04u ISO
•attar or Sbart 40 3545 Otc 7540 ».l 10 Days 24 044
Honey SePr Laag MM 05 •045 4.45 4 Me 0.3
0.34 15 154 304» 150
250
Caady Long as 40-70 55 5045 B4 4 Mo 043 40
Carter 40 4*45 OS 5046 314
(la T«b<) Lang. 14 34-34 05b 0045
24 a in
24J 15 Day 04 ISO
CtHMta Sbart 40 43-45 •0b 7540
Americaa » 30-34 7540
21.1
Mo
2.3 0.44 044 70 (54 17 W
21.1 16 04 to
Camembert Sbart 40 4*45 K 0045 314 2.5 0.70 040 M 104 10 a
Laaa 40 34-34 05b 0045 314 fODays 04
Umbargar 40 40-45 05 1045 314 24 0.70 040 04 404 it a
fteqeerort
Laaa
Sbart
31
41
30-34
4540
06b
10
•046
7540
214
354
MDayi M
Lena 40 30-34 00b 7540 40 Dan
24 046 042 70 S5.0 3 a
2».l 04
4) 40-45 00 7540
u£ SO 3044 00b 7540
21.1
274 MDayi
24 •44 044 7» 55.0 IS a !

04 . a
Lang <0 40-70 55 5045 4 Mo
Coating)
42.1 0.1 044 041 40 04 0545 a
Craam Sbart 35 3540 •0c 7540 234
(40%) 5 MM M 0046 5.41 4 Mo
24u 045 0.40 SO 554 a ISOd
0.lu in
Eggs Crated Sbart 40 4045 Kb •046 314
(See Doc Laag 30 30-31 05b 0547 204 Mo
34 045 0.46 100 744 314 a
2D4S) Chill Start 40 05b 314
12
45
04 u
Chill R.ltt 30 05b 20.4
30 10 046 74f a
04 40d
Eggs, Frozen
10 lb MM
Long
CMII Start
S MM 40 4.24 10 Mo 04lu 046 100 2»
Otc 445 40 5 24 047 »4f
Oac.2D.es 250
Chill Raid Otc 445 044 2a
Far, Woolen Famlgated 35 36-40 45 4045 l»4 4 Mo 0.1 0.40 la
(See Doc.
1041)
let* only 15 15-10 70 45-70 04 4 Mo 0.1 la
Fleer Laag 70 71-02 40 4045 044 4 Mo 040 040 134 a
Howan,Cat 40k 33-40 05 •540 314 Dayi
•aaaral 3-14 I20 0tu\ 042 27-31 u
Orchldl
Oardeales
46 45-50 05 •5-10 374 1 Wk PerSq
20-31 a
FtFloei
Hides. Caring SS 5*46 OS •045 54.5 04 0.40 la
Storage l~g Si 32-40 75 70-75 23.1 5Yr 0.1 0.40 la
leaCraam Hardaalag
SSal Can Start •Sc 4.16 -10
Ruhr. -a Be
22 0.75 l.lf.u 0.77 37 H4 2044 2a
(SaaDec. 1.55
Start 05c
0.1a 2B
JD44) 445 24 -10 0.75
Rahb -20 05c 145
l.3f,ii 42 2a
O.lu
so
Lard Sbart 45 45-50 Mc 7540 354
32 3244 •0c 7540 21.1 4Mo
24 040 B a la
04
Maple Sbart 45 45*0 70c 45-70 2».f 0.7 044 041 7 5 2a
Loag II 31-32 70c 45-70 17.7 6 Mo 0.1
2a
4*0 PRINCIPLES OF REFRIGERATION

TABLE 10-13 (Continued)


DESI0N ROOM SKCIFIC
CONDITIONS Eat
HEAT Mait-
Tomparatura Ralatlva Gralai Prod
Maxi- Ota/lb/ Latant Wa-
MISCEL- TYPEOF Humidity par lb Product Lataat
DaoF Haal tar Fran- Alr
STORA6E Air at mum Tamp. Haat lag Ma-
Rac- Sac- Storago DagF Unto Rata Ota/lb la-
of
Attar Futlon
COR'
Parrnh)- om- Farmlf- Racom- Hr Fac- tant
•aflat ttel
Parted 24 Hr tora Fraaa- Otu/lb DagF la
maad- tlbla •Ibla
Coadl- Start
sr
tor
(Es.m Fraaa- i>g % Room
ad Raago "ad lUaga
DagF DagF % V tlon Notof) la. Ft./Mla

Mapla Short « 4S-50 70c 45-70 0.7 0.40 0.11 53 34.0 ISO
Strip II 31-11 70c 45-70 17.7 5 Mo 0.1 no
MDt B
lorHad
Short 05-40 70c 45-75 20.1 SDayt 0.00 Ml 124 07.5 31 ISO
HdWil Chill Start 41 Mc ».l 45 IS 10 0.K BO
Doc»-» Chill Ftaha M 10c 0.1 B0
Nab, la Short « 40-41 70c 45-75 25J O.SO 0JS 0.22 3-10 2-0 in
SMk Loag 32-40 70c 45-75 11.4 0-12 M; 0.01 ISO
Nats Short « 1-45 70 45-75 B.3 0.50 0.3 0.24 4-14 3-10 ISO
IMM Loag 11 n-«> 70 45-75 10.4 4-10
Mfl 0M ISO
Olao Short 45 45-H 00c 7540 38.1 2J> 0.40 150
Loag M M-U 00c 7540 23J> ODDayi 0.3 ISO
VkcIim Loag 4MS
Saram
41 70 •5-70 20.5 4 Mo 0.0 150

Saraba Loag J» 24-» 70 •MO 16.4 i-OMo DM 0.* 50.0

From Carrier Design Data. Reproduced by permission of Carrier Corporation.


TABLES 461

TABLE 10-14. Reaction Heat from Fruits and Vegetables

FRUITS VEGETABLES
Temperature Btu per hr
Temperature Btu per Kr
Commodity D.gP per lb
Commodity O.gF per lb
Applet 32 Asparagus 32 .5S
.018
40 .030 40 .170
40 .120 Beans, Lima 32 .170
60 .820
Apricots 32 .023 Beans, String 32 .099
40 .036
40
M .170
60
.140
.470
Bananas Beets 32 .055
Holding 54 40 .085
.069
Ripening M .190
60 .150
Chilling 70-S6 Brussel 32 .059
.500§
Sprouts 40 .095
Borrios 34 .115 60 .280
M .345 Cebbage 32 .059
40 .095
Cherries 32 Ml 60 .280
60 .250 Cauliflower 32 .059
40 .095
Cranberries 32 .014 60 .280
40 .019
Carrot. 32 .045
SO .036
40 .073
Dates, Froth 32 60 .170
.014
40 .019
Celery 32 .059
50 .036 40 .095
60 .280
Grapefruit 32 .0096 Corn, Sweet 32 .035
40 .022 40 .170
60 .058 Cucumber 32 .028

Grape*
40 Ml
32 .0075 60 .175
40 .014 Endive 40 .200
60 .050
Lettuce 32 .240
_«moM 32 .012 40 J30
40 .017 60 .960
60 .062 Melons 32 .028
(Except 40 .041
Limos 32 .012 Watermeionf) 60 .175
40 .017 Mushrooms 32 .130
60 .062 50 .460
Onions 32 .018
Orangos 32 .017
50 .039
40 .029
70 .075
60 .104
Parsnips 32 .045
Poachoi 32 .023 40 .073
40 .036 60 .170
60 .170 Poas 32 .170
60 .820
Poars 32 .016 Peppers 32 .057
60 .230 60 .180
Potatoes 32 .014
Plums 32 .032
40 .030
60 .250

32
40
.018
Spinach
Sweet Potatoes
70
40
40
M
.060

.070
.030
60 .120 Tomatoes
(Green) 60 .130
Strawborriot 32 .068 (Rip.) 40 .027
40 .120 Turnips 32 .040
1
60 .360 I 40 | .050

From Carrier Design Data. Reproduced by permission of Carrier Corporation.


462 PRINCIPLES OF REFRIGERATION

TABLE 10-15. Heat Equivalent of


Electric Motors
Btu/hp-hr

Motor Connected
Connected Losses Load
Load in Outside Outside
Motor Refr. Refr. Refr.
hp Space 1 Space2 Space3 TABLE 10-16. Heat Equivalent of
Occupancy
|to£ 4250 2545 1700 Cooler
*to3 3700 2545 1150 Temperature, Heat Equivalent/Person
3 to 20 2950 2545 400 F Btu/hr

1
For use when both useful output and motor 50 720
losses are dissipated within refrigerated space; 40 840
motors driving fans for forced circulation unit 30 950
coolers. 20 1050
2
For use when motor losses are dissipated 10 1200
outside refrigerated space and useful work of 1300
motor is expended within refrigerated space; -10 1400
pump on a circulating brine or chilled water
system, fan motor outside refrigerated space From ASRE Data Book, Design Volume,
driving fan circulating air within refrigerated 1949 Edition, by permission of the American
space. Society of Heating, Refrigerating, and Air-
3 Conditioning Engineers.
For use when motor heat losses are dissi-
pated within refrigerated space and useful work
expended outside of refrigerated space; motor
in refrigerated space driving pump or fan located
outside of space.
From ASRE Data Book, Design Volume,
1949 Edition, by permission of the American
Society of Heating, Refrigerating, and Air-
Conditioning Engineers.
TABLES 463

TABLE 10-17. Usage Heat Gain, Btu per 24 Hour for One Cubic Foot Interior
Capacity
Temperature Reduction in ° F
Volume (Outside temperature minus storage temperature)
Cubic
Feet Service 40° 45° 50° 55° 60° 65° 70° 75° 80°

15 Normal 108 122 135 149 162 176 189 203 216
Heavy 134 151 168 184 201 218 235 251 268

50 Normal 97 109 121 133 145 157 169 182 194


Heavy 124 140 155 171 186 202 217 233 248

100 Normal 85 96 107 117 128 138 149 160 170


Heavy 114 128 143 157 171 185 200 214 228

200 Normal 74 83 93 102 111 120 130 139 148


Heavy 104 117 130 143 156 169 182 195 208

300 Normal 68 77 85 94 102 111 119 128 136


Heavy 98 110 123 135 147 159 172 184 196

400 Normal 65 73 81 89 97 105 113 122 130


Heavy 95 107 119 130 142 154 166 178 190

600 Normal 61 68 76 84 91 99 106 114 122


Heavy 91 103 114 125 137 148 160 171 182

800 Normal 59 67 74 81 89 96 104 111 118


Heavy 89 100 112 123 134 145 156 167 178

1000 Normal 57 64 72 79 86 93 100 107 114


Heavy 86 97 108 119 130 140 151 162 173

1200 Normal 55 62 69 76 83 90 97 104 110


Heavy 84 95 105 116 126 137 147 158 168

1600 Normal 51 58 64 70 77 83 90 96 102


Heavy 79 89 99 108 118 128 138 148 158

From ASRE Data Book, Design Volume, 1949 Edition, by permission of the American Society
of Heating, Refrigerating, and Air-Conditioning Engineers.
464 PRINCIPLES OF REFRIGERATION

TABLE 10-18. Wall Heat Gain


(Btu per sq ft per 24 hr)
Insulation

Cork or Temp. Difference (Ambient Temp Minus Refrigerator Temp) ., F


.

Equivalent
in. 1 40 45 50 55 60 65 70 75 80 85 90 95 100 105 110 115 170

3 2.4 96 108 120 132 144 156 168 180 192 204 216 228 240 252 264 ?67 788
4 1.8 72 81 90 99 108 117 126 135 144 153 162 171 180 189 198 7,07 7.16
5 1.44 58 65 72 79 87 94 101 108 115 122 130 137 144 151 159 166 173
6
7
1.2
1.03
48
41
54
46
60
52
66
57
72
62
78
67
84
72
90
77
96
82
102
88
108
93
114
98
120
103
126
108
m
113
138
118
144
174

8 0.90 36 41 45 50 54 59 63 68 72 77 81 86 90 95 99 104 108


9 0.80 32 36 40 44 48 52 56 60 64 68 72 76 80 84 88 97 96
10 0.72 29 3r 36 40 43 47 50 54 58 61 65 68 72 76 79 83 86
11 0.66 26 30 33 36 40 43 46 50 53 56 60 63 66 69 71 76 79
12 0.60 24 27 30 33 36 39 42 45 48 51 54 57 60 63 69
66 77

13 0.55 77 75 28 30 33 36 39 41 44 47 50 52 55 58 61 63 66
14 0.51 70 73 76 28 31 33 36 38 41 43 46 49 51 54 56 59 61
Single
glass 27.0 1080 1220 1350 1490 1620 1760 1890 2030 2160 2290 2440 2560 2700 2840 2970 3100 3240
Double
glass 11.0 440 500 550 610 660 715 770 825 880 936 990 1050 1100 1160 1210 1270 1320
Triple
glass 7.0 280 320 350 390 420 454 490 525 560 595 630 665 700 740 770 810 840
Note: Where wood studs are used multiply the above values by 1.1.
From ASRE Data Book, Design Volume, 1955-56 Edition, by permission of the American Society of Heating.
Refrigerating, and Air-Conditioning Engineers. °
~M

TABLES 465

6.34 7.82 8.97 9.94 29.


©tx*'*-
*©©©*>
10.82 11.67 12.37 13.92 14.43 15.06 15.66 16.26 16.81 17.39

* 17.93 18.51 18.99 19.23 22.87 23.33 23.77 24.22 24.66

ifi i« t*i ««
27.28 27.64 28.48 28.23

<efsx«>©
«»tA©x« 3 « e <tfx « »# » c tr.
«o«x^
©
t^XX©*©

«IHf9(4M ©*«*>© 3 x*«©in©


X ©tn©f4©
•r*t>ao©© © »» r« -J w *©fr* XXX « * •• tft ift


SSSS2 RS8S8 #8?SS S3S58 ^rN, — Kx
mt*xx© • *-««
2S2SS
-e
*>t*>t*3<3< «>©*»x 8RSSS| *S3x& >
2««rx»*
©<© <et^tC
N M ** *"l <»•
^ *i * »r" ^s

Xf««3©f4
ift
S5S5J2 S8S8S SSS2S S5SS3
W*0*0^t«ft

w t*©r«*<© n»oo*x O«
c «• *©*>©tiv ©#©f«X IT. va to

£ **n'qox »'©©«*.*
mmmmm l^*SIN****N
9Mf4IAA » X © t« (S
*c t9W9 S8SSS t* M * « «m in
f*o*xn
IT. 5j»Mt«©
r<> ir, tft

o ©*©*•••-« ^n p^ ffl f^ f^ kWVo


«
3S388 tttqxiA© 8SS8S XO<S»^i M9«P«X xStft?*

x©*»*©©
•noK M 0*4 *! ** W*
! r*xr>r<*w
»* 85#W SR.SS8 5SRS8 «x^m» X«X*«S4
xod©*©©
o- "
t^rCxxx
E
•i
H 9 Sft#SS tort«»n *3©^S
t^xx©'© <«l>t>l>X ocaeV»V ©'©©M*
>
?ss*s *t*©©rt 9>to»Ntf)
©
©in©^t* 8SR2« S=«S2 SS25S
I ©OOmm rCxoex»
22SRS
««>«»* t*rt
»*«©©*
©«©<*© 3SSC2 3>*3S2 M^tAtoX
X *•« *« rt r« t«)

mo^xco t*r*.t^©r*
SSS8R nmitKo ro©fs^©
tv.
wm^tcio X©*©'©© t» X XXX
M(NM«K«4
t*.^i«5©^i
©r*«©w t*(*.t0t*iA
m3<£SiS (StfiX — t l>©^<©
« ©O© — •et^tCtotC
III
-I «MiA»n
a «<h©<«S ^X~*>X M«r.M<««i
»fttft*«r*
58SKS
« N0D««e> r^f^
MMMMNf) f)*fl

2
•«x>-«9<r
enisMir, ©r«>S5f*s 3©">«© K«£2« >o*5©to»*j
»AX«4t4©
* aox©*©'©

x*5»© 3E283 -.«X-.? acnae«»


Mlftt>©N
« t* XXX X
3©t%©X tOtOMMA
e%
^©*«<s ^ ©«<•»© rtioSof • mSSm SS8S2 S8S5S
xxxx^ OOtMMM
1

•n
M825 t*©f«3© r«mx«ot*
528=S 5S2SS t*. t* »> t* t*

M f*S fj Hi f*> t%t^XXX BOOVVV ©©©•©


1

P*<1«9<riA «mo»o — M«^>»fi ©t*X©0


««r»«roj *«NN« o-f^^JJift
i^ *<5 ^ fO ^
466 PRINCIPLES OF REFRIGERATION

TABLE 11-2. Evaporator Design TD


Design TD, ° F
neiauve
Humidity, Natural Forced
% Convection Convection

95-91 12-14 8-10


90-86 14-16 10-12
85-81 16-18 12-14
80-76 18-20 14-16
75-70 20-22 16-18

For temperatures 10° F and below, an evapo-


rator TD of 10° F is generally used for forced
convection evaporators.

TABLE 11-3. Properti es of Pu re Calc ium Chli aride Brine

Specific Specific Weight per gallon Weight per cubic foot


Pure
gravity Baume heat Crystal-
CaCli Water
60 F density 60F lization CaCl, Brine
%by 60F Btu per starts
CaClt Water Brine
lb/cu lb/cu lb/cu
wt lb/gal lb/gal lb/gal
60 P lbF F ft ft ft

1.000 0.0 1.000 32.0 0.000 8.34 8.34 0.00 62.40 62.40

5 1.044 6.1 0.924 27.7 0.436 8.281 8.717 3.26 61.89 65.15
6 1.050 7.0 0.914 26.8 .526 8.234 8.760 3.93 61.59 65.52
7 1.060 8.2 0.898 25.9 .620 8.231 8.851 4.63 61.51 66.14
8 1.069 9.3 0.884 24.6 .714 8.212 8.926 5.34 61.36 66.70
9 1.078 10.4 0.869 23.5 .810 8.191 9.001 6.05 61.22 67.27

10 1.087 11.6 0.855 22.3 0.908 8.168 9.076 6.78 61.05 67.83
11 1.096 12.6 0.842 20.8 1.006 8.137 9.143 7.52 60.81 68.33
12 1.105 13.8 0.828 19.3 1.107 8.120 9.227 8.27 60.68 68.95
13 1.114 14.8 0.816 17.6 1.209 8.093 9.302 9.04 60.47 69.51
14 1.124 15.9 0.804 15.5 1.313 8.064 9.377 9.81 60.27 70.08

15 1.133 16.9 0.793 13.5 1.418 8.034 9.452 10.60 60.04 70.64
16 1.143 18.0 0.779 11.2 1.526 8.010 9.536 11.40 59.86 71.26
17 1.152 19.1 0.767 8.6 1.635 7.984 9.619 12.22 59.67 71.89
18 1.162 20.2 0.756 5.9 1.747 7.956 9.703 13.05 59.46 72.51
19 1.172 21.3 0.746 2.8 1.859 7.927 9.786 13.90 59.23 73.13

20 1.182 22.1 0.737 - 0.4 1.970 7.883 9.853 14.73 58.90 73. 61
21 1.192 23.0 0.729 - 3.9 2.085 7.843 9.928 15.58 58.61 74.19
22 1.202 24.4 0.716 - 7.8 2.208 7.829 10.037 16.50 58.50 75.00
23 1.212 25.5 0.707 -11.9 2.328 7.792 10.120 17.40 58.23 75.63
24 1.223 26.4 0.697 -16.2 2.451 7.761 10.212 18.32 58.00 76.32

25 1.233 27.4 0.689 -21.0 2.574 7.721 10.295 19.24 57.70 76.94
26 1.244 28.3 0.682 -25.8 2.699 7.680 10.379 20.17 57.39 77.56
27 1.254 29.3 0.673 -31.2 2.827 7.644 10.471 21.13 57.12 78.25
28 1.265 30.4 0.665 -37.8 2.958 7.605 10.563 22.10 56.84 78.94
29 1.276 31.4 0.658 -49.4 3.090 7.565 10.655 23.09 56.53 79.62

29.87 1.290 32.6 0.655 -67.0 3.16 7.59 10.75 23.65 56.80 80.45
30 1.295 33.0 0.653 -50.8 3.22 7.58 10.80 24.06 56.70 80.76
32 1.317 34.9 0.640 -19.5 3.49 7.49 10.98 26.10 56.04 82.14
34 1.340 36.8 0.630 + 4.3 3.77 7.40 11.17 28.22 55.35 83.57
From ASRE Data Book, Design Volume, 1957-58 Edition, by permission of The American Society of
Heating, Refrigerating, and Air-Conditioning Engineers.
TABLES 467

TABLE 1 1-4. Properties of Pure Sodium Chloride Brine

Specific Weight per gallon Weight per cubic foot


Pure Specific Crystal'
Baumfi heat
NaCl gravity lization
density 59 F NaCl Water Brine
%by 59 F 60F Btu/lb
starts NaCl Water Brine
lb/cu lb/cu lb/cu
wt 39 F degF F lb/gal lb/gal lb/gal
ft ft ft

1.000 0.0 1,000 32.0 0.000 8.34 8.34 0.000 62.40 62.4

6 1.035 5.1 0.938 27.0 0.432 8.22 8.65 3.230 61.37 64.6
6 1.043 6.1 0.927 25.5 0.523 8.19 8.71 3.906 61.19 65.1
7 1.050 7.0 0.917 24.0 0.613 8.15 8.76 4.585 60.91 65.5
8 1.057 8.0 0.907 23.2 0.706 8.11 8.82 5.280 60.72 66.0
9 1.065 9.0 0.897 21.8 0.800 8.09 8.89 5.985 60.51 66.5

10 1.072 10.1 0.888 20.4 0.895 8.05 8.95 6.690 60.21 66.9
11 1.080 10.8 0.879 18.5 0.992 8.03 9.02 7.414 59.99 67.4
12 1.087 11.8 0.870 17.2 1.090 7.99 9.08* 8.136 59.66 67.8
13 1.095 12.7 0.862 15.5 1.188 7.95 9.14 8.879 59.42 68.3
14 1.103 13.6 0.854 13.9 1.291 7.93 9.22 9.632 59.17 68.8

15 1.111 14.5 0.847 12.0 1.392 7.89 9.28 10.395 58.90 69.3
16 1.118 15.4 0.840 10.2 1.493 7.84 9.33 11.168 58.63 69.8
17 1.126 16.3 0.833 8.2 1.598 7.80 9.40 11.951 58.36 70.3
18 1.134 17.2 0.826 6.1 1.705 7.76 9.47 12.744 58.06 70.8
19 1.142 18.1 0.819 4.0 1.813 7.73 9.54 13.547 57.75 71.3

20 1.150 19.0 0.813 + 1.8 1.920 7.68 9.60 14.360 57.44 71.8
21 1.158 19.9 0.807 - 0.8 2.031 7.64 9.67 15.183 57.12 72.3
22 1.166 20.8 0.802 - 3.0 2.143 7.60 9.74 16.016 56.78 72.8
23 1.175 21.7 0.796 - 6.0 2.256 7.55 9.81 16.854 56.45 73.3
24 1.183 22.5 0.791 + 3.8 2.371 7.51 9.88 17.712 56.09 73.8

25 1.191 23.4 0.786 + 16.1 2.488 7.46 9.95 18.575 55.72 74.3
25.2 1.200 +32.0

From ASRE Data Book, Design Volume, 19S7-S8 Edition, by permission of The American Society or
Heating, Refrigerating, and Air-Conditioning Engineers.

TABLE 1 1-5. Freezing Points of Aqueous Solutions


Alcohol Glycerine Ethylene Glycol Propylene Glycol
% by Wt Deg F %byWt DegF % by Vol DegF % by Vol DegF

5 28.0 10 29.1 15 22.4 5 29.0


10 23.6 20 23.4 20 16.2 10 26.0
15 19.7 30 14.9 25 10.0 15 22.5
20 13.2 40 4.3 30 3.5 20 19.0
25 5.5 50 -9.4 35 -4.0 25 14.5
30 -2.5 60 -30.5 40 -12.5 30 9.0
35 -13.2 70 -38.0 45 -22.0 35 2.5
40 -21.0 80 -5.5 50 -32.5 40 -5.5
45 -27.5 90 +29.1 45 -15.0
50 -34.0 100 +62.6 50 -25.5
55 -40.5 55 -39.5
59 -57.0
Above 60% fails to crys-

tallize at -99.4 F
From ASRE Data Book, 1957-58 Edition, by permission of the American Society of Heating,
Refrigerating, and Air-Conditioning Engineers.
468 PRINCIPLES OF REFRIGERATION

TABLE 12-1. Approximate Volumetric Efficiency of Refrigerant- 1 2 Compressors


at Various Compression Ratios

Compression Ratio Volumetric Efficiency Compression Ratio Volumetric Efficiency

2 87.3 62 6Z2
2.2 86.0 6.4 61.2
2.4 84.9 6.6 60.2
2.6 83.5 6.8 59.2
2.8 82 7 58.2
3- 80.8 7.2 57.2
3.2 79.5 7.4 56.3
3.4 78.3 7.6 55.3
3.6 77.2 7.8 54.4
3.8 76.0 8 53.5
4 74.9 8.2 52.6
4.2 73.7 8.4 51.7
4.4 72.5 8.6 50.8
4.6 71.3 8.8 49.9
4.8 70.1 9 49
5 69.0 9.2 48.1
5.2 67.9 9.4 47.2
5.4 66.8 9.6 46.4
5.6 65.7 9.8 45.7
5.8 64.5 10 44.9
6 63.3
TABLES 469

Temperature of Water 125° F or Less

Types of Water Water Velocity Ft/Sec

3 Ft and Less* Over 3 Ftf

Sea water 0.0005 0.0005


Brackish water 0.002 0.001
Cooling tower and artificial spray pond: Treated make-up 0.001 0.001
Untreated 0.003 0.003
City or well water (Such as Great Lakes; 0.001 0.001
Great Lakes 0.001 0.001
River water: Minimum 0.002 0.001
Mississippi 0.003 0.002
Delaware, Schuylkill 0.003 0.002
East River and New York Bay 0.003 0.002
Chicago Sanitary Canal 0.008 0.006
Muddy or silty 0.003 0.002
Hard (Over 15 grains) gal 0.003 0.003
Engine jacket 0.001 0.001
Distilled 0.0005 0.0005
* 2.16 gpm per tube is equivalent to a water velocity of 3 ft per second.

t This table is presented by permission of the Tubular Exchanger Manufacturers Association,


Inc., New York.

TABLE 14-2. BI«ed-Off Rates


Cooling Range Percent
Deg. F Bleed-off

6 0.15
7* 0.22
10 0.33
15 0.54
20 0.75

Courtesy The Marley Company.


470 PRINCIPLES OF REFRIGERATION

OOm irt O o n
MN«Mn
f> r» f» o
«s
O oo o vt tN ©
m w*
f"-

vi v» «N * oo tno\ift«in o\ uo i> vi ^nift


vo 5K L!

© O © m SO m
HflOm^M
^ ON ©
00 vt
^0Q^«-4tN *N ^ vi so vi -h S©

OOWhw 1 ***
voo ^ O
OS
H^mmift _
^h
-- vi
tn
»n ^©
^
*-« vi
oo vi
©
•rtSSJ*" '"'

iOOOO ("-^^"Os^* «* onm © v* v. «n o\

iZ
•a OOnineO Vt tN 00 tN 00 tN
^ »-hO
^ ~* ^ »n ** o -*
^
B ^OOONqo*^ ,-h ,-« *TS fO ^ 00 SO "^

s vt © © oo so ^oninm
*n m f>
.-*»-< <-• .-**
so O VI 00 SO Vt
>

o I OQO«^ ^ON'OOsO OS
^ oo O
^ -h
rn
SO 00
^
O
tri
«*"»

»>
m
'™
l

? a Opoen © oo <4- so «t oo n o i> oo m i>


*"*
l> «N vi ^ in
o *
v a, O so vt ©
B 73 ri S^« vi ci «n so

* e © VI Vi Vl © O OO o so o o
*^
oo SO v»
_! OS * _^ ~« <S«N »N ~* m c4
3 I vj <s

£23 00m © © On t>


""> oo oo r* vt
OO Vt l> 00 f»
oo 00 ^ P4 (*5 <N -! *
"O
< ^ irt ^ o <o «N ^- ^" w-i ^ <* VJ 00 "* O <rj ^ so
so (*i ph*

.Q
vj vi ci m cs
^ Om iN vj oo m
V tn *"" *"* vi «*> *J ri -J ^
•w
4) v> © os o OS O Vt SO Os OVtOs ^ O
41 •*> ^ ri d tN ^d *n *
IL
© v» vj oo v> VJ tN oo so M 00 O
fO IN »> 1-^ **> »N O Hrtd *N

\© *© * fO "A «rj © wso v» O vj \o so m O so vt bo

C *n" tN v» vi O
01
00 VI Vt VJ Vt 0<*)«n O oo vj ro O
^ **^ * Nrtd p^dd ** (s
e
« m so oo oo v. ** o © »n o oovon o\ vi

>
© *"*
©~ »-<«-« en fi *n odd d ^
3
Ul

III
o
_l
co oo
.5 s
2 a o
2*S
.§ -as § •s°.
**« o. oo bo *^o c

i ii ii ii II II
5.* ^
e a » u I 1 DQQ °QQQ &.§.§

•Sop
iJla o u
_

TABLES 471

TABLE 15-2. Pressure Drop Correction Factors*


Specific
Gravity Friction Correction Factor
at(°F) Temperature— F
Freeze
Liquid at°F -20 +20 -20 -10 10 20 30 40 50 60

Calcium brine
Sp gr = 1.10 20.3 1.11 1.21 1.19 1.15 1.12 1.11
Sp gr = 1.20 -5.8 1.21 1.49 1.44 1.38 1.33 1.28 1.26 1.24
Sp gr = 1.25 -26.0 1.27 1.26 1.85 1.75 1.66 1.57 1.50 1.44 1.40 1.37 1.34

Sodium brine
Sp gr - 1.10 14.9 — 1.11 1.27 1.21 1.19 1.15 1.12 1.11
Sp gr = 1.18 -6.0 1.19 1.58 1.50 1.44 1.39 1.33 1.28 1.25 1.22

Ammonia (liquid) -107.8 0.68 0.65 0.65 0.65 0.65 0.65 0.65 0.65 0.65 0.65 0.65
Alcohol (ethyl) (100%) -114.6 0.83 0.81 0.97 0.95 0.93 0.92 0.91 0.91 0.90 0.90 0.90
Alcohol (ethyl) (40%) -22 0.93 0.91
1.45 1.39 1.33 1.29 1.23 1.19 1.15 1.12 1.10
Alcohol (methyl) (100%) -97 0.84 0.82 0.85 0.85 0.85 0.84 0.84 0.84 0.83 0.83 0.83
Alcohol (methyl) (30%) -5.8 0.96 1.32 1.26 1.22 1.19 1.16 1.12 1.09 1.07 1.05

Ethylene glycol (60%) -59.0 1.10 1.09 1.87 1.83 1.78 1.72 1.62 1.57 1.46 1.40 1.36
Ethylene glycol (50%) -38.0 1.09 1.08 1.83 1.74 1.64 1.54 1.48 1.42 1.37 1.31 1.26
Ethylene glycol (30%) 2.0 1.06 1.34 1.27 1.22 1.17 1.13 1.11

Refrigerant-1 1 (liquid) -168 1.60 1.55 1.42 1.42 1.42 1.42 1.42 1.42 1.42 1.42 1.42
Refrigerant-12 (liquid) -252 1.49 1.42 1.32 1.32 1.32 1.32 1.32 1.32 1.32 1.32 1.32
Methyl chloride -144 1.02 0.97 1.13 1.13 1.13 1.12 1.12 1.12 1.11 1.11 1.11
Methylene chloride -142.1 1.40 1.33 1.65 1.63 1.62 1.60 1.59 1.58 1.56 1.54 1.52

* To obtain pressure drop from flow of above liquids through pipes, multiply pressure drop for
water flow (of equal quantity through same pipe) by factors from above table.
Courtesy York Corporation.
472 PRINCIPLES OF REFRIGERATION

TABLE 16-1. ASRE Refrigerant Numbering System


ASRE
Standard Boiling
Refrigerant Chemical Molecular Point,
Designation Chemical Name Formula Weight F Status1

Halocarbon Compounds
10 Carbontetrachloride CC1 4 153.8 170.2
11 Trichloromonofluoromethane CC1.F 137.4 74.8 C
12 Dichlorodifluoromethane CC1.F, 120.9 -21.6 c
13 Monochlorotrifluoromethane CC1F, 104.5 -114.6 c
13B1 Monobromotrifluoromethane CBrF, 148.9 -72.0 s
14 Carbontetrafluoride CF4 88.0 -198.4 s
20 Chloroform CHC1, 119.4 142
21 Dichloromonofluoromethane CHC1.F 102.9 48.1 D
22 Monochlorodifluoromethane CHC1F, 86.5 -41.4 C
23 Trifluoromethane CHF, 70.0 -119.9 D
30 Methylene chloride CH.C1, 84.9 105.2 C
31 Monochloromonofluoromethane CH.C1F 68.5 48.0
32 Methylene fluoride CH.F, 52.0 -61.4
40 Methyl chloride CH.C1 50.5 -10.8 C
41 Methyl fluoride CH.F 34.0 -109
(50 Methane CH4 16.0 -259 Q«
110 Hexachloroe thane ccucci, 236.8 365
111 Pentachloromonofluoroethane CC1.CC1.F 220.3 279
112 Tetrachlorodifluoroethane CC1,FCC1,F 203.8 199.0
112a Tetrachlorodifluoroethane CC1.CC1F, 203.8 195.8
113 Trichlorotrifluoroethane CC1.FCC1F, 187.4 117.6 C
113a Trichlorotrifluoroethane CC1.CF, 187.4 114.2
114 Dichlorotetrafluoroethane CCIF.CC1F, 170.9 38.4 C
114a Dichlorotetrafluoroethane CC1.FCF, 170.9 38.5 c
114B2 Dibromotetrafluoroethane CBrF.CBrF, 259.9 117.5 D
113 Monochloropentafluoroethane CCIF.CF, 154.5 -37.7 D
116 Hexafluoroethane CF.CF, 138.0 -108.8
120 Pentachloroethane CHCl,Ca, 202.3 324
123 Dichlorotrifluoroethane CHa,CF, 153 83.7
124 Monochlorotetrafluoroethane CHC1FCF, 136.5 10.4
124a Monochlorotetrafluoroethane CHF.CC1F, 136.5 14 D
125 Pentafluoroethane CHF.CF, 120 -55
133a Monochlorotrifluoroethane CH.C1CF, 118.5 43.0 D
140a Trichloroethane CH,CC1, 133.4 165
142b Monochlorodifluoroethane CH.CC1F, 100.5 12.2 S
143a Trifluoroethane CJH5CF3 84 -53.5
150a Dichloroethane CH.CHC1, 98.9 140
152a Difluoroethane CH.CHF, 66 -12.4 c
160 Ethyl chloride CH.CH.C1 64.5 54.0
(170 Ethane CH.CH, 30 -127.5 Q*
218 Octafluoropropane CF.CF.CF, 188 -36.4
(290 Propane CsngGHfCrig 44 -44.2 c^
Cyclic Organic Compounds
C316 Dichlorohexafluorocyclobutane C4C1,F. 233 140
C317 Monochloroheptafluorocyclobutane C.C1F, 216.5 77
C318 Octafluorocyclobutane C4F, 200 21.1 D
AZCUlFUpCS
500 Refrigerants-12/152a 73.8/26.2 wt %* CC1,F^CH,CHF, 99.29 -28.0 c
501 Refrigerants-22/12 75/25 wt % CHC1F./CC1.F, 93.1 -42
502 Refrigerants-11/115 48.8/51.2 wt % CHClF^CaF,CF, 112 -50.1
TABLES -473

TABLE 16-1 {Continued)


ASRE
Standard
Boiling
Refrigerant Chemical Molecular Point,
Designation Chemical Name Formula Weight F Status 1

Miscellaneous Organic Compounds


Hydrocarbons
50 Methane CH4 16.0 -259 C
170 Ethane CH.CH,
290
30 -127.5 C
Propane CHsCHjCHj
600
44 -44.2 c
Butane CH,CH,CH,CH, 58.1 31.3
601 Isobutane CH(CH„), 58.1 14
(1150 Ethylene CrIj=CHj 28.0 -155.0 C)«
(1270 Propylene CH,CH=CH, 42.1 -53.7 C)»
Oxygen Compounds
610 Ethyl ether C,H,OC,H, 74.1 94.3
611 Methyl formate HCOOCH, 60.0 89.2
Sulfur Compounds
620
Nitrogen Compounds
630 Methyl amine CH.NH, 31.1 20.3
631 Ethyl amine C,H6 NH, 45.1 61.8
Inorganic Compounds
717 Ammonia NH 8 17 -28.0 C
718 Water H,0 18 212
729 Air 29 -318
744 Carbon dioxide CO, 44 -109 C
(subl.)
744A Nitrous oxide N.O 44 -127
764 Sulfur dioxide SO, 64 14.0 c
Unsaturated Organic Compounds
1 1 1 2a Dichlorodifluoroethylene CC1,=CF, 133 67
1113 Monochlorotrifluoroethylene CC1F=CF, 116.5 -18.2
1114 Tetrafluoroethylene CF,=CF, 100 -105
1120 Trichloroethylene CHC1=CC1, 131.4 187
1130 Dichloroethylene CHC1=CHC1 96.9 118
1132a Vinylidene fluoride CHr=CF, 64 -119
1140 Vinyl chloride CH,=CHC1 62.5 7.0
1141 Vinyl fluoride CH,=CHF -98
46
1150 Ethylene CJij==Cjij
1270
28.0 -155.0 c
Propylene CHjCrr=CHj 42.1 -53.7 c
* Carrier Corp. Document 2-D-127, p. 1.
1. Denotes that as of October 1956, the status of these refrigerants as regards commercial
evolution is as
follows: C, S, or D. C—
Commercial S—Semi-commercial D—
Development.
2. The compounds methane, ethane, and propane appear
in the halocarbon section in their proper
numerical positions, but in parentheses since these products are not
halocarbons.
3. The compounds ethylene and propylene appear in
the hydrocarbon section as parenthetical items in
order to indicate that these compounds are hydrocarbons. Ethylene
and propylene are properly
' identified
under Unsaturated Organic Compounds.
From the ASRE Data Book, Design Volume, 1957-58 Edition, by permission of the American
Society of
Heating, Refrigerating, and Air-Conditioning Engineers.
474 PRINCIPLES OF REFRIGERATION

TABLE 16-2. Refrlgerant-ll (Trichloromonofluoromethane) Properties of


Liquid and Saturated Vapor

Liquid, Vapor, Enthalpy, datum -40 F Entropy, datum —40 1


Pressure Btu per lb Btu per lb F
Temp SPTOl sp vol
F Vapor
1 euft/lb cuft/lb Liquid Vapor Liquid
psla psig
V V •»

-90 0.100 29.72* 0.00964 288.6 -9.89 81.7 -0.0250 0.2226


-80 .157 29.60* .00961 189.0 -7.89 82.9 - .0197 .2192
-70 .240 29.43* .00967 127.58 -5.91 84.0 - .0146 .2160
-60 .356 29.19* .00974 87.5 -3.94 85.2 - .0096 .2133
-SO .518 28.86* .00981 61.1 -1.97 86.3 - .0047 .2108

-40 0.739 28.42* 0.00988 44.21 0.00 87.48 0.0000 0.2085


-36 0.847 28.20* .00991 38.93 0.79 87.96 .0019 .2076
-32 0.968 27.95* .00993 34.37 1.58 88.44 .0037 .2068
-28 1.103 27.67* .00996 30.44 2.36 88.91 .0055 .2060
-24 1.253 27.37* .00999 27.03 3.15 89.39 .0073 .2053

-20 1.420 27.03* 0.01002 24.06 3.94 89.87 0.0091 0.2046


-16 1.605 26.65* .01005 21.47 4.73 90.35 .0109 .2040
-12 1.810 26.24* .01008 19.20 5.52 90.83 .0127 .2033
- 8 2.035 25.78* .01011 17.21 6.31 91.31 .0145
.0162
.2027
.2021
- 4 2.283 25.27* .01015 15.47 7.10 91.79

2.555 24.72* 0.01018 13.94 7.89 92.27 0.0179 0.2015


4 2.852 24.11* .01021 12.58 8.68 92.75 .0197 .2010
St 2.931 23.95* .01022 12.27 8.88 92.88 .0201 .2009
8 .
3.179 23.45* .01024 11.38 9.48 93.24 .0213 .2005
12 3.534 22.73* .01027 10.31 10.28 93.72 .0231 .2000
16 3.923 21.94* .01031 9.359 11.07 94.21 .0248 .1996

20 4.342 21.08* 0.01034 8.519 11.87 94.69 0.0264 0.1991


24 4.801 20.15* .01037 7.760 12.68 95.18 .0281 .1987
28 5.294 19.14* .01041 7.087 13.48 95.66 .0297 .1983
5.830 18.05* .01044 6.481 14.28 96.15 .0314 .1979
32
6.411 16.87* .01048 5.934 15.08 96.63 .0330 .1976
36
40 7.032 15.61* 0.01051 5.447 15.89 97.11 0.0346 0.1972
7.702 14.24* .01055 5.006 16.70 97.60 .0362 .1969
44
8.422 12.78* .01058 4.607 17.52 98.08 .0378 .1966
48
9.199 11.20* .01062 4.245 18.33 98.56 .0394 .1963
52
10.02 9.53* .01066 3.921 19.15 99.05 .0410 .1960
56
60 10.90 7.73* 0.01069 3.636 19.96 99.53 0.0426 0.1958
64 11.85 5.80* .01073 3.356 20.78 100.01 .0442 .1955
68 12.87 3.72* .01077 3.107 21.61 100.49 .0457 .1953
72 13.95 1.53* .01081 2.883 22.43 100.97 .0473 .1950
76 15.09 0.39 .01085 2.679 23.26 101.45 .0489 .1948

80 16.31 1.61 0.01088 2.492 24.09 101.93 0.0504 0.1947


84 17.60 2.90 .01092 2.322 24.93 102.41 .0519 .1945
18.28 3.58 .01094 2.242 25.34 102.65 .0527 .1944
86t .0535 .1943
88 18.97 4.27 .01096 2.165 25.76 102.89
20.43 5.73 .01101 2.020 26.60 103.36 .0550 .1941
92
96 21.97 7.27 .01105 1.887 27.43 103.83 .0565 .1940

100 23.60 8.90 0.01109 1.765 28.27 104.30 0.0580 0.1938


104 25.33 10.63 .01113 1.652 29.12 104.77 .0595 .1937
108 27.15 12.45 .01117 1.548 29.97 105.24 .0610 .1936
112 29.05 14.35 .01122 1.452 30.82 105.71 .0625 .1935
116 31.07 16.37 .01126 1.363 31.67 106.17 .0639 .1934

120 33.20 18.50 0.01130 1.281 32.53 106.63 0.0654 0.1933


124 35.42 20.72 .01135 1.206 33.38 107.09 .0669 .1932
128 37.74 23.04 .01139 1.135 34.24 107.55 .0683 .1931
132 40.23 25.53 .01144 1.068 35.10 108.00 .0698 .1930,
136 42.80 28.10 .01149 1.007 35.97 108.46 .0712 .1929'

140 45.50 30.80 0.01154 0.9505 36.84 108.91 0.0727 0.1929


144 48.35 33.65 .01159 .8970 37.71 109.35 .0741 .1928
148 51.31 36.61 .01163 .8476 38.59 109.80 .0755 .1927
152 54.41 39.71 .01168 .8014 39.46 110.24 .0770 .1927
156 57.65 42.95 .01173 .7581 40.35 110.69 .0784 .1927

100 61.04 46.34 0.01179 0.7176 41.23 111.12 0.0798 0.1926

*Inehei i of mercury below one etu dard atmosphere


t Stand »rd eyele temperatures.

Court et) E. I. du Pont de Nemours It Co.. Inc.


1 1 1 1I

TABLES 475

a,
"5
10
QU5QUJQ
SS^i^ieoco
lOOiOOiO
nnhho P"5©"?Q
oataooa «0©«S©»0
t«Neoio Q "5
25'*
a &•
I I I I I I I I I I I I I I I I I I I I I II

•" ih»N lOCOHOI St-iom


.."SCOON
mMeoeo^
i-H©OCOt«-
t»o
cow
iQwOi 9909C 00

6*
p>i
« P P
*

a 8
Wg a6 8
00
SiOQCOCN
»-l00'*'<-l
O00«0»Q>0
00^f»-ICOiO
10 CO 00©
NOffiWH tNtOCOCNCO
1(5
MiONON QiO
CO 5D CO lO iO kO * *
00000 00000 00000 00
<* CO CO hhhiiO o©cN ifi

00000 CO CN CN CN CN

a T fiiii Tiiii fiiii Tun Ti


(A 8 t»^<c«©oo
iftrtt^COOO
»*wnh
^OOCNCO qaaooh
*«iOiiN coio^cocm
COO A .-if-
>-it-i
coo
ft 7 «•
St 9< S'r-Ii-JcNcN «2^S!'2i2 t" <59S ooo»-<i-i cnn
s COCOCOCO cOCOcOCOCO «SSS
CO CO CO CD CO CO CO t>» t» t*> t»l»

NCOrtaoO
©ioo^o 000C00OCM
•* o ©
•>* •*>
^t^O'Jir-
o«o-hcoi-i
niOOXON
t-eNt»eooo
C4b
•*©
«
it
s "tfeoeogir-j
00 00 00 00 00
s'ooo'oi
co So 00 35 1-
o 06 06
t^t^t^t^t^
£»'
f.'
to cd
t~t»t-t»t-
«a° »o° •* *<»
t-t-
2 M

IS
W"6
•0.3
SOOOO
COCNt-k-i
tN>*»O00^H
©O00t-r»
tfOOHSSO
cOIQiOtP^
>OQ>OpiO
COCQCNCN1-1
»HtO
^HO
3 s
S'cNrHpoJ
eN«NCN»-i
odoiorjieo
hihhhih ci<-<ooo6
1-I1H11
t^«d»OT(Jeo cii-i

3g 1 1 1 1 1 1 1 1 1 1 1 1 1 1 1 1 II

1 _
(OIHHNO
.

lOrtOCOH
.. tNIOOOrHCO
•>*<<NO'«*t» ^cNooicco k>co
ooShh _ ©toeoeoio
* _5-00 .- oto
« 00000
00000 i-hcmcncoco © OOOO OH»IOt«
.
l-H I-H I-H 1-H 1-H
OlM
I-4CN
e
M
X wean*
ts §§o8S COiONCO'* SiPSlSO
(Nt^cococo a^cuso 100
usfc-

E S^^coeoej ©O©eooo
£ 3£^ 00000 ooooo t^ixocpcp
00000 23!^!iriS 00000 1010
00
^tO ^ CN
Or «g«-ii^ 0000
8 gSocSS ^S^ctS CNt^OOlOiO ioqnho
CJCDTflCt- —
, i-i

1 _ 00000
0< t»«oco«
t-COOOi-iCO c»oo»oco^h OOOt^COlQ
£3 NHHHrt
s. £ OOCOt-CNCD h<Dh»h t^O*
88SS
Hi-tHH 8§8SS SS
3« OOOO 00000 00 1-4 1—I 1—I r-» I— *-H 1-H

O '
o* o* ' *
'
* ' '
'

Ill
_l co co«oSe*o oococooto O^t^OO OOOiOOCO <f03
CD Sf-9 3 o 00000 oooooot^t** t»coi«'*>* NHoaN loco
85 ?5c5ncncn cncnScncncn CNCNCNCNCN tNtMC4^H<-i ^i-i
2
SrHCOIOiO >QCQ
it 1-1
«-t
iOO>OcN<-i
t-i -1 CN CO * HjftbW
^iOt-00
iocot-Ori
eoi-iSS!
Tt<t- o-*o0
_ t>CN<00
_
")o«oeo>-H
eo<
CNCO
»-n-«
O O o " " **h «-<»He«eitM' eoeo^'to'cd i>oo

&•
S S^ScSS 88S2§ 8S888 158S8S S!§
I T TTTTT TT7TT Ti 1 1 1 1 1 1 1 1 i i
476 PRINCIPLES OF REFRIGERATION

oaotcftc* ooo«p->*in ooote-^N ©oo<©*n on*io««o


fc » *«eoweo »Snnn «»-i.hihi-i .-•

I I I I I II II I I I 1 I 1 I I I I

58S82
,o <*-*<* t*
&3S&8
weoeSeoN
J2SSS2S
S?£?£?£?r!
ESSS©-
rl^CCC
SgggSi
SS2SSS
^H £_, J-l i-H «-l »H »H »-l »-l 1-H i—« •—i i—< »-l ^H »1
,-( i-l rH i-l ,-1 i-H »-« l-H 1-«
,-J
& o * ' * '
o * * * *
e>' -

d o
£l *oom«o ^«oo»eo«o oeotoooo
no^io^i SSSBSSSif5 SS^fcffiS
«8 s • Shmh
osoo&oP-
QOHNto ©ooooot-
S««S« 9cqc4i-k-h gi-*ig»b- $S^'22'*£S
<»®agr;sa
'3-° «*.
OOO© OOOOO
*''©'"'© OOOOO OOOOO oooooo
"J 2 od o o
n

?.»«>* fi3>»-<-*5 00HCCIO00 ONiflNOl N^ONftH


ggg gigggggg ggggg SEiSfcE: gggggg
8SS3S &S8S8 SS5SSS SESSJS gfcfe^&S
§1
ggg gg ggggg gggsg ggggg gggggg
SSSSS 83338 ©;$$SK 2&85S8 3&S8SS
OOO^ri C4C4NCOCO ^'^'^idiO <6<6<Ot^t^ OOQOOJOJOJOJ

§
e

J
2SS S8852SS §2888 s>sP°<3S §88838
ill SiSis fsiss iisis§ sss^If:
"8

s
ID
Q a-. . 8888S gg§2Sf2 2§g82fefc JS838S 23533888 1
•3«JS>
Hi%% 38888 88833 33388 888888
>opc *OHt» 00 NNIQNg t^Tfl-
~ BN^L
r-it^eg®;
^ iirtooi
EE IX
ST* "J!
51^555565
ooSbc5o>65
** sv rs*
Sc6S6©»S .
t>t>b»F»o6
^»«f>ra H g ^
© © 00 ** "
t*» -
CJ t» r-< 00 US
«© U9 Hi * ©
"* ^<

eoc«icoe<ieo weiciwN ci«eie<iN c*-*<-«-«.-i


i
"3
> t»t-t»oooo 00929922 22222
Sto«p*t» 22 ——2—
_2222 22222 22222
OOOOO 22333
OOOOO 333333
OOOOOO
OOOOO OOOOO
I • t • • • • • • •
gss&ss ssssa ss&sfe sssas 52833$
doJoor^td iQ^cit-id ©*-ie<io»eo ^HQ<Ct«O0 ©©«-H**C«W

I Ml 8SS8S? gSfcSJS SSSSfc SoSSSgSS


gSSScSfeSS
••ssa g'aass asssig sssaa aasass%
eo c ,N ON^tfXOOO '*
SS8SS8 88SSJ8 82233' 2
«

f z - r I I I I
1 1 1 1 1 1 1 1 1 1 1 1 1 I
1
TABLES 477

sassa saass sssssssss 99999


8» -
1

>2 0000
-s&Ss §i^ 2 i &g838 S^sss

0000
n
fa oooJO^igj £5WJ2{S£2 PSSS^Sisi S©«9Sfc
§1
00000 SsasS 00000
Ss^ss 00000 00000
ssiasssssa fassasas
33
n
(OONiCN
mioooON SMiONOI
^rt^a»»Hco
hMiJNOI
<ooooc«^<
*4COiO<0t«
nohhh)
gggSS 38833 338S38 SS2S8SS8
4*

S3 &&83S ssssfe sss&s sssiss


5/ 25 2) 2) 2* to to <© <© <© 3? 3? ^ff 9? *tf <o <o <o <p ^5
« 8

SSSSS S8S38S8 SS3S3 8938$?


©Oi-h-h'^ c< eo eo co * <•*'»«> in <p co r^t^t^oooo

u
3 1H1HOOQ
©JH--i-t00Q0
oooooooSk e>»>ooo«5ao u;moqh _^
<9^*q>moq
3e *SS>8§
t»l— t-OOOO
o»SJ2s5S
aOO>0)0)0 kS^oBq
OhhhN S9S2S
CiTOCOW^ 1

v3
*s
a
a>
Q 9S33S S525585S SSSSS 288892
j 3£- So » oo S SB S888000000 §000888 So 3 c2 3 ao
a
t-n-ieorseo
r r2S2fc?' 4
9!3 o>oot».oQ« S3£=£a o
2 88882 2SS8S ••••• 8SSK8
• •••• 88SSS3
• • • •••••
a
a
"3
Si etnc«Nn
MMHHH eoeoeo^"* loujioceco
i-HHrtiSH
«o«ot*»»*5-
HHHHrt
8>
hmih hh f-H F-l rH 1-H
1-H

OOOOO
3s
OOOOO OOOOO OOOOO s£ri'S'sJisJ
S3 S3 S3 S3 £3:2 22:5 S3

88282 9SS92 83288


SSSfcSS*
£ I^ 2 S2J522 558888 88388 88993
9

1 88889 J22SBS SS9538 83988


8838S 8fc893 99999 SS'SfeS
S* « SS3SS 8$?S88 883S88 99999
1
478 PRINCIPLES OF REFRIGERATION

8S3SS8 S8S8S 8KSgS 8S3S888 SS5SSS SSSSS

l^^^l 2??£?£l gll^l S3333 S2SSS5 3S£S83


d ' * ' * d '.''
,
'
b ''' ,
'

Q ••' •
o o • • • • • • • '

82SSS
i-!?3»?8?28 K^Smm 8S>&?2§
SfeSSSSS bSSSS <e^!' «?"»
x£co{n3o>
n****
H a'S^as <or3ot^co
ooouWoS .-iQ*ir~tc
3SI3;23< !222S;222
SSSSS SSSsS 88SKS3
SSSaS ooooo oos^o^e*
^>$<Pt>N
^>rtS!c»«s io>o3ffio cococococo
99999 99999 "Rrr^ ooooo ooooo
J2 2 ~S. d ' ' ' •
d ' ' ' '
d ' ' ' "

S8S8SSS S8&88 8SSS8 SgSJfeSs fcS5§8g88§ 88883888

SfcgfcS 33SS8 88823 S3S3S!§8 3S28S8 $28582


SSSSS 88888 33338 88888 88885; £o£;88£
&K833 feSS&S 815388 SgSgS gSSSfe 28282
"8
ssasa ss'sissi sisisaa gg&sig gsgssg'g zzs&%
e SOO"* eoco«io»»o
» HOionop ic*(Opk
onhhi
v3
S3! >
S c*
_ e»
1-4
"5
88 «*
co
?3
-4* 2 co c»
^ «o r-
o> 55 «5 §<
oo o> O mnm^i
1->
HrtrtHH hhhmh NNNNN <N<N<NtN<N cNtNCNtNCO CO CO CO CO CO

3SS8S JSSSS28 Sfc2fc8 82SSSS; S^SSS 00>O(N<


(0
SSSSS S8S3SSS8SS 88833 338SS SgSSgi 8888E:
2 COgJgJtMM 00 t^ t^ 00 00 OSONCOlO
COcocOcpto fe33S3S 35$$$ 55ql§Sco
CO
<OUJ3|COCN
CO CO CO CO CO COCOCOCNCN

0000 00 O O
FHHHMH
« 2§§§S SSS88S3 SgJSSS i/5U>«0<0«0
tNtN MMNNN
O0OQC»
IN CM CM
r^ r~
OOOOO f*
*-4 •-<
ssssssssss i-H
sssgs ooooo ooooo t—4 i-H r-H f* f—

o • • ' "
b'*'d"*''b'"' d
-
* " '
d
»>-©§$S«Sr-i8S82 o 8'He«•<*
)

, OO CO 00 CO ttOWHH
88888 fesssfe esses sssss's SJgSgS <© d -t oQ cm
i

.
,
*I9™1 5«t>.8coS oo8§8<3 St-.ooo>.H «ot*0">0 oeoooooo
SSSfeg SSg&S' 3SS888 SSS&d 3J5SS8 S5S8S8
88388 SSSSSSSSSS 88388 8SSSS 8oS§8 «-| r-* i-H rH 1-4
TABLES 479

ft S2222 88388 §8388 §


a * ~
<2

sbsss SSK|S ilsii 1


Issss sssss ggsss §
« o • • • •
d ' ' ' ' d ' ' ' '
©
MS
fa
oxoeogo-* SSS^ErH feSiSKSJ S
SS3S58S 28898 ||fclg I
S8SSS SfeSfeS SfeSSS 8
o
« o '
o'
© '

98ES53 2898S £8882 3


IS* 88888 88888 88833 3
S3 298S8 8838$ ©-£889 8
si
« c
SSSSS J33S388 83388 8
3£ 82823 282S2 82S3S 3
•53* 8553588 88S5S58 888895 3
?
&T S<-ie4co^<
<0t»000>0
e»H«N
»ie5l^ U9« »»ON*
,
SJSSBSS £?
ui
»3 >£*"'

S
«
Q $2888 8K92K 92392 J2
ui
-I 32- fc£gg£ gEiSiSS SttSSS S
a
t t^osN^rj 2£?fc83 §8838 8
• d d © d
a
3
> 3* 88833
HHHMM cowcSeow nwwraw $>
ooooo ooooo ooooo So
»H SSfififi
i-H i-l i-H -H

as d d o o

I
O;0 89398 J338£iS8888g |
1
He

SSSsSto F?r>oo8S aftfifiH §


fa

» S2222 §S3i88 88S88 9
480 PRINCIPLES OF REFRIGERATION

TABLE 16-4. Refrigerant- 1 3 (monochlorotrifluoromethane) Properties of


Liquid and Saturated Vapor

Enthalpy Entropy
Specific volume
Temp Pressure cu ft per lb
datum —40 F datum —40 F
F psia
Btu per lb Btu per lb R
Liquid Vapor Liquid Latent Vapor Liquid Vaporization Vapor
-200 0.4329 0.009466 61.33 -34.551 73.096 38.545 -0.10081 0.28147 0.18066
-190 0.7490 0.009574 36.74 -32.429 72.029 39.600 -0.09313 0.26708 0.17395
-180 1.238 0.009685 22.99 -30.298 70.790 40.672 -0.08575 0.25375 0.16800
-170 1.967 0.009801 14.942 -28.208 69.904 41.696 -0.07858 0.24131 0.16273
-160 3.104 0.009920 9.750 -26.083 68.808 42.725 -0.07213 0.22960 0.15747
-150 4.464 0.01004 6.976 -24.010 67,783 43.773 -0.06491 0.21887 0.15396
-140 6.455 0.01017 4.950 -21.902 46.696 44.794 -0.05844 0.20863 0.15019
-130 9.080 0.01031 3.605 -19.792 65.596 45.804 -0.05209 0.19896 0.14687
-120 12.48 0.01045 2. 631 -17.671 64.473 46.802 -0.04590 0.18980 0.14390
-uo 16.81 0.01060 2.031 -15.527 63.316 47.789 -0.03977 0.18106 0.14129
-100 22.23 0.01075 1.5642 -13.387 62.138 48.751 -0.03286 0.17275 0.13889
- 90 28.89 0.01091 1.2232 -11.241 60.941 49.700 -0.02806 0.16484 0.13678
- 80 36.98 0.01109 0.9689 - 9.052 59.672 50.620 -0.02230 0.15716 0.13486
- 70 46.68 0.01127 0.7766 - 6.843 58.362 51.519 -0.01665 0.14977 0.13312
- 60 58.19 0.01146 0.6289 - 4.604 56.993 52.389 -0.01106 0.14259 0.13153
- 50 71.71 0.01167 0.5139 - 2.320 55.546 53.226 -0.00548 0.13558 0.13009
- 40 87.43 0.01189 0.4234 0.000 54.023 54.023 0.00000 0.12872 0.12872
- 30 105.6 0.01213 0.3512 2.363 52.416 54.779 0.00545 0.12199 0.12744
- 20 126.4 0.01239 0.2930 4.809 50.668 55.477 0.01096 0.11524 0.12620
- 10 150.1 0.01268 0.2454 7.484 48.630 56.113 0.01686 0.10814 0.12500
176.8 0.01299 0.2066 10.052 46.638 56.690 0.02234 0.10146 0.12380
10 206.8 0.01335 0.17443 12.696 44.479 57.175 0.02789 0.09470 0.12259
20 240.4 0.01375 0.14732 15.443 42.100 57.543 0.03351 0.08777 0.12128
30 277.9 0.01422 0.12437 18.247 39.472 57.719 0.03921 0.08061 0.11982
40 319.6 0.01477 0.10455 21.370 36.450 57.820 0.04516 0.07295 0.11811
45 342.2 0.01509 0.09565 22.979 34.769 57.748 0.04826 0.06889 0.11715
50 365.9 0.01546 0.08734 24.651 32.958 57.609 0.05143 0.06466 0.11609
55 390.8 0.01588 0.07945 26.418 30.946 56.364 0.05473 0.06013 0.11486
60 417.0 0.01637 0.07189 28.310 28.677 56.987 0.05824 0.05518 0.11342
65 444.5 0.01696 0.06468 30.322 26.137 56.459 0.06193 0.04981 0.11174
70 473.4 0.01771 0.05767 32.515 23.193 55.708 0.06591 0.40379 0.10970
75 503.7 0.01874 0.05207 35.110 19.382 54.492 0.07059 0.03625 0.10684
80 535.5 0.02047 0.04131 38.527 13.565 52.092 0.07672 0.02513 0.10185
83.93 561.3 0.02772 0.02772 45.271 - 45.271 0.08898 — 0.08898

Courtesy E. I. du Pont de Nemours & Co.. Inc.


TABLE 16-5. Refrigerant-22 (Monochlorodifluoromethane) Properties of
Liquid and Saturated Vapor

Pressors
Liquid, Vapor, Enthalpy, datum -40 F Entropy, datum —40 F
T density •p vol Btu per lb Btu per lb F
f¥>
t
pala lb/cnft cuft/lb Liquid Vapor Liquid Vapor
P<<S
«/
-155 0.19901 29.51* 97.67 188.1 -29.07 86.78 -0.0808 0.2996
-150 0.2605 29.39* 97.33 146.1 -27.79 87.36 -0.0767 0.2952
-145 0.3375 29.23* 96.99 114.5 -26.52 87.94 - .0727 .2912
-140 0.4332 29.04* 96.63 90.61 -25.25 88.53 -0.0687 0.2874
-135 0.5511 28.80* 96.27 72.33 -23.99 89.11 - .0647 .2837
-130 0.6949 28.51* 95.91 58.21 -22.73 89.70 -0.0609
-125 0.2803
0.8692 28.15* 95.53 47.23 -21.47 90.29 - .0571 .2770
-120 1.079 27.72* 95.15 38.60 -20.22 90.88 -0.0534
-115 0.2738
1.329 27.21* 94.76 31.77 -18.98 91.47 - .0497 .2708
-110 1.626 26.61* 94.37 26.33 -17.73 92.07 -0.0461
-105 0.2680
1.976 25.90* 93.97 21.96 -16.48 92.67 - .0425 .2653
-100 2.386 25.06* 93.56 18.43 -15.23 93.27 -0.0390
- 0.2627
95 2.865 24.09* 93.14 15.54 -13.98 93.87 - .0356 .2602
- 90 3.417 22.96* 92.72 13.20 -12.73 94.47 -0.0322 0.2579
- 85 4.055 21.67* 92.29 11.26 -11.47 95.08 - .0288 .2556
- 80 4.787 20.18* 91.85 9.650 -10.22 95.68 -0.0255 0.2535
- 78 5.100 19.55* 91.67 9.086 - 9.72 95.92 - .0242 .2526
- 76 5.430 18.87* 91.49 8.561 - 9.21 96.16 - .0229 .2518
- 74 5.79 18.14* 91.31 8.072 - 8.70 96.40 - .0216 .2510
- 72 6.17 17.37* 91.13 7.616 - 8.20 96.64 - .0203 .2502
- 70 6.57 16.55* 90.95 7.192 - 7.69 96.88 -0.0253 0.2494
- 68 6.99 15.70* 90.77 6.795 - 7.19 97.12 - .0177 .2487
- 66 7.40 14.86* 90.58 6.426 - 6.68 97.36 - .0164 .2479
- 64 7.86 13.93* 90.39 6.079 - 6.17 97.60 - .0151 .2472
- 62 8.35 12.93* 90.21 5.755 - 5.67 97.84 - .0138 .2465
- 60 8.86 11.89* 90.03 5.452 - 5.16 98.08 -0.0126 0.2458
- 58 9.39 10.81* 89.84 5.166 - 4.65 98.32 - .0113 .2451
- 56 9.94 9.69* 89.65 4.900 - 4.13 98.56 - .0100 .2444
- 54 10.51 8.53* 89.46 4.650 - 3.61 98.80 - .0087
- .2438
52 11.11 7.31* 89.27 4.415 - 3.09 99.04 - .0075 .2431
- 50 11.74 6.03* 89.08 4.192 - 2.58 99.28 -0.0062 0.2425
- 48 12.40 4.68* 88.88 3.986 - 2.06 99.52 - .0050
- .2418
-
46 13.09 3.28* 88.68 3.793 - 1.54 99.76 - .0037 .2412
-
44 13.80 1.83* 88.49 3.611 - 1.02 100.00 - .0025 .2406
42 14.54 0.326* 88.30 3.440 - 0.51 100.23 - .0012 .2400
- 40 15.31 0.610 88.10 3.279 0.00 100.46 0.0000 0.2394
- 38 16.12 1.42 87.90 3.126 0.53 100.70 .0013 .2389
- 36 16.97 2.27 87.70 2.981 1.05 100.93 .0025 .2383
- 34 17.85 3.15 87.50 2.844 1.58 101.17 .0037 .2377
- 32 18.77 4.07 87.29 2.713 2.10 101.40 .0050 .2372
- 30 19.72 5.02 87.09 2.590 2.62 101.63 0.0062 0.2367
- 28 20.71 6.01 86.89 2.474 3.15 101.86 .0074 .2361
- 26 21.73 7.03 86.69 2.365 3.69 102.10 .0086
- 24 .2356
22.79 8.09 86.48 2.262 4.22 102.33 .0099 .2351
- 22 23.88 9.18 86.27 2.165 4.75 102.56 .0111 .2346
- 20 25.01 10.31 86.06 2.074 5.28 102.79 0.0123 0.2341
- 18 26.18 11.48 85.85 1.987 5.82 103.02 .0135 .2336
- 16 27.39 12.69 85.64 1.905 6.40 103.25 .0147 .2331
- 14 28.64 13.94 85.43 1.827 6.90 103.48 .0159 .2326
- 12 29.94 15.24 85.21 1.752 7.43 103.70 .0170 .2321
- 10 31.29 16.59 84.99 1.681 7.96 103.92 0.0182 0.2316
- 8 32.69 17.99 84.78 1.613 8.49 104.14 .0194 .2312
- 6 34.14 19.44 84.56 1.549 9.02 104.36 .0205 .2307
- 4 35.64 20.94 84.34 1.488 9.55 104.58 .0217 .2302
- 2 37.19 22.49 84.12 1.429 10.09 104.80 .0228 .2298
38.79 24.09 83.90 1.373 10.63 105.02 0.0240 0.2293
2 40.43 25.73 83.68 1.320 11.17 105.24 0.0251 0.2289
4 42.14 27.44 83.45 1.270 11.70 105.45 .0262 .2285
5 43.02 28.33 83.34 1.246 11.97 105.56 .0268 .2283
6 43.91 29.21 83.23 1.221 12.23 105.66 .0274 .2280
8 45.74 31.04 83.01 1.175 12.76 105.87 .0285 .2276
10 47.63 32.93 82.78 1.130 13.29 106.08 0.0296
12 0.2272
49.58 34.88 82.55 1.088 13.82 106.29 .0307 .2268
14 51.59 36.89 82.32 1.048 14.36 106.50 .0319 .2264
16 53.66 38.96 82.09 1.009 14.90 106.71 .0330 .2260
18 55.79 41.09 81.86 0.9721 15.44 106.92 .0341 .2257

* Inches mercury below one atmosphere.

481
TABLE 16-5 (Continued)

Liquid, Vapor, Enthalpy, datum -40 F Entropy, datum —40 F


Pressure Btu per lb F
Temp density sp vol Btu per lb
F
1 lb/cu ft cu ft/lb Liquid Vapor Liquid Vapor
p«l» psig
1/Vf V
20 57.98 43.28 81.63 0.9369 15.98 107.13 0.0352 0.2253
22 60.23 45.53 81.39 .9032 16.52 107.33 .0364 .2249
24 62.55 47.85 81.16 .8707 17.06 107.53 .0375 .2246
26 64.94 50.24 80.92 .8398 17.61 107.73 .0379 .2242
28 67.40 52.70 80.69 .8100 18.17 107.93 .0398 .2239

30 69.93 55.23 80.45 0.7816 18.74 103.13 0.0409 0.2235


32 72.53 57.83 80.21 .7543 19.32 108.33 .0421 .2232
34 75.21 60.51 79.97 .7283 19.90 108.52 .0433 .2228
36 77.97 63.27 79.73 .7032 20.49 108.71 .0445 .2225
38 80.81 66.11 79.49 .6791 21.09 108.90 .0457 .2222

40 83.72 69.02 79.25 0.6559 21.70 109.09 0.0469 0.2218


42 86.69 71.99 79.00 .6339 22.29 109.27 .0481 .2215
44 89.74 75.04 78.76 .6126 22.90 109.45 .0493 .2211
46 92.88 78.18 78.51 .5922 23.50 109.63 .0505 .2208
48 96.10 81.40 78.26 .5726 24.11 109.80 .0516 .2205

50 99.40 84.70 78.02 0.5537 24.73 109.98 0.0528 0.2201


52 102.8 88.10 77.77 .5355 25.34 110.14 .0540 .2198
54 106.2 91.5 77.51 .5184 25.95 110.30 .0552 .2194
56 109.8 95.1 77.26 .5014 26.58 110.47 .0564 .2191
58 113.5 98.8 77.01 .4849 27.22 110.63 .0576 .2188

60 117.2 102.5 76.75 0.4695 27.83 110.78 0.0588 0.2185


62 121.0 106.3 76.50 .4546 28.46 110.93 .0600 .2181
64 124.9 110.2 76.24 .4403 29.09 111.08 .0612 .2178
66 128.9 114.2 75.98 .4264 29.72 111.22 .0624 .2175
68 133.0 118.3 75.72 .4129 30.35 111.35 .0636 .2172

70 137.2 122.5 75.46 0.4000 30.99 111.49 0.0648 0.2168


72 141.5 126.8 75.20 .3875 31.65 111.63 .0661 .2165
74 145.9 131.2 74.94 .3754 32.29 111.75 .0673 .2162
76 150.4 135.7 74.68 .3638 32.94 111.88 .0684 .2158
78 155.0 140.3 74.41 .3526 33.61 112.01 .0696 .2155

80 159.7 145.0 74.15 0.3417 34.27 112.13 0.0708 0.2151


82 164.5 149.8 73.89 .3313 34.92 112.24 .0720 .2148
84 169.4 154.7 73.63 .3212 35.60 112.36 .0732 .2144
86 174.5 159.8 73.36 .3113 36.28 112.47 .0744 .2140
88 179.6 164.9 73.09 .3019 36.94 112.57 .0756 .2137

90 184.8 170.1 72.81 0.2928 37.61 112.67 0.0768 0.2133


92 190.1 175.4 72.53 .2841 38.28 112.76 .0780 .2130
94 195.6 180.9 72.24 .2755 38.97 112.85 .0792 .2126
96 201.2 186.5 71.95 .2672 39.65 112.93 .0803 .2122
98 206.8 192.1 71.65 .2594 40.32 113.00 .0815 .2119

100 212.6 197.9 71.35 0.2517 40.98 113.06 0.0827 0.2115


102 218.5 203.8 71.05 .2443 41.65 113.12 .0839 .2111
104 224.6 209.9 70.74 .2370 42.32 113.16 .0851 .2107
106 230.7 216.0 70.42 .2301 42.98 113.20 .0862 .2104
108 237.0 222.3 70.11 .2233 43.66 113.24 .0874 .2100

110 243.4 228.7 69.78 0.2167 44.35 113.29 0.0886 0.2096


112 249.9 235.2 69.45 .2104 45.04 113.34 .0898 .2093
114 256.6 241.9 69.12 .2043 45.74 113.38 .0909 .2089
116 263.4 248.7 68.78 .1983 46.44 113.42 .0921 .2085
118 270.3 255.6 68.44 .1926 47.14 113.46 .0933 .2081

120 277.3 262.6 68.10 0.1871 47.85 113.52 0.0945 0.2078


122 384.4 269.7 67.75 .1825 48.6 113.57
124 391.6 276.9 67.40 .1772 49.4 113.61
126 398.8 284.1 67.05 .1724 50.2 113.65
128 306.1 291.4 66.70 .1675 50.8 113.69

130 313.5 298.8 66.35 0.1629 51.5 113.71


132 321.0 306.3 66.00 .1585 52.3 113.74
134 328.7 314.0 65.65 .1538 53.1 113.77
136 336.6 321.9 65.25 .1492 53.8 113.79
138 344.6 329.9 64.85 .1449 54.6 113.80

140 352.7 338.0 64.45 0.1408 55.3 113.81


142 361.0 346.3 64.05 .1368 56.1 113.80
144 369.7 355.0 63.65 .1330 56.9 113.79
146 379.0 364.3 63.25 .1292 57.7 113.78
148 388.8 374.1 62.85 .1253 58.4 113.76

150 399.0 384.3 62.45 0.1216 59.2 113.74


152 407.0 392.3 62.02 .1179 60.0 113.71
154 416.0 401.3 61.58 .1141 60.8 113.67
156 426.0 411.3 61.13 .1105 61.6 113.62
158 436.5 421.8 60.67 .1070 62.5 113.56

160 448.0 433.3 60.20 0.1035 63.5 113.50

Courtesy E. I. du Pont de Nemours & Co., Inc.


482
TABLES 483

TABLE 16-6. Refrigerant-717 (Ammonia) Properties of Liquid and Saturated Vapor

Pressure Liquid, Vapor, Enthalpy,datum -40 F Entropy, datum -40 F


Temp density sp vol Btu per lb
F Btu per lb F
( lb/cuft
pel* euft/lb Liquid Vapor Liquid Vapor
prig
•« »/ V
-105 1.00 27.9 45.71 223.14 -68.5 570.3
-104 1.04
-0.1774 1.6243
27.8 45.67 214.23 -67.5 570.7 — .1744
1.08 27.7 45.63 205.90 -66.4 571.2
1.14 1.6167
27.7 45.59 197.70 -65.4 571.6
1.19 1.6129
27.5 45.55 190.08 -64.3 572.1 - .1655 1.6092
-100 1.24 *27.4 45.51 182.90 -63.3
- 99
572.5 —0.1626 1.6055
1.29 27.3 45.47 175.42 -62.2 572.9 -
- 98 .1597 1.6018
1.35 27.2 45.43 168.48 -61.2 -
- 97 573.4 .1568 1.5982
1.41 27.0 45.40 161.98 -60.1 573.8 - .1539
- 98 1.47 26.9 45.36
1.5945
155.92 -59.1 574.3 - .1510 1.5910
- 95 1.52 •26.8 45.32 150.30 -58.0
- 94 574.7 -0.1481 1.5874
1.S9 26.7 45.28 144.68 -57.0 575.1 -
- 93 .1452 1.5838
1.66 26.5 45.24 139.27 -55.9 575.6 -
- 92 1.73 26.4 45.20 134.06 -54.9 -
.1423 1.5803
- 91
576.0 .1395 1.5768
1.79 26.2 45.16 129.06 -53.8 -
576.5 .1366 1.5734
- 90 1.86 *26.1 45.12 124.28 -52.8
- 89 1.94 26.0
576.9 -0.1338 1.5699
45.08 119.75 -51.7 577.3 -
- 88 2.02 25.8 45.04 115.37 -50.7 -
.1309 1.5665
- 87 577.8 .1281 1.5631
2.11 25.6 45.00 111.31 -49.6 -
- 86 578.2 .1253 1.5597
2.18 25.5 44.96 107.39 -48.6 578.6 - .1225 1.5504
- 85 2.27 25.3 44.92 103.63 -47.5
- 84 2.36
579.1 -0.1197 1.5531
25.1 44.88 99.87 -46.5 579.5 -
- 83 2.46 24.9 44.84
.1169 1.5498
- 82 2.55
96.28 -45.4 579.9 - .1141 1.5465
24.7 44.80 92.86 -44.4 -
- 81 2.65 24.5 44.76
580.4 .1113 1.5432
89.65 -43.3 580.8 - .1085 1.5400
- 80 2.74 •24.3 44.73 86.54 -42.2
- 79 2.85 24.1 44.68 83.50 -41.2
581.2 -0.1057
-
1.5368
- 78 2.96 581.6 .1030 1.5336
23.9 44.64 80.61 -40.1 -
- 77 3.07 23.6 44.60
582.1 .1002 1.5304
- 76 3.19 23.4
77.90 -39.1 582.5 - .0975 1.5273
44.56 75.30 -38.0 582.9 - .0947 1.5242
- 75 3.30 •23.2 44.52 72.80 -37.0
- 74 3.43 22.9
583.3 -0.0920 1.5211
44.48 70.35 -35.9 -
- 73 3.56 22.7 44.44
583.8 .0892 1.5180
- 72 3.69 22.4 44.40
68.01 -34.9 584.2 - .0865 1.5149
- 71 3.82 22.2
65.78 -33.8 584.6 - .0838 1.5119
44.36 63.70 -32.8 585.0 - .0811 1.5089
- 70 3.94 •21.9 44.32 61.65 -31.7
- 69 4.09 585.5 -0.0784 1.5059
21.6 44.28 59.60 -30.7 -
- 68 4.24 21.3 44.24 57.64 -29.6
585.9
-
.0757 1.5029
- 67 4.39 21.0 44.19
586.3 .0730 1.4999
- 66 4.54 20.7
55.78 -28.6 586.7 - .0703 1.4970
44.15 54.01 -27.5 587.1 - .0676 1.4940
- 65 4.69 •20.4 44.11 52.34 -26.5
- 64 4.86 587.5 -0.0650 1.4911
20.1 44.07 50.79 -25.4 -
- 63 5.03 19.6 44.03 49.26
588.0 .0623 1.4883
- 62 5.20 19.3 43.99
-24.4 588.4 - .0596 1.4854
- 47.74 -23.3 588.8 - .0570 1.4826
61 5.38 18.9 43.93 46.23 -22.2 589.2 - .0543 1 .4797
- 60 5.55 •18.6 43.91 44.73 -21.2
- 59 5.74 18.2 43.87 43.37
589.6 -0.0517 1.4769
- 58 5.93 17.8 43.83
-20.1 590.0 - .0490 1.4741
- 42.05 -19.1 590.4 - .0464 1.4713
57 6.13 17.4 43.78 40.79
- 56 6.33 17.0 43.74
-18.0 590.8 - .0438 1.4686
39.56 -17.0 591.2 - .0412 1.4658
- S5 6.54 •16.6 43.70 38.38 -15.9
- 54 6.75 16.2 43.66 37.24
591.6 -0.0386 1.4631
- 53 6.97 15.7 43.62
-14.8 592.1 - .0360 1.4604
- 36.15 -13.8 592.4 - .0334 1.4577
52 7.20 15.3 43.58 35.09
- 51 7.43
-12.7 592.9 .0307 1 .4551
14.8 43.54 34.06 -11.7 593.2 .0281 1.4524
mercury Deiow one standard atmosphere (29.92 in.)
t-
and
i
ctcSa?No
A
«T<1948)'.
^^ " Standards Thermodynamic Properties of Ammonia, Circular
' No. 142 (1923)
484 PRINCIPLES OF REFRIGERATION

TABLE 16-6 (Continued)

Liquid, Vapor, Enthalpy, datum -40 Fl Entropy, datum —40 F


Pressure Btu per lb Btu per lb F
Temp density sp vol
F
1 lb/cu ft cuft/lb Uquid Vapor Liquid Vapor
ssia pais //»/ •« V *« •/ ««

— 50 7.67 14.3 43.49 33.08 -10.6 593.7 -0.0256 1.4497


- 49 7.91 13.8 43.45 32.12 - 9.6 594.0 .0230 1.4471
- 48 8.16 13.3 43.41 31.20 - 8.5 594.4 .0204 1.4445
- 47 8.42 12.8 43.37 30.31 - 7.4 594.9 .0179 1.4419
- 46 8.68 12.2 43.33 29.45 - 6.4 595.2 .0178 1 .4393

- 45 8.95 *11.7 43.28 28.62 - 5.3 595.6 .0127 1.4368


- 44 9.23 11.1 43.24 27.82 - 4.3 596.0 1.4342
- 43 9.51 10.6 43.20 27.04 - 3.2 1.4317
- 42 9.81 10.0 43.16 26.29 - 2.1 1.4292
- 41 10.10 9.3 43.12 25.56 - 1.1 1.4267

- 40 10.41 *8.7 43.08 24.86 0.0 597.6 0.0000 1.4242


- 39 10.72 8.1 43.04 24.18 1.1 598.0 .0025 1.4217
- 38 11.04 7.4 42.99 23.53 2.1 598.3 .0051 1.4193
— 37 11.37 6.8 42.95 22.89 3.2 598.7 .0076 1.4169
- 36 11.71 6.1 42.90 22.27 4.3 599.1 .0101 1.4144

- 35 12.05 •5.4 42.86 21.68 5.3 599.5 0.0126 1.4120


- 34 12.41 4.7 42.82 21.10 6.4 599.9 .0151 1.4096
- 33 12.77 3.9 42.78 20.54 7.4 600.2 .0176 1.4072
— 32 13.14 3.2 42.73 20.00 8.5 600.6 .0201 1.4048
- 31 13.52 2.4 42.69 19.48 9.6 601.0 .0226 1.4025

- 30 13.90 *1.6 42.65 18.97 10.7 601.4 0.0250 1.4001


- 29 14.30 0.8 42.61 18.48 11.7 601.7 .0275 1.3978
- 28 14.71 0.0 42.57 18.00 12.8 602.1 .0300 1.3955
- 27 15.12 0.4 42.54 17.54 13.9 602.5 .0325 1 .3932

- 26 15.55 0.8 42.48 17.09 14.9 602.8 .0350 1.3909

- 25 15.98 1.3 42.44 16.66 16.0 603.2 0.0374 1.3886


- 24 16.42 1.7 42.40 16.24 17.1 603.6 .0399 1.3863
- 23 16.88 2.2 42.35 15.83 18.1 603.9 .0423 1.3840
- 22 17.34 2.6 42.31 15.43 19.2 604.3 .0488 1.3818
- 21 17.81 3.1 42.26 15.05 20.3 604.6 .0472 1.3796

- 20 18.30 3.6 42.22 14.68 21.4 605.0 0.0497 1.3774


- 19 18.79 4.1 42.18 14.32 22.4 605.3 .0521 1.3752
- 18 19.30 4.6 42.13 13.97 23.5 605.7 .0545 1.3729
- 17 19.81 5.1 42.09 13.62 24.6 606.1 .0570 1.3708
- 16 20.34 5.6 42.04 13.29 25.6 606.4 .0594 1.3686

- 15 20.88 6.2 42.00 12.97 26.7 606.7 0.0618 1.3664


- 14 21.43 6.7 41.96 12.66 27.8 607.1 .0642 1.3643
- 13 21.99 7.8 41.91 12.36 28.9 607.5 .0666 1.3621
- 12 22.56 7.9 41.87 12.06 30.0 607.8 SS'P 1.3600
- 11 23.15 8.5 41.82 11.78 31.0 608.1 .0714 1.3579

- 10 23.74 9.0 41.78 11.50 32.1 608.5 0.0738 1.3558


- 9 24.35 9.7 41.74 11.23 33.2 608.8 .0762 1.3537
- 8 24.97 10.3 41.69 10.97 34.3 609.2 .0768 1.3516
- 7 25.61 10.9 41.65 10.71 35.4 609.5 .0809 1.3495
- 6 26.26 11.6 41.60 10.47 36.4 609.8 .0833 1.3474

- 5 26.92 12.2 41.56 10.23 37.5 610.1 0.0857 1.3454


- 4 27.59 12.9 41.52 9.991 38.6 610.5 .0880 1.3433
- 3 28.28 13.6 41.47 9.763 39.7 610.8 .0904 1.3413
- 2 28.98 14.3 41.43 9.541 40.7 611.1 .0928 1.3393
- 1 29.69 15.0 41.38 9.326 41.8 611.4 -.0951 1.3372

30.42 15.7 41.34 9.116 42.9 611.8 0.0975 1.3352

1 31.16 16.5 41.29 8.912 44.0 612.1 0.0998 1.3332


2 31.92 17.2 41.25 8.714 45.1 612.4 .1022 1.3312
3 32.69 18.0 41.20 8.521 46.2 612.7 .1045 1.3292
4 33.47 18.8 41.16 8.333 47.2 613.0 .1069 1.3273
5 34.27 19.6 41.11 8.150 48.3 613.3 .1092 1.3253

6 35.09 20.4 41.07 7.971 49.4 613.6 0.1115 1.3234


7 35.92 21.2 41.01 7.798 50.5 613.9 .1138 1.3214
g 36.77 22.1 40.98 8.629 51.6 614.3 .1162 1.3195
9 37.63 22.9 40.93 7.464 52.7 614.6 .1185 1.3176
10 38.51 23.8 40.89 7.304 53.8 614.9 .1208 1.3157

11 39.40 24.7 40.84 7.148 54.9 615.2 0.1231 1.3137


12 40.31 25.6 40.80 6.996 56.0 615.5 .1254 1.3118
13 41.24 26.5 40.75 6.847 57.1 615.8 .1277 1.3099
14 42.18 27.5 40.71 6.703 58.2 616.1 .1300 1.3081
15 43.14 28.4 40.66 6.562 59.2 616.3 .1323 1.3062

'
Inches of mercury below one standard atmosphere (29.92 in.).
TABLES 485

TABLE 16-6 (Continued)

Pressure
Liquid, Vapor, Enthalpy, datum —40 F Entropy, datum —40 F
Temp density sp vol Btuperlb Btu per lb F
F
1 lb/cttft cuft/lb Uauid Vapor Uauid Vapor
Pita P*>8 1/v, A,

16 44.12 29.4 40.61 6.425 60.3 616.6 0.1346 1.3043


17 45.12 30.4 40.57 6.291 61.4 616.9 .1369 1.3025
18 46.13 31.4 40.52 6.161 62.5 617.2 .1392 1.3006
19 47.16 32.5 40.48 6.034 63.6 617.5 .1415 1.2988
20 48.21 33.5 40.43 5.910 64.7 617.8 .1437 1.2969

21 49.28 34.6 40.38 5.789 65.8 618.0 0.1460 1.2951


22 50.36 35.7 40.34 5.671 66.9 618.3 .1483 1.2933
23 51.47 36.8 40.29 5.556 68.0 618.6 .1505 1.2951
24 52.59 37.9 40.25 5.443 69.1 618.9 .1528 1.2897
25 53.78 39.0 40.20 5.334 70.2 619.1 .1551 1.2879

26 54.90 40.2 40.15 5.227 71.3 619.4 0.1573 1.2861


27 56.08 41.4 40.10 5.123 72.4 619.7 .1596 1.2843
28 57.28 42.6 40.06 5.021 73.5 619.9 .1618 1.2825
29 58.50 43.8 40.01 4.922 74.6 620.2 .1641 1.2808
30 59.74 45.0 39.96 4.825 75.7 620.5 .1663 1.2790

31 61.00 46.3 39.91 4.730 76.8 620.7 0.1686 1.2773


32 62.29 47.6 39.86 4.637 77.9 621.0 .1708 1.2755
33 63.59 48.9 39.82 4.547 79.0 621.2 .1730 1.2738
34 64.91 50.2 39.77 4.459 80.1 621.5 .1753 1.2731
35 66.26 52.6 39.72 4.373 81.2 621.7 .1775 1.2704

36 67.63 52.9 39.67 4.289 82.3 622.0 0.1797 1.2686


37 69.02 54.3 39.63 4.207 83.4 622.2 .1819 1.2669
38 70.43 55.7 39.58 4.126 84.6 622.5 .1841 1.2652
39 71.87 57.2 39.54 4.048 85.7 622.7 .1863 1.2635
40 73.32 58.6 39.49 3.971 86.8 623.0 .1885 1.2618

41 74.80 60.1 39.44 3.897 87.9 623.2 0.1908 1.2602


42 76.31 61.6 39.39 3.823 89.0 623.4 .1930 1.2585
43 77.83 63.1 39.34 3.752 90.1 623.7 .1952 1.2568
44 79.38 64.7 39.29 3.682 91.2 623.9 .1974 1.2552
45 80.96 66.3 39.24 3.614 92.3 624.1 .1996 1.2535

46 82.55 67.9 39.19 3.547 93.5 624.4 0.2018 1.2519


47 84.18 69.5 39.14 3.481 94.6 624.6 .2040 1.2502
48 85.82 71.1 39.10 3.418 95.7 624.8 .2062 1.2486
49 87.49 72.8 39.05 3.355 96.8 625.0 .2083 1.2469
50 89.19 74.5 39.00 3.294 97.9 625.2 .2105 1.2453

51 90.91 76.2 38.95 3.234 99.1 625.5 0.2127 1.2437


52 92.66 78.0 38.90 3.176 100.2 625.7 .2149 1.2421
53 94.43 79.7 38.85 3.119 101.3 625.9 .2171 1.2405
54 96.23 81.5 38.80 3.063 102.4 626.1 .2192 1.2389
55 98.06 83.4 38.75 3.008 103.5 626.3 .3214 1.2373

56 99.91 85.2 38.70 2.954 104.7 626.5 0.2236 1.2357


57 101.8 87.1 38.65 2.902 105.8 626.7 .2257 1.2341
58 103.7 89.0 38.60 2.851 106.9 626.9 .2279 1.2325
59 105.6 90.9 38.55 2.800 108.1 627.1 .2301 1.2310
60 107.6 92.9 38.50 2.751 109.2 627.3 .2322 1.2294

61 109.6 94.9 38.45 2.703 110.3 627.5 0.2344 1.2278


62 111.6 96.9 38.40 2.656 111.5 627.7 .2365 1.2262
63 113.6 98.9 38.35 2.610 112.6 627.9 .2387 1.2247
64 115.7 101.0 38.30 2,565 113.7 628.0 .2408 1.2231
65 117.8 103.1 38.25 2.520 114.8 628.2 .2430 1.2216
66 120.0 105.3 38.20 2.477 116.0 628.4 0.2451 1.2201
67 122.1 107.4 38.15 2.435 117.1 628.6 .2473 1.2186
68 124.3 109.6 38.10 2.393 118.3 628.8 .2494 1.2170
69 126.5 111.8 38.05 2.352 119.4 628.9 .2515 1.2155
70 128.8 114.1 38.00 2.312 120.5 629.1 .2537 1.2140
71 131.1 116.4 37.95 2.273 121.7 629.3 0.2558 1.2125
72 133.4 118.7 37.90 2.235 122.8 629.4 .2579 1.2110
73 135.7 121.0 37.84 2.197 124.0 629.6 .2601 1.2095
74 138.1 123.4 37.79 2.161 125.1 629.8 .2622 1.2080
75 140.5 125.8 37.74 2.125 126.2 629.9 .2643 1.2065

76 143.0 128.3 37.69 2.089 127.4 630.1 0.2664 1.2050


77 145.4 130.7 37.64 2.055 128.5 630.2 .2685 1.2035
78 147.9 133.2 37.58 2.021 129.7 630.4 .2706 1.2020
79 150.5 135.8 37.53 1.988 130.8 630.5 .2728 1.2006
80 153.0 138.3 1
37.48 1.955 132.0 630.7 .2749 1.1991
486 PRINCIPLES OF REFRIGERATION

TABLE 16-6 (Continued)

Prosstav
Liquid, Vapor, Enthalpy, datum -40 F Entropy, datum —40 F
Temp density sp vol Btu per lb Btu per lb F
F
t lb/cu ft cu ft/lb Liquid Vapor Liquid Vapor
psla psig A,
lit, «/

81 155.6 140.9 37.43 1.923 133.1 630.8 0.2769 1.1976


82 158.3 143.6 37.37 1.892 134.3 631.0 .2791 1.1962
83 161.0 146.3 37.32 1.861 135.4 631.1 .2812 1.1947
84 163.6 149.0 37.26 1.831 136.6 631.3 .2833 1.1933
85 166.4 151.7 37.21 1.801 137.8 631.4 .2854 1.1918

80 169.2 154.5 37.16 1.772 138.9 631.5 0.2875 1.1904


87 172.0 157.3 37.11 1.744 140.1 631.7 .2895 1.1889
88 174.8 160.1 37.05 1.716 141.2 631.8 .2917 1.1875
89 177.7 163.0 37.00 1.688 142.4 631.9 .2937 1.1860
90 180.6 165.9 36.95 1.661 143.5 632.0 .2958 1.1846

91 183.6 168.9 36.89 1.635 144.7 632.1 0.2979 1.1832


92 186.6 171.9 36.84 1.609 145.8 632.3 .3000 1.1818
93 189.6 174.9 36.78 1.584 147.0 632.3 .3021 1.1804
94 192.7 178.0 36.73 1.559 148.2 632.5 .3041 1.1789
95 195.8 181.1 36.67 1.534 149.4 632.6 .3062 1.1775

96 198.9 184.2 36.62 1.510 150.5 632.6 0.3083 1.1761


97 202.1 187.4 36.56 1.487 151.7 632.8 .3104 1.1747
98 205.3 190.6 36.51 1.464 152.9 632.9 .3125 1.1733
99 208.6 193.9 36.45 1.441 154.0 632.9 .3145 1.1719
100 211.9 197.2 36.40 1.419 155.2 633.0 .3166 1.1705

101 215.2 200.5 36.34 1.397 156.4 633.1 0.3187 1.1691


. 102 218.6 203.9 36.29 1.375 157.6 633.2 .3207 1.1677
103 222.0 207.3 36.23 1.354 158.7 633.3 .3228 1.1663
104 224.4 210.7 36.18 1.334 159.9 633.4 .3248 1.1649
105 228.9 214.2 36.12 1.313 161.1 633.4 .3269 1.1635

106 232.5 217.8 36.06 1.293 162.3 633.5 0.3289 1.1621


107 236.0 221.3 36.01 1.274 163.5 633.6 .3310 1.1607
108 239.7 225.0 35.95 1.254 164.6 633.6 .3330 1.1593
109 243.3 228.6 35.90 1.235 165.8 633.7 .3331 1.1580
110 247.0 232.3 35.84 1.217 167.0 633.7 .3372 1.1566

111 250.8 236.1 35.78 1.198 168.2 633.8 0.3392 1.1552


112 354.5 239.8 35.72 1.180 169.4 633.8 .3413 1.1538
113 258.4 243.7 35.67 1.163 170.6 633.9 .3433 1.1524
114 262.2 247.5 35.61 1.145 171.8 633.9 .3453 1.1510
115 266.2 251.5 35.55 1.128 173.0 633.9 .3474 1.1497

116 270.1 255.4 35.49 1.112 174.2 634.0 0.3495 1.1483


117 274.1 259.4 35.43 1.095 175.4 634.0 .3515 1.1469
118 278.2 263.5 35.38 1.079 176.6 634.0 .3515 1.1455
119 282.3 267.6 35.32 1.063 177.8 634.0 .3556 1.1441
120 286.4 271.7 35.26 1.047 179.0 634.0 .3576 1.1427

121 290.6 275.9 35.20 1.032 180.2 634.0 0.3597 1.1414


122 294.8 280.1 35.14 1.017 181.4 634.0 .3618 1.1400
123 299.1 284.4 35.08 1.002 182.6 634.0 .3638 1.1386
124 303.4 288.7 35.02 0.987 183.9 634.0 .3659 1.1372
125 307.8 293.1 34.96 0.973 185.1 634.0 .3679 1.1358
TABLES 487

TABLE 16-7. Relative Safety of Refrigerants


Toxicity
Lethal or Serious Injury*

Products of or Explosive
Decomposition Limits of
ASAB9 Nafl Refrigerant in Air by Flame Concen-
Safety Fire
Under- Duration Duration in Air
Code
Group writers of of A
Classifi- Group Exposure % lb/1000 Exposure by %
Refrigerant cation Number (hr) by Vol cuft (min) Vol 4 by Vol

4.9-15.0
Methane 31 +5
1 61 Nonflam.
R-14 l
3.0-25.0
Ethylene 31 +5
8 0.0025 Nonflam.
Nitrous oxide
1 61 Nonflam.
R-13 l

5 2 37.4-51.7 3.3-10.6
Ethane 3
5 itol 29-30 33.2-34.3 Nonflam.
Carbon dioxide 1

Kulene-131 l
1 61 Nonflam.
5 2 37.5-51.7 42.4-58.5 2.3-7.3
Propane 3
5A 16 1.0 Nonflam.*
R-22 1

2 0.5-0.6 0.221-0.256 16.0-25.0


Ammonia 2 i
Carrene-7 1 5A 2 19.4-20.3 50.2-52.2 25 1.1 Nonflam.
6 2 28.5-30.4 89.6-95.7 20 1.0 Nonflam.
R-12 1
4 2 2-2.5 2.62-3.28 30 2.4 8.1-17.2
Methyl chloride 2
3 +5 1.8-8.4
Isobutane

2
i
It 0.7 1.165 Nonflam.
Sulfur dioxide 1
5 2 37.5-51.7 1.6-6.5
Butane 3
6 2 20.1-21.5 90.5-96.8 15 1.0 Nonflam.
R-114 1

i 10.2 27.1 Nonflam.


R-21 1
2 4 1 4.0 6.72 18 2.0 3.7-12.0
Ethyl chloride

R-ll 1 5 2 10 35.7 5 1.0 Nonflam.


2 3 2-2.5 3.12-3.9 4.5-20.0
Methyl formate 1

Methylene chloride 1 4A i 5.1-5.3 11.25-11.7 20 1.0 Nonflam.


R-ll 3 1 4 1 4.8-5.2 23.3-25.2 16 1.2 Nonflam.
2 4 1 2-2.5 5.04-6.3 5 2.1 5.6-11.4
Dichlorethylene

1 Unofficial.
2
Very slightly flammable, but for practical purposes considered nonflammable.
* To guinea pigs.
* Initial concentration. .
.

From the ASRE Data Book, Design Volume, 1957-58 Edition, by permission of the American Society of Heating
Refrigerating, and Air-Conditioning Engineers.
488 PRINCIPLES OF REFRIGERATION

c
o
N
N
<-«<•»
VINO
hHOinO
»mnOO NOh
N00O>
CO «N«
00 •* ©
W>
O O N
1e .••CSC
4>« o u n=
.doo.
«~ «*" _i _i _« n _• N r* .-« .-« .-«
00
_ ,_, N 00 00
N
« !>
O <-*
^» » OWh
Q 0DO«hO
^^"i <"ONOh
OON-<lfl
omooooot
M*W»-tNOi
OONWO ^p
N«»
n
o>
<*>no
COCO -h
c frtlift .1 n *o m^^< ^«^t^j»^i^i i^^^^^t ^inniow «* tp tp^tj*
o
323* OxONN
O Sin CO O 00« rt-MNp NcOQQOQco W«<OlflN
h^qwn N«
no O
o fOOtn
ooaoo ooooNin © noon
**
s o> <*»000© o*oo>o»o> oootooi ooca o\o>^
u. Jeu:-
Wo. n ^ <-4 ~*»Jw ^'^©'J^* o'^'ooo hoooo do d do~
S
3 .si a* co oo o o» oo oo^&oe ottooNio w^eoon o^co \N won
N
oo
-» O © N »0 «rf>rO^*10* 1ft <-4 dO d d •* 00 « in in ^«
lft*
»-ti-ti-i.-ieMP*«*i<*> t*
O ~ Nd
••* lft
hon
© -Htn^«
'

.-«

c
n
•*>
in
°?
n ^tSJ'S
c 1*"!*
OOIftlftNCH
^n-hiO *
NftN^OO nhOkin MOO
m o» NfOONS ° OOO
***& ^ ^^-^
do ««
1

«<f"tf
1

a-3 *">
.-*
^
oon nhddhm ^ddind
* Vo»nnn
r4Cft*H
oowp;
^CO*
n
CM
rod^
ON*
CO > N-*
N
8. B.S ox n '~;ooonoi nn»o© oxaox in mow
Q0
.
,ft ©o<omo>i
ono o" oo -i in
1

?OiNNN diowdd
It
> !* aS ^
«£ N
r* <ONO ^ooddco
tfi- ««hI>00 d--o OO^^^IO N««l O m ^ oo* ift

'a a*>
g
N
J o ^
-*
»ONO
00
**> <<
VIONSN
oao^NOi (OOiHOO
nN^'Oin
^•^Ift©*
<o<N*oool
oooox
om<4*
«o
r*
mooo.
r^*-t<-i
t ,^
I> ro <*• mo-* HNOrtrt rt^^o-4 ^»CM«-'0N f<*Hrt co -«P40
in o W u^"*® OfJ^HH NVI^OO »-«^«coiOIO N")<0 !>• cot*co
W 00 >2 JG 01 ?* -0*h-i
NOMOIA P'-'i-«^'00* «4 0<NV)N
* 00 ^« CM O
OX'O^ fO ^f-H»n
P* lO 00 Ml CSt*
IM ^ lft<-i^*ON
^t^H^CO^* HN OOC«CO
HXrt Ift .-» OS N
O
8*
pH —I

a 2 !35!2 o«^mn oo ^ co co n«coOth tpoo n Nino


CD 00 O "1^^ l>00t*-<0 «U)OhO -^OvOO^N NNO O -tfOO
in.

Ift N «OcOcO
«*> co ^ ^* ^* 1* ^^lddlO W)IOIOI>'d t*0000 00 00 -4 to
8
e
id 00 ** « °^°. tft00V>-4*f4 00 00 00 OS OW^OO OOtlft ON 00 NO
E 00 +*-"
* M **n! fcR pa^cofo d^Hio'd nonoco «-<^, o* co inooo"
k «> §j §J52 **" °^* OO^IO^N , N^^^« -• ^hNN

I OOco
-.^ ***.—. NcoO-^00
,~J ,°. .
*#*»
^ incoOXOiN *****
hNV)<hO ***
cOO>"0
*
o\
#ft*
roOt-
£ 04-i-m-o>^
CON
S!t*^!!2t
N^CO
i>ood'0^ <ocoioo"«o dddco^j" *«i> i> ooda*

OO^TQOco °. w*'^ ,
f
^ uioooo noiaoh ooooo oooo o ooo
CON ^00 SSnfcfe -idlN^N
o« OOfN^NQ
oco^ooin
'^Q'^tfJlO
t-ujo-Hco S*QP
Poot>
« m" 00 «d
onn
SIN
r»^?coo KSfc?5S? in co oo o>
Olftt-r* OWOIOO 3l>*0>0lft OMnOiA ^t*fc*00O oroo * NNN
•"«•-* r+ w*~* CO
a i
O00O00--4 **I S
C
^
,
'? OhO ^r OO^h
M
*I
O00^«i-*N <«N00OO m<«)00*f

a
aSM'S
Nm SSSSS NNNNN NNcococo N«Oco*OcO 2«8
SSPSS! SS228 SSS'SS
«-<»h
^«W>'* * Sii
£ *WN
I I 1

OOOO ® €J°1 ,°, 0000>00 OOOMOO OONOO IOCOO O OroO


NNN 00 wft«NO o N^Nwioo' NNN ^ O* ^ N
o.Ht»N SS9SSS o in in ^dcoV-H
*
2mSfc** «no>o<-i co^-^^o ^n^ co nnco
, s

NfONN T*? Nf? mN^NN HNhhn ^NN-^^ -^ ~< ^


2 MM 'I II
.

1 II M I Mill II II I Ml I M
^h

& OONOin OWco^N N^OOO OOcoOcocO ^"OOOON N">N O OOO


oo* wn fc^Sfc^S
oo
r-iinOXinNN^S^
op*
10
*-ioooo-H
**NNN oo-vo**
h*4^N»)
oo*oo^^^«
co-tflftON
d«*in
00*0 n
— oo'nn
hOOh
IONHi>4r4 ... ** H ^H.ftC"*

I I I I II I I I
.

I
.

I I I I I I I

> S383S S98SS SSSSo. $2882 SSS8S SSS % SS3


S2SSS SS?'«!S J858S SSSSS S3S$^ 8SS fe SSS
I

H
s.
E
o ouuo Rwooo UUK-K) UUOQUU ooooo ooo o OOtU
u
•« © m ^ 5 *> 3 9^
t " *
Il§^
CO *a»««
«smhm ss-s-sir
K«o«« 15 IH Hill
£«5«« aa«s«
lilti
««ww« s«a « ill*
ah^
fgm
TABLES 489

TABLE 19-1. Dimensions and Physical Data—Copper, Brass, or Seamless


Steel Tubing

TransTerse Lin. Ft. of Pipe Length


Area Per Square Length Length of Pipe
Shw Nomina Enema
1 Internal Thick- Square Inches
in Inches Pipe
1
Foot of o/Rpe ofHpe in Ft. Weight
Dia. Type Dia. Occupy Per
OD Sin Inches Inches Metal
in Ft.
Contain Contain ing 1-Cu Foot
Inches Inchef External Internal External Internal ing 1-Cu ing 1- Ft. of Pounds
Sorfaoe Surface Foot Gallon Space
X X .250 .190 .030" .049 .028 15.25 20.00 5090.0 681.0 2940.0 .080

X X .375 K .311 .032** .110 .076 10.45 12.29 1895.0 253.0 1310.0 /.134

X H .500 K .402 .049 .196 .127 7.65 9.50 1135.0 151.0 735.6 .269
L .430 .035 .144 8.89 1001.0 133.5 .198

H X .625 K .527 .049 .306 .218 6.10 7.25 660.5 88.0 470.0 .344
L .545 .040 .232 7.00 621.0 82.6 .284

X X .750 K .652 .049 .539 .333 5.10 5.85 432.5 S7.5 267.0 .418
L .660 .042 .341 S.79 422.0 56.1 .362

X X .875 K .745 .065 .598 .435 4.36 5.12 331.0 44.0 240.5 .641
L .785 .045 .482 4.86 299.0 39.8 .454

IX 1 1.125 K .995 .065 .989 .775 3.39 3.84 186.0 24.7 145.9 .839
L 1.025 .050 .825 3.72 174.7 23.2 .653

IK IK 1.375 K 1.245 .065 1.481 1.215 2.78 3.06 118.9 15.8 97.3 1.04
L 1.265 .055 1.255 3.02 115.0 15.3 .882

\H IX 1.625 K 1.481 .072 2.070 1.725 2.35 2.57 83.5 11.1 69.6 1.36
L 1.505 .060 1.771 2.54 81.4 10.8 1.14

2X 2 2.125 K 1.959 .083 3.540 3.000 1.80 1.95 48.0 6.39 40.6 2.06
L 1.985 .070 3.090 1.92 46.6 6.20 1.75

XX iX 2.625 K 2.435 .095 5.400 4.620 1.45 1.57 31.2 4.15 27.6 2.92
L 2.465 .080 4.760 1.55 30.2 4.01 2.48

3H 3 3.125 K 2.907 .109 7.750 6.620 1.22 1.31 21.8 2.90 18.6 4.00
L 2.945 .090 6.810 1.29 21.1 2.80 3.33

m *X 3.625 K
L
3.385
3.425
.120 10.350 8.96 1.05 1.13 16.1 2.14 13.9 5.12
.100 9.21 1.11 15.6 2.07 4.29

*H 4 4.125 K 3.857 .134 13.320 11.620 .93 .99 12.4 1.65 7.50 6.51
L 3.905 .110 11.920 .98 12.1 1.61 5.38

SK S 5.125 K 4.805 .160 20.530 18.100 .7S .79 7.95 1.06 7.04 9.67
L 4.875 .125 18.600 .78 7.75 1.03 7.61

6X 6 6.125 K 5.741 .192 89.400 25.80 .62 .67 5.59 .74 4.90 13.87
L 5.845 .140 86.61 .66 5.41 .72 10.20

»H 8 8.125 K 7.583 .271 il.700 14.80 .47 .50 3.22 .43 2.78 .5.90
L 7.725 .200 <16.60 .49 3.09 .41 1 9.29
<

490 PRINCIPLES OF REFRIGERATION

NMW)Ht*JO^OCOO
ro ^ ~* fO *-t t~-
l-H
W rH PO O

*-* *-i CS -* VO O t—

oo oooooooo
00 00 00 00 00
"S CO
> ^to «|oo wfoo -"I* wW **» «** >-fl 08

CO
He.
JO
*rt rt MN c^ ^

^ ro (S
SSSSwo^oooooo CO 00 ^
^H CM lO ~H
co *s>

O
*

CNJ

o 00 CN
oooooo
O oo
00o
„rHPOOO>0«SO\N rt >-H CO
©O
00 00 co CS 00
O On
O
~« CO IT)

r* *o to
S8SSoi;ow)ooooo O CM CO © CM - CM 00 oooooo
rt N t»< O0 lO <(<
^H CS

cs vo *o
^SSSw^oo^ooooo ^Ovt^-OOT^vDt^OOOOOOO
dnfONrHrH^HiflOOlfliOOO
-H CS

^h 00 O O vO O
>"<-<^ pot2t:2
oo^-tfoioootH OiHNIOCO^i'fOOiOCO'HNO
fC Tj< t^ O 00 a
i

^H »"H CS

SSSSSS-nooooo WJVON00N *—— — —*—.—»


donfJiOONON^N'OOOO
rt CM HHWJOO
O ^ CS CS *© o 'O
^'OMiON'<) t yOW^OOOO
«1 (S f) ^* w f) ^ (N ^t^MiOlON

r^ cs *—^ ^ ^* _ _ »
c*i

d © d ^ n ^ oo 2 5 g ^ £ *H CS T*

(S iO 00 •«* Ov n^** **. »


w ^-* w HrttOW)

?3qw©ww©OCM 12222°
^t ^* ^ ^ < 1—4

^^T^HCS NrOfO^iOVO CO
ft. HH«NN")*lO'0«ON
TABLES 491

•I

dH31 NOIlVUniVS Nl 30NVH3 ONIQNOdS3UUO0

d(l\7)SSOn dW31 1N31VAI0D3

<

o
PS

Or*

D ft.
*
H O Q
< a!

S as II
w 5 s'l
en <§<§
W SP
* 8 3£
^1
3 q
M
,?<?
*3
492 PRINCIPLES OF REFRIGERATION

M
c
to

ra
f «OCStJ*iOOOOOOO
U Receiver
System

<"S
IB
P
<
s
e u.
4* to
3 ^ *-» «-«
s =>100fpm

a. CM«*5*"»CMO\*^t*»*'0^»'HfO
M
Receiver
Velocity

a. Condenser

*"H <-H »-t T-H »-* «-H


< o
V c
M 1
a.
3
(A <U

L.
SOOQOOOOOOO
Ooooooooooo^^^-^Tti
£. Size

M i; TypeL Copper
OD
He* inM t>tto
HHHN
i-tn e«|« >o|t« h» m|» Hw >o*» H«
CM'f) fO ^
X x; Line

L.
tt.
c
a a

y wiflf<)ooioo>w*e(oooo ^O^^^CMOOOOOOOOO
(A £ e
1
is Nioaooyooaoo^ioNNOo
f? c
*• 3 ^ fO IO
#
c a > n
<
> ^fO^t^vOOfOOOOOO CMiOmxO»00000000000
14
£ = £
,j

CO
HNIOOOO^OIOOICOO-*
H ^ N\0 O N »0 lO 't N^OONOhm^moOOOOO
rtNlO00Tj«O»Nl«^HlO00
(I .5T 5 -hCSNIOOOOOihO
M 111,2 < u
»
^ (V) to
I S § i
*9
©«-iO\00©^<©©©©©0 OihoOhhonoOOQOOOO
£ «1 <

3 CO 1
N«*N»0^«^NaQMOOOO
L.
c5 «-i CM TO tO
-2 *«
©</! O O O CM 00 »0
-<lNTt<

Hi
a.
iO«H0000O\0\MHHIOOO M IO N r* In m a ^0*0000 O ©

2 ©*-i'*-HlOO»OfO00<s'5O^0»O iHM^O\^WiOO*O^NNvOQ>0
rttNr}<M^IOOi4>OiOt>
o 3i
w CM ©\ **
^ »— •-* .^t

a CM
o io O CO fO *o
<4<r«>0©©cMCMfOiooo©o 3jr-cMoo^©NO\.o©e^©©©©©
01 6 ©*-HPO^o©CN^^o^rM**ON^>o»o
e L.
Ik * fc.
-^ CM
-I S

e
<l
e f » l^ ^
)0
lO^N^OOOiOOHNvOOOOO
Tl<
-j
<s ©•^CM^«t*ro-^00>OvO00^^©©
b e
II

CM «# »© 00 *0
« 1u Ah *-<

M c c < *-• CM Ti" O0 fO

* CM
"4
3 3 CM -* 00 t-
*-.
POOONONOMOihO\NN <p* O ro ©
O0 00 «-h

ae *>
for*^ooa«N(N'-iooo^oooo
3 HN^IOO ©©»-ieo^oo^io»o©'-ioovo©r*
«NrOtf)Oi^^V)N
w ro iO 00
of
C*
" c foooo5csoofc»o\rt & •*# m o © © o*
IU
O
0©*-«CM»000^-«fO«M»^'^'C>'<4'«0
g 'A ^^ CM lO

< M &322229 OO0O0o00O


H *« 1
^ . »"» ^* ^f "•T """I* ^* *4* ^f ^* ^* ^J* **lf ^* ^* ^* ri
^^
B£ & *|n wt* M» H« nfc» "*• "+• taw r^m «N *+• H* Xco
TypeL Copper
< OD

|2
TABLES 493

I
I
I

I
O oo 10 «• eu
<M3i Noavtmjys ni bonvho ONKiNOdsauuoo
d(!V)SS01 dl/M31 lN3nVAinD3

S 5

<
•2.

X v a
o
3 ^^^ **>i *-*

r i
a •»
8 .
- a 3 tq
i s
1-8 t
J
I •a

5
x S
£ a^ <*> 00 **^ t^ *^

5 .8 | •5 'IS »-«'«««© o
a a, I -5 I
3 £T i
S **"
"8 .8 u *• R
a
£<
§ 8*^
?
lie £
in
(5
is
t
X
3 '£> 3-£ S'
•f a .5 «i »i *«
to
III > k « 8 a fc

5
**. <. *£
s? <1
I
494 PRINCIPLES OF REFRIGERATION

^^ «
w
e
M
C 8 •s
a
.c E^ 3
U ts »
^ \a O ^ 0\ 4 1
Ov «-i ©
1 S "3
a
a
< E
< ™" Li.

« .8
DO q
J M to f
in c fpm.
JC ae 3
.8
M 8 Receiver
Velocity

=100
.c Condenser
«-H ^h C^ ** «*•
X 8.
L.
i.
•a
L.
o 1
«l

M U
e .c SCH
SooS^S^S^^^^^^I-t Ex
vt BO
OJ
Size
•3 a
_l
1?
c <u
a. Line
a
IPS
£ Ex
< X
E
E c
< \0»^N"O^NNOOOO
> Q- 6 8<3 fONin*iNOONOOVOGOVOO
^ 3 E •5-S ii X f
£v
a-
ui ^H *-. -#
3c Q < "I
.r c s
It _ 5? Rio^^fONOOOOOOO X
«
M 8! f0N0sO>NHNNN00O>OO .8

*
ee
u
u
a.
o
ft,
<
§ 00 ^
lO^H-^rs-^iOOOOOOOO
Ex
<
ft.
a

£ O <5 <S lOfO O Os NihO00M")OO


II

c «> ft.
*-< CS t|< vO <*3 -^ On
o <
4* ao a t
'u tt 00 »ooo 1
X t_ & ^ONt-^csesOvOOOOOO
a. u_
M 41
# t T^ Q
u c
8
«>
ft, t"x T?«
w _l
.e <
e A .1
»o
*J ^ t-4
Ex
-J C S3 ^lO^-^OOtOOOOOO
*-( t-»
<
c M J OiOOO^O^O^NOOOO
—*NTjiaN00NfO*O 8 9
it rt n >o oo
iu
3 'i u a
M M *.#
q£ 3 *o ^
CNCOWJOOn^hOOOOO
-«**

ItQ 1a
|
BE o 7.

t'
A Ex<
s
!5
0> M
IU Size
SCH
oooooooooooogoQ *&.*
•3 ^
-i o£
a •s
o
<
h c
Line IPS

H
TABLES 495

•8

I
s

a •=>

8 * *1

£
11
•**

Is *1
S3
£ .

•Scq
S
I*8
§ *
.2 3

8 IS
.8
.8
Line
J?
O
•3
3
_ co <e * n
dW3i Nouwanivs ni 30nvho 0NKiNOdS3UMO0
8*. SS01 dW31 1N3~I\/Ain03
J 3 (IV)
.1

.3 8 1

a
TABLE 19-5. Refrigerant Line Capacities for Intermediate or Low Stage
Duty (Tons) for Refrigerants 12, 22, and Ammonia
Suction lines' Lint
Refrigerant and AT Line Site Liquid line*
Equivalent of TyPaL Die- Silt
Capper Suction Tamp F charge TfptL Condenser to Receiv-
friction Drof
OD -90 -so -60
Una? Copper Receiver
-59 -40 -30 OD V-lOOfpm Sgstem'
1
Refrigerant 12 1 2.1
1
t
{ 0.2 0.3 0.4 0.5 0.9
H 0.17 0.24 0.3 0.4 0.6 0.8 1.0 1.8 U
i 11.0
21.5
if 0.30 0.42 0.6 0.8 1.1 1.4 1.7 3.2 «t 37.0
11 0.47 0.67 0.9 1.2 1.7 2.2 2.7 5.0 it 60.0
2J 1.00 1.40 1.9 2.5 3.5 4.6 5.7 10.5 2» 125.0
2 F AT Per 100 ft 1.7 2.4 3.3 4.5 6.1 8.0 10.0 18.5 2t 220.0
Equiv. Length 31 2.8 3.9 5.4 7.3 10.0 13.0 16.2 30.0 3t See Table 350.0
31 4.1 5.9 8.2 10.8 15.0 19.5 24.3 44.0 3|
*J 6.0 8.5 11.7 15.6 21.5 28.0 35.0 65.0 4i
Si 10.6 15.1 20.8 27.8 38.5 50.0 62.5 113.0 5i
61 18.1 25.8 35.4 47.2 65.4 85.0 106.0 180.0 6}

1
Refrigerant 22 1 3.6
0.6 t 7.0
1 0.16 0.23 0.31 0.44 0.57 0.75 0.94 15 t 18.0
H 0.34 0.48 0.65 0.91 1.19 1.55 1.93 3.0 ii 36.0
11 0.59 0.81 1.12 1.59 2.07 2.7 3.4 5.2 it 63.0
it 0.93 1.34 1.8 2.5 3.3 4.3 5.4 8.5 ij 100.0
21 1.9 2.8 3.7 5.2 6.8 8.9 11.1 17.5 2i
2 F AT Per 100 ft 21 3.5 5.0 6.6 9.4 12.3 16.0
210.0
20.0 31.0 2t 375.0
Equiv. Length 31 5.5 8.0 10.6 15.0 19.6 25.5 32.0 50.0 3t
3t 8.4 12.0 16.0 22.6 29.5 38.5 48.0 75.0 3t 19-3
41 12,0 17.2 22.9 32.3 42.3 55.0 68.8 105.0 4J
51 21.2 30.6 41.0
6^ 34.8 50.0 66.5
57.5
94.0
75.0
123.0
98.0
160.0
122.0
200.0
190.0 a
305.0 6t
See*/
Steel
IPSSCH -60 -SO -40 -30 IPSSCH
i 80 17.0
i 40 0.26 0.38 0.50 0.62 1.0 i 80 34.0
t 0.55 0.76 1.05 1.30 2.1 f 80 75.0
1.05 1.53 2.00 2.50 4.1 80
Refrigerant 717 li 40
1 150.0
2.15 3.15 4.10 5.10 8.5 It 80
40 305.0
(Ammonia) li 3.4 5.0 6.5 8.1 12.5 It 80 490.0
2 40 6.3 9.2 12.0 15.0 25.0 2 40 See Table
F AT Per 100
n 40 10.3 15.0 19.5 24.3 40.0 21 40 19-4
1 ft 3 40 18.4 26.8 35.0 43.7 71.0 3 40
Equiv. Length 31 40 27.3 39.8
37.8 55.2
52.0
72.0
65.0 105.0 n 40
90.0 145.0 4 40
5 68.3 100.0 130.0 162.0 260.0 < 40
40 110.0 161.0 210.0 262.0 425.0 6 40
40 258.0 376.0 490.0
|
610.0 8 40

Lint Capacity Multiplier!


Sat. Dis-
charge Refrigerant 12 Refrigerant 22
Temp, F Suction Discharge Suction Discharge Discharge

-30 1.12 O.SS 1.09 O.SS


-20 1.07 0.70 1.06 0.71
-10 1.03 O.SS 1.03 O.SS 0.77
1.00 1.00 1.00 1.00 1.00
10 0.96 1.2S 0.97 1.20 1.23
20 0.93 ISO 0.94 1.45 1.45
30 0.90 1.80 0.90 1.S0 1.67

(3) For other A7"j ani BqmaUnt Lengths,

Line Capadly (Tens)-TaUe TensX (-


100 Actual a.T Less Desired,
aired, F \"
\'-
< Actual Rente. Length, ft Table AT Lees,<,F J
(4) For ether Tons and Equivalent Lengths in a given pipe site.

A '*«aBP*'L~tl*.f' „ Actual Tens \"


nrrn-TWir ftr?t (
100 *\ ToUeTous J

(5) Values attained from Carrier Corp. data.

ar^Alr^o^U^tog aSr2eS
J n *'*""*' 19S7~58 Editi0n ' ** pmMoa of "» American Society of Heating. Refrigerating.
496
CHARTS 497

CHART 14-1. Btu per Minute Removed in Refrigerant- 1 2 Condenser per Ton of
Refrigerating Effect Occurring in Evaporator
340

-30 -20 -10 10 20 30 40 50


Suction temperature F

From ASRE Data Book, Design Volume 1957-58 Edition, by per-


mission of the American Society of Heating,
Refrigerating, and
Air-Conditioning Engineers.

CHART 14-2. Btu per Minute Removed in Refrigerant-22


Condenser per Ton of
Refrigerating Effect Occurring in Evaporator

-»5Q -40 -30 -20 -10 10 20 30 40

^
Suction temperature F
From ASRE Data Book, Design Volume 1957-58 Edition, by per-

A^ndlSg £gS. **» °f *%«** »<


498 PRINCIPLES OF REFRIGERATION

CHART 14-3. Btu per Minute Removed in Ammonia Condenser per Ton Refrigerating
Effect Occurring in Evaporator

300
u-
E _
O Q)
E
oh a) 280
M
re co

ton in
s
per
moved
O
e ;
no
100
to 90
g.8.220 8U
3 3 70
60
200
!0 -10 10 20 30 40
Suction temperature F

From /iSltE Data Book, Design Volume 1957-58 Edition,


by permission of The American Society of Heating, Refrigerat-
ing and Air-Conditioning Engineers.

CHART 15-1. Resistance to Flow of Water Through Smooth Copper Tubing

o
•-«

ci 000^ cio o h
CM CO tf)U> CCJ oj m <* into 00
1

Pressure drop (psi per 100 lin. ft)

National Bureau of Standards Report BMS 79.


CHARTS 499

CHART 15-2. Resistance to Flow of Water Through Fairly Rough Pipe

CMCO ^ mio CO »-H CM CO «t If)*© CO o mu5 ©


o o o do c>
"" cm co *f 00

Pressure drop (psi per 100 lin. ft)

National Bureau of Standards Report BMS 79.


500 PRINCIPLES OF REFRIGERATION

c c
E M
c o
«
b rt

e t
ill
£
c f
s
L. « 1 /l 71 ii
o <*
rf
*2 _l
77
1
Sco 8
A
u >*

t5
// s
>M a) 1 '//, //

.3*
cH
E i
3
>
L.
01

?
,s ^"XX I/*'
/y
V/
/I
I


0)
O S r« * * « a *& iiC ^P rS
1

T<3
Minim 1
Copp

in I
1

to

_l
U

§§§§
uidj 'Apoisa sb8 uinuijuiw
* <
L.

£ £
oe
< BO
X 8
u o

4J
e
M
C
« g
E O
e
M .

k n
C e
111
£
o r I
*
o
-1

*c 3
>s
u
V
a •
>
L.

El l
i • O
l £
£ o
>?
i/>

f Oe CM

s
< _l
^
*
*>
iud) '<ipo(9A sb3 uinuiiuiim

<
L.

H £
AC c
< M
X s
u Q
CHARTS 501

e
«
E
e M 00 satpuj am sazis sdri ||v

2 1S
t -J
iu

c
^ 8 8 o
.8
2 J 1
£8 -•
•SO 8 f
J2 «*
>s
« o w

1*1 I
§ g
UKlj '%M|8A SB8 UinUI|UJM

<
I
u

«
E M
& 00 ssqsui 3JB sazjs sdjd nv

c 8
IU

IS
£ o (d
I

a
1

i*

13

wdj 'X}!0O|3A se8 umujiujh


<
i
o
502 PRINCIPLES OF REFRIGERATION

CHART I9-2A. Refrigerant- 1 2 Flow Rate


5.0
NX \ \
s N.
4.6
\\ \ s^fe^
^130°

\ ^f
a 4.2
Icnr^ 110°

10! r

£3.8
igo;

:3.4

20"
40° ^
3.0 0°

—20
-40°
2.6 1 i
1

-100 -80 -60 -40 -20 20 40 60


Average refrigerant temperature in evaporator-T

Courtesy York Corporation.

CHART I9-2B. Refrigerant-22 Flow Rate

|3-8

1 3.6
^_130°
£3.4 ^120°
*^ <*f
^3.2
t3.0
^ ^ 'hire.

12.8 80°
a .

f 2.6
60"
40°
1^2.4
20°
J 2.2 0°
-20"
^2.0 -60° ^40
3 1.8 _—— — ^_—
-80-60-40-20 . 1

-100 20 40 60
Average refrigerant temperature in evaporator-'F

Courtesy York Corporation.


CHARTS 503

CHART I9-2C. Ammonia Flow Rate


504 PRINCIPLES OF REFRIGERATION

The equipment rating tables reprinted in this book are intended only to illustrate methods of
equipment rating and selection. For this reason, many are incomplete and therefore do not
represent the full line of the manufacturer.

TABLE R-l. Natural Convection Cooling Coils Capacities


Btu/hr/inch Finned Length
Surface
(7* less than overall) No. Fin SqFt Btu per Hr per in.
Model Width Tubes Spac. per in. 1°TD 15° TD
Single Row High Coils
J-14 13* 4 1.26 1.47 22.1
i
i 0.86 1.21 18.2
K-16 18^ 6 i 1.89 2.21 33.1
i 1.29 1.82 27.3
U8 24' 8 i 2.52 2.94 44.1
i 1.72 2.44 35.5
N-1.10 29J* 10 i 3.15 3.68 55.2
i 2.15 3.04 45.9
J-18 30' 8 i 2.52 2.94 44.2
i 1.72 2.44 36.6
K-1.12 42* 12 i 3.78 4.42 66.3
i 2.58 3.65 54.8
L-1.16 54* 16 i 5.04 5.88 88.4
i 3.44 4.87 73.2
PJ-14 13** 8 2.52 2.94 44.2
i
i 1.72 2.44 36.6
PK-16 18i*» 12 i 3.78 4.42 66.2
i 2.58 3.65 54.8
PL-18 24** 16 5.04 5.88
i 88.3
i 3.44 4.87 73.0
PN-1.10 29J* 20 i 6.30 7.33 110.0
i 4.30 6.10 91.1

Two Row High Coils

J-24 13' 8 2.52 2.65


i 39.8
i 1.72 2.23 33.5
K-26 18i* 12 i 3.78 3.96 59.4
i 2.58 3.34 50.1
L-28 24* 16 5.04 5.30
i 79.5
i 3.44 4.45 66.8
N-2.10 29$* 20 i 6.30 6.62 99.3
i 4.30 5.57 83.6
J-28 30* 16 5.04 5.30
i 79.5
i 3.44 4.45 66.8
K-2.12 42* 24 i 7.56 7.92 118.8
i 5.16 6.68 100.2
L-2.16 54* 32 10.08 10.60 159.0
i
i 6.88 8.90 133.5
PJ-24 13'* 16 5.04 5.30
i 79.5
i 3.44 4.45 66.8
PK-26 18J'* 24 i 7.56 7.92 118.8
i 5.16 6.68 100.2
PL-28 24** 32 10.08 10.60
i 159.0
i 6.88 8.90 133.5
PN-2.10 29J** 40 i 12.60 13.25 198.8
i 8.60 11.12 166.8

* Width of each section.


Courtesy Dunham-Bush, Inc.
TABLES 505

h 13*l r- 24 -*i (*— 29% -*j

Type J 14 Type K 16 Type L 18 Type N 1.10

Type J 24 Type N 2.10

e 30 — »-)
f<
42
KJ model 42" wide

("J" plasti coil in a


"K" tube sheet)
Type J 18 Type K 1.12 Type L 1.16

U model 54"'wide

ssl w
Type J 28
sfli iS?
Type K 2.12
sIKo^w Type L 2.16
("J" plasti coil
"L" tube sheet)
in a

|*13H |-d3*| fel8%*| (*18iJ->| |-«— 24 ->j l-e— 24 —>| (-« — 29Js — »-j h«— 29% — »-|

Type PJ 14 Type PK 16 Type PL 18


'km! Type PN 1.10
lw^
— *k jfs^

Type PJ 24 Type PK 26 Type PL 28 Type PN 2.10

Installation dimensions

^ *D
<-A-»4___J< 5») K
SSSSSKW

<-E-l
WZMmk-E JMWm. wmm.
Dimensions A B C D E
Maximum 12 in. 5 ft. 8ft 8 in. 4ft
Minimum 6 in. 10 in. 6 in. 4 in. 4 in.

Ifthe width of the box requires additional plasti-units, care should be


taken to see that they are installed in a similar manner as shown
above.
— —

506 PRINCIPLES OF REFRIGERATION


TABLE R-2. Single Nate Evaporators
Total Btu's per Hour
Total "K" per Plate at 15° TD
Catalog Width, Length, feetot sqri
Number Inches Inches Pass Surface Below 32° Above 32° Below 32° Above 32°

12 24 9.9 4.18 10 13 150 195


1224
12 36 15.8 6.26 16 18 240 270
1236
12 48 21.8 8.35 21 24 315 360
1248
12 60 27.8 10.45 26 30 390 450
1260
12 72 33.8 12.55 31 36 465 540
1272
12 84 39.8 14.65 37 42 555 630
1284
108 51.8 18.83 48 54 720 810
12108 12
144 69.8 25.2 63 72 945 1080
12144 12
22 30 25.7 9.64 24 28 360 420
2230
36 31.6 11.58 29 33 435 495
2236 22
48 43.6 15.4 38 44 570 660
2248 22
60 55.6 19.3 48 55 720 825
2260 22
72 67.6 23.2 58 66 870 990
2272 22
79.6 27.24 68 79 1020 1185
2284 22 84
108 103.6 34.9 87 100 1305 1500
22108 22

Courtesy Kold-Hold Division, Tranter Manufacturing Co.

TABLE R-3. Plate Banks

No. of No. of Total Btu's per Hour


Expansion Expansion at 15 TD
Catalog Size, No. Feet of Valves Valves
Ammonia Below 32° Above 32°
Number Inches Plates Pass R-12

4 87 1260 1440
4-1248-B 12 x 48
1575 1880
5-1248-B 12 x48 5 109
6 131 1890 2160
6-1248-B 12 x 48
111 1560 1800
4-1260-B 12 x 60 4
139 1950 2250
5-1260-B 12 x 60 5
166 2340 2700
6-1260-B 12 60x 6
1860 2160
4-1272-B 12 72x 4 136
170 2325 2700
5-1272-B 12 72x 5
204 2 2790 3240
6-1272-B 12 72x 6
160 2220 2520
4-1284-B 12 84x 4 1

200 2 2775 3150


5-1284-B 12 x 84 5
240 2 3300 3780
6-1284-B 12 x 84 6
4 207 2 2880 3240
4-12108-B 12 x 108
258 2 3600 4050
5-12108-B 12 x 108 5
310 2 4320 4860
6-12108-B 12 x 108 6
4 279 2 3780 4320
4-12144-B 12 x 144
348 2 4725 5400
5-12144-B 12 x 144 5
418 2 5670 6480
6-12144-B 12 x 144 6

Courtesy Kold-Hold Division, Tranter Manufacturing Co


TABLES 507

TABLE R-4. Plate Stands

No. of No. of Total BTU' per hour


>

Catalog Width, Length, No. of Feet of Expansion Expansion at 15° TD


Number Inches Inches Plates Pass Valves Valves
F-12 Ammonia Below 32° Above 32°

4-2230-S 22 30 4 103 1 1 1440 1680


5-2230-S 22 30 5 128 1 1 1800 2100
6-2230-S 22 30 6 154 1 1 2160 2520
7-2230-S 22 30 7 180 2 1 2520 2940
8-2230-S 22 30 8 206 2 1 2880 3360

4-2248-S 22 48 4 175 1 1 2280 2640


5-2248-S 22 48 5 218 2 1 2850 3300
6-2248-S 22 48 6 262 2 1 3420 3960
7-2248-S 22 48 7 306 2 1 3990 4620
8-2248-S 22 48 8 350 2 1 4560 5280

4-2260-S 22 60 4 222 2 1 2880 3300


5-2260-S 22 60 5 278 2 1 3600 4125
6-2260-S 22 60 6 333 2 1 4320 4950
7-2260-S 22 60 7 390 3 1 5040 5775
8-2260-S 22 60 8 445 3 2 5760 6600

4-2272-S 22 72 4 270 2 1 3480 3960


5-2272-S 22 72 5 338 2 1 4350 4950
6-2272-S 22 72 6 405 3 1 5220 5940
7-2272-S 22 72 7 474 3 2 6090 6930
8-2272-S 22 72 8 540 3 2 6960 7920

4-2284-S 22 84 4 319 2 1 4080 4740


5-2284-S 22 84 5 398 3 1 5100 5925
6-2284-S 22 84 6 479 3 2 6120 7110
7-2284-S 22 84 7 558 3 2 7140 8295
8-2284-S 22 84 8 637 4 2 8160 9480

4-22108-S 22 108 4 415 2 1 5220 6000


5-22108-S 22 108 5 518 3 2 6525 7500
6-22108-S 22 108 6 622 3 2 7830 9000
7-22108-S 22 108 7 725 4 2 9135 10500
8-22108-S 22 108 8 830 4 2 10440 12000

Courtesy Kold-Hold Division, Tranter Manufacturing Co.


508 PRINCIPLES OF REFRIGERATION

TABLE R-5. "1C" Factors fc»r Bare Pipe Co ils


1 in Liquid*
Desired Desired
Liquid Refr, Liquid Refr.
Temp. Temp. "K" Temp. Temp. "K"

65° (a) 38° 15.7 35° (c) 19° 13.5

60 (a) 38 15.5 30 (c) 15 13.0

55 (a) 38 15.2 25 (c) 11 12.5

50 (a) 36 15.0 20 (c) 7 12.0

45 (a) 32 14.5 15 (c) 3 11.2

40 (c) 28 14.0 10 (c) -1 10.5

35 (a) 24 13.5 5 (c) -5 9.8

35 (b) 19 12.5 (c) -9 9.0

35 (6) 15 10.8 -5 (c) -12 8.2

35 (b) 11 10.0 -10 (c) -16 7.5

35 (ft) 7 9.0

(a) Water cooling, (b) Water cooling, ice formation on coils, (c) Brine cooling.
* For dry expansion tubing or pipe submerged in water or brine without agitation. (Courtesy
Vilter Manufacturing Company.)

TABLE R-6. "K" Factors for Bare Pipe Coils in Air*

Refrig. Room Temperature Degrees Fahrenheit


Temp.
-20° -10 10° 20° 30° 36° 40° 44° 50° 60°
°F

32 2.30 2.49
28 2.11 2.50 2.52
24 2.11 2.49 2.51 2.52
20 2.11 2.49 2.49 2.47
14 1.79 2.50 2.52 2.48 2.52
12 1.80 2.49 2.49 2.52
9 1.40 1.79 2.50 2.49 2.49
6 1.39 2.01 2.48 2.51

3 1.40 1.99 2.48 2.53


1.39 1.59 1.99 2.51
-4 1.39 1.59 1.99 2.50
-8 1.30 1.49 1.80 2.01
-13 1.39 1.60 1.74 1.98
-17 1.50 1.70 1.79
-25 1.50 1.60 1.80
-30 1.39 1.70 1.79 1.80
-40 1.59 1.80 1.80
-50 1.79 1.80

* For iron pipe coils with gravity air circulation. (Courtesy Vilter Manufacturing Company.)
TABLES 509

TABLE R-7. Lineal Foot of Pipe per Sq Ft


of External Surface

Pipe Size Lineal Feet

*' 4.55

r 3.64
i' 2.90
li- 2.30
if 2.01
2* 1.61

Courtesy Vilter Manufacturing Company.

TABLE R-8. Unit Cooler Capacity Ratings and Specifications


Btu/Hr Rating Core Motor and Fan

Surf. Motor Heat Air


Model 10° TD 15° TD FT* Circuits Tube HP Btu/24 Hr Fan Rpm Cfm Throw
1
UC25 2,500 3,750 67 1 ¥ 25 7,600 10' 1500 390 20
UC35 3,500 5,250 93 1 V h 8,000 12' 1500 510 20
UC45 4,500 6,750 156 11,500 14' 1500 700 17
1 i" 20

1
UC65 6,500 9,750 210 1 ¥ 12
A
12,600 16' 1140 1,000 23
UC85 8,500 12,750 266 1 ¥ 12 13,350 16' 1140 1,480 27
UC105 10,500 15,750 328 Split ¥ 4 15,100 18' 1140 1,730 25

UC120 12,000 18,000 378 Split ¥ (2) 2*0 18,000 (2) 14' 1140 1,950 25
UC180 18,000 27,000 566 Split ¥ (2) A 25,200 (2) 16' 1140 2,550 20
UC240 24,000 36,000 755 Two ¥ (2)4 34,000 (2) 18' 1140 4,050 27
UC320 32,000 48,000 1030 3 ¥ (2)i 75,000 (2) 22' 1140 6,000 28

Courtesy Dunham-Bush Inc.


TABLE R-9 Water Chillers
Acme Tube
Effective Capacity Std. No. Number
Model No.* Area Sq Ft Range Tons of Circuits of Tubes

DXH-805 41
806 50
807 59
808 67
809 76 5.4 to 19 1 44
810 85
811 94
812 102
813 111
814 119
DXH-1005 64
1006 77
1007 91
1008 104
1009 118
1010 131 8 to 30 1 68
1011 145
1012 158
1013 171
1014 184
1015 197
1016 211
DXH-1206 105
1207 123
1208 141
1209 159
1210 177
1211 195 11 to 40 2 92
1212 213
1213 231
1214 249
1215 267
1216 285
DXH-1406 136
1407 159
1408 184
1409 207
1410 231
1411 255 14 to 52 2 120
1412 278
1413 302
1414 325
1415 349
1416 372
DXH-1606 187
1607 219
1608 251
1609 283
1610 315
1611 348 20 to 71 2 164
1612 380
1613 412
1614 444
1615 477
1616 509
DXH-2006 286
2007 335
2008 384
2009 437
2010 487
2011 535
2012 583 31 to 111 2 252
2013 633
2014 683
2015 733
2016 782
2017 830
2018 880
2020 976

510
TABLES 511

For higher loading -use loading of 3000 maximum

K 0.35 0.4 0.6 0.8 1.0 L5 2.0 25 3.0 3.5

i l I 1 L_ I l l I J I

L 0.2 0.4 0.6 0.8 1.0 15 2.0

_L J_ J_ _l_ > ' l i i l _|


0.15 0.2 0.4 0.6 0.8 1.0 U
Water pressure drop
Feet head /foot of shell length

Courtesy Acme Industries.


512 PRINCIPLES OF REFRIGERATION

+ 00
in <o
N
oo'
111


q
N N N N N -
c> rs M;
**:
ix * O "> co m
r*>
« « ' W
•<) n -o
•*•'
\rtcm
-6 oo
^ P
° 2
o o
11.111 iilil 8i|8§
N O t 9 fill <N 5o 00 II
© m
O** V * n a
— £ in"
N
«'
£;
oft I*

N n o * m — * — d - m eo in
— ro xO (^ f«*
M; in -O rs oq
V in -n* is' oo ftOodo <4- U)
00
<o' OO (h
«*»
© — — -*
'
fx.

ro lo xQ 00 o"

88888 8888 O O
8 cm o Oo O Q
in S
1 T • N ". 5 • 8 * nN -
* I M* o m is ^
U) N n' N « i* a rs CM CM is «o

— — — —
Ifl
cm m ^ «5 oo •* Is
CM >

•o ^ m N p
ifl *n -o i/> pn xO xO xO in cm in is in ro xO 00 q
M t 4 N 09Ifl 00 00 00 CO ui >o is oo* o* o* o* c*M* in <> 00

II lllil 8 88 llll
00 CM
o o _ _
M- CM M" CM
?" 00"m" V in 00"
3 - - W M Ifl S" 8 2f — cm m M^

o>_ -o M"# — CO CO O- CO N is -o in *n cm 00 M" N CO N in in in in m*


n n V in lii <o <o <o <o 4 Cm «
M- m" -o" N N N CM* ro M* in -O

I §8888
9 _ 88888
^ © m
C* cs| -. *n tM § 888—
O- CM
8 8888
-O CM CM 00
Q o <o in us a» <** V
n n t «**" M" Mj- CO
rs ©* — V
*
M
C
"-: *. — r~ q m; M- M; cm cm q r^ ro <N — — :
o
fM <"* V ^ Uft ^ -o «o ^' ^ V in m' CnJ rn V in xO
tc

Sn a§ fi'
88888
O if
P> In 00
88 CM
88888m
So oo oo oo ?6
w _' m m ©~ —

2
o
m
to
o> m' oo" cm 15 in,«o •o
— — co" -o
<o
cm m
a u. N m , in t oo — cn — q *- o -o — in 000*00 C> <> iv) — 00
E in N « ri ri + m* **> in in in pi ^ •<-'
in* in in in in — cm rn m* M-*

o
U in «o 8S8

N « <
<> m o
«"
11888
B» c> o * m
o* »"
<» 9 i» e> —
° s o ©.
-•jo
— «"
ix" -a" in" <0" ix"
© ©
M 2?§88
* - O
M.
in" o" in —
CO
o*

co m k n <> o <m m cm — rx CN 00 O CO in cm ?s M- O
tc — CM cm fn m — CN M* V M* - <n N m V
§8888 OOoo
* n « S 5 88888 88
111
-I
O li II o S 5 o o
o O ~
CM li
to O CM CM CO CM CM CM CM O CM
o O O -" in c" — is
Is" T rs V is"
< ifl o»*
A Is 00
M-" *>i"

I-
Ik un oo <n lo o» cm m; in M; m M N - CO O; CO l"-; CM is <S|
-, -
«N f>i N ro ro *^ r^
.

r*> cm m cm* - (N « m
88888 SoS S 55 6S poo5
So o o o o O
*
•"l T.

in ©_
g* ©~
ix_
m -o V ix
illli ?S2§
— CM ot m o V
I p>~ -o o>
m ** in •« ^

iv £•oo; in
•* -q in ix —
«•< in ix o N *U> O
k
IA M ix— in o ix r4o-
m -<r <o in t<4

•2 in * » V — •• IX -d o w —
<•• o» f» -4
— es m m # o 10 "«; o»

o o oo e g o o e w c? ^ ^
F 777 T7 7 7 177T
w

O 1 " 00 oo o- 8 |lf
r -

TABLES 513

en ^ m **> *> O c* p U) ^ N in GO
i^ o o - q h;
o ™ P m go in q n -
V •+ u> -a ~6 in < n s ~ in in n go O'
•rt in «b r*i
in 2 £
i^: I** 0>

§8888 8§88 88888 llll 88888 888


o 819
3 * w •
o m
* * » *«N * 18 —
i» cm i» CO CM CM CM CM
§' S S" J" — o a - a » c£
? n n t 4 ooj oo
m <T
^ in
-o a uf
— t
ia
— a n
— cm
^f *-» <*»

oo »o -^ 00 <>
O — O in nt r*> 9*.
O -
O-; r-*
t
rn -O
« -O "t, « N q-ON
— CN (> N U)
— m ^ *-
- N «' * W
*•;
CM <N C»i LI") -o oo o>
(si rn t -O GO (h rn c*»
U) N (^

888 N »« » 8 8 co S 8 88§8 8881 ©» 3 IN 11888


o' in in » V
=? si *' a' N O N
< Q N» tt 9- - m n in 9
< a— >
- * >>
oo cm

~ **. °°. a n s q — o r» eo »• * 00 w m CM ^.
eft

» » 2 ° 2 V uS -d <> 2 ~ - m uj 4) a 2 2 £ Z o-' M-' -O 1-^ » 2 z — —


o o o
8188 ooo g* oCM 8 fill 1188 88888
S 3 888 §8888

o *
a
CM CM CO 35 cm «o
ri o" ••"
ia r-> eK — *
(So! r». c*\-O «*0 m ^h in ^> o^ in *> — — OJ ^ ^; m p ^> o* «n t> cn en
s r^ oo co oo ro -^ in -o I-* 00 00 O* en + m -o S OO* CO O"' (^ m ui 4 N CO C O
8 888 II 8 88888 2 888 888 888
OWN 1188
n* ci O* w m
cn r- r-% N t> 00 C4 O m
V in
* t o-*
in
u>
n - »

r*> oi r-; o o p p — p mJ- o> ro * co •>


p q oo in p in oq S CO 0> o>; r** in —
in
«0 s,»
« i"** oo oo V in -n t*n r-J r»' co' co w pi in -o O N O0 CO CO m t m' -o r-" oo o» o*

88888
m — — r^ ©»
a
m So
co 0>
8880
n - rt t <*» m r>* i|8S§
t nt t
ffl ||8i8
n f n8 a
Q m
8* f^ M> CO
r» o> — if t a «o
<o ia m)
o
* CO <N ^f- •*- in co — c* *
-p oo W N O 00 in p mco — in <s eo n n
n n t
I

in u> -b -d -o in *b -b »o <N en V V in >o -o -*o r^ in -o .b i-** fv

88888 o m o
o in O O Q O O m o in
o o 88888
S S * r«. cm
»*•
IA O* *A 88 fill a o «t co « o n n Aa t
«o — «o*
«n V *" o oo"
?" CM
ia
£
m>
CM
oo

o
ci rn
— d
CM
of iV •** o*
m ^- m >• oo
^* co — en rn — n ^ o en in -4> * ^ m q in p 't; N O; t N O; * N N - *
^ V in in in N N W t in in ui — cm m •* o*" in in in m «N C*» i**> ** in in *o <i

88888
^^
IN 00 ^_ t> N SO 8888
*
CM
8888 88881 %%% §§§18
CM_ » 3 * 5«
O- CM CM M» CO

m ^ m
of —T -o —?
in -S oo ? S 5 R
O O* -M O N N N -o. o «* •*»' "^ ui — rs. <m •n om ** «*•* ^ (M -O O; —

8SS8S o o o
r*.
« *
55 cm cm Mr
a' n
CM CO IA
K cm r»
u» o> m
88888 fill §§§88
» p m 8i§
in' co m'
*> r« CM
t* »*
S S CM 8
Nh
cm m ^> IA M)
oo'
N mt w3 o>"

•-.- "! P
— -
«/ aN4
»». !? 4> u,
IA * 525 9*. ». «5 *i 5-1S Mf ©» * — •
— cm r»
cm m
-o
CM CM m M?

§
2gS?S o o oo S8S9S ««9 «i* «^
?s 28S?S 1
777 777 It
I
&!5*

s
o
U
514 PRINCIPLES OF REFRIGERATION

li 31

i 3^

3J

3 31
Q

SB. Jd
8
a.
a kid
E
o
U

3^
III
-I
a
2
3
_2 u 5 a li
-8~ =1
* 8 a.
6-
E
1* 6l C M
! « !
C

1}o li 3
E

i s
ft'
ui

5k 3s 5S li
.

lib Tj j: T3
c
o
A
£
-

53
§

1
II \t 5| I
5
TABLES 515

TABLE R-IOC. Specified Suction Temperature at Compressor Inlet (R- 1 2 and R-500)
Saturated suction -40 —30 -20 -10
temperature ° F and above

Actual suction 35 43 55 65 65
temperature ° F
Courtesy Carrier Corporation.

TABLE R-IOD. Correction Factors


Saturation temperature
of suction gas (°F) -40 -30 -20 -10 10 20 30 40 50

Factor 0,90 0.91 0.92 0.93 0.94 0.95 0.96 0.97 0.987 0.997

Courtesy Carrier Corporation.


516 PRINCIPLES OF REFRIGERATION

©
oo
oo v> co
(- t- t- I—
© v9 ^
rt 9( VS Cl
t- t- t- VO VO vo
© *
co r- -h
vo vS u-> >o ti
5 ooddd dddodd ©© ©©©©

ON <S <r% S^t oo co On *o ~* oo vo


oo oo t- vo vo r~ r- vo vn Tf Tt ** co co
8 lis
©©©© ©
HDOIftNO S8S© O O
cn oo O0 CO CN
©©©
35.
"3"
°°. H
t, °\ "! .
on in <s oo co ON h oon
CO CS <N CN -h -h

r~ f- f- vo v© 3 VO
t~ r» VO vo
H M
vo vn
IO N*
vn vn -^ Tt
Ift

3 ©©©'©© ©d dddd ©'©OO © ©


2
8
*e
D
-a ON <N *
00 CM
* on
nm
•<t <r\ © on
« r-

M
C
I §N US vo vo <n tj- rf >n «-> tj- co co is (S -<

c
B o oo o
»n n©S S 8nS«s © © Q ©
f» in ^ »n
O
m
VO N » *

to * * * m «
O ! a ^i
vo"
°°- '*!.
>n >o •*
°V. 't.
x-T *"
tT co" co" co* co co (N rN cs «
< pq
•<r

I o,

O
*
H-
co <-< oo «o «s W h OO vO wo Sf-
vi >o m ^ 7
•* - oo 3
I
t-
©'©'©'
P-; vo vo vo
©' d dddddd*
e- r- vo vo vo
©©©'©©"©

of)
^J- * co co (S ^nn N -> « « — N
O0 »1 » |v «
-H © © ©
oS (D efl
ui
o
oo
to
<

382§
hn»A «5 ONn a is
© *
m § •* »8
©
m^
3" f-^ co oo o> ^h r-_ co

n vo" <o" m" m" »i m" * ^" n *" co" co" ts" cs" csf


oo
f-; © vo V> vo VO i-< t-; 1-; 00 (N f>J (~- vn n vo (N
« n*
*
r*. *m^n
N » n ^ Mh
M « «t «
FH M
ol ON vo CN ©
<N

3
CO

m ©
9 8 »n & *n
<s|
<n
CM CM
om
—©— t> © © vn
CO
|6 I I

c
9- § 9- o*
I6
*
>-
£
111 .cm
£
" «">
(N
cm 09
as •O I—
TABLES 517

« O 3 »m
*— »* O On On © © O; On 00
>-h
n & S£ r NO
t~-
ON 00 00
:« f-; |~-

o©© ©©©
i

fi MM «od ^ ^ ^^ © © ©

fc •f? ai OB \o
oo oo
hm I--
»N
vo no
m
t-
ON »S t— rm
vo vo w> «n
m
<*• m in t *^

5 t: S
Sue „ «o J-u ^T
© © ©o ©
00 OO © — NO
© ©
rom © ©©©
vo © © >n
o
fO ggsss SP

"3" t? r-_ <s vo O 00 m © m©


c-l I- fsjf ml^ r^
oo r-" r-^" r-^" vo «\0 m m t * * tt to ro ri r-f
«

£ «a
-h
^O
mp
cj O; w
05
« © socioon On On 00 00
^-<in ©
^" On ^
On 00 00 t-; VO VO {jSSallt
I »"H©OO *-M
;

— .— © © © © © ©©©©
a 2SS£S.sg
X = 1 8 e S s

"8.11"-
O
r~
»S •* 00 rs
VO V> Tf •* «^^nnN mo
00
<*>
On
«s m©
^ ts
r-i «s
I a At •B.»2B.9S.Sa

c
V
"3"
So p © om
f» «s «M
•* oo_ ts vo
•-<_
© ©
00 00
ro 1-^ ©©
_
«n
op On
On tK
3
Pi!
flio o [! * «
II
£ oC oo r-* r~ vo vo «n <
< « t-^ vd" tJ-"

O N «©m
^* O On On 00
Otompno
** On On On 00 00
on ro 00 <S t- r-1
00 00 r~ .t— vo vo
3 « d ©' © «© ©©©© © ©©©©©

oo ro t~ <s vo
^ * ro to «s S ^t 00 »o r-
f> <s <s ——
— 00 VO t— ON g
o .a y ^"
oo

s 38$ 2© O
t- .-< in
co in
in
v> "n vt ^mm

00
3 3> I-; ©
VO in VO VO »-« f; t- 00 Cn) «s r— m m vo <*-»

c
o
-j t~ cs oo TT
*nnNN N «hTf-Ol
Nt
-< rt -.
(n)
on no •* r-i © <s
•a

en


«n
•*
©
^ in ©m
f> co <s
•n©h
N N m©
-<
m <n O
« <ri Q
>n
-h ts (S
I I I I I
jlilll
sa°§S3
C

v>
<
"8 5
.

518 PRINCIPLES OF REFRIGERATION

Q O»
O « m N
<S ~* ^H ^H v-N f» eft
v© m ^ «»i <s \o >n ^ en f*4 *-" tS CN ON ©\
A — ©' ©

S»P?s On fS Ov * m* *
m 3» r- ro os in
"ft ift ^t ^"
N
^woo
t— t— vo
8 *Qfi

o °»88 © O o
© OO Q
<s *
O O Q O OO
"3"
oo
in » On m «> <s
^ i— c> »« cs
f-_ VO o\ M_ ift_ 00 M_
« *-h o Ov 0\ oo o\" oo" t-" vo* vo" in vo" "ft" >n ^ to ro

v>
«n

o
•>*
t» f~
<S >-i
t~ vo in vo \e
IT! •* C> CS *-"
«
© s O o1
— N »
o\ 00
£ — -I © ©

E •g 8®> (S v©*
r— v© in >n
£2 ™^ »N|55
in »n ^ mo -q- fft rft
00 Tt
co m O eft
"ft no
cN cs CN
|2

"8
3 c
8§£8g © © © © ©
^ ©
§ 9 © n »
O ^ (N
.C t^
3 <N o\ eft
"3" <n in I— oo ©__ im ift vo r- oo in r- — * r~
C — T ©* ©"
of o\" on" oC oo* r-* vo* vf t^ vo "ft "ft ^" *n
v3

©
m i-h
^-
<N co
n cs >-
m fS -*
in •'*'
©
rft
«-h
cni
™ —
~* ©
— <
rv|
>* r- CT\ <N VO
-h ©
0\ On 00
ui
£ ©© ©
<
fi a "Sfc ^nm
-h in Ov
n f»
N *nnN « ©
N 00 vo © V©
„ „->o rft
n. 3 V « ift •• rt
o
00 a*fe

8© ©©©
•n oo m ©
©©© m© ©©
(S -j ®
"5"
_ ** *© f^ ct\ »©
rn
hN n*«
« eft" © oC
rvf i-" © © oC vo oo" r~"

00
©
r~ vo v© m VO -h I— t-; 00 N (M r^ "n in vo
a .— i~ r4 od ^r
^m rft (N »s
Tf
rs)
« — — «
l~ Tt
rs| — ON
.
0\ VO •* rs es o
1
00
3°- N ©
in
«©
N <n -m© © "ft ©mom
IS i i i i i

c
o
Ire- E &• 6 &
Si * "-• m
<s
u in

a aS •8 5 o "
1-1
TABLES 519

t~ i- •* (S M « in ro N rt n
»-H t-h O OS OO 00 00 t- so so m
M

» n « o\ 5 m oo <s r; <s
in "^ ^* ^^ t-- *a in
5 fill I
S Sit oo oo t~ so so ^O

llflll
o o SOo © © ©©
ro -- © r; s O O OOO
00 s m_ »s CI CI C4
en o c^ ^
<-i
33 (S
OS o p- so
— m N O O^ m" *
in^

3 t-~ so «n * ts cs
"*
' o" oC oo r~ oC oo" t-" so"
m

r~ oo
8 SO o s ON
m CI
m 00 oo t- so in t- oo
^ w N
Ph. O Os i- r- ^- t^ so >n
3
ffiliff
B. 8 2 o -a >
i j

N h M m
«s ^m n oomo\
rt oo ^t <s os
I 00 <s
•* •* *n NNn n nM N fsl -h
§|| ||s!
s.Sf s 8
S o-a
.a a £ a* g
ai S o
8

B uuifl
o O QO
<s s© t-
OO OQOO 8 ©
N O
© © ©
<s <n t—
« -—
I--

3" t~ CI Os OS NO CI ^H ON 00
©" oC oo r» so m
H*isfe
mn ci tN rt O 0O t^"
"
0\ r~ so -*S.S8gv>g
n

»ONm» oo
so so
so n m -23
m ^ *r h so "n fi pi N
os os oo r- so
3

oo
•*
* mo
* on m s©
cs NN «m
N oo
so
-—
"Ot OWn
« »
tN ©t
oS© -*

O
oo l££

8 ooo 8 ©no©
oo 88|
m O t- 5
ON 2
5. 00 -. SO N oo f~

«-*

o"
Os_
Os" t-T so"
0\
m"
o" oo so" >n" in c"T r-i
"
»-» os ob
m CN *-
Os"
1 —— '
*-

00
r- o so <n so SO y— t~i t"; 00 tN tN r- mm
so c-j

C — r~ <s ori * tt r> « -- os *— ON SO •* fN tN © I Ifilil


CS tN -H -H

o
Is
9
co
O >n
NO
in <n O
(S H H
OmO
Sm—O >n
CO § <N
I I I I
(N
I
frill's
jl

H sl§6'§8

a
o
«4
« m
11
IS
Oh
™ CO

is
< <

520 PRINCIPLES OF REFRIGERATION

<3\90 P; VO VI
c> PI —
PI PI PI
vj Tf ro PI »-< O S3 o v> m —
A pi pi pi pi pi pi N
pi ri n n pi pi pi pi pi --

o w .Sir 00 PI VO -* V>
vo vo vt vi •*
Q
^ mn
» t to
m 00 ^ ffl

8 si*

80000
* m v> v> © © 8 s 00000*
~3
1 c> ©*
" ^.
°°.
frf —I" 00 VO
*~. P-^
vo" *
00
f> >n -- >«
m" pf o* oC
m
ro
_
(S 0\ 10 -1
p- pi oo_ vo ro
>-T o" 00 t~ vo
PI N PI ««

n
00
N in
vo wt
r»_
«^
p hn
w
if;
pi
PI

O m
ft »
I pi pi pi pi pi
1

pi pi pi pi pi — pi ri N pi -h ph

1
-Sb f»> r- Q t p-
» *m* *^ m mpi
p- pi p» f> 00 * » Tt
m nm O » H
^N
t» pi nn
5£&
B
U

p-mS 2
© <=> © o ©© ©
pi — p> — 10
OOOOOO
« f «
e 1 rn^ 00
00
rf h
« P»
o
m
»
rn « » n h
00 vo* t" n «
<n v9 r-
vo a\ -* o^ v6 ro
•*t pT —T oC 00" p»
3 < »1
N PI PI PI «

cs

Ul
Ov ©O©
VO vo V> ^O t»>
OOOOOO
«*>O
PI <-« CT\ 00
f»>00 *t ON P- Vl
T: PI rn 0\ 00 P
-J £ pi pi pi pi pi pi pi pi pi -» « pi pi pi -^ ««
a
2
m 00
m * vo
* pi nn o o vo pi on
« -«
vi pi PI 00 vn
— « o\ P-
fa nriPl PI -< — ©o
o
00 III '

S S M °°.
N S5 §8*. S". *. © OOOOOO
§ *. n P m *pi (S 0\ vo
. *. ^
"3"
°. — . 00 pi vo « r~ t»v
p-" m" n" •«" o" 6"• h « »—
* «' pi" vT •^r pf —T o\ 00
p| pi p| p| pi PI 1—
I *-H t »-< *—

CA 'bb O VO
p; v; vo «H p. h 00 N PI p- 10 in vo pi
e
o if ^ pi
•;
tn n 00 Tf
pi n *
PIN H ^

«n« o\
i P-' o\* vo (N * PI ©

o
3
W
a, ©
^ vi
«m ©«
vi vi
PI ts
o—O
vi
t-i
Vi o O vi O V> O Vl
pi
1 1 1 1

•s Oh
s
IH

8. a £ tl
1)
r
V)
pi
.
1
PI
& •& "Si: O
r~

-J
T

TABLES 521

§ •* t co °o oe»
«;
n nO
in * co r5
v£>
i<i
2°2P •* *~-

A * «» CO CO CO co to CO CO CO CO CO CN| <vj f\j f\j

^ Sm 9\ (<) * **tft n oo -. Iff


ft,

s 111 ft oo r~ r~ $ » <5 <t n


ifl
On cS vo f-, oo in
•n •»*•mTf co co
feiha
It' 1sil

8om <o
o o 88 82 S88
O* — * oo •
v> cs
^ ** 00 oo
vo * -;
nn -J
t~
mmoC N
t»- vo
N NN ft vo~ co
i" «r. vo* < co f-7 o\

a co vo <N oo 00 VO ^
CO CO CO p oo oo oo <s m
3 ^f ^* co co co
in >* co csj --
co co" co co co' CO
p <s o\ t-- >n ** fv| iiihii
co' cn r*i <s" cj <s

tfllllf
ro <n oo is vp
s ft,
Jjf 5? *nnNN oo co ft in co O
8
co co <v| cs <n <s
mlln
f'Sis
e eg SSflS^
oe «.*« Z.& s »,
M X 8©mom oo©o©o
» N in o <o m
O OOOOO
oo oo f^ m in m
3 r- ft
5 — ^r n * in oo — 8.Saiioa«
< 9 t-
CO co co
oo
CO «S
00 VO CO f-* 00 in* oo vo" •*" CN* ©* ft"
CL,

O p^
00 t~- IT) e>
sa >s O—
*m
«S p- £ vo o
q»« 8£$S32
A ^ CO CO CO CO
Tj-

co co co co r4 cs co cm ri oi cs r4

ft, «*tnn 2
(O M SS w
)£ N °^ ">
fn rt
N 82228©-
s

© o
m <N 8 ©©©©
"5" co
p ©* m go oo cs
© © <*" m n
©
co"
*t co co
(«•" ^—
co co
in rsf
-h* oo*
co rs cs cvj c5 -i
©* (-"
©
on,
t-T vo*
fN F< FH
<s"
©"
F-l F-« F-,

*
p VO in VO
t-; vo -N r-. e-. oo cm
00
cs t-> >n in vo cN
*hnN oo *
a 2
_o
II nN N <H CS <~4 F-4 ~H
o\ vo •* ri cn ©
tj
s Ifllll
CO

en
Sft7 ^^ CO CO cs
mom©
rs ts f-. f-,
<n © •n
—m©
© cn »-i
in
<s
3 ?
I I I I I

e
o Irs* 8B
!§ 6
u
&<
8 •-
ail PI
**
_:
©
J"*
"" *- to Jlilli
a.
< Spin ?1^
8 I '

522 PRINCIPLES OF REFRIGERATION

^ ^ ft +
^^©
ft >n
^ vo e>j
en en
»* p- en ft v* -j
nNN -< nH CS
ft
-* O ©
»-i
ft
ft
3

en o
r- en
en en cJ r^ P4
m O
CN
OOtOTf
*-* i-H i-H
NO
»— »—
O
— 00
i*
© © en© © r-i oo
ft
2 8 #
<

s*

a o o oo©
-h »h m fn <N
© © ~ S8?
~. rt ft O* OO
"3" «0 •* (S_ <S en O O O ft ft ft "V ^1 '"I '*!.

•^ en pi w ©" n" « d « oo" r-"


'"i,
oo* t-" « vT 't
«
m O vo
-O — f £ p- © vo — vo <

NN 9; ft ft 00 00 00 00 oo oo p- r- vo
I ^H ^1 -1 © 1-H dd dddd ©©© © © ©

s3> — « «o©
ft r- •* cs oo m <s
= 8© o ©o
p~
a, n (S M NN >H -H ft

O2 Q O O ©
oo m oo
© ©©© ©©
fS ft m « p- en
o o o s
in 8 en
e A
oo oo «> eN <n h »n« r»_ ©„ ft_ en vq_ m
D "a"
© ft" oo" oo" r- «? *"
r-" K m" m* en* en ri
M
e
'SS
c * ^m
r- P-
N r- vo vo 3 vo vo s ft
VO in
VO
in m
« M o oooo
t-»
©odd©©" ©o O© © ©
e
o
S.8 9 »s <s cs <s cs
oo so en P4
i— i »-« »~< »— i
© ft
»—© i 8 S3 O 8 t--
ft

o
u a mi» ©©©© O © o
00 8 in in
I
L.
3"
i»?
IO oo_ eN t~_ <S
in u"T
f© nSNWn
^ oo
\e « m" ^" <t •^" e»T
«—
en
es 00
es
P-" vo" vo"

^ * ?®
* <s
f»l _ «
^ >-< _ .
*
•*
N p»
Tf «* <* * en
<»i •-<
^^©
eS
^ ft oo r-
<-h
en en en
I odcidd © ©©© ©© ©©©© © ©
ui
J a,
a^ Oh»>nC>
nNNNN t~ m en ts © oo
©m
©m
r- >h
©o ft
ft ft
<
ea ja

p © ©* S8 © o o©
en eN ^ oo
8 P- en ©
s-
ft «> t—
n
Si *
» mM
« N OS VO en
_
ft VO^ eN_
ft"
*. "^ "*. "".
^" en" en en en" en" en en of r-f r4" N cf N « - >i
oo
^3> r> O« vi »e vq -h p- t-. oo e^j es p- in m vo fN
ca
o £ h t^ rioo* ^ r— P- -^ i— ft
« ft vo © fN
NN I I

i. *5|- en en CN fS ii i-i

«n © m ©mo <n © O <n O in ©


in
3 en en >-c -h es cN
I
I I I

.s6
oja
&
gee i- id
H *j *j
S « *
5 o o ff o o
>3 mm
35 p- ft SP ft
TABLES 523

v> o <n is oo
N «»»» x »nO
vO w> ^ 1
1*1
oo vo
>-c © ©m©nN
OS oo »ift
© <r> s
X
I c> »n is is is IS is IS IS IS IS is IS IS IS IS is

<n is os r-
(»1 nNNN
«*• * N»
IS IS »-t
r- <r> rr> is
1— SSSSg; ft
£ § *3!8
£ eidd
i

1]1 fc
x
A3
a
oo ©oo ©vn ©^
© >rt
© ©©
Wl OO 00
o
00 S8822
0.
tt

s -. a
00 O 00 OS
f*S 32,940 30,120 27,350 IO t-A » ©_ © *o en ^t no -«*>
"3"
«* •*"
1 s
r* oo *rf en i-T oC
t-^ is*
^ ^ os* m is"
roi*s IS IS
oo"
T-l ^ i—* r^ i-H

«"> oo O t»>«n >n is ©


IS W> 00
m VO f**» © t-
m **
NMO ON h« oo r- ve >/i 1- s On oo Is- ^
—i os oo
s »-H *-* »-H f-« f* f* IS ,—1 tH M 1"^ ^"* HI
8SSS l I
11
|

m
l in
a,
43 •2 £ «?
n C) (S NN •<*IS OS I—
IS IS <m
en
SSSsaS; f ill
is IB
1 1
si
£
©©go©
» m9 om ©©©
moon P.
iiili W
9
os r» r> oo
*"
"-i
©*
»©m
©*
15,670 13,800
12,010
oo
©*
OS* f»* IS* 1--" ve Tt* IS* OS* 1-*
PQ
N N N N (S is «
o\"
—«
II I
0. u.

£ m©
W vn ©
Bin m• <rt <n
•*
©m
^ © lO ©
in n N N
oo ©
1*1 «n OS rr,
vn w> Tf ci is IS
*

nNNNN is os ve
— v-> * m r~ m is ©
s© ©©© r~
43 lit is -* OS

© ©© o©© o
s ©» Os^ * 0_
IS OO IS
l»>
o_ so n os in N
r*S

1O *J
<n r^ *n
m ch
>/->
^,
3 i«"©* oo" r-* ve" so" -* ©*
•* c" O* ©*
IS* oo* i—* ve <n
« IS is — <-i -<

iKlliilll

oo
I- © ve vn ve vo -< r~ f- 00 OS is r~ m w> vo is
-« r^
•* nnn oo
nn
<« * —
N «<S
r- •* i OS — OS VO 'l- IS IS©
^iillllill

m © <n © ,. m©
N -©
N vn sn © © vo ©
—m ©m
-« IS IS
1 1 1 1 1
8 la|
4|l5f|
.
iiHflllt
3 Out IlE s Il| s i
.&6 e o In
•z SllJUliL
S3
temp Water Water
temf
Water Water
Temp.
Water Water
! gffif pi J
High 75° 95° Med. 75° 90° Low 75° 85° H 32 o o
524 PRINCIPLES OF REFRIGERATION

TABLE R-13. Air-Cooled Condenser Ratings


TABLE A. Basic Rating Table 40° Suction 90° Air Entering
Compr. Capacity-
Btu/Hr 120°
Condensing Temp. Fan Motors Fan
MnHel Dia. Speed
No. Tons Btu/hr Cfm Inches HP Total Watts Amps Rpm Rpm

RC75 0.88 10,600 1,125 14 A 78 1,500 1,500


R.C100 1.11 13,340 1,450 16 A 175
230
1,050 1,050
1,140
RC150 1.61 19,300 2,450 18 * 1,140
RC200 2.36 28,300 2,850 20 * 230 1,140 1,140

RC300 3.18 38,200 3,400 20 * 330 1,140 1,140


BD300 3.31 39,800 3,450 24 I 1.3 1,725 625
RC500 5.14 61,700 5,500 20 2-* 460 1,140 1,140
BD500 5.13 61,600 5,500 30 * 1.7 1,725 500

RC750 7.92 95,000 6,900 24 2-i 860 1,140 1,140


BD750 7.57 90,900 7,100 36 1 2.8 1,725 455
RC1000 10.34 124,000 9,100 20 3-i 1,290 1,140 1,140
BD1000 10.00 120,000 8,800 36 1 3.2 1,725 485

BD1500 17.32 208,000 14,750 48 It 4.8 1,725 325


BD2000 19.75 237,000 13,600 48 1* 4.8 1,725 325
BD3000 34.64 416,000 29,500 48 2-1* 9.6 1,725 325
BD4000 39.50 474,000 27,200 48 2-lf 9.6 1,725 325

Courtesy Kramer-Trenton Company.


TABLES 525

TABLE B. Correction Factors for Suction Temperature Lower than 40°


Suction temp. °F -30 -20 -10 +10 +20 +30 +40

Conversion factor 0.76 0.81 0.85 0.89 0.92 0.95 0.98 1.00

Courtesy Kramer-Trenton Company.

TABLE C. Correction Factors for Temp. Diff. (Condensing temp.—ent. air. temp.)
Condensing Temperature ° F
entering
Air D.B. 100 105 110 115 120 125 130

70 1.00 1.17 1.33 1.50 1.67 1.83 2.00


80 0.665 0.834 1.00 1.17 1.33 1.5 1.67
90 0.333 0.50 0.665 0.834 1.00 1.17 1.33
100 — — 0.333 0.50 0.665 0.834 1.00

Courtesy Kramer-Trenton Company.


w w

526 PRINCIPLES OF REFRIGERATION

ii& s on «k» f->*<0 vorj maw <oot «w« o«k —


•jo*
•-»- •--•- www wnn
*
n •» •» « Mnn n * * id in m <o fi-fiZ '•S^ *•>*« e>ta «0« »-K

oo-to w--« woo-* on- k 1 •own n»m t-"Ow oo -to 4 woo -to
*
#
5k*> ont
*>•—•—
hikS
—*-•» ?{*<•> w
WWW W
owin now •okqn nmoe
•» •- »-ww w nnn o-r rtKb «tK«- moon »)»A ibn k?
»-»- w. w w« rtn^- -t*«> in9<0 t»K

ft* tn«r
rtix « — o nooo «k *»«-o nooo niv «-^o now
++ *
P-'WO V«K •»•«••*'• >o> ++ ' FN'ri f'tjk *««»»OB *Oe *-*' a-^wn < «d k «««*»»oi •»:•**•*• tot »•*

2 i
I
Q i
M W WW'
. e i W U| 0> •«•»•«• ton©-*** tot-ono- to* Iff Qt aOt*0«»OB *Oca0*tOt (OttOBkOB 10c WW*> 1 *»t*0**0» tOttOltea i«ck9»e* (0HO>

"E 4
«
4)
e
M o e KK 09 — W W*-W q»- wkw «*n ON K«K *» K •*

wnn »DIMQ W
**««> H»-^00 -eMO-totto.. **0* •«•••*•• *t»aoa»et
e ss ---- w<5 ww0* 00*00
n^n00 t> O*
<o«»
afltaai

in
>.
1
#
M
id
* 00p-9»
(jtaf Q« »«*»»» tOctOMO* tottOMO* «ot *f aJ Q, aOtiOitOB tectetiot aecaOBtOa te« («,*)• «£K«O9&03 UT'.-Jniot cocientos US* COB WW eoctot
*•
i
1
it
i
8* 3o"8 k«*> « 8$S88K88 3 w* -rmo oo pm w
PQ <o;n k«o» «oiom «oo
1 i " *** ** ' ' ' ' - ^ «' « <> ++++++++ *
'*-^>-*wwn^rift«K
I
1 2 4
I i
cjkS Snn kwo»
«NO
mkw HI
>ee<im k«» wkK own , k no;
e p. cJ <r <d ww •*» «*.omo» *©» *
^ ci V <o* n'ciin »«•«=•»» «* 'r. cj-f y> ajriui *»•*•••«• »*t *»t »ot aot»«

ove
o
y o
m
m wo ->nn an«-< Oo f> 0"0 m«K
£ •
M"M"M '
**oo-o <omm mu*-ti -okk
W
•»— Sw« ««* <t wn <**fQ nooo o*-w ooe>
m^
it w M
L.
II
M «
• 5«* ©•*©• K«« — ^» o>«* mom « <on w_ w o; on co
n w<b '-"•o-- *»«**«*t """""- (

S. wio* ri
•- *-ww »n*>
-=»
"28 ass sj'ss
(ij

»-
ri *J *± *1 r^ CO 't
wn ««« N»e> Iti <Ot tOttOt fO= aOetOt atX 10c

4
«*
WWW PI H WWW W
WHW Ho»-n <maq « k no>m o <0 w eo *n *> -to
in Ko>->,nwK mK» -
** ,-*-,_ _ww www n •-•-- www wnn n*"t * -ww O*no> -ton cowk >-«q mo
>o^«
nnn tt« m«« kkS oo*

111
-I
BO o*-w rt^io <fiA>p •>««» 2-12 222 2 •O-O K00O a-^— a----- ^-a-a-
_^
««<e . Koot>
^
--»-»- >->-•-
00 00 00 CO 00 CO «0 00 00 00 00 00
•*
•-
00
ooo S8S ooo ooo o WWW WWW ««« WWW WWW WW *. ft

2 H-^-H- &fcfc M>iku.


hHh •tHH h-Pi- kPF ift.it.it. u.
sttst sitfc sttt tfcst stftit stjt
it
MiAtfi wtf»(A
*yitf»t/» <y»i/»v» M«AM MM MVltA WIIAIA MM
vi (A v> W1 Irt

5 s
*i
St
I:
00
I w
'

TABLES 527

88S3R3WS8SSS28S8 S5SSRS?«:8 2fta5S3RS

88? 5.8E 25*1 558 §25 k« 8JSS835SS632 2RS8S


8Sr> S3? SSe KKe •>»>•• 06
« •I
a
»«« «mr> »** rtw-0 *kk £§ M
<•
—o
ft. °c
^
8.
an m — o nqe t

II
,2
a"**
»—m K<i<
ri n» •»«•«•• • p| H) 0t •OI«OCMt
HtiMM) l «««t^«^ *«ie* 8 as
t>E
1 1-8
tt £'=
e
*• — *«ei -0 KO »
*+«.« »£
*«•) orj'O o°«*" '

88 «D338 833' • -
pin m<OK o>oci
3
Si I
o.

E o
NO — eflt—a*W •«(•<
u
38* >

c 81
M
C £=
2 .e
83 388 £88 3«.«, ,. 83 388 R88 S-.. <»-
* '-• J tin B»w >ok
1-
*
*6 3$
hkI >eein kh» *s US
i -own ism*'
o
'
m***<o" Wrim' • fa

*; si*» 1

• 2 <1
«o «t>m *k So
din «*- ««•-
K3 cie>K otfiH <0«c
"«£ 28S S8 91

r+
_c

85! 35
2m
w—• kmk •-

?«# nKQ
«e>d worn ' e> o'? •c
««• o>>-«
H oio
fac the
Si
°-3

QQ»
KSn »•>«• SKK K<« 522 n 5 sec. availa

«*m K«a
Sa-
«««>> n"»»o
m** *>ss; aia —"tjj S»i»IE!i2HMB9 scale
of drainage

V
of are

a 80%
free
on
533 «33 333 333 333 3* on
on
shell,
flow,

maximum condensers

U.M.H. U.U.U. U.U.U. U.U.U. U. U. U. H.U. based

'SfSsS 555 555 555 52 SS9SSS


atetat atatac aeotat atatae at at
Sat at ™55>
SSS
«v>v> v>mm »S based

based
of
turbulent

EEEEEEEEEEEEEssss oti t oo o S o See eeo 5 S are


is
SRF
bottom
exceeds

capacities
rating
capacity
Model
in below

u d .ity

= i-
" t •
3-8 8 !??<
s.ss
•Noir
Pum
•Con
"5 s|
NOTE


528 PRINCIPLES OF REFRIGERATION

2 3 4 5
Gpm water per tube

Fig.
TABLES 529

ou

130 25

1.25

1.20
i 20
|. C
3
1.15 E
I" 8.15
1.10 ?. E
a ca &
1.05 v^/V
^ ;s S 10

1.00 ^ .v
fe ^ «, 120
>>>;

0.95 s^ t^> 90
110
105
100
5

80
0.90 n
-40-30-20-10 10 20 30 40 50 2 3 4
Evaporator temperature *F Gpm per ton

Fig. 2 Fig. 3

30 20

At
1

t
20

10
4 Pass

W
Am
iw uu
f
j
Jk*\
l

8 zz
i //'//
§ 7
| 6 M VJ '///*
W-l >/.,
Wtt tV?
/
Z
i ///// //
I'
* 4 ////, '//
w //t

/. '//,
'/;/ //
'//a /

m % ///
'///, (/ V
7
Jf '//

M% 2
*

Gpm
3
per tube
45678 10
Gpm per tube

Fig. 43
530 PRINCIPLES OF REFRIGERATION

TABLE R-15. Quick Selection Table—Water-Cooled Condensers


Sizeand Type of Connections
Water Refrigerant Cleaning
In and Out In Out Shipping Tool
Catalog Stock •Nominal SAE SAE SAE Dimension in Inches No. of Weight Catalog
Number Number HP Rating Flare Flare Flare Height Length Depth Sections (Appr.) Number

EL-33 1-EL i ¥ i * 8f 18 1 1 13 836


EL-50 2-EL i ¥ i # H 21 1 1 18 836
EL-75 3-EL \ ¥ i * lOf 21 1 1 20 836
EL-100 4-EL 1 ¥ i * lOf 27 1 1 25 836
EL-1J0 5-EL li ¥ i * 12f 33 1 1 30 836
O.D. Swt.
EL-200 6-EL 2 ¥ i * 14 34 1 1 35 1036
EL-300 7-EL 3 ¥ I i 16i 34 1 1 39 1036

* For Booster application —


Use one size smaller when used in combination with air-cooled condensers.
Courtesy Halstead Mitchell.
TABLES 531

e
©o oq o r-
o OPPPOPO
en r- © © o iiiiniii 88888888
i

• i** t*t t> r^ f^ <*i

«Q Q so *o «n «" en


onomoQn
* m 888.
<i •> «s «s «»>
IS 8S8888888
to a ^<ftsOhflOO\OH
6

nintocodntt r*- «n o **> *** © t> mooooooor- oooooooo


m soonsoonso
^" ^tN c* «s m ro en

o op opoo ? mSooS^S 8S888°888 888S


si* ON' [888
L.

*
o
H VO\AOO
oo oj cj «n * r;
n in vc o\ ^ ^ d
i*»so
S© 00
«rt <*
-"^ t""
»•* so
»-*
C* ooMcntno
nifHOH^^iH^

fl o
w dNwm
0««ONOO
so * ^tfcoQNtnQO
u s
> s
o
ri i^' t-" o\ ci o\ ri
^t^o^f^-o«oo
t« T,
C
*
i > «* I*, oo w* el in ^
©
**)sq ^" iN ~h
«> oo ** * ^ oo 'fr *«
8! «o »n vd on -* •* I-*
n^-inwvorsojOiH ^ r^ ^h
it ^

?" qooqons imo*nocoo o«^o*no>nooo inininmininpp


1 ° ninvoaoONtt) rit^r<ic^rii^«np
n m * «n so so i^ ft o rt«WH«MN(*l
ao 0\ o

£ -a

» « §Hn O; 00 l~; r-; « O M missoq^q


<4
ri ^
in f- o\ c$ t-^ v* r«" o\ r* ^r oo **

"S I
x"»«NNNN
o-
_
•o
rt
8.
.Sf

* U £
J \q i-;Cj »m
«n *n
« 00 m vi vd od o' r* od mcn^iosot^t-osO tnptocsoomOO
*^oosor*^ot^M«
rK
2 O fO
a\oo r»
v" r" ©* ci *n os v"
_4 ^4 »-. *4 (S| •^*\OO»^<^00^
IU
-I
a
oq oo t-~ vc> in oo •* is *-* «r» oo r* oo r-
2 f*i ^? in r^ ol »* i^ • -" * oo rj vc •-.©
ON 01 >
< «« « « o r*
<n in i^
f>«
vo oo
ts cs «s

(3

s oo so oo © o» «n
h o\ooao
ri ^ in Koo^in vomoosfoosoosoo
r*m^^»Asi5 oi>ooi

_m<mT*»<Stt<S

r;««NqrjN o ^ q n vj » in
ri ^ in r^ on •« *d vo o\ m vd N n«n^mmsosooo<
•ri «-<

m <-> m m ^ r4 <4 (S

& 3
bea.
o i-« «s « <»» rs m ^ K so t^ o 888SSS88i
-««MNNnn'
I's^
**i m
^f
^m
* «n no QOQNVlvOOOO'tt toento«Q»
cs en en ** in no
vo « vo « vo <e io o) N«N«NNMM
MMM09CAMM aiBiaaeoeotaMcota to ce co co to to « to
532 PRINCIPLES OF REFRIGERATION

TABLE R-17. Evaporative Condenser Ratings


TABLE A. (In terms of evaporator load at +40° F evaporator)
Cond. Wet Bulb Temperature of Entering Air (°F)

No. (°F) 60° 65° 70° 75° 78° 80° 85°

90 2.6 2.2 1.9 1.5 1.2 1.1 0.6


95 3.2 2.9 2.5 2.1 1.9 1.7 1.2
E-80F 100 3.8 3.5 3.2 2.8 2.5 2.3 1.9
105 4.5 4.2 3.9 3.5 3.3 3.1 2.6
110 5.3 5.1 4.8 4.4 4.1 4.0 3.5
115 6.3 6.0 5.7 5.3 5.1 4.9 4.5

90 4.4 3.9 3.3 2.6 2.1 1.8 0.9


95 5.5 5.0 4.3 3.7 3.3 2.9 2.1
E-135F 100 6.6 6.1 5.5 4.8 4.4 4.1 3.2
105 7.8 7.3 6.7 •6.0 5.6 5.4 4.5
110 9.4 8.9 8.3 7.6 7.2 6.9 6.1
115 11.0 10.5 9.9 9.3 8.9 8.6 7.8

90 8.8 7.8 6.6 5.2 4.2 3.6 1.8


95 11.0 10.0 8.6 7.4 6.6 5.8 4.2
E-270F 100 13.2 12.2 11.0 9.6 8.8 8.2 6.4
105 15.6 14.6 13.4 •12.0 11.2 10.8 9.0
110 18.8 17.8 16.6 15.2 14.4 13.8 12.2
115 22.0 21.0 19.8 18.6 17.8 17.2 15.6

* ASRE standard rating conditions.

TABLE B
Evaporator Temperature Correction Factors

Evaporator Correction Evaporator Correction


Temp.(°F) Factor Temp. (° F) Factor

50 0.97 1.11
40 1.0 -10 1.16
30 1.03 -20 1.20
20 1.05 -30 1.26
10 1.09

Courtesy McQuay, Inc.


TABLES 533

TABLE R-18. Water Valve Selection Table


,
300 1 1 1

200 flowchart 2^,


All R-12 and R-22 in h" through US"
valve sizes only
2^
100 —i"p '

——
[<1

80
US

J
,
5
60
50
^-
•|

8.
40
30 •-< ^/C^X, < •^ -
t3 1

^^-^

Eallons
o si

c
i
\^
1 io
t 8
£
« 6 ""^ -^
= : c sample w lw>
VIM tion proce mire
4
i
lection 14
3
i

2 t1

1_36*- 60 '
1
t
„ 1

vz_ / / 1 / /
BO 1 Water pressure drop through valve (Ib/sq in.)

1 30*- 50 "
2/ 5 -t-io, JU/ WT^WH Wf
r /
V
| 1

//
24"-40 t
| 1

1 18*- 30 4L/
,'
Q.
E
£
g 12°- 20
-- ~ /--V

£
^ 6*- 10
<0

x •-
t "F Psi

TABLE R-19. Centrifugal Pump Capacity Table


Series (1531) Capacity Chart
160

|
140

120 ^ ft*

^^
E 100 ^fc *
«31-«
I» ---^ =J-«
a* — SJ r^"
I 60 «&£
—JH*40 ] « i-a
20
25 SO 75 100 125 150 175 200 225 250 275 300 325
Capacity in U.S. gallons per minute
Courtesy Bell & Gossett Company.

534 PRINCIPLES OF REFRIGERATION

cn o *r (0 o»

8 CM

3 » s a » s m
in »> in to m to 10 cn cn CM cn 40 O
8 CM CM to n 10 cn
CM

Si
a a » 8 CM cm s 8 co in CM * CM CM CM CM °? 1*.
i

8 CM co 2 in 10
2 2 CO

in
a s s s 8 ^1 ^ cn M eo co CM
^ - CM CM co W in - o 2 10

c CO CM cn «o m
a CM

CM CO » 10 CM in «o o
cm CM

i
Si cn a mt (0 o cn

SJ CM
.2
s !X M> •» (0 10 f» (0 en
c in
e
CM 1 8 . CM S s. Ok » CO m 10 CM o "*. co
tl
s •4 2 CM CM * w in n oo co

s en «n co » CO
in
CM CM
& -l~ CO

> o
7 Z B 3! K «. 1 CO s <M
co © CO CM to o C3 .

Ui
ae 8 o CM CM to co to 9*
2 a co
CM
•>
S oe a s cn «o CO O (0 O (O CO
g K CI CM CM
s ae
UJ
3 O
z
u.
Ul
- » m en
ae 8 S1 o• co 3. ^ O) cn in cn to 00
Ul
oe a u. 4 CM CM co or in co <n CO CO
3 (O
Ul O
i 5 s
CO
o
X » 5f s -- in 3 &£ r* 40
(M
CM
co
in cn
40
to
ad
in
o
co
CM
(0
•0
s
40

i Ul
oe
-I-
CM CO CO

> & 3 8 » » CO O CO o CO
w 8 CM CM CM

£ in cn •* in o en
in CM

o o
CM in CM •-. oe ^ CM <o
E CO *
L. g w* tM Co CM CM in
« if! o a CM 40 40 CO

I-
8 — < •4 CO CM CO co

is
8 . * - <n m a cn CB 10 in CM CO CO 40 o> CO

8 •i CM - CO CO m in CO CO
CM CM
in
CO
CM

3 » CM CO CO
s 8 4 CM
CM

Ul
_l S CO <* C9 CM CO CO (0 CM 40

g — M, "O CM
CO
S O m M o * CM o m (0

2 8 im m CM CM <o <o in

1 s
8 *4 H M • H M M * ta < M a a a R R a 3 8
a 5 - g cn cn m cn ca * in cn o cn co CO
9 -" tsi 2 CM co" in to cn CO 40
3 9

!
ft 1
I & si II ll si It SS Si IS SS s 3 1 i 1 p P H 1
Index
Absolute, humidity, 59 Brines, 199-201
pressure, 5 British thermal unit, 17
temperature, 13, 14 Butane, 297
Adiabatic, compression, 37-39, 82 By-pass, condenser, 263
expansion, 37-39, 207-209 cylinder, 242
Air, composition of, 57 hot gas, 391
conditioning, 122
dry, 57 Canning, 123
latent heat of, 62 Capacitor start motors, 414
quantities, 57 Capacitor start and run motors, 415
sensible heat, 62 Capacity, centrifugal pump, 278, 279
specific heat, 246 compressor, 87, 205, 206, 210-214, 222, 223,
standard, 58 229
total heat, 62 condenser, 245-253
Air circulation, over cooling coils, 174, 179, 185- cooling tower, 259-261
187 evaporator, 170, 174, 177-179, 180, 181, 228
over condensers, 245-247 expansion valve, 318
in refrigerated space, 131, 179 system, 85, 230, 239
Air change load, 145, 152, 153 water valve, 265-267
Air changes in refrigerated space, 145, 152 Capacity control, compressor, 241, 242, 363
Air cooled condensers, 248-255 condenser, 267-271
Air cooled condensing units, 79, 223 evaporator, 241
Air cooling coils, 180-187 system, 239-241, 243
Ammonia, anhydrous, 293 Capillary tubes, 318-321
piping, 381 Carrene 7, 297
Antifreeze solutions, 201 Carbon dioxide, 287, 293

Approach, cooling tower, 259 Cascade systems, 395, 396, 399


Atmosphere, 1, 2 Cases, display, 140
Atmospheric cooling tower, 260, 261 Centrifugal compressors, 203, 352-363
Atmospheric pressure, 1, 2 Centrifugal pumps, 278, 279
Atoms, 10 Centrifugal refrigerating machines, 363, 364
Autolysis, 125 Clearance factor, volumetric efficiency due to,

Automatic expansion valve, 298-302 207


used as condenser by-pass, 301 Clearance gas, 203
Azeotropes, 297 Clearance volume, 203
Centrigrade temperature, 12, 13
Bacteria, 126, 127 Charles' law, 26, 29
Baffle assembly, cooling coil, 181 Chart, psychrometric, 64-69
Barometer, 2 pressure-enthalpy, «9, 93, 98, 101, 107, 113,
Barometric pressure, 2 116, 118, 119
Baudelot coolers, 188-190 pressure-volume, 34-36, 38, 39
Booster compressors, 395, 396 temperature-entropy, 103, 105
Bourdon tube gage, 5 Chillers, oil, 379, 380
Boyle's law, 27, 28 liquid, 188-197, 201
Brake horsepower, 216 Chilling coolers, 132, 133
Brine cooling, 199-201 Chilling of foods, 132
535
536 INDEX

Cloud point, 345 Compressor (s), pumping limit, 361


Coefficient, of expansion, 24 reciprocating, 203-223, 334-344
of heat transfer, 148 rotary, 203, 250-252
of performance, 97 selection of, 222
Coils, air cooling, 174, 175, 180-187 service valves, 78
bare tube, 165 single acting, vertical, 334-337
dry expansion, 164-177, 193, 194 speed of, 220, 221, 362, 363
finned, 168, 170, 172 surging, 362
flooded, 164, 194, 195 types, 203, 334
forced air, 185-187 valves, 337-341
liquid cooling, 188-197, 201 volumetric efficiency, 206-210
natural convection, 180-185 volute type, 357
Cold storage, 130 water jacketing of, 219
Cold storage rooms, 130-132 Compressor capacity, 87, 205, 206
air circulation in, 131, 179 control of, 241, 242, 363
air leakage into, 145, 146, 152 ratings, 222, 223
air velocity in, 131, 179 variation with condensing temperature, 212-
humidity in, 131, 179 214
temperature in, 130 variation with suction temperature,210-212
Compression, adiabatic, 37-39, 82 Condenser load, 96, 244, 245
compound, 395-401 Condensers, air cooled, 248-255
constant pressure, 25, 26, 219 air quantities for, 245-247
cycle, PV diagram of, 204 air temperature rise in, 245-247
heat of, 95 booster for, 255
isentropic, 95, 219 capacity, 245-253
isothermal, 27, 28 35-37, 219 capacity control, 267-271
polytropic, 39-41, 219 chassis mounted, 249
processes, PV diagrams of, 34-36, 38, 39 forced air, 249
ratio of, 208 function of, 244
wet, 219 gravity flow, 248, 249
work of, 34-41 location of, 249
Commercial refrigeration, 122 rating and selection, 251-253
Commercial refrigerators, 139 remote, 249
Compressor (s), action of, 203, 204 temperature split, 252
bore and stroke, 342, 343 Condensers, water cooled, 253-259
centrifugal, 203, 352-363 by-pass, 263
classification of, 203 capacity control, 267-271
compression efficiency of, 216-218 cleaning, 255, 256, 271-273
crankcase heaters, 348, 350 double tube, 255
crankshafts, 343 rating and selection, 257-259
crankshaft seals, 343 recirculating systems, 253
cycling devices, 231-239 scale factors for, 255
cylinder arrangement, 337 scaling rate, 254-255
discharge temperature, 83, 96 shell-and-coil, 255, 256
function of, 78 shell-and-tube, 256, 257
hermetic, 79 water piping for, 253, 254, 278
horizontal, double acting, 334-337 water pressure drop through, 253, 254
horsepower, 214-220 water quantities for, 245-247
impellers, 352-354 water regulating valves, 265-267
lubrication of, 344-350 water temperature rise in, 245-247
mechanical efficiency of, 216-218, 221 water velocity in, 253, 254
oil check valve, 349 Condensers, evaporative, 264, 265
piston displacement, 205 capacity control of, 267-271
pistons, 337 rating and selection, 265
prerotation vanes, 357 Condensation of vapors, 49, 50
pumping head, 353-356 Condensing pressure, 83
INDEX 537

Condensing pressure, effect of high, 101-103, 208 Defrosting, water, 389, 390
effect of low, 267, 268 Dehydration, of foods, 123, 131, 179
Condensing temperature, 43, 49, SO, 83, 101— of refrigerants, 297
105, 253 Dehydrators, 297
Condensing units, 79, 223 Delta connections, 407
Conductance, 148 Depression, wet bulb, 61
over-all coefficient of, 147-149 Desiccants, 297
Conduction, 14, 148 Desiccation of foods, 131, 179
Conductivity, 15, 148 Deterioration of foods, 123, 124
Controls, compressor cycling, 231 Dew point temperature, 58
condenser capacity, 267-271 Diagrams, cycle, 89-120
electric motor, 418-427 pressure-enthalpy, 89
evaporator capacity, 228 pressure-volume, 34-36, 38, 39, 204
pressure, 236, 237 temperature-entropy, 103, 105
refrigerant, 75, 76, 298-333 Differential, control, 232
temperature, 231-236 Dilution, of oil by refrigerant, 290-292
Conversions, energy, 8, 21 Discharge, gas by-pass, 391
head-pressure, 275 head in pumps, 279, 280
heat-work, 21 pressure, 83
pressure, 4, 5 service valves, 78
temperature, 13 temperature, 83
Convection, natural, 15 valves, 337-341
Coolers, liquid, 188-197, 201 Discharge lines, 372-375

reach-in, 140 311-314


Distributor, refrigerant,
walk-in, 140 Domestic refrigeration, 122
Cooling load, see Heat load Double-pipe condensers, 255
Cooling towers, 259-262, 271 Double-pipe evaporators, 188
Copper plating, 290 Driers, refrigerant, 297
Corrosion, 272, 273 Drift, water loss by, 260
Crankcase heaters, 348, 350 Dry bulb temperature, 61
Crankcase oil equalizing, 382, 383 Dry ice, 49
Critical pressure, 50
Critical temperature, 50 Efficiency, compression, 216-218
Cycle, compression, 203-205 mechanical, 216, 217, 221
diagrams, 89-119, 204 over-all, 216-218
reverse, 392 system, 97
simple saturated, 92-106 volumetric, 206-208
subcooled, 112-117 Electric motors, 407-416
superheated, 107-112 Eliminators, vibration, 367
Cycling controls, 231 Energy, conservation of, 8
Cylinder heating, 209 definition, 7
Cylinder unloading, 242 equation, general, 30
Cylinders, compressor, 337 heat, 10
heat- work equivalents, 21-23
Dalton's law, 58 internal, 10, 11
Defrosting, brine spray, 389, 390 kinetic, 7, 11
electric, 390,391 molecular, 10
frequency, 388, 389 potential, 7, 11
hot gas, 391-395 transformations of, 8, 21-23
multiple evaporator systems, 391, 392 Enthalpy, 50, 51
off-cycle, 388, 389 of air, 62
re-evaporator coils, 391 Enthalpy-pressure diagrams, 89, 93, 98, 101, 107,
reverse cycle, 392 113, 116, 118, 119
Thermobank, 393, 394 Entropy, 51, 52
timers, 389 Entropy-temperature diagrams, 103
Vapot, 394, 395 Enzymes, 124, 125
538 INDEX

Equalizer, crankcase oil, 382, 383 External work, 30-33


external, thermo expansion valve, 304-306, External equalizer, 304-306, 317, 318
317, 318
Equivalents, head-velocity, 275, 276 Fahrenheit scale, 12, 13
heat-work, 21-23 Feather valve, 339
pressure, 5 Finned coils, 168, 170-172
pressure-head, 275 Fins, advantage of, 172
temperature, 13, 14 inner, 172

Ethane, 287, 288, 297 Flammability of refrigerants, 285


Ethylene, 297 Flapper valves, 339
Ethylene glycol, 201 Flash intercoolers, 397
Eutectic solutions, 199-201 Float, high side, 321, 323-325
Evaporation, 47 low side, 321-323
Evaporative condensers, 264, 265 Float switch, 325, 326
Evaporators, air-cooling, 174, 175, 185-187 Film, air, 149, 171
air flow through, 174, 179, 185-187 oil, in evaporator, 291
baflOing, 180, 181 Fluid, flow, 274-278
bare tube, 180, 181 friction, 278
Baudelot, 188-190 pressure, 274, 275
capacity, 170, 174, 177-181 viscosity, 291, 345,346
circuiting, 175-177 Fluorocarbon refrigerants, 287, 288, 294-297
coil and baffle assembly, 181 Foaming, oil, 347-350
defrosting, 388-395 Foods, absorption of odors by, 132
double-pipe, 188 blanching of, 133
dry expansion, 164-177, 193, 194 chilling, 132
film coefficients, 171 cold storage, 130-132
finned, 168, 170, 172 dehydration, 131
flooded, 164, 194, 195 deterioration, 123, 124
forced air, 185-187 enzymic action in, 124, 125
liquid chilling, 188-197, 201 freezing, 133-139
natural convection, 180-185 microorganisms in, 125-129
plate type, 165-168 packaging, 139
rating and selection, 177, 181-187, 195-197 preservation of, 122, 123, 128-140
shell-and-coil, 191, 192 Freezing, of foods, 133-139

shell-and-tube, 192-195 temperature of water, 12


spray type, 195, 201 Friction, fluid, 278
tank type, 190, 191 head, 278
losses in piping, 278
temperature difference, 173, 178, 179
mechanical, in compressors, 216, 221
types of, 164
Frost accumulation on coils, 165
U factors, 171, 181
Frozen storage, 133-139
Evaporator pressure regulators, 331-333
Fusible plugs, 384
Exchangers, heat, 114-117
Fusion, latent heat of, 19
Expansion, adiabatic, 37, 38
Fusion temperature, 19
coefficient of, 24
of water, 12
constant pressure, 25, 26
isothermal, 27, 28, 35-37
Gages, 3-5
of gases, 25-28 Gas(es), 12
of solids and liquids, 24 adiabatic expansion and compression of, 37-39
polytropic, 39-41 Boyle's law of, 27, 28
processes, PV diagrams of, 34-36, 38, 39 Charles' law of, 26, 29
throttling, 93 clearance, 203
wiredrawing, 93, 208, 209 constant pressure expansion and compression
Expansion valve, automatic, 298-302 of, 25, 26
hand, 298 constants, 29
thermostatic, 302-318, 326, 327 critical temperature and pressure, 50
INDEX 539

Gas(es), ideal or perfect, 31 Horsepower, brake, 216, 217


isothermal expansion and compression of, 27, compressor, 214-220
28, 35-37 indicated, 217

law, general, 29 pump, 281,. 282


liquefying of, 49, 50 theoretical, 97, 214-216

polytropic expansion and compression of, 39- Hot gas defrosting, 391-395
41 Hot gas line, 372-375
processes, PV diagrams of, 34-39 Humidity, absolute, 59
properties of, 50 in storage coolers, 131, 179
specific heat, 32, 33 percentage, 60
General energy equation, 30 59
relative,

Glazing of fish, 139 60


specific,

Glycols, 201 Hydrocarbons, 297

Halide leak detectors, 293 Ice, banks, 167


Halogens, 287 latent heat of, 19

Halocarbon refrigerants, 287 melting temperature, 12


Hand expansion valves, 298 refrigeration, 72, 73
Head, centrifugal compressor, 354-356 specific heat, 17

friction, 278 Ideal gas, 31


pumping, 279-281 Impeller, centrifugal compressor, 352-354
static, 275-277 centrifugal pump, 278
total, 275-277 Indicated horsepower, 217-219
velocity, 275-277 Indicator diagrams, 217-219
Head-pressure ratio, 275 Indirect refrigeration, 197
Heat, 10 Induction motors, 407-415
intensity, 12 Industrial refrigeration, 122
latent, 18, 19 Infiltration, 146
mechanical energy equivalent of, 21-23 Insulation, thermal, 71
of compression, 95 Intercoolers, 396-399
quantity, measurement of, 17 Internal energy, 10
respiration, 155 kinetic, 11, 33

sensible, 18, 20 potential, 11

total, 21 Isentropic compression, 219


Heat exchangers, 114-117 Isothermal processes, 27, 28, 35-37
Heat loads, 71, 144-162
air change, 145, 152, 153 Jacketing, water, of compressor, 219
equipment, 147
lights, 147 Kelvin temperature, 13, 14
motors, 147 Kinetic energy, 7, 11, 33
occupancy, 147, 155 k factors, 148
product, 146, 153-155
sample calculations, 156-162 Labyrinth seal, 357
short method calculations, 155 Latent heat, 18-20
solar, 150 calculations, 19, 20

wall gain, 145, 147-152 of air, 62, 63


Heat transfer, 14-17 of fusion, 19
conduction, 14 of ice, 19
convection, 15 of vaporization, 20
radiation, 15-17 Leakage, air into refrigerated space, 148, 149
Heaters, crankcase, 348, 350 piston and valve, 209
Hermetic motor-compressors, 79 Liquid, 12
High pressure control, 236 chillers, 188-197, 201
High side float, 321, 323-325 intercoolers, 396-399
History of industry, 121 lines, refrigerant, 375, 376
Horsepower, 6, 7 risers, 376
540 INDEX
Liquid, saturated, 43 Oil, return to crankcase, 290-292
subcooled, 43 separators, 292, 379-381
subcoolers, 114 slugging, 347-350
Liquid-suction heat exchangers, 114-117 viscosity, 290, 291, 345, 346
Loads, heat, 144-162 Over-all coefficient of heat transmission, 148, 149
Locker plants, 141, 142 Overload protection for motors, 418, 422
Log mean temperature difference, 173
Low pressure control, 236, 237 Perfect gas, 31, 32
Low temperature systems, 395-401 Performance, coefficient of, 97
Lubricants, see Oil Percentage humidity, 60
Lubrication, methods of, 346, 347 Pilot valves, 327-328
Pipe, connections, 365
Manometers, 3, 4 fittings, equivalent length of, 278
Marine refrigeration, 122 friction loss in, 278
Matter, 10, 11 joints, 365
Mean effective temperature difference, 173 Piping, refrigerant, 365, 383
Mechanical efficiency, 216, 2i7, 221 condenser to receiver, 376-379
Mechanical energy, 7, 8 discharge, 372-375
heat equivalent of, 21-23 general specifications, 365-367
Mercury gages, 3, 4 noise in, 366, 367
Methane, 297 receiver to system, 375, 376
Methyl chloride, 287, 294 suction, 368-372
Methylene chloride, 287, 294, 295 supports, 366
Microorganisms in food, 125 types, 365
bacteria, 126 vibration in, 366, 367
molds, 128 Piping, water, 253, 254, 278
yeasts, 127 friction loss in, 278
Milk, growth of bacteria in, 127 Piston, clearance, 203
Moisture, in air, 59, 60 pins, 337
indicators, 383 rings, 337
289
in refrigerating systems, 288, speed, 341, 342
removal from system, 297, 383, 384 types, 337
Mold, 128 Polytropic processes, 39, 41, 219
Molecular theory of matter, 10 Potential energy, 7, 11
Molecules, 10 Potential relay, 417, 418
Motor 416-422
controls, Pour point of oil, 344, 345
magnetic starters, 419-422 Power, 6
manual starters, 418, 419 415
factor, 413,
416-418
starting relays, single phase, measurement of mechanical, 6, 7
Multiple compression systems, 395-401 Preservation of food, 122, 123
Multiple temperature systems, 402-406 Pressure, absolute, 5
atmospheric, 1
Oil, bleed lines, 381, 382 condensing, 43, 46, 50, 78, 83
check valve, compressor, 249, 250 constant, gas processes at, 25, 26
chemical stability, 290, 344 critical, 50
chillers, 379,380 defined, 1
cloud point, 344 effectof saturation temperature, 43-46
dielectric strength, 345 evaporator, 75
dilution by refrigerant, 290-292, 347-350 fluid, 274, 275
film, effect in evaporator, 290-292 gages, 3-6
floe point, 344,345 losses in refrigerant system, 117-120
foaming, 347-350 losses in water piping, 278
miscibility with refrigerant, 290-292 measurement of, 12
moisture in, 288, 289 oil, 422-425
pour point, 344, 345 saturation, 43-46
pressure failure control, 422-425 static, 274, 275
INDEX 541

Pressure, vapor, 58 Refrigeration loads, 71, also see Heat loads


velocity, 274, 275 Refrigerators, commercial, 139, 140
Pressure-enthalpy diagrams, 89, 93, 98, 101, 107, Relative humidity, 59
113, 116, 118, 119 Respiration heat, 155
Pressure-head equivalents, 275 Reverse cycle defrosting, 392
Pressure limiting expansion valves, 306-310 Rotating magnetic fields, 409-412
Pressure regulators, crankcase, 331, 333 Rotors, squirrel cage, 412
evaporator, 331-333 wound, 412, 413
Pressure relief valves, 384 Rotary compressors, 203, 250-252
Pressure-volume diagrams, 34-36, 38, 39
Pressure-weight equivalents, 274, 275 Safety controls, electrical overloads, 418, 422
Pulsations, discharge gas, 366, 367 fusible plugs, 384
Pumping head, 279-281 high pressure, 236
Pumping limit, centrifugal compressors, 360-362 low pressure, 236
Pumps, capacity, 279 oil pressure, 422-425

centrifugal, 278-282 pressure relief valves, 384


rating and selection, 279-281 Saturated vapor, 43
Propane, 297 Saturation pressure, 43-46
Propylene glycol, 201 Saturation temperature, 20, 43-46
Psychrometric properties of air, 57-69 Saybolt viscosity, 345, 346
Seals, crankshaft, 343
Quick freezing, 135 labyrinth, 357
Secondary refrigerants, 197-201
Radiation, 15-17 Sensible heat, 18
Range, control, 233 calculation of, 17, 21
cooling tower, 259 Separators, oil, 292, 379-381
Rankine temperature, 13, 14 Service valves, 78
Ratio, compression, 208 Shaded pole motors, 415
Reciprocating compressors, 203-223 Sharp freezing, 135
Refrigerant controls, 75, 76, 298-333 Shell-and-coil condensers, 255, 256
Refrigerant distributors, 311-314 Shell-and-coil evaporators, 191, 192
Refrigerant-oil mixtures, 290-292 Shell and tube condensers, 256, 257
Refrigerant piping, 365-383 Shell and tube evaporators, 192-195
Refrigerants, 71, 284-297 Sight glass, 383
ammonia, 293 Single phase motors, 413-415
butane, 297 Slip, rotor, 412
carbon dioxide, 287, 293 Sludge formation in oils, 288, 290
characteristics of, 284-287 Sodium chloride brine, 200
comparison of, 287 Solder joints, 365
ethane, 287, 288, 297 Solenoid valves, 328-331
fluorocarbons, 287, 288, 294-297 Specific gravity of liquids, 200
halocarbons, 287 Specific heat, 17
liquid, 73 of air, 246
safe properties, 284, 285 of gases, 32
secondary, 197-201 of water, 17
sulfur dioxide, 287, 293 Specific humidity, 60
thermodynamic properties of, 285, 287 Specific volume, 24
Refrigerating effect, 83-85 of gases, 21
Refrigerating systems, 78 of liquids, 20
Refrigeration, 71 of solids, 18
applications of, 121 Speed, compressor, 220, 221, 362, 363
commercial, 122 motor, 412
domestic, 122 Spoilage agents, 124
industrial, 122 control of, 128
marine, 122 Spray-type evaporators, 195, 201
transportation, 122 Squirrel cage rotor, 412
542 INDEX
Staging, 395-402 Temperature-entropy diagrams, 103
Standard 58
air, Thermal insulation, 71
Starters, motor, 418-422 Thermometers, 12
Starting relays, single phase motor, 416-418 Thermostatic expansion valve, 302-318
Starting torque, 412 cross-charged, 308, 309
Static head, 275, 276 external equalizer, 317, 318
Static pressure, 274, 275 gas-charged, 308, 309
Stator, motor, 408, 409 operation of, 302-304
Storage, cold, 130 pressure limiting, 306-310
conditions of, 130 and selection, 318
rating
diseases, 131 remote bulb location, 315-317
frozen, 133-139 superheat adjustment, 303, 304
humidity, 131, 179 Thermostatic sensing elements, 231, 232
mixed, 131 Thermostats, 231-236
refrigerated, 130 Torque, motor starting, 412-415
temperature, 130 Total heat, 21
Strainers, refrigerant line, 384 of air, 62-64
Stuffing box, 343 Towers, cooling, 259-261
Subcooled liquid, 43, 112-117 Timers, defrost, 388, 389
Subcoolers, 114 Toxicity of refrigerants, 284, 285
Sublimation, 49 Transportation refrigeration, 122
Suction line, 368-372 Tube, capillary, 318-321
Sulfur dioxide, 287, 293 pitot, 277
Superheat, 21 Tunnel freezers, 135
Superheated cycle, 107-112
Superheated region, 89 U factor, 148, 149, 171, 181
Superheated vapors, 21, 43 Unit coolers, 185-187
Surging in centrifugal compressors, 362 U tube manometers, 3, 4
Synchronous motors, 413 Universal viscosity, 345, 346
System, balance, 225-230 Unloaders, cylinder, 242
capacity, 85
vapor compression, 78 Valves, automatic expansion, 298-302
compressor, 337-341
Tank-type chillers, 190, 191 condenser by-pass, 301
Temperature, absolute, 13 condenser water regulating, 265-267
centigrade, 12, 13 crankcase pressure regulating, 331-333
compressor discharge, 83 float, high side, 321, 323-325
condensing, 49, 83, 101-103, 105, 253 low side, 321-323
float,
constant, processes at, 27, 28 hand expansion, 298
controls, 231-236 hand stop, 386, 387
conversions, 13, 14 hot gas by-pass, 391
critical, 50 pilot operated, 327, 328
dew point, 58 pressure 384
relief,
dry bulb, 61 receiver tank, 334, 335
eutectic, 199-201 service, 78
evaporating, 20 solenoid, 328-331
Fahrenheit, 12, 13 thermostatic expansion, 302-318
freezing, 19 Vapor, 12
fusion, 19 clearance, 203
in cold storage rooms, 130 condensation, 49, 50
Kelvin, 13, 14 pressure, 43, 48, 58
measuring instruments, 12 saturated, 43
Rankine, 13, 14 superheated, 43
saturation, 20, 143 tables, 52-56
wet bulb, 61 water, in air, 57-62
INDEX 543

Vaporization, 47 TABLES
ebullition, 47
Allowance for Solar Radiation, 450
evaporation, 47
Approximate Volumetric Efficiency of Refriger-
latent heat of, 20
ant-12 Compressors at Various Compres-
sublimation, 49
sion Ratios, 468
Vegetable, blanching of, 135
ASRE Refrigerant Numbering System, 472
Velocity, air, in refrigerated space, 131, 179
Average Air Changes per 24 Hours for Storage
over condensers, 245-247
Rooms, 452
over cooling coils, 174, 179, 185-187
Bleed-Off Rates, 469
Velocity, piston, 341, 342
Btu per Cubic Foot of Air Removed in Cooling
Velocity, refrigerant, in evaporator, 175-178
to Storage Conditions, 451
in refrigerant lines, 368-371, 373
Comparative Refrigerant Characteristics, 488
Velocity, water, in condenser, 253, 254
Design Data for Fruit Storage, 453
in piping, 282
Design Ground Temperatures, 447
Velocity head, in centrifugal compressors, 353-
Design Date for Meat Storage, 457
356
Design Data for Miscellaneous Storage, 459
in centrifugal pumps, 275-277
Design Data for Vegetable Storage, 455
Velocity pressure, 275-277
Dimensions and Physical Data—Copper, Brass,
Viscosity, effect of refrigerant dilution on oil,
or Steel Tubing, 489
290, 291
Equivalent Length of Valves and Fittings, 470
Saybolt universal, 345, 346
Evaporator Design TD, 466
Voltage, standard, 407
Freezing Points of Aqueous Solutions, 467
Volume, clearance, 203
Heat Equivalent of Electric Motors, 462
constant, processes, 28, 29
Heat Equivalent of Occupancy, 462
displaced per ton capacity, 86
effect of heat on, 24
Heat Transmissions Coefficients for Cold Stor-
age Rooms, 440
specific, 24
Volute compressors, 357 Mean Effective Temperature Differences, 465
Pressure Conversion Factors, 430
188-197 Pressure Drop Correction Factors, 471
Water, chillers,
cooling towers, 259-262 Properties of Gases, 430
defrosting, 389-390 Properties of Pure Calcium Chloride Brine, 466
density, 24, 25 Properties of Pure Sodium Chloride Brine, 467
fouling factors, 258 Properties of Saturated Steam, 431
gages, 3 Properties of Saturated Water Vapor with Air,
intercoolers, 399 432
latent heat of fusion, 19 Reaction Heat from Fruits and Vegetables, 461
latent heat of vaporization, 20 Refrigeration Design Ambient Temperature
pumps, 278-282 Guide, 444
regulating valves, 265-267 Refrigerant-11, Properties of Saturated, 474
saturation temperature, 20 Refrigerant- 12, Properties of Saturated, 475
specific heat, 17 Refrigerant-13, Properties of Saturated, 480
vapor in air, 57-62
Refrigerant-22, Properties of Saturated, 481
velocity in condensers, 253, 254
Refrigerant-717, Properties of Saturated, 483
Wax in lubricating oil, 344, 345
Refrigerant Line Capacities in Tons for Refrig-
Weight of refrigerant circulated per ton capacity, 490
erant- 12,
86, 368
Refrigerant Line Capacities in Tons for Refrig-
Wet bulb depression, 61
erant-22, 492
temperature, 61
Refrigerant Line Capacities in Tons for Refrig-
Wiredrawing, 93, 207-209
erant-717, 494
Work, 6
Refrigerant Line Capacities for Intermediate or
external, 30, 31, 33
heat equivalent, 21-23
Low Stage Duty, 496
of compression, 35-40 Relative Safety of Refrigerants, 487
Scale Factors—Water, 469
Yeasts, 127 Surface Conductance for Building Structures, 444
544 INDEX
Temperature and Enthalpy of Discharge Vapor, Resistance to Flow of Water through Copper
439 Tubing, 498
Thermal Conductivity of Materials Used in Cold Resistance to Flow of Water through Fairly
Storage Rooms, 443 Rough Pipe, 499
U Factors for Class, 444
Usage Heat Gain, 463
EQUIPMENT RATING TABUS
Wall Heat Cain, 464
Bare Pipe Coils in Air, 508
CHARTS Bare Pipe Coils in Water, 508
Btu per Minute Removed in Refrigerant- 12
Compressors, 512
Condensers, 497 Condensers, 524, 532
Btu per Minute Removed in Refrigerant-22 Condensing Units, 516
Condensers, 497 Cooling Towers, 531
Btu per Minute Removed in Refrigerant-717 Lineal Feet of Pipe per Square Foot, 509
Condensers, 498 Natural Convection Cooling Coils, 504
Minimum Gas Velocity in Discharge Risers, SOI Plate Evaporators, 506
Minimum Gas Velocity in Suction Risers, 500 Thermostatic Expansion Valves, 534
Flow Rate, 502
Refrigerant-12 Unit Coolers, 509
Flow Rate, 502
Refrigerant-22 Water Chillers, 510
IP^gwant-717 Flow Rate, 503 Water Valves, 533
"0
This Wiley International Edition is part of a continuing pro-
gram of paperbound text books especially designed for stu-
z
dents and professional people overseas.
reprinting of the original hardbound
It

edition,
is an unabridged
which is also
o
available from your bookseller.

Wiley Internationa) Editions


m
CO
include titles in the fields of:
\
o
-n
Chemical Engineering Psychology
Engineering Agricultural Engineering 20
Civil

Electrical Engineering and Agriculture m


Mechanics Biological Science
Metallurgy Chemistry
Mechanical Engineering Earth Sciences CD
Business Management Mathematics m
20
Statistics Physics
>

Q
O
CO
CO
-..
>

JOHN WILEY & SONS, INC., U.S, A.


TOF>PAN COMPANY LTD., JAPAN v<n
WILEY
TOPPAM
JW-0J3(P-1I)

^fa-

Das könnte Ihnen auch gefallen